Download Syllabus PDF

Survey
yes no Was this document useful for you?
   Thank you for your participation!

* Your assessment is very important for improving the workof artificial intelligence, which forms the content of this project

Document related concepts

Vectors in gene therapy wikipedia , lookup

Gene wikipedia , lookup

Gene therapy of the human retina wikipedia , lookup

Tay–Sachs disease wikipedia , lookup

Genetic testing wikipedia , lookup

History of genetic engineering wikipedia , lookup

Gene therapy wikipedia , lookup

Therapeutic gene modulation wikipedia , lookup

Site-specific recombinase technology wikipedia , lookup

Birth defect wikipedia , lookup

NEDD9 wikipedia , lookup

Population genetics wikipedia , lookup

RNA-Seq wikipedia , lookup

Epistasis wikipedia , lookup

Fetal origins hypothesis wikipedia , lookup

DiGeorge syndrome wikipedia , lookup

Epigenetics of neurodegenerative diseases wikipedia , lookup

Artificial gene synthesis wikipedia , lookup

Saethre–Chotzen syndrome wikipedia , lookup

Mutation wikipedia , lookup

Public health genomics wikipedia , lookup

Oncogenomics wikipedia , lookup

Cell-free fetal DNA wikipedia , lookup

Nutriepigenomics wikipedia , lookup

Frameshift mutation wikipedia , lookup

Neuronal ceroid lipofuscinosis wikipedia , lookup

Genome (book) wikipedia , lookup

Designer baby wikipedia , lookup

Microevolution wikipedia , lookup

Medical genetics wikipedia , lookup

Point mutation wikipedia , lookup

Transcript
2017 ACMG
GENETICS AND GENOMICS
REVIEW COURSE
SYLLABUS
TAMPA, FLORIDA
MAY 4 - 7, 2017
©American College of Medical Genetics and Genomics 2017
8th Edition
All Rights Reserved
Printed in the United States of America
2017 ACMG GENETICS AND GENOMICS REVIEW COURSE
SYLLABUS CONTENTS
Course Information
Agenda ................................................................................................................vi
Course Faculty ................................................................................................viii
Faculty Disclosures and HIPAA Compliance Statement .................................x
(Note: Educational Credits are Not Available for the Purchase of the Syllabus Only)
Thursday, May 4
ABMGG Review &
Exam Preparation Tips ..........................................................................................3
Friday, May 5
Cell Biology/Genomics ........................................................................................17
Genomics - Basics ..............................................................................................51
Clinical Cytogenetics .........................................................................................111
Clinical Molecular Genetics ...............................................................................139
Genetic Transmission........................................................................................177
Newborn Screening ...........................................................................................201
Developmental Genetics ...................................................................................227
Cancer Genetics I .............................................................................................259
Saturday, May 6
Genetic Counseling & Risk Assessment ...........................................................285
Biochemical Genetics I ......................................................................................311
Neurogenetics ...................................................................................................341
Reproductive Genetics I ....................................................................................367
Cancer Genetics II.............................................................................................395
Biochemical Genetics II .....................................................................................421
Systems-Based Disorders I ...............................................................................453
Sunday, May 7
Systems-Based Disorders II ..............................................................................475
Reproductive Genetics II ..................................................................................501
Genomic Medicine.............................................................................................531
Appendix
Syndromes Every Geneticist Should Know .......................................................551
Writing Exam Questions ....................................................................................635
Sample Exam Questions & Answers .................................................................651
American College of Medical Genetics and Genomics, 7101 Wisconsin Avenue, Suite 1101,
Bethesda, Maryland 20814, www.acmg.net, Telephone: (301) 718-9603
ii
2017 ACMG GENETICS AND GENOMICS REVIEW COURSE
COURSE INFORMATION
Course Description & Course Format
The ACMG Genetics and Genomics Review Course offers a 3 day format that provides an
intense learning environment with exam preparation lectures that cover a broad range of genetic
and genomic topics presented by recognized experts in the field. Topics include:
• Cell Biology/Genomics
• Genomics - Basics
• Clinical Cytogenetics
• Clinical Molecular Genetics
• Genetic Transmission
• Biochemical Genetics
• Developmental Genetics
• Cancer Genetics
• Genetic Counseling & Risk Assessment
• Newborn Screening
• Neurogenetics
• Reproductive Genetics
• Systems-Based Disorders
• Genomic Medicine
The Course will feature a pre-course sample examination, exam preparation and exam taking
tips, interactive examination workshops with in-depth coverage of exam content areas. There
will be an interactive breakout sessions with faculty on-site.
Course Objectives
At the conclusion of this activity, participants should be able to:
• Identify common genetic syndromes and discuss their clinical features
• Interpret standard molecular data and explain how to communicate results to families
• Perform simple quantitative genetic calculations and solve related problems
• Describe basic cytogenetics and identify features of common chromosomal disorders
• Recognize clinical features of selected metabolic disorders and describe their molecular
basis and review how to provide counseling about them
• Discuss the extents and limits of prenatal tests and explain how to perform routine
prenatal counseling
• Explain clinical and molecular aspects of inherited cancer syndromes and know how to
provide counseling for common human cancers
Course Objectives approved by the NSGC for Genetic Counselors
At the conclusion of this activity, participants should be able to:
• Identify common genetic syndromes
• Interpret standard molecular data
• Perform simple quantitative genetic calculations
• Describe basic cytogenetics
• Recognize clinical features of selected metabolic disorders
• Explain how to perform routine prenatal counseling
• Explain clinical and molecular aspects of inherited cancer syndromes
iii
Target Audience
This course is designed to assist genetics healthcare professionals who are seeking to update
and reinforce their general knowledge of medical genetics. This course is designed to assist
genetics healthcare professionals who are seeking to update and reinforce their general
knowledge of medical genetics. This course also allows ABMGG individuals to prepare for
Board Certification or Recertification. Genetic counselors are also welcome.
HIPAA COMPLIANCE
The ACMG supports medical information privacy. While the ACMG is not a “covered entity”
under HIPAA 1996 and therefore is not required to meet these standards, ACMG wishes to take
reasonable steps to ensure that the presentation of individually identifiable health information at
ACMG-sponsored events has been properly authorized. All presenters have completed a form
indicating whether they intend to present any form of individually identifiable healthcare
information. If so, they were asked either to attest that a HIPAA-compliant consent form is on
file at their institution, or to send ACMG a copy of the ACMG HIPAA compliance form. This
information is on record at the ACMG Administrative Office and will be made available on
request.
Content Validation
ACMG follows the ACCME policy on Content Validation for CME activities, which
requires:
Content Validation and Fair Balance
1. ACMG follows the ACCME policy on Content Validation for CME activities, which
requires:
a) All recommendations involving clinical medicine must be based on evidence that is
accepted within the profession of medicine as adequate justification for their
indications and contraindications in the care of patients.
b) All scientific research referred to, reported or used in CME in support or justification
of patient care recommendations must conform to the generally accepted standards
of experimental design, data collection and analysis.
2. Activities that fall outside the definition of CME/CE; “Educational activities that serve to
maintain, develop, or increase the knowledge, skills, and professional performance and
relationships that a physician uses to provide services for patients, the public, or the
profession” (source: ACCME and AMA) will not be certified for credit. CME activities that
promote recommendations, treatment, or manners of practicing medicine or pharmacy
that are not within the definition of CME/CE or, are known to have risks or dangers that
outweigh the benefits or, are known to be ineffective in the treatment of patients.
3. Presentations and CME/CE activity materials must give a balanced view of therapeutic
options; use of generic names will contribute to this impartiality. If the CME/CE
educational materials or content includes trade names, where available, trade names
from several companies must be used.
Off-label Uses of Products
When an off-label use of a product, or an investigational use not yet approved for any purpose,
is discussed during an educational activity, the accredited sponsor shall require the speaker to
disclose that the product is not labeled for the use under discussion, or that the product is still
investigational. Discussions of such uses shall focus on those uses that have been subject of
objective investigation.
iv
Disclaimer:
This review course is designed as an educational resource for medical geneticists and other
health care providers. Its use does not, and should not be considered to ensure a successful
outcome on the certification examinations offered by the American Board of Medical Genetics
and Genomics or the American Board of Genetic Counseling, or any other examinations. The
course should not be considered inclusive of all appropriate information or all available sources
of information that may be useful in preparing for the examinations or for any other purpose.
The ACMG does not endorse, or recommend the use of this educational program to make
patient diagnoses, particular by individuals not trained in medical genetics. Adherence to the
information provided in these programs does not necessarily ensure a successful diagnostic
outcome. The program should not be considered inclusive of all proper procedures and tests or
exclusive of other procedures and tests that are reasonably directed at obtaining the same
results. In determining the propriety of any specific procedure or test, a healthcare provider
should apply his or her own professional judgment to the specific clinical circumstances
presented by the individual patient or specimen.
v
2017 ACMG GENETICS AND GENOMICS REVIEW COURSE
AGENDA
Thursday, May 4, 2017
4:00 pm – 7:00 pm
Registration
7:00 pm - 7:30 pm
Introduction to Course
Bruce R. Korf, MD, PhD, FACMG
7:30 pm – 8:30 pm
ABMGG Review and Exam
Preparation Tips
Miriam G. Blitzer, PhD, FACMG
Gary S. Gottesman, MD, )$$3, FACMG
Friday, May 5, 2017
7:30 am – 8:00 am
Continental
Breakfast/Registration
8:00 am – 9:00 am
Cell Biology/Genomics
Christa Lese Martin, PhD, FACMG
9:00 am – 10:00 am
Genomics - Basics
Madhuri Hegde, PhD, FACMG
10:00 am –10:30 am
Break
10:30 am – 11:30 am
Clinical Cytogenetics
Christa Lese Martin, PhD, FACMG
11:30 am – 12:30 pm
Clinical Molecular Genetics
Madhuri Hegde, PhD, FACMG
12:30 pm – 2:00 pm
Lunch (on your own lunch)
2:00 pm – 3:00 pm
Genetic Transmission
Bruce R. Korf, MD, PhD, FACMG
3:00 pm – 4:00 pm
Newborn Screening
John A. Phillips, III, MD, FACMG
4:00 pm – 4:30 pm
Break
4:30 pm – 5:30 pm
Developmental Genetics
Tony Wynshaw-Boris, MD, PhD,
FACMG
5:30 pm – 6:30 pm
Cancer Genetics I
Sharon Plon, MD, PhD, FACMG
6:30 pm – 7:30 pm
Dinner (on your own)
7:30 pm – 9:00 pm
Advanced Genetic Topics Informal Sessions
•
Quantitative
•
Biochemistry
•
Molecular
•
Cytogenetics
•
Prenatal
•
Genetic Counseling
This session is not live
streamed or recorded
vi
Faculty
Saturday, May 6, 2017
7:30 am – 8:00 am
Continental Breakfast
8:00 am – 9:00 am
Genetic Counseling & Risk
Assessment
Pamela L. Flodman, MSc, MS, LCGC
9:00 am – 10:00 am
Biochemical Genetics I
Gerard Berry, MD, FACMG
10:00 am – 10:30 am
Break
10:30 am – 11:30 am
Neurogenetics
Bruce R. Korf, MD, PhD, FACMG
11:30 am – 12:30 pm
Reproductive Genetics I
Louise E. Wilkins-Haug, MD, PhD,
FACMG
12:30 pm – 2:00 pm
Lunch (on your own lunch)
2:00 pm – 3:00 pm
Cancer Genetics II
Sharon Plon, MD, PhD, FACMG
3:00 pm – 4:00 pm
Biochemical Genetics II
Gerard Berry, MD, FACMG
4:00 pm – 5:00 pm
Systems-Based Disorders I
Bruce R. Korf, MD, PhD, FACMG
05:00 pm – 6:30 pm
Dinner (on your own)
6:30 pm – 8:00 pm
Exam Workshop
8:00 pm – 9:00 pm
Networking Reception
Gary S. Gottesman0')$$3
FACMG
Sunday, May 7, 2017
7:30 am – 8:00 am
Continental Breakfast
8:00 am – 9:00 am
Systems-Based Disorders II
John A. Phillips, III, MD, FACMG
9:00 am – 10:00 am
Reproductive Genetics II
Louise E. Wilkins-Haug, MD, PhD,
FACMG
10:00 am – 10:30 am
AM Break
10:30 am – 11:30 am
Genomic Medicine
Bruce R. Korf, MD, PhD, FACMG
11:30 am – 12:00 pm
Course Wrap Up
Bruce R. Korf, MD, PhD, FACMG
John A. Phillips, III, MD, FACMG
vii
2017 ACMG GENETICS AND GENOMICS REVIEW COURSE
COURSE FACULTY
COURSE DIRECTORS
John A. Phillips, III, MD, FACMG
David T. Karzon Professor of Pediatrics
Professor of Pathology, Microbiology and
Immunology and Professor of Medicine
Director, Division of Medical Genetics &
Genomic Medicine
Vanderbilt University School of Medicine
DD-2205 Medical Center North
Nashville, TN 37232-2578
Tel: (615) 322-7602
[email protected]
Bruce R. Korf, MD, PhD, FACMG
Wayne H. and Sara Crews Finley Chair in
Medical Genetics
Professor and Chair, Department of
Genetics
Director, Heflin Center for Genomic
Sciences
University of Alabama at Birmingham
1720 2nd Ave. S., Kaul 230
Birmingham, AL 35294
Tel: (205) 934-9411
[email protected]
FACULTY
Gerard Berry, MD, FACMG
Harvey Levy Chair in Metabolism Director,
Metabolism Program Division of Genetics
and Genomics Boston Children’s Hospital
Professor of Pediatrics, Harvard Medical
School, Center for Life Science Building,
Suite 14070
3 Blackfan Circle
Boston, MA 02115
Tel: (617) 355-4316
[email protected]
Madhuri Hegde, PhD, FACMG
Associate Professor
Emory Genetics Lab Scientific Director
Sr. Director, Emory Genetics Lab, Molecular
Lab
Department of Human Genetics
Emory University School of Medicine
2165 North Decatur Road
Decatur, GA 30033
Tel: (404) 727-5624
[email protected]
Pamela L. Flodman, MSc, MS, LCGC
Adjunct Professor, Pediatrics
School of Medicine
Director, Graduate Program in Genetic
Counseling
Department of Pediatrics
University of California, Irvine
101 The City Drive
Mail Code: 4482
Orange, CA 92868
Tel: (714) 456-5789
[email protected]
Christa Lese Martin, PhD, FACMG
Autism & Developmental Medicine Institute
Geisinger Health System
120 Hamm drive, suite 2A, M.C. 60-36
Lewisburg, PA 17837
Tel: (570) 522-9427
[email protected]
Sharon E. Plon, MD, PhD, FACMG
Professor, Pediatrics/Hematology-Oncology
Professor, Molecular and Human Genetics
Human Genome Sequencing Center
Director, Medical Scientist Training Program
Department of Pediatrics
Baylor College of Medicine
Feigin Center Room 1200.18
1102 Bates Street
Houston, TX 77030
Tel: (832) 824-4251
[email protected]
Gary S. Gottesman, MD, FAA3, FACMG
Medical Geneticist
Center for Metabolic Bone Disease
Shriners Hospitals for Children - St. Louis
2001 S. Lindbergh Blvd.
St. Louis, MO 63131
Tel: (314) 872-8305
[email protected]
viii
Louise E. Wilkins-Haug, MD, PhD, FACMG
Division Director, Maternal Fetal Medicine
and Reproductive Genetics
Brigham & Women's Hospital
Professor, Obstetrics/Gynecology
Harvard Medical School
75 Francis Street
Boston, MA 02115
Tel: (617) 732-4208
Fax: (617) 264-6310
[email protected]
CONTRIBUTOR
Miriam G. Blitzer, PhD, FACMG
Executive Director
American Board of Medical Genetics and
Genomics
9650 Rockville Pike
Bethesda, MD 20814-3998
Tel: (410) 706-1429
[email protected]
Tony Wynshaw-Boris, MD, PhD, FACMG
James H. Jewell Professor of Genetics
Chair, Department of Genetics and Genome
Sciences
Case Western Reserve University, School
of Medicine
University Hospitals Case Medical Center
One 10900 Euclid Avenue, BRB731
Cleveland, OH 44106-4955
Tel: (216) 368-0581
[email protected]
ix
FACULTY DISCLOSURES
As a sponsor accredited by the ACCME, the American College of Medical Genetics and
Genomics must ensure balance, independence, objectivity and scientific rigor in all its
sponsored educational activities. All faculty participating in a CME-certified activity are
expected to disclose to the audience any relevant financial interest(s) or other relationship(s)
with the manufacturer(s) of any commercial product(s), provider(s) of commercial services or
any commercial supporters, including diagnostic laboratories, of the activity discussed in an
educational presentation. Relevant financial interest(s) or other relationship(s) can include such
things as grants or research support, consultancy, major stock holder, etc. The intent of this
disclosure is not to prevent a planner or speaker with a relevant financial or other relationship
from course planning or making a presentation, but rather to provide learners with information
on which they can make their own judgments. It remains for the audience to determine whether
the speaker's interests or relationships may influence the presentation with regard to exposition
or conclusion. All conflicts of interests have been reviewed and resolved by the education and
CME subcommittee.
The following have reported disclosures:
Bruce R. Korf, MD, PhD, FACMG
Dr. Korf receives grant/research support from Novartis. He is on the Advisory Board for
Accolade and Genome Medical. He is on the Board of Directors for the American College of
Medical Genetics and Genomics and the Children’s Tumor Foundation. He is an advisor for the
Neurofibromatosis Therapeutic Acceleration Project, a Founding Member of Envision
Genomics. He receives a salary from the University of Alabama at Birmingham.
Madhuri Hegde, PhD, FACMG
Dr. Hegde is an advisor for Genzyme (Pompe program), PTC (DMD program), Coriell Cell
Repositories, and PerkinElmer Genetics Inc.
Christa Lese Martin, PhD, FACMG
Dr. Martin is employed by Geisinger Health System and is a consultant the The Jackson
Laboratory.
John A. Phillips, III, MD, FACMG
Dr. Phillips is a Site Investigator for the following: 1) PKU: BMN 015 &165, 2) Achondroplasia:
BMN 111-901, 201 & 202 Clinical Trials BioMarin Pharmaceutical Inc. & 3) FAOD: UX007
Ultragenyx Pharmaceuticals, Inc. He is Principal Investigator for TN State Genetics Contract &
Member TN Genetics Advisory Committee. A Co-PI: Vanderbilt Undiagnosed Disease Network
(UDN) Clinical Center and a Co-I: Dr Blackwell’s PPG “Mechanisms of Familial Pulmonary
Fibrosis”.
Sharon Plon, MD, PhD, FACMG
Dr. Plon is an employee of Baylor College of Medicine (BCM) which derives revenue from
genetic testing, including whole exome sequencing. BCM and Miraca Holdings Inc. have agreed
on a joint venture with shared ownership and governance of the clinical genetics diagnostic
laboratories. Dr. Plon will also discuss off-label use and/or investigational use of drug target
tumors.
The following faculty members have nothing to disclose:
Gerard Berry, MD, FACMG
Pamela L. Flodman, MSc, MS, LCGC
Gary S. Gottesman, MD, FAAP, FACMG
Louise E. Wilkins-Haug, MD, PhD, FACMG
Anthony J. Wynshaw-Boris, MD, PhD, FACMG
Miriam G. Blitzer, PhD, FACMG
Jane Radford, MHA, CHCP (ACMG Staff)
x
Exam Preparation Tips
EXAM SKILLS WORKSHOP
Gary S Gottesman, MD, FAAP, FACMG
Medical Geneticist
Center for Metabolic Bone Disease and Molecular Research
Shriners Hospitals for Children – Saint Louis
Gary S. Gottesman, MD, FAAP, FACMG
Center for Metabolic Bone Disease and Molecular Research
Shriners Hospitals for Children – Saint Louis
4400 Clayton Ave.
St. Louis, MO 63110
(314) 872-8305 Telephone
(314) 872-7844 Fax
[email protected]
1
2
Examination Skills Workshop
Gary S Gottesman, MD
Medical Geneticist
Center for Metabolic Bone Disease and Molecular Research
Shriners Hospital for Children – Saint Louis
Disclosure(s)
I have no conflicts of interest to disclosure
ABMGG Website
• Training & Certification
• ITE Practice Exam (External Website)
• http://www.starttest.com/ITDVersions/11.1.0.1/ITD
Start.aspx?SVC=7478ee5d-5cf8-4042-a4790038337ce40b
3
ABMGG Website
Exam Tutorial
• Overview of Your Test
• Tutorial
• Exam Section 1
• Exam Section 2
• Exam Section 3
• Exam Section 4
• Total Session Time 1 Hour 15 Minutes
Items
5
5
5
5
20
Exam Tutorial
• Navigating Throughout Your Exam
• Information Panel
• Scrolling the Screen
• Zooming
• Navigation Panel
• Keyboard Accessibility
• Button
4
Exam Tutorial
• Time Management
• Section Time Remaining
• Session Time Remaining
• Examination Overview
• Unauthorized Breaks
• Taken during test block
• Timer continues to run
Exam Tutorial
• Answering and Marking Items
• Click with your choice of answer with the mouse
• You may change your answer at any time
• You may opt to mark an item with “Mark” check box
Exam Tutorial
Help Menu
• Highlight
(Keywords)
• Strike Out
(Distractors)
• Notes
Calculator
5
Exam Tutorial
• Overview of the Review Screen
• Item Review Screen
• Can be accessed at any point
• Select Review Button
• Appears after the last item in the section
• If time remains:
• You can return to
• Incomplete
• Marked items
Exam Item Formats
• Matching Format
• This item format consists of a series of items related
to a common logic.
• Item set may be used for multiple problems.
Exam Tutorial
• Multiple Item Format
• “The following vignette applies to the next 2 items.”
• May also be a single option set that is associated with multiple
vignettes:
• “The response options for the next 4 items are the same.
Select one answer for each item in the set.”
• “For each description, select the associated disorder (A-E).”
6
Exam Tutorial
• End of Examination Survey
• End of Session Notice
NBME Resources
• www.nbme.org
• Register with NBME – Customer Access Portal
• Find Lessons
• Item Writing Manual - 2016
• Download this manual (>90 pages)
• Recommend looking at Appendix B
• Sample Lead-Ins
• Online Interactive Tutorial
• http://www.nbme.org/IWTutorial
Template for NBME Exam Questions
• Stem:
• Typically includes a vignette that provides some
clinical relevance to the topic being addressed in
the question. The vignettes are supposed to be as
succinct as possible.
7
Template for NBME Exam Questions
• Stem:
• The stem ends with a question composed in the
following manner:
• “Which of the following . . (identify the characteristic
that links all the distractors) . . is the best (or most likely)
response . . ?”
• What it the chance that the . . . (numerical answer) ?
Template for NBME Exam Questions
• Option Set:
A. Options should be lettered from A to E (must have 5
total)
B. Place options in alphabetical order based on first word,
second word, etc. or numerical order (increasing or
decreasing)
C. Distractors should all be about the same length as the
correct option
D. Avoid: “All of the above,” EXCEPT, NOT, Least, etc.
E. Avoid multiple-multiple choice problems
Template for NBME Exam Questions
• Test takers should be able to read the stem and come
up with possible answers and the correct answer
without ever referring to the option set.
8
Test Taking
• READ the stem
• Note key words or key information
• Highlight them if the stem is long
• SIMPLIFY stem
• Translate stem into your own words
• ANSWER the question
• Answer in your own words before looking at options
Test Taking
• ELIMINATE answers that are unlikely or include absolutes
• Never, always, every, at no time, etc.
• Answers that mean basically the same thing
• COMPUTATION
• Determine formula or principle required
• Complete computation before looking at options
• If your answer is not in option set check your work
Test Taking
• REMEMBER
• Longest answer is often the correct one
• Loaded with qualifying adjectives or phrases
• Key words from stem in option may identify it as correct
• CHOOSE the best answer
• Correct choice often contains relative qualifier
• Usually, generally, sometimes, often, etc.
9
Test Taking
• GUESSING STRATEGY
• If two options are opposites one is likely correct
• If you cannot identify answer – guess
• There is no penalty for guessing.
• Always guess the same answer
• Unless you can eliminate it
Additional Recommendations
• Make sure you get a good night’s
sleep prior to the exam.
• Consider reading a medical school genetics text from
cover to cover for general exam preparation.
Additional Recommendations
• Study photos and common signs and symptoms of
genetic disorders from a dysmorphology text and
GGRC 100
• Syndromes section (few photos/x-rays).
• Check with others who took the test in the past for
any study aides they may have used.
10
Additional Recommendations
• Arrive at least 30 minutes prior to your exam
• Take some snacks to eat during breaks.
Genetics Education Resources
• Genetics texts:
• Thompson & Thompson Genetics in Medicine by Nussbaum,
McInnes and Willard
• USMLE-type questions
• Medical Genetics by Jorde & Carey
• medgen.genetics.utah.edu/index.htm
• Human Genetics and Genomics (Human Genetics: A ProblemBased Approach) by Bruce R. Korf, MD, PhD and Mira B Irons
(Fourth edition)
• http://www.korfgenetics.com/
Genetics Education Resources
• Genetics Websites
• GeneReviews.org
• Genetics Education Center (KUMC)
• http://www.kumc.edu/gec/
• University of Michigan Medical School
http://www.med.umich.edu/lrc/coursepages/M1/humange
netics/genetics/humangeneticsquiz.html
• Not in NBME format
11
Acknowledgements
Debra L. Schindler, PhD, Office of Curricular Affairs, Saint Louis University SOM
References
www.abmgg.org
www.nbme.org
Barbara Swanson, I.S.U. Learning Strategies –
http://www2.isu.edu/success/strategies/handouts/docs/test_taking_and_money/Systematic%20Method%20of%20Answering%20Multiple%20Choice%20Questions.pdf
http://www.studygs.net/tsttak3.htm
12
Cell Biology/Genomics
CELL BIOLOGY
GENOMICS
Christa Lese Martin, PhD, FACMG
Geisinger Health System
Autism & Developmental Medicine Institute
Director and Senior Investigator
Christa Lese Martin, PhD, FACMG
Autism & Developmental Medicine Institute
Geisinger Health System
120 Hamm Drive, Suite 2A, M.C. 60-36
Lewisburg, PA 17837
(570) 522-9427 Telephone
(570) 522 9431 Fax
[email protected]
15
16
Cell Biology:
Chromosomes in Gametogenesis and Cell Division
Christa Lese Martin, PhD, FACMG
Director and Professor
Autism & Developmental Medicine Institute, Geisinger Health System
Disclosure(s)
Employed by Geisinger Health System
Consultant for The Jackson Laboratory
Overview
•
•
•
•
History of Cytogenetics
Chromosome Structure
Mitosis and Meiosis
Numerical and Structural Chromosome Abnormalities
17
History of Cytogenetics
1923: T.S. Painter established human chromosome number as 48 and identified the Y
chromosome (XX/XY mode of sex determination)
1953: T.C. Hsu accidentally discovered hypotonic treatment of cells to spread
chromosomes
1956: Only 46 chromosomes in humans by Tjio & Levan
1959: Trisomy 21 causes Down Syndrome, by Jerome Lejeune in Paris. 1st chromosome
abnormality, birth of clinical cytogenetics
1969-1970: Banding techniques discovered by Caspersson in Sweden (fluorescent Qbanding first, followed by G-banding and others)
1970-75: Amniocentesis shown to be safe and accurate method for prenatal diagnosis
1976: “High-resolution” methods developed
by J. Yunis
4
History of Cytogenetics (cont’d)
1976-1980: Fragile sites on human chromosomes re-discovered, including fragile X
1982-87: CVS introduced and shown to be safe and effective for first trimester prenatal
diagnosis
1990: FISH – molecular cytogenetics
1990: Genomic imprinting and UPD understood as important mechanisms of human
genetic disease
1992: CGH (Comparative Genomic Hybridization) developed for solid tumor studies
1996: 24 color FISH (including SKY, spectral karyotyping)
2001: Cytogenomic microarrays, including array CGH
2010: Cytogenomic microarrays become the first-tier test replacing G-banded karyotype
5
ADD DATE FOR CNV CALLING FROM WES/WGS
DATA
Chromosome Structure
Challenges of DNA packaging:
Each human cell contains ~2 meters of DNA if stretched end-end; yet the nucleus of a
human cell is only ~6 μm in diameter
DNA packaging into a set of chromosomes:
• The complex of DNA + proteins is called “Chromatin”
• These proteins include:
1. Proteins involved in DNA packaging
2. Proteins associated with gene expression, DNA replication and DNA repair
• Each human cell contains two copies of each chromosome (one maternal and one
paternal). These pairs are called “Homologous Chromosomes”
18
General model for
the many levels of
chromatin packaging
Molecular Biology of the Cell, 4th edition. Alberts B,
Johnson A, Lewis J, et al. Garland Science; 2002
Chromosome Structure
Metacentric
Submetacentric
Acrocentric
telomere
satellite
short arm - p
centromere
stalk
long arm - q
chromatid
telomere
• Metaphase chromosomes are comprised of two sister chromatids
connected at the centromere (primary constriction)
• All human chromosomes are bi-armed
telomere
3
Chromosome Structure
Acrocentrics (13, 14, 15, 21, and 22) are a special class of human chromosomes.
They have very small p arm comprised of large, tandem arrays of rDNA genes
(stalks; stain positive only by silver staining).
“Associate” in interphase to form nucleolus
(also called nucleolus organizer regions or NORs).
At end of stalks are “satellites” (which are made up of highly repetitive “junk”
DNA and no known coding sequences) which stain positive by C-banding.
Length/size of stalks and satellites are polymorphic.
19
G-banded Karyotype
acrocentric
Image from ISCN 2013
Courtesy of N.L. Chia
G-banding pattern and DNA content
G negative
(light bands)
G positive
(dark bands)
Higher GC content
Lower AT content
Lower GC content
Higher AT content
Rich in SINE and Alu repeats
Rich in LINE repeats
Early replicating
Late replicating
Contain housekeeping genes
Genes are tissue specific
Rich in transcribed sequences
Sparse genes
Human genome sequencing project
supports these findings
G-banded Ideograms
Left - Computer generated ideogram
Right – ISCN 1995 schematic and measurements
20
Centromeres and Telomeres
of Human Chromosomes
Centromere Structure
Alpha-satellite DNA:
í 171 bp tandemly repeated sequence clustered at all human centromeres
í 300 kb - 5,000 kb present at each human centromere
í Sequence divergence in 171 bp repeat allows chromosome-specific alpha-satellite FISH
probes (except 13 and 21, 14 and 22, whose sequence are too similar for unique
hybridization)
í The inner plate of a kinetochore is “seeded” in the centromere, followed by cooperative
assembly of the entire group of special proteins that form the kinetochore
Telomere Structure
TTAGGG - simple sequence repeat:
í 3-20 kb (T2AG3) n
í Substrate for telomerase (solves the end replication
problem)
í Shortens with age
í True terminal deletions can generate new telomeres
TAR (telomere associated repeats); aka “subtelomeric repeats”:
í 100-300 kb most chromosome ends
í Sequence homologies to subsets of chromosomes
Chromosome specific (unique) DNA:
Most begin ~200-300 kb from end of chromosome
21
Telomere Structure – cont’d
í Highest gene density in human genome (high GC content; G-negative bands)
í Very high genetic recombination in females and males, but especially in males
(only chromosome regions male recombination is higher than females)
í Critical role in meiosis: synapsis of homologous chromosomes begins at
telomeres
í Polymorphic: evidence of length and sequence polymorphism for some
human telomeres
(e.g., 16p has 3 alleles; 2q deletion)
Mitosis and Meiosis
ƒ Mitosis
growth and development of somatic cells
one cell divides to give rise to two identical “daughter” cells
–
–
ƒ Meiosis
production of germ cells
diploid cells of the germline give rise to haploid gametes
(sperm/egg)
–
–
Mammalian Cell Cycle
alternation between mitosis and interphase (resting state)
Mitosis
PM
AT0
hours
G2
25
4 -6 hrs
2n, 4c
5
G1 Phase
S Phase
12 – 24 hrs
7 hrs
2n, 2c
20 2n, 3c
S - DNA replication occurs
(synthesis of RNA and proteins)
G2 - interval between S and mitosis (repair occurs)
10
15
Interphase (DNA decondensed):
G1 - interval between mitosis and replication
Mitosis and Cytokinesis:
cell division processes (nuclear and cytoplasimic division)
22
Mammalian Cell Cycle
n = haploid chromosome #
Mitosis
PM
AT0
hours
c = DNA content
G2
25
4 -6 hrs
2n, 4c
S Phase
7 hrs
20 2n, 3c
46, each chromosome
has 2 chromatids
5
G1 Phase
46, each chromosome
has 1 chromatid
12 – 24 hrs
2n, 2c
10
15
Many sources use “n” incorrectly to
refer to DNA content; it is important
to understand the difference
Mitosis
•Prophase
Chromosomes condense
Mitotic spindle/centrioles
begin to form
Figure 2-10 from Thompson & Thompson, Genetics in
Medicine, edition 7, copyright 2007, Elsevier
Mitosis
•Prometaphase
Nuclear membrane
disappears
Figure 2-10 from Thompson & Thompson, Genetics in
Medicine, edition 7, copyright 2007, Elsevier
23
Mitosis
•Metaphase
Chromosomes fully
condensed
Chromosomes line up on
the “metaphase plate”
Spindle fibers begin to
contract
Figure 2-10 from Thompson & Thompson, Genetics in
Medicine, edition 7, copyright 2007, Elsevier
Mitosis
•Anaphase
Centromeres divide in two
Spindle fibers pull
chromatids toward opposite
sides of the cell (centromere
first))
Figure 2-10 from Thompson & Thompson, Genetics in
Medicine, edition 7, copyright 2007, Elsevier
Mitosis
•Telophase
Cytokinesis (forms two
identical daughter cells)
Two nuclear membranes form
Spindle fibers disappear
Chromosomes decondense
and return to interphase
Figure 2-10 from Thompson & Thompson, Genetics in
Medicine, edition 7, copyright 2007, Elsevier
24
Mitosis Summary
1
1
1
2n2c
1
1
2n4c
1
1
1
2n2c
1
1
1
1
1
1
(only showing chr 1)
1
1
Meiosis
Meiosis is a specialized form of cell division
that occurs only during gametogenesis
• comprised of 1 round of DNA replication,
but 2 cell divisions -- M I and M II
• divides genetic material in half
• shuffles genetic material by recombination
Meiosis
Meiosis I
• reduction division
(46ї23, 2n ї 1n, diploid ї haploid)
• occurs only in meiosis
NO INTERVENING DNA REPLICATION
Meiosis II
• identical to mitosis in somatic cells, except only
23 chromosomes are present
25
Male Meiosis I
MI:
2n,2c
Male Meiosis II
MII
1n,2c
2n,4c
1n,2c
1n,1c
From Vogel and Motulsky, Figure 3.34, 4th edition,
Copyright 2010, Springer
MI:
2n,2c
From Vogel and Motulsky
2n,4c
1n,2c
From Vogel and Motulsky, Figure 3.34, 4th edition, Copyright 2010, Springer
Prophase I:
- leptotene: chromosomes begin to condense
- zygotene: homologs pair (telomere); Synaptonemal complexes form
- pachytene: crossing over occurs
- diplotene: homologs separate; remain attached at chiasma
- diakinesis: separation of homologs
MII: 1n,2c ĺ 1n,1c. 4 gametes in males.
Note recombinant and nonrecombinant chromosomes.
MII
1n,2c
1n,1c
From Vogel and Motulsky, Figure 3.34, 4th edition,
Copyright 2010, Springer
26
1
Recombination
Only one sister chromatid involved in each crossover event
• Centimorgan used to describe recombination frequency;
1 cM equals 1% chance for recombination; translates to ~1 Mb
•Avg. 50 chiasmata in male meiosis; Number correlates with length
of chromosome arms
• ≥ 1 chiasma/chromosome arm is required for normal segregation
Consequences of Meiosis
Random segregation during Meiosis I makes the likelihood of
any two gametes from an individual having the exact same
chromosomes equal to 1 in 223 (1 in 8 million)
Shuffling of the DNA through recombination makes the above
likelihood even smaller
That’s why no two people are exactly the same!!
Meiosis I Summary – homologs separate
(only showing chr 1 and chr 4)
1 11 1
2n4c
1
11
1n2c
11
4 44 4
1
4
1
4
4
44
44
1n2c
1
4
27
Meiosis II Summary – chromatids separate
Essentially same as mitosis except only 23 chromosomes are present
1n2c
1n2c
1
1
4
4
1n1c
1n1c
1
1
1
4
4
4
1
4
Female vs. Male Meiosis
Male
Commences
Puberty
Duration of
meiosis
60-65 days
Gametes/
meiosis
4
spermatids
Gamete
production
100-200
million per
ejaculate
Female
Early
embryonic
life
10-50 years
Only complete
after fertilization
1 ovum and
2 (3) polar
bodies
1 ovum per
menstrual
cycle
Begins
prenatally
Arrested in first meiotic
prophase = dictyotene
(does not occur in males)
Female Meiosis
Meiosis I
completed
at ovulation
1n, 2c
2n, 2c
1n, 1c
From Vogel and Motulsky, 3rd edition, Springer
28
Male vs Female Meiosis
Image from Pearson Education
http://www.mun.ca/biology/desmid/brian/BIOL2060/BIOL2060-20/20_09.jpg
Errors of Meiosis and Mitosis:
Nondisjunction
• Failure of homologous chromosomes (MI) or sister
chromatids (MII, mitosis) to separate;
• Trisomy and monosomy can originate from meiotic or
mitotic nondisjunction;
• Parental origin of the extra chromosome in trisomy is most
often maternal and most often a result of an M I error.
Mitotic Nondisjunction – Somatic:
Results in Mosaicism
46
46
46
46
46
46
46
NORMAL
47
46
47
45
47
45
45
MOSAICISM
(45 cell line often not viable)
Don’t confuse mosaicism with chimerism!
Most common form: two zygotes fuse to form one embryo
29
Meiotic Nondisjunction
1st meiotic
division
nondisj.
nl
nl
2nd meiotic
division
nl.
nl
nondisj.
Meiotic Nondisjunction (cont’d)
fertilization
MI nondisj.
trisomy and monosomy
MII nondisj.
Normal, trisomy & monosomy
Meiotic Origin of Nondisjunction
MI
MII
Molecular genetic polymorphic markers can be used to
determine at which stage of meiosis the error occurred.
MI errors will show three different homologs.
MII errors will show only two different homologs.
30
Parental Origin of Aneuploidy
% mat
MI
MII
% pat
92
75
25
8
3
+21
+18
97
31
69
+16
100
100
0
0
XXX
90
78
22
10
XXY
54
54
74
26
46
X
30
30
MI
MII
100
70
XXX cases are the same as autosomal trisomies.
XXY: mat = pat nondisjunction.
45,X is mainly paternal nondisjunction.
Maternal Age Effect – Trisomy 21
Incidence
(per 1,000 births)
25
20
15
10
5
0
20
25
30 35 40
Mother’s Age
45
Meiotic Origin of Nondisjunction
Studies have shown that aberrant recombination is also associated with nondisjunction:
• Absent (achiasmate) or reduced recombination
- reported for nearly all acrocentric trisomies, some non-acrocentrics (chr 18 and sex
chromosomes)
• Suboptimally located crossovers
- exchanges among MII nondisjunction tri21 cases clustered at the pericentromeric region
(not tel) – suggesting location may also cause susceptibility to MI or MII non-disjunction
(Lamb et al., 1996; Lamb et al., 1997; Brown et al., 2000; Garcia-Cruz et al., 2009)
31
Meiotic Errors Leading to Aneuploidy
ƒ Most errors occur at Meiosis I (MI)
ƒ Non-disjunction is associated with
abberations in both the frequency and
placement of chiasmata:
9 Too few recombinations
9 Too close to the centromere
9 Too distal
ƒ Premature separation of sister
chromatids: another mechanism for
aneuploidy in meiosis
Hassold and Hunt, Nature Vol2:280-292 (2001)
Multiple Mechanisms Contribute
to Maternal Age Effect
Human maternal age effect involves different “hits” that work
together to increase frequency of errors in eggs
• Decreased interactions between homologous chromosomes in
prophase during recombination (females seem to be more
prone)
• Long prophase arrest in females
• Failed repair checkpoints in oocytes with longer arrest time
(males seem to maintain better checkpoints and default to
spermatocyte “death”)
Nagaoka et al. (2012) Nat Rev Genet
Frequency of Chromosome Abnormalities
• Spontaneous miscarriage (first trimester)
50.0 %
• ~15% of all pregnancies result in miscarriage
• Stillbirths and perinatal deaths
5.0 %
• Livebirths
• Congenital anomaly with ID
• Congenital heart disease
0.5 - 1%
23.0 %
13.0 %
• Couples with multiple spontaneous abortions
(balanced rearrangements)
5.0 %
32
First Trimester Spontaneous Miscarriage
Š ~15% of all first trimester pregnancies end as miscarriage:
Š NORMAL CHROMOSOMES
40 %
Š ABNORMAL CHROMOSOMES
ƒ Autosomal trisomy
- most common tri 16 (~16%);
never observed in liveborns
ƒ Monosomy X (~99% abort)
ƒ Polyploidy
ƒ Structural abnormalities
60 %
50 %
25 %
20 %
<5 %
Numerical Chromosome Abnormalities
ƒ EUPLOID = exact multiple of haploid set
• Haploid = normal number in gametes (n=23 in humans)
• Diploid = normal number in zygote and somatic cells (2n=46)
• Polyploidy = complete set(s) of extra chromosomes
ƒ Triploidy = 3n = 69/cell in humans
ƒ Tetraploidy = 4n = 92/cell in humans
ƒ ANEUPLOID = loss/gain of single chromosomes
• Monosomy = chromosome number = 45
• Trisomy = chromosome number = 47
Triploid Karyotype
69,XXY
1
2
3
6
7
8
13
14
15
19
20
9
21
Tetraploid Karyotype
92,XXYY
4
5
10
11
12
16
17
18
22
X
1
2
3
6
7
8
13
14
15
19
20
9
21
4
5
10
11
12
16
17
18
22
X
Y
Y
No increased risk for recurrence
33
Mechanism of Triploidy
Dispermy (66%) - most common cause
Meiotic failure (34%)
- in spermatogenesis (24%)
- in oogenesis (10%)
Paternal
large placenta
small fetus
partial hydatidiform mole
IUGR
oligohydramnios
congenital heart defect
syndactyly
Maternal
small placenta
large fetus
early loss
Numerical Chromosome Abnormalities
ƒ EUPLOID = exact multiple of haploid set
• Haploid = normal number in gametes (n=23 in humans)
• Diploid = normal number in zygote and somatic cells (2n=46)
• Polyploidy = complete set(s) of extra chromosomes
ƒ Triploidy = 3n = 69/cell in humans
ƒ Tetraploidy = 4n = 92/cell in humans
ƒ ANEUPLOID = loss/gain of single chromosomes
• Monosomy = chromosome number = 45
• Trisomy = chromosome number = 47
Gene Dosage Effects in Cytogenetic Disorders
•
Normal chromosome and gene copy number is two
(one paternal, one maternal)
- but can see Copy Number Variants (CNVs)
•
Whole chromosome aneuploidy (gain or loss) usually lethal
(except sex chromosomes)
•
Monosomy more severe/lethal than trisomy
(no viable autosomal monosomies)
34
Gene Dosage Effects in Cytogenetic Disorders
• Monosomy and deletions cause more severe phenotypic
consequences than trisomy and duplications
- No viable autosomal monosomies (only 45,X)
• Larger imbalances (more genes) more severe phenotype than
smaller imbalances
• Imbalance of G-negative bands (gene-rich) more severe than Gpositive bands (gene-poor)
(chr 13, 18, 21 – all gene poor)
Chromosome Abnormalities in Newborns
Abnormality
Birth Frequency
Trisomy 21
Trisomy 18
Trisomy 13
47,XXY (Klinefelter synd.)
47,XYY
47,XXX
45,X (Turner synd.)
1 in 800
1 in 6000
1 in 10000
1 in 1000 males
1 in 1000 males
1 in 1000 females
1 in 5000 females
Most chromosome abnormalities are prenatal lethals
35
Trisomy 21: 47,XX,+21
1
2
3
6
7
8
13
14
15
19
20
9
21
4
5
10
11
12
16
17
18
22
X
Y
Common Features
- Moderate ID
- Hypotonia
- Almond-shaped eyes,
epicanthal folds
- Depressed nasal bridge
- Brachycephaly (round head shape w/
flattened occiput)
- Excess nuchal skin
- Simian crease; clinodactyly
- Heart defect - 40% (most A-V canal)
~95% free trisomy; others translocation, mosaicism
Image from: ttp://2011gtms8f.wikispaces.com/trisomy+13,18,21+Diego+r
Trisomy 18: 47,XY,+18
1
2
3
6
7
8
13
14
15
19
20
9
21
4
5
10
11
12
16
17
18
22
Common Features
• Severe ID
• IUGR, microcephaly
• Weak, feeble activity and cry (hypoplastic
muscles)
• Clenched hand position w/ 2 & 5 over 3 & 4
• Clubfoot or rocker-bottom feet
• Small, low set ears
• Omphalocele
• Early lethality
• most die in first month;
5-10 % survive > 1 yr
X
Y
Images from:
http://library.med.utah.edu/WebPath/jpeg3/PERI228.jpg
medgen.genetics.utah.edu
Trisomy 13: 47,XY,+13
1
2
3
6
7
8
13
14
15
19
20
9
21
4
5
10
11
12
16
17
18
22
X
Common Features
• Severe ID
• IUGR, microcephaly,
• Midline anomalies
• cleft lip/palate,
holoprosencephaly, scalp
defect, CHD, omphalocele
• Polydactyly, postaxial
• Early lethality
• most die in first month;
5% survive > 6 months
Y
Image from: Medscaape.com
36
Structural Chromosome Abnormalities
Terminal
Deletion
Interstitial
Deletion
Duplication
Translocation
OR
Pericentric
Inversion
Ring
Isochromosome
Marker
Reciprocal translocations
Incidence: 1 in 500
Most common: t(11;22)(q23;q11)
Most balanced translocations have normal phenotype,
unless bkpt disrupts a gene or deletion occurs at bkpt.
If balanced de novo translocation in prenatal setting,
~5% risk of some developmental delay or other abnormality.
t(2;8)(q33;q24.1)
q24.1
der(8)
2
der(2)
8
der(2)
8
q33
der(8)
2
37
-----------Common-------------
Theoretical: 25% 25%
Empirical:
85-90%
-----Rare-----
25%
25%
10-15%
Reciprocal translocations
Meiosis:
Pairing of translocation products to form quadrivalents.
Major segregation products are:
alternate (normal and balanced)
adjacent 1 (two unbalanced forms).
Four products with theoretical risk of 50% unbalanced,
but empirical risk is 10-15% abnormal.
Adjacent 2 products are more rare, and have two identical
centromeres in each gamete (probably occurs less often).
They represent much greater imbalance (less viable).
Robertsonian Translocation
• Incidence: 1 in 1,000
• A translocation between two acrocentric chromosomes (13, 14, 15, 21, 22)
resulting in loss of the short arms of both chromosomes, but does not
affect the DNA content of the long arms. Usually dicentric.
• rob(13;14) most common
21
45,XY,der(14;21)(q10;q10)
OR
45,XY,rob(14;21)(q10;q10)
14
21
14
38
Robertsonian Translocation
In Meiosis, Robertsonian translocations form a trivalent configuration.
Alternate segregation produces roughly equal frequencies of normal
and balanced gametes.
Adjacent 1 segregation produces four potential gametes, although
both monosomies are lethal and trisomy 14 in this example is lethal.
Although T21 has a theoretical risk of 33% in this situation, the
empiric risk is 10-15% in female carriers and 0-2% in male carriers.
Risks for UPD – Robertsonian Translocations
ƒ Homologous and non-homologous Rob. translocation involving
chromosome 14 or 15;
ƒ Inherited or de novo
ƒ For prenatally identified non-homologous [e.g. rob(13q14q)], risk for
UPD is ~0.6%; for homologous risk is ~66%.
Shaffer et al., ACMG Statement for UPD
Genet Med. (2001), vol 3(3), pg 206-11
39
Inversions
A
B
C
D
A
C
B
D
A
B
C
D
E
A
D
C
B
E
per-i-centric
par-ACENTRIC
(acentric or dicentric products)
Neither stable, so risk for
abnormal offspring is low.
(duplication and deletion products)
Viability depends on size of
segments involved.
Risk for Abnormal Offspring:
Translocations vs. Pericentric Inversion Carriers
Translocations:
The larger the segments involved in the
translocation are, the less likely that an unbalanced
offspring will be liveborn.
Pericentric Inversions:
The larger the inversion, the more likely it is to to
produce unbalanced offspring.
Marker Chromosomes
Called a marker chromosome until the origin is identified, then referred
to as a derivative chromosome with the origin identified [e.g., der(3)].
Important to determine whether euchromatic material is present or not
to define phenotypic consequences.
Most common: chr15, chr22 (Cat Eye syndrome)
40
Acknowledgements
David Ledbetter, PhD, Katrin Leuer, PhD, and Fady
Mikhail, PhD, for some of the slides/content used in
this lecture.
Additional Slides for Your Reference
Banding techniques
*G-banding: Giemsa staining after trypsin
treatment. AT rich regions stain darkly (G+);
GC rich regions stain lightly (G-). By far most
used for clinical and research purposes.
Q-banding: Quinacrine fluorescence dye. AT
rich regions stain positive (fluoresce brightly);
GC rich regions stain negative (fluorescence
weakly)
R-banding: Reverse banding. Reverse pattern
to Q or G; used primarily to highlight
telomeric regions
41
Banding techniques
C-banding: Constitutive heterochromatin.
Heterochromatin stains darkly, euchromatin lightly
Ag-NOR Silver nitrate (AgNO3) stains proteins at sites
of active rDNA synthesis (the “stalks” on acrocentric
chromosomes)
Polymorphic regions of the human karyotype:
•Acrocentric stalks and satellites.
•Pericentromeric regions of 1, 9, and 16 vary
tremendously in amount of heterochromatin (Cband positive material) and large variants are
referred to as 1qh+, 9qh+, etc.
•Long arm of Y chromosome highly polymorphic.
Chromosome 14 Band Designation
p 1
1
2
1
1
1
2
q
3
2
1
1
3
3
4
2
2
1
3 2
2
3
1
2
1 3
1
.1
.2
.3
2
.1
.2
.3
14q32.3
.1
.2
.3
2 3
3
2
.1
.2
.3
.1
.2
.3
.1
.2
.3
.1
.2
.3
4
1
Band
Region
1
2
3
4
14q32
Arm
1
1
.1
.2
.3
.2
.11
.12
.13
.31
.32
.33
14q32.33
Cytogenetic Nomenclature
1) Number of chromosomes present
2) Sex chromosome composition
3) Descriptive characters of abnormalities
+
add
del
der
dic
dup
gain of a whole chromosome
loss of a whole chromosome
additional material of unknown origin
deletion
derivative chromosome
dicentric chromosome
duplication
42
Cytogenetic Nomenclature
inv
iso
mat
pat
mos
r
t
rob
inversion
isochromosome
maternal origin
paternal origin
mosaic
ring
translocation
Robertsonian translocation
Cytogenetic Nomenclature
45,XX,-9
45,X
47,XX,+13
47,XXY
46,XY,del(5)(q13q21)
46,XY,t(2;6)(p23;q32)
47,XY,t(3;7)(p21;p15),+22
Inversions
43
Paracentric Inversions
par-ACENTRIC (does NOT include the centromere; two
breaks in one arm). Cytogeneticists should be able to
draw pairing diagram and derive meiotic products
(noncyto should understand difference between repro
risks of para vs. peri-. Unbalanced products are acentric
or dicentric. Neither is stable, so risk for abnl. liveborn low.
Pericentric Inversions
nl
inv
dupA, dupD,
delD delA
Per-i-centric (i-ncludes the centromere in inversion segment;
one break in each arm). Each unbalanced product has a
duplication and a deficiency. Viability depends on the size of
the segments involved as well as what genes are involved.
Generally should be treated similarly to a balanced reciprocal
translocation in terms of risk of abnormal offspring.
Cytogenetics References: General
• Thompson & Thompson Genetics in Medicine
Nussbaum, McInnes & Willard
Elsevier, 7th ed., 2007; 8th edition out 06/15
Basic and clinical chapters
• Chromosome Abnormalities and Genetic Counseling
Gardner and Sutherland
Oxford University Press, 4th ed., 2011
Detailed mechanisms, segregation, recurrence risks Essential for all
clinicians, GCs, lab directors
44
Cytogenetics References: Advanced
• Human Genetics: Problems and Approaches
Vogel and Motulsky
Springer-Verlag, 4th ed., 2010
- Chapter 2 - Human Chromosomes Comprehensive chapter on
cytogenetics, including history, methods, *meiosis,
abnormalities, and clinical features.
• AGT Cytogenetics Laboratory Manual
Barch, Knutsen, and Spurbeck
Lippincott, 3rd ed., 1997
• Detailed laboratory methods and protocols.
45
46
Genomics - Basic
GENOMICS - BASIC
Madhuri Hegde, PhD, FACMG
Associate Professor
Emory Genetics Lab Scientific Director
Sr. Director Emory Genetics Lab, Molecular Lab
Madhuri Hegde, PhD, FACMG
Department of Human Genetics
Emory University School of Medicine
2165 North Decatur Road
Decatur, Georgia 30033
(404)727-5624 Telephone
(404)727-3949Fax
[email protected]
49
50
Genomics -Basics
Madhuri Hegde
Emory University School of Medicine
• The following relationship(s) exist related to this
presentation:
– Category of relationship – Advisor, Employment
– Name of commercial entity – Genzyme (Pompe
program), PTC (DMD program), Coriell Cell
Repositories, PerkinElmer Genetics Inc
Learning Objectives
• Mutations & their effects:
– transitions & PTCs
– Recombination & gene rearrangements
– Trinucleotide repeats & disease
• Methods of detecting genetic variation:
– Southern blotting, FISH, CGH, PCR & MPLA
– Polymorphisms, Linkage, LOH & MSI
– Sequencing (Sanger and Next Generation Sequencing)
– Gene discovery (Linkage analysis)
51
Gene Structure & Transcription
Promoter
Enhancer
Start Transcription
AUG: Start
Translation
UAG: Stop
Translation
4
Gene Structure: Exons & Introns
5
Gene Families & Pseudogenes
Expressed genes: transcribed & functional
Pseudogenes (ψ): non functional copies
Conventional ψ: not expressed & often
occur in gene families
Expressed ψ: non functional transcripts
6
52
Human Repetitive DNA Sequences
SINEs: short interspersed nuclear elements, ie
Alu (~300 bp, 106 copies); ~13% of genome)
LINEs: long interspersed nuclear elements, ie
LINE 1 (~800 bp, ~5x105 copies); LINES ~20%
of genome
7
Effects of Mutations
• Loss of function causes reduced activity or
product (hypomorphs or null alleles)
• Haploinsufficiency when ½ normal levels
result in phenotype
• Dominant negative in heterozygous state
causes loss of normal allele function, usually
from multimer formation
• Gain of function have increased expression
levels or product acquires a new function
ACMG Genetics Review Course June 4, 2011
Methods of Detecting Genetic Variation
Promoter
Enhancer
Start Transcription
AUG: Start
Translation
UAG: Stop
Translation
9
53
Properties of DNA
•Double stranded
•Denatures with heat
•Complementary basepairing
•Primers and enzymes to
replicate
•Large
•Negatively charged
DNA Isolation
Transfer
supernatent
Spin to collect
WBC pellet
RBC lysis
WBC lysis
RNAaseA
Proteinase K
Protein
DNA
precipitation precipitation
Procedures vary by sample type. Example – cultured cells
Original sample conditions are important! Example – freezing
Contents of blood collection tube are important! Example - heparin
The Known Versus the Unknown
One of a few mutations
account for a large proportion
of mutations in the population
Many private mutations or an
unknown spectrum of
mutations comprise mutant
alleles in the population
54
The Known: Targeted Mutation Testing
Benefits:
- Low cost
- Can be high throughput
- Pre-defined possible outcomes (Interpretation)
Limitation:
- Limited number of mutations detected
- Other changes will not be detected
- Other changes could interfere with the assay
Most Appropriate for:
- Carrier screening
- Population based mutation screening (NBS)
Methods to Detect Pathogenic Variants
Detection of specific mutations:
Restriction Enzyme digest
Allele-specific PCR
Allele-specific hybridization
Oligo ligation assay
Detection of any nucleotide change:
Full gene sequencing
Mutation scanning
- DHPLC
- SSCP
Detection of large copy number mutations:
Southern blotting
Quantitative PCR
MLPA
Array comparative genomic hybridization
Restriction Enzyme Digest
- Bacterial immune / defense system
- Enzymes that recognize and cut specific DNA sequences
- Commercially purified for laboratory use
- Systematic naming convention
Dde I – from Desulfovibrio desulfuricans
5’….CTNAG….3’
3’….GANTC….5’
HbA
55
Manipulation of Target DNA
Dde I
5’….CTNAG….3’
3’….GANTC….5’
http://www.larasig.com/node/4912
Restriction Digest
HbA
HbS
Not specific!
233
178
55
Amplification-Refractory Mutation System (Allele-specific PCR)
3’
5’
Extension with polymerase
A
C
G
T
G
T
T
G
G
T
3’
G
C
A
C
A
A
C
C
Normal
G
G
A
A
T
G
G
T
C
T
T
G
T
A
5’
Test for a C>T mutation at this site
3’
5’
A
C
G
T
G
T
T
G
A
T
3’
G
C
A
C
A
A
C
T
Mutant
G
G
A
A
T
G
G
T
C
T
T
G
T
A
5’
Test for a C>T mutation at this site
Basic Principle: The 3’ end of the PCR primer must perfectly match the template for
amplification to occur
56
Allele-Specific Oligonucleotide Hybridization (Dot Blot)
Basic Principle: Normal and mutant probes that differ by as little as a single basepair
mismatch are hybridized to test DNA under stringent conditions. A perfect match is
necessary for binding.
3’
5’
A
C
G
T
G
T
T
G
Genomic DNA hybridized to a membrane:
G C C T T
Normal
Heterozygote
Mutant
Normal Probe
3’
5’
A
C
G
T
G
T
T
G
A C C T T
Mutant Probe
Oligo Ligation Assay
Probe B
Probe A - normal
Ligation
Probe A - mutant
Template DNA
Labeled PCR primers specific to Probe A and Probe B are used to amplify
ligated probes. Ligation and amplification occur only if the 3’ end of probe
exactly matches template DNA.
Normal and mutant probes can be differentiated by length.
Can be multiplexed (MLPA – Multiplex Ligation-dependent Probe
Amplification)
Can be quantitative.
Methods to Detect Mutations
Detection of specific mutations:
Restriction Enzyme digest
Allele-specific PCR
Allele-specific hybridization
Oligo ligation assay
Detection of any nucleotide change:
Full gene sequencing
Mutation scanning
- DHPLC
- SSCP
Detection of large copy number mutations:
Southern blotting
Quantitative PCR
MLPA
Array comparative genomic hybridation
57
Sanger Sequencing
PCR region of
interest
Separate by
capillary
electrophoresis
Perform two sequencing reactions: one with a
forward primer, one with a reverse primer
Sanger Sequencing
Raw sequence:
Sequence assembled, analyzed, and displayed by software:
Reference for
regions of
interest
Reference
sequence
Forward
sequence
Software comparison of
reference and patient
sequence
Reverse
sequence
Reference
sequence
58
Methods to Detect Mutations
Detection of specific mutations:
Restriction Enzyme digest
Allele-specific PCR
Allele-specific hybridization
Oligo ligation assay
Detection of any nucleotide change:
Full gene sequencing
Mutation scanning
- SSCP
- DHPLC
Detection of large copy number mutations:
Southern blotting
Quantitative PCR
MLPA
Array comparative genomic hybridation
Single Stand Confirmation Polymorphism Analysis
Denaturation of DNA
Heat
59
Heteroduplex Analysis
C
G
Allele 2
T
A
1. PCR
2. Melt
3. Reanneal
Basic Principle: Heteroduplex (mismatch-containing)
DNA fragments will have different melting properties
than homoduplex DNA
C
G
T
A
Homoduplex
Homoduplex
C
^
A
Heteroduplex
T
^
G
Heteroduplex
^
Allele 1
^
Person heterozygous for a
C/T mutation
Denaturing Gradient Gel Electrophoresis and variations
DHPLC
Melting curve analysis
Denaturing High-performance Liquid Chromatography (DHPLC)
•Heteroduplex DNA elutes
earlier than homoduplex
DNA
• Several denaturing
temperatures per
fragment must be tested
Fragment
detection
Melting Curve Analysis
60
Methods to Detect Mutations
Detection of specific mutations:
Restriction Enzyme digest
Allele-specific PCR
Allele-specific hybridization
Oligo ligation assay
Detection of any nucleotide change:
Full gene sequencing
Mutation scanning
- DHPLC
- SSPC
Detection of large copy number mutations:
Southern blotting
Quantitative PCR
MLPA
Array comparative genomic hybridation
Gene Deletion / Duplication Testing
Southern blotting:
Benefit:
- Technically simple, can be performed in most laboratories
Limitations: - High cost, labor intensive
- Low resolution: may miss small deletions or duplications
Quantitative PCR:
Benefits:
- Many small regions (exons) can be tested
- Accurate
Limitations: - Labor intensive
- Significant QA/QC investment
MLPA:
Benefits: - Many small regions (exons) can be tested
- Accurate
Limitations: - Labor intensive
- Significant QA/QC investment
- Single base pair changes can interfere with probe binding
Array-based technologies:
Benefits:
- Highly accurate
- High resolution
Limitations: - Technically sophisticated
- Significant platform investment required
Most appropriate for:
- Individuals in whom a mutation was not detected by sequencing
- Gene in which deletions and duplications are common
Southern Blot Analysis
EcoRV
Uncut
P - +
Mitochondrial DNA as probe
Brief protocol:
1.
2.
3.
4.
5.
6.
Digest genomic DNA with a restriction enzyme
Run on an agarose gel
Denature DNA to create single-stranded DNA
Transfer to a nitrocellulose membrane
Probe with a labeled gene-specific probe (100’s – 1000’s bp)
Visualize labeled blot
61
Quantitative PCR
F2
F3
Probe 1: Monitors PCR for
the gene tested for
deletions
Probe 2: Monitors PCR for
the gene serving as a
control
Brief protocol:
1. Amplify target sequences in presence of labeled probes for test and control genes.
2.
Fluorescence is monitored during each PCR cycle.
3.
Compare amplification curve to standard curve (as above) to determine starting
concentration of test and control template.
4.
If no deletion, test and control template will have the same starting concentration. If
one copy of the test template is deleted, the ration of test to control will be .5.
Multiplex Ligation-dependent Probe Amplification (MLPA)
Concepts:
-Binding of probes to known target
- Ligation (joining of perfect match probes)
- PCR amplification
www.MLPA.com
Gene Targeted Array CGH
Array CGH is a better way to detect deletions and duplications than older methods.
DMD deletion of exons 3 – 9 in a male
Each point represents a single probe
62
Detection of Abnormal Repeat Expansions
Repeat Expansion Disorders
5’UTR
Exon
Intron
3’UTR
Myotonic Dystrophy
Autosomal dominant with anticipation
CTG expansion in the 3’UTR of the DMPK gene
1:100,000, but as high as 1:10,000 in some populations (Japan, Iceland)
Mild
Classic
Congenital
Cataracts
Mild myotonia
Weakness
Myotonia
Cataracts
Balding
Cardiac arrhythmia
Others
Infantile hypotonia
Respiratory deficits
Intellectual disability
Classic signs present in adults
63
Myotonic Dystrophy – DMPK Gene
Phenotype
Normal
CTG Repeat
Size
5-34
Onset
Death
None
Average
Premutation
35-49
None
Average
Mild
50-150
20-70
60 – Average
Classical
100-1500
10-30
48-55
Congenital
1000->2000
Birth-10
Neonatal - 45
**Premutation – an allele that can expand to a full mutation in one generation
Myotonic Dystrophy - Anticipation
Mild
Classic
Congenital
Cataracts
Mild myotonia
Weakness
Myotonia
Cataracts
Balding
Cardiac arrhythmia
Others
Infantile hypotonia
Respiratory deficits
Intellectual disability
Classic signs present in adults
Myotonic Dystrophy – Anticipation
Large expansions occur in female germline.
Pedigree –
Monckton and Caskey Circulation. 1995; 91: 513-520.
64
Myotonic Dystrophy - Anticipation
Large expansions do not occur in male germline.
Myotonic Dystrophy – Allele Calculation
Myotonic Dystrophy – Normal alleles
Capillary electrophoresis
65
Myotonic Dystrophy – Normal alleles
Zoom in
Myotonic Dystrophy – Allele Calculation
Fragment size – non-repeat sequence
= Repeat size
3
Myotonic Dystrophy – Normal alleles
147
171
147 – 108 = 39 / 3 = 13 CTG repeats
171 – 108 = 63 / 3 = 21 CTG repeats
66
Myotonic Dystrophy – Abnormal alleles
28,000
0
Zoom in
1,000
0
123 – 108 = 15 / 3 = 5 CTG repeats
378 – 108 = 270 / 3 = 90 CTG repeats
Myotonic Dystrophy – Abnormal alleles
Phenotype
Normal
Premutation
CTG Repeat
Size
5-34
Death
None
Average
None
Average
Mild
50-150
20-70
60 – Average
Classical
100-1500
10-30
48-55
Congenital
1000->2000
Birth-10
Neonatal - 45
BglI
|
|
BamHI
________
Probe A
35-49
Onset
3.4kb
|
CTG repeat
|
BamHI
BglI
|
500bp
| poly
BglI
|
67
BglI
|
|
BamHI
________
Probe A
3.4kb
|
CTG repeat
|
BamHI
BglI
|
500bp
| poly
BglI
|
BglI
~ 5,510 bp
5510 – 3400 = 2110 / 3 =
~700 CTG repeats
~3400 bp
Myotonic Dystrophy – Abnormal alleles
Phenotype
Normal
CTG Repeat
Size
5-34
Premutation
35-49
Onset
Death
None
Average
None
Average
Mild
50-150
20-70
60 – Average
Classical
100-1500
10-30
48-55
Congenital
1000->2000
Birth-10
Neonatal - 45
PCR – precise, but limited in ability to amplify large alleles
Southern – imprecise, but detects large expansions
**Premutation – an allele that can expand to a full mutation in one generation
Huntington Disease
Autosomal dominant – one of the only true human autosomal dominant conditions
CAG expansion in the HTT (also known as IT-15 gene)
1:100,000 in most Western populations, as high as 15:100,000
World’s highest prevalence in Maracaibo region of Venezuela
Adult onset (most cases):
Progressive motor disability - chorea
Cognitive decline
Psychiatric disturbances
Personality changes
Depression
Juvenile onset (5-10% of cases):
Rigidity
Seizures
Progressive motor disability - chorea
Cognitive decline
Psychiatric disturbances
68
Huntington Disease
Juvenile HD >60
Well-defined repeat size / phenotype correlation – precision critical in sizing repeat
Alleles can expand or contract
Large expansions occur through paternal germline (stable paternal transmissions also occur)
Huntington Disease
Precision in the 35-40 CAG range is important due to repeat size / phenotype correlation
ALWAYS run 40 (or similar) CAG control
If potential for juvenile onset, Southern may be needed
109 – 46 = 63 / 3 = 21 CAG repeats
166 – 46 = 120 / 3 = 40 CAG repeats
Friedreich Ataxia
Autosomal recessive
GAA expansion in intron 1 of the FXN gene (98%) of alleles and rare mutations
1:25,000 – 1:50,000 in Europe, Middle East, India, North Africa, rare elsewhere
Carrier frequency: 1:60 – 1:100
Early onset (10-15 years old) slowly progressive ataxia
Dysarthria
Muscle weakness
Spasticity of lower limbs with loss of reflexes
Scoliosis
Bladder dysfunction
69
Friedreich Ataxia
Phenotype
GAA Repeat Size
Normal
5-33
Premutation
34-65 pure GAA
Borderline
44-66 pure GAA
Full penetrance
66-1700
(most 600-1200)
(GAGGAA)n and (GAAAGAA)n
may be stable
May be associated with reduced
penetrance
Friedreich Ataxia
Long range PCR
Southern blot
How would you add this to a large carrier screening panel?
Spinal and Bulbar Muscular Atrophy
(Kennedy Disease)
X-linked
CAG expansion in the AR gene – codes for the androgen receptor
1:150,000 males in Europe and Asia, not yet reported in other populations
Most common in Japan
Gradually progressive degeneration of lower motor neurons
- Proximal muscle weakness
- Muscle atrophy
- Fasciculations
Mild androgen insensitivity in affected males
- Gynecomastia
- Testicular atrophy
- Reduced fertility
70
Spinal and Bulbar Muscular Atrophy
Phenotype
CAG Repeat Size
Normal
> 34
Unknown /
questionable
Reduced penetrance
35
Never reported
36-37
Full penetrance
>38
Family history and clinical
presentation important
Up to at least 55
Fragile X Syndrome
Head circumference >50th %
Prominent forehead and jaw
Large ears (>2 st dev for age)
Macroorchidism (> 2 st dev for age)
Hyperextensible joints
Pectus excavatum
Pes planus
Mitral valve prolapse
Intellectual disability (IQ 20-60)
ADD / ADHD
Autistic behavior
Tactile defensive
Poor eye contact
Hand-flapping
Fragile X Mental Retardation 1 (FMR1)
CpG
Island
5’
X
Transcription
X
(CGG) n
~ 44 repeats
Translation
3’
Common
~44-54 repeats
~55-199 repeats
> 200 repeats, methylated
Male
Intermediate
Premutation
Full mutation
Female
1/25
1/12
1/800
1/250
1/4000
1/4000
Most common form of inherited intellectual disability.
S. Sherman
71
Fragile X syndrome and
associated disorders
Fragile X related
tremor/ataxia syndrome
(FXTAS)
58,30
75
62,29
90,31
70,30
>200
Ovarian
insufficiency
1% POF
15% POF
2 yrs
5 yrs
22 yrs
Fragile X syndrome
40
51
46
Age at menopause
S. Sherman
Fragile X – Allele sizing by PCR
CGG repeat size analysis
- In-house developed PCR and capillary electrophoresis
- Large alleles will be not amplified
Commercially available kit for amplification of full mutations
Fragile X – Allele sizing by PCR
Normal female
Normal male
72
Fragile X – Allele sizing by PCR
Abnormal male
Fragile X – Assessment of methylation by msPCR
Neg male
Pos male
Neg female
271 bp unmethylated
allele
223 bp methylated
allele
Bisulfite treat DNA
Amplify with methylated and unlemtylated-specific primers
Validated to ~5% methylated – can detect methylation mosaics
Fragile X – Southern Blot
EcoRI
|
2.7kb
Probe
Patient
Male
Normal
2.7
Female
2.7
5.2
XhoI*
|
2.4kb
EcoRI
|
|
CGG repeat
Premutation
>2.7
2.7 + >2.7
5.2 + > 5.2
Full mutation*
>5.2
2.7 + Methylated* >5.2
5.2 + >5.2
* Xho I methylation sensitive
If Xho I does not cut EcoRI fragment is ~ 5.2kb plus length of CGG repeats
73
Fragile X – Southern Blot
Fragile X – Mosiacism
Mosiacism of large expansions not uncommon
Mosiacism of premutation and full mutation expansions is consistent with a
diagnosis of Fragile X syndrome
Gene identification
Familial aggregation
Twin and adoption studies
Is the trait
influenced by
genetic
factors?
Linkage analysis
Identifying
mutations that
determine trait
Identifying the
position of the
gene(s) in the
genome
Precise
location
Identifies candidate
genes in region
Specific gene
structure, function
and effect of
mutation
74
Gene Identification for Mendelian Traits
• These diseases usually involve a
rare, highly penetrant mutation
• Environmental component usually
minor
• They are often phenotypically
distinct, which reduces locus
heterogeneity
Where do we start looking?
Linkage Analysis of AD Trait: Microsatellite
1, 2
3, 4
1, 3
1, 4
2, 3
2, 4
1
2
3
4
Determines if DNA variations near gene of interest cosegregate with a phenotype in a family
Steps: 1) Get needed samples, 2) Is the study
informative? 3) Do you see co segregation? & 4) Infer
status (Allele 4 co segregates with the AD trait)
74
Microsatellite Instability (MSI) &
Mismatch Repair Failure in HNPCC
Blood
MSI
Normal
Tumor 1
Microsatellite
Instability
Tumor 2
Tumor 3
Change in
nucleotide
repeats
MSI
Tumor 4
75
75
Evolution of Sequencing
• 1st Gen: 2 reads (Forward and Reverse)
Sanger Sequencing Technique: ABI, Beckman
• 2nd Gen: Millions of reads
Next Generation Sequencers:
Roche 454, Junior
ABI SOLiD
Illumina (HiSeq and MiSeq)
• 3rd Gen: Single molecule sequencing, nanopore sequencing
Ion Torrent, Pacbio, The MinION
Dideoxy DNA Sequencing
Homozygous
Point Mutation
Heterozygous Heterozygous
Point Mutation Insertion/Deletion/Fusion
77
Multiple Options for Sequencing
NextGen
Targeted Panel
Exome
• Target Enrichment
• PCR based
• In Solution
• Microarray
• Target Enrichment
• Nimblegen SeqEZ
• Illumina Truseq
• Agilent SureSelect
• Sequencing
• Sequencing
Genome
• Random Shearing of
Genome
• No target
enrichment needed
• Sequencing
76
Next Generation Sequencing Process
Genomic DNA
Fragment
Randomly shear DNA + end
repair + size select
Enrichment
Amplification
Addition of adaptors
Modification of products
NextGen
Sequencing
Raw Data
Amplification
Enrichment
NGS Sequencing
Illumina – sequencing by synthesis
77
Interaction between panels and Exome/Genome
Informs design of panels
• Bioinformatics
• Phenotype Driven Analysis
Sequencing
panels
Exome
• Functional Analysis
Genome
• Gene Expression analysis
Clinical – 2nd Testing
Clinical – 2nd Testing
Drop in cost of sequencing
Technological advances- sequence once read often
Targeted panels/ exome / genome
Targeted panels
Exome
Genome
Smaller target region
Allows interrogation of entire coding
region of the genome(~92% coverage);
Exome is 1-2% of the genome
Targets the 85% of the genome; 15% are
drak regions (repeats, pesudogenes etc)
100X coverage (clinical grade)
100X coverage (clinical grade)
30X (clinical grade)
uniform coverage across all targeted
regions; 30-200 genes
Enhanced coverage only across disease
associated region (5300/22,000). All
22,000 genes sequenced but coverage
enhanced over disease assoc regions*(see
note)
Near uniform coverage across entire
genome (85% sequencable genome)
Difficult for detection of CNVs,
Trinucleotide repeats,
pseudogenes
Difficult for detection of CNVs, indels
Trinucleotide repeats,
Pseudogenes
Cannot detect deep intronic mutations
unless specifically targeted in the design
Can detect CNVs (both genomic and
intragenic), deep intronic mutations, and
indels
Difficult to detect Trinucleotide repeats,
Pseudogenes but new algorithms are
ebing developed
No incidental findings (Allelic changes)
Need to address incidental findings
Need to address incidental findings
Supplemental Study Slides
78
Components of the Human Genome
85
Transcription: Cis & Trans Acting Factors
CAAT box binds NFI & CBF transcription factors
& regulates the amount of transcription
GC (Sp1) modulates transcription with CAAT
TATA box binds TFIID & determines transcription
start site (not in all promoters)
Initiator (Inr) (PyA+1NT/APyPy) complements TATA
to localize transcription start site
Inr
86
Promoter Mutation That Affects Transcription
(Hemophilia B Leyden)
87
79
Alternative Transcription Start Sites
DNA
Transcription Start
Transcription Start
Site 1
Site 2
5㵭
2
B
1
A
G
1
A
2
B
3
C
3
C
3㵭
4
D
G
3
C
4
D
AAAAAAAA
4
D
AAAAAAAA
Different mRNAs are produced from
different transcription start sites
88
Processing & Translation
Promoter
Enhancer
Start Transcription
AUG: Start
Translation
UAG: Stop
Translation
89
Splicing Consensus Sequences
90
80
Splicing Mutation Causing Exon Skip
91
Alternative Splicing (AS) Can Give
Multiple Transcripts
92
Alternative Splicing of CFTR mRNAs
Regulated by IVS8 5T/7T
93
81
㵰Silent Changes㵱 Can Perturb SEs
• Splicing enhancers (SEs) can be in
exons (ESEs) or introns (ISEs)
• SEs bind Serine Rich (SR) proteins &
other factors to select splice sites
• ESEs can encode AAs(GAAGAA=
GluGlu) but also regulate splicing
• Synonymous substitutions (silent
mutations) in ESEs can encode the
same AA but derange splicing
94
ESE Mutation Causes Exon Skip
Growth Hormone Deficiency
I
1
1
2
?
?
2
3
4
&
6
5
II
A/A
A/G
A/A
1
3
2
III
A/G
IV
agGAAG
A G
A/G
A/A
1
2
A/G
A/G
A
95
Translation: Nonsense Mediated Decay (NMD)
• NMD degrades mRNAs
with premature termination
codons (PTCs)
• Destroys truncated proteins
from PTCs
• Efficiency of NMD lessens
as PTC moves 5㵭 to 3㵭
• No NMD for PTCs in last 50
bp of next to last, last exon,
or single exon genes
96
82
Translation: MicroRNAs (miRNAs)
• Short RNAs not
mRNAs
• Hundreds are known
• miRNA (~20 nts long)
loads the RNA Induced
Silencing Complex
(RISC)
• RISC represses
mRNA translation
Translational
dsRNA
Small RNA
duplex
miRNA/
siRNA
RISC
siRNA
function
mRNA
97
destruction
repression
Translation: MicroRNAs (miRNAs)
MicroRNA Expression in
Malignancy
• miRNAs are
important in
regulating gene
expression in
development
• miRNAs are
also biomarkers
of malignancies
98
Translation: mRNA Editing
Apo B gene
CAA
TAA
5’
3’
CAA > UAA editing
No editing
Apo B mRNA
CAA
UAA
UAA
Apo B100 protein- Liver
UAA
Apo B48 protein- Intestine
• ApoB100 isn㵭t edited in liver
• ApoB48 translated in small intestine
from edited ApoB100 mRNA
• Editing changes CAA > UAA (Term) &
this converts ApoB100 to ApoB48
mRNA without changing the gene
99
83
Silent (Synonymous) Mutations
mRNA AUG GAA GCU AGU
Protein Met Glu Ala Ser
mRNA AUG GAG GCU AGU
Protein Met Glu Ala Ser
Don’t alter an amino acid ie Glu109Glu
Be Careful these may create splice sites,
affect ESEs, or alter mRNA secondary
structure (Science 314: 1930, 06)
100
Missense (Nonsynonymous) Mutations
mRNA AUG GAA GCU AGU
Protein Met Glu Ala Ser
mRNA AUG GAC GCU AGU
Protein Met Asp Ala Ser
Changes codon base 1: almost always, 2:
always & 3: sometimes causes missense
Missense changes amino acid ie Glu10Asp
Be Careful the amino acid change may be
harmful or neutral
101
Nonsense (Termination) Mutations
mRNA AUG GAA GCU AGU
Protein Met Glu Ala Ser
mRNA AUG TAA GCU AGU
Protein Met Stop Ala Ser
Changes an amino acid to premature
termination codon (PTC) ie Glu132Ter
RT -PCR
Be careful PTCs can cause
1
2
exon skips, encode truncated
1 WT, 2 Missense & 3 PTC
proteins, or trigger NMD
102
84
Frameshift Mutations
mRNA AUG GAA GCU AGU
Protein Met Glu Ala Ser
mRNA AUG
Protein Met
AGC UAG
Ser Stop
In/del of anything but (3)N bases alters
reading frame & amino acid sequence
ie 2 bp Del, 185GA
Be Careful frameshift may not produce
truncated protein if NMD occurs
103
Splicing Mutations
• Can destroy/create
splice site, ESE/ISE
& cause exon skips,
activate cryptic splice
sites, change
alternative splicing or
cause read through
• Be careful difficult to
predict the effects of
mutations on mRNA
splicing & translation
104
Hb E Mutation: Splicing & Missense
105
85
Transitions = Pu
Purine
Pu or Py Py Transversions
= Pu Py
Transitions
Adenine
AdenineTransitions
Guanine
Guanine
Transversions
Transversions
Cytosine
Pyrimidine Cytosine
ACMG Genetics Review Course June 4, 2011
Uracil
Uracil
Transitions
Thymine
Thymine
Methylation of CGs Causes
C to T & G to A Transitions
107
Repetitive Sequences & Recombination
ACMG Genetics Review Course June 4, 2011
86
How do Trinucleotide Repeat Expansions
(TNREs) Cause Disease?
AUG
(CGG)n
TAA
(GAA)n
(CAG)n
(CTG)n
Coding TNREs (ie HD) can cause polyglutamine runs
that affect protein function
Non coding TNREs (ie FXS) can affect gene
expression by triggering methylation
TNREs are associated with anticipation in pedigrees
ACMG Genetics Review Course June 4, 2011
Slipped Mispairing Causes TNR Expansions
ACMG Genetics Review Course June 4, 2011
Properties of DNA
•Double stranded
•Denatures with heat
•Complementary basepairing
•Primers and enzymes to
replicate
•Large
•Negatively charged
87
Ways to Visualize DNA
-
+
Basic concept: separation based on chemical and physical properties
- Negative DNA migrates toward positive cathode
- Smaller pieces get through gel faster
Ways to Visualize DNA – Dyes and Labeling
EtBr – ethidium bromide
DNA run on an agarose gel containing
EtBr shown under and UV lamp
*intercalating agent: reversible inclusion of a molecule between two other molecules
Ways to Visualize DNA – Dyes and Labeling
23 kb
2 kb
Genomic DNA – large pieces of DNA
88
Regions of Interest
(Exploiting) Complementary Basepairing
5’
3’
A C G T G T T G G
replication / extension
primer / probe / oligonucleotide binding
T G C A C A A C C G G A A T G G T C T T G T A
3’
5’
Enzyme - protein that catalyzes chemical reactions of other substances
without itself being destroyed or altered upon completion of the reactions.
Enzymes Used in Molecular Biology
Enzyme - protein that catalyzes chemical reactions of other substances
without itself being destroyed or altered upon completion of the reactions.
Polymerase – used to replicate DNA
Restriction enzymes – used to cut DNA at specific sequences
Ligase – Joins two pieces of DNA
89
Properties of DNA
•Double stranded
•Denatures with heat
•Complementary basepairing
•Primers and enzymes to
replicate
•Large
•Negatively charged
Ways to Visualize DNA of Interest
Amplification – polymerase chain reaction (PCR)
PCR video
Advantages
– amplification of area of interest
- design for specificity (18-25 bp)
Disadvantages
- variability in DNA can interfere with primer binding
- assay designs may not work as intended
- some regions of the genome are not unique
- relatively limited range (4 – 10 kb max)
- repetitive regions can interfere with amplification
Ways to Visualize DNA of Interest
1500 bp
1000 bp
500 bp
Resolution: 10’s to 10,000’s of basepairs (even whole chromosome)
Fast, relatively cheap, run and visualize any piece of DNA
90
Ways to Visualize DNA – Dyes and Labeling
Incorporation of fluorecently labeled primers
3’
5’
A
C
G
T
G
T
T
G
G
T
3’
G
C
A
C
A
A
C
C
G
G
A
A
T
G
G
T
C
3’
C
G
T
G
T
T
G
G
T
3’
G
C
A
C
A
A
C
C
G
G
A
A
T
G
T
G
G
T
C
T
T
A
5’
A
T
C
A
T
T
Incorporation of fluorecently labeled dNTPs
5’
C
A
G
T
G
T
A
5’
Ways to Visualize DNA – Dyes and Labeling
Capillary electrophoresis
Basic concept: separation based on chemical and physical properties
- Negative DNA migrates toward positive cathode
- Smaller pieces get through polymer faster
Resolution: 1 to 1,000’s of basepairs
Relatively expensive, run labeled DNA
Ways to Visualize DNA – Dyes and Labeling
Capillary electrophoresis
An array is a systematic arrangement of objects, usually in rows and columns.
91
Ways to Visualize DNA – Dyes and Labeling
smaller
bigger
Targeted Mutation Testing
The Known: Mutation Detection Methods
Restriction digestion of PCR-amplified DNA
Allele-specific amplification
Allele-specific hybridization
Allele-specific ligation
92
High-throughput screen for a common mutation
Factor V Leiden Thrombophilia
- Caused by a specific mutation in the F5 clotting factor gene (c.1691G>A (p.R506Q))
- Increased risk of venous thromboembolism and other thrombotic events
- Can be considered autosomal dominant or autosomal recessive:
Heterozygote: 3-8-fold increased DVT risk
Homozygotes: 9-80-fold increased DVT risk
- Carrier frequency: 1/12 – 1/33
-Prevalence of homozygotes: 1/5,000
- Lifetime penetrance for heterozygotes: ~10%
Melting curve analysis
of a PCR product with
an “extra” probe over
mutation site
The Known: Targeted Mutation Testing
Benefits:
- Low cost
- Can be high throughput
- Pre-defined possible outcomes
Limitation:
- Limited number of mutations detected
- Other changes will not be detected and could interfere with the assay
Most Appropriate for:
- Carrier screening
- Population based mutation screening (NBS)
93
Cystic Fibrosis
Most common genetic disease in the Caucasian population (~1 / 3,300)
Autosomal recessive inheritance
Dysfunction of ion transport in epithelial cells results in a multisystem
disorder
www.genetests.org
Moskowitz et al., (2008) Genet Med 10: 851-868
The Known: Cystic Fibrosis
CFTR gene mutations
ACMG Recommendations for CFTR Mutation Selection:
A carrier screening mutation panel should include:
- mutations present in > 0.1% of CF patient chromosomes
- only mutations associated with classic CF
ACMG Recommendation:
- 25 mutations recommended for carrier screening in 2001, revised to 23
mutation in 2004 based on additional data
- Addition of mutations may be appropriate in the future.
ACMG Practice Guidelines:
Grody et al., 2001 Genet Med 3: 149-153
Watson et al., 2004 Genet Med 6: 387-391
39 Mutation Panel
dF508*
dI507*
3120+1G>A* G85E*
R117H*
W1282X*
Y122X
R334W*
V520F
R347P*
R347H
A455E*
G542X*
S549R
S549N
A559T
G551D*
R553X*
R560T*
Y1092X
M1101K
R1162X*
S1255X
N1303K*
394delTT
621+1G>T*
711+1G>T*
1717-1G>A* 1898+1G>A* 1898+5G>T
2184delA*
2307insA
3849+10kbC>T*
1078delT
2183AA>G
2789+5G>A* 3659delC*
3876delA
3905insT
* ACMG recommended panel
94
c.443T>C (p.I148T)
- Rare variant
- Originally identified in individual with CF
- Eventually found to be more common in controls than CF patients
CFTR Mutation Detection Rate
Estimated detection rate
Racial/ethnic group
ACMG 23
39 mutation panel
Ashkenazi Jewish
94%
94%
Non-Hispanic Caucasian
88%
90%
Hispanic American
72%
74%
African American
65%
68%
Asian American
49%
49%
CFTR mutation detection in different populations
Racial/ethnic group
Detection rate
Ashkenazi Jewish
94%
Non-Hispanic Caucasian
90%
Hispanic American
74%
African American
68%
Asian American
49%
Percentage of CF patients
carrying at least one mutation
99.6%
99.0%
93.2%
89.8%
74.0%
95
Allele Specific Primer Extension
Wild type
Mutant
a
A
T
3’
3’
3’
G
C
3’
PCR-amplified
target DNA
b
Two universally-tagged
ASPE primers whose 3’
end defines the allele
3’
Tag 2
3’
T
c
Only correctly hybridized
primer will extend and
incorporated a biotin-dCTP
A
T
3’
Tag 1
B
A
T
3’
Tag 1
3’
C
3’
B
G
C
G
C
Tag 2
B
Allele Detection After Primer Extension
Mutant
Wildtype
Fluorescent signal
SA
PE
B
Tag 1
Anti-tag 1
ID read by machine
Bead 1
Tag 2
Anti-tag 2
Bead 2
Data from Allele Detection
96
CFTR Assay Positive Results
37 Out of 39 Mutations Observed
dF508*
dI507*
3120+1G>A* G85E*
R117H*
W1282X*
Y122X
R334W*
R347P*
R347H
A455E*
V520F
G542X*
S549R
S549N
A559T
G551D*
R553X*
R560T*
Y1092X
M1101K
R1162X*
S1255X
394delTT
621+1G>T* 711+1G>T*
N1303K*
1078delT
1717-1G>A* 1898+1G>A* 1898+5G>T 2183AA>G
2184delA*
2307insA
2789+5G>A* 3659delC*
3849+10kbC>T* 3876delA
3905insT
* ACMG recommended panel
Unexpected Results: p.R117H / p.R117H by NBS
Negative carrier screen
- mother’s report only
- test method unknown
- most likely a carrier panel
p.R117H / p.R117H by GA NBS
97
Unexpected Results: p.R117H / p.R117H at Screening
Negative carrier screen
p.R117H / p.R117H
Elevated IRT
- Large deletion (not detected by mutation panel or sequencing)
- Allele drop out (can occur with any PCR-based method)
- Sequence interfering with assay (can occur with any PCR-based method)
- Sample mix up
- Mom’s screening result incorrect
- Information about mom’s screening result incorrect
p.R117H / p.R117L
Reference:
Patient:
G
A/T
CGC – Arg
CAC – His
CTC – Leu
Unexpected Results: R117H / R117H at Screening
p.R117H
p.R117L
p.R117H / p.R117L
Elevated IRT
Positive sweat test
98
McArdle Disease
- A glycogen storage disorder (also known as GSD type V)
- Autosomal recessive, caused by mutations in the PYGM gene
- Characterized by myopathy, exercise intolerance, rapid fatigue, myalsia , muscle cramps
- Myoglobinuria can result in acute renal failure
- Mutations common in some populations
PYGM Mutation Detection Rate
Panel 1
p.R50X
Panel 2
p.R85X
p.G205S
p.F710del
European
50%
25%
10%
0%
Japanese
0%
0%
0%
65%
What percent of affected individuals of European decent would have at
least one mutation detected by panel 1?
What percent of affected individuals of Japanese decent would have at
least one mutation detected by panel 2?
PYGM Mutation Detection Rate
Panel 1
p.R50X
Panel 2
p.R85X
p.G205S
p.F710del
European
50%
25%
10%
0%
Japanese
0%
0%
0%
65%
What percent of affected individuals of European decent would have at
least one mutation detected by panel 1? 97.75%
What percent of affected individuals of Japanese decent would have at
least one mutation detected by panel 2? 87.75%
99
Populations with Common Mutations
Ashkenazi Jewish
Sephardic Jewish
North America:
Old Order Amish
Old Order Mennonite
Dutch-German Mennonite
Bethren in Christ
French Canadian
First Nations
Finland
The Netherlands
Iceland
Any isolated population
Common Mutations in the Ashkenzi Jewish Population
- 1 in 4 to 1 in 5 individuals carry at least one “common” mutation
-Guidelines from the ACMG (Gross et al., 2008 Genet Med 10: 54-56)
- 8 disorders
- Specific mutations
- Many panels offer more diseases and more mutations
- Some include Sephardic Jewish mutations as well
- Preconception / prenatal carrier screening
Common Mutations in the Other Populations
Larger carrier screening panels are being designed and offered
Not all common mutations are tested for in carrier screens – from Strauss and
Puffenberger (2009) about the Clinic for Special Children in Strausburg, PA:
Before the Clinic was established, Amish children with nemaline rod myopathy were
repeatedly subjected to invasive and costly interventions (e.g., muscle biopsies,
nerve conduction testing, electromyography, magnetic resonance imaging,
echocardiograms, etc.) that collectively cost the community millions of dollars. A
thoughtful physician informed with the right genetic diagnosis can help a family
determine appropriate limits of medical care while also protecting the child from
futile interventions and the common miseries of hunger, thirst, dyspnea, and pain.
100
Spinal muscular atrophy – technically difficult to screen for
- Severe neuromuscular disease caused by degeneration of the anterior motor neurons
- Severe progressive hypotonia, muscle weakness
- Three types:
Type I – severe, lethal in infancy
Type II – childhood form
Type III – mild
- Caused by mutations in the SMN1 gene: 95% deletions, 5% point mutations
- Carrier frequency:
1 in 35 Caucasian
1 in 41 Ashkenazi
1 in 53 Asian
1 in 66 African American
1 in 117 Hispanic
- Recommendation to offer carrier screening by ACMG, ACOG, and AMP
SMN1 – telomeric – active copy
SMN2 – centromeric – less active copy
SMN1 deletion detection by TaqMan assay
Fluorescent tag
Quencher
Amplify with fluorescent probe for SMN1 and a reference gene
** Add a non-fluorescent, non-hydolysable probe specific to SMN2 - competes for binding
Sugarman et al., 2012 Eur J Hum Genet 20: 27-32
101
SMN1 and SMN2 Alleles
SMN1 Alleles
SMN2 Alleles
[1+0]
[1+1]
[2+0]
[2+1]
[1+0]
[1+1]
[1+2]
[1+3]
[2+0]
[2+2]
[2+3]
Etc.
Sugarman et al., 2012 Eur J Hum Genet 20: 27-32
Large scale population-based screening for SMN1 deletions: n= 68,471
Examples of differences by ethnicity:
1-copy
2-copies
>3-copies
Caucasian
~2.0%
~90.9%
~7.1%
African-American
~1.0%
~51.9%
~47.1%
2 copies: [0+2] OR [1+1]?
Sugarman et al., 2012 Eur J Hum Genet 20: 27-32
102
How Southern Blots Work
Palindrome
157
Southerns: TNR Expansion Size & Methylation
• Methylation sensitive
enzymes
2
2
• Separates active &
inactive FMR1 gene of
females
• Premutations (1) are
not methylated
• Methylation of full
mutations (2)
inactivate the FMR1
gene
1
158
DNA Methylation Silences FMR1 Expression
(CGG)
GENE
RNA
n
PROTEIN
Mild
Severe
Severe
GENE + METHYLATION
159
103
PCR of TNR: Huntington Chorea
Allelic
Drop out
160
Missed Deletions: Multiplex Ligation dependent
Probe Amplification (MLPA)
Sequence of cDNA
Exon 1
BMPR2 genes
Exon 3
MLPA Control
Exon 2 Del
ACMG Genetics Review Course June 4, 2011
Gene Fusions: Quantitative PCR of BCR/ABL
11 copies of BCR/ABL / 100,000 cells
BCR-ABL/BCR= 0.00012
8,421 copies of BCR/ABL / 100,000 cells
BCR-ABL/BCR= 0.11275
162
104
Types of Polymorphisms
Single nucleotide polymorphisms (SNPs):
substitution of a single base
Short tandem repeats (STRs): tandem bi, tri or
tetra nucleotide repeats such as (TG)n, (CAA)n
or (GATA)n (aka microsatellite markers)
Variable number of tandem repeats (VNTRs):
includes STRs but usually refers to unstable
minisatellites (repeats of 9-65 bases)
Restriction fragment length polymorphisms
(RFLPs): changing a restriction site or an
internal repeat or in/del alters fragment length
163
Single Nucleotide Polymorphism (SNP)
TGC/TGC
CGG/TAC
Exon 1
Intron
Exon 2
GTC/TGC
Intron
ACG/TAC
Exon 3
SNP: single base substitution (does not
include insertions or deletions)
Occur ~1300 bp so there are millions
Have a population frequency of at least 1%
SNPs may or may not affect gene function
ACMG Genetics Review Course June 4, 2011
The Unknown: Exome Sequencing
Benefits:
- Interrogates all* genes in the genome
- Uses sophisticated bioinformatics
- Highly automated data proccessing
Limitations:
-Difficult to rule out genes
- Amplification or capture required
- High cost for infrastructure
- Requires sophisticated bioinformatics
-Indel detection
- Gene discovery in a clinical setting?
-Identification of mutation (truncating vs.
missense) in new gene of unknown function
(e.g. biochemical)
- Reagent cost vs. cost of the test
- Complicated interpretation and reporting
- How will the reports be written?
105
Whole Exome Sequencing (WES)
Ng et al Nat Genet 42: 30-36, 2010
Filter
NS/SS/I
Fam 1
Fam 1+2
Fam 1+2+3
2,362
1,810
1,525
Not dbSNP129
53
25
21
Not HapMap8
46
7
4
Neither
9
1
1
Predict Damaging
1
0
0
DHODH: 10 missense & 1 bp del in 6 Kindreds
ACMG Genetics Review Course June 2-5, 2011
㵰DNA is Important!㵱 OT Avery 1944
167
106
Clinical Cytogenetics
CLINICAL CYTOGENETICS
Christa Lese Martin, PhD, FACMG
Geisinger Health System
Autism & Developmental Medicine Institute
Director and Senior Investigator
Christa Lese Martin, PhD, FACMG
Autism & Developmental Medicine Institute
Geisinger Health System
120 Hamm Drive, Suite 2A, M.C. 60-36
Lewisburg, PA 17837
(570) 522-9427 Telephone
(570) 522 9431 Fax
[email protected]
109
110
Clinical Cytogenetics
Christa Lese Martin, PhD, FACMG
Director and Professor
Autism & Developmental Medicine Institute, Geisinger Health System
Disclosure(s)
Employed by Geisinger Health System
Consultant for The Jackson Laboratory
Overview
•
•
•
•
Molecular Cytogenetics/genomics Techniques
Microdeletions/Microduplications Syndromes
Recurrent Genomic Disorders
X Chromosome Abnormalities
111
Clinical indications for cytogenetic analysis
Intellectual disability
Evolution of G-banding to Molecular Cytogenetics
G-band designation
(subjective)
7q34 (+/- a band)
vs.
Array mapping
(objective)
7q35 – q36.1
Karyotype courtesy of N.L. Chia
ISCN2009
G-banding resolution
Š Standard idiograms for 400, 550, and 850 band stages of
resolution per haploid genome
Š High-resolution (850-1000 band stage) usually requires cell
synchronization methods or the addition of chemical agents to
prevent chromosome condensation.
Š Smallest detectable imbalance (deletion, duplication) by Gbanding ~2-3 Mb
ƒ But, analytical sensitivity at 5 Mb probably only ~70%
112
Copy Number Variation (CNV)
• Class of mutation resulting from the loss (deletion)
or gain (duplication) of genomic material
• > 1 kb in size
• Recurrent – common breakpoints mediated by
underlying mechanism, such as segmental
duplications (e.g., 16p11.2)
• Non-recurrent – variable breakpoints throughout
the genome
CNVs can be observed in normal populations
or cause disease
Human Disease
Normal Individuals
• Common cause of normal variation
• Identified in ~35% of human genome
(Iafrate et al., 2004)
• In general:
– Smaller in size
– Contain fewer genes
– Highly variable regions (e.g.,
pericentromeric DNA, segmental
duplications)
– Often inherited
• One of most common causes of human
disease
• Diagnostic yield of 10-20% in DD, ID, ASD,
birth defects
• In general:
– Larger in size
– Contain more genes
– Located in unique regions of the
genome
– Often de novo
Cataloging CNVs in Shared Databases
Normal Variation
DGV
Database of Genomic Variants
dbVar
Database of Genomic Structural Variation
Human Disease
ClinVar
DECIPHER
Database of Chromosomal Imbalance
and Phenotype in Humans Using
Ensembl Resources
OMIM
Online Mendelian Inheritance in Man
113
Pathogenic vs. Benign Imbalances
1. Evidence from literature/databases
- known del/dup or Mendelian disorders
OMIM, DECIPHER, ClinGen
- known CNV in normal population
DGV, dbVar
- comparison with patient population
data, case reports
PubMed, DECIPHER, ClinGen/ClinVar
2. Genomic/Gene Content
- correlates with size and location
UCSC, Ensembl
3. Inherited or de novo
ACMG Guidelines for CNV interpretation
• PATHOGENIC
- Reported as pathogenic in multiple
publications/databases, rarely identified in controls,
and/or high genic content
• UNCERTAIN - LIKELY PATHOGENIC
• UNCERTAIN CLINICAL SIGNIFICANCE
• UNCERTAIN - LIKELY BENIGN
• BENIGN
- Reported in multiple publications/databases as
benign or is a known polymorphism
Kearney et al. (2011) Genet Med
Methods for Copy Number Detection Genome Size and Resolution
Human genome
Chromosome
Metaphase band
(500 bands)
Prometaphase band
(1,000 bands)
FISH resolution
array/PCR/MLPA
Mb
3,000
kb
3,000,000
150
150,000
# Genes
~20,000
1,000
6
6,000
50
3
3,000
25
20-100
<1-100
1
1
114
Copy Number Tests
G-banding
+
+
+
+/-
+
-
-
-
Array
CGH
+
-
-
+
-
-
-
+/-
SNP Array
+
+/-
-
+
-
+
+
+/-
Exon-level
Arrays
-
-
-
-
-
-
-
+
Confirmation Studies
Š Different methods used to confirm copy number array results include:
ƒ FISH
ƒ Q-PCR (quantitative-PCR)
ƒ MLPA (multiplex ligation-dependent probe amplification)
Š Only FISH can determine the mechanism of the imbalance – important for
recurrence risk
FISH Analysis
1
2
3
6
7
8
13
14
15
19
20
9
21
4
5
10
11
12
16
17
18
22
X
Utilizes a DNA probe specific
to a targeted chromosomal region
(e.g., 22q11.2)
Probe binds to complementary DNA
in the cell
Visualized by fluorescent tag
attached to probe
Y
115
FISH
(Fluorescence In Situ Hybridization)
Patient
Cells
G
A
T
T
Patient
Cells
Hybridize
Probe/clone
DNA
Denature
probe DNA
ds
ssDNA
Denature
target DNA
FISH Probes
Whole
Chromosome
“Painting”
Probe
Centromere
Probe
LocusSpecific
Probe
Metaphase FISH
116
Interphase FISH
BENEFITS:
Cells do NOT need to be cultured
Metaphase cells not necessary
LSI 13
LSI 21
CEP 18
CEP X
CEP Y
FISH Analysis
1
2
3
6
7
8
13
14
15
19
20
9
21
4
5
10
11
12
16
17
18
22
X
Resolution:
Size of probe (~100 kb); but
not equal across entire genome
Y
Requires at least 500-600 evenly spaced DNA
probes to match the power of the karyotype!!!
Human Telomere Clone Set
cen
gene rich
3 -20 kb
100 - 300 kb
Unique Telomere Probe
Subtel. Repeats
(TTAGGG) n
117
Telomere FISH Analysis
1ptel del
1p tel green
1q tel red
Most common: 1p del, 9q del, 22q del
Knight, Lese et al. (2000) AJHG
Unexplained Intellectual Disability
~2.5 - 5% clinically significant
60% of unbalanced translocations were inherited from
parent with balanced form of rearrangement
Biesecker (Am J Med Genet 2002 107:263-266)
Reviewed 14 studies – 1,718 patients with ID, 2-29% yield
Conclusion – G-banding alone is insufficient
Array-based
Copy Number Microarrays
Molecular cytogenetic method to detect copy number imbalances
(also called copy number variations, or CNVs)
2005 BAC arrays used clinically
2007 - genome-wide interrogation using oligonucleotide and SNP
arrays
Objective method compared to routine cytogenetic and FISH analyses
118
All microarrays are NOT created equal!
Depends on:
1) Purpose of Microarray:
Gene expression
Sequence changes
Copy number imbalances
2) Content of Microarray
cDNA
Single nucleotides
Pieces of DNA (genomic clones, oligonucleotides)
Targeted vs Whole Genome Array
Focus on specific areas
Telomeres
Centromeres
Microdeletion/duplication
regions
~400-2,000 BAC clones or
oligo probes
Analyze entire genome
Tiling path BAC arrays –
32,000 overlapping BACs
Oligo arrays – 44,000 to 2M
probes
SNP arrays – 500K to 2M
probes
Array-based CGH
Patient DNA
Genomic
Probes
Loss: ratio < 0.8
Normal: ratio 0.8 - 1.2
Gain: ratio > 1.2
Control DNA
Will not detect balanced chromosome rearrangements or
polyploidy (triploidy, tetraploidy)
119
FISH
Copy Number Arrays
Patient
Cells
Clone DNA
(1 probe)
Patient DNA
Microarray
with Genomic
Clones
(probes)
Control DNA
aCGH = hundreds of FISH probes
SNPs allow AOH and UPD to be detected
Future
Directions
• Copy Number Variants are now being called from whole exome
and whole genome sequencing data
• Identification of sequence and structural variants can be achieved
in one assay
Example Array Results
Normal
results
Chr 4
Trisomy 21
120
Targeted Coverage: PW/AS Region
arr 15q11.2q13.1(20,249,886-26,884,937)x1
PWS/AS deletion
Targeted Coverage: PW/AS Region
arr 15q11.2q13.1(20,249,886-26,884,937)x1
PWS/AS deletion
Atypical deletion
UBE3A
45 kb loss
Previous nl:
Methylation
UBE3A seq.
22q tel.
Understand
limitations/purpose
of testing
Unbalanced translocation between 5p and 17p
GAIN 5p
LOSS 17p
Arrays can identify imbalances,
but not determine mechanisms
121
Nomenclature for Reporting
arr 16p11.2(29,649,997-30,199,855)x1
array
cytogenetic
band
genomic
coordinates x1 = loss
(and genome x3 = gain
build)
Miller et al. (2010)
Array Types and Diagnostic Yield
Targeted
Low
Targeted with Backbone
Whole Genome
High
Higher resolution studies increase the yield:
Telomere FISH
+ 2-3% (over G-banding)
Targeted Array
+ 7-11%
Genomic Array
+ 9-20% compared to 2-3% by karyotype!
Miller et al. (2010)
122
Array Analysis Now First-Tier Cytogenetic Test
Microdeletion/microduplication syndromes
Complex phenotypes due to dosage imbalance of multiple, unrelated
genes which happen to be contiguous on chromosome. In some cases,
clinical syndrome defined before genetic basis known.
AKA
Contiguous gene syndromes
Segmental aneusomy syndromes
Genomic Disorders (subset mediated by segmental duplications –
seg dup)
Mechanisms include deletion, duplication, and
UPD = any deviation from normal, biparental inheritance.
Microdeletion/duplication Syndromes
Wolf-Hirschhorn
Cri-du-Chat
Williams
Langer-Giedion
Wilms tumor-aniridia (WAGR)
Beckwith-Wiedemann
Prader-Willi/Angelman
Smith-Magenis
Miller-Dieker
DiGeorge/VCFS
4p16.3
5p15
7q11.23
8q24
11p13
11p15
15q11-13
17p11.2
17p13.3
22q11.2
123
Microdeletion/duplication Syndromes
Mechanism
Wolf-Hirschhorn
Cri-du-Chat
Williams
Langer-Giedion
Wilms tumor-aniridia (WAGR)
Beckwith-Wiedemann
Prader-Willi/Angelman
Smith-Magenis
Miller-Dieker
DiGeorge/VCFS
4p16.3
5p15
7q11.23
8q24
11p13
11p15
15q11-13
17p11.2
17p13.3
22q11.2
non-rec
non-rec
seg dup
non-rec
non-rec
non-rec
seg dup
seg dup
non-rec
seg dup
Mechanisms of structural rearrangements
Recurrent rearrangements (e.g., microdeletions/duplications and
translocations) in human are mediated by large blocks of DNA sequence
homology (25-400 kb) with very high sequence identity distributed along
chromosomes.
Mispairing between non-allelic copies produces unequal crossing over and
deletion or duplication.
= Non-Allelic Homologous Recombination (NAHR)
“Segmental duplications” represent ~5% of total genomic sequence in human
Segmental duplications (Low Copy Repeats; Direct Repeats)
100-400 kb, >99% sequence identity
Unequal Recombination
(non-allelic homologous
recombination or NAHR)
Deletion
and
Duplication
Primary mechanism for common, recurring microdeletion/
microduplication syndromes in humans (Genomic Disorders)
124
Genomic Disorders in Humans
Disorder
Type
Size
Williams; autism
Location
7q11.2
del/dup
2 Mb
Duplicon
100 kb
PWS/AS; aut mat
15q11-q13
del/dup
4 Mb
450 kb
24 kb
CMT; HNPP
17p12
del/dup
1.5 Mb
SMS; PLS
17p11
del/dup
5 Mb
200 kb
NF 1
17q11.2
del
1.5 Mb
100 kb
DGS/VCF
22q11.2
del/dup
3 Mb
200 kb
Male infertility
Sotos
Yq
del
3.5 Mb
200 kb
5q35
del
2.2 Mb
140 kb
see review by Mefford and Eichler (2009) Curr Opin Genet Dev
del(22)(q11.2)
DiGeorge syndrome/VCFS:
~1/4,000 – most common mdel syndrome
Thymus hypo/aplasia
→ cellular immunodeficiency
Parathyroid hypo/aplasia → hypocalcemia
DD, ID
Cardiovascular:
Conotruncal heart defects, aortic arch defects
Dysmorphic features:
Micrognathia, ear anomalies, cleft palate, short
palpebral fissures, short upper lip
del(22)(q11.2)
Image from www.thelancet.com
Image from www.pediastaff.com
22q Foundation - www.22q.org
125
16p11.2 Deletions
• Most exhibit developmental and/or psychiatric disorders
(e.g., autism spectrum disorder, intellectual disability)
• Macrocephaly
• Obesity
• Seizures
Zufferey et al. (2012) JMG; Shinawi et al. (2010) JMG; Hanson et al. (2015) Biol Psych
16p11.2 Duplications
• Most exhibit developmental and/or psychiatric
disorders (e.g., intellectual disability, ADHD)
• Microcephaly
• Seizures
Zufferey et al. (2012) JMG; Shinawi et al. (2010) JMG; Hanson et al. (2015) Biol Psych
Miller-Dieker syndrome (MDS)
Š Lissencephaly, type I - complete agyria (absent gyri) or w/ limited
pachygyria (broad gyri)
ƒ severe ID, spasticity, seizures
Š Microcephaly, bitemporal narrowing, vertical furrows on forehead
Š prominent forehead, short nose, upturned nares, protuberant upper
lip, thin vermillion border, small jaw
Š Isolated lissencephaly sequence (ILS) - same or milder brain
malformation with normal or subtle facial features
126
17p13.3: Normal vs. Lissencephalic Brain
Image from Bill Dobyns
MDS and ILS genetics
del(17)p13.3 including the LIS1 gene (variable or random
breakpoints, no “hotspots”)
MDS:
visible cytogenetic deletion in ~50%
deletion by FISH in 100%
ILS:
normal high-resolution cytogenetics
deletion by FISH in 30-40%
point mutations found in LIS1 in ~40% non-deletion cases
LIS1 gene - b subunit of platelet activating factor acetylhydrolase,
brain isoform Ib
del(4p)
(Wolf-Hirschhorn syndrome)
IUGR, microcephaly, hypotonia, severe ID
Dysmorphic facial features
hypertelorism,prominent glabella, arched eyebrows,
nose broad or beaked,CL/P, short upper lip
Other
scalp defect, hypospadias, heart defect, seizures,
preauricular pit
Most de novo, 10-15% from balanced carrier parent
127
del(5p)
(Cri du Chat syndrome)
Cat-like cry in babies (hypoplastic larynx)
IUGR, microcephaly, hypotonia, ID
Hypertelorism, round face, epicanthal folds, downslanting
palpebral fissures, strabismus
Heart defect
Transverse palmar creases
Most de novo, ~15% from balanced carrier parent
Smith-Magenis syndrome
• First microdeletion syndrome with blocks of duplicated material
flanking deletion
• Reciprocal duplication product identified
• Three blocks of sequence, SMS-REPs, mapped to 17p11-12
• SMS-REPs are >200 kb in length
Recurrent Deletions
Deleted Region
Syndrome/Phenotype
1q21
TAR syndrome
1q21.1
ID/Microcephaly
3q29
3q29 deletion syndrome
5q35
Sotos syndrome
7q11.23
Williams syndrome
8p23.1
8p23.1 deletion syndrome
15q11.2-q13
PW/Angelman (BP1/2-3)
15q13.2-q13.3
ID/Epilepsy (BP4-5)
16p13.11
Autism/ID/Schizophrenia
16p11.2
Autism
17p12
HNPP
17p11.2
Smith-Magenis syndrome
17q12
Renal cysts and Diabetes; Autism/ID/SCZ
17q21.31
17q21 deletion syndrome
22q11.2
22q11.2 Deletion syndrome
128
Recurrent Duplications
Duplicated Region
Syndrome/Phenotype
1q21
TAR region
1q21.1
ID/Autism
3q29
Variable phenotype
5q35
Short stature, microcephaly
7q11.23
Autism
8p23.1
Variable phenotype
15q11.2-q13
Autism (BP1/2-3)
15q13.2-q13.3
Psychiatric disease (BP4-5)
16p13.11
Variable phenotype
16p11.2
Autism
17p12
CMT1A
17p11.2
Potocki-Lupski syndrome
17q12
Epilepsy
17q21.31
Behavioral problems
22q11.2
Variable phenotype; LD
Recurrent CNVs
Many show broad phenotypic presentation (some observed in
apparently normal individuals) due to incomplete penetrance and
variable expressivity
Duplications tend to have more variable phenotypes than deletions
Many being identified across multiple neurodevelopmental
disorders (ID, ASD, Schizophrenia)
Significant Phenotypic Variability in CNV disorders
(as in all chromosome disorders)
Lancet Neurol 2013
129
X chromosome abnormalities
Sex chromosome aneuploidy
Karyotype
Incidence
Name
45,X
(1/3000)
Turner syndrome
47,XXX
(1/1000)
Trisomy X
47,XXY
(1/1000)
Klinefelter syndrome
47,XYY
(1/1500)
47,XYY syndrome
X-Inactivation
There are many important genes on the X
chromosome…
So, how can males, with only one X chromosome, and
females, with two X chromosomes, not differ in the
products encoded by most of these genes???
Explained by X-inactivation resulting in dosage
compensation.
130
The Lyon Hypothesis - 1
In female somatic cells:
•One X chromosome is active;
•The second is inactive and remains
condensed and appears in interphase
cells as the Barr body.
Barr Body
Chromosome
Complement
45,X; 46,XY; 47,XYY
46,XX; 47,XXY; 48,XXYY
47,XXX; 48,XXXY; 49,XXXYY
48,XXXX; 49,XXXXY
49,XXXXX
Number of
Barr Bodies
0
1
2
3
4
# Barr Bodies = n-1, where n=# X chromosomes
The Lyon Hypothesis - 2
z
X-Inactivation occurs early in embryonic life.
~2 weeks after fertilization, at several hundred cell stage.
Note: The inactive X must become re-activated in the female’s germ line
so that each egg can receive an active X chromosome.
z
X-inactivation is random.
The inactive X may be either the paternal or the maternal X;
with a mix of cells, females are mosaics for the X
chromosome.
z
X-inactivation is clonal.
After one X chromosome has become inactivated in a cell,
all of that cell’s descendants have the same inactive X.
131
XX XX XX
Paternal X
Maternal X
XX XX XX
Random
Inactivation
XX XX XX
X
X
X
X X
X
Clonality
X X
X
X
X
X
X
X
X X
X
X X
X
X-Inactivation is Incomplete
Subject to Inactivation
Inactivation
Escape
Pseudoautosom
al
Genes
Carrel et al. (1999) PNAS 96:1440-1444
Copyright (1999) National Academy of Sciences, U.S.A.
X Chromosome Mechanism
XIST gene (X-inactivation specific transcript)
The XIST gene is located in the X inactivation center of Xq13 and
is transcribed only from the inactive X chromosome.
XIST mRNA transcripts are only detected in normal females, not
normal males.
But, the RNA transcript is not translated into a protein, rather it
remains in the nucleus and coats the inactive X chromosome,
which affects replication (inactive chromosome is late
replicating) and condensation.
132
Properties of the Inactive X chromosome
•
•
•
•
•
•
transcriptionally inactive
late replicating in the cell cycle
most genes inactivated
undergoes inactivation from a specific initiation region
stable and remains inactive
inactivation “reset” in oocytes
X Chromosome Inactivation
Inactive X chromosome forms Barr body, associated with nuclear membrane
If one normal X and one abnormal X chromosome (deleted, duplicated),
abnormal X will be inactive (by selection in most cases)
In balanced X;autosome translocations, the normal X is inactive
5-10% of normal females demonstrate extreme skewing of inactivation
pattern – “unfortunate Lyonization” or “skewed X-inactivation” for X-linked
disease gene.
Pseudoautosomal Regions (PAR)
X and Y chromosomes share TWO regions of homology which
undergo very high levels of genetic recombination
PAR1 is at distal Xp/Yp; obligatory cross-over in 2. 6 Mb region
required for proper pairing and segregation of X and Y
SRY is just proximal to PAR1 in Y specific region; unequal
recombination leads to XX males and XY females
PAR2 in distal Xq/Yq
133
Pseudoautosomal Region:
•Acts like an autosome;
•Exchanges with the PAR on Xp.
SRY gene
(Sex-determining Region on the Y chromosome)
•Expressed during embryonic development;
•Encodes product that interacts with other genes
to initiate development of undifferentiated
embryo into a male.
Heterochromatic Region
SRY
X
Y
SRY
= XX Male
= XY Female
Acknowledgements
David Ledbetter, PhD, and Fady Mikhail, PhD, for some
of the slides/content used in this lecture.
134
Clinical Molecular Genetics
CLINICAL MOLECULAR
GENETICS
Madhuri Hegde, PhD, FACMG
Associate Professor
Emory Genetics Lab Scientific Director
Sr. Director Emory Genetics Lab, Molecular Lab
Madhuri Hegde, PhD, FACMG
Department of Human Genetics
Emory University School of Medicine
2165 North Decatur Road
Decatur, Georgia 30033
(404)727-5624 Telephone
(404)727-3949Fax
[email protected]
137
138
Clinical Molecular Genetics
Madhuri Hegde
Emory University School of Medicine
භ The following relationship(s) exist related to this
presentation:
භ Category of relationship – Advisor, Employment
භ Name of commercial entity – Genzyme (Pompe
program), PTC (DMD program), Coriell Cell
Repositories, PerkinElmer Genetics Inc
Learning Objectives
භ Describe testing methods for types of mutations, limitations,
and how to interpret results for the following types of
disorders:
භ Mutation Nomenclature
භ Sequence interpretation
භ Sequencing as a clinical tool
භ Detection of different types of mutations
භ Test validation
භ Future
139
The Human Genome
¾ ~3 billion base pair locations
ƒ Gaps in sequence still exist
ƒ Exact position of a specific base changes with builds
ƒ Current build – Hg19
¾ Variation
ƒ Millions of single nucleotide polymorphisms (SNPs)
ƒ Copy number variants (CNVs)
¾ Nomenclature
ƒ Standard
ƒ Historic
Standard Nomenclature
¾ Use of standard nomenclature allows for the
precise, unambiguous identification of a
genomic position
¾ Citing a specific reference sequence is critical
for longterm understandability of results
¾ Necessary for targeted testing and proper
interpretation of family studies
Standard Nomenclature
¾ Recommendations by the Human Genome Variation Society
¾ DNA / RNA / protein identity
ƒ
ƒ
ƒ
ƒ
ƒ
Genomic denoted as “g.”
Coding denoted as “c.”
Mitochondrial denoted as “m.”
RNA denoted as “r.”
Protein denoted as “p.”
¾ Recommendations for
ƒ
ƒ
ƒ
ƒ
ƒ
Single basepair changes
Small deletions, duplications, insertions
Large rearrangments
Intronic changes
Nearly any scenario that has ever been reported
http:// www.hgvs.org/mutnomen/
140
Standard Nomenclature:
Recommendations by the Human Genome Variation Society
Nomenclature for Variation from Reference Sequence
Type of Variation
Genomic
cDNA
Protein
Missense change
g.5248232T>A
c.20A>T
p.Glu7Val or p.E7V
g.20763582C>A
c.139G>T
p.Glu47* or p.E47X
Deletion
g.117199646_
117199648delCTT
c.1521_1523delCTT
p.Phe508del or
p.F508del
Intronic change
g.103234177C>T
c.1315+1G>A
-
Nonsense change
¾ Specifying the reference sequence is critical
ƒ These changes are meaningless without a reference
ƒ Different references result in different correct ways to
describe a variant
http:// www.hgvs.org/mutnomen/
Standard Nomenclature
The most common sickle cell disease mutation
Reference
Hemoglobin S
Historic
HbS
dbSNP:rs334
dbSNP:rs77121243 (retired)
HBB: Glu6Val
NC_000011.9:g.5248232T>A
NP_000509.1:p.Glu7Val
NM_000518.4:c.20A>T
Standard
What is a mutation?
MedicineNet.com:
Mutation: A permanent change, a structural alteration, in the DNA or RNA. mu·ta·tion
The FreeDictionary.com:
n.
1. The act or process of being altered or changed.
2. An alteration or change, as in nature, form, or quality.
3. Genetics
a. A change of the DNA sequence within a gene or chromosome of an organism resulting in the
creation of a new character or trait not found in the parental type.
b. The process by which such a change occurs in a chromosome, either through an
alteration in the nucleotide sequence of the DNA coding for a gene or through a change in the
physical arrangement of a chromosome.
c. A mutant.
Pathogenic variant is now the
preferred term in clinical genetics
141
Mutation Nomenclature
භ Database reference: RefSeq (curated)
භ Largest transcript; NM_012654.3
භ Note numbering starts at 㵬start site㵭, usually ATG
භ Nucleotide substitution
භ g (genomic), c (coding), m (mitoDNA), r (RNA)
භ g.1162G>A
භ c.621+1G>T or IVS4+1G>T (intronic)
භ r.957a>u
භ Amino acid substitutions: p (protein) 1 or 3 letter code
භ p.R117H or p.Arg117His
භ Frameshifts: p.Arg83fs or p. Arg83SerfsX15
භ Deletions and insertions
භ p.F508del
භ c.6232_6236delATAAG
භ g.409_410insC
භ Report all variants to appropriate database
Human Genome Variation Society
Human Variome Project
www.hgvs.org
www.humanvariomeproject.org
Gene: Disease Evidence
Gene
Evidence
Disease
Variant
Function (LoF)
Spectrum
Effect
Penetrance
Expressivity
Does not change interpretation
Does not change disease causality
142
The Growing
Complexity…changing the
fundamentals
Many
genes/
one
disorder
One
gene/
many
disorders
One
gene/
one
disorder
One
gene/ AR,
AD
Somatic
(cancer)
One
gene/
unrelated
disorders
LGMD and Other Muscular Dystrophies
More than 25 LGMD types are linked to specific gene loci
Variable expressivity
143
Sequence variant interpretation
Pathogenic
Likely pathogenic
Unknown
Likely benign
Benign
Richards et al., ACMG Recommendations for standards for interpretation and reporting
of sequence variations: Revisions 2007. Genet Med 2008: 10(4): 294-300.
Maddalena et al., Technical standards and guidelines: molecular genetic testing for ultrarare disorders. Genet Med 2005: 10(8):571-583. Reviewed and Revised 2009.
The ACMG Laboratory Practice Committee Working Group, ACMG Recommendations for
Standards and Interpretation of Sequence Variants. Genet Med 2000: 2(5): 302-303.
Pathogenic
Likely pathogenic
Unknown
Likely benign
Benign
1. Variants predicted to result in the loss of protein function (may or may not have been previously reported in
patients with disease)
a. frameshift (an insertion or deletion that is not a multiple of 3 nucleotides)
b. nonsense (introduction of a premature stop codon)
c. splice junction (at positions +1,+2, -1 and -2 in an intron)
d. change in an initiation codon
e. change in the termination codon
2. Variants predicted to result in an amino acid replacement (missense) with one of the following conditions met:
a. variant demonstrated to result in reduced protein function (loss of function), or aberrant protein function
(gain of function) in an appropriate functional assay
b. common disease causing pathogenic variant in a specific population based on evidence in the literature
c. variant reported in multiple affected individuals and demonstrated to segregate with disease in multiple
families
3. Variants demonstrated to result in aberrant splicing in an appropriate functional assay (eg. intronic or silent)
http://genetics.emory.edu/egl/emvclass/EGLClassificationDefinitions.php
The Unknown: DMD Full Gene Sequencing
- Duchene / Becker muscular dystrophy
- X-linked
- 79 exons – amplify each exon by PCR and sequence
Example: exon 32
144
The Unknown: DMD Full Gene Sequencing
c.4405C>T (p.Q1469X)
Kabuki Syndrome
• Autosomal dominant
භ Characteristic facies
භ Long palpebral fissures
භ Lower lateral eyelid eversion
භ Dispersed lateral one-third
of eyebrows
භ Widely spaced teeth
භ Depressed nasal tip
භ Malformed / prominent ears
භ
භ
භ
භ
Skeletal anomalies
Dermatolyphic abnormalities
Mild to moderate intellectual disability
Postnatal growth deficiency
භ Caused by pathogenic variants in KMT2D
Adam, Hudgins Clin Genet 2004: 67: 209-219
Missense Variants
KMT2D: c.15536G>A (p.R5179H)
Control
Patient
Reported in Ng et al., (2010) in two patients, demonstrated de novo
Reported in Hannibal et al., (2011) in one additional individual
145
Pathogenic
Likely pathogenic
Unknown
Likely benign
Benign
1. Recessive conditions: (all the following conditions must be met)
a. diagnosis has been confirmed by biochemical testing or patient phenotype is specific for
disease
b. variant located on opposite chromosome from a known disease causing pathogenic or likely
pathogenic variant
c. variant occurs at an evolutionarily conserved nucleotide and/or amino acid
d. variant not present in dbSNP, EVS (Exome Variant Server) or other publically available
database at a frequency consistent with being a benign variant
2. Dominant conditions: (all of the following conditions must be met)
a. variant segregates with phenotype in the family being tested or
b. testing parental samples demonstrates that the variant occurred de novo
c. variant not present in dbSNP, EVS (Exome Variant Server) or other publically available
database at a frequency consistent with being a benign variant
Pathogenic
Likely pathogenic
Unknown
Likely benign
Benign
MSUD is caused by the inability to metabolize branched-chain amino acids
- 1 / 185,000 live births
- 1 / 175 live births in the Mennonite population
- Autosomal recessive
- Caused by mutations in one of three genes: BCKDHA, BCKDHB, DBT
c. 670G>T (p.E224X)
DBT:
c. 670G>T (p.E224X) – nonsense mutation
c.752T>C (p.V251A) – unknown
c.752T>C (p.V251A)
DBT:
c. 670G>T (p.E224X) – nonsense mutation
c.752T>C (p.V251A)– likely pathogenic
KMT2D: c.10740G>A (p.Q3580Q)
1 year old
Reference
Proband
Exon 38
Intron 38
146
KMT2D: c.10740G>A (p.Q3580)
Reference
Reference
Father
Mother
1 year old
Reference
Proband
Exon 38
Pathogenic
Likely pathogenic
Intron 38
Unknown
Likely benign
Benign
Likely benign variant (one of the following conditions must be met)
1. Variant found heterozygous (for dominant) or homozygous (for recessive) in
multiple unaffected individuals.
2. Variant found in cis with a pathogenic variant in multiple unrelated individuals
3. Variant found in an unaffected family member (for dominant)
Pathogenic
Likely pathogenic
Unknown
Likely benign
Benign
Benign variant (one of the following conditions must be met)*
1. Variant reported in dbSNP, EVS, locus specific databases or EGL database at a
population frequency higher than expected given the prevalence of the disease
and mode of inheritance.
2. Variant reported in a control population at a frequency inconsistent with being
causative of disease
3. Other evidence from published literature that indicates the variant has no effect
on function
*Benign variants are interpreted as described above and not reported in clinical reports.
A list of these variants is available upon request.
147
Benign variation GJB2
¾ The NHLBI GO Exome Sequencing Project
ƒ
ƒ
ƒ
Large scale next-generation sequencing project
Focus on heart, lung, and blood disorders
European and African American populations
¾ GJB2: c.-34C>T
European allele frequency:
African allele frequency:
0.08% (A=7 / G=8587)
23.38% (A=1030 / G=3376)
¾ GJB2: c.79G>A (p.V27I)
European allele frequency:
African allele frequency:
0.21% (A=18 / G=8582)
0.34% (A=15 / G=4391)
http://evs.gs.washington.edu/EVS/
Benign variation GJB2: c.79G>A (p.V27I)
1.2
Genotype frequency
1
0.163
0.186
0.8
0.395
0.6
1
0.442
1
0.4
0.2
0.442
0.372
HapMap-HCB
(86 alleles)
HapMap-JPT
(172 alleles)
A/A
G/A
G/G
0
HapMap-CEU
(120 alleles)
HapMap-YRI
(120 alleles)
Genotype frequencies of the c.79G>A (p.V27I) variant in HapMap populations. CEU - Utah Residents,
YRI - Yoruba in Ibadan, Nigeria, HCB - Han Chinese in Beijing, China, JPT - Japanese in Tokyo, Japan
http://www.ncbi.nlm.nih.gov/projects/SNP (rs2274084)
http://hapmap.ncbi.nlm.nih.gov/
Pathogenic
Likely pathogenic
Unknown
Likely benign
Benign
Variant of unknown clinical significance (VOUS) (if unable to classify the nucleotide change in one of
the four categories above, it will be classified as a VOUS)
a. not reported in HGMD, locus specific databases, published literature, dbSNP or EVS
b. reported in dbSNP or EVS, but at an allele frequency insufficient to rule out clinical significance
based on mode of inheritance and severity of disorder.
c. reported in a single individual with insufficient segregation and/or functional data
d. reported in a single individual with inadequate clinical information.
Note: In the presence of conflicting data the term of variant of uncertain significance may be used.
148
KMT2D: c.2428_2508del81 (p.810del27)
c.2428_2508del81 / +
c.2428_2508del81 / +
exon 10
c.1539
c.2797
exon 10
c.1539
c.2716
Rare variant vs. incomplete penetrance
Variant Proficiency- Programs
භ VITAL (Variant Interpretation Testing Across
Laboratories)
භ CAP NGS program PT; CAP Variant Assessment
Programභ
භ
භ
භ
CAP-accredited Molecular Pathology laboratories: 821
International Mol Path: 169
CAP-accredited NGS laboratories: 280
International NGS: 51
භ Train the Trainer (R25 grant; Richard Haspel; Beth
Deaconess) (Pathologist +Geneticist)
149
1 st Exome (2012)>Reanalysis;2013>2015)
Disorders Requiring Sequencing
භ Rare and ultra-rare disorders
භ Common inherited disorders as 2nd tier strategy if:
භ Targeted mutation panel (CFTR) fails to identify mutation
in affected individual
භ Point mutations are a less frequent type of mutation
(DMD)
භ Gene has many novel/nonrecurring mutations
භ Limitations: will not detect gross alterations
භ Scanning used to screen for mutations prior to
sequencing
භ dHPLC
භ HRM analysis
Clinical Sequencing
One or a few pathogenic
variants account for most
cases in a population
Targeted genotyping assays
Many private pathogenic variants
or an unknown spectrum of
mutations in the population
Full gene sequencing
150
The Known Versus the Unknown
Genetic Disorder
Unknown Gene(s)
Known Gene(s)
Single Gene
One or a few mutations
account for most cases in
a population
(The Known)
Single
gene
Multiple Genes
Many private mutations or an
unknown spectrum of
mutations in the population
(The Unknown)
Sequencing
panels
Exome
Genome
Clinical Information
භ 10-day-old male patient
භ
භ
භ
භ
භ
Passed away in 2009
Hypotonia
Polyhydramnios due to inability to swallow
Reduced fetal movement
Diagnosed with congenital muscular dystrophy in 3rd
trimester
භ type unknown
භ Mother is 38 years old
භ Now pregnant: 24 weeks
භ Reports some bleeding in 1st trimester and feeling a lot of
movement
භ Normal family history
151
Previous Testing
භ 46,XY (G-banded chromosome analysis)
භ Normal microarray
භ X-linked myotubular myopathy: MTM1 gene sequencing was normal
භ Mother was tested for myotonic dystrophy: 5 and 12 CTG repeats
භ SMA - normal
භ Muscle biopsy – was abnormal but non-diagnostic (increased connective
tissue suggestive of a congenital muscular dystrophy)
Patient’s Exome Stats
භ Total genes covered: 19,160
භ Total exons covered: 212,786 (66,474 HGMD)
භ Total low coverage: 6,842 HGMD exons (11%)
භ Total variants detected: 29,493
භ Total variants selected for confirmation: 11
භ True positive variants that match phenotype:
භ RYR1 gene: c.12612G>A p.W4204X + c.14416A>G p.N4806D
Pedigree
p.N4806D
p.W4204X
p
p.W4204X
p.N4806D
24 weeks
46,XY
Normal chromosomes via CVS
Normal microarray
152
RYR1 gene
භ Chr 19q13.3
භ 106 exons
භ Encodes Ryanodine receptor 1 protein
භ Form Ca2+ channels in skeletal muscle cells
භ Involved in muscle contraction
භ Mutations cause different diseases:
භ Malignant hyperthermia (AD)
භ Myopathies:
භ Central Core disease (AD>AR)
භ Multiminicore disease (AR<AD)
භ Congenital Fiber-type disproportion (AD)
Genomic Medicine: Exome, Whole Genome, Next
Gen Panels
භ
භ
භ
භ
භ
භ
භ
භ
භ
භ
භ
භ
භ
භ
The Geneticist is the Genomicist
The Exome is now
The Genome is still a work in progress to clinical
Clinically available
Standards & Guidelines in process
Panels for multigene disorders
Technology addresses single nucleotide changes and CNV
Informed consent issues
Filtering of data
Interpretation issues
Cloud computing
False Positives/False negatives
Cost effective?
Decision tree for when to test using this method: Strategy
Possibilities if a Sequence Variant is
Not Detected
භ Patient has a mutation in a different gene
භ Patient has a mutation type that is not detected by
the testing method due to allele drop-out
භ Large gene deletions/duplications require a
different test technology
භ
භ
භ
භ
භ
MLPA (multiplex ligation probe amplification)
Multiplex PCR & dosage
RT PCR, quantitative PCR, long-range PCR
FISH
Array technologies for CNV
භ Patient has a mutation in a regulatory region that
was not tested
153
Detection of Single base pair changes and small
insertions/deletions
භ Forward Allele-specific oligonucleotide (ASO)
භ Reverse dot blot hybridization (RDB)
භ Amplification Refractory Mutation System (ARMS)
භ Oligonucleotide Ligation Assay (OLA)
භ Fluorescence Resonance Energy Transfer (FRET)
භ Liquid Bead Array
භ End-point and real-time PCR analysis (TaqMan)
භ Melting curve analysis using FRET hybridization probes
Duchenne/Becker MD
භ X linked recessive: males affected, females carriers
භ One third of cases are new mutations
භ Two thirds of patients have carrier mothers
භ Incidence: 1 in 3000 male births
භ Female carriers can rarely be affected due to skewed Xinactivation
භ Extremely high new mutation rate ~ 10-4
භ Alu repeats in introns lead to del/dup
භ Dystrophin gene located on Xp21.2
භ Large size of gene - 79 exons >2 million bases
භ Deletion hot spots
භ Genetic Testing: Deletion/duplication analysis &
Sequencing
DMD/BMD: Deletion/Duplication
භ PCR-Multiplex analysis, qPCR, Southern, MLPA, Array
භ Assess presence or absence of exons in DMD:
භ 65% Deletions DMD; 85% in BMD
භ 5-10% Duplications; 6-10% BMD
භ Multiplex detects ~98% del/dup; MLPA,array: 100%
භ Use Reading frame rule to predict in-frame (milder BMD)
vs. out-of-frame (severe DMD)
භ “Out of frame” deletion predicts no functional dystrophin protein
and severe disease (DMD)
භ “In frame” deletion predicts some dystrophin protein may be
present, and less severe disease (BMD)
භ Deletions cannot be detected by sequence analysis
භ Sequencing to detect point mutations
භ DMD-25-30%
භ BMD-5-10%
154
Multiplex PCR Analysis
P1 = del exons 45-48
P2= del exons 48-51
P3= del exon 44
Assay detects both deletions &
duplications when done in
log phase
Thompson & Thompson
Genetics in Medicine, 6th Edition, Nussbaum, McInnes, Willard, eds., p228,
Copyright Elsevier (2004)
cDNA Probes Detect Exons in DMD Gene
1
2
3
4
5
6
7 Exons
47
52
• Scan entire gene for
deletions/duplications
• Carrier testing/dosage
48/50
1= Wild-type control
2= Del 49-50
3= Del 49-52
4= Del 47-52
5= Del 49-50
49
6= Del 47-50
48
7= Del 47-50
Tom Prior
51
Principle of MLPA
antisense
primer
labelled
sense primer
ligation point
㵰stuffer㵱 sequence
with variable length
• denaturation
• probe annealing (O/N)
• ligation
• PCR amplification (universal primers)
• separation by capillary electrophoresis
Courtesy, G. Pals
155
Principle of MLPA
antisense
primer
labelled
sense primer
ligation
stuffer sequence
with variable length
• denaturation
• probe annealing (O/N)
• ligation
• PCR amplification (universal primers)
• separation by capillary electrophoresis
Courtesy G. Pals
Principle of MLPA
antisense
primer
labelled
sense primer
PCR
stuffer sequence
with variable length
• denaturation
• probe annealing (O/N)
• ligation
• PCR amplification (universal primers)
• separation by capillary electrophoresis
Courtesy, G. Pals
MLPA
DMD: Female Carrier Test
M. Hegde
156
Gene Targeted Microarray
භ Gene centric design
භ 60K probes tiled on the array
භ Average spacing in coding region = 210 bp
භ Average spacing in intronic region = 25
bp
භ Length of probes ranges from 45-60
bases; isothermal Tm across array (Tm
determines length of probe)
භ CGH performed using same sex
controls
භ Array analyzed using manufacturer
software
භ Data masking feature - ability to extract
data for single gene
DMD 385k Male
Ex79
Ex1
del Ex17 – Ex44
del Ex48 – Ex52
dup Ex2 – Ex4
DMD/BMD Gene Deletion
•2 hot spot regions for
deletions
•Size of deletion does not
determine phenotype
•Exceptions exist
•Reading frame rule
correctly predicts >90%
case phenotypes
Thompson & Thompson
Genetics in Medicine, 6th Edition, Nussbaum, McInnes, Willard, eds., p227,
Copyright Elsevier (2004)
157
DMD/BMD Reading Frame Rule
EXON 1
Nucleotide triplets
encoding
functional amino
acids will be
shifted out of
reading frame (to
create an unstable,
non-functional
protein) by deletion
of exons
containing a total
number of
nucleotides not
divisible by 3
TAG CTA GCT
EXON 2
AGC TAG CT
EXON 3
T GGC
EXON 4
CTA GCT
NORMAL
DMD (out of frame)
BMD (in frame)
Germline (Gonadal) Mosaicism
භ Mother of affected male with known mutation
in dystrophin gene may not have mutation in
her somatic cells but may carry the mutation in
her germ cells
භ Germline (or gonadal) mosaic females may
have a second affected child
භ Risk is estimated at ~15%
භ Lab reports should indicate that a mother of an
affected male who has a negative test result for
the dystrophin mutation present in her son has
a risk of having another affected child.
Spinal Muscular Atrophy (SMA)
telSMN (survival motor neuron) is the primary SMA-determining gene
• SMN1
•5q12.2-13.3
•AR
• 1:6000 newborns
• Homozygous
deletion in SMA
Types I, II, & III
• Exons 7/8 show
differences between
SMN1 & SMN2
• Exon 7/8 deletion
• Assay is PCR &
RE digest
Courtesy T. Prior
158
SMA telSMN Deletion Analysis
Homozygous deletion of exon 7 in telSMN is
diagnostic of SMA
telSMN
cenSMN
1
2
3
4
5
Uncut ___________DraI ____________
Digests of Exon 7 PCR
Courtesy T. Prior
200ng DNA Template; 20 Cycles
smnT,smnC
0,3 (Affected)
Ratio (0, 0.9)
2,2 (NON-Carrier)
Ratio (0.68, 0.64)
1,1 (Carrier)
Ratio (0.36, 0.33)
1,2 (Carrier)
Ratio (0.36, 0.61)
Tom Prior
Limitations of SMA Carrier Test
භ Non Deletion Mutations
භ
Lack of Phenotypic Prediction
භ
De-novo Mutations
භ
2 copy Cis SMN1 Chromosomes
Tom Prior
159
ͳ
ʹ
‘ ƒ””‹‡”
ͳ
ͳ
ʹ
ʹ
ƒ””‹‡”ȋͳǦ‘’›Ȍ
ƒ””‹‡”ȋʹǦ‘’‹‡•Ȍ
Tom Prior
Prader Willi/Angelman Syndromes
භ Promoter region of SNRPN gene contains CpG islands
which are heavily methylated in the maternally-derived allele
and unmethylated in the paternally-derived allele
භ PW = only methylated (maternal) allele present
භ AS = only unmethylated (paternal) allele present
භ Test method: Methylation analysis
භ Restriction digest with methylation-sensitive RE and Southern using
SNRPN probe
භ Genomic DNA is treated with sodium bisulfite, converting cytosine to
uracil except where cytosine is methylated.
භ methylation-specific PCR
භ methylation-specific melting analysis
භ Detect 80% AS
භ Detects 99% PWS
PWS/AS: DNA Methylation Analysis
•
Patient 2
Patient 1
Normal Ctl
Deletions 15q12
AS deletion Maternal
PWS deletion paternal
Southern blot uses
methylation-sensitive
enzymes & 5㵭 SNRPN
probe
Other methods of detection
Deletion Ctl
AS
•
•
•
•
maternal
paternal
160
DNA Methylation Detected by Methylation
Specific PCR (MSP-PCR)
…GTCMeGATCMeGATCMeGTG…
…GTCGATCGATCGTG…
Bisulfite treatment
converts
unmethylated C
residues to U.
…GTCMeGATCMeGATCMeGTG…
ÅG CTAG CTAG CAC
…GTUGATUGATUGTG…
CTAGCTAGCACG
PCR
PCR primer
PCR primer
Product
E. Lyon & R. Mao
No product
Prader-Willi/Angelman Methylation
Methylation-Specific Melting Analysis
Lower Tm for unmethylated
(≈83°C)
= Angelman Syndrome
U
m
Higher Tm for methylated (≈87°C)
= Prader-Willi Syndrome
E. Lyon & R. Mao
The Mitochondrial Genome
භ Circular; 16,659 bp
භ Polymorphic, 0.3% variation
between individuals; 7-10x
mutation rate of nuclear genome;
limited DNA repair
භ 2-10 copies/mt; 100’s to
1000’s per cell
භ Unique genetic code
161
Genome Collaboration
NuDNA
MtDNA
>300 genes for
37 genes for
Resp chain proteins
mtDNA
Replication
Expression
Repair
Antioxidant defense
Fe homeostasis
13 Resp chain proteins
2 rRNAs
22 tRNAs
Import machinery
Adapted from SIMD NAMA
Mitochondrial OXPHOS System *
Totals
13
~ 77
*DiMauro & Schon, NEJM 348: 2656, 2003
Special issues with Mitochondria
භ Heteroplasmy is found in mito disorders
භ Not all sample types will have the same proportion of the variant
භ Blood for mito deletions
භ Urine has been found to be suitable for a number of mito point mutations
භ Muscle
භ Sample type depends on which mito disorder is being tested
භ Testing methods
භ
භ
භ
භ
PCR-based
Southern for deletions
Array-based
Next Gen
භ A larger proportion of variant is generally associated with disease
භ Unaffected individuals may have low levels of variant
භ When a variant is identified, recommend testing mother and siblings
162
Human Disorders Due to Mitochondrial
Mutations
භ Kearnes Sayre syndrome (KSS)
භ Pigmentary retinopathy, chronic progressive external
ophthalmoplegia (CPEO)
භ Leber hereditary optic neuropathy (LHON)
භ Mitochondrial myopathy, encephalopathy, lactic
acidosis, and stroke-like episodes (MELAS)
භ Myoclonic epilepsy with ragged red fibers (MERRF)
භ Deafness
භ Neuropathy, ataxia, retinitis pigmentosa (NARP)
භ Subacute necrotizing encephalomyelopathy with
neurogenic muscle weakness, ataxia, retinitis
pigmentosa (Leigh with NARP)
E. Lyon & R. Mao
Mitochondrial Mutations Associated with Disease
HV 1
HV 2
PH1
MELAS
3243A>G
PH2
LHON
14484T>C
PL
LHON
3460G>A
Areas
deleted in
KSS
LHON
11778G>A
MERRF
8344A>G
NARP
8393T>G
E. Lyon & R. Mao
Detection of KSS Mitochondrial
Deletion Mutation by Southern Blot
M M + +
PvuII U C U C
The restriction enzyme,
PvuII cuts once in the circular
mitochondrial DNA.
M = Mutant
Heteroplasmy)
+ = Normal
U = Uncut, No PvuII
C = Cut with PvuII
E. Lyon & R. Mao
16.6 kb (normal)
Deletion mutant
Autoradiogram
163
Detection of NARP Mitochondrial Point Mutation (ATPase
VI 8993 TĺC or G) by PCR-RFLP
U = Uncut, no MspI
C = Cut, with MspI
MspI U C U C U C
551 bp
The presence of
the mutation
creates an MspI
restriction
enzyme site in the
amplicon.
345 bp
206 bp
Mutation
present
E. Lyon & R. Mao
Agarose gel
Cancer Tests
භ Inherited
භ Screening tumor
භ Screening patient germline
භ Screening family
භ Algorithm of strategy
භ Somatic
භ Testing tumor
භ patient therapy
භ Patient prognosis
භ Difference: Complexities of tumor analysis
Hereditary Non-Polyposis Colorectal
Cancer (HNPCC): Lynch Syndrome
භ
භ
භ
භ
භ
භ
භ
භ
භ
Represents ~3% of patients with CRC
Can be confused with FAP – rule out first
Autosomal dominant inheritance
Family history is big clue (Amsterdam/Bethesda)
Due to mutations in mismatch repair genes (MMR)
භ MLH1, MSH2 Major
භ MSH6, PMS2 Minor
Tumors display Microsatellite instability (MSI)
IHC can identify which MMR protein to test
Testing done by sequencing (+/- scanning) and deletion
analysis
EGAPP recommends screening all newly diagnosed CRC
patients
164
Microsatellite Instability
95% of HNPCC tumors have MSI at multiple loci
10%–15% of sporadic tumors have MSI
Marker #1
Marker #2
Patient 1 Tumor
Patient 1 Normal
Patient 2 Tumor
Patient 2 Normal
Patient 1= MSI-High
Patient 2 = MSS -stable
S. Thibodeau
Immunohistochemistry
MMR Proteins
M. Troxell
BRAF & methylation for predicting sporadic vs
inherited CRC
භ Serine/threonine kinase
භ Part of Ras/Raf/MEK/MAP signal transduction
pathway
භ Oncogenic mutations in 66% metastatic melanoma,
36% papillary thyroid cancer and 10% colon cancer
භ Most BRAF mutations are V600E
භ BRAF V600E mutation affects methylation of MLH1
promoter and is nearly 100%
somatic
භ Absence of MLH1 protein by IHC in
the presence of BRAF V600E is
most likely somatic, not HNPCC
V600E
E. Lyon & C. Vaughn
165
FAP-AFAP-MAP – INDICATIONS FOR TESTING
ƒColorectal cancer diagnosed in an individual younger than 40 years of age
ƒColorectal cancer diagnosed in one or more first-degree relatives
ƒColorectal cancer diagnosed in two or more first- or second-degree relatives of any age
FAP/AFAP testing: 10-100 adenomatous polyps with
dominant inheritance
MAP testing: 10-100 adenomatous polyps with family
history suggestive of recessive inheritance
Test for APC mutations
(Point mutation, deletions/duplications)
Positive
Test for p.Y165C and p.G382D MutYH mutations
One heterozygous
mutation detected
Negative
Diagnosis of FAP/AFAP
•Test at-risk family members
•Offer genetic counseling
Patient may have APC
mutation not detected by
testing method – Consider
MAP testing (Important for
patients with no/weak FHx*
and fewer polyps)
Negative
MutYH gene sequencing
Biallelic mutations
detected
Diagnosis of MAP
•Offer testing for
offspring ( obligate
carriers) and
relatives
One heterozygous mutation detected
Biallelic mutations detected
Negative
•Patient is, at minimum, a carrier of MAP
•Patient may have MutYH mutation not detected
by testing method – Consider FAP testing
•Offer screening for relatives and\or genetic
counseling
Diagnosis of MAP
•Offer testing for offspring
(obligate carriers)
•Offer testing for relatives and
genetic counseling
• Screen based on FHx*
and clinical presentation
• Consider FAP testing
*FHx: Family History
Lynch Syndrome- INDICATIONS FOR TESTING
ƒColorectal cancer diagnosed in all newly diagnosed CRC*
ƒSynchronous, or metachronous, colorectal, or other HNPCC-associated tumors
ƒColorectal cancer diagnosed in one or more first-degree relatives with an Lynch-related tumor, with one diagnosed before 50
years of age
ƒColorectal cancer diagnosed in two or more relatives of regardless of age
*see EGAPP recommendation3,4
Microsatellite instability and Immunohistochemical analysis
Recommend performing both tests when there is a high index of suspicion for LS*
MSS or MSI-L
No loss of proteins on IHC
MSI-H and loss of expression of MLH1 or PMS2 only or
both MLH1/ PMS2 based on IHC
Probably not Lynch Syndrome
Test for MLH1 promoter methylation and
BRAF p.V600E mutation
MSI-H and loss of expression of
MSH2 or MSH6 only or
MSH2/MSH6
based on IHC
Test for MSH2 gene mutations
No Mutation detected
Hyper-methylation of
MLH1 and BRAF
mutation detected
CRC not due to
MMR defect
Hypermethylation of
MLH1 and no BRAF
mutation detected
Hypermethylation
absent in normal
tissue: CRC not due to
MMR defect
Normal methlyation of
MLH1 and no BRAF
mutation detected
Hypermethylation
present in normal
tissue: Inherited
epimutation
No mutation detected
Test for EpCAM
Deletion
Mutation detected
Lynch Syndrome:
Test at-risk family
members
No Mutation detected
Test for MSH6 gene mutations
Test MLH1
gene
mutations
No mutation detected
Mutation detected
Test PMS2 gene mutations
Lynch Syndrome
with unidentified
mutation
No mutation
detected
Mutation
detected
Lynch Syndrome:
Test at-risk family
members
Lynch Syndrome with
unidentified mutation
භGene-targeted therapies
භBCR/ABL1 : imatinib, dasatinib, nilotinib
භPML/RARα : ATRA, arsenic
භHER2/neu : trastuzumab
භEGFR : erlotinib, getfitinib
භKRAS : panitumumab, cetuximab
Mutations may predict drug response
or absence of response.
Barbara Zehnbauer
166
Gene
Drug
CYP2C9
warfarin
TrimGen Corporation eQPCR LC Warfarin Genotyping
PGx Diagnostic Test
CYP2C9*2, CYP2C9*3
VKORC1
warfarin
TrimGen Corporation eQPCR LC Warfarin Genotyping
VKORC1:G-1639A
CYP2C19
clopidogrel,
esomeprazole,
omeprazole,
phenytoin,
others
Infiniti CYP450 2C19
CYP2C19*2, CYP2C19*3,
CYP2C19*4, CYP2C19*5,
CYP2C19*6, CYP2C19*7,
CYP2C19*8, CYP2C19*9,
CYP2C19*10
CYP2D6
codeine,
fluoxetine,
metropolol,
risperidone,
tamoxifen,
others
Roche AmpliChip
Cytochrome P450
Genotyping test and
Affymetrix GeneChip
Microarray Instrumentation
System
CYP2D6*1, CYP2D6*2ABD,
CYP2D6*3, CYP2D6*4ABDJK,
CYP2D6*5, CYP2D6*6ABC,
CYP2D6*7, CYP2D6*8, CYP2D6*9,
CYP2D6*10AB, CYP2D6*11,
CYP2D6*15, CYP2D6*17,
CYP2D6*19, CYP2D6*20,
CYP2D6*29, CYP2D6*35,
CYP2D6*36, CYP2D6*40,
CYP2D6*41, CYP2D6*1XN,
CYP2D6*2XN, CYP2D6*4XN,
CYP2D6*10XN, CYP2D6*17XN,
CYP2D6*35XN, CYP2D6*41XN
UGT1A1
irinotecan
Invader UGT1A1 Molecular
Assay
Variants Assayed
UGT1A1*28
Test Sensitivity & specificity
භ Analytic validity: ability to accurately measure a specific analyte, or
identify a mutation of interest in the sample type(s)
භ Analytic sensitivity: proportion of samples that have a positive test
result and that are correctly classified as positive
භ Analytic specificity: proportion of samples that have a negative test
result and that are correctly classified as negative.
භ Clinical validity: ability to accurately identify individuals who have (or
will develop) the disorder or phenotype of interest.
භ Clinical sensitivity: proportion of individuals who have (or will develop)
the phenotype of interest and who have a positive test result.
භ Clinical specificity: proportion of all unaffected individuals identified by
the proposed test as being negative
Predictive Value of Genetic Testing
භ The positive and negative predictive values of
testing in the target population measure the ability
of the test to give accurate clinical information.
භ The positive predictive value is the proportion of
positive test results that correctly identify an
individual who has the phenotype of interest
(number of true positives / true positives + false
positives).
භ The negative predictive value is the proportion of
negative tests that correctly identify an individual
who does not have the phenotype of interest
(number of true negatives / true negatives + false
negatives).
167
Clinical Utility
භ Addresses risks & benefits of testing:
භ Results of pilot trials
භ Quality assurance processes that monitor
effectiveness of lab testing
භ Adverse effects of testing on health or social
consequences
භ Follow-up treatment/interventions available for
patients based on genetic test results
භ Ret mutations guide therapy for MEN-2 patients
භ EGFR mutations guide drug use for lung cancer
භ Financial costs and economic benefits of testing
භ ELSI issues
ACCE
EGAPP
Haddow & Palomaki
Current State
Screening
Symptomatic medicine
Precision
medicine
Future State
Screening
•
•
NBS 2nd tier (panel)
NBS confirmatory (panel)
Symptomatic
medicine
Precision medicine
•
•
•
Single gene testing (pharma)
Gene Panels (pharma)
Exome/ Genome sequencing
168
Pitfalls in Genetic Testing
භ
භ
භ
භ
භ
භ
භ
භ
භ
භ
භ
PCR: PCR Contamination
PCR: SNPs in primers
Wrong gene….diagnosis incorrect
Linkage: Paternity incorrect; recombination error rate
Prenatal: Maternal cell contamination R/O is critical!
Incomplete testing….test doesn㵭t detect certain types of mutations
Patient has had a bone marrow transplant
Mutation in patient is not on panel
Potential for false negative results
VUS – may be uninformative or misclassified
Technical problems
භ Sample mix-ups at pre- or analytical stage
භ Sample mix-up at blood draw site
භ SNPs/rare variants at the primer/probe site
භ Human Error
Helpful Study Guides
භ GeneTests Reviews www.genetests.org
භ ACMG Standards & Guidelines
භ ACMG Practice Guidelines www.acmg.net
භ Human Molecular Genetics, Strachan & Read
භ Human Genetics & Genomics, Korf
භ Genetics in Medicine, Thompson & Thompson
භ CAP/ACMG Proficiency Testing Program (targets
most common genetic tests)
භ ACMG Genetics Review Course
භ CLSI Molecular Methods, 2011 edition
Supplemental Study Slides
169
භ BCR/ABL1 is molecular signature of
Philadelphia chromosome t(9;22)
භ Fusion gene of BCR and ABL1 tyrosine kinase
භ Diagnostic for CML
භ Prognostic for ALL
භ Monitor MRD by quantitative PCR
භ TKI therapy targeted to fusion gene – imatinib,
dasatinib, nilotinib
භ Mutations in ABL in kinase domain detected by
sequencing may inhibit binding of drug
B. Zehnbauer
Proposed mode of action of STI571
-Functions through competitive inhibition of the ATP binding site
on the BCR-ABL product
-This leads to inhibition of tyrosine phosphorylation of proteins
S. Olson
involved in signal transduction
Druker et al., 2001
EGFR (ERBB1, HER1)
භProto-oncogene (7p12)
භEncodes a receptor
tyrosine kinase
භOverexpressed in various
cancers (lung, head and
neck)
භApproximately 10% of
patients with NSCLC
respond to small molecule
inhibitors gefitinib or
erlotonib
Barbara
Zehnbauer
170
Mutations Associated with
Sensitivity to EGFR Inhibitors
EGF
binding
EGF
TM
binding
Tyrosine kinase
Autophosphorylation
Exon 1
28
Exon 18
Exon 19
Exon 20
Exon 21
5%
45%
<1%
40-45%
G719C
ΔE746-750
% of all mutations
Examples
L858R
Zehnbauer
Mutations Associated with
Resistance to EGFR Inhibitors
EGF
binding
EGF
TM
binding
Tyrosine kinase
Autophosphorylation
Exon 1
28
Exon 18
Exon 19
Exon 20
% of all mutations
1%
5%
Examples
D761Y
T790M
Exon 21
Zehnbauer
panitumumab
cetuximab
Zehnbauer
EGFR inhibitors will be ineffective when KRAS is mutated
and constitutively active.
http://www.kras-info.com/slide_set
171
Normal
ITD
FLT3
Mutation = poor prognosis
In AML
dHPLC analysis
NPM
Mutation =
good prognosis
In AML
Insertion of 4bp = frameshift
Special issues with Tumor analysis
භ FFPE: formalin-fixed paraffin-embedded slides of tumor/normal
tissue
භ Requires stained slides & microscopic review to identify regions of
normal & tumor
භ There is a minimal amount of tumor required for analysis
භ Tumor heterogeneity can influence result
භ Requires method of detection with high sensitivity to detect small
amount of variant in tumor sample
භ Tumors are very different from germline analysis!
Other considerations….
භ Evidence-based tests
භ See requirements of GTR
භ Test validation
භ Review Guidelines
භ Pitfalls of DNA tests
172
Genetic Transmission
GENETIC TRANSMISSION
Bruce R. Korf, MD, PhD, FACMG
Wayne H. and Sara Crews Finley Chair in Medical Genetics
Professor and Chair, Department of Genetics
Director, Heflin Center for Genomic Sciences
University of Alabama at Birmingham
Bruce R. Korf, MD, PhD, FACMG
Department of Genetics
University of Alabama at Birmingham
1720 2nd Ave. S., Kaul 230,
Birmingham, AL 35294-0024
(205) 934-9411 Telephone
(205) 934-9488 Fax
[email protected]
175
176
Genetic Transmission
Bruce R. Korf, MD, PhD
Professor and Chair, Department of Genetics
University of Alabama at Birmingham
Disclosure(s)
Relationship
Entity
Grant Recipient
Novartis
Advisory Board
Accolade, Genome Medical
Board of Directors
American College of Medical Genetics and Genomics
Children’s Tumor Foundation
Advisor
Neurofibromatosis Therapeutic Acceleration Project
Founding Member
Envision Genomics
Salary
University of Alabama at Birmingham
Objectives
• Recognize patterns of Mendelian transmission
• Describe deviations from classical Mendelian
transmission
• Perform basic population genetic calculations
• Describe models of multifactorial inheritance
• Use odds ratios in risk assessment based on GWAS data
177
Autosomal Recessive
A
a
AA
A
A
A
a
a
Aa
unaffected
Genotype
homozygous
Phenotype
unaffected
heterozygous
Aa
aa
a
affected
homozygous
unaffected
affected
Counseling
2/3
Consanguinity
Aa
AA
AA
Aa
Aa
AA
AA
Aa
Aa
AA
aa
Identical by descent
178
Recessive Mechanisms
Gene
Loss of function mutations:
deletion, frameshift, stop,
missense
Protein
Complete absence of
product or significant
reduction of function
Heterozygote – sufficient
activity to avoid phenotype
Homozygote – profound loss of
activity results in phenotype
Autosomal Dominant
Aa
a
aa
a
a
a
A
A
homozygous
Phenotype
unaffected
heterozygous
affected
Aa
aa
Aa
aa
A
aa
Genotype
aa
A
Aa
Aa
aa
Aa
Aa
aa
homozygous
affected
Pseudodominance
Aa
Aa
aa
aa
179
Penetrance
Fraction of individuals who carry a gene who
manifest a specified phenotype
Age-Dependent Penetrance
Expressivity
different modes or degrees of expression of trait
in population
Neurofibromas in NF1
180
Dominant Mechanisms
•
•
•
•
•
• Haploinsufficiency
• Dominant negative
• Tumor suppressor
Deletion
Stop
Frameshift
Missense
Structural
Loss of
Function
Gain of
Function
• Signaling pathway
• Missense
Mosaicism
• Germ line
• Somatic
X-linkage
Male
y
A
Y
a
unaffected
A
Y
Aa
affected
y
A
y
Aa
a
Female
A
A
unaffected
A
a
unaffected
a
a
affected
Aa
A
y
AA Aa
a
A
AA
AA
a
No male to male transmission
181
X-linked Dominant
Male transmits to all daughters, not to sons
X-linked Dominant Male Lethal
Males who inherit mutation die in utero
Females who inherit mutation are affected
X Chromosome Inactivation
182
Genetic Heterogeneity
• Locus
• Mutations in different genes result in same phenotype
• Allelic
• Different mutations in same gene
Locus Heterogeneity
Allelic Heterogeneity
gene
aa
BB
AA
bb
Compound
Heterozygote
Genetic Heterogeneity
Gene
A
Gene
D
Phenotype
1
Gene
E
Gene
B
Gene
C
Phenotype
2
Gene
F
Gene
G
Sex Limited Expression
autosomal dominant
transmission
expressed only in one sex
get male to male transmission
requires sex-specific factor for
expression
male pattern baldness
183
Epistasis
gene interaction results in
modification of phenotype
A
O
AB
Bombay
Phenotype
A
H
precursor
B
3
Digenic Inheritance
Gene Locus 1
Gene Locus 2
Maternal Transmission
184
Mitochondrial DNA
• 16.5 kb circular double
stranded DNA
• Multiple copies per
mitochondrion
• Heteroplasmy – mixture of
mitochondria with different
genotypes in same cell
• 13 subunits of mitocondrial
proteins, tRNAs, rRNAs
• Most mitochondrial proteins
encoded in nucleus
Genomic Imprinting
maternal
copy
expressed
paternal
copy not
expressed
Imprinting: differential
expression
of maternal and paternal copy
of a gene
Hereditary Paraganglioma
185
Hardy-Weinberg Equilibrium
• Large population
[A] = p
[a] = q
p + q =1
• No mutation
• No selection
[AA] = p2
[Aa] = 2pq
[aa] = q2
• Random mating
• No migration
frequencies remain stable
eggs
sperm
allele
frequency
A
=
a
=
A
a
p
q
AA
p
p2
aA
q
pq
allele
frequency
Aa
pq
aa
q2
Autosomal Recessive
Cystic fibrosis: 1/2,500 = q2
q = 1/50
2pq = 2(1/50)(49/50) ≈ 1/25
2/3
1/25
risk to offspring = (2/3)(1/25)(1/4) = 1/150
186
Selection
AA
Aa
aa
Before
p
2
2pq
q2
After
p2
2pq
0
Reduction in reproductive fitness
Genetic Lethal
p
q
0.5
0.5
0.66
0.33
0.75
0.25
generation
1
aa
Aa
AA
gene pool
2
Aa
AA
aa
gene pool
3
AA
Aa
aa
Genetic Lethal – Change in Allele Frequency
q
0.6
0.5
0.4
0.3
0.2
0.1
0
0
1
2
3
4
5
6
7
8
9
10
11
12
13
14
15
16
17
18
19
20
generation
187
Mutation-Selection Equilibrium
a
a
mutation
A
A
A
A
A
A
A
A
A
A
A
A
A
A
A
A
A A
A
A
A A
A
A
A
a
a
A
A
A
A
A
A
selection
A
A
A
Mutation-Selection Equilibrium
q
0.6
0.5
0.4
0.3
selection
0.2
0.1
0
0
1
2
3
4
5
6
7
8
9
10
11 12
13
14
15
16
17
18
19
20
generation
mutation
Mutation-Selection Balance
Autosomal Recessive
2
Before
p
2pq
Fitness
1
1
After
2
p
2pq
2
q
1-s
2
q (1-s)
lose 2sq2 alleles each generation
at equilibrium, 2μ = 2sq2
q=
μ
s
188
Mutation-Selection Balance
Autosomal Dominant
AA
Aa
aa
Before
p2
2pq
q2
Fitness
0
1-s
1
After
0
2pq(1-s)
q2
lose 2ps alleles each generation
at equilibrium, 2μ = 2ps
p = μ/s
Balanced Polymorphism
• Maintenance of deleterious allele in heterozygotes
• “Heterozygote advantage”
• Polymorphism: occurrence of at least two alleles at
locus having frequency of at least 1%
Malaria
AA
Globin Disorder
aa
AA
Aa
Genetic Drift
• Fluctuation in gene frequency
due to small size of breeding
population
• Fixation or extinction of allele
possible
189
Founder Effect
• High frequency of gene
in distinct population
• Introduction at time when
population is small
• Continued relatively high
frequency due to
population being “closed”
Consanguinity
Coefficient of Inbreeding:
Probability that two alleles in child are
identical by descent; symbolized by “F”
Coefficient of Inbreeding
190
Coefficient of Inbreeding
Coefficient of Inbreeding
(1/2)(1/2)(1/2)+(1/2)(1/2)(1/2) = 1/4
Coefficient of Inbreeding
191
Multifactorial Inheritance
• Trait clusters in family
• Increased concordance in identical twins
• Multiple genetic and/or non-genetic factors
Familial Clustering
λR = ratio of risk in relatives of type R compared with population
risk
λs = ratio of risk in sibs compared with population risk
Cystic fibrosis:
Risk in sibs = 0.25; risk in population = 0.0004 λs = 500
Huntington disease
Risk in sibs = 0.50; risk in population = 0.0001 λs = 5000
Disorder
Population
Frequency
Recurrence Risk in First Degree
Relatives
λ
Cleft lip 㫧 palate
0.001
0.049
49
Congenital hip dislocation
0.001
0.035
35
Pyloric stenosis
0.002
0.032
16
Type 1 diabetes mellitus
0.002
0.071
35
Twin Studies
• Dizygotic
– Genetically equivalent to
full sibs
– Distinct placentas,
chorions, amnions, but
placentas/chorions may
fuse
• Monozygotic
– Genetically identical
– Fetal membranes may be
separate or shared
% concordance
Disorder
MZ
Sibs
Cleft lip 㫧 palate
40
5
Pyloric stenosis
22
4
Clubfoot
32
3
Congenital hip dislocation
33
4
Diabetes mellitus type 1
36
7
192
Heritability
genetic environmental
variance
variance
measurement
variance
VP = VA + VD + VE + VI + CovGE + VM
VA = additive genetic variance
VD = deviation due to dominance and epistasis
VE = environmental variance
VI = interaction variance
CovGE = covariance of genetics and environment
Heritability in narrow sense
Heritability in broad sense
VA
VP
h2 =
h2 =
VG
VP
Additive Polygenic Model
QTL: quantitative
trait locus
aabb
Aabb
aaBb
Dominant: 2 cm
Baseline: 150 cm
AaBb
aaBB
AAbb
AaBB
AABb
AABB
Threshold Model
Threshold
Number of
Individuals
Trait
Liability
193
Sex Differences
females
males
Common Disease-Common Variant
Hypothesis
Common diseases accounted for by genetic
variants found in 1-5% of population
Linkage disequilibrium
Case-Control Study
ACTAGGA
Allele 1
ACTCGGA
Allele 2
Asthma
No Asthma
Allele 2
Present
30
10
Allele 2 Not
Present
70
90
Hypothesis: Allele 2 is associated with an increased risk of asthma
194
Odds Ratio
Asthma
No Asthma
Allele 2 Present
30
10
Allele 2 Not
Present
70
90
Odds Ratio (used on case-control studies):
Odds of disease given allele = 30/10 = 3
Odds of disease given not allele 2 = 70/90 = 0.78
Odds ratio = 3/0.78 = 3.85
Odds Ratio and Risk
ܴܴ ൌ ܱܴ
ͳ െ ܴܿ ൅ ሺܴܿ ȉ ܱܴሻ
ܴ݅‫ ݇ݏ‬ൌ ܴܴ ȉ ܴܿ
Say the risk of asthma in the population is 0.1 and we have calculated the OR
ଷǤ଼଺
to be 3.86. The relative risk would be
= 3.00
଴Ǥଽାሺ଴ǤଵȉଷǤ଼଺ሻ
The risk to an allele 2 carrier of getting asthma would then be (0.1)(3.00) = 0.3
GWAS
https://www.ebi.ac.uk/gwas/diagram
195
Missing Heritability
100%
90%
Non-genetic
factors
80%
70%
60%
50%
40%
Calculated
heritability
Missing
heritability
30%
Possible Explanations
• Heritability overestimated
• Rare variants account for
some heritability
• Non-detected variants
(e.g., CNVs)
20%
Heritability
accounted for
by GWAS
10%
0%
196
Newborn Screening
NEWBORN SCREENING
John A. Phillips, III, MD, FACMG
David T. Karzon Professor of Pediatrics
Professor of Pathology, Microbiology and Immunology and
Professor of Medicine
Director Division of Medical Genetics and Genomic Medicine
Vanderbilt University School of Medicine
John A. Phillips, III, MD, FACMG
Division of Medical Genetics
Vanderbilt University School of Medicine
DD-2205 Medical Center North
Nashville, TN 37232-2578
(615) 322-7602 Telephone
(615) 343-0959 Fax
[email protected]
199
200
Newborn Screening
John A Phillips III
David T Karzon Prof of Pediatrics
Vanderbilt University Medical Center
4/21/2017
Site Investigator: 1) PKU: BMN 015 &165, 2) Achondroplasia: BMN 111-901, 201 &
202 Clinical Trials BioMarin Pharmaceutical Inc. & 3) FAOD: UX007 Ultragenyx
Pharmaceuticals, Inc.
PI: TN State Genetics Contract & Member TN Genetics Advisory Committee
Co-PI: Vanderbilt Undiagnosed Disease Network (UDN) Clinical Center
Co-I: Dr Blackwell’s PPG “Mechanisms of Familial Pulmonary Fibrosis”.
4/21/2017
Learning Objectives
•Understand the approach used in Newborn
Screening (NBS)
• Understand clinical presentations, Dx & Rx of
selected disorders in the NBS Panel
• Be able to access & use ACMG ACT sheets &
New England Emergency Protocols
4/21/2017
201
NBS: Key Points
• Done by state health programs
• Emphasizes disorders with effective, necessary
& available early Rx
• Samples obtained by hospital, but tests & follow
up done by state
• Methods (ABR, DNA, enzyme, hormone, HPLC,
MS/MS, OAE) & disorders may differ between
states
4/21/2017
NBS: Criteria
• Serious & has reasonable frequency
• Clinical Dx is difficult & requires test
• UnRx causes irreversible damage
• Test is rapid, sensitive & specific
• Feasible intervention is available that improves
outcome
• NBS program is cost effective
4/21/2017
NBS: Approach
• Traditional Medicine- based on signs, symptoms, or FH
• NBS- done on all to identify those to Rx
• Preclinical-period before disease onset
4/21/2017
202
Sensitivity: Fraction of affecteds
who screen positive = TP/(TP+FN)
Sample screen
= TP+TN+FP+FN
4/21/2017
1- Sensitivity = False Neg Rate
Specificity: Fraction of unaffected
who screen neg = TN/(FP+TN)
4/21/2017
1- Specificity = False Pos Rate
Positive Predictive Value (PPV):
Fraction with + screen who are affected =TP/(TP+FP)
4/21/2017
203
Management of NBS Results
96%
1%
4/21/2017
3%
Unsatisfactory NBS Samples
4/21/2017
Abnormal NBS Results
Mildly abnl (PPV < 10%)
• Only one analyte ~cut off
• Lab notifies PCP/facility to repeat NBS
Highly abnl (PPV > 60%)
• Single/multiple analytes >>cut off
• Lab notifies PCP ASAP & gives Metabolic Specialist
contact info
• ACMG ACT sheets & Algorithms, GeneReviews &
New England Emergency protocols
4/21/2017
204
ACMG NBS Info
(www.acmg.net)
4/21/2017
Phenylketonuria (PKU): Dx & Rx
• AR def of Phenylalanine
Hydroxylase (PAH)
• ~1/15,000
• UnRx IQ < 50
• Dx: Phe > 1200 μmol/l (μM) (20
mg%; Tyr & nl blood Biopterin or
urine pterins (Genet Med 16: 188,
14)
• Rx: Phe diet if >360 μM
4/21/2017
PKU: NBS History
Folling
1934
Guthrie Bacterial
Inhibition Assay 1961
MS/MS
2017
4/21/2017
205
PKU: NBS by MS/MS
• Sample prep & add internal standard (IS)
• Ionize & inject into 1st MS/MS mass analyzer (MA)
• Select ions & fragment by argon
• Separate fragments by 2nd MA & determine signatures
4/21/2017
PKU: MS/MS Phe & Phe/Tyr ratio
4/21/2017
Case 1: What Should You Do?
• It is 4:30 PM on a Friday
• You (Baby Gale’s PCP) are called by the State
NBS lab
• Baby Gale’s Phe level was 410 μM (nl <152) &
Phe/Tyr was 3.02 (nl < 2.01) at 24 hrs of age (3
days ago)
• What should you do?
4/21/2017
206
ACMG NBS Info
(www.acmg.net)
4/21/2017
4/21/2017
ACMG Algorithm http://www.acmg.net
4/21/2017
207
Case 1: What should you do?
a) Inform family of NBS result
b) Evaluate baby & consult metabolic specialist
c) Initiate confirmatory/diagnostic tests with
metabolic specialist
d) Provide family basic info on PKU
e) All of above
4/21/2017
PKU (PAH) vs BH4 Defects
98%
2%
4/21/2017
Hyperphenylalaninemia
Category
Phe Level μM/L
Mutations
Classic PKU
>1200 (20mg%)
PAH
Mild/Moderate
PKU
360-1200 (6-20mg%)
PAH
Non-PKU HPA
120-360 (2- 6 mg%)
PAH
Transient HPA
Premature, TPN, Liver
disease & Mat HPA
None
BH4 Deficiency
~2% of those 120>1200 (2-20 mg%)
PCD, DHPR,
GTPCH & PTPS
BH4 Responsive
PKU
~400-1100
PAH
4/21/2017
208
Treatment of PKU
4/21/2017
PKU: Treatments
• Standard: Phe free formula & low protein diet
Goals: Phe 120-360 μM (all ages)
Ref: Genet Med 16: 188, 14
• Cofactor: BH4 (Sapropterin™)
@ 20mgm/kg/day
ź Phe >30% in ~50%
May be used in pregnancy
4/21/2017
PKU: Treatments
• Competition: Large neutral amino acids
• Competes with Phe for uptake by membrane transporters &
Lysine impairs Phe absorption
• May reduce CNS Phe but compliance critical
• Not recommended in pregnancy
• Enzyme Substitution: Recomb PEGylated Phe Ammonia Lyase
• Phase 3 trial testing safety, effect & dose on Phe levels
• Injection site reaction & immune reaction problems
• 0.1mg/kg lowers Phe by 62%
4/21/2017
209
Maternal PKU
• Phe teratogenic causes miscarriage, IUGR, ID, CHD,
microcephaly, CL/P & pyloric stenosis
• Phe<360μM 3 months before conception
4/21/2017
Tyrosine: Pathway
Tyrosinemia Type II
Tyrosinemia Type I
4/21/2017
Tyrosinemia Type I (Hepatorenal)
• AR defect in FAH
• Sx: liver failure/cirrhosis & hepatic cancer
(35-40%) by 5yrs; renal Fanconi syndrome,
FTT; neurologic crises
• Dx: UOA + succinylacetone; PAA & NBS
Tyr & Met; 1/5000 French Canadians
• Rx:
NTBC &
Phe/Tyr diet, liver transplant ?
4/21/2017
210
Tyrosinemia Type II (Oculocutaneous)
• Clin: ID, corneal dystrophy,
hyperkeratosis & erosions
of palms/ soles
• AR deficiency of Tyr aminotransferase
• Dx: PAA Tyr, UOA 4-OH-phenyl
lactate & N-acetyltyr
• Rx: low Tyr diet
4/21/2017
Classical Homocystinuria
• Cystathionine ɴ Synthase Deficiency
• AR ~1/150,000
• NBS for Methionine rises slowly; likely true + if Met > 100
μM
• Dx:Ÿplasma Methionine & Homocysteine
• źCysteine & urine + nitroprusside
4/21/2017
Classical Homocystinuria
• Clin: ID, lens dislocateź
thrombosis & osteoporosis
• Rx: Pyridoxine (B6) & Folate;
prn low Met diet, Betaine,
Dipyridamole & ASA
• ~½ respond B6
(usually Ile278Thr) vs
non responders
(Ser307Gly)
4/21/2017
211
Branch Chain AA (BCAA) Catabolism
4/21/2017
Maple Syrup Urine Disease (MSUD)
• AR def branched chain (BC) ketoacid decarboxylation
• Freq ~1/ 250,000 (Mennonites 1/176)
• Clin: Urine maple syrup odor; lethargy, irritability,
emesis & coma
• NBSŸLeucine; Dx:Ÿblood BCAA (Leucine, Isoleucine
&Valine) & Alloiso-leucine (diagnostic); urineŸBC
ketoacids & ketones
• Rx: Thiamine (B1), diet źBCAA
4/21/2017
ACMG NBS Info
(www.acmg.net)
NBS often +
due to mom’s
levels
4/21/2017
212
Medium Chain Acyl-CoA
Dehydrogenase (MCAD) Deficiency
• AR, most common FAOD ~1/12,000
• Fasting > lethargy, hypoketotic hypoglycemia & SIDS
(first crisis fatal in 25% before NBS)
• NBS: MS/MSŸC6, C8 & C8/ C10
• Dx: Acylcarnitine profile,źfree carnitine, UOA ŸC610 dicarboxylics, >90% of mutations are Lys304Glu
(K304E)
• Rx: Carnitine & avoid fasting
4/21/2017
MCAD MS/MS Acylcarnitine Profile
Normal
100
%
0
100
MCAD
Deficiency
%
0
225
250
275
300
325
350
375
400
425
450
475
500
m/z
4/21/2017
NBS for Other FAODs
• SCAD: hypotonia & metabolic acidosis; NBSŸC4 &
UOA have ethylmalonic acid; common mild variants
of ? Significance
• LCHAD: cardiomyopathy, hypotonia, rhabdomyolysis
& moms have HELLP; NBS ŸC14-OH, C16-OH , C18OH & C18:1-OH
• VLCAD: cardiomyopathy, hepatomegaly,
rhabdomyolysis & SIDS; ŸC14:1 & C14:1/ C12:1
4/21/2017
213
ACMG NBS Info
(www.acmg.net)
D
D
NBS often +
due to mom’s
D
4/21/2017
3Methylcrotonyl CoA Carboxylase Def (3MCC)
• Clin: highly variable; some decompensate but
great majority are asymptomatic; metabolic
acidosis; Ÿ AG & NH3;źglu
• AR, heterodimer MCCA*, MCCB*
• NBS:ŸC5-OH, mom’s levels can cause
false + infant NBS; UOA show 3-methylcrotonylglycine
• Rx with diet (low Leu if needed) & add carnitine
& Glycine
4/21/2017
Case 2: What Should You Do?
• It is 4:30 PM on a Friday
• You (baby Adon’s PCP) are
called by State Lab
• Baby Adon had a C3
acylcarnitine level of
19.92 (nl <6.35) at 25 hrs
of age (two days ago)
• What should you do?
4/21/2017
214
ACMG ACT Sheet
http://www.acmg.net
4/21/2017
New England Emergency Protocols @newenglandconsortium.org
4/21/2017
Case 2: What Should You Do?
a) Inform family of NBS result
b) Evaluate ASAP & consult metabolic specialist
c) Check BS, AG, ketones & NH3
d) If indicated stop protein, hydrate & Rx
chemical imbalances per metabolic specialist
e) All of above
4/21/2017
215
Propionic Acidemia
Dx
4/21/2017
Rx
Normal
Newborn Screening
Sx
Dx
Rx
Preclinical stage
ACMG NBS Info (www.acmg.net)
4/21/2017
Hemoglobin Disorders: HPLC
Sickle cell
FS
anemia (HbSS
or HbS/ߺ Thal)
Hb SC disease FSC
(HbS/C)
Hb S/beta
FSA
Thalassemia
(HbS/ß+ Thal)
4/21/2017
216
Biotinidase Deficiency
4/21/2017
Cystic Fibrosis
• AR, 1/3000 Caucasians (80% neg FH)
• NBS: two tier test with Immunoreactive
Trypsinogen (IRT); IRT>IRT or IRT>DNA
• Dx Iontophoresis (sweat test)
or DNA
• Rx: CF Clinic, pulmonary Rx,
pulmozyme, antibiotics; Kalydeco
(ivacaftor for G551D); diet, oral
enzymes & genetic counseling
4/21/2017
Hearing: Auditory Brainstem Response (ABR)/
Evoked Otoacoustic Emissions (OAE)
~15% prelingual deaf are either GJB2 (Connexin 26) 35delG
homozygotes 98% (AR) or GJB2 35delG/GJB6 (Connexin 30)
del double heterozygotes (2%)
4/21/2017
217
Classical Galactosemia (GALT)
Galactose
Galacitol
Galactokinase
Galactonic
ATP
ADP
Galactose-1-phosphate
GALT
UDPGlu
UDPGal
Epimerase
Glucose-1-phosphate
Phosphoglucomutase
4/21/2017
Glucose-6-phosphate
Classical Galactosemia
• AR, ~1/62,000, Galactose 1 PO4 Uridyl Transferase
(GALT) deficiency
• Clin: emesis, diarrhea, jaundice & E Coli sepsis;
ź liver & renal function; cataracts only develop later
• NBS:ŸGal / Gal1P & Absent GALT = EMERGENCY.
You should immediately:
1) Repeat Gal, Gal 1 P & GALT
2) Start Gal free formula
4/21/2017
ACMG ACT Sheet @ www.acmg.net
4/21/2017
218
Congenital Hypothyroidism
• ~1/4,000; 85% thyroid develop, 10%
synthetic & 4% hypopit, only~15% AR
Pendred: deaf as NB; deaf +
adolescent euthyroid goiter
• Clin: Poor growth, NB have
subtle signs but 2/3 ź IQ if not Rxd
• NBS:TSH >24 hrs; Dx T4 ,TSH & TRH
• Rx: L thyroxine
4/21/2017
Congenital Adrenal Hyperplasia (CAH)
ACTH causes adrenal hyperplasia & over-production of
17 OHP, testosterone & estradiol
4/21/2017
Congenital Adrenal Hyperplasia (CAH)
• AR, ~1/12,000; > 90% CYP21OH (salt wasting);
3ɴHSD similar; 11ɴOHŸ BP
• Clin:źaldosterone & cortisol cause hyponatremic
hyperkalemic dehydration & death;Ÿandrogen
virilizes females
• NBS:Ÿ17OHP (false + 20 toźbirth wt & < 24 hrs)
• Rx: Cortisone & mineralocorticoid
4/21/2017
219
ACMG NBS Info (www.acmg.net)
4/21/2017
Severe Combined Immunodeficiency (SCID)
• XL, AR & źADA ~1/50-100,000
• Clin: Cellular & humoral due to
B & T cell); presents 1-3/12 with
FTT, oral/diaper candidiasis, absent tonsils/lymph
nodes, recurrent & persistent infections despite Rx
•Gen: NBS T cell receptor excision circles (TRECs)
•Rx: Antibiotics (Pneumocystis), IVIg, avoid live viral
vaccines; HSCT within 3/12, gene therapy?
4/21/2017
Fabry Disease
• Clin: XL ~1/3-4k accumulate globotriaoslyceramide (Gb3). Onset 4-8 yrs:
angiokeratomas, acroparesthesia,
hypohydrosis, corneal/lens opacities
& proteinuria. Adults: LVH/ischemia, TIA/stroke, tinnitus,
hearing loss & ESRD. 70% Ƃ affected,variable & later.
• NBS: ɲ galactosidase A (GLA), if low WBC GLA in ƃ (not
reliable in ֯). Dx: low WBC GLA in ֱ do DNA testing
(classical vs later onset variants). In ֯ DNA testing.
• RX: Test ֱ for proteinuria q y after 2 y with
GFR; ֯ less frequently. ERT agalsidase ɲ/ɴ
[Replagal/ Fabrazyme] as needed.
4/21/2017
220
Gaucher Disease
• Clin: AR ~1/40-60k; Type 1: Non neuronopathic infancy to adult; anemia &
љplatelets; HS megaly, bone disease but
nl CNS. Type 2: Acute neuropathic (np)
Gaucher cells
onset < 2y with CNS љ & death by 2-4 y.
(crushed tissue
paper) in bone
Type 3: Chronic np longer survival.
marrow (BM).
• NBS: glucocerebrosidase (GBA), if low WBC GBA. Dx: low
WBC GBA, N370S/N370S; N370S/L444P; N370S/ 84GG
~65% cases. N370S/? np protective vs L444P/L444P not.
• RX: ERT: Cerezyme (Imiglucerase)/VPRIV (Velaglucerase
ɲ) or Elelyso (Taliglucerase ɲ) for Type 1, +/- Type 3; not
Type 2. Monitor ACE, TRAP & Chito. Substrate reduction:
Eligustat (? CYP2D6) or Zaveca (Miglustat) Type 1 adults.
4/21/2017
Krabbe Disease
• Clin: AR ~1/100k; Infantile onset (IO) has
progressive CNS љ/death < 2 y (85-90%).
Irritabile, spastic, blind, deaf & delay with
startle <6/12 & become decerebrate.
Onset > 1 y slower & variable (10-15%).
• NBS: galactocerebrosidase (GALC), if low reflex to
psychosine (p), if p high, reflex to 30 kb del (seen in
~45%), if not (del/del) sequence. Dx: 0-5% activity in
WBC/ fibroblasts + CSF protein (2), brain MRI (2), NCV (2)
& BAER (1). If ш 4 refer for HSCT.
• Rx: IO & later onset with mild symptoms, may improve &
preserve cognition but many show peripheral nervous
system deterioration. Supportive care IO in later stages.
4/21/2017
Niemann-Pick Disease
• Clin: AR ~1/250k (Ashkenazi 1/40k)
Type A: neonatal onset, hypotonia,
FTT, CNS deterioration, deaf, blind
(cherry red spot), lung infiltrates,
HS megaly & fatal by 1.5 - 3 y. Type B:
later onset, HS megaly, interstitial lung
& nl intelligence.
• NBS: sphingomyelinase, if low, sphingomyelin
phosphodiesterase 1 (SMPD1) DNA which may indicate
Type A or B. Foam “Niemann-Pick foam cells” in BM.
• Rx: supportive, liver/HSC transplantation, or ERT may be
considered. ERT is highly complicated.
4/21/2017
221
Pompe Disease (PD)(GSD Type II)
• Clin: AR ~1/40k; Early onset:2 m hypotonia,
weakness, cardiomegaly/cardiomyopathy,
FTT & hearing loss. Usually fatal < 12 m.
Late-onset: proximal muscle weakness &
respiratory failure; cardiac uncommon.
Genotype to determine CRIM status (null
alleles) but genotype & phenotype may not correlate.
• NBS: acid ɲ glucosidase (GAA), if low urine hexose
tetrasaccharide (HEX4), CPK, chest X ray, EKG, ECHO,
GAA sequence & CRIM status.
• RX: ERT ASAP if WBC GAA low & heart involved. Monitor
Ab for immune modulation. CRIM- often develop rhGAA
IgG Ab with poor outcome without immune modulation.
4/21/2017
Hurler Disease (MPS1H) & Scheie (MPS1S)
•Clin: AR~1/107k; Infants: hernias, recurrent URIs; then
growth & development slow,
progressive HS megaly, course
facies, corneal clouding (>1 yr),
hydrocephalus, skeletal & cardiac
disease. Fatal by 5-10 y. Scheie is
milder, onset adolesence or adulthood.
•NBS: ɲ L Iduronidase (IDUA), if low glycosaminoglycans
(GAGs) to RO Ɏdeficiency & sequence IDUA
•RX: ERT Laronidase (Aldurazyme) for non CNS
symptoms; HSCT & ERT for severe genotype < 2y age.
4/21/2017
References
ACMG NBS ACT sheets & algorithms @ http://www.acmg.net (ACMG Act
Sheets App)
Genetic Home References @ https://ghr.nlm.nih.gov
GeneReviews @ https://ghr.nlm.nih.gov
New England Emergency Protocols @ http://newenglandconsortium.org
SIMDNAMA course
Vademecum Metabolism (EVM) @ vademetab.org
4/21/2017
222
Developmental Genetics
DEVELOPMENTAL
GENETICS
Tony Wynshaw-Boris, MD, PhD, FACMG
James H. Jewell Professor of Genetics
Department Chair of Genetics and Genome Sciences
Case Western Reserve University School of Medicine
University Hospitals Case Medical Center
Tony Wynshaw-Borris, MD, PhD, FACMG
Dept. of Genetics and Genomic Sciences
Case Western Reserve University School of Medicine
University Hospitals Case Medical Center
One 10900 Euclid Avenue, BRB731
Cleveland, Ohio 44106-4955
(216) 368-0581 Telephone
(216) 368-3832 Fax
[email protected]
225
226
Developmental Genetics
Tony Wynshaw-Boris, MD, PhD
Chair, Department of Genetics and Genome Sciences
Case Western Reserve University School of Medicine
University Hospitals Cleveland Medical Center
Disclosure(s)
None
Congenital Anomalies
1-3% of all newborns
Leading cause of neonatal morbidity and
mortality
20% of infant deaths
10% NICU admissions, 25-35% of deaths
Pediatric admissions
25% to 30% have major birth defect
227
Causes of Congenital Anomalies
Congenital Anomalies
භ Isolated Anomaly
Incidence per livebirths
Undescended testes
Heart defect
Club foot
Neural tube defects
Cleft lip + cleft palate
Hypospadias
Polydactyly
Cleft palate
Craniosynostosis
Syndactyly
1:30
1:150
1:300
1:500
1:1000
1:1000
1:1500
1:2000
1:2000
1:2000
Deformation
Developmental Process is normal
Mechanical force alters structure
Examples:
Oligohydramnios
Breech presentation
Bicornuate uterus
228
Deformation
Clubbed feet
• spina bifida
Moore. The Developing Human. Saunders, 1994
Disruption
Developmental process is normal, but
interrupted
Examples:
Amniotic band sequence
Fetal Cocaine exposure
Disruption
Porencephaly
http://www.neuropat.dote.hu/develop.htm#Porencephaly
Amniotic Band
Wiedemann and Kunze. Clinical Syndromes. Mosby-Wolfe, 1997
229
Dysplasia
Abnormal tissue organization,
microscopic structure
Examples:
Skeletal or connective tissue dysplasias
Ectodermal dysplasias
Dysplasia
Ectodermal Dysplasia
Buyse. Birth Defects Encyclopedia. Blackwell Science, 1990;
Baraitser and Winter. Color Atlas of Congenital Malformation Syndromes, Mosby-Wolfe, 1996;
Bergsma. Birth Defects Compendium, Alan R. Liss, 1979.
Malformation
Morphological defect from an intrinsically
abnormal developmental process
Examples: holoprosencephaly, congenital
heart disease, neural tube defect
230
Malformation
Unilateral Cleft Lip and Palate
Moore, Persaud, and Shiota. Color Atlas of Clinical Embryology. Saunders, 1994
Syndrome
A recognizable pattern of anomalies
presumed to be causally related
Genetic: chromosomal, single gene
Environmental: alcohol, retinoic acid
Complex: more than one genetic and/or
environmental factor
Syndrome: Environmental
Causes
Fetal Alcohol
Growth retardation
Microcephaly
Mental retardation
Short palpebral fissures
Short nose
Smooth philtrum
Thin upper lip
Small distal palanges
Hypoplastic finger nails
Cardiac defects
Clarren and Smith. NEJM 298:1063, 1978
231
Normal Development
http://embryo.soad.umich.edu/carnStages/carnStages.html
Developmental
Pathways and Mechanisms
Development
232
Germ Cells and Stem Cells
Lineages from Stem Cells: Blood
Fate, Specification, and
Determination
233
Fate, Specification, and
Determination
Differentiation
CS 7, day 15-17
Gastrulation occurs as
cells migrate from the
epiblast, to form
mesoderm.
Mesoderm lies between
the ectoderm and
endoderm as a continuous
layer
From the primitive node a
tube extends under the
ectoderm to form the
notochord
http://embryology.med.unsw.edu.au/
wwwhuman/Stages/Stages.htm
http://www.med.unc.edu/embryo_images/
unit-bdyfm/bdyfm_htms/bdyfm003.htm
Pattern Formation
CS 10, week 4
Ectoderm: Neural folds
fuse
Mesoderm: continued
segmentation of
paraxial mesoderm
(4 - 12 somite pairs)
http://embryology.med.unsw.edu.au/wwwhuman/Stages/Stages.htm
234
Organogenesis
CS 13, week 5
Ectoderm: sensory placode,
lens placode, otic vesicle,
early nasal placode, forebrain
Mesoderm: 30 somite pairs,
heart prominence
Head: 1st, 2nd and 3rd
pharyngeal arch, stomodeum
Body: heart, liver, umbilicus,
upper limb bud, lower limb
bulge
http://embryology.med.unsw.edu.au/
wwwhuman/Stages/Stages.htm
Organogenesis
CS 16, week 6
CS 18, week 7
Nasal pits moved ventrally,
auricular hillocks, foot plate
Finger rays,
Ossification commences
http://embryology.med.unsw.edu.au/wwwhuman/Stages/Stages.htm
Growth
CS 20, week 8
CS 23, week 9
Upper limbs longer
and bent at elbow
Rounded head,
body and limb
http://embryology.med.unsw.edu.au/wwwhuman/Stages/Stages.htm
235
Establishment of Body Axes
A-P: anterior-posterior (cranial-caudal)
[Proximal-distal for limbs]
D-V: dorsal-ventral (back-front)
L-R: left-right axes
Patterning program of the embryo is overlaid onto these
axes
Rotation of the Proximo-Distal (P-D) to Anterior-Posterior
(A-P) axis and Mesoderm Induction
Nodal cilia rotate in a clockwise fashion to drive leftward fluid flow
Nonaka et al. PLoS Biology 3:e268 (2005)
236
Posterior Localization of Nodal Cilia in a Single Node Cell
Hashimoto et al. (2010) Nat Cell Biol. 12:170
Proximal
Distal
Anterior
Dorsal
(WNT7A)
Ventral
AER
(FGF4
FGF8)
Posterior
ZPA
(SHH)
HOX Genes: Transcription Factors for
Positional Information
237
Hox Gene Mutation: Syndromes
Anterior - Head
HOXA1
Athabaskan Brainstem Dysgenesis
Bosley-Salih-Alorainy Syndrome (Duane Syndrome, Deafness, Delayed
Motor Milestones, Autism)
Posterior - Tail
HOXA11
Radioulnar Synostosis with Amegakaryocytic Thombocytopenia
HOXA13
Hand-Foot-Uterus Syndrome
Preaxial Deficiency, Postaxial Polydactyly and Hypospadius
Hox Gene Mutation: Syndromes
Posterior - Tail
HOXD10
Vertical Talus, Congenital (Rocker-Bottom Foot)
HOXD13
Synpolydactyly 1 (Syndactyly, Type II)
Brachydactyly, Types D and E
Cellular and Molecular Mechanisms
During Development
238
Transcriptional Regulation:
Model for Wnt Pathway and Pitx2
during Development
Kioussi et al. Cell (2002)
Morphogens: Sonic Hedgehog
(SHH) in Neural Tube and Limb
Cell Shape and Organization
239
Neurogenesis: Neuroepithelium
vs. Radial Glia
The cortex forms by radial
and nonradial migration
3V
cortex
hip
caudate,
putamen
HIP
LV
NCX
LGE
PCX
MGE
globus
pallidus
Neocortical Layering in the Mouse
240
Neuronal Proliferation & Migration
Proliferation
Microcephaly
AR – multiple loci
Migration
Lissencephaly
Miller-Dieker LIS1
X-linked: DCX (doublecortin)
X-linked with abnormal genitalia
(ARX)
Cobblestone dysplasia (Fukuyama
MD, Walker-Warburg, muscle-eyebrain)
Heterotopia
Periventricular nodular (FLN1)
Cortical Organization
Pachygyria/polymicrogyria
Schizencephaly
EMX2
schizencephaly
Normal
Type I
Lissencephaly
(Severe)
Lissencephaly
Normal
Severe MR
Seizures
Early Death
Incidence: 1/50,000-1/100,000
241
Isolated
Lissencephaly
Sequence
LP87-001
Isolated
Lissencephaly
Sequence
LP86-003
Miller-Dieker
Syndrome
LP82-002
Heterozygous Deletions of
17p13.3 in ILS and MDS
X-linked Lissencephaly and
Subcortical Band Heterotopia
LP95-136m
LP95-136a1
242
Other Lissencephaly Syndromes
Lissencephaly with Cerebellar Hypoplasia
Reelin mutation
VLDL Receptor mutation
Lissencephaly with Abnormal Genitalia
ARX mutation
Lissencephaly Pathways
Programmed Cell Death During
Development
243
Developmental
Pathways and Mechanisms
Developmental Pathways
Core Pathways
Developmental Pathways
Cell Cycle, Proliferation, Apoptosis
244
Developmental
Pathways/Processes
Developmental Pathways- General
Ligand
Cell
Membrane
Receptor
Signaling
Molecules
Transcription
Factor
Nucleus
245
FGF
FGFR
Ras-MAPK
STAT
PI3K, PLCγ
Transcription
Transcription
Factors
Factors
STAT
Fibroblast Growth Factor Signaling
Pathway
Bonaventure and El Ghouzzi. Expert Rev Mol Med 2003:1-17, 2003
FGFR Craniosynostosis Syndromes
Autosomal dominant
Genetic heterogeneity
Crouzon
Phenotypic variability
Gain of function
mutations, missense
and in-frame deletions
and insertions, splicesite mutations in 85 to
90%
Jackson-Weiss
Apert
Crouzonodermoskeletal Beare-Stevenson
SADDAN
Jabs. ed. Jameson, Principles of Molecular Medicine, 1998
246
Fibroblast Growth Factor Receptors
Expert Reviews in Molecular Medicine © Cambridge University Press
Bonaventure and El Ghouzzi. Expert Rev Mol Med 2003:1-17, 2003
SHH
SMOH
CHOL
Sterol delta-7
reductase
PTCH
GLI
GLI
Transcription Factors
247
syndactyly
polydactyly
upturned nose
ptosis
cryptorchidism
CNS hypoplasia
holoprosencephaly
dysmorphic features
(short metacarpals,
rib defects, broad face,
dental abnormalities)
cancer predisposition
(rhabdomyosarcoma,
medulloblastoma
variable midline defects
(single maxillary incisor ,
hypotelorism,
holoprosencephaly,
cyclopia)
postaxial polydactyly
syndactyly
hypothalamic hamartomas
imperforate anus
hypertelorism,
syndactyly,
preaxial polydactyly with
broad thumbs, and great toes
248
Bardet-Biedl
Syndrome
Bardet Biedl Syndrome
Beales et al. J Med Genet 36:440, 2007
Ocular rod-cone dystrophy
Hearing loss
Anosmia
High arched palate
Situs inversus
Congenital heart defects
Liver disease
Truncal obesity
Renal abnormalities
Postaxial polydactyly and
Brachydactyly
Hypogenitalism in males
Cognitive deficits
Genetic Heterogeneity in BardetBiedl Syndrome
249
BBS Genes
Tobin and Beales. Pediatr Nephrol 22(7):926-936, 2007
Joubert Syndrome
AR group of inherited conditions
Congenital ataxia
Hypotonia,
Episodic breathing
Mental retardation
Associated finding in some patients:
Retinal dystrophy
Nephronophthisis (renal fibrocystic
disease)
Molar Tooth Sign:
Specific malformation of the
brainstem, cerebellum and
the cerebellar peduncles,
250
Figure 2. Cystoproteins are proteins of genes that are mutated in cystic kidney diseases of humans, mice, or
zebrafish
Hildebrandt, F. et al. J Am Soc Nephrol 2007;18:1855-1871
Copyright ©2007 American Society of Nephrology
251
Ligand
Cell
Membrane
Receptor Tyrosine Kinase
Adapter
RAS
RAF
ERK/MAPK
Transcription
Factor
Nucleus
Ras/MAPK Pathway
Kate Rauen
Genetic Syndromes of the Ras/MAPK Pathway
Neurofibromatosis 1
Cap-AV
Malformation
Gingival
Fibromatosis 1
RTK
RasGaps
active
RAS
SOS1
GRB2
SHP2
Costello
KRAS HRAS
SPRED1
CRAF BRAF
MEK1 MEK2
ERK1 ERK2
LEOPARD
pERK
nucleus
Noonan
Kate Rauen
Legius
pERK
Cardio-facio-cutaneous
252
254
Cancer Genetics I
CANCER GENETICS I
Sharon E. Plon, MD, PhD, FACMG
Professor, Department of Pediatrics/Hematology-Oncology
Molecular and Human Genetics
Human Genome Sequencing Center
Director, Medical Scientist Training Program
Baylor College of Medicine
Sharon E. Plon, MD, PhD, FACMG
Department of Pediatrics
Feigin Center Room 1200.18, 1102 Bates Street
Houston, TX 77030
(832) 824-4251 Telephone
(832) 825-4276 Fax
[email protected]
257
258
Cancer Genetic 1
Sharon E. Plon, MD, PhD, FACMG
Professor
Baylor College of Medicine
• I have the following financial relationships to disclose:
• I am a employee of Baylor College of Medicine (BCM) which derives revenue from
genetic testing, including whole exome sequencing.
• BCM and Miraca Holdings Inc. have entered into a joint venture, Baylor Genetics,
with shared ownership and governance of the clinical genetics diagnostic
laboratories
• I am a member of the BMGL Scientific Advisory Board
• I will discuss off label use and/or investigational use in my
presentation.
Outline of topics – Cancer Genetics 1
• Basic principles
• Oncogenes
• Somatic cell genetics/targeted therapy
• Basic principles
• Tumor suppressor genes (LOH)
• Familial predisposition syndromes
•
•
•
•
Retinoblastoma
Li-Fraumeni syndrome
Telomere Disorders
miRNA Disorders (miRNA)
259
Key points regarding mutation mechanisms in
Oncogenes
• The types of mutations differ significantly between oncogenes
and tumor suppressor genes (loss of function variants).
• Proto-oncogene - formal name for gene prior to
mutation/activation in tumor
• Mutations in oncogenes activate the gene product (missense)
or cause the gene to be misexpressed or overexpressed
(translocations, amplification).
• Mutations in oncogenes are frequently somatic, e.g., found in
the tumor but not in matched normal DNA from the patient
with the second allele unaltered.
1. Activation by Single Nucleotide Variation
SPECIFIC MISSENSE VARIANTS frequently
alter the normal activity of a protein to
become transforming.
• RAS proto-oncogene proteins have
GTPase activity that is normally
regulated.
• RAS activity becomes constitutively
active due to specific missense mutations
that block ATPase function.
• HRAS codon 12 is one of the most
common mutations in all human cancer.
Active
Ras-GTP
G12V
Inactive
Ras-GDP
Common Somatic Ras Missense Mutations
Ras Gene
Codon
Tumor Type
H-ras
12 GGC ÆGTC
Bladder
K-ras
12 GGT Æ GAT
12 GGT Æ GTT
12 GGT Æ GAT
61 CAA Æ CGA
Pancreas, Colon, Lung,
Uterine
N-ras
Leukemia, small number of
colon cancers
It is not entirely clear why there are different Ras
mutations in different tumor types.
260
2. Oncogene Amplification
• Increases the copy number of oncogene to be activating due to
increased expression – can result in hundreds of copies of the oncogene.
• Amplification often results from two overlapping mechanisms that are
distinguished by use of FISH:
• Double minutes (multiple extrachromosomal fragments)
• Homogeneously staining regions (HSR) – seen within chromomsomes
• MYCN (n-myc) frequently amplified in high risk neuroblastoma.
• Her-2/Neu/c-ERBB2 amplification frequent in human breast cancers.
• Therapeutic target of Trastuzumab/Herceptin
DzE Amplification - Associated w/ Poor Prognosis
in Neuroblastoma
100
MYCN Not Amplified (N = 71)
EFS (%)
80
60
P < 0.0001
40
MYCN Amplified (N = 31)
20
0
0
1
2
3
4
5
6
Years from Dx
7
8
Courtesy of Garret Brodeur – Children’s Hospital of Philadelphia
Translocations/Fusion Proteins
• Often results in oncogenes being abnormally expressed with the
creation of fusion proteins.
• Many translocations are specific to certain tumor types and can be
used to confirm diagnosis.
• Initially hallmark of hematopoietic malignancies but now clear that
translocations are found in many solid tumors.
• Transcription factor or kinase are most common downstream partner
of the translocation.
• Highly variable geography of translocations
• Intrachromosomal deletions and inversions can also result in
oncogenic fusion proteins.
261
Rearrangement types
• Imprecise translocations result in an oncogene being moved to the
proximity of a transcriptionally active gene with varying geography.
• Overexpression of the “normal” coding region of the oncogene in a
cell type normally silenced.
• Precise translocations result in the precise joining of two genes to
make a novel fusion gene:
• The 5' end of the gene controls the expression pattern +/functional domains and 3' end of the gene often controls function.
• Intrachromosomal deletions/inversions
• Deletions – P2RY8 exon 1 is fused to CRLF2 exon 1 and activates
transcription in high risk ALL and DS-ALL.
Imprecise Fusions t(8;14)
MYCC proto-oncogene at 8q24
IgG locus at 14q32
Chromosome 8
Chromosome 14
IgG – c-Myc
t(8;14) Translocation
Activation of MYCC oncogene by juxtaposition of MYCC
with the Immunoglobulin locus in lymphoid cells in Burkitt㵭s
Lymphoma – no fusion protein is made
Precise Translocation: Philadelphia
Chromosome in CML
Images from R. Naeem, TCH
262
Impact of BCR-Abl Inhibitor – Imatinib/Gleevac
• Development of a specific inhibitor
against the BCR-Abl protein has
resulted in improved treatment of Ph+
leukemia (CML and ALL) with
improved survival.
• Now also used for tumors which have
activation of other related kinases,
like gastrointestinal stromal tumors
(GIST) with activation of c-Kit kinase.
Courtesy – Stephen Kornblau (MDACC)
D>ϰͲ><Fusion Results from Recurrent Inversion
Chr 2p
KD
EML4
ALK
Normal cell
KD
ALK
Tumor cell
Somatic inversion
event
EML4
KD = kinase domain
• Somatic inversion event in ~3% of non-small cell lung cancer.
• Results in activation of ALK kinase because the EML4 domain
aids in heterodimerization.
• FDA approved companion diagnostic test to decide upon use of
ALK inhibitors – crizotinib - in treatment of lung cancer.
Specific Kinase Alterations in Cancer
• JAK2 mutations in high-risk childhood ALL and ALL associated
with Down syndrome.
• ALK missense and amplication (neuroblastoma), inversion
(NSCLC) and translocation (lymphoma) - crizotinib.
• FLT3 internal tandem duplication in AML.
• EGFR missense mutations in lung cancer associated with
sensitivity to tyrosine kinase inhibitors (gefitinib was first of
class FDA approved in 2003).
• Secondary somatic mutations V843I or T790M then associated with
resistance to same class of drugs.
• BRAF (B-raf) gene V600E in a substantial percentage of
malignant melanoma (multiple inhibitors FDA approved)
263
Metabolic Genes Implicated through
Whole Exome Sequencing
• Whole exome sequencing of high grade gliomas by Hopkins
group (Parsons et al, Science, 2009) led to discovery of specific
missense mutations in IDH1/2 in astrocytomas and secondary
glioblastoma multiforme.
• IDH1/2 also frequently mutated in AML and sarcomas
• Functional assays reveals that these are gain of function
mutations which alter substrates for chromatin remodeling
• Multiple other Krebs cycle proteins implicated in cancer, e.g.
fumarate hydratase
From Wallace, DC Nat Rev Cancer 2012, 10:685
Sampling FDA approved targeted drugs
Drug
Target
Tumors - off label use in italics
Imatinib
BCR-ABL and C-KIT
CML ,GIST, PH+-ALL
Trastuzumab
ERBB2/HER2/neu
Breast cancer, gastric or GEJ junction cancer
Crizotinib
EML4-ALK
ALK missense mutation
NSCLC
Neuroblastoma and other ALK+ tumors
Vermurafinib
BRAF V600E
Melanoma
Erlotinib
EGFR missense
Lung cancer
Vandetanib
RET missense
Medullary thyroid cancer
Everolimus
TSC1/TSC mutant
Subependymal giant cell astrocytoma
Olaparib
BRCA1/2 germline mutation
Ovarian cancer (PARP inhibitor)
Germline status
264
Genetic susceptibility to
cancer
Multiple Mechanisms of Cancer Susceptibility
• Common alleles with modest risk – GWAS
• Numerical chromosomal abnormalities: autosomes
and sex chromosomes
• Microdeletion/rearrangements
• Overgrowth syndromes including imprinting
disorders and mosaicism for somatic mutations.
• Autosomal dominant disorders – LOF mutations in
TSG and activating mutations in proto-oncogenes
• Autosomal and X-link recessive disorders
Multiple Mechanisms/Same Tumor- Wilms tumor
Syndrome
Location - Gene
Molecular Basis
Trisomy 18
Trisomy 18
Trisomy
WAGR
11p13 - WT1
Microdeletion
MYCN-DDX1
2p24.3
Microduplication
Dennys-Drash
11p13 - WT1
Point mutation (AD)
Perlman
syndrome
2q37 -DIS3L2
Autosomal recessive
BeckwithWiedemann
syndrome
11p15, IGF-2, H19
p57KIP2, LIT1/
KCNQ10T1
Loss of imprinting,
deletion, UPD &
mutation
265
GWAS experiments – large case:control
cohorts genotyped for common variants
Note: Many
candidate SNPs
have not validated
in larger studies
Taylor et al Trends Mol Med. 2001 Nov;7(11):507-12.
Breast cancer GWAS hits demonstrate the
typical modest impact on cancer risk
Gene/SNP
Chromosome
Risk allele
frequency
Relative Risk per
allele
FGFR2
10q
0.38
1.26
TNRC9
16q
0.25
1.2
5q
0.28
1.13
LSP1
11p
0.30
1.07
?/rs13281615
MAP3K1
8q
0.40
1.08
?/rs13387042
2q
0.50
1.2
CASP8
2q
0.86
1.13
From Pharoah, (2008) N Engl J Med 358;26; 2796
Chromosomal Abnormalities and Cancer Risk
– Trisomy 21
• Results in ~20 fold increase in leukemia and also
shifts the myeloid: lymphoid leukemia ratio to 40:60.
• Transient myeloid proliferation (TMP) which may or may
not evolve into leukemia is seen in infants.
• Somatic GATA1 mutations found in TMP cells
• Acute megakaryocytic leukemia is 400 fold RR.
• In DS associated ALL see JAK2 missense mutations
and CRLF2 activation due to intrachromosomal
deletion.
• Little increased risk of other cancer types
266
Pathways towards DS Associated Leukemia
TMP
GATA1 somatic variant
AML
Often cytogenetically
“normal”
Patient with
Trisomy 21
ALL
JAK2 variant
CRLF2 activation
(intrachromosomal deletion)
Sex Chromosome Abnormalities
• Girls with mosaic Turner syndrome or gonadal dysgenesis are
at increased risk for gonadoblastoma.
• Correlates with presence of Y chromosome containing
material.
• XXY males have an increased risk of breast cancer.
• Long-term survivors of trisomy 18 have increased risk of
Wilms tumor.
Inherited Structural Chromosome Defects
• WAGR – Wilms tumor, Aniridia, Genital abnormalities, mental Retardation
• Contiguous gene syndrome on 11p13.
• WT1 - Wilms Tumor and GU abnormalities
• PAX6 - aniridia
• Denys-Drash syndrome caused by missense or nonsense mutations in
WT1.
• Increased risk of end stage renal failure
• Very significant risk of WT (~50%) in patients with WAGR and estimated to
be 90% in patients with Denys-Drash
• Recommend screening children with deletion by abdominal U/S q 3
months until age 8.
267
Other microdeletions associated with cancer risk
through deletion of tumor suppressor genes
• Retinoblastoma - ~3% constitutional
deletions of 13q14 overlying the RB1 gene.
• Deletions of NF1 - ~3-5% of patients.
• Earlier onset neurofibromas
• Increased risk malignancy (PMNST)
• More severe developmental phenotype
• Syndromic thrombocytopenia with AML:
• Associated with deletions of RUNX1 at 21q22
(see photo from Ramawi et al, Blood 2008).
Imprinting Disorders and Tumor Risk
• Beckwith-Weideman Syndrome –excessive growth, macroglossia,
organomegaly, ear creases, hemihyperplasia
• Overall Wilms Tumor risk 2-5% and hepatoblastoma risk ~1-2%.
• Cancer risk >> BWS including HH or organomegaly - ~30% risk.
• Gain of methylation IC1 (28%), loss of methylation IC2 (2.6%),
pUPD11 16%, and CDKN1C mutations (6.7% ) – Maas et al,
AJMG, 2016
• Recommend screening kidneys by ultrasound and AFP for
hepatoblastoma as per WAGR.
Mendelian (high risk) Forms of Cancer Susceptibility
• Inherited cancers are a minority of the total number of cases
but the proportions can range from 1-60%.
• For most tumor types, e.g. breast, the inherited fraction fall in the
range of 1-10%.
• For an increasing number of rare tumors, adrenocortical carcinoma,
rhabdoid tumors, retinoblastoma and optic gliomas are recognized
to have a very high inherited fraction (30-60%).
• Specific histological subtypes can have higher frequency, e.g.
pediatric ALL ~1% but hypodiploid ALL is ~50%.
• The majority of conditions result from loss of function
mutations in tumor suppressor genes
268
Diagnosis with >10% hereditary form
Genetic Loci
Retinoblastoma
RB1
Adrenocortical or choroid plexus carcinoma,
hypodiploid ALL, anaplastic rhabdomyosarcoma
TP53
Pheochromocytoma/ paraganglioma
VHL, NF1, RET,
SDHB, SDHD…
Retinal or cerebellar hemangioblastoma,
Endolymphatic sac tumor (ELST)
VHL
Optic pathway tumor, malignant peripheral nerve
sheath tumor, JMML
NF1
Medullary thyroid cancer
RET
Atypical teratoid/ malignant rhabdoid tumor
SMARCB1/A4
Ovarian small cell carcinoma, hypercalcemic type
SMARCA4
Acoustic or vestibular schwannomas
NF2
Pulmonary pleuroblastoma
DICER1
General Features or Autosomal Dominant
Hereditary Cancer Disorders
• Multiple generations affected with cancer w/highly
variable proportion of de novo mutations.
• Transmission through mothers and fathers independent
of tissue type.
• Earlier age of onset of cancer c/w sporadic cases
• Increase in multiple and bilateral tumors
• Clustering of specific tumor types w/in family
• Variable penetrance with unaffected mutation carriers
Autosomal Dominant Cancer Disorders
Associated with Inherited Oncogene Mutations
Gene
Disorder
RET
Multiple endocrine neoplasia type 2
MET
Hereditary papillary renal cell cancer
HRAS
Costello syndrome with skeletal abnormalities,
developmental delay, bladder
KRAS
Cardio-Facio-Cutaneous syndrome – no known cancer
phenotype
ALK
Hereditary neuroblastoma (specific missense alleles)
EGFR
Familial lung cancer (V843I or T790M)- germline
When somatic these variants associated with acquired
resistance to tyrosine kinase inhibitor therapy
269
Multiple Endocrine Neoplasia 2
• Type 2A includes- in this order of diagnosis:
•
•
•
•
Medullary thyroid carcinoma (MTC)
Pheochromocytomas
Parathyroid disease
Missense mutations in cysteine residues found in >90%.
• Type 2B presents in infancy with
• Ganglioneuromas of the GI tract, lips and skeletal abnormalities. Can be confused
with Hirschsprung’s megacolon.
• Onset of MTC in early childhood
• Two specific tyrosine kinase domain missense mutations: M918T >> A883F
• Familial MTC - without other MEN features.
Clinical Evaluation for MEN2
• Genetic testing is standard of care in MEN2 families and anyone
with MTC especially at young ages.
• Testing is sequencing of either the entire cDNA or regions with
recurrent missense mutations in RET.
• If mutation positive, recommend prophylactic thyroidectomy
(Chen et al., Pancreas, 2010):
• Level 1 mutations age 5 – 10 outside conserved cysteine
residues
• Level 2 mutations by age 5 (cysteine alleles in exon 10-11)
• Level 3 mutations by age 1 (including MEN2B alleles).
• Specific RET alleles also associated with Hirschsprung’s
Tumor Suppressor Genes
Many dominant disorders result from
loss of function (LOF) mutations in
tumor suppressor genes.
TSG normally function to:
• Inhibit proliferation
• Down regulate the cell cycle
• Repair DNA
• DNA damage checkpoints
Active
Ras-GTP
NF1
Inactive
Ras-GDP
270
Retinoblastoma
• 1 in 20,000 children affected
• Unilateral or bilateral tumors develop in early
childhood
• Occurs in heritable and nonheritable forms
• 15% of unilateral and ~100% bilateral are heritable
• 3-4% demonstrate somatic mosaicism for RB1 mutation
• Identifying at-risk infants substantially
reduces morbidity
• Developed concept of tumor suppressor gene
and loss of heterozygosity.
Features of Zϭ gene
• RB1 gene was linked to 13q14.2 by obvious cytogenetic
deletions in ~5% of children with Rb, developmental delay
and anomalies.
• Majority of germline and somatic variants are a variety of
LOF alleles (nonsense, frameshift, deletions, splicing – 90%).
• RB1 gene encodes a cell cycle regulator.
• Inhibits the G1 to S phase by recruiting histone
deacetylases to promoters to inhibit transcription of genes
required for S phase.
• RB1 protein itself regulated by phosphorylation.
Familial Retinoblastoma
B-Rb
dx 1
5y
B-Rb
Dx 6 mo
7y
• Truncating RB1 germline variants results in ~90%
penetrance for diagnosis of Rb (not necessarily
bilateral).
• 80% of patients with bilateral RB results from de
novo mutations w/o family history of Rb.
• Only 20% of patients with bilateral Rb have a
positive family history.
• Can see families with incomplete penetrance due
to missense or splicing alleles.
• Germline mosaicism in parents (fathers >
mothers) leads to substantial recurrence risk
271
Recurrence Risk for Rb in absence of
testing
Clinical scenario
Offspring of bilateral cases
Retinoblastoma
Risk
45%
Offspring of unilateral cases
7.5%
Sibling of bilateral cases (with
unaffected parents)
Sibling of unilateral cases (with
unaffected parents)
5-7%
1%
Concept: Somatic versus Germline Variants
• Fundamental aspect of cancer genetics is differentiating
inherited from somatic variants that occur as tumors develop.
• The same gene may undergo mutation in both the sporadic and
inherited forms.
• For TSGs you typically see a mixture:
• One inherited & one somatic (second hit)
• Two somatic (one smaller/point mutation and often second larger
event).
• One somatic or inherited (dominant negative)
• No mutation – silencing by methylation instead
Two Hit Hypothesis of TSGs
Normal genes
(prevent cancer)
1st mutation
( if inherited
susceptible carrier)
GROWTH
2nd mutation or loss
(leads to cancer)
272
Two Hit Hypothesis
• Inactivation of both copies of a TSG is required to develop
cancer.
• Both inactivation events can occur somatically during
development of childhood (unilateral tumor)
OR
• Inherit a single mutation in a TSG and then somatically
acquires loss/inactivation of the second normal copy of the
gene to develop tumor (more likely to be bilateral).
• At cellular (somatic) level these genes are recessive because both
copies of the gene are lost in tumors.
• Autosomal dominant inheritance of susceptibility to cancer.
Loss of Heterozygosity Experiment Using
Heterozygous Markers Flanking TSG
SNPchip data near TSG
Nl Tumor
Polymorphic marker near TSG
Molecular Basis of Second Hit
• Genetic events associated with LOH
• Loss of whole chromosome
• Loss of chromosome and reduplication of
chromosome containing mutation
• Loss of whole arm or large interstitial deletion
• Mitotic recombination between chromosomes
• Not associated with LOH
• Point mutation or intragenic deletion
• Silencing of gene by methylation of promoter
273
Test Results of Two Unilateral Rb patients
Sample
Allele 1
Allele 2
Tumor 1
Q347X
LOH
Blood 1
Q347X
Normal
Patient 1: Hereditary form of RB
Tumor 2
Blood 2
Methylation
Promoter
Normal
567delAG
Normal
Patient 2: Sporadic Rb due to somatic mutation
and promoter methylation
Recurrence Risk for Retinoblastoma w/genetic testing
– assume Zϭtesting is 95% sensitive
Bilateral Proband
• If blood testing reveals RB1
mutation
• Offspring and siblings tested
for mutation identified.
• If blood tested with
negative result:
• Uninformative
• Maximize detection of mosaic
results
Unilateral Proband
• If blood sample is positive
• Offspring and siblings tested for
mutation identified.
• If tumor positive and blood negative
- confirmed sporadic result:
• Offspring <<1%
• Siblings – population risk
• If blood only is tested with negative
result:
• Offspring <0.5%
• Siblings – <0.1%
Second Malignancies after Rb
• Long term survivors of bilateral Rb have a very high rate of
second malignancies.
• Bone and soft tissue sarcomas are the most common
second primary cancer in childhood.
• Uterine leimyosarcomas and lung cancer seen in adults
• Radiation therapy of RB significantly increases the risk
of malignancy, particularly for sarcomas.
• Cohort study suggest 68% risk of second primary
malignancy through ~age 50 with epithelial tumors, e.g.
lung cancer common in adult RB1 mutation carriers.
274
dWϱϯ Tumor Suppressor Gene
• Guardian of genome, regulates DNA damage responses
(checkpoint, repair and apoptosis)
• Somatic TP53 mutations in cancers are very frequent in
osteosarcoma, breast, colon, pancreatic and other solid
tumors.
• Majority of mutations are missense mutations that interfere
with functional domains.
• Also see deletions other truncating alleles
• Accompanying LOH in ~50% of tumors
• Constitutional mutations associated with Li Fraumeni Cancer
Predisposition Syndrome.
Li-Fraumeni syndrome (LFS)
• First identified in children with sarcomas and family
history of early onset breast cancer.
• Autosomal dominant inheritance of cancer susceptibility
• Small proportion of germline de novo mutations.
• Penetrance nearly 85% for lifetime tumor development:
women greater and earlier than men
• High breast cancer risk w/ 31 average age of diagnosis (NCCN
recommends TP53 testing for breast cancer <age 31)
• 15% of TP53 carriers develop more than one cancer
LFS Pedigree
Breast
dx 35
Lung
dx 45
Sarcoma
dx 35
Sarcoma,
Dx 5
Adrenocortical
Ca, dx 10
Stomach Breast,
dx 38
dx 25
ALL,
Dx 3
45
275
LFS Clinical Diagnostic Criteria Can Aid in
Decision on dWϱϯ testing
• LFS Classic definition
• Sarcoma <45 yrs PLUS
• 1° relative with any cancer <45 yrs
PLUS
• Another 1°/2° relative with any
cancer <45 yrs OR sarcoma at any
age.
• High suspicion: Chompret Criteria for
LFS Diagnosis
• Li-Fraumeni-Like Syndrome (LFL)
• Tumor within LFS spectrum <46 yrs PLUS
1°/2° relative with LFS-related tumor
<56 yrs OR with multiple tumors OR
• Multiple tumors, 2 within LFS spectrum
and 1st tumor <46 yrs OR
• Any adrenal cortical carcinoma or CPT,
regardless of FH
• Less stringent parameters
Tinat et al, JCO, 2009
Molecular Genetics of LFS
• 80% of LFS families have mutations detectable by sequencing TP53.
• Missense variants are the most common pathogenic variant
• 5-10% have deletions of TP53
• TP53 also on most BRCA and other hereditary cancer panels:
• NCCN guidelines: TP53 testing for woman w/ BRCA <age 31
• No other LFS gene has clearly been identified except one paper on
POT1 missense variants and LFS with angiosarcomas.
• CHEK2 is LFS2 in OMIM but data that causes LFS is very slim.
• Mainly associated with breast and colon cancer risk.
Common Tumor Types in Li-Fraumeni Syndrome
• Sarcomas – both soft tissue and osteosarcoma all ages, not Ewing’s sarcoma.
• Particularly rhabdomyosarcoma embryonal anaplastic subtype.
• Breast cancer – most common malignancy in LFS families overall
• Average age of onset ~ 31.
• Leukemias and lymphomas – including hypodiploid ALL
• Adrenocortical carcinoma – in children, not necessarily adult onset.
• 60% of children with ACC have germline TP53 mutation
• Specific Brazilian variant, R337H, with ~10% penetrance for ACC
• Brain tumors – in both children and adults.
• Choroid plexus carcinoma are highly indicative of TP53 mutations.
• GI malignancies including colon cancer, colorectal, laryngeal
• Others: kidney, testicular, head/neck cancers, neuroblastoma
276
Modifiers of cancer risk in dWϱϯmutation
carriers
• p53 pathway (Fang et al PLoS One 2010):
• TP53 codon (P72R) - (RR/RP vs PP carriers age of
cancer diagnosis (21 vs 34.4 years, P = 0.05)
• MDM2 - 309T>G (GG/GT vs TT carriers age of
cancer diagnosis (18.6 vs 27.6 years, P = 0.009)
• Telomere length –short telomeres assoc. w/ greater
risk childhood cancer diagnosis.
• Increasing copy number variation assoc. w/cancer risk.
Surveillance for dWϱϯMutation Carriers – Toronto
Protocol
• Demonstrated to improve morbidity/mortality
• Evaluations q3-4 months:
• Physical exam
• Imaging: abdominal ultrasound (ACC)
• Blood tests: adrenal androgens, AFP, ɴ-hCG (ACC), CBC (leukemia)
• Urine test: Urinalysis (ACC)
• Annual brain MRI (brain tumor)
• Annual whole body MRI (sarcoma other soft tissue tumors)
• Adults include breast MRI (beginning age 18-21) and
colonoscopy beginning age 25.
Recent update of the Toronto Protocol Outcomes
Lancet Oncology, 2016
277
Telomere Disorders: Dyserkatosis congenita
• Disorder due to dysfunctional
telomeres (telopathy)
• Classical clinical triad
• Nail dystrophy
• Oral leukoplakia
• Abnormal (reticulate) skin
pigmentation
• Causes of DC mortality
• Mean age of death 3rd decade
• Bone marrow failure or
immunodeficiency (60-70%)
• Pulmonary complications 10-15%
• Malignancy 20% (including AML
and head and neck cancer in
young adults)
From Savage and Bertuch, Genetics in Medicine, December 2010.
• Diagnosis:
• Clinical Triad (minority of patients)
• 1 or 2 classical features + hypoplastic bone
marrow
• Known pathogenic germline variant
• Very short telomeres (<1st %ile) in 3 or more
lymphocyte subsets using flow cytometry.
• Features of telomere length study
• Test of fresh blood sample is now clinically
available in several centers.
• Needs to be compared with aged matched
controls.
• Flow cytometry allows separation of the
different types of blood cells and then
telomere length is measured for each cell
type in the blood sample.
Telomere components
From Savage and Bertuch, Genetics in Medicine, December 2010.
278
DC: Genetic heterogeneity (at least 10 genes)
Hoyeraal-Hreidarsson
syndrome - DC-related
disorder with
additional features of
IUGR,
immunodeficiency,
and cerebellar
hypoplasia.
Associated with DKC1,
TERT, TINF2, RTEL1,
ACD, or PARN
DKC1 (Dyskerin)
30%
XLR
TINF2 (TIN2)
10-15%
De novo AD mutations,
Revesz syndrome
TERC (RNA
component)
5-10%
AD
TERT (TERT)
5%
AD, AR,
More minor genes
<5% each
NHP2 (NHP2)
AR
NOP10 (NOP10)
AR
TCAB1/WRAP53
AR (compound het missense)
CTC1
AR (Coats Plus syndrome & DKC)
PARN
AR
Pleuro Pulmonaryblastoma (PPB) & /Zϭ
• Rare developmental tumor disorder associated with lung cysts and PPB
• Patients with DICER1 germline mutations also at increased risk for:
•
•
•
•
•
•
•
Multicystic goiter
Cystic nephroma
Ovarian Sertoli-Leydig-type tumors
Wilms tumor
Rhabdomyosarcomas (including ovarian and CNS)
Intraocular medulloepithelioma
Pituitary blastoma and Pineoblastoma
Foulkes et al., Nat Rev Cancer, 2014
Genetics of /Zϭ Syndrome
Pre-miRNA
• Autosomal dominant inheritance w/incomplete
penetrance and variable expressivity.
• Caused by inactivating mutations in DICER1
• First evidence of a cancer susceptibility gene which
encodes a miRNA processing enzyme that cleaves premiRNA
• 2nd hits missense mutations specifically clustered in
the RNAse IIIb domain (not LOF mutations).
• Young children found to carry DICER1 mutations
undergo screening focused on surveillance of the
pulmonary cavity
• Somatic mutations in other miRNA components
(DROSHA) are also seen in Wilms tumor.
• DROSPHA works at an earlier step to cleave pri-miRNA to
form pre-miRNA
Figure from National Institute of General Medical Sciences:
https://ghr.nlm.nih.gov/condition/dicer1-syndrome#genes
279
Genetic Counseling &
Risk Assessment
GENETIC COUNSELING &
RISK ASSESSMENT
Pamela L. Flodman, MSc, MS, LCGC
Adjunct Professor, Pediatrics
School of Medicine
Director, Graduate Program in Genetic Counseling
University of California Medical Center, Irvine
Pamela L. Flodman, MSc, MS, LCGC
Department of Pediatrics
University of California, Irvine
101 The City Drive
Mail Code: 4482
Orange, CA 92868
(714) 456-5789 Telephone
(714) 456-5330 Fax
[email protected]
283
284
Genetic Counseling and Risk Assessment
Pamela Flodman, MSc, MS, LCGC
Adjunct Professor and Director, Genetic Counseling Program
University of California, Irvine
Disclosure(s):
Nothing to disclose
Overview
භ Principles of genetic counseling
භ Ethical, legal and social issues
භ Community services / advocacy
භ Risk assessment
භ Carrier screening
285
y Genetic counseling is the process of helping people understand and adapt to
the medical, psychological and familial implications of genetic contributions
to health and to disease.
y This process integrates:
• Interpretation of family and medical histories in combination with results of genetic
testing, to assess the chance of disease occurrence or recurrence.
• Education about inheritance, testing, management, prevention, resources and
research.
• Counseling to promote informed choices and adaptation to the risk or condition.
Non-directive: Help individuals to make their
own best decisions.
Components of a genetic counseling session
• Case preparation: records, lit review, etc.
• Contracting
–
–
–
–
–
–
Establish rapport
Identify client concerns
Establish an agreed-upon agenda
Assess prior knowledge
Understand the cultural context
Acknowledge and respond to anxiety, anger, emotion
• Obtain and evaluate medical & family history
– Recognize relevant symptoms and findings
– Identify need for specific records and test results
Components of a genetic counseling session
• Interpret family history; risk assessment
– Determine inheritance pattern
– Bayesian modification of risk if necessary
– Apply appropriate risk assessment models
• Explain diagnosis, natural history, inheritance, risk, and reproductive
alternatives
• Discuss options for genetic screening & testing
– Assess prior perceptions
– Decide who and how to test
– Discuss testing options and possible outcomes, including risks / limitations,
and impact on family
– Interpret and explain results
– Determine need for further testing
286
Components of a genetic counseling session
• Psychosocial assessment & support
– Support systems; cultural beliefs and values
– Coping strategies and defense mechanisms
•
•
•
•
Denial; suppression; anger
Projection; displacement
Rationalization; intellectualization
Acceptance
• Identifying resources
–
–
–
–
Educational materials, and address social media
Advocacy and support groups
Public and private agencies
Option to participate in research; clinical trials
• Follow-up
Genetic counseling techniques
• Primary empathy
– A continuum
– Minimal encouragers (including non-verbal)
– Reflection of content and feeling
• Advanced empathy
– Goes beyond what is explicitly stated
– Tentative: explore together with client
• Anticipatory guidance
• Confrontation
– Not commonly used, but can be powerful
Professional issues
භ Transference and counter-transference
භ Ethical and legal issues:
භ Privacy and confidentiality
භ Informed consent
භ Access to resources
භ Professional Codes of Ethics
භ Ethics boards
287
භ Transference: Occurs when a patient redirects or projects feelings that
they have for another person into their relationship with their counselor,
without being aware they have done so. Examples:
භ Anger
භ Mistrust
භ Extreme dependence
භ Counter-transference: Occurs when a counselor redirects feelings for
another person into their relationship with their client.
භ Important to be aware when this happens
භ Can occur in response to transference
GINA: Genetic Information
Non-discrimination Act of 2008
•
Provides a baseline protection to prevent discrimination in health care coverage and
employment based on genetic information.
•
Some states have even more protective laws.
•
Prohibits health insurers from requesting genetic information or using it for decisions re:
coverage, rates, or preexisting conditions.
•
Prohibits most employers from using genetic information for hiring, firing, promotion, or
any decisions regarding employment.
•
Does not extend to life, disability, or long-term care insurance.
•
Employment provisions don’t apply if < 15 employees.
http://www.genome.gov/Pages/PolicyEthics/GeneticDiscrimination/GINAInfoDoc.pdf
Counseling Issues:Pre-symptomatic testing for adult-onset disease
ƒ Reasons for seeking evaluation at this time
ƒ Perceived risk
ƒ Current emotional well-being and mental health history
ƒ Emotional response to family history of disease
ƒ Coping strategies with respect to perceived risk
ƒ Reactions and responses during counseling
ƒ Support mechanisms
ƒ What specifically will the counselee do differently depending on the results
288
Guidelines: Genetic testing in minors
ƒ For detecting conditions in which treatment or preventative measures exist:
• Testing at the earliest age where health benefits accrue
ƒ For determining genetic risk or diagnosis only for reproductive decision-making:
• Minor should be the primary decision maker (assess voluntariness)
ƒ Parents or minor request testing with no immediate benefit to the minor
• Advisable to defer testing to adulthood
• No ethical justification for testing before age 11 or 12
ƒ Testing for the benefit of another family member
ƒ Must have a clear medical benefit
ƒ Both parents and minor must consent / assent
ƒ Reassess
Relevant Practice Guidelines
ƒ National Society of Genetic Counselors
ƒ Genetic cancer risk assessment and counseling
ƒ Counseling of consanguineous couples and their children
ƒ Genetic counseling and testing for FMR1 mutations
ƒ Communicating a prenatal or postnatal dx of Down syndrome
ƒ Position statements on topics including non-discrimination, disability, reproductive freedom,
ƒ … many others
ƒ American College of Medical Genetics
ƒ Carrier screening in individuals of Ashkenazi Jewish descent
ƒ Genetic counseling for advanced paternal age
ƒ Genetic testing and counseling for Alzheimer disease
ƒ Carrier screening for Spinal Muscular Atrophy
ƒ Non-invasive prenatal screening for fetal aneuploidy
ƒ Recommendations for utilization of array-based technology
ƒ … many others
Tools for genetic risk calculation:
භ Mendelian genetics
භ Empiric risk figures
භ Hardy-Weinberg Equilibrium
භ (Linkage analysis)
භ Bayesian calculation
289
Sometimes, there is additional information which must be
included in the calculation of genetic risk:
ƒ Further pedigree information
ƒ E.g., having unaffected sons may reduce but not eliminate the chance that a
woman is a carrier of an XLR mutation.
ƒ Test results
ƒ E.g., a test result that reduces but does not eliminate the chance that a person is a
carrier (such as a screening test result)
ƒ Age of unaffected at-risk individuals
ƒ Reduces but does not eliminate chance the person carries a predisposing mutation
Æ Bayesian analysis, a statistical technique for incorporating
additional information.
Example: Conditional probability
භ A man knows that he has been adopted from a region in which ¼
of the population is Ashkenazic Jewish (and ¾ is not).
භ A particular mutation is carried by 3% of the Ashkenazic
population, and by 1% of the non-Ashkenazic population.
භ If the man finds out that he is a carrier of this mutation, what is
the probability that he is Ashkenazic?
Bayesian analysis:
Graphic representation
Ashkenazic
Not Ashkenazic
The population is
represented by the
rectangle.
•¼ is Ashkenazic
•¾ is not
The proportions carrying
the mutation are shown by
the shaded areas
.03
.25
.75
.01
290
Bayesian analysis:
Graphic representation
Ashkenazic
Not Ashkenazic
The proportion of carriers is
represented by the area of the
shaded boxes:
(.03 x .25) + (.01 x .75)
= 0.015
Probability of being Ashkenazic given
that he is a carrier is:
(.03 x .25)
(.03 x .25) + (.01 x .75)
= 1/2
.03
.01
.75
.25
Bayes’ theorem
P(A|B) =
P(A ∩ B)
P(B)
(Statistical statement of Bayes’ theorem)
Bayes’ theorem
(Another way to state Bayes’ theorem)
291
Bayesian calculations: A method
of combining probabilities
භ Consider 2 or more alternative hypotheses.
භ Start with a prior probability for each.
භ Bring in relevant evidence to support or oppose each hypothesis:
conditional probabilities.
භ Combine these to obtain overall posterior probabilities.
Prior probability
Hypothesis 1
Hypothesis 2
He is Ashkenazic
He is not
.25
.75
Hypothesis 1
Hypothesis 2
He is Ashkenazic
He is not
.25
.75
.03
.01
Conditional
probability
(carrier)
Joint probability
Posterior
probability
Prior probability
Conditional
probability
(carrier)
Joint probability
Posterior
probability
292
Hypothesis 1
Hypothesis 2
He is Ashkenazic
He is not
.25
.75
.03
.01
.25 x .03 = .0075
.75 x .01 = .0075
Hypothesis 1
Hypothesis 2
He is Ashkenazic
He is not
.25
.75
.03
.01
.25 x .03 = .0075
.75 x .01 = .0075
Prior probability
Conditional
probability
(carrier)
Joint probability
Σ=
.015
Posterior
probability
Prior probability
Conditional
probability
(carrier)
Joint probability
Posterior
probability
.0075
.015
= 1/2
.0075
.015
Σ=
.015
= 1/2
Bayesian Calculations: Steps
1. Write down all possible mutually exclusive hypotheses (one to each column).
2. Assign a prior probability to each.
3. Consider the additional information. For each column, determine the
likelihood of this observation IF the hypothesis is true (conditional
probabilities).
4. Multiply down the columns Æ joint probabilities.
5. Scale the joint probabilities so that they sum to 1. These are the posterior
probabilities.
293
Hypothesis 1
Hypothesis 2
Event “A”
“Not-A”
P(A)
P(Not-A)
P(B | A)
P(B | Not-A)
P(B | A) x P(A)
P(B | Not-A) x P(Not-A)
Prior probability
Conditional
probability
“B”
Joint probability
Posterior
probability
P(A | B) =
P(Not-A | B) =
P(B | A) x P(A)
P(B)
P(B | Not-A) x P(Not-A)
P(B)
X-linked recessive: Example 1
II:1
I:1
I:2
II:2
II:3
: Duchenne muscular
(DMD)
dystrophy
Mrs. Y
IV:1
III:1
III:2
IV:2
IV:3
Prior probability
III:3
III:4
IV:4
What is the probability that
Mrs. Y is a carrier?
Hypothesis 1
Hypothesis 2
Mrs. Y is a carrier
Not a carrier
1/2
1/2
Conditional
probability
(3 unaff sons)
Joint probability
Posterior probability
294
Hypothesis 1
Hypothesis 2
Mrs. Y is a carrier
Not a carrier
1/2
1/2
(1/2)3 = 1/8
1
Hypothesis 1
Hypothesis 2
Mrs. Y is a carrier
Not a carrier
1/2
1/2
(1/2)3 = 1/8
1
Prior probability
Conditional
probability
(3 unaff sons)
Joint probability
Posterior probability
Prior probability
Conditional
probability
(3 unaff sons)
Joint probability
1/2 x 1/8 = 1/16
1/2 x 1 = 1/2
= 8/16
Σ=
9/16
Posterior probability
Hypothesis 1
Hypothesis 2
Mrs. Y is a carrier
Not a carrier
1/2
1/2
(1/2)3 = 1/8
1
Prior probability
Conditional
probability
(3 unaff sons)
Joint probability
Posterior probability
1/2 x 1/8 = 1/16
1/16
9/16
= 1/9
1/2 x 1 = 1/2
= 8/16
8/16
= 8/9
9/16
Σ=
9/16
295
For the next example:
• Sometimes we need to consider more than two alternative
possibilities
• For example, if there is information to incorporate from multiple
generations
X-linked recessive: Example 2
: Duchenne muscular
(DMD)
Mom
A
Daughter
B
C
dystrophy
What is the probability that Mom is a carrier?
That Daughter is a carrier?
At conception, a 50% chance that Mom inherited the mutation.
Her risk is reduced because she has an unaffected son, and grandson.
Mom:
Daughter:
carrier
carrier
carrier
not carrier
not carrier
not carrier
Prior probability
Conditional
A is unaff.
B is unaff.
Joint probability
Posterior
probability
296
Mom:
Daughter:
carrier
carrier
carrier
not carrier
not carrier
not carrier
½x½
½x½
½x1
carrier
carrier
carrier
not carrier
not carrier
not carrier
Prior probability
½x½
½x½
½x1
Conditional
A is unaff.
B is unaff.
½
½
½
1
1
1
carrier
carrier
carrier
not carrier
not carrier
not carrier
Prior probability
½x½
½x½
½x1
Conditional
A is unaff.
B is unaff.
½
½
½
1
1
Prior probability
Conditional
A is unaff.
B is unaff.
Joint probability
Posterior
probability
Mom:
Daughter:
Joint probability
Posterior
probability
Mom:
Daughter:
Joint probability
1
(½)4 =
(½)3 =
½
1/16
1/8 = 2/16
= 8/16
Σ =
11/16
Posterior
probability
297
Mom:
Daughter:
carrier
carrier
carrier
not carrier
not carrier
not carrier
Prior probability
½x½
½x½
½x1
Conditional
A is unaff.
B is unaff.
½
½
½
1
1
Joint probability
Posterior
probability
Mom:
Daughter:
1
(½)4 =
(½)3 =
½
1/16
1/8 = 2/16
= 8/16
1/16
11/16
= 1/11
2/16
11/16
= 2/11
8/16
11/16
= 8/11
carrier
carrier
carrier
not carrier
not carrier
not carrier
Prior probability
½x½
½x½
½x1
Conditional
A is unaff.
B is unaff.
½
½
½
1
1
Joint probability
Posterior
probability
1
(½)4 =
(½)3 =
½
1/16
1/8 = 2/16
= 8/16
1/16
11/16
= 1/11
2/16
11/16
= 2/11
8/16
11/16
Σ =
11/16
Σ =
11/16
= 8/11
P(Mom is a carrier) = 1/11 + 2/11 = 3/11 = 27%
P (Daughter is a carrier) = 1/11 = 9%
For the previous examples:
• We knew that an XLR mutation was segregating in the family,
because there were multiple affected individuals
• Æ We could start with a Mendelian risk for the prior probability
• BUT – in a family in which there is an ISOLATED case of an affected
male, we also have to take into account the possibility that this
occurred as a result of a de novo mutation
298
X-linked recessive: Example 3
Mom
Daughter
: Duchenne muscular
(DMD)
dystrophy
Son
No other family hx of DMD
What is the probability that Mom is a carrier?
Mom may be a carrier of a DMD mutation which she passed to her son.
Alternatively, her son may have a new mutation.
Three fundamental probabilities:
ʅ = mutation rate per locus per generation (per meiosis)
ʅ is very small
N
ʅ
N
2ʅ
4ʅ
(New μ from father or
mother)
Digression: deriving 4ʅ
There are three possible ways that a woman in the general
population is a carrier of the XLR mutation, if you know nothing
about her parents:
ƒ New mutation on the X from her mother: ʅ
ƒ New mutation on the X from her father : ʅ
ƒ Her mother is a carrier and passed the mutation to the woman.
Let C be P(woman in general pop’n is a carrier)
C = ʅ + ʅ + 1/2 C
1/2 C = 2 ʅ
C=4ʅ
299
X-linked recessive: Example 3
Mom
Daughter
: Duchenne muscular
(DMD)
dystrophy
Son
No other family hx of DMD
What is the probability that Mom is a carrier?
Mom may be a carrier of a DMD mutation which she passed to her son.
Alternatively, her son may have a new mutation.
Prior probability
Mom is a carrier
Mom is not a carrier
4ʅ
1 - 4ʅ у 1
Mom is a carrier
Mom is not a carrier
4ʅ
1 - 4ʅ у 1
½
ʅ
Conditional
probability
1 aff’d son
Joint probability
Posterior
probability
Prior probability
Conditional
probability
1 aff’d son
Joint probability
Posterior
probability
300
Prior probability
Conditional
probability
1 aff’d son
Joint probability
Mom is a carrier
Mom is not a carrier
4ʅ
1 - 4ʅ у 1
½
ʅ
2ʅ
ʅ
Mom is a carrier
Mom is not a carrier
4ʅ
1 - 4ʅ у 1
½
ʅ
2ʅ
ʅ
Σ =
3ʅ
Posterior
probability
Prior probability
Conditional
probability
1 aff’d son
Joint probability
Posterior
probability
2ʅ
3ʅ
ʅ
3ʅ
= 2/3
Σ =
3ʅ
= 1/3
AD w/ age-dependent penetrance
: Huntington disease
(onset in early-mid 40s)
No other family hx of DMD
50 y.o.
What is the probability that your 50 y.o. patient has
inherited the HD mutation?
Cumulative incidence: 75% of individuals with an HD
mutation have onset by age 50
301
Prior probability
Carrier
Not a carrier
1/2
1/2
Carrier
Not a carrier
1/2
1/2
0.25
1
Carrier
Not a carrier
1/2
1/2
0.25
1
.5 x .25 = .125
.5 x 1 = .5
Conditional
probability
Unaff at age 50
Joint probability
Posterior
probability
Prior probability
Conditional
probability
Unaff at age 50
Joint probability
Posterior
probability
Prior probability
Conditional
probability
Unaff at age 50
Joint probability
Posterior
probability
Σ =
.625
302
Prior probability
Conditional
probability
Unaff at age 50
Joint probability
Posterior
probability
Carrier
Not a carrier
1/2
1/2
0.25
1
.5 x .25 = .125
.5 x 1 = .5
.125
.625 = 1/5
.5
.625
Σ =
.625
= 4/5
Heterozygote carrier screening and testing:
• Bayesian analysis is also used to incorporate the results of
carrier testing, when the testing is not fully predictive of
carrier status
• I.e. when sensitivity and/or specificity is < 1
• With a negative test result, a residual risk may remain
Carrier testing in a family
with an AR disorder
Mr. G had a brother with CF; no molecular testing was
done for the brother.
Mr. G chooses to have carrier testing.
Mr. G
The mutation panel detects 93% of CF mutations in the
Caucasian population (sensitivity), with a specificity of
> 99.9%.
Two possible results:
: cystic fibrosis
Ethnicity: Caucasian
Mutation detected
Mr. G is a carrier
No mutation detected
Bayesian calculation needed
303
Prior probability
Hypothesis 1
Hypothesis 2
Mr. G is a carrier
Not a carrier
2/3
1/3
Hypothesis 1
Hypothesis 2
Mr. G is a carrier
Not a carrier
2/3
1/3
0.07
0.999
Hypothesis 1
Hypothesis 2
Mr. G is a carrier
Not a carrier
2/3
1/3
0.07
0.999
2/3 x 0.07 = 0.047
1/3 x 0.999 = 0.333
Conditional probability
(neg carrier test)
Joint probability
Posterior probability
Prior probability
Conditional probability
(neg carrier test)
Joint probability
Posterior probability
Prior probability
Conditional probability
(neg carrier test)
Joint probability
Σ=
0.380
Posterior probability
304
Hypothesis 1
Hypothesis 2
Mr. G is a carrier
Not a carrier
2/3
1/3
0.07
0.999
2/3 x 0.08 = 0.047
1/3 x 0.999 = 0.333
Prior probability
Conditional probability
(neg carrier test)
Joint probability
Posterior probability
0.047
0.380
= 12%
0.333
0.380
Σ=
0.380
= 88%
Strategies for problem solving
ƒ Determine whether a Bayesian approach is needed
ƒ “If”, “Given that”, “Among”, “Conditional on”
ƒ Lay out the alternate hypotheses
ƒ Identify all of the relevant information
ƒ Prior probabilities
ƒ Conditional information
ƒ Do the math – avoid the common mistakes
ƒ Check your answer against common sense
ƒ Practice working problems through, and identify your local resources!
305
Biochemical Genetics I
BIOCHEMICAL GENETICS I
Gerard Berry, MD, FACMG
Harvey Levy Chair in Metabolism Director, Metabolism Program
Division of Genetics and Genomics Boston Children’s Hospital
Professor of Pediatrics, Harvard Medical School
Gerard Berry, MD, FACMG
Harvard Medical School
Center for Life Science
Building, Suite 14070
3 Blackfan Circle
Boston, MA 02115
(617) 355-4316 Telephone
(617) 730-4874 Fax
[email protected]
309
310
Biochemical Genetics I
Gerard T. Berry, MD, FACMG
Founding Fellow ACMG
Division of Genetics and Genomics
Boston Children’s Hospital
Harvard Medical School
Director of Harvard Medical School BG training program
ACMG Genetics and Genomics Review Course
June
20-23, 2013
ACMG Genetics and Genomics Review Course
June
20-23, 2013
Nothing to disclose
Inborn Errors: A Few Principles
• Rare monogenic disorders typically with extensive allelic
heterogeneity
• Complex interactions of involved proteins
• Integrate into pathways, cycles, organelles
• Genetic and environmental modifiers
• Example: varying severity with age of onset
• Early onset – “classic” severe phenotype
• Later onset – milder variants and phenotypic spectrum
311
Some Terms and Abbreviations
• Sx – symptoms
• PFVLCSz – poor feeding, vomiting, lethargy, coma,
seizures
• Dx – diagnosis
• Ge – genetics (AR=autosomal recessive; AD - autosomal
dominant; XL=X-linked)
• Rx – treatment
• PAA – plasma amino acids; UOA – urine organic acids;
ACP – plasma acylcarnitine profile
• IDD – intellectual disability
Acute Metabolic Disease – The Newborn Crash
• Consider in neonate with presumed sepsis or acidosis;
older child with acidosis, lethargy
• Usually term infant, good Apgars, well interval
• Family Hx usually negative – may see consanguinity, XL
pedigree, hx sibling deaths
• Sx – generally non-specific with poor feeding, irritability,
vomiting, seizures, progressing to lethargy, coma, apnea
(PFVLCSz)
• Occasionally specific sx/clues – urine odors, skin/hair
(see Appendices)
Differential Diagnosis
• Amino acid disorders - MSUD
• Urea cycle disorders - all except arginase
• Organic acid disorders
• Carbohydrate disorders - congenital lactic acidosis,
fructose disorders, occ. GSDs
• Mitochondrial disorders, FAO disorders
• Other - sepsis, adrenal insufficiency, CHD, asphyxia
312
Acute Metabolic Disease
• Initial labs - sepsis w/up with CBC, lytes, anion gap
(AG), NH3, lactate, glucose, urine ketones; check
the NBS results
• STAT PAA and UOA; ACP; genetics consult
• Acute rx - high IV glucose; eliminate protein,
lactose, fructose; hemodialysis for NH3 >400 to 500
or high leucine; ?vitamins, carnitine
• Specific therapy tailored to individual disease
Neonatal Hemodialysis
Urea Cycle Disorders
• All AR except OTC which is X-linked; overall incidence
~1/30,000
• Sx – all can present in neonatal period with NH3
except arginase deficiency; milder partial defects exist
• AS lyase deficiency often develop
hepatomegaly/fibrosis
• Arginase deficiency – spastic diplegia; IDD
313
Urea Cycle
Orotic acid
NH3 + HCO3
+
CAP
CPS I
OTC
N-Ac Glu
Citrulline
Ornithine
NAG
synthetase
Glutamate
ASA
synthase
Arginase
Urea
Arginine
ASA
lyase
ASA
X-linked Pattern of Inheritance
Biochemical Diagnosis in Urea Cycle Disorders
භ Dx -
NH3, acidosis only late in course
භ Plasma amino acids – elevated glutamine; citrulline level is
key
Cit is
absent in CPS/OTC
in citrullinemia
in AL deficiency
භ Urine orotic acid - in OTC and low in CPS
භ Confirmation by DNA (or enzyme assay)
314
Urea Cycle Disorders
භNBS - cit , arg; some states screen for low ( ) cit or
gln/cit ratio to detect OTC, CPS, NAGS deficiency
භRx - low protein diet; meds remove toxic NH3 via alternate
pathways; arg or cit replacement; liver transplant for OTC
භIV ammunol (Na benzoate and Na phenylbutyrate) – for
acute hyperammonemia
භBuphenyl (Na phenylbutyrate) – oral powder
භRavicti (glycerol phenylbutyrate) – liquid NEW!
Removal of NH3 via Alternate Pathways
(phenylbutyrate)
(ravicti)
NH3 =
From Gelehrter & Collins, Principles of Medical Genetics, 1990, Fig. 7-20
OTC Heterozygotes
භMay be symptomatic in late infancy or childhood; can
present with lethargy out of proportion to degree of
illness
භDx - NH3 and orotic acid increased when symptomatic;
allopurinol or protein challenge; molecular testing
best way to diagnose
භRisk of post-partum coma in OTC heterozygotes
315
Urea Cycle Disorders
භOutcome - virtually all have MR with
neonatal onset; outcome depends on
duration of hyperammonemia
භPrenatal dx – by DNA for all with known
mutations; enzyme assay (CVS, amnio) for AS,
AL, arginase; OTC & CPS only expressed liver,
intestine
Mitochondrial Carrier Family Transporters Involved in the Urea Cycle
Aspartate /
glutamate
Transporter
“Citrin”
SLC25A13
Mitochondrion
NH4 + HCO3
TCA cycle
OAT
Ornithine
Transporter
ORNT1
CAP
Ornithine
OTC
Ornithine
Urea
Arginine
Citrulline
Citrulline
Asp
ASA
Cytoplasm
Other Disorders Related to the Urea Cycle – Transporter
Disorders
භHHH syndrome
භSx - episodic NH3
භDx - ornithine, homocitrulline
භGe – AR; gene ORNT1; redundant fx of related
genes
භRx – low protein; + citrulline
භType II citrullinemia (citrin def)
භSx – neonatal hepatitis, adult-onset citrullinemia;
භDx - orn, NH3
භGe – AR; gene SLC25A13
භ Rx – low protein
316
Disorders of Ornithine Metabolism-Gyrate Atrophy
භSx - blindness & chorioretinal degeneration
භDx – PAA with orn, nl NH3
භGe - AR defect in OAT enzyme
භRx - low arg diet
Glutamate
1
Δ - P5C
Urea
cycle
OAT
Retina
Ornithine
Proline
Disorders of Ornithine Metabolism – Creatine Deficiency Syndromes
භSx - MR, seizures, hypotonia, autism
භDx - plasma and/or urine guanidinoacetic acid (GAA);
MRI with MRS (Cr); molecular
භGe - 2 very rare AR (AGAT, GAMT); XL Cr transporter
(SLC6A8; ?~1% XL MR)
භRx - Cr (AGAT, GAMT); low arg + orn (GAMT); ?none for
XL transporter deficiency
AGAT = arginine:glycine amidinotransferase deficiency
GAMT = guanidinoacetate methyltransferase deficiency
CNS
Muscle
SAM
Creatine
Creatine
transporter
GAA
Ornithine
GAMT
AGAT
Glycine
Arginine
GAMT
AGAT
SLC6A8
Urine GAA
High
Low
Nl
Urine Creatine (Male)
Low
Low
High
Nl
Nl
High
High
Low
Nl
Urine Creatine/Creatinine
Plasma GAA
317
Alkaptonuria
භOne of original inborn errors
recognized by Garrod
භSx - osteoarthritis; ochronosis
භAR; deficiency of homogentisic acid
oxidase in tyr pathway
භDx - urine dark upon standing;
homogentisic aciduria
භRx - ?high dose ascorbic acid; ?NTBC
Homogentisic
Acid
MAA
HGA
oxidase
Cystinosis
භSx (infantile or nephropathic) – onset <1 yr; FTT, renal
Fanconi, low PO4 rickets, fair hair and skin,
photophobia, corneal erosions, hypothyroidism (~age
10), nl IQ, delayed puberty, diabetes, male infertility
භJuvenile form (onset 5 -10 yrs) - Nl hair & skin
භAdult onset – Mild eye disease only
Cystinosis
භAR; defect in egress of cystine (cys-cys) from lysosome;
gene (CTNS) - cystine transporter
භDx - +/- nitroprusside; plasma cys nl; assay cys content in
WBC; molecular
භRx - cysteamine; symptomatic for renal disease
භLate sx post transplant (non-renal) include myopathy, CV &
GI disease, resp failure; need to continue cysteamine rx
318
Nonketotic Hyperglycinemia
භSx (classic infantile) – most common form; intractable
seizures (may have prenatal onset), hypotonia, severe
IDD
භRare later onset (milder) variants as well as transient
form resolving by 8 wks
භGe - AR; defect in glycine cleavage enzyme (4 subunits –
P*, T*, H, L )
Nonketotic Hyperglycinemia
භDx - gly in CSF; plasma gly usually ; CSF:plasma gly ratio
> 0.08 is diagnostic; molecular
භRx
භNa benzoate (excrete gly as hippurate)
භlow protein diet
භdextromethorphan (inhibits CNS NMDA receptors)
භnone very effective
Glutaric Aciduria Type I
භSx - macrocephaly; acute encephalopathic crises produce
IDD, dystonia; MRI with basal ganglia changes and cortical
atrophy
භAR; defect in lys metabolism (glutaryl CoA
dehydrogenase)
භDx - urine organics with elevated glutaric,
3-OH glutaric; on NBS panels (
C5-DC)
භRx - low protein diet, carnitine, riboflavin
319
Canavan Disease
භGe - AR; increased incidence Ashkenazi Jewish pop (~1/40
carrier frequency); deficiency ASPA
භDx - N-acetylaspartic acid (NAA) on UOA, NAA on MRS;
3 mutations account for~99% Jewish alleles (E285A 84%)
භRx -Sx - onset 3-5 mo; hypotonia, progressive
leukodystrophy, macrocephaly; later optic
atrophy, seizures
භ None; Phase II gene therapy clinical trial
N-acetylaspartate
L-aspartate + acetate
Aspartoacylase (ASPA)
(reaction occurs in CNS only)
Mitochondrial Fatty Acid Oxidation
භ During fasting FAO provides up to 80% total body energy needs
භ Long chain (LC) fats preferred substrate for cardiac & skeletal
muscle
භ LC free fatty acids (FA; C18, C16) released from TG in adipose
tissue
භ Peripheral tissues oxidize FA to CO2 and H2O Liver oxidizes FA to
ketone bodies for energy for gluconeogenesis and ureagenesis
භ Ketone bodies used as fuel in CNS
Mitochondrial Fatty Acid Oxidation
Medium
Chain FA
LC
Ac-CoA
Carnitine
Cycle
LC
Ac-CoA
mitochondrion
ß-Oxidation
Cycle
TCA
Ac-CoA
Electron
transfer
Ketone
synthesis
Four components of fatty acid oxidation:
Carnitine cycle
ß-oxidation cycle
Electron transfer
Ketone body synthesis
320
Fatty Acid Oxidation Disorders (FAO)
භLiver - glu, ketones, LFTs/NH3, coagulopathy,
enlarged liver, fasting intolerance
භCardiac - cardiomyopathy, arrhythmias
භSudden death - liver (hypoglycemia), cardiac
භSkeletal disease
භ Acute - rhabdomyolysis
භChronic – weakness, fatigue, lactic acidosis
භ Risk HELLP (hemolytic anemia with elevated LFTs and
low platelets) in females with LCHAD fetus
භRx - avoid fasting; low fat diet (Lipistart, Monogen);
carnitine
Mitochondrial FAO: Carnitine Cycle
Carnitine
Extracellular
fluid
Plasma
Membrane
Carnitine Cycle Defects - Clinical
CU
Liver
Heart
CU
+
+
CPTI
+
CT
+
+
CPTII
+
+
Cytosol
Skel muscle
Acute Chronic
Fatty acid
Outer Mito
Membrane
CPT I
(+)
Carnitine
Acylcarnitine Carnitine
Acyl-CoA
Inner Mito
Membrane
CT
+
(+)
+
CPT II
Acyl-CoA
Carnitine
Acylcarnitine
Carnitine
Mito
matrix
ß-Oxidation
cycle
Mitochondrial FAO: ß-Oxidation Spiral
Clinical phenotype
ß-Oxidation Spiral
Liver
Heart
Skel muscle
Acute
Acyl-CoA
Ac-CoA
1
4
3 keto
Acyl-CoA
Enoyl-CoA
2
3
3 OH
Acyl-CoA
1.
Acyl-CoA
dehydrogenase
3.
3-OH Acyl-CoA
dehydrogenase
Chronic
Acyl-CoA
dehydrogenases
VLCAD
+
MCAD
+
+
+
SCAD
+
3-OH Acyl-CoA
dehydrogenases
LCHAD
SCHAD
+
+
+
+
+
321
Mitochondrial FAO: Ketone Synthesis
ß-OxidationSpiral
Acetoacetyl-CoA
Clinical phenotype
Ac-CoA
Onset in infancy or childhood
Hypoketotic hypoglycemia
1
HMG-CoA
2
Acetoacetate
3
4
βȕ-Hydroxybutyrate
Acetoacetyl-CoA
1.
2.
3.
4.
HMG CoA synthase
HMG CoA lyase
SCOT
D-3-OH-butyrate dehydrogenase
FAO Genetics
භAt least 25 enzymes and 22 distinct disorders
භAll AR – MCAD most common (~1/10-15,000); others rare
භDx – ACP; UOA (short chains); carnitine levels (<10 in
Carnitine Uptake Deficiency, CUD)
භMolecular
භLCHAD common mutation (>80% alleles)
භ2 common SCAD polymorphisms 7% population
PA and MMA Metabolism
Gut bacteria
Ile, Val
Met, Thr
Odd chain FA
Propionate
Propionyl CoA
PCC, Biotin
D-Methylmalonyl CoA
L-Methylmalonyl CoA
Mutase, B12
Succinyl CoA
Adapted from SIMD NAMA
322
Branch Chain Amino Acid Catabolism
Leucine
Isoleucine
Valine
Keto-isocaproic
Keto-methylvaleric
Keto-isovaleric
MSUD
Isovaleryl-CoA
IVA
Methylcrotonyl-CoA
MCC
Methylglutaconyl-CoA
3-OH-3methylglutaryl-CoA
Acetoacetate
Methylbutyryl-CoA
Isobutyryl-CoA
Tiglyl-CoA
3OH-Isobutyryl-CoA
Methyl-3OH-butryl-CoA
3OH-Isobutyric acid
2-Methylacetoacetyl-CoA
Methylmalonic
semialdehyde
Acetyl-CoA
Malonyl-CoA
Acetyl-CoA
Propionyl-CoA
PA
Methylmalonyl-CoA
MMA
Succinyl-CoA
Methylmalonic Acidemia (MMA)
භ Sx - PFVLCSz; later onset with hematologic sx (macrocytic
anemia), altered gait & cognition in some Cbl (B12)
defects
භ Dx - Metabolic acidosis, NH3; PAA: Gly;
homocysteine (cbl C,D,F); UOA: MMA, PA metabolites &
ketones
භ Rx - like PA; OH-B12 for cbl defects
Methylmalonyl-CoA
mutase
Succinyl-CoA
AdoCbl
L-Methylmalonyl-CoA
Cobalamin Metabolism
From OMMBID, Ch155fg12
323
Methylmalonic Acidemia Genetics
භMethylmalonyl-CoA mutase - adenosylcobalamin
(AdoCbl) co-factor
භMut0 - (0 enzyme activity); mut- - residual activity
භLocus heterogeneity
භcbl A – MMA only
භcbl B – MMA only
භcbl C - combined MMA & homocystinuria, relatively
common
භcbl D & F – combined MMA and homocystinuria, rare
භcblE, G – homocystinuria only
PA and MMA: Some Complications
භBone marrow suppression during acute crises and as
part of chronic disease
භSevere feeding difficulties
භProgressive renal disease
භCardiomyopathy
භBasal ganglia infarcts
“metabolic strokes”
භPancreatitis
භEye and vision problems
Treatment of MMA and PA
භRx - low protein (Propimex); restrict precursors
[Val, Met, Ile, Thr, odd chain FA (VOMIT)];
carnitine, biotin, ?metronidazole, carbiglu
324
Isovaleric Acidemia (IVA)
භSx- PFVLCSz; odor of sweaty feet
භDx- Metabolic acidosis; Anion Gap; NH3; UOA:
isovaleric & 3-OH-isovaleric acid; isovaleryl-glycine;
ketones
භGe- AR deficiency of Isovaleryl CoA dehydrogenase
භRx- low protein + formula (Valex), glycine, (carnitine)
Metabolic Disorders of Morphogenesis
භGroup of inherited disorders in which major malformations
occur prior to birth
භImply lack of product or presence of increased substrate or
metabolite is teratogenic
භExamples
භCholesterol biosynthetic disorders (SLOS)
භGeneralized peroxisomal disorders (Zellweger syndrome)
භSevere MADD/ glutaric acidemia Type II (an FAOD)
භPyruvate dehydrogenase deficiency
භSome mitochondrial disorders
Cholesterol Biosynthetic Disorders
භ 10 disorders reported
භ Mevalonic aciduria, Hyper IgD
syndrome
භ CHILD syndrome
භ X-linked dominant chondrodsyplasia
punctata (CDPX2)
භ Lathosterolosis
භ Smith-Lemli-Opitz syndrome (SLOS)
භ Desmosterolosis
භ SC4MOL deficiency
භ Lanosterol demethylase
deficiency
භ (Antley-Bixler) – defect
in P450 oxidoreductase
භ HEM dysplasia
Dx: urine organics (mevalonic aciduria, Hyper IgD);
plasma sterol profile for others
325
Smith-Lemli-Opitz Syndrome
භMost common disorder
භSx – MR/autism, microcephaly, hypotonia, FTT, 2-3 toe
syndactyly, ptosis, cataracts, hypogenitalism, cleft palate,
occ. CDP
භDx – low cholesterol; elevated 7-DHC
භRx – cholesterol supplementation
may improve growth & behavior;
no effect on development
From Porter & Herman,
Jl Lipid Res., 52: 6-34,
2011.
CDPX2; Happle Syndrome
භ X-linked dominant, us. male lethal
භ Sx - epiphyseal calcification (CDP), asymmetric
dwarfing; asymmetric cataracts/ microphthalmia;
linear hyperkeratotic erythroderma, alopecia
භ Distinct phenotype (MR, hypotonia, szs, CNS &
other malformations) in >10 males with
hypomorphic mutations
භ Dx - plasma sterol analysis; molecular defect in Δ8−Δ7 sterol isomerase (EBP)
Disorders of Carbohydrate Metabolism
භCongenital lactic acidosis
භ(Galactosemia – see NBS)
භDisorders of fructose metabolism
භGlycogen storage disorders
326
Congenital Lactic Acidosis
භIncludes some CHO (F-1,6-bisphosphatase def), disorders
of pyruvate metabolism (PDH, PC), & some mitochondrial
(mt) disorders
භSpectrum of clinical phenotypes
භSevere neonatal lactic acidosis +/- congenital
malformations (agenesis of corpus callosum)
භChronic with MR, szs, hypotonia
භEpisodic +/- FTT, MR
භDx - elev. plasma/CSF lactate, PAA: ala; normal lactate to
pyruvate ratio (PDH/PC); hypoglycemia (PC; F-1,6-BP);
molecular
Congenital Lactic Acidosis
භPDH: Most common cause
භ5 proteins in complex (E1ɲ,E1ɴ,E2,E3,X, PDP)
භMutations in XL E1ɲ most frequent; most de novo; males
with hypomorphic muts or lethal; females often with sx
භF-1,6-BP
භ½ have 1st sx <1 week of age
භDo not need to ingest fructose
භSx – acidosis, glu, +ketones, liver with dysfx , FTT,
hypotonia
භRx - poor outcome for neonatal cases; biotin and high CHO
for PC; low CHO (ketogenic diet), thiamine for PDH; limit
fructose & rx acute attacks for F-1,6-BP
Hereditary Fructose Intolerance
භSx - Asymptomatic in the absence of dietary fructose;
Nausea, vomiting, hypoglycemia & metabolic acidosis
following fructose intake (sucrose, fruits, honey); occ
chronic sx - FTT
භDx - Controlled fructose load; AR defect aldolase B (A149P
accounts for ~70% of mutant alleles)
භRx - Fructose avoidance; tolerance may improve with age
ATP
Fructose
Aldolase B
F1P
DHAP + Glyceraldehyde
327
Glycogen Metabolism & Liver GSDs
Glycogen
1,4
1,6
Debrancher GSD III
Brancher - GSD IV
1,4
Glycogen
synthase
Phosphorylase
GSD VI
GSD 0
G1P
Glucokinase
G6P
G6Pase
complex
Lactate
Glucose
Brain & other
peripheral
tissues
GSD I
Pyr
TCA cycle
Glycogen Storage Disease (GSD) - Type I
භTwo types: Ia – glucose-6-phosphatase deficiency
Ib – translocase deficiency
භSx Type Ia – most severe of GSDs with glu,
hepatomegaly, short stature; irritable infant with chubby
cheeks; epistaxis and bleeding
භSx Type Ib – same + neutropenia and chronic
ER
infections, oral/intestinal ulcerations, chronic G6P G6Pase Glucose + Pi
inflammatory bowel disease
G6P translocase
G6P
භLate complications - osteopenia
cytosol
and fxs; gout; pancreatitis;
Glycogen
renal disease; hepatic adenoma/Ca
GSD - Type I
භ Dx - hypoglycemia, elev. TG/chol, lactate, uric acid; no response to
glucagon; LFTs usually nl; neutropenia with GSD Ib; molecular dx
(liver bx often not necessary now)
භ Ge - Both Ia and Ib are AR
භ Type Ia (G6PC gene) ~80-90% and
Type Ib (SLC374A gene) ~10-20%
භ Some common mutations with founder effects
භ Rx - freq. feeds of complex carbs, cornstarch/ Beneprotein; avoid
lactose, fructose, gal; DDAVP for bleeding, G-CSF for Ib neutropenia;
allopurinol for uric acid; monitor renal fx; liver US and AFP for
adenomas, liver Ca
328
GSD 1A
From OMMBID, Ch71Fg8
GSDs - Type II Pompe Disease
භSx - hypotonia, cardiomegaly, large tongue, hepatomegaly;
juvenile form with progressive muscle weakness
භGe - Lysosomal storage disease (α-glucosidase deficiency); AR
භDx - elev. CPK, massive QRS on EKG; enzyme assay or
molecular for dx
භRx - enzyme replacement therapy
භLate sx with ERT in infantile
Pompe – hearing loss,
osteopenia, motor/speech
delay, GE reflux/dysphagia
329
GSD Type III Debrancher Deficiency
භSx - hepatic sx like Ia but often milder; muscle involved
(hypotonia); liver sx often resolve with puberty; adult
cardiac/skeletal myopathy
භDx - lactate, uric acid nl; elev. CPK and LFTs; molecular or
enzyme assay fibroblasts, liver
භGE - AR; most pts with defective enzyme in liver & muscle
Glycogen
(IIIa);10-15% liver only (IIIb) with nl CPK
(ass’d with 2 early frameshift, nonsense
1,6
1,4
1,4
mutations and truncated protein)
භRx -like Ia, but can have some fructose, galactose Debrancher
deficiency
Phosphorylase Related Defects (GSD VI, IX)
භDefects of phosphorylase itself (Type VI) or
components of phosphorylase kinase (Type IX)
භSx - hepatomegaly and short stature; mild
hypoglycemia; some with more severe sx
භMost common form of phosphorylase
kinase deficiency (IX) is X-linked (“short
males with pot bellies”); rest AR
Glycogen
1,4
1,6
1,4
Glycogen
synthase
Phosphorylase
system
G1P
භPrognosis good for most &
many require no rx
Glucokinase
G6P
Lactate
G6Pase
complex
Glucose
Pyr
Other forms of GSD in Appendix
Disorders of Metal Metabolism - Menkes Disease
භX-linked; ~1/100,000
භSx - hypotonia; szs; depigmented steely, brittle hair;
FTT; typical facies; hypothermia; lax skin; tortuous
vessels; osteoporosis; death by age 2
භCation transporting ATPase (ATP7A gene); defective Cu
uptake placenta, gut, BBB
භDx - low ceruloplasmin and low serum Cu; occ. elevated
lactate
භRx - symptomatic; early rx with Cu histidinate improves
outcome in less severe cases
330
Menkes Disease
Pili torti
Wilson Disease
භAR; ~1- 4/100,000
භSx - hepatic with jaundice, hepatic failure, hemolysis, bone
disease or neurologic with dementia, intellectual loss,
dysarthria, basal ganglia sx; Kayser-Fleischer rings (eye)
භDx - low ceruloplasmin, increased Cu in urine and liver
භLiver ATPase; H1069Q accounts for ~40% N. Europe
alleles; >100 mutations identified
භRx - penicillamine, Zn, liver transplant
for hepatic failure
Molybdenum Cofactor Deficiency & Sulfite Oxidase Deficiency
භMo complexes to 3 enzymes (sulfite oxidase,
aldehyde oxidase, xanthine dehydrogenase); sx with
cofactor deficiency or isolated sulfite oxidase
deficiency (both AR)
භSx – severe progressive szs, dislocated lenses, MR
භDx – low uric acid (<2; in the combined deficiency),
increased sulfite excretion (sulfocysteine)
භRx – Trials for Rx with cyclic pyranopterin
monophosphate (cPMP) in MoCD Type A
331
Neurodegenerative Disorders of Iron Accumulation
• PKAN pantothenate kinase ass’d neurodegen)
• Dystonia, retinopathy, acanthocytosis
• Depletion of CoA in mitochondria
• PLAN (P’lipase A2 ass’d neurodegen)
• Weakness to spasticity, muscle wasting, optic atrophy
• BPAN
• Neurodegenerative, XL dom with mosaic males, can
resemble Rett or Angelman
• WDR45 ɴ-propellor protein; fairly frequent
APPENDICES
Holocarboxylase Synthase (HCS) and the Biotin Cycle
භHCS catalyzes the covalent addition of biotin to 4
carboxylases
භ3-Methylcrotonyl-CoA carboxylase
භPropionyl-CoA carboxylase
Holocarboxylase
Aposynthase
carboxylases
භAcetyl-CoA carboxylase
භPyruvate carboxylase
B
භBiotinidase catalyzes
recovery of proteinbound biotin
B
Holocarboxylases
Diet
Biotinidase
proteolysis
66
332
Holocarboxylase Synthase (HCS) Deficiency
භSx – PFVLCSz, odor of cat’s urine; skin rash and alopecia in
later onset cases
භDx – metabolic acidosis & ketosis; +anion gap,
NH3, UOA: 3-OH-isovalerate, methylcitrate,3methylcrotonylglycine, propionylglycine, 3-OH-propionate
භGe – AR; functional deficiency of all 4 biotin-dependent
carboxylases
භRx – support during acute crises; 10-50 mg/dy biotin is
virtually curative
Porphyrias
භGroup of enzyme disorders responsible for heme
synthesis from gly + succinyl CoA
භTwo types of major sx – neurovisceral and/or
photosensitivity
භExamples of heterozygous enzyme deficiency
causing disease
භExamples of pharmacogenetic disorders where
avoidance of certain drugs may prevent sx
Porphyria Cutanea Tarda (PCT)
භMost common type
භSx - adult disorder; lesions on sun-exposed skin; liver
disease (cirrhosis 40%); alcohol and estrogen aggravate sx
භDx - urine/stool coproporphyrins; some inherited (AD),
most acquired, defects in UROD (uroporphyrinogen
decarboxylase)
භRx - avoid triggers;
phlebotomy
333
Acute Intermittent Porphyria (AIP)
භIncidence ~1,20,000
භSx - after puberty, abdominal pain and neuropsychiatric;
attacks can be precipitated by certain drugs (dilantin,
sulfa drugs, etc)
භ80-90% of gene carriers asymptomatic
භAD deficiency of PBG deaminase
භDx - urine porphyrins; enzyme assay/molecular
භRx - high CHO diet; IV glucose and hematin for attacks;
avoid offending drugs
Other Liver GSDs
භGSD 0 - Glycogen synthase deficiency
භFasting hypoglycemia in infancy; postprandial
lactate
භFrequent developmental delay
භNormal liver size with reduced hepatic glycogen
භAR inheritance; GYS2 mutations
භGSD IV - Brancher deficiency (Anderson)
භChildhood cirrhosis with hepatosplenomegaly
භAbnormal glycogen - Amylopectinosis
භAR inheritance; GBE mutations
GSD- Type V McArdle’s Disease
භAR; muscle phosphorylase deficiency
භSx - muscle weakness and cramping, usually ass’d with
exercise; sx generally start as adult
භDx - myoglobinuria (50%); lack of rise in lactate on
ischemic exercise test is virtually diagnostic
භRx - avoid precipitating factors; glucose with exercise; B6
334
Other Muscle Glycogen Storage Diseases
භ GSD VII - AR; Phosphofructokinase deficiency
භSimilar to GSD V plus hemolysis
භ GSD X - AR; Phosphoglycerate mutase deficiency
භExercise intolerance, cramps, myoglobinuria
භ GSD XI - AR; Lactate dehydrogenase deficiency
භExercise intolerance, cramps, myoglobinuria
Defects of Purine and Pyrimidine Metabolism
භLesch-Nyhan syndrome (HGPRT def)
භMR, choreiform movements, selfmutilation, gouty arthritis; milder
variants
භXL; Dx - elevated uric acid, enzyme
assay/molecular
භAPRT def – renal sx, stones
භAdenysuccinate lyase def – MR, szs, autistic features
භADA and NP deficiency – AR immuno- deficiency syndromes
භHereditary orotic aciduria - megaloblastic anemia & orotic
acid crystals; +/- MR; rare AR
Disorders of Bilirubin Metabolism
භUnconjugated hyperbilirubinemia
භCrigler-Najar I and II – AR; deficiency of bilirubin-UDPglucuronosyl transferase 1; kernicterus; Type I more
severe and don’t respond to phenobarbitol
භGilbert – mild with no clinical sx; pts with homozygous
promoter mutations
භConjugated hyperbilirubinemia
භDubin-Johnson – AR; defect in liver anion transporter
භRotor syndrome – AR; rare, usually benign; recent
demonstration defect is biallelic mutations in linked
bilirubin transporter genes
335
• Online Metabolic and Molecular Basis of Disease
• Blau et al., Physician’s Guide to Laboratory Diagnosis of
Metabolic Disease
• Nyhan et al., Atlas of Metabolic Disease
• NAMA slide sets from SIMD
• Lee and Scaglia, eds., Inborn Errors of Metabolism (2014)
ACMG Genetics and Genomics Review Course
June
20-23, 2013
336
Neurogenetics
NEUROGENETICS
Bruce R. Korf, MD, PhD, FACMG
Wayne H. and Sara Crews Finley Chair in Medical Genetics
Professor and Chair, Department of Genetics
Director, Heflin Center for Genomic Sciences
University of Alabama at Birmingham
Bruce R. Korf, MD, PhD, FACMG
Department of Genetics
University of Alabama at Birmingham
Kaul 230, 1530 3rd Avenue South
Birmingham, AL 35294-0024
(205) 934-9411 Telephone
(205) 934-9488 Fax
[email protected]
339
340
Neurogenetics
Bruce R. Korf, MD, PhD
Professor and Chair, Department of Genetics
University of Alabama at Birmingham
Disclosure(s)
Relationship
Entity
Grant Recipient
Novartis
Advisory Board
Accolade, Genome Medical
Board of Directors
American College of Medical Genetics and Genomics
Children’s Tumor Foundation
Advisor
Neurofibromatosis Therapeutic Acceleration Project
Founding Member
Envision Genomics
Salary
University of Alabama at Birmingham
Objectives
• Formulate genetic differential diagnosis for
neurological problems
• Recognize indications for genetic testing for
neurological disorders
• Describe natural history and management for selected
neurogenetic disorders
341
Approach to Neurogenetic Disorders
Anatomical localization
Brain
Spinal cord
Peripheral nerve
Muscle
Sensory organs
Temporal course
Congenital or acquired
Static or progressive
Continuous or paroxysmal
Physiology
Developmental mechanisms
Intercellular signaling/signal transduction
Structure/function
Control of cell growth
Metabolic
Phakomatoses
• Neurofibromatosis
• NF1
• NF2
• Schwannomatosis
• Tuberous Sclerosis complex
• von Hippel-Lindau syndrome
+/+
-/-
+/-
Tumor suppressor mechanism
Neurofibromatoses
NF1
Inheritance/
Penetrance
Frequency
Features
Gene/Protein
Function
NF2
Schwannomatosis
AD/Complete
AD/Complete
1:3,000
1:25,000
AD/Incomplete
1:40,000
neurofibromas; café-au-lait macules;
learning disabilities; skeletal dysplasia;
gliomas; malignant peripheral nerve
sheath tumors
vestibular schwannomas; other
schwannomas; meningiomas;
ependymomas; cataracts
Schwannomas; pain
NF1 – chromosome 17
neurofibromin
NF2 - chromosome 22
merlin/schwannomin
INI1/SMARCB1 or LZTR1
GTPase activating protein
cytoskeletal protein
chromatin remodeling
342
NF1
•
•
•
•
•
•
Frequency: 1:3,000
Inheritance: AD, complete penetrance
Gene: NF1
Diagnosis: clinical criteria, genetic testing
Pathophysiology: tumor suppressor, control of RAS signaling
Surveillance: blood pressure, growth, puberty, vision, tumors,
malignant peripheral nerve sheath tumor
• Treatment: surgery, chemotherapy for low grade gliomas,
clinical trials (MEK inhibitor)
• Counseling: AD, new mutation, mosaicism
NF1 Diagnostic Criteria
• At least six café-au-lait macules
• 5 mm pre-puberty
• 15 mm post-puberty
• Skin-fold freckles
• Two or more neurofibromas/one plexiform neurofibroma
• Two or more iris Lisch nodules
• Optic Glioma
• Characteristic Skeletal Dysplasia
• tibia
• orbit
• Affected first-degree relative
Diagnosis requires fulfillment of two criteria
343
Differential Diagnosis of
Multiple Café-au-Lait Spots
Condition
Gene(s)
Comments
NF1
NF1
Six or more
NF2
NF2
Usually fewer than six; bilateral vestibular schwannomas
Legius syndrome
SPRED1
Café-au-lait spots, skin fold freckles; no tumors
Noonan syndrome
Ras pathway genes
Dark café-au-lait spots; some with lentigines
Mismatch Repair Deficiency Syndrome
Mismatch repair genes
High risk of malignant tumors
Fanconi anemia
Various DNA repair genes
Congenital anomalies, bone marrow failure
Ataxia-Telangiectasia
ATM
Ataxia, telangiectasia
McCune-Albright
GNAS1 mosaicism
Large, irregularly shaped; precocious puberty; fibrous dysplasia
Chromosomal mosaicism
Various aneuploidies
Irregularly shaped along lines of Blashko
Bloom syndrome
BLM
Short stature, risk of cancer
Tuberous sclerosis complex
TSC1/TSC2
More often hypopigmented macules
NF1 Pathogenesis
23a
9br
1 2 3 4a 4b 5 6
7
8
10a 10b 10c 12a
13
16
17 1819a
20
2
1
22 23-2
48a
24 25 26 27a27b
28
OMGP EVI2B
29
30 31
32
33
34
48
49
EVI2A
EGF
EGF
Grb
36 37 38 39 40 41 42 4344 45 46
Sos
NF1 +/-
Ras
neurofibromin
NF1-/-
Schwann cell
Grb
Sos
Ras
NF1-/Other changes
MPNST
Neurofibroma
NF2
• Frequency: 1:25,000
• Inheritance: AD, complete penetrance
• Gene: NF2
• Diagnosis: clinical criteria, genetic testing
• Pathophysiology: tumor suppressor, cytoskeletal
protein
• Surveillance: hearing, brain and spinal imaging,
eye exam
• Treatment: surgery, clinical trials (bevacizumab)
• Counseling: AD, new mutation, mosaicism
344
NF2 Diagnostic Criteria
Bilateral vestibular schwannomas, or
First degree relative with NF2 and
any two of:
meningioma
ependymoma
schwannoma
juvenile posterior subcapsular cataract/cortical wedge opacity
Schwannomatosis
• Frequency: ~1:40,000
• Inheritance: AD, incomplete penetrance
• Gene: INI1/SMARCB1 or LZTR1
• Diagnosis: clinical criteria, genetic testing, mosaic staining INI1 in
schwannoma (for SMARCB1-related schwannomatosis)
• Pathophysiology: tumor suppressor, complex findings in tumors;
chromatin remodeling protein(s)
• Surveillance: tumor growth, pain
• Treatment: surgery, pain management
• Counseling: AD, incomplete penetrance, new mutation, mosaicism
Schwannomatosis
Germline
SMARCB1
SMARCB1
NF2
NF2
Tumor
SMARCB1
Deletion
NF2
Similar mechanism with LZTR1
345
Tuberous Sclerosis Complex
• Frequency: 1:6,000
• Inheritance: AD, complete penetrance
• Gene: TSC1 (9q34 hamartin); TSC2 (16p13 tuberin)
• Diagnosis: clinical criteria, genetic testing
• Pathophysiology: tumor suppressor – mTOR inhibition
• Surveillance: development, seizures, SEGA, renal,
pulmonary, eye
• Treatment: everolimus for progressive SEGA or AML
• Counseling: AD, new mutation, mosaicism
TSC Diagnostic Criteria
Major
•
•
•
•
•
•
•
•
•
•
•
Angiofibromas (ш3) or fibrous
cephalic plaque
Ungual fibromas (ш2)
Hypomelanotic macules (ш3,
ш5mm)
Shagreen patch
Multiple retinal hamartomas
Subependymal giant cell
astrocytoma
Subependymal nodules (ш2)
Cortical dysplasias (ш3)
Cardiac rhabdomyoma
Renal angiomyolipoma
Lymphangioleiomyomatosis
Minor
Dental enamel pits (>3)
Intraoral fibromas (ш2)
Non-renal hamartomas
Retinal achromic patch
Confetti skin lesions
Multiple renal cysts
Definite TSC
Two major
One major, two or more minor
Possible TSC
one major, or
two or more minor
TSC Skin Lesions
hypomelanotic
macule
Shagreen
patch
facial
angiofibroma
periungual
fibroma
346
TSC CNS Lesions
TSC
Normal
TSC Other Lesions
Cardiac Rhabdomyoma
Levine, D., Barnes, P., Korf,
B., Edelman, R. Am. J.
Roentgenol. 2000;175:10671069
Angiomyolipoma
Renal cysts
Lymphangioleiomyomatosis
TSC Surveillance
• Brain
• MRI (repeat every 1-3 years) • Eye
• EEG
• Ophthalmology exam annually
• Developmental screening
• Kidney
• Heart
• Echo on children
• ECG all ages (every 3-5 years)
• MRI abdomen (repeat every
1-3 years)
• Skin – exam annually
• Blood pressure annually
• Teeth – exam every 6
• GFR annually
• Lung (female >18)
months
• PFT, 6 minute walk test
• High resolution chest CT (repeat
every 5-10 years)
347
TSC Treatment
hamartin
tuberin
Rheb GDP
mTOR
SEGA
Everolimus
Renal AML
growth & proliferation
Von Hippel-Lindau Syndrome
• Frequency: 1:36,000
• Inheritance: AD
• Gene: VHL
• Diagnosis: clinical criteria, genetic testing (genotype-phenotype
correlation)
• Pathophysiology: tumor suppressor – vascular response to hypoxia
• Surveillance: eye, hearing, brain, kidney, pheochromocytoma
• Treatment: some clinical trials
• Counseling: AD, 20% new mutation, potential risk for those with
sporadic hemangioblastoma
Von Hippel-Lindau Syndrome
•
Hemangioblastoma
• cerebellum
• retina
• spinal cord
•
•
•
Pheochromocytoma
Renal cell carcinoma
Endolymphatic sac tumor
HIF
VHL
O2
angiogenesis
348
VHL Surveillance
• Birth
• Physical exam/hearing screen
• Ages 1-4
• Annual ophthalmological exam
• Physical exam
• Ages 5-15
•
•
•
•
Annual PE and eye exam
Annual fractionated metanephrines
Every 2-3 year audiology
MRI if repeated ear infections
• Age 16 and beyond
• Annual eye, PE, fractionated metanephrines
• Annual abdominal u/s with MRI every other year
• Every 2 years brain MRI, audiology
• During Pregnancy
• Test for pheochromocytoma early, mid, and late pregnancy
• MRI without contrast at 4 months brain and spine
www.vhl.org
Screening for Pheochromocytoma
• Screening for PPGLs should always include measurements of
plasma-free metanephrines (obtained from a blood sample) or
urinary fractionated metanephrines (obtained from a urine
sample).
• For a blood sample, patients are now required to be supine (lying
down) for a minimum of 20 (ideally 30) minutes between the
time the needle is inserted and the time the blood is drawn.
• Blood sample analysis should be done using norms (a reference
standard) from supine tests, not seated tests, to minimize the
chance of a false negative result (missing a PPGL that is present)
www.vhl.org
Epilepsy
• Affects 1% population
• At least 2 unprovoked seizures
• Classification
• Focal Onset
• Aware/Impaired Awareness
• Motor/Non-motor onet
• Focal/Bilateral Tonic-Clonic
• Generalized Onset
• Motor (tonic-clonic, other)
• Non-Motor (Absence)
• Unknown Onset
• Motor
• Non-Motor
• Monogenic mostly ion channelopathies
• Multifactorial: epilepsy with complex genetics
• CNVs: 15q11.2, 15q13.3, 16p13
• Epilepsy syndromes
• Part of some genetic syndromes
349
Channelopathies
Na Channel
Na+
Generalized epilepsy
with febrile seizures
Severe myoclonic
epilepsy of infancy
Idiopathic childhood
epilepsy with
Generalized t/c seizures
Primary erythermalgia
Paroxysmal extreme pain disorder
Insensitivity to pain
Myotonia
Periodic paralysis
Empirical Recurrence Risk Counseling for Epilepsy
Prasad AN, Prasad C. Genetic Aspects of Human Epilepsy.
Emery & Rimoin, Principles and Practice of Medical Genetics, 6th Ed.
Movement Disorders
• Bradykinesia – slow movements
• Dystonia – sustained muscle
contraction resulting in abnormal
posture
• Chorea – sudden involuntary
movement
• Myoclonus – muscle jerking
• Tremor – rhythmic oscillatory
movement
• Tic – sudden stereotyped motor
movement or vocalization
By Mikael Häggström, based on images by Andrew Gillies/User:Anaru and Patrick J. Lynch
[CC-BY-SA-3.0 (www.creativecommons.org/licenses/by-sa/3.0)], via Wikimedia Commons
350
Parkinson Disease
Tremor, rigidity, bradykinesia
1% >60 yo; 4% >80 yo
Most cases sporadic (multifactorial)
Glucocerebrosidase heterozygosity
Multiple risk alleles by GWAS
5-10% monogenic
autosomal dominant
SCNA, LRRK2, VPS35
autosomal recessive
PINK1, DJ-1, Parkin, DNAJC6
Atypical PD
ATP13A2, FBX07, PLA2G6, SYNJ1
By Sir_William_Richard_Gowers_Parkinson_Disease_sketch_1886.jpg:
derivative work: Malyszkz [Public domain], via Wikimedia Commons
Fragile X Tremor Ataxia Syndrome
• Ataxia
• Intention tremor
• Short term memory loss
• Dementia
• Parkinson symptoms
• Premutation
• >50 yrs
• Penetrance males older than 50 >33%; females 5-10%
• Toxic gain of function of FMR1 mRNA
Dystonia
• DYT1
• Torsin A(9q34)
• AD
• GAG deletion most common mutation
• Focal to generalized dystonia
• DYT5
• AD
• GTP cyclohyrolase I/tyrosine
hydroxylase
• DOPA-responsive
By James Heilman, MD (Own work) [CC-BY-SA-3.0
(www.creativecommons.org/licenses/by-sa/3.0) or
GFDL (www.gnu.org/copyleft/fdl.html)], via Wikimedia
Commons
351
Dystonia
Condition
Genetics
Isolated Dystonias
Childhood/Adolescent-onset
AD: DYT1 (TOR1A), DYT6 (THAP1), DYT13; AR: DYT2,17
Adult-onset
DYT7, DYT21, DYT23 (ClZ1), DYT24 (ANO3), DYT25 (GNAL) (all AD)
Combined Dystonias
Persistent
Dystonia with Parkinsonism
Dystonia with Myoclonus
Dystonia with Chorea
Paroxysmal
Nonkinesigenic
Kinesigenic
Exercise-induced
AD: DYT5(GCH1, TH, SPR), DYT12 (ATP1A3); AR: DYT5, 16; XR: DYT3 (TAF1)
DYT11, DYT15 (SGCE) (AD)
DYT4 (TUBB4) (AD)
All AD
DYT8 (MR-1), DYT20
DYT10 (PRRT2), DYT19
DYT18 (SLC2A, GLUT1)
Neurodegeneration with Brain Iron Accumulation
• Clinical features
• Movement disorder
• Dystonia, choreoathetosis, rigidity
• Ophthalmologic signs
• Retinitis pigmentosum, optic atrophy
• Iron accumulation in basal ganglia
• Genetics
• PKAN – pantothenate kinase deficiency
• PLAN – infantile neuraxonal dystrophy or later onset (PLA2G6)
• MPAN – mitochondrial membrane protein (C19ORF12)
• BPAN – beta-propeller (WDR45) – X-linked
• Aceruloplasminemia (CP)
• FAHN – fatty acid hydroxylase (FA2H)
• Kufor-Rakeb (ATP13A2)
• Neuroferritinopathy (AD)(FTL)
• Woodhouse-Sakati (DCAF17)
• Idiopathic
Curr Opin Neurol. 2016 Aug; 29(4): 486–495.
doi: 10.1097/WCO.0000000000000352
Monogenic Choreas
Disorder
Huntington chorea
Gene
HTT (CAG expansion)
Inheritance
AD
Notes
Dementia, psychiatric disturbance
Prion
PRNP (octapeptide expansion)
AD
Dementia, psychiatric disturbance
HDL2
JPH3 (CAG/CTG expansion)
AD
Parkinsonism
AD
Ataxia, dementia, Parkinsonism
SCA17
TBP (CAG expansion)
Dentatorubral-pallioluysian atrophy
ATN1 (CAG expansion)
AD
FTD/MND
C9orf72 (GGGGCC expansion)
AD
Dementia, psychiatric disturbance
Neuroferritinopathy
FTL
AD
Facial dystonia
Seizures, myoclonus, dementia
Idiopathic basal ganglia calcification
SCLC20A2, PDGFB, PDGFRB, XPR1
AD
Basal ganglia calcification
Choreoacanthocytosis
VPS13A
AR
MacLeod syndrome
XK
XLR
Neuropathy, cardiomyopathy, inc. CK
Ataxia-telangiectasia
ATM
AR
Ataxia, telangiectasia, immune deficiency
Ataxia with oculomotor apraxia
APTX, SETX, PNKP
AR
Gordon-Holmes syndrome
RNF216
AR
Hypogonadism, cerebellar atrophy
NKX2-1 related chorea
NKX2-1
AD/de novo
Hypotonia, learning disabiliites, pulmonary and thyroid dysfunction
ADCY5-related chorea
ADCY\5
AD/de novo
PDE10-A-related chorea
PDE10-A
AR/de novo
Delayed milestones and language
GPR88-related chorea
GPR88
AR
Delayed language/learning disabilities
Early infantile epileptic encephalopathy 17
GNAO1
de novo
Congenital Rett disease
FOXG1
de novo
Intellectual disability, microcephaly, abnormal MRI
Severe motor delay and intellectual disability
SYT1
de novo
Intellectual disability, motor delay, no seizures
Early infantile epileptic encephalopathy type 13
SCN8A
AD/de novo
Paroxysmal dystonia/chorea
Oromandibular dystonia, neuropathy, inc. CK
Neuropathy, hypoalbuminemia, hypercholesterolemia, cerebellar atrophy
Diurnal variation, dystonia, myoclonus, axial hypotonia
Developmental delay, some with seizures
352
Huntington Disease
• Clinical features
• psychiatric: depression, mood swings
• cognitive: dementia
• motor: chorea, bradykinesia
• Early onset with rigidity inherited from father
• Pathology: caudate atrophy
• Genetics
• 4p16.3; huntingtin
• CAG triplet repeat (polyglutamine)
• Age-dependent penetrance
• Peak age of onset 3rd to 4th decades
• repeat expansion
By Frank Gaillard (Own workvia Wikimedia
• general population: <26 repeats
Commons
• intermediate alleles: 27-35
• HD: >36 (arise from intermediate, more often in sperm)
• Reduced penetrance 36-39 repeats
Triplet Repeat Disorders
AUG
CGG
TAA
GAA
CAG
CTG
Friedreich ataxia
SCA
Fragile X syndrome
Huntington disease
DRP atrophy
Spinal & bulbar atrophy
Machado-Joseph disease
myotonic dystrophy
increasing severity from one
generation to next
Severity and onset correlate
with repeat size
Larger size = greater instability
Anticipation
Paroxysmal Dyskinesias
• Episodic abnormal movements
• Paroxysmal kinesigenic dyskinesia – PRRT2
• Triggered by voluntary movements
• Infantile convulsions, choreoathetosis
• Treated with carbamazepine
• Paroxysmal exercise-induced dyskinesia – SLC2A1
• Exercise-induced dystonia
• Migraine, hemiplegia, ataxia, epilepsy
• Paroxysmal non-kinesigenic dyskinesia – PNKD
• Triggered by alcohol, stress, caffeine
353
Hereditary Ataxias
• Spinocerebellar ataxias
•
•
Multiple SCAs – most AD
Some triplet repeat disorders
• Friedreich ataxia
•
Clinical
•
•
•
•
Genetics
•
•
•
•
•
•
autosomal recessive
GAA repeat normal 7-34 times
Premutation 34-65 repeats (borderline clinical 44-66)
Pathological 66-1,700
Other point mutations also may occur
Pathogenesis
•
•
•
ataxia, impaired position and vibration sensation
loss of DTR’s; pes cavus; extensor plantars
hypertrophic cardiomyopathy; diabetes mellitus
Conjunctival telangiectasia
Repeat interferes with transcription
Impaired mitochondrial Fe metabolism
Ataxia telangiectasia
•
•
•
•
•
ataxia
telangiectasia
immune deficiency
abnormalities chromosomes 7 and 14 (Ig chains and T cell receptor)
ATM mutations
Metabolic Ataxias
Disorder
Metabolic Abnormality
Clinical Features
Treatment
Bassen–Kornzweig syndrome
Abetalipoproteinemia
Acanthocytosis, retinitis
pigmentosa, fat malabsorption
Vitamin E
Ataxia with isolated vitamin E
Deficiency of alpha-tocopherol
transfer protein
Progressive ataxic syndrome
Vitamin E deficiency (AVED)
Hartnup disease
Tryptophan malabsorption
Pellagra rash, intermittent ataxia
Niacin
Riboflavin, CoQ10, dichloroacetate
Mitochondrial complex defects
Complexes I, III, IV
Encephalomyelopathy
Multiple carboxylase deficiency
Biotinidase deficiency
Alopecia, recurrent infections,
variable organic aciduria
Biotin
Pyruvate dehydrogenase
deficiency
Block in energy metabolism
Lactic acidosis, ataxia
Ketogenic diet, chloroacetate
Refsum disease
Phytanic acid, alpha hydroxylase
Retinitis pigmentosa,
cardiomyopathy, hypertrophic
neuropathy, ichthyosis
Dietary restriction of phytanic acid
Urea cycle defects
Urea cycle enzymes
Hyperammonemia
Protein restriction, arginine benzoate,
alpha ketoacids
Opal, P, Zoghbi, H. The Hereditary Ataxias.
Emery & Rimoin, Principles and Practice of Medical Genetics, 6th Ed.
Alzheimer Disease
•
•
Progressive dementia
Autosomal Dominant Genes
• (ख़2% cases)
• Early onset AD
• PSEN1 (presenilin-1)
• PSEN2 (presenilin-2)
• Amyloid precursor gene (APP)
• Late onset AD
• APOE ε4 allele
• 15x risk homozygote; 3x risk
heterozygote
• Effect varies with sex and ethnicity
354
Prion-Associated Dementia
• PRNP mutations
• May be familial (AD) or infectious
• Disease associated with
misfolded proteins
• Spongiform encephalopathies
• Creuzfeld-Jacob disease
• Gerstmann-Straussler-Scheinker
disease
• Fatal familial insomnia
By Lopez-Garcia, F., Zahn, R., Riek, R., Wuthrich, K. &
RCSB [Public domain], via Wikimedia Commons
Cerebrovascular Disorders
•
Small vessel disease
•
•
•
Large vessel disease
•
•
•
Marfan syndrome
Arrhythmias
Ischemic stroke
•
•
•
•
Fabry disease
Homocystinuria
Cardioembolic stroke
•
•
•
EDS vascular type
Pseudoxanthoma elasticum
Mixed small and large vessel disease
•
•
•
Cerebral autosomal dominant arteriopathy with subcortical infarcts and
leukoencephalopathy (CADASIL): NOTCH3
CARASIL: HTRA1
Sickle cell disease
Moyamoya (NF1)
Collagen IV mutations
Hemorrhagic stroke
•
•
•
•
Cerebral cavernous malformations
Hereditary hemorrhagic telangiectasia
Cerebral Amyloid antipathy
Cerebral venous thrombosis (prothrombin)
Hereditary Spastic Paraplegia
• Clinical
• Progressive weakness and spasticity in lower extremities
• Includes bladder disturbance, LE sensory changes
• May be “complicated” by other features, including
seizures, dementia, movement disorder
• Genetics
• Multiple subclasses, dominant, recessive, and X-linked
• AD: most common are SPAST, ATLI, KIF5A, REEP1
• AR: SPG11 (intellectual disability, thin corpus callosum,
axonal neuropathy), many others
• XLR: L1CAM, PLP1
355
Anterior Horn Cell
Type
• Weakness, absent
reflexes, fasciculation
• neuropathic EMG and
muscle biopsy
• normal level of
alertness
Locus
Siddique, T, et al. Motor Neuron Disease,
Emery and Rimoin Principles and Practice
of Medical Genetics, 6th edition.
Phenotype
OMIM
21q22
SOD1
AD-ALS
105400
2q33
ALSIN
Juvenile AR-ALS
Juvenile PLS
Infantile onset spastic paraplegia
205100
606353
607225
Incorrectly assigned to 18q21, a mutation in
FUS (ALS6) has been identified in the family
that was used for initial mapping
606640
ALS3
18q21
–
ALS4
9q34
SETX
AD- Juvenile ALS
ALS5
15q21
SPATACSIN
AR- Juvenile ALS
SPG11
602099
ALS6
16p11
FUS
AD- ALS
AD-ALS-FTD
608030
ALS7
• Distal spinal muscular
atrophies chaparonopathies
Gene
ALS1
ALS2
602433
20p13
–
AD-ALS
608031
ALS8
20q13
VAPB
AD distal SMA
AD typical and atypical ALS
608627
ALS9
14q11
ANG
AD- ALS
611895
ALS10
1p36
TDP43
AD-ALS
AD ALS-FTD
612069
ALS11
6q21
FIG4
AD-ALS
CMT4J
612577
611228
ALS12
10p15
OPTN
ALS linked to chromosome 9
9p21
C9ORF72-
AD ALS
AD ALS-FTD
105550
ALS linked to chromosome 12
12q24
DAO
AD-ALS
AD-ALS
–
613435
ALS-VCP
9p13
VCP
AD-ALS
IBMPFTD
–
167320
ALSX
Xp11.23
UBQLN2
X-juvenile ALS
X-Adult ALS
X-ALS/Dementia
Spinal Muscular Atrophy
cen
tel
NAIPD
P44C
SMN2
SMN1
NAIP5
P44T
ƒ 5q11.2-q13.3
ƒ inverted, duplicated segment
ƒ SMN: survival motor neuron
ƒ base differences in exons 7 & 8
ƒ interacts with RNA-binding protein
ƒ increased dosage of SMN2 correlated with milder phenotype
ƒ 95-98% homozygous for SMN1 deletion or truncation
ƒ 2-5% compound heterozygotes for deletion or truncation and SMN1 intragenic
mutation
ƒ Nusinersin – FDA approved treatment- antisense oligonucleotide that includes
exon 7 in SMN2
SMN Therapy
SMN1
SMN1
SMN2
6
7
8
6
C
7
U
8
6
7
8
7
8
Antisense
oligonucletide
Splicing
6
8
C
Functional
protein
6
U
Splicing
mRNA
SMN2
Transcription
Transcription
6
7
8
U
Non-functional
protein
Functional
protein
356
Peripheral Neuropathy
• Absent deep tendon reflexes, weakness, muscle
atrophy
• Inflammatory demyelinating
• Hereditary
• motor and sensory (Charcot-Marie-Tooth)
• sensory (including familial dysautonomia)
• Friedreich ataxia
Familial Dysautonomia
• Clinical Features
•
•
•
•
•
•
•
Feeding difficulty
Episodic vomiting
Autonomic neuropathy
Insensitivity to pain
Absent tearing
Absent fungiform papillae
Increased sweating
• Genetics
• IKBKAP gene
• Splicing mutation in Ashkenazi Jewish form
Metabolic Neuropathy
• Diabetes
•
•
•
•
•
•
Uremia
Porphyria
Pernicious anemia
Abetalipoproteinemia
Refsum disease
Tangier disease (alpha lipoprotein)
357
Hereditary Sensory & Motor Neuropathy
Normal
Duplication - CMT
Deletion -HNPP
+
ƒ Clinical
ƒ distal weakness, pes cavus, absent DTR’s
ƒ Genetics
ƒ AD, XLR, AR
ƒ CMT1: PMP22 gene duplication, point mutations
ƒ flanked by 24 kb repeat, with unequal crossing over
ƒ HNPP: PMP22 gene deletion, truncating mutations
ƒ other genes: P0, EGR2, connexin 32 (XL)
Neuromuscular Junction: Myasthenia Gravis
• Transient neonatal
(mother with MG)
• Immunological
• Rare genetic forms
By Posey & SpillerCumulus at nl.wikipedia [Public domain], from
Wikimedia Commons
Muscle Biopsy
dystrophic
metabolic
neurogenic
mitochondrial
myopathic
Pictures from Dubowitz, V. Color Atlas of Muscle Disorders in Childhood, Chicago: Yearbook Medical Publishers, 1989
358
Myotonic Dystrophy
CTG repeat
5-35 copies
DMPK gene
3’ untranslated region
>50 copies
DMPK gene
• Clinical
• Myotonia, weakness
• Hair loss, diabetes mellitus, cataract, ECG changes
• Genetics
• CTG repeat expansion
• Maternal transmission of severe neonatal syndrome (>2,000 repeats)
• DM2: CCTG expansion in zinc finger protein ZNF9
Muscular Dystrophy
progressive; high muscle enzymes; loss of
muscle cells by biopsy
Duchenne/Becker dystrophy
Facio-scapulo-humeral dystrophy
Congenital muscular dystrophy
Duchenne/Becker Dystrophy
• Clinical
• High CPK
• Prominent calves
• Steroids
• Genomic Treatments (exon skipping,
nonsense suppression)
• Genetics
• XLR
• Dystrophin mutations (2/3 deletions)
• Duchenne – loss of dystrophin
• Becker – abnormal dystrophin
359
Facioscapulohumeral Dystrophy
Clinical
Weakness of facial and upper shoulder
girdle muscles
Other features
Retinal vasculopathy 40-60%
Sensorineural hearing loss 60%
Genetics
Abnormal DUX4 expression
Deletions in D4Z4 3.3 kb repeat
11-100 repeats normal
1-10 repeats abnormal
Laminopathies
• Lamin A/C – alternative splicing of LMNA gene
• Emery-Dreifuss Muscular Dystrophy
• scapula-humero-peroneal
• Elbow contractures
• Cardiac – arrhythmias, cardiomyopathy
• Missense mutations, usually AD, but may be AR
• Limb-Girdle Muscular Dystrophy 1B
• Pelvic-scapular
• Contractures
• Cardiac – arrhythmias, cardiomyopathy
• Frameshift mutations
• LMNA-congenital muscular dystrophy
• Diffuse or dropped head syndrome
• Contractures
• Cardiac – arrhythmias, cardiomyopathy
• Respiratory failure
• Missense mutations
Congenital Myopathy
Variable progression;
enzymes may be normal
Characteristic
pathology
central core
nemaline
centronuclear
congenital fiber type
disproportion
360
Metabolic Myopathy
• Periodic paralysis (SCN4A)
• hypokalemic
• hyperkalemic
• Thyroid disorders
• Steroid
• Glycogen storage
• Mitochondrial
361
362
Reproductive Genetics I
REPRODUCTIVE GENETICS I
Louise E. Wilkins-Haug, MD, PhD, FACMG
Division Director, Maternal Fetal Medicine and
Reproductive Genetics
Brigham & Women’s Hospital
Professor of Obstetrics, Gynecology and
Reproductive Biology
Harvard Medical School
Louise E. Wilkins-Haug, MD, PhD, FACMG
Division of Maternal Fetal Medicine
Brigham & Women’s Hospital
75 Francis Street
Boston, MA 02115
(617) 732-4208 Telephone
(617) 264-6310 Fax
[email protected]
365
366
Reproductive Genetics 1
Louise Wilkins-Haug
MFM Division Director
Brigham and Women’s Hospital
Disclosure(s) - None
Objectives
1) Examine current guidelines for aneuploidy screening
මCompare serum screening to NonInvasive
Prenatal Testing (NIPT, cfDNA, ffDNA)
මUnderstand “false positive” and “false negative”
2) Assess the role of fetal ultrasound
මUse of “soft markers”
මAnalysis for structural anomalies
367
Case
Mr and Mrs Smith present to the Obstetrician at 8
weeks. This is their first pregnancy, achieved by in
vitro fertilization due to “multifactor etiology” infertility
ම She is 37 yo, healthy, no surgeries or contributing family
history, Northern European ancestry
ම He is 38 yo, healthy, no surgeries or contributing family
history, Northern European ancestry
What is our risk of Down syndrome?
Screening for Down Syndrome Maternal Age and A Priori Risk
American College of OBGYN Guidance
All women - offer screening and diagnostic testing for aneuploidy ideally
in the first trimester (2007)
Options – serum screen vs cfDNA
ම 2012 ACOG/SMFM
ම cfDNA - screening for high risk women, singletons for common
aneuploidies
ම 2015 SMFM update – women’s autonomy respected, if cfDNA
requested by low risk women, pretesting counseling needed, routine
screening remains the preferred option
ම 2016 ACOG/SMFM – cfDNA an aneuploidy screening method
without delineation to maternal age
(ACOG and SMFM update, PB 163, May 2016)
368
Down Syndrome Screening in the Second or
First Trimester
Screen
positive
Sensitivity
PPV
5.0 %
81.0 %
1 / 40
(2.5%)
Nuchal lucency
4.2%
76.8%
1 / 50
(2.0%)
NL + PAPP- A,
hCG
5.0%
87.0%
1 / 25
(4.0%)
Quad – MSAFP, estriol,
hCG + inhibin
(Nicolaides, 2004)
Down Syndrome Screening – Combined
1st and 2nd Trimester
Sequential Screening – 3 types
ම “Integrated” - Nondisclosure
ම PaPP-A and NL 1st trimester + Quad second trimester
ම Results released 2nd trimester
ම Stepwise – disclosure
ම NL + PaPP-A + hCG
ම Disclosure of high risk results 1st trimester
ම All remaining return for quad
ම Contingent – disclosure
ම NL + PaPP-A + hCG
ම Disclosure of high risk results 1st trimester
ම Only intermediate risk return in second trimester
(Cuckle, 2008)
Serum Screening for Down Syndrome
(5 % screen positive rate) – 1990-2000
100
% trisomy 21
90
NT+serum
80
Quad
NT
70
60
50
Triple
40
4 % PPV
30
20
10
2% PPV
Maternal
Age
2% PPV
0
First trimester
Second trimester
(Malone, 2005; Cuckle, 2008)
369
Serum Screening for Down Syndrome
(5 % screen positive rate) – 2000-2010
120
% Trisomy 21
100
NT+serum
CS
CS
SS
IS
SS
Triple
60
40
20
Quad
NT
80
2 % PPV
4 % PPVV
2 % PPV
2 % PPV
Maternal
Age
0
First trimester
Second trimester
(Malone, 2005; Cuckle, 2008)
Noninvasive Prenatal Genetic Testing
2010 -2017
Cells pass between mother and fetus
ම Extracted, quantified and studied
Cell free nucleic acids in adult serum since 1947
ම Fragments of DNA / RNA without cell membranes
Increased with cell turnover
ම Inflammatory diseases (Lupus, glomerular nephritis, pancreatitis)
ම Cancer
ම Tissue injury (trauma, stroke, myocardial infarct)
(Desai and Cregel, 1963)
How does this apply to
pregnancy?
Presence of fetal DNA in maternal
plasma and serum
• Lo, Y M, Corbetta, N, Chamberlain, P F, Rai, V,
Sargent, IL, Redman, C W, Wainscoat, J S1997
Lancet, 350:845-7
Fetus-derived Y sequences:
ම80% (24/30) maternal plasma
ම70% (21/30) maternal serum
ම17% (5/30) fetal cells
370
Characteristics of cffDNA
ƒ Comes from the
placenta
ƒ cffDNA is 5-10% total
cell free DNA in
maternal circulation
ƒ Present at 5-7
weeks, cleared within
hours
Levels not altered
Levels altered
Maternal age
Gestational age
Race
BMI
Parity
Aneuploidy
Mode of conception
Smoking
Placental volume
(Bianchi, 2006, Pergament, 2014)
Distinguishing Fetal from Maternal
Free Nucleic Acids
RhRhRh-
Rh+
Rh+
Rh+
21
21
21
21
Rh+
Fetal gene is different from
mother’s gene
SRY – fetal sex
Father’s genes different from
mother’s
21
Maternal free nucleic acids
21
21
Fetal free nucleic
acids
Fetal gene is the same
as mother’s gene
Aneuploidy
Detection of Aneuploidy Next Generation Sequencing (Massively parallel genomic
sequencing)
o 10s of millions
DNA fragments
sequenced at
same time
o First 36 bases
are sequenced
o “Binned” by
chromosome
(Chiu , 2008; Palomaki, 2011,)
371
Screening for Down Syndrome in Women > 35
2nd trim
quad
1st trim
combined
Integrated
DNA testing
DR
80%
90%
95%
99%
Screen
Positive Rate
5%
15%
2%
0.2%
Chance of true
positive
2%
2-3%
4%
80-99 %
Complexity
1 Blood
draw
US and 1
blood draw
US and 2
blood draws
1 blood
draw
Failure Rate
<<1%
<1%
<1%
0.3 – 3%
Unanswered Questions with NIPT
#1 Should it be used in low risk women?
o Advanced screen?
o Primary screen?
#2 What do discordant results mean ?
o“False positives”
o“False negatives
#3 What should be done with a “no call” result?
#4 Should it expand beyond the common aneuploidies?
As an Advanced Screen In Women < 35
Years Old ?
2,800,000 women
(5,000 trisomy 21 fetuses)
Combined 1st trimester
screening
(75% detection)
1250 trisomy 21
(25% missed)
140,000 positive
(3,750 trisomy 21)
140,000 CVS /Amniocentesis
Loss of 1,050 normal fetuses
3,750 trisomy 21
(75% detection)
cfDNA
(3675 tri 21, 740 nl
1374 failed)
5,789 Amniocentesis
Loss of 10 normal fetuses
3675 trisomy 21
(73.5% detection)
372
As A Primary Screen in All Women?
Fetal fractions, sensitivities and specificity are
independent of maternal age
Positive predictive value dependent on a priori risk
මBallpark estimates
Indication
Positive predictive value
> 35 years old
< 35 years old
US + serum screening
80%
50%
2-4%
Positive Predictive Value for Aneuploidy
120%
100%
80%
60%
PPV
40%
20%
0%
45 yo
35 yo
20 yo
12 weeks, trisomy 21
45 yo
35 yo
20 yo
12 weeks, trisomy 18
Serum Screen – “Hidden” Value vs Risk
Value – abnormal karyotype not detected by cfDNA
but found by 1st US / serum screen – 2%
o Reflects abnormal placentation, increased nuchal lucency
o 1.0% if exclude US anomalies, karyotype changes without
abnormal phenotype
Risk- invasive studies for all positive serum screen
(5-7% of screened population)
o Predominantly pregnancies with normal chromosomes
(Norton, 2014; Peterson, 2014)
373
# 2 What About the Discordant Results?
“False Positives” (0.1%)
“cfDNA” positive
මIncreased chromosome specific DNA segments but
not its origin
Possible origins
මPlacenta
ම“Vanishing “ twin
මMaternal
Confined Placental Mosaicism
1-2% of all pregnancies at 10–12 wks
Normal fetal karyotype with normal
and aneuploid cell lines in the placenta
Adverse pregnancy outcomes
ම IUGR
ම Uniparental disomy
Multiple case reports, overall
prevalence?
(Futch, 2012, Hall, 2013, Pan 2013)
“Vanishing Twins”
1-2% of singletons originate as twins
Discordancy with vanishing twins
ම 2 of 3 of discordant trisomy 13 cases
occurred in setting of an early twin demise
Impact on NIPT results
ම 0.43% of cases with two paternal
haplotypes
ම Second twin cfDNA detected >8 weeks
after demise
(Futch, 2013; McAdoo, 2014)
374
Maternal Cancer - “Widespread genomic
imbalance” NIPT Results
37yo, NIPT + 13 and -18; amniocentesis, neonate, placental
biopsies – all normal
ම Postpartum pelvic pain - small cell carcinoma of the vagina
ම Majority of cancer cells with trisomy 13
Recent report of 12 cases with various malignancies
One study – 0.03% of tests > 2 aneuploidies
ම affected fetuses - 4, normal fetuses – 14
ම 5/14 (36%) of women with a cancer diagnosis
(Osborne, 2013; Bianchi, 2015, Snyder, 2016)
Maternal Aneuploidies
Sex chromosome aneuploidies :
ම44 yo with an IVF conception
ම NIPT - abnormal X chromosome ratios, newborn karyotype normal
ම Maternal karyotype 45, X[3]/46, XX[27]
ම25 yo, normal height, intellect and fertility
ම NIPT positive for triple X, amniocentesis normal
ම Maternal karyotype 47, XXX
8.6% of NIPT positive for sex aneuploidy have
maternal sex chromosome mosaicism
(Nicolaides, 2013; Lau, 2013; Wang, 20
Discordant Results - False Negatives
o Liveborn trisomy 13 and 18
fetuses have mosaicism
(euploid and aneuploidy) in
their placentas
o Case reports as source of
a false negative for trisomy
18
(Kalousek, 1989; Pergament, 2014)
375
#3 What Should be Done with “no call”
Results? (2-8% of reports)
Low fetal fraction = higher false negatives
1) Early gestational age
ම 60% get a result on redraw
2) Higher BMI and lower fetal fraction
ම 20% in women > 250 lbs
ම 50% in women > 350 lbs
3) Aneuploidy (13,18,21, triploidy)
ම Increased aneuploidy rate with low fetal fraction / no results
ම As high as 20% ( 1 in 5 are chromosomally abnormal)
(Ashoor, 2013; Pergament, 2014, Williams, 20
#4 Expanded Beyond the Common
Aneuploidies?
Micro deletions and duplications
ම 1/100 neonates but widespread across genome
ම 5 most common are 1/1000, most are 22q deletion
ම Emerging technology, not currently supported by professional
societies
Whole fetal exome
ම Technically possible at 7 Mb level
ම Extends analyses to other aneuploidies
ම Emerging technology
(ACOG and SMFM statement, May 2016)
35 yo Positive NIPT for 21
Prenatal care 1st trimester
මChoose NIPT – positive for 21
මPPV – 83%
Continuation rate influenced by path
to result
ම NIPT
ම Amniocentesis
ම CVS
42%
33%
8%
376
Antepartum Management Down
Syndrome
Antepartum risks
ම IUFD rate (4/70)
ම Average gestational age = 37.0 wks
ම Growth restriction in 20.0%
ම Not associated with anomalies, or maternal
age
Surveillance
ම 35.0% delivered for new onset nonreassuring
fetal testing
ම Not associated with anomalies, growth
restriction or maternal age
Newborn Genetic Follow-up
ම Karyotype (not microarray)
(Guseh, 2017)
20 yo with NIPT Positive for Trisomy 13
o Ultrasounds normal
o Declined invasive testing
o Had “arranged for palliative care
team”
o Positive predictive value=16 %
o Could her placenta have CPM for
trisomy 13?
40 yo, NIPT “negative” for
21,18,13, X,Y Aneuploidy
o NIPT at 11 weeks – negative, NL 5.0
mm
o US - early onset IUGR,
polyhydramnios
o Newborn karyotype - 47,XY, +18
o NIPT is screening with false negatives
o True false negative or possible CPM ?
377
35 yo, NIPT “negative” for 21,18,13,
X,Y Aneuploidy
Ultrasound at 20 wks delayed growth,
possible VSD, and club foot
Chose NIPT for aneuploidies to
exclude severe conditions (+13, + 18)
මResult = negative
Microarray reveals “Cri du Chat”
(46,XX,5p- syndrome)
What is the Role of cfDNA?
1) Appropriate first line for advanced
maternal age for trisomy 21 screening.
මSingletons, not twins
මNot validated for microdeletions
මDoes not replace diagnostic study for
ultrasound anomalies
(ACOG and SMFM joint statement, May 2016)
What is the Role of cfDNA in Women
< 35 years Old?
Pros
ම High detection, very low false-positives
ම Single blood test any time past 10 weeks
ම Provides a noninvasive risk assessment
Cons
ම Calculation of patient’s positive predictive value essential
ම Traditional serum screen identifies additional karyotype
anomalies
ම Cost efficacy remains to be established
(Norton, 2016)
378
Resources – ACOG / SMFM / NSGC
1) NIPT/Cell Free DNA Screening Performance Calculator (ACOG endorsed)
ම www.perinatalquality.org
2) “Free Webinar: Prenatal Cell-Free DNA Screening”
ම http://cfweb.acog.org/obpractice/webinars
3) “Prenatal Cell-Free DNA Screening: Q&A for Healthcare Providers”
ම http://nsgc.org/page/non-invasive-prenatal-testing-healthcare-providers (ACOG endorsed)
4) “Abnormal Prenatal Cell-Free DNA Screening Results: What Do They Mean?”
ම http://nsgc.org/page/abnormal-non-invasive-prenatal -testing-results (ACOG endorsed)
5) Resources for Women
ම "Cell-free DNA Prenatal Screening Test“ (http://www.acog.org/Patients/FAQs/Cell-freeDNA-Prenatal-Screening-Test-Infographic)
ම “Prenatal Cell-Free DNA Screening” -FAQ for patients (http://nasgc.org) (ACOG endorsed)
Clinical Case – Initiates First Trimester
Screening
She has her 11 week US
A nuchal lucency of 4.0
mm is reported
Clinical Case
She returns with her
husband one week later.
Should she have a CVS?
මWhat does it mean if it is
now smaller?
මShould she get NIPT?
මRisk of other conditions ?
Nuchal lucency = 2.0 mm
379
Should You Cancel the CVS?
Trisomy 21 with increased nuchal lucency
මMajority (5/6) resolved by 2nd trimester
ම1st trimester NT of 10, 7, 5, 5, 4, 8 mm
මNot associated with cardiac anomalies
මNot predictive of spontaneous loss
“If an abnormal fetus is likely to miscarry, shouldn’t I wait
until amniocentesis?”
මTrisomy 21 - About 10% chance (CVS to Amniocentesis)
(Comstock, 2006; Pandya, 1999; Sawa, 2006)
Outcomes for Aneuploidies at Midtrimester
1) Trisomy 21
මLoss rate from 24 weeks about 20%
2) Trisomy 18
මLoss rate about 80% to term
3) 45, X
මLoss rate lower with mosaicism
මCardiac anomalies and nonmosaic 45,X have highest
chance of in utero loss
If She Gets cfDNA ? Nuchal Lucency
and cfDNA
Microarray anomalies not
addressed
මCMA abnormalities associated
with increased NL
Role of other genetic syndromes
මNoonan syndrome
මCystic hygroma
16%
මIncreased NL
2%
(Bakker, 2011, Lee, 2009)
380
Clinical Case
Couple get cfDNA – negative for major
aneuploidies
Decide to wait for 2nd trimester ultrasound and
fetal DNA studies (amniocentesis)
What other concerns exist? What else do they
need to be prepared for?
ම Structural abnormality
ම Undetected birth defect / genetic condition in child
Structural Anomalies and
Inherited Disease
Cardiac anomalies – 15% risk if NL > 5.5 mm
Increased venous pressures, mediastinal compression
ම congenital diaphragmatic hernia
ම narrow thorax skeletal dysphasia
Altered extracellular matrix
ම collagen disorders (chondrodysplasias)
Impaired fetal movement
ම fetal akenesia sydromes
Noonan, Smith Lemli Opitz Syndromes
Long Term Outcomes with NT > 4.0
Increased nuchal lucency
50% abnormal karyotype
50% Normal karyotype
25% abnormal second trimester US
Cardiac and other anomalies
Fetal demise
75% Normal second trimester US
94% Normal at birth
94% Developmentally normal 12-36 mths
6% abnormal at birth
Cardiac (TGV)
Multiple malformaitons
6 % developmentally abnormal
NTs 6.3; 6.3 and 12
(Senat, 2002)
381
Ultrasound in the Second Trimester
Ultrasound for structural anomalies
Ultrasound for aneuploidy screening
ම“soft markers”
Ultrasound in the Second Trimester :
Structural Survey and Aneuploidy Risk
Multiple anomalies
ම18.8 %
Isolated anomalies
ම9.3 %
Issue of an “isolated” ultrasound anomaly – minor
features and dysmorphia difficult to appreciate
(Staebler, M., 2005)
Classic Presentations – Second Trimester
Cardiac
Other
Growth / Amniotic fluid
Trisomy 21
AV canal
Absent nasal bone, clinodactyly,
sandal gap, double bubble,
omphalocele
Mild third trimester growth
restriction, normal amniotic fluid
Trisomy 18
Variety
Clenched hands, CNS including
NTD, CDH, omphalocele
Significant growth restriction
(2nd trimester), polyhydramnios
Trisomy 13
Variety
Midline defects - Midline facial
clefting, holoprosencephaly
/cyclopia, omphalocele,
polydactyly
Third trimester growth
restriction, normal amniotic fluid
382
Classic Presentations – Second Trimester
Cardiac
Other
Growth / Amniotic
fluid
45, X
HLHS,
coarctation
Cystic hygroma, hydrops,
horseshoe kidneys
Normal growth and
amniotic fluid
Triploidy
Variety
CNS, omphalocele,
hypertelorism,
micrognathia
2nd trimester IUGR
(skeleton more than
head), 2nd trimester
placental thickening or
calcification
TABLE 1 – isolated ultrasound findings and risks of chromosome
abnormality (appendix)
2nd Trimester Ultrasound “Soft Markers”2nd
(Reddy, 2014)
2nd Trimester Ultrasound “Soft Markers”
Echogenic bowel
LR 5.5 – 6.7 fold increase;
also CMV, cystic fibrosis, IUGR
Absent nasal bone,
80 fold increase
Nuchal thickening
11 – 18 fold increase
383
Incorporating NIPT and “soft markers”
Occur individually in 1-5% of
normal fetuses
Emphasis is on isolated findings,
multiple “soft markers” should be
considered differently
(SMFM, 2017)
Case – Returns for 18 week ultrasound
මWhat is it?
මCould it have been seen sooner?
මRisk of undiagnosed aneuploidy ?
මRisk of genetic etiologies?
මHow to proceed?
Clinical Case – Abdominal Wall Defect
Centers for Disease Control and Prevention, National Center on Birth Defects
and Developmental Disabilities
384
Abdominal Wall Defects
Omphalocele
Gastroschisis
Location of umbilical
vessels
Into peritoneal covering
Lateral to mass
Membrane covered
Yes
No
Defect
Altered timing of normal
physiologic herniation /
enlarged internal organs
Vascular accident
Associations
Other malformations,
genetic syndromes and
aneuploidy
Young age, smoking,
cocaine
Concerns?
What is it? - Omphalocele
Could it have been seen sooner?
මPossibly, can be confused
though with physiologic
herniation of small bowel into
umbilical stalk at 8-10 wks
Concerns?
Risk of undiagnosed aneuploidy ?
මRisk of false negative for the major aneuploidies
මLow (< 2.0%)
Risk of other chromosome abnormalities ?
මAbout 10 - 15% (aneuploidy and micro del/dups)
Risk of genetic etiologies ?
385
To Be Continued . . .
Thank You for Your Attention
Table 1 - Aneuploidy Risks among Isolated Major Anomalies
High risks
Cystic hygroma
Hydrops
Holoprosencepahly
Complete AV Canal
Omphalocele
Duodenal atresia
Bladder outlet obstruction
Risk
> 50%
> 50%
50%
40%
30%
30%
20%
Most Common
45,X
13,21,18, 45,X
13, 18, 18p21
13, 18
21
13, 18
Lower risks
Hydrocephaly/
Ventriculomegaly
10%
Cardiac defects
10%
Meningomyeloceles
7%
Anencephaly
2%
Encephalocele
10%
Limb reduction
8%
Clubfoot
6%
Facial clefts
1%
Minimal risk
Gastroschesis-provided differentiated from ruptured omphalocele
Hydranencepahly
Single umbilical artery
Above numbers are estimates which are influenced by gestational age at detection.
21, 13, 18, triploidy
21, 18, 13, 22-, 8, 9
18
18
47,XXY,47,XXX,18,21
13, 18, 22q-
References
American College of Obstetricians and Gynecologists. Screening for fetal aneuploidy. ACOG
Practice bulletin no. 163. (2016) Obstet Gynecol, 127:e123-37.
American College of Obstetricians and Gynecologists. Cell-free DNA screening for fetal
aneuploidy. ACOG Committee opinion no. 640.(2015) Obstet Gynecol,126:e31-7.
American College of Obstetricians and Gynecologists. Ultrasound in pregnancy. ACOG Practice
bulletin no. 175. (2016) Obstet Gynecol,128:e241-56.
Bianchi, D. W., T. Wataganara, et al. (2006). "Fetal nucleic acids in maternal body fluids: an
update." Ann N Y Acad Sci 1075: 63-73
Cuckle, H. S., F. D. Malone, et al. (2008). "Contingent screening for Down syndrome--results
from the FaSTER trial." Prenat Diagn 28(2): 89-94.
Fan, H. C., Y. J. Blumenfeld, et al. (2008). "Noninvasive diagnosis of fetal aneuploidy by
shotgun sequencing DNA from maternal blood." Proc Natl Acad Sci U S A 105(42):
16266-71.
Gregg A, Van den Veyver I, Gross S. J., et al. (2014)”Noninvasive prenatal screening by nextgeneration sequencing” Annu Rev Genomics Hum Genet 15:327-47.
Gregg AR, Skotko BG, Benkendorf JL, et al. (2016) Noninvasive prenatal screening for fetal
aneuploidy. 2016 update: a position statement of the American College of Medical
Genetic and Genomics. Genet Med,10:1056-65.
Lefkowitz RB, Tynan JA, Liu T, et al. (2016) Clinical validation of a noninvasive prenatal test for
genomewide detection of fetal copy number variants.AmJ Obstet Gynecol, 215:227.e116.
Malone, F. D., J. A. Canick, et al. (2005). "First-trimester or second-trimester screening, or both,
for Down's syndrome." N Engl J Med 353(19): 2001-11
Nicolaides, K. H. (2004). "First-trimester screening for Down's syndrome." N Engl J Med 350(6):
619-21
Norton M, Jelliffe-Pawlowski L, Currier R (2014). “Chromosome abnormalities detected by
current prenatal screening and noninvasive prenatal testing” Obstet Gynecol 124(5):97986.
Norton, ME, Biggio, JR, Kuller, JA, Blackwell SC, (2017)The role of ultrasound in women who
undergo cell-free DNA screening Society for Maternal-Fetal Medicine (SMFM) Consult
Series #42
Pergament E, Cuckle H, Zimmermann B, Banjevic M, Sigurjonsson S, et al. (2014) “Singlenucleotide polymorphism-based noninvasive prenatal screening in a high-risk and lowrisk cohort” Obstet Gynecol 124 (2):210-8.
Reiff ES, Little SE, Dobson L, Wilkins-Haug L, Bromley B. (2016).What is the role of the 11- to
14-week ultrasound in women with negative cell-free DNA screening for aneuploidy?
Prenat Diagn, 36:260-5.
Society for Maternal-Fetal Medicine (SMFM) Publications Committee. Consult Series 36 (2015):
prenatal aneuploidy screening using cell-free DNA. Am J Obstet Gynecol 212:711-6.
Swaminathan, R. and A. N. Butt (2006). "Circulating nucleic acids in plasma and serum: recent
developments." Ann N Y Acad Sci 1075: 1-9.
Lo YM, Corbetta N, Chamberlain PF, Rai V, Sargent IL, Redman CW, Wainscoat JS. 1997.
Presence of fetal DNA in maternal plasma and serum. Lancet. 350(9076):485-7.
386
Reproductive Genetics: Prenatal Diagnosis Table 1 and References
Table 1 - Aneuploidy Risks among Isolated Major Anomalies
High risks
Cystic hygroma
Hydrops
Holoprosencepahly
Complete AV Canal
Omphalocele
Duodenal atresia
Bladder outlet obstruction
Risk
> 50%
> 50%
50%
40%
30%
30%
20%
Most Common
45,X
13,21,18, 45,X
13, 18, 18p21
13, 18
21
13, 18
Lower risks
Hydrocephaly/
Ventriculomegaly
Cardiac defects
Meningomyeloceles
Anencephaly
Encephalocele
Limb reduction
Clubfoot
Facial clefts
10%
10%
7%
2%
10%
8%
6%
1%
21, 13, 18, triploidy
21, 18, 13, 22-, 8, 9
18
18
47,XXY,47,XXX,18,21
13, 18, 22q-
Minimal risk
Gastroschesis-provided differentiated from ruptured omphalocele
Hydranencepahly
Single umbilical artery
Above numbers are estimates which are influenced by gestational age at detection.
Nyberg D, Mahony B, Pretorius D. Diagnostic Ultrasound of Fetal Anomalies: Text and
Atlas. Mosby Year Book, 1990
Sanders R, Hogge W, Spevak P, Wulfsberg E. Structural Fetal Abnormalities-the Total
Picture. Mosby, 2002.
Shipp, T. D, Benacerraf, B. R., The significance of prenatally identified isolated clubfoot:
is amniocentesis indicated? Am J Obstet Gynecol 178 : 600-2.
387
References
American College of Obstetricians and Gynecologists. Screening for fetal aneuploidy. ACOG
Practice bulletin no. 163. (2016) Obstet Gynecol, 127:e123-37.
American College of Obstetricians and Gynecologists. Cell-free DNA screening for fetal
aneuploidy. ACOG Committee opinion no. 640.(2015) Obstet Gynecol,126:e31-7.
American College of Obstetricians and Gynecologists. Ultrasound in pregnancy. ACOG Practice
bulletin no. 175. (2016) Obstet Gynecol,128:e241-56.
Bianchi, D. W., T. Wataganara, et al. (2006). "Fetal nucleic acids in maternal body fluids: an
update." Ann N Y Acad Sci 1075: 63-73
Cuckle, H. S., F. D. Malone, et al. (2008). "Contingent screening for Down syndrome--results
from the FaSTER trial." Prenat Diagn 28(2): 89-94.
Fan, H. C., Y. J. Blumenfeld, et al. (2008). "Noninvasive diagnosis of fetal aneuploidy by
shotgun sequencing DNA from maternal blood." Proc Natl Acad Sci U S A 105(42):
16266-71.
Gregg A, Van den Veyver I, Gross S. J., et al. (2014)”Noninvasive prenatal screening by nextgeneration sequencing” Annu Rev Genomics Hum Genet 15:327-47.
Gregg AR, Skotko BG, Benkendorf JL, et al. (2016) Noninvasive prenatal screening for fetal
aneuploidy. 2016 update: a position statement of the American College of Medical
Genetic and Genomics. Genet Med,10:1056-65.
Lefkowitz RB, Tynan JA, Liu T, et al. (2016) Clinical validation of a noninvasive prenatal test for
genomewide detection of fetal copy number variants.AmJ Obstet Gynecol, 215:227.e116.
Lo YM, Corbetta N, Chamberlain PF, Rai V, Sargent IL, Redman CW, Wainscoat JS. 1997.
Presence of fetal DNA in maternal plasma and serum. Lancet. 350(9076):485-7.
Malone, F. D., J. A. Canick, et al. (2005). "First-trimester or second-trimester screening, or both,
for Down's syndrome." N Engl J Med 353(19): 2001-11
Nicolaides, K. H. (2004). "First-trimester screening for Down's syndrome." N Engl J Med 350(6):
619-21
Norton M, Jelliffe-Pawlowski L, Currier R (2014). “Chromosome abnormalities detected by
current prenatal screening and noninvasive prenatal testing” Obstet Gynecol 124(5):97986.
Norton, ME, Biggio, JR, Kuller, JA, Blackwell SC, (2017)The role of ultrasound in women who
undergo cell-free DNA screening Society for Maternal-Fetal Medicine (SMFM) Consult
Series #42
388
Pergament E, Cuckle H, Zimmermann B, Banjevic M, Sigurjonsson S, et al. (2014) “Singlenucleotide polymorphism-based noninvasive prenatal screening in a high-risk and lowrisk cohort” Obstet Gynecol 124 (2):210-8.
Reiff ES, Little SE, Dobson L, Wilkins-Haug L, Bromley B. (2016).What is the role of the 11- to
14-week ultrasound in women with negative cell-free DNA screening for aneuploidy?
Prenat Diagn, 36:260-5.
Society for Maternal-Fetal Medicine (SMFM) Publications Committee. Consult Series 36 (2015):
prenatal aneuploidy screening using cell-free DNA. Am J Obstet Gynecol 212:711-6.
Swaminathan, R. and A. N. Butt (2006). "Circulating nucleic acids in plasma and serum: recent
developments." Ann N Y Acad Sci 1075: 1-9.
389
390
Cancer Genetics II
CANCER GENETICS II
Sharon E. Plon, MD, PhD, FACMG
Professor, Department of Pediatrics/Hematology-Oncology
Molecular and Human Genetics
Human Genome Sequencing Center
Director, Medical Scientist Training Program
Baylor College of Medicine
Sharon E. Plon, MD, PhD, FACMG
Department of Pediatrics
Feigin Center Room 1200.18, 1102 Bates Street
Houston, TX 77030
(832) 824-4251 Telephone
(832) 825-4276 Fax
[email protected]
393
394
Cancer Genetics II
Sharon E. Plon, MD, PhD
Professor
Baylor College of Medicine
Disclosure(s)
• I have the following financial relationships to disclose:
• I am a employee of Baylor College of Medicine (BCM) which derives revenue from genetic testing,
including whole exome sequencing.
• BCM and Miraca Holdings Inc. have entered into a joint venture, Baylor Genetics with shared
ownership and governance of the clinical genetics diagnostic laboratories
• I am a member of the BMGL Scientific Advisory Board
• I will discuss off label use and/or investigational use in my presentation.
General features of hereditary
cancer genetic testing
395
Examples of hereditary cancer syndromes w/ Management
specific care guidelines
modalities
Hereditary breast/ovarian cancer (BRCA1/2)
Lynch syndrome (MSH2, MLH1, MSH6, PMS2)
Familial adenomatous polyposis (APC)
Von Hippel Lindau syndrome (VHL)
Tuberous sclerosis complex (TSC)
Multiple endocrine neoplasia 2 (RET)
Li Fraumeni syndrome (TP53)
Imaging
Other diagnostic
modalities
Prophylactic
surgery
Targeted
medications
Genetic testing approaches when specifically
performing hereditary cancer evaluation
Single Genes
Very Specific
Phenotype Sanger
2-4 Genes – Sanger/NGS
Multiple gene panels – Highly
variable in size (7 – 150 genes)
Performed by a variety of different
NextGen sequencing methods w/
copy number analysis
Whole exome or whole genome analysis (maybe
part of tumor study) – NextGen methods +/- copy
number analysis
RB1 or VHL
TSC1 & TSC2 or
APC & MUTYH
Hereditary breast
cancer or hereditary
colon cancer panel
Whole exome
analysis
Outcomes of hereditary panel testing
• Simplifies the genetic testing process and it is more efficient for patients.
• Can provide “surprising results” with regard to pathogenic variants
despite atypical family history or the “right gene: wrong tumor”:
• Panel may include genes for which there is limited information with
regard to pathogenicity
• ClinGen consortium is now systematically reviewing this evidence in
what is referred to as “clinical validity framework”
(www.clinicalgenome.org)
• Using panels increases the likelihood of variants of uncertain significance
• In one study with breast panel (30 genes), per patient, the average
number of VUS across all genes was 2.1 (Kurian, JCO, 2014)
396
In addition there are many situations were one can
infer germline findings from TUMOR ONLY panel or
WES/WGS sequencing
• Certain specific founder mutations are almost always found
in constitutional DNA:
• Ashkenazi founder mutations in BRCA1/BRCA2
• BRCA1 loss of function mutations in breast cancer
• EPCAM deletion in colon cancer sample
• Specific tumor subtypes with high germline yield:
• TP53 mutation in hypodiploid ALL (90% TP53; 48% germline)
• Medullary thyroid cancer (RET)
• Mutation profile associated with germline findings:
• Hypermutated colon cancer (POLE)
• Chromothripsis in medulloblastoma (TP53 mutation)
Zhang J et al. N Engl J Med 2015. DOI: 10.1056/NEJMoa1508054
Distribution of Germline Mutations in Different
Gene Categories and Cancer Subtypes in n=1120
childhood cancer patients
• 8.5% had pathogenic variants in dominant
genes (entire cohort – not unselected tumors)
• 5.6% if exclude hypodiploid ALL and ACT
• Only one biallelic recessive diagnosis
• 8.5% of single recessive P/LP variants
Zhang J et al. N Engl J Med 2015. DOI:
10.1056/NEJMoa1508054
397
Data from the MSKCC (adult) analysis of germline findings in
IMPACT panel (341 genes)
• 246 of 1566 subjects (15.7% 95% CI 14.0%
- 17.6%) with germline findings
o 198 of 246 individuals, (80.5%; 95%
CI, 75.1%-85.0%) of findings in cancer
susceptibility genes
• Often found P/LP variants in genes not
currently associated with the tumor
diagnosis of the patient.
Summary of germline findings in unselected
cancer patients
• Multiple studies find ~10-15% of diverse pediatric and adult cancer populations
carry P/LP variants in wide range of dominant cancer susceptibility genes.
• Mixture of genes with with and without prior association with the patient’s tumor
• Another ~6% carry single recessive alleles (unclear whether this is increased over control
populations).
• Metastatic Prostate cancer patients – 12% carry P/LP variant in DNA repair genes
potentially associated with sensitivity to PARP inhibitors.
• Initiating genetic testing in prostate cancer patients
• Many studies demonstrate that only about 50% of patients with germine
findings would meet clinical criteria for genetic testing (e.g. NCCN, Bethesda).
• Most existing criteria are designed to be highly specific but not necessarily sensitive for
identifying cancer patients with germline cancer susceptibility mutations.
Hereditary Colon Cancer and
Polyposis Syndromes
398
Risk factors for Colon Cancer
Situation
Lifetime Risk of CRC
General Population
6%
Personal history of CRC
Inflammatory Bowel
Disease
Hereditary Nonpolyposis
Colon Cancer (HNPCC)
Familial Adenomatous
Polyposis
15-20%
15-40%
60-80%
>95%
Familial Adenomatous Polyposis (FAP)
• Autosomal Dominant disorder due to loss of function mutations in APC
with very high penetrance for polyps and colorecal cancer
• Acts as a classic tumor suppressor gene with two hits
• >90% are truncating variants; 5-10% result from deletions
• Some genotype:phenotype correlations.
• Extracolonic symptoms assoc/w variants in long exon 15 in middle of gene.
• Attenuated FAP (fewer polyps, later CRC onset) variants cluster at far 5’ & 3㵭ends.
• 15-30% of patients with FAP result from de novo mutations.
• Adenomatous colonic polyps begin in childhood to adolescence.
• Nearly 100% lifetime colorectal cancer risk with 50% risk by age 33.
• Extracolonic features often referred to as Gardner Syndrome.
Polyposis Associated with FAP
Twin A
Twin B
Typical Adenomatous Polyp from
colon of a teenager with FAP
Photographs courtesy of M. Finegold, MD
– Texas Children㵭s Hospital
Colons of twin teenage boys who presented
with history of rectal bleeding and abdominal
pain and underwent prophylactic colectomy
399
Extracolonic Manifestations of FAP
• Desmoid tumors – often abdominal.
• Painful and can be difficult to treat.
• More likely to occur at sites of surgical resection
• Osteomas of the jaw, skull, or other bones
• Epidermoid cysts on face or trunk
• Congenital Hypertrophy of the Retinal Pigment Epithelium (CHRPE)
• present at birth, asymptomatic but useful clinically.
• Pediatric hepatoblastoma (~0.5-1% risk)
• Some studies suggest that ~10% of hepatoblastoma patients have FAP
• Thyroid cancer (1% risk)
• Medulloblastoma (<1% risk)
Inverse relationship between somatic &
germline mutations
• Hepatoblastoma and desmoid tumors are associated with two apparently
exclusive events:
• Somatic activating mutations (exon 3 mutations) of beta catenin (CTNNB1 gene).
OR
• Germline inactivating mutations in APC.
• Only one is needed to upregulate beta catenin signaling so I use somatic
information to decide upon need for germline testing for hepatoblastoma
and desmoid tumors.
• There are other examples of these types of inverse relationships between
activating oncogenes in tumor or germline LOF in tumor suppressor
genes.
Management of W+ Patients
• Colonoscopy surveillance beginning age 10–15; continuing every 1-2 yrs.
• Colectomy late teens to early twenties (depending on polyp load or
dysplasia).
• Restorative proctocolectomy with ileal pouch-anal anastomosis
• Total colectomy with ileorectal anastomosis with annual surveillance of rectal
stump.
• Consider Sulindac or NSAID’s for residual stump to decrease size of polyps.
• Recent study exploring erlotinib for decreasing polyp burden
• Upper GI follow-up by endoscopy for risk of gastric adenomas and
duodenal carcinomas beginning late teens or early twenties.
• Annual thyroid exam
• Hepatoblastoma screening in children controversial.
400
Dhdz,Associated Polyposis
• Autosomal recessive polyposis with >100 polyps.
• Two MUTYH pathogenic variants, p.Y165C & p.G382D.
• The mean ages of CRC diagnosis:
• 58 years (homozygous G382D)
• 52 years (compound heterozygous)
• 46 years (homozygous Y165C)
• Conflicting data over whether there is any increased CRC risk in
heterozygous patients (~1% allele frequency).
• MUTYH encodes base excision repair protein.
• Absence of BER leads to increased mutation rate with APC
somatic mutations in the polyps.
70
55
CRC
dx 66?
48
52
Siblings with >100
polyps and CRC
= MUTYH mutation
Juvenile Polyposis Coli
• Present with bleeding, rectal prolapse, pain.
• Childhood onset of juvenile/ hamartomas
colonic polyps (>5 polyps).
• Mutations in BMPR1A and SMAD4 found in
~50% of patients.
• Severe form with deletion including PTEN and
BMPR1A
• ~1-2% result from PTEN mutations.
• CRC rate as high as 50% with average age
diagnosis 43 yrs.
• SMAD4 > BMPR1A mutation
• Remaining JPC loci still not discovered.
Photographs courtesy of M. Finegold, MD
– Texas Children’s Hospital
JPC Management
• Monitor for rectal bleeding, anemia, abdominal pain, constipation,
diarrhea or change in stool size, shape, and/or color;
• CBC, colonoscopy, and upper endoscopy beginning by 15 years of age.
• Goal is removal of polyps and then periodic surveillance.
• Prophylactic colectomy not recommended but maybe required if polyp
burden is very high w/dysplastic changes or excess bleeding.
• Some patients with SMAD4 P/LP variants with hereditary hemorrhagic
telangiectasia and JPC clinical phenotypes :
• Can see severe HHT complications including chronic epistaxis, pulmonary AVMs.
• Screening for HHT complications recommended in all JPC patients with SMAD4
mutations (Zbuk and Eng, Nature Practice, 2007)
• One study found 6/16 (38%) SMAD4 mutation carriers had evidence of aortopathy
(Heald et al., AJMG, 2015) – may result in updated guidelines.
401
Peutz-Jeghers Syndrome
• Pigmented spots on lips, buccal mucosa and GI tract
• Can fade w/age
• Hamartomas of the small and large bowel.
• Intussusception – serious complication which can present at later
ages than in general population (need to warn families).
• 50% deletions/45% truncating mutations in STK11:
• Highlights the need for sequence AND deletion analysis
• Lifetime cancer risk of 81% (Zbuk & Eng, Nature Practice Oncol, 2007)
Photograph courtesy of S. Plon
– Texas Children’s Hospital
• GI cancers (66%- small intestine, CRC, esophageal & pancreatic)
• Breast cancer risk 32% in women.
• Surveillance should include breast MRI
• STK11 on most BRCA panel tests although rarely positive (small contributor)
• Benign ovarian sex-cord tumors with annular tubules and Sertolicell testicular tumors – indication for STK11 testing.
'ZDϭ Polyposis
• Described in 2012 as another form of “mixed” polyposis
including both adenomatous, juvenile & hyperplastic polyps.
• Major Ashkenazi disease allele associated with disease is
40kb duplication upstream of GREM1.
• Other rearrangements/duplications have been reported in a few
families.
• Maybe missed on panels that don’t include copy number.
• Recent review of four extended Ashkenazi families
demonstrate (Lieberman et al., Gastroenterology. 2017)
• GREM1 mutation carriers can phenotypically resemble either
FAP or Lynch syndrome.
Lynch Syndrome or Hereditary Non-Polyposis
Colon Cancer
• Autosomal dominant CRC w/o polyposis associated with
endometrial ca, bile duct, ovarian, ureteral and gliomas.
• ~70% CRC lifetime risk and 50-70% endometrial Ca in classic
Lynch.
• Right-sided CRC cancer is more frequent.
• Better prognosis of CRC stage for stage.
• Common to see individuals with 2 or 3 different primary LS-tumors
• Due to high mutation rate these patients respond more
effectively to immune checkpoint inhibitors
• Definitely increasing the interest in identifying LS patients
402
Family History Criteria for HNPCC
• Amsterdam Criteria (CRC based) – first exclude FAP
• At least 1 CRC < age 50
• 2 affected generations
• 3 affected relatives, 2 are 1o relatives of other one
• Revised Bethesda Criteria – based on proband characteristics
• CRC <50 yrs
• Synchronous, metachronous CRC, or other LS tumors.
• CRC with the MSI-H† histology‡ in a patient <60.§
• CRC plus 1st relative with LS tumor (<age 50).
• CRC in 2 or more relatives with LS tumors.
• These criteria have low sensitivity (but high specificity).
• Many groups now argue for screening all colorectal cancer
patients for tumor features of LS
HNPCC- Results from Heterozygous P/LP
variants in one of 4 Mismatch Repair Genes
• MSH2 and MLH1 mutations make up 80-90% of Amsterdam criteria families.
• Most are typical LOF mutations in MMR gene (truncating and deletions); MSH2 inversion
• MSH6 (MIS-low tumors) CRC: 44% for men & 22% for women (Baglietto, JNCI 2010).
• PMS2 testing difficult due to multiple pseudogenes w/gene conversion events.
• Very poorly covered in WES or tests that don’t have focused approach to PMS2.
• Lower CRC penetrance BUT ~8% of CRC diagnoses <30 yrs (McKinsey et al., GIM, 2015).
• EPCAM deletion (fusion transcript results in epigenetic silencing of adjacent MSH2 gene)
/EPCAM
Microsatellite Instability (MIS+) in MMR mutant tumors
• NCI definition using five markers –
MSI - Boland et al., Cancer Research, 1998
o Stable – 0
o Low – 1 unstable
o High - >2 unstable
• Previously standard approach to
evaluating tumors for evidence of Lynch
syndrome.
• However, most pathology labs more
routinely do immunohistochemistry.
• MIS studies do not tell you anything about
which gene may be responsible for the
MIS phenotype.
403
Work-up of CRC patients/specimens
More studies arguing to to evaluate all CRC or at least those that meet age cut-off (<70)
see Schneider et al., GIM, 2015 and Leenen et al, Genet Med. 2016
CRC +/- age
MMR Protein Expression
by immunohistochemistry (IHC)
MSI
Yes
MLH1 absent
MSH2, MSH6
or
PMS2 absent
BRAF p.V600E or
MLH1 methylation
No
Consider mutation testing
of blood for MMR mutations
Yes
Sporadic
HNPCC Guidelines for mutation positive
individuals – NCCN 2016
• Colon cancer: Colonoscopy beginning at age 20–25 y or 2–5 y prior to the earliest CRC (if
<25y age of diagnosis) and repeat every 1–2 y.
• Unclear how useful aspirin treatment is in cancer prevention
• Endometrial and ovarian cancer:
• Prophylactic hysterectomy and bilateral salpingo-oophorectomy (BSO) should be considered by women
who have completed childbearing.
• Dysfunctional uterine bleeding warrants evaluation.
• No clear evidence to support further endometrial cancer screening. However, annual endometrial
sampling is an option.
• Transvaginal ultrasound for ovarian & endometrial cancer not been shown to be sufficiently sensitive or
specific to support a positive recommendation. Similar caveats for serum CA-125.
• Other LS cancers - no active surveillance other than clinic suspicion is recommended.
• LS surveillance has been shown to increase survival.
• However, only 50% of close relatives of Lynch syndrome patients undergo genetic testing – we need to
do better!
WK>ͬWK>ϭAssociated Hereditary CRC
• Specific missense variants in POLE/POLD1 exonuclease domain were first
identified as somatic mutations in colorectal cancer samples found to be
hypermutated (many more somatic mutations c/w MMR deficient tumors).
• Mutations increase the polymerase error-prone repair activity of these
proteins (not standard tumor suppressor gene with 2 hits).
• In a subset of patients variants are germline with hereditary cancer.
•
•
•
•
•
Attenuated or oligo-adenomatous colorectal polyposis (average 16 adenomas)
Risk of CRC (~60%)
Gastric and duodenal adenomas (57%)
Also maybe more responsive to immune checkpoint inhibitors
Recent article Belido et al., Genet Medicine, 2016 Apr;18(4):325-32
404
Specific missense variants in the
conserved region of these error
prone polymerases results in a
hyper-mutated tumor which occurs
in germline or as a somatic finding.
Germline and somatic polymerase İ and į mutations
define a new class of hypermutated colorectal and
endometrial cancers.
Briggs S, Tomlinson I - J. Pathol. (2013)
Colon Cancer Molecular Pathology Summary
• 85% of CRC demonstrate mutation and LOH of the APC gene.
• Initiating events for polyp formation.
• 1% due to germline APC mutation
• 15% of CRC – demonstrate microsatellite instability or abnl MMR IHC.
• 13% Results from silencing of both copies of the MLH1 promoter by methylation.
• These patients may not respond to 5-Fluoro-uracil.
• 2% due to germline MMR mutations
• Rare individuals with hypermutated tumors and germline findings
• There are a variety of NextGen CRC panels including: APC, MUTYH,
MSH2, MLH1, MSH6, PMS2, STK11, SMAD4, BMPR1A, POLE, POLD1.
Constitutional Mismatch Repair Deficiency
Syndrome - cMMRD
• Turcot Syndrome (original name) – association of brain tumors and colon
polyps/cancer in childhood.
• Dominant forms with FAP and medulloblastomas or HNPCC and glioblastomas
• Autosomal Recessive form due to biallelic inactivation of MMR genes =
MMR deficiency – possibly highest pediatric tumor risk of any syndrome
• Atypical café au lait spots and axillary freckling
Genes
Age of Diagnosis
of 1st tumor
Leukemia
Lymphoma
Brain Tumors
HNPCCTumors
MLH1, MSH2
n=20 pts.
3.5y
(0.4 -35)
11
4
6
MSH6, PMS2
n=55 pts.
9y
(1-31)
16
32
35
Modified from Wimmer & Etzler (2008)
405
cMMRD Management
• Skin findings (NF1 like), tumor types and consanguinity key clues to
diagnosis:
• IHC should demonstrate absence of MMR protein in both normal and
tumor DNA.
• Surveillance recommendations (Durno et al., Eur J Can, 2015) includes:
• GI tract include Colonoscopy (> age 6) and EGD and video endoscopy
(>age 8) annually
• Brain MRI q6 months (from birth)
• Adults – add ultrasound of uterus and urinary tract annually.
• New guidelines considering adding whole body MRI as there are a wide
variety of other tumor types seen in these children/young adults.
Hereditary Breast and
Ovarian Cancer
Hereditary Breast cancer syndromes
• Breast-Ovarian Families – BRCA1>BRCA2
• Li-Fraumeni –average onset 32 yo in TP53 carriers
• Peutz-Jeghers Syndrome (STK11)– 32% by age 60
• Hereditary breast cancer – PALB2 (some risk of pancreatic cancer)
• Cowden syndrome – PTEN
• Hereditary diffuse gastric cancer & lobular breast cancer – CDH1 mutations
• diffuse gastric (80% risk) and lobular BRCA (40-50% in women) and cleft lip/palate
• RECQL – discovered 2015 in Polish, French Canadian kindreds
• Moderate risk alleles RR~2.0
• CHEK2 (1100delC founder), RAD51C
• Ataxia telangiectasia (ATM) heterozygotes.
406
Cowden syndrome – WdE Hamartoma
syndrome
• Breast Cancer risk (~30% lifetime risk).
• Thyroid cancer (~10% lifetime risk).
• Adult Lhermitte-Duclos disease (LDD), cerebellar dysplastic gangliocytoma
• Mucocutaneous lesions
•
•
•
•
•
Trichilemmomas (facial)
Acral keratoses
Papillomatous lesions
Mucosal lesions
Pigmented spots on penis
• Macrocephaly
• Autism
Zϭ;17q21) – Hereditary Breast/Ovarian Cancer
• Associated with loss of function mutations found throughout large gene.
•
•
•
•
Most are truncating: frameshift or nonsense mutations.
Rare missense alleles in ring finger and BRCT domain
Deletions or rearrangements make up 2-5% of disease alleles.
Hundreds of different rare missense variants most of which are benign or VUS.
• In hereditary cancers follows the two hit hypothesis with loss or inactivation
of second allele in tumors.
• However, somatic mutations in sporadic breast cancer are almost never
found in breast cancer and very rare in other tumors.
• Tumors - basal-like gene expression profile.
• Triple negative (ER/PR/HER2) more likely.
ZϮ – 13q12
• Similar to BRCA1 with LOF mutations spread throughout the gene.
• Rare somatic mutations in sporadic breast cancers and other tumors.
• Compared with sporadic breast cancer it is difficult to distinguish the
specific gene expression profile in BRCA2 mutant tumors.
• Notable for significant predisposition to male breast cancer (6%) and
pancreatic cancer (~1%) in men and women.
• There is genomic instability – referred to BRCAness or HRD (homologous
recombination deficient) associated with BRCA1/2 or related DNA repair
genes being mutant.
407
Ashkenazi Founder Mutations
• Three founder mutations :
• Two specific mutations in BRCA1 (185delAG and 5382insC) and one in BRCA2
(6174delT).
• Carrier frequency of 2.4% for all three mutations.
• Start testing with these three mutations.
• Responsible for ~50% of Jewish BRCA families.
• If negative for founder mutations recommend full sequence mutation
analysis for higher risk families.
• Can calculate BRCAPro score as non-Ashkenazi to measure residual risk of
BRCA1/2 mutations.
• Population screening of Ashkenazi individuals not yet recommended by
US guidelines but underway in Israel
Panels and Variants of Uncertain Significance (VUS)
• VUS in ~3-5% of BRCA1/2 sequence tests.
• Predominantly missense mutations in protein regions without known function.
• A variety of approaches including conservation, computational predictions, segregation
with cancer and population studies are utilized to try and determine the significance.
• Different laboratories may report out same variant as a VUS or likely pathogenic
or likely benign based on their laboratory’s criteria.
• Data sharing through ClinVar and other databases helps to decrease discordance across
laboratories.
• ENIGMA consortium is a ClinVar Expert Panel (3*) for evaluation of BRCA1/2 variants.
• Breast cancer patients – dozens of panel studies:
• BRCA1, BRCA2 are the most often seen genes with P/LP variants
• Can be used for PARP inhibitor treatment selection
• In BRCA1/2 negative about 11% panel positive. CHEK2, ATM some of the most frequently
detected (Maxwell et al, Genet Med. 2015 Aug;17(8):630-8)
ClinVar Variant View
408
BRCA1, BRCA2, PALB2, RAD51C Proteins
• Response to DNA damage regulates homologous recombination
• ATM phosphorylates BRCA1;
• BRCA1 interacts with Rad51
• BRCA2 interacts with Rad52 and PALB2
• BRCA1 or BRCA2 mutant cells are synthetic lethal with PARP inhibitors (a
second back-up path of DNA repair)
• Ovarian tumors which are BRCA1 or BRCA2 mutant have increased sensitivity
to PARP inhibitors.
• Olaparib – FDA approved with positive Myriad Genetics Laboratory companion diagnostic test.
• About 11% of men w/metastatic prostate cancers demonstrate HRD much
higher than those with localized prostate cancer
Risk Prediction Models
• There are well established computer models which we can use to predict
• Risk of developing breast cancer for someone of 㵰average㵱 risk – Gail
Model.
• Not appropriate for very high risk families.
• Has to be at least 35 years old
• Risk of developing breast cancer based on family history of cancer –
Claus Tables.
• Likelihood that genetic testing will yield a mutation in BRCA1 or BRCA2:
• BRCAPro (US model)
• BOADICEA (UK model)
Cancer risk for Zϭ͕ZϮ͕W>Ϯpathogenic
variant carriers
Tumor type
BRCA1
BRCA2
PALB2
Female breast
cancer
50-85%
50-85%
33-58%
Male breast
cancer
<1%
6%
Ovarian cancer
15-40%
15-25%
Prostate cancer
8-16%
8-16%
Pancreatic
cancer
1.5-2%
Not increased
(make up 1%
PRCA families)
409
Guidelines for ZϭͬϮ carriers
Screening test
Interval of Screening
Clinical breast exam
q6-12 months from age 25
Mammogram
Yearly beginning age 25
Breast MRI
Yearly beginning age 25
Transvaginal ultrasound
Q6 mo beginning age 30
Blood CA-125 level
Q6 mo beginning age 30
Comments
May defer to age 30
No data available if defers BSO
No data available – if
defers BSO
Prophylactic Surgery
Bilateral mastectomy
Discuss option with patient
Risk reducing Bilateral
salpingo-oophorectomy
(RRSO)
Recommend once childbearing is complete around age 35-40.
OVCA develops about 10yrs later so can consider delaying RRSO if patient
had bilateral mastectomy
No clear pancreatic cancer or melanoma screening guidelines
but may be recommended depending on family history
Modified from NCCN 2017 guidelines.
Multiple Endocrine Neoplasia 1
• MEN1 classic TSG with LOF and 2nd hits; Tumor spectrum includes:
•
•
•
•
Parathyroid
Pancreatic islet cell tumors
Anterior pituitary hyperplasia
Zollinger-Ellison Syndrome (much higher mortality in sporadic ZES versus MEN1
associated ZES).
• Biochemical investigations (yearly)
• Serum concentration of prolactin from 5 yo
• Fasting total serum calcium calcium from 8 yo
• Fasting serum gastrin from 20 yo
• Imaging - every 3-5 yrs
• Head MRI from 5 yo
• Abdominal CT or MRI from 20 yo
Screening guidelines in Thakker et al, J Clin Endocrinol Metab 2012
Pheochromocytomas - Paragangliomas
• 40-50% of pheochromocytomas/paragangliomas patients carry P/LP variants.
• Succinate dehydrogenase subunits (SDHB, SDHD, SDHC, SDHA, SDHAF2) encode
mitochondrial proteins.
• SDHD imprinted, affected when inherited from father.
• SDHB mutations associated with malignant forms of the tumors.
• More likely to have extra-adrenal tumors including the chemoreceptor organs (glomus and
carotid body tumors)
• MAX, KIF1B, TMEM127, EGLN1
• VHL – Von Hippel Lindau syndrome – type 2 (missense mutations) associated with
high risk of adrenal and extra-adrenal pheochromocytomas with elevated
metanephrines
• RET – MEN2 syndrome
• Rarely NF1
• Can also see GIST tumors
410
AIP Related Familial Isolated Pituitary Adenomas
• Pituitary adenomas usually expressing GH (somatotropinoma),
PrL (prolactinoma), TSH (thyrotropinoma), ACTH
(corticotropinoma) or non-functioning. Age of onset 20-24 yrs
• AD with reduced penetrance resulting from heterozygous
mutations in AIP
• Aryl hydrocarbon receptor Interacting Protein
• Mutation: ~90% sequence & 10% del/dup
• Treat by surgery, medical or radiotherapy
Genetics of Renal Cell Cancer
• Histology typically helps decide which gene to test for or use NextGen
panel.
• VHL – almost always clear cell histology.
• Conversely 80% of sporadic RCC has somatic VHL mutations.
• Balanced translocation carriers involving chromosome 3
• A variety of different translocations result in AD RCC (clear cell)
• Papillary renal carcinoma – due to activating mutations in c-MET
oncogene
• Hereditary leiomyomatosis RCC (aka Reed Syndrome): mutations in
fumarate hydratase (FH)
• Autosomal dominant
• uterine fibroids and cutaneous leiomyomata
• Birt-Hogg-Dubé Syndrome – chromophobe/oncocytic
Birt-Hogg-Dubé Syndrome
• Chromophobe/oncocytic
histology to RCC
• Benign fibrofolliculomas
• Colonic polyps
• Medullary thyroid cancer
• Spontaneous pneumothorax
• BHD tumor suppressor gene
fibrofolliculomas
411
Autosomal Recessive Disorders
• Much rarer disorders associated with increased cancer risk,
often during childhood.
• Genes involved in either DNA repair or checkpoint response.
• Patients are often sensitive to specific DNA damaging agent
requiring modification of treatments (see supplemental table).
• Specific syndromes frequently have founder mutations in
specific ethnic groups, e.g., Bloom syndrome in the
Ashkenazi population.
Ataxia Telangiectasia
• Ataxia, telangiectasias (conjunctiva), immunodeficiency, and
lymphomas and leukemias +/- solid tumors.
• Cancer can be the first sign of the disease.
• Most patients wheelchair bound by the teen-age years.
• Cells are very sensitive to ionizing radiation.
• A-T patients with cancer need substantial dose reduction to avoid lifethreatening toxicity.
• Diagnosis - elevated AFP or increased radiation sensitivity on clonal
assay.
• Can see t(7;14) translocations in peripheral blood
Heterozygous dD mutation carriers
• The ATM gene encodes a large DNA damage checkpoint protein which
phosphorylates proteins encoded by cancer susceptibility genes.
• Mutations in ATM include truncating alleles and missense changes in
functional domains.
• Decades worth of research in 2-4xfold cancer risk associated with being
heterozygous carrier of ATM pathogenic variants: Breast, colon,
pancreatic.
• Goldgar et al., Br Cancer Treatment, 2011 estimated breast cancer risks
for heterozygous rare missense ATM mutations:
• Overall, 30% risk of BRCA by age 60 (large CI)
• One specific c.7271 C>T p.V2424G mutation in ATM is associated with a
significantly higher risk of breast cancer similar to BRCA2.
412
Fanconi Anemia
• Autosomal recessive syndrome with 19 different complementation
groups
• FA – A, B (X-linked), C, D1 (BRCA2), D2, E, F, G, I, J (BRIP1), K, L, M, N (PALB2), O
(RAD51C), P (SLX4), Q (ERCC4), R (RAD51A), T (UBE2T), V (MAD2L2), U (XRCC2),
• Associated with varying clinical features:
• Congenital anomalies
• Bone Marrow Failure resulting in pancytopenia with typical onset between 5-10
years treated by BMT.
• Significantly increased risk of malignancy (Rosenberg et al. Blood, 2003).
• By age 17, 12% FA pts had died from BMF, leukemia or solid tumor.
• By age 24, 10% had AML alone.
• Overall, solid tumors much more likely in adulthood (including head and neck
cancer and GYN malignancies).
FA – Complementation Analysis
MMC Sensitive
FA-A
MMC Sensitive
FA-B
MMC Sensitive
or
FA-?
Fused Cells are
MMC Resistant
MMC Sensitive
+
FA-?
=
Fused Cells are
MMC Sensitive
MMC Sensitive
+
FA-?
Retrovirus
expressing FANCB
FA-?
FA-A/FA-?
=
=
FA-B/FA-?
Infected FA-? Cells
are MMC Resistant
= Fanconi Type B
FA - Congenital Anomalies
• Skeletal: radial ray, hip, vertebral
• Skin hypo/hyperpigmentation
• Short stature
• Microphthalmia
• Microcephaly
• Renal: unilateral aplasia,
hypoplasia, horseshoe
• Genital: hypogenitalia
413
FA Breakage Assay
• Cells from FA patients (both
lymphoblasts and fibroblasts)
demonstrate increased
sensitivity and chromosome
breakage after exposure to
crosslinking agents:
• Diepoxybutane (DEB)
• Mitomycin C (MMC)
• Used as the gold standard
diagnostic test
Quadri-radial formations Tri-radial formations in MMC
in DEB treated PB from treated PB from FA patient
FA patient
• When in doubt, order the test,
particularly prior to decisions
about bone marrow transplant.
Images from M. Folsom, R. Naeem, Cytogenetics Laboratory, TCH
Core Fanconi Anemia Complex
FA Core Complex (in yellow) is required
to Monoubiquitinate FANCD2 and FANCI
D2
I
B E
F
A
M
G
L
C
Weak evidence that
heterozygous
mutations in core
complex genes
convey cancer risk
I
D2
Ub
P
Ub
ATM
ATM protein phosphorylates
FANCD2 after DNA damage
FA Proteins and DNA Damage Response
D2
I
B E
F
A
M
G
L
C
I
RAD51C/O BRCA2/D1
BRIP1/J
PALB2/N
D2
Ub
P
ATM
ATM phosphorylates
FANCD2 after DNA
damage
Ub
SLX4/P
DNA
MAD2L2
XRCC2
Genes that encode proteins that play
important roles in processing of DNA
damage are implicated in both FA and
hereditary breast cancer (in green)
414
Summary: Cancer risk of heterozygous carriers
for rare FA subtypes
Gene
Type
BRCA2
FA-D1 High risk breast, ovary, pancreatic…
Heterozygous cancer risk
PALB2
FA-N
Moderately high risk breast (2-4 fold increased),
pancreatic cancer (familial pancreatic cancer)
RAD51C FA-O
High grade serous epithelial ovarian cancer; GWAS
hit for testicular germ cell cancer; Breast cancer
risk unclear
BRIP1
FA-J
Twofold increased risk of breast cancer
SLX4
FA-P
No clear breast cancer risk from several large
studies
XRCC2
FA-U
RAD51 paralog; LOF mutations identified in breast
cancer families and one FA-like patient cell line
RecQ Helicase Disorders
Disease
Clinical Features
Cancer
Predisposition
Gene/Chromosome
location
Hereditary Breast
Cancer (AD)
Autosomal dominant
non-syndromic
Breast cancer
(2015)
RECQL 12p12
Bloom (AR)
Small stature,
photosensitive rash,
immunodeficiency
Multiple tumor
types including
leukemia/lympho
ma and solid
tumors
BLM 15q26.1
Werner (AR)
Premature ageing,
cataracts, diabetes,
atherosclerosis
Soft tissue
sarcomas and
skin cancers
WRN 8p11
Rothmund Thomson
(AR)
Poikiloderma rash,
sparse hair, radial ray
defects, cataracts
Osteosarcoma
and skin cancers
RECQL4 8q24.3 and
unidentified gene(s).
Osteosarcoma
associated with
RECQL4 mutation
RAPIDILINO (AR)
Baller-Gerold (AR)
Poikiloderma in RTS
• Children born with normal skin (may have
absent hair).
• Acute phase starts on the cheeks during
infancy and spreads to extremities
(spares the thorax and buttocks).
• The rash persists as stable poikiloderma.
• Seen in both Type 1 and Type 2 RTS.
C.
• Type 2 has extremely high risk of
osteosarcoma although response to
treatment similar to tumors in general
population.
• Specific RECQL4 variants associated with
lymphoma
415
Phenotypic tests for hereditary cancer syndromes
Syndrome
Gene
Phenotypic test
Dyskeratosis
congenita
Multiple
Telomere length by FISH/flow cytometry
Bloom
BLM
Sister chromatic exchange
Fanconi
Multiple
DEB and MMC breakage study
Ataxia Telangiectasis ATM, NBN,
(and Nijmegen)
LIG4
Sensitivity of lymphoblastoid cell lines to
ionizing radiation
Mosaic Variegated
Aneuploidy
BUB1B &
others
Random trisomies in peripheral blood
XRCC2
FA-U
RAD51 paralog; LOF mutations identified
in breast cancer families and one FA-like
patient cell line
416
Biochemical Genetics II
BIOCHEMICAL GENETICS II
Gerard Berry, MD, FACMG
Harvey Levy Chair in Metabolism Director, Metabolism Program
Division of Genetics and Genomics Boston Children’s Hospital
Professor of Pediatrics, Harvard Medical School
Gerard Berry, MD, FACMG
Harvard Medical School
Center for Life Science
Building, Suite 14070
3 Blackfan Circle
Boston, MA 02115
(617) 355-4316 Telephone
(617) 730-4874 Fax
[email protected]
419
420
Biochemical Genetics II
Gerard T. Berry, MD, FACMG
Founding Fellow ACMG
Division of Genetics and Genomics
Boston Children’s Hospital
Harvard Medical School
Director of Harvard Medical School BG training program
ACMG Genetics and Genomics Review Course
June
20-23, 2013
Nothing to disclose
Disorders of Organelles & Large Molecules
භ Mitochondria
Cilia
භ Trafficking defects
Ly
භ Lysosomes
භ Peroxisomes
Px
Nucleus
ER
421
Organelle Targeting Signals
භ Mitochondria
භ Presequence - N-terminal amphipathic helix
භ Nucleus
භ Internal basic AA di-peptide
භ ER
භ N-terminal hydrophobic peptide; binds signal
recognition particle (SRP) with co-translational
import followed by cleavage
භ Lysosome
භ Mannose-6-P (M6P) added post-translationally
භ Peroxisome
භ PTS1 -- C-terminal -SKL
භ PTS2 -- near N-terminal -RLX5H/QLභ PTS3 - ?
Mitochondrial Disease
භ~1/5000-8000 overall incidence
භNuclear and mt DNA mutations
භ>300 nuclear genes involved mt function
භ37 mt genes – some resp chain proteins (for all
but Complex II), 2 rRNAs, 22 tRNAs
භMaternal, cytoplasmic inheritance (for mt DNA)
භHeteroplasmy – mitochondria
form a population within cells;
threshold effect
භPhenotypes may vary with age
භAffect tissues with high energy demands
Symptoms Suggesting a Mt Disorder
භCNS - hypotonia, ataxia, IDD, seizures, migraines,
dementia, sensorineural hearing loss
භEyes – retinitis pigmentosa (RP), optic atrophy,
nystagamus, ophthalmoplegia
භMuscle - weakness, exercise intolerance, red
ragged fibers
භCardiac - hypertrophic cardiomyopathy,
arrhythmias, heart block
භHematologic - macrocytic anemia, pancytopenia
422
Symptoms Suggesting a Mt Disorder
භEndocrine - diabetes mellitus, diabetes
insipidus, exocrine pancreatic dysfunction,
short stature
භGI - dysfx, intestinal pseudo-obstruction
භLiver - dysfx, failure
භRenal - RTA, Fanconi syndrome
භMany pts, particularly infants, do not present
with classic phenotypes; consider in differential
if pt has 2 or more suggestive findings
Genetic Defects in Mt Function
භ Defects in nuclear genes
භSingle function deficiencies
භMt biogenesis, complex assembly, and
replication defects
භ
භ
Mt DNA point mutations
Mt DNA deletions / insertions
Many Mt Disorders Caused by Mutations
in the Nuclear Genome
භ Defects in OxPhos
භ ~ 80-90% of the patients have nuclear encoded defects
භAll complexes of the e- transport chain have subunits
encoded in nuclear genome
භ Typically AR inheritance
භ Respiratory chain assembly factors
භ ACAD9 (Complex I) SURF1 (Complex IV), SCO1&2 (Cu++
homeostasis, complex assembly)
භ MNGIE – AR defect in TYMP gene
භ(Mt neurogastrointestinal encephalomyopathy)
භ CoQ synthesis defects - 5 genes
423
Nuclear Encoded Mitochondrial Genes Recently
Identified by Exome Sequencing
from Taylor et al., JAMA, 312: 68-77, 2014 (Fig. 2)
Leigh Syndrome
භ Sx - onset late infancy with regression; MRI
abnl with
white matter and basal ganglia changes; +/- increased
serum lactate
භ Heterogeneous with nuclear & mt mutations
භ ~50% SURF1 (involved assembly Cyt Oxidase,
Complex IV)
භ PDH mutations
භ Complex I, II, IV deficiency
භ NARP (neuropathy with ataxia & RP) mt point
mutations
භ mt DNA depletion
Mitochondrial Depletion Syndromes
භAR - ratio of mt/nuclear DNA
භGe - heterogeneous and caused by
defects in nuclear genes involved
in mt DNA replication (POLG1, TK2,
DGUOK, TWINKLE, others)
භSx - hepatic failure,
glu, CNS
see Copeland, Ann. Rev. Med. 59: 131-46, 2008
424
Some Disorders Caused by Mt Point
Mutations
භ LHON - adult onset optic neuropathy; most
homoplasmic missense muts
භ NARP - Neuropathy, ataxia & RP; most
patients with heteroplasmic missense muts
in ATP synthase (Complex V)
භ Maternally inherited deafness - point
muts in mt rRNA; also associated with
susceptibility to aminoglycoside ototoxicity
භ Several others
Some Disorders Caused by Mt Point Mutations
භ Mt DNA tRNA mutations
භMERRF
- heteroplasmic point muts in mt
lys
tRNA (~80%)
භMELAS - heteroplasmic point muts in tRNAleu
(most)
භ Mechanism not known; ?Sx related to inability
to translate several mt proteins and lack of nl
processing of transcripts
MERFF = myoclonic epilepsy with red ragged fibers
MELAS = myoclonic epilepsy with lactic acidosis and stroke
MELAS: Myoclonic Epilepsy, Lactic Acidosis & Stroke
භEpisodes of metabolic
decompensation ass’d with
high stroke risk
භAcute Rx – arginine
භ3243 A>G tRNALeu (~80%);
3271 T>C tRNALeu (~7%)
භHeteroplasmic
425
Some Disorders Caused by Mt DNA Deletions
&/or Duplications
භDiabetes and deafness
භPearson syndrome –anemia
2° marrow failure; lactic
acidosis; exocrine
pancreatic failure; RTA
භCPEO – chronic progressive
external ophthalmoplegia
භKearns-Sayre – PEO, cardiac conduction block,
RP, ataxia, lactic acidosis, ataxia; sporadic
Laboratory Diagnosis of Mt Disorders
භ Lactate, pyruvate (peripheral, CNS),
ratio;
alanine
භ MRI of brain
භ Consider muscle and/or liver bx (most
involved tissue)
භ OxPhos analysis including enzyme assays
භ DNA analysis for specific mtDNA/nuclear
mutations; mt DNA panels; exome sequencing
3 Parent Embryos to Treat Mt-Encoded Disease
from Science, 343: 827, 2014
426
Congenital Disorders of Glycosylation
(CDG)
භ Glycosylation pathways very complex
භ~2% genes encode proteins involved in glycosylation
භ>100 human disorders identified to date; most very
rare
භ Clinical spectrum very broad: CNS, eye, skeletal, skin,
clotting, immune , endocrine, GI, liver, & more
භ New disorder nomenclature: gene symbol-CDG
භCDG1a becomes PMM2-CDG
භCDG1b becomes MPI-CDG
Classes of Glycosylation Disorders
භN-glycosylation defects (ш28 human disorders)
භ Amide linkage to asparagine
භ N-glycan assembly ER or cytosol; sugars transferred en bloc
from dolichol; processing in ER or Golgi
භ~50% of all known proteins have ш1 N-gly site
භ O-glycosylation defects (ш34)
භLinkage through –OH on serine or threonine; transfer single
sugars onto growing glycan backbone
භIncludes ABO blood groups, Exostoses I & 2 proteins;
proteoglycans (with skeletal & connective tissue sx), some
congenital muscular dystrophies (POMT1&2, Fukutin, etc)
Classes of Glycosylation Disorders
භ Combined N- and O-glycosylation defects (ш7)
භ Lipid glycosylation defects (ш3)
භ GPI-anchor defects (ш12)
භ Trafficking defects of Golgi COG complex proteins
භ Defects of dolichol synthesis or recycling (ш5)
Farnesyl-PP
Dolichol-PP
(ш13)
N-gly
ER
(3 steps) Dolichol-P
Cholesterol
O-gly, others
Modified from Wolfe et al., AJMG, 160C: 322-8, 2012
427
Summary of Types of Glycosylation
PROTEOGLYCANS
GLYCOSPHINGOLIPIDS
GLYCOPROTEINS
from Freeze et al., Inborn Errors of Metabolism, 2015, Fig. 2.1
Classical PMM2-CDG (Ia) Clinical Features
භ AR; phosphomannomutase 2 def
(most common CDG, 60-70%)
භ Multisystem disorder
භhypotonia, IDD, szs, ataxia
(cerebellar hypoplasia)
භRP, strabismus
භliver disease, coagulopathy
භFTT, inverted nipples, lipodystrophy
භ Some die <1 yo; others survive to
adulthood; range of cognitive skills
භ Abnl transferrin isoelectric focusing
Other CDGs
භ MPI-CDG (Ib) - Mannose-6-phosphate isomerase def
භHepatic/GI sx with vomiting, GI bleeding, protein-losing
enteropathy, liver disease, coagulopathy, hepatic fibrosis
භMinimal neurologic involvement
භOnly form with rx – oral mannose
භ SRD5A3-CDG (Iq) – steroid 5-ɲ-3 reductase def (dolichol
synthesis defect)
භ1st sx between 6 mo – 12 yrs
භSevere IDD, ataxia, cerebellar hypoplasia
භProminent eye anomalies – coloboma, optic atrophy,
cataracts, glaucoma, micro-ophthalmia
භHeart defects, liver dysfx, ichthyosis
428
CDG Diagnosis
භ Transferrin (Tf) isoelectric focusing – Dx for Nlinked disorders only
භ Various mass spec techniques of purified serum
proteins (Tf, apoCIII, others) and urine – N- and Olinked disorders
භ False positives – young infants (<30 days),
galactosemia, HFI, recent EtOH use, liver disease,
hemolytic uremic syndrome, Tf protein polymorphisms (sugars don’t bind)
CDG Diagnosis: Transferrin Isoelectric Focusing
See Jaeken & Matthijs Ann Rev Gen & Hu Genet, 2001
Lysosomal Storage Diseases
භLysosomes - cytoplasmic organelles that contain ~50
acidic degradative enzymes
භAlso include membrane proteins
භTransporters (cys-cys, sialic acid)
භSAP activator proteins
භDeficiency results in accumulation of macromolecules
usually degraded by that enzyme/protein
භStored material may cause enlargement of organs and may
be visualized in membrane bound vesicles by EM
429
Lysosomal Storage Diseases (LSDs)
භTarget organs affected by each disease are determined
by normal sites of degradation of each compound
භAll are recessive, most are autosomal
භMost patients are normal at birth; as material
accumulates there is a plateau and then regression
භMost disorders are progressive and often fatal
භMany have classic infantile (occ prenatal) as well as
later onset, milder forms
Classification of LSDs
භ Mucopolysaccharidoses (Hurler, Hunter)
භ Sphingolipidoses (Tay-Sachs, Gaucher)
භ Transport disorders (Cystinosis)
භ Mucolipidoses – trafficking (I-Cell)
භ Glycoprotein (Mannosidosis)
භ Neutral Lipid (Wolman)
භ Glycogen Storage (Pompe)
LSDs : Some General Phenotypic Features
භCoarse facies
භOrganomegaly (liver, spleen)
භEye abnormalities
භCorneal clouding
භCherry red spot
භOptic atrophy
භPigmentary retinopathy
භSkeletal abnormalities
භNon-immune hydrops
Cherry red macula
Dysostosis multiplex
430
LSDs: General Diagnostic Approach
භ
භ
භ
භ
භ
භ
භ
භ
භ
Serum lysosomal enzymes
Blood smear
Radiologic exam
Ophthalmologic exam - fundoscopic & slit lamp
Urine mucopolysaccharides and glycoproteins
Consider bone marrow
Biochemical studies of fibroblast +/- leukocytes
Molecular/gene sequencing
Other -- depending on specific disorder
Mucopolysaccharidoses
භ Usually normal at birth
භ Gradual slowing of development, regression
භ Coarse facies
භ +/- Corneal clouding
භ Macrocephaly, IDD (intellectual/developmental disability)
භ Skeletal involvement ( ROM, claw hand)
භ Dysostosis multiplex on x-ray
භ Otitis and hearing (sensorineural & conductive)
භ Recurrent herniae, thickened mucous
භ Late cardiac involvement
Hurler Syndrome (MPS I)
භPrototypic MPS disorder
භAR; incidence ~1/100,000
භSx - onset 6-12 months; death by 5-10 yrs; milder
variant w/o CNS (Scheie IS)
භDx - +MPS spot test; enzyme assay; DNA
භRx - HSCT transplantation with matched donor slows
disease if performed early (including CNS; no effect
on skeletal sx, corneal clouding); ERT – improved
somatic sx, no effect on CNS; protocols now with ERT
before HSCT
431
Hurler Syndrome
Dysostosis Multiplex
Hunter Syndrome (MPS II)
භSx - like Hurler but usually no corneal
clouding; prominent deafness
භXL; ~1/70-150,000; 20% of pts with
gene deleted have more severe ID
භDx: +MPS spot; enzyme assay; carrier
females best diagnosed by DNA
භ Rx: ERT gives improved somatic function in pts pts with
mild disease (reduces viscero-megaly & GAG excretion,
improves joint mobility, preserves linear growth); no
effect on CNS; efficacy of HSCT not proven
432
Sanfilippo Syndrome (MPS III)
භ4 distinct loci (all AR); A and B most common
භSx - more CNS, less somatic features; onset usually
2-4 yrs; chronic diarrhea, insomnia, szs, aggression
prominent
භDx – MPS may be + or -; enzyme assay/DNA
භRx - none; no benefit from HSCT; some enzyme
replacement to CNS and gene therapy clinical trials
Other MPS Disorders
භMorquio (Type IVA & B)
භShort-trunk dwarfism with nl IQ; severe odontoid
hypoplasia; ERT (elosulfase alfa)
භMaroteaux-Lamy (Type VI)
භSomatic sx may be as severe as Hurler; usually nl IQ
භDefect in Arylsulfatase B; ERT (Naglazyme)
භSly (MPS VII)
භSevere infantile form like Hurler; prenatal form with
hydrops/fetal ascites
භDefect in β-glucuronidase
Other Features of MPS Disorders
භHydrocephalus
භObstructive airway disease; difficulty with
intubation; excessive secretions
භAtlantoaxial instability; odontoid hypoplasia
භCardiac disease - valvular, conduction
disturbances, EFE, occ cardiomyopathy
භPulmonary and systemic hypertension
433
Gaucher Disease
භMost common lysosomal storage disease
භ3 types based on clinical symptoms
භType I - nonneuronopathic; splenomegaly,
pancytopenia, bone pain/lytic bone lesions;
1:400-1:1000 US Ashkenazi Jews
භType II - acute neuronopathic - rapidly progressive
neurologic disease with hepato-splenomegaly;
all ethnic groups
භType III - subacute
neuronopathic - later onset
Gaucher Disease
භDx - “foam cells” in bone marrow, smear; enzyme
assay; molecular for carrier screening in Jewish
population
භGene (GBA; glucosidase-ɴ acid) on chr 1; nearby
pseudogene; certain alleles appear protective
(N370S) against CNS disease; L444P usually Type II or
III
භRx - symptomatic; splenectomy; ERT for Type I pts
(no effect Type II); newer substrate reduction therapy
(miglustat; D-glu analog)
Tay-Sachs Disease (TSD), GM2 Gangliosidosis
භAR; incidence ~1/100,000; ~1/4000 in Ashkenazi
Jewish pop; also increased in French Canadians
භSx classic infantile - onset 6-12 mos; loss of
milestones, hyperacusis, apathy; cherry red spot;
later onset of szs, blindness, spasticity; death by age
2-5
භMilder juvenile & adult
forms
434
Cherry Red Spot
භTay-Sachs Disease
භSandhoff Disease
භSialidase deficiency
භNiemann-Pick Disease Type A
භGM1 Gangliosidosis
Tay-Sachs Disease
භMolecular defect of Hexoseaminidase A;
defects in Hex B cause Sandhoff Disease
(Sandhoff may have some somatic
features + CNS)
භDx - enzyme assay; DNA
භRx - none
භPrevention – heterozygote/carrier
screening (recommended enzyme + DNA,
even in Ashkenazi Jewish pop)
Fabry Disease
භX-linked; ~1/40-60,000 males; CNS spared
භSx (males) – median age of onset 9 yrs;
peripheral neuropathy; acroparesthesias;
angiokeratomas; lens/corneal opacities; late
renal and cardiovascular disease; chr lung
disease with fibrosis
භAccounts for ~1% chr renal failure & 5%
cryptogenic stroke;
incidence cardiac
variant ~1/3500
435
Fabry Disease
භMost females have sx – median age onset 13 yrs; fatigue,
stroke & ~10% develop renal failure; can be detected by
slit lamp
භDx - enzyme assay males (may miss females);
heterogeneous mutations; DNA best for females
භRx – dilantin/tegretol for neuropathy; renal transplant;
ERT may decrease pain, GI sx, slow renal disease; does
not proteinuria
භAdult male – start ERT at time of dx
භPediatric male and all females – start rx at 1st sign of sx
Krabbe Disease, Globoid Cell Leukodystrophy
භ Sx infantile - onset <6 mos; hypotonia, irritability;
optic atrophy; occ. macrocephaly; elev. CSF
protein; leukodystrophy on MRI
භ Dx - enzyme assay/molecular (GALC gene);
pseudo-deficiency can complicate prenatal dx &
requires sulfatide loading assay
භ Rx - HSCT has some efficacy in later onset cases or
if performed very early in infantile cases
GALC = galactosylceramidase
Lysosomal Processing Defects
භ I-Cell Disease (Mucolipidosis II)
භ Multiple sulfatase deficiency
(Appendix)
436
I-Cell Disease
I-Cell Disease
භAR defect in targeting enzymes to lysosome via
mannose-6-P (2 step Golgi rxn; defect 1st step);
MLIII is allelic, milder variant
භSx – like severe Hurler; may see neonatal or
prenatal onset
භDx - plasma activity multiple lysosomal
enzymes with deficient activity in fibroblasts; MPS spot
භRx - None
Lysosomal Transport Disorders
භDefective transport out of lysosomes of
products of lysosomal degradation
භExamples
භCystinosis
භSialic acid storage disease (infantile and adult
forms)
භNiemann-Pick Type C (NPC) Disease
437
Niemann-Pick Disease, Type C
භAR; ~1/150,000
භSx – like NP A, B; infantile form with neonatal jaundice;
later onset forms with ataxia and progressive dementia,
psychosis; vertical ophthalmoplegia
භDx - nl sphingomyelinase; abn. lysosomal accumulation of
unesterified cholesterol
භ2 genes identified, NPC1 (95%), NPC2 (~5%);
role in intracellular cholesterol trafficking
භRx – supportive; clinical trials with drugs to increase chol
removal from cells
Enzyme Replacement Therapy (ERT) for LSDs
භNeed to reach cells with defect
භGaucher – macrophages
භFabry – endothelial cells
භTay-Sachs - neurons
භThreshold for effectiveness may differ with each
disorder & site (bone resistant in MPS)
භMay be most effective early in course of disease (MPS,
neuronal involvement)
භActs as foreign protein in CRM- patients
භCost and availability
ERT for LSDs
භ FDA approved
භ Cerezyme, Vpriv – Gaucher (I,III)
භ Fabrazyme, Replagal – Fabry
භ Myozyme - Pompe
භ Aldurazyme – MPS I (Hurler)
භ Elaprase - MPS II (Hunter)
භ Naglazyme – MPSVI (Maroteaux-Lamy)
භ Elosulfase alfa MPSIVA (Morquio)
භ Synageva – Lysosomal acid lipase def (Wolman)
භ Ongoing clinical trials
භ Sanfilippo IIIA, MLD
භ www.clinicaltrials.gov
438
ERT for Type I Gaucher Disease
Other Therapies for LSDs
භSubstrate reduction
භZavesca (miglustat; Gaucher & others) iminosugar
analog of glucose; crosses BBB
භNeed residual enzyme or alternate pathway
භEnzyme enhancement – chaperone therapy
භRequire residual activity, stabilize mutant protein
භNot specific to a single enzyme, ?mutation specific
භCross BBB
භCan be complementary & synergistic to ERT
Genetic Counseling for LSDs
භDx usually relies on enzyme assay with molecular
testing for confirmation, carriers &/or prenatal
භReliability of biochemical carrier testing variable
(excellent for TSD; poor for many)
භPseudodeficiency can complicate prenatal dx
(MLD, Krabbe)
439
Features of Peroxisomes (Px)
M
භUbiquitous (except RBCs)
භBound by single membrane
භContain no nucleic acid;
proteins encoded by
nuclear genes, translated
on free cytoplasmic
ribosomes & imported into
peroxisome membrane or matrix
භSeveral hundred/cell; ~70 enzymes in matrix
භPerform a variety of cellular anabolic and catabolic
functions
ER
Medically Relevant Peroxisomal Functions
භDegradation of very long chain fatty acids
භEarly steps of plasmalogen biosynthesis
භDegradation of phytanic acid
භSelected steps in cholesterol biosynthesis
භDegradation of pipecolic acid, synthesis of bile
acid intermediates, glyoxylate metabolism
Peroxisome Biogenesis
භDivide by fission to form new organelles
භBiogenesis and protein import co-ordinated by
16 PEX proteins
භImport of matrix proteins uses at least 3 targeting
signals
භPTS1 - C-terminal SKL - most common
භPTS2 - N-terminal sequence
භSome proteins use neither of the known signals
භTargeting of peroxisomal membrane proteins not
completely understood
440
Classification of Peroxisomal Disorders
(PSDs)
භPeroxisome biogenesis disorders (PBDs) multiple deficiencies
භ Zellweger syndrome spectrum
භRhizomelic chondrodysplasia punctata
(RCDP)
භSingle enzyme defects
භ X-linked adrenoleukodystrophy
භRefsum disease
Zellweger Syndrome Spectrum
Zellweger
syndrome
Neonatal ALD
Infantile
Refsum Disease
භ Combined developmental & metabolic disorders
භ Overall ~1/50,000 incidence
භ Genetically heterogeneous - 12 complementation
grps
භ 50% defect in PEX1 gene encoding PTS1 receptor
Zellweger Syndrome
භPrototypic peroxisomal biogenesis disorder
භSx - dysmorphic facies, hypotonia, seizures,
IDD, neuronal heterotopias, cataracts &/or
glaucoma, renal cysts, 50% with epiphyseal
calcifications (CDP)
භEarly death, usually by 6-12 months
භVariants present a bit later, usually w/o
dysmorphisms/malformations; survive longer
භDx - all peroxisomal functions abnormal; no
peroxisomes or ghost membranes seen by
EM; biochemical, then molecular
441
Rhizomelic Chondrodysplasia Punctata
භ ~1/100,000
භ Skeletal dysplasia, cataracts, ichthyotic skin rash, IDD; occ CHD,
cleft palate
භ Mutations in PEX7, encoding the PTS2 receptor is most
common cause
භ Dx – low plasmalogens; elevated phytanic acid
භ Phenotype of single enzyme defects in plasmalogen synthesis
result in similar phenotype (no phytanic acid)
X-linked Adrenoleukodystrophy
භ Progressive X-linked neurodegeneration associated with adrenal
involvement
භ ~1/20,000 prevalence
භ Highly variable clinical phenotype
භ Childhood cerebral - childhood onset, rapid progression
භ Adrenomyeloneuropathy (AMN) - onset 20’s - 30’s with spastic
paraparesis
භ Adrenal only
භ ~50% het female carriers develop mild neurological sx in
adulthood
භ 20% gait and spinal cord involvement like AMN
X-linked Adrenoleukodystrophy
භResponsible gene, ABCD1, encodes an ABC
transporter in the peroxisome membrane
භNo genotype / phenotype correlation VLCFA
භSome males with sx & others
asymptomatic in same family
Px
භDefective metabolism of VLCFA
භDx – VLCFA; molecular (esp. carrier females)
භRx – HSCT early in course for males as soon
as MRI changes noted; corticosteroids
for adrenal insufficiency
442
X-ALD: Childhood Cerebral Form
Age 2
Age 5
Cerebral X-ALD
T1 MRI with gadolinium
Inflammatory demyelination with
perivascular infiltration
Diagnostic Abnormalities in the Px Disorders
භ PBDs - Zellweger spectrum
භ VLCFA
භ RBC plasmalogens
භ plasma pipecolic acid
භ PBDs - RCDP
භ RBC plasmalogens
භ VLCFA are normal
භ Other
භ phytanic acid (requires dietary intake)
භ bile acid intermediates
භ Dicarboxylic aciduria by urine organic acids
භ Molecular screening
* Steinberg et al, Mol Genet Metab 83: 252, 2004
443
Refsum Disease
භSx - Cerebellar ataxia,
polyneuropathy & RP;
elevated CSF protein
භGe - AR deficiency of
phytanoyl-CoA hydroxylase
භDx - Phytanic acid; molecular
භRx - Phytanic acid-restricted diet
Clinical Features Suggesting a PX Disorder
භ Failure to thrive, developmental delay
භ Hypotonia, cerebral atrophy, decreased myelination,
neuronal heterotopias
භ Dysmorphia similar to Zellweger
භ Cataracts, glaucoma, retinitis pigmentosa
භ Chondrodysplasia punctata
භ Hepatomegaly, renal cysts
Thank you and good luck!
444
• Online Metabolic and Molecular Basis of Disease
• Blau et al., Physician’s Guide to Laboratory Diagnosis of
Metabolic Disease
• Nyhan et al., Atlas of Metabolic Disease
• NAMA slide sets from SIMD
• Lee and Scaglia, eds., Inborn Errors of Metabolism (2014)
ACMG Genetics and Genomics Review Course
June
20-23, 2013
APPENDICES
Nuclear Membrane: The Laminopathies
භ Lamins - multifunctional filamentous proteins of the nuclear
lamina, just under the inner nuclear membrane
භ Three genes - LMNA, LMNB2, LMNB1
භ 13 known disorders, including 11 discrete phenotypes caused by
LMNA mutations including:
භ Hutchinson-Gilford progeria
භ Emery-Dreifuss muscular dystrophy
භ Mandibuloacral dysplasia
භ Generalized lipodystrophy
භ Restictive dermopathy
445
Niemann-Pick Disease,Types A and B
භAR - deficiency of acid sphingomyelinase;
increased in Ashkenazi Jews (carrier freq ~1:60)
භSx - neurodegenerative with spleen > liver;
cherry red spot (~50% type A); pulmonary (Type
B)
භDx - enzyme assay; molecular; sea blue
histiocytes in marrow
භRx - none
SMPD1 gene = sphingomyelinase phosphodiesterase 1
Protein = ASM, acid sphingomyelinase
GM1 Gangliosidosis
භSx – somatic + CNS affected; hypotonia, szs, MR;
½ pts have cherry red spot
භMilder juvenile and adult forms exist
භDx – foamy histiocytes in bone marrow; enzyme
assay of ɴ-galactosidase; gene (GLB1=
galactosidase, beta 1)
භVariants
භ Morquio Type B pts with residual GM1 activity,
skeletal sx, CNS spared
භCombined galactosialidosis with protector
protein deficiency
Tay-Sachs Disease (TSD)
TSD = deficiency of Hex A, a+b dimer
Sandhoff = deficiency of Hex B – b+b dimer
Hex A is heat labile, Hex B is not. Std serum screening
assay measures total Hex A+B, then heat inactivates
Hex A (or uses low pH) and measures residual Hex B.
Then Hex A = Total – Hex B. Molecular testing used to
distinguish rare pseudo-deficiency alleles in healthy
persons with low HexA
Pregnancy, oral contraceptives and some illnesses
can make serum screening test inconclusive.
446
Metachromatic Leukodystrophy
භSx late infantile - most pts walk; regression
before age 2; white matter changes; elev. CSF
protein
භMilder variants
භAR defect in Arylsulfatse A (gene is ARSA)
භDx - enzyme assay; pseudodeficiency
භGene cloned and 2 single bp changes associated
with pseudodeficiency
Glycoprotein Disorders
භIncludes mannosidosis, aspartylglycosaminuria
(AGU), sialidosis, fucosidosis
භSx – like mild/mod MPS; fucosidosis has false +
sweat test & angiokeratomas; congenital form of
sialidosis with fetal ascites
භDx – MPS spot neg; enzyme assay; characteristic
urine oligosaccharides
භAGU common in Finland – C163S missense ~98%
Finnish alleles (founder effect)
Multiple Sulfatase Deficiency
භSx – like severe MPS+ichthyosis, mild skeletal
භAR deficiency of Formylglycine enzyme - catalyzes
posttranslational modification of conserved cys in
all sulfatases; very rare
භFeatures of 8 monogenic disorders
භ5 MPS - II, IIIA, IIID, IVA, VI
භX-linked ichthyosis (steroid sulfatase)
භMetachromatic leukodystrophy (Aryl A)
භXL rcessive CDP (ARSE)
භDx - +urine MPS, enzyme assay(s); molecular
(Gene is SUMF1, sulfatase modifying factor 1)
447
Selected Other LSDs
භFarber lipogranulomatosis (ceramidase def) – very rare;
painful, deformed joints + subcutaneous nodules; IDD
භAcid lipase def (Wolman disease) – GI disorder with
hepatosplenomegaly, FTT, adrenal Ca++; milder variant is
cholesterol ester storage disease
භSchindler disease – progressive IDD, blindness, szs,
hypotonia; rare cause of neuraxonal dystrophy due to
deficiency of lysosomal N-acetylgalactosaminidase
භPycnodysostosis –skeletal dysplasia; defect in lysosomal
cathepsin K
LSD: Sphingolipid Activator Protein (SAPs)
භSmall proteins that interact with some lys
hydrolases to stabilize them or stimulate activity
භ2 SAP Genes
භ5q GM2 activator (Tay-Sachs, Sandhoff); gene is
GM2A, GM2 ganglioside activator
භChr 10 – One gene encodes SAPs A-D; processed
to individual proteins (gene is PSAP, prosaposin)
භSx – SAP B resembles juvenile MLD; SAP C juvenile
Gaucher; All are AR
Other Single Function Px Disorders *
භ Total at least 13 disorders
භ In addition to XALD & Refsum includes:
භ Px β-oxidation disorders - most resemble ZS
භ Ether phospholipid synthetic defects resemble RCDP
භ Mevalonate kinase - classic & Hyper IgD/
periodic fever
භ Catalase deficiency - oral ulcers
භ Glyoxylate detoxification - Hyperoxaluria type I
* Wanders et al, MMBID edt 8, 2001; Wanders & Waterham BBA 1763: 1707, 2006
448
Systems-Based Disorders I
SYSTEMS-BASED
DISORDERS I
Bruce R. Korf, MD, PhD, FACMG
Wayne H. and Sara Crews Finley Chair in Medical Genetics
Professor and Chair, Department of Genetics
Director, Heflin Center for Genomic Sciences
University of Alabama at Birmingham
Bruce R. Korf, MD, PhD, FACMG
Department of Genetics
University of Alabama at Birmingham
1720 2nd Ave. S., Kaul 230,
Birmingham, AL 35294-0024
(205) 934-9411 Telephone
(205) 934-9488 Fax
[email protected]
451
452
Systems Based Disorders I
Bruce R. Korf, MD, PhD
Professor and Chair, Department of Genetics
University of Alabama at Birmingham
Disclosure(s)
Relationship
Entity
Grant Recipient
Novartis
Advisory Board
Accolade, Genome Medical
Board of Directors
American College of Medical Genetics and Genomics
Children’s Tumor Foundation
Advisor
Neurofibromatosis Therapeutic Acceleration Project
Founding Member
Envision Genomics
Salary
University of Alabama at Birmingham
Outline
• Skin
• GI
• Sensory
• Psychiatry
• Cardiovascular
• Epigenetic
453
Pigmentation
Skin
Eyes
Hair
Other
Gene(s),
(Inheritance)
Hearing loss
Dystopia canthorum
Melanocyte Development
Waardenburg Syndrome
Congenital leukodermia
Heterochromia iridis; choroidal
hypopigmentation
White forelock
Piebaldism
Patchy hypopigmentation
Heterochromia iridis
White forelock
KIT, SNAI2 (AD)
PAX3 (AD)
Melanogenesis
OCA1
White
Decreased acuity
White
TYR (AR)
OCA2
White
Decreased acuity
White/Yellow
OCA2 – tyrosine
transporter (AR)
OCA3
Copper-red, freckles
Nystagmus, strabismus
Copper-red
TRYP1 (AR)
OCA4
White
Decreased acuity
White
OCA5
White
Decreased acuity
Yellow
?
OA1
Normal
Decreased acuity
Normal
GPR143 (XLR)
SLC45A2 (AR)
Melanocyte Biogenesis/Transport
Hermansky-Pudlak
Creamy white
Decreased acuity
White to brown
Bleeding, cardiomyopathy, pulmonary
fibrosis, colitis, renal failure, absent dense
bodies in platelets
HPS1 (AR) prevalent
in Puerto Rico; other
genes
Chediak-Higashi
Hypopigmentation
Decreased acuity
Hypopigmentation
Hepatosplenomegaly, neuropathy,
Intellectual disability, anemia,
thrombocytopenia, infections
LYST (AR)
Griscelli
Hypopigmentation
Poor fixation
Silver-gray
Developmental delay
MYO5A, RAB27A,
MLPH
Ectodermal Dysplasia
Hypohidrotic
Ectodermal
Dysplasia
Skin
Teeth
Hair
Nails
Other
Gene(s)
(Inheritance)
Dry, lack of
sweat pores,
hypohidrosis
Adontia/hypodontia
Fine, brittle, sparse
Spooned, brittle
Depressed nasal
bridge, respiratory
problems
EDA (XLR)
EDAR, EDARADD
(AD/AR)
Odonto-OnychoDermal Dysplasia
Hyperkeratosis,
erythema,
hyperhidrosis
Hypodontia
Absent, dry, thin
Dystrophic
Smooth tongue
WNT10A (AR)
P63 Related
Hyperkeratosis
Adontia/Hypodontia
Sparse
Dystrophic
Various syndromes:
EEC, Hay-Wells,
Rapp-Hodgkin, etc.
P63 (AD)
Clouston
Hyperkeratosis,
hyperpigmentati
on
Normal
Fine, brittle
Dystrophic
Short stature, eye
anomalies
Witkop
Normal
Small primary,
partial/total absence
permanent
Normal
Thin, friable
GJB6 (AD)
MSX1 (AD)
Incontinentia Pigmenti
• Clinical
• Streaky hyperpigmention (erythema,
vesicular verrucous phases)
• Abnormal teeth and hair
• Neovascularization of retina
• Neurological problems (seizures,
developmental delay)
• Genetics
• XLD male lethal
• IKBKG gene exons
• 65% deletion exons 4-10
454
Ichthyosis
• Ichthyosis vulgaris – most common –
semidominant – filaggrin
• X-linked – steroid sulfatase
• AR congenital ichthyosis – collodion
membrane
• Severe form – harlequin ichthyosis
• Other forms – erythroderma and lamellar
ichthyosis
• Multiple genes – TGM1 50-60%; 90%
lamellar; ABCA12 >93% harlequin
Epidermolysis Bullosum
• Blistering of skin
• EB Simplex – splitting in or above basal layer
• Types
•
•
•
•
Localized
Generalized Intermediate
Mottled Pigmentation
Generalized Severe
• Genes: Genes: EXPH5, KRT5, KRT14, TGM5
• Junctional EB – AR – within basement membrane
• LAMB3 (70%), COL17A1, LAMC2, LAMA3
• Dystrophic EB - scarring below basement
membrane
• AD or AR – COL7A1
• Kindler syndrome – multiple cleavage planes
• kindlin-1 (FERMT1)
Alpha-1-Antitrypsin
• Clinical
• Pulmonary emphysema (heterozygotes and homozygotes
• Hepatic cirrhosis (homozygotes, specific alleles)
• Panniculitis
• Pi type - electrophoretic mobility
• inhibitor of neutrophil elastase
• M allele in 95% Caucasians
• SERPINA1 alleles with decreased production or function
• null mutations only associated with emphysema
• Z allele: glu to lys at codon 342
• 10-15% Pi activity
• impaired release from hepatocytes (liver toxicity)
• Treatment
• No smoking
• Antioxidants (vitamin E)
• Transplantation (lung, liver)
• Alpha-1-AT intravenous augmentation
By Laura Fregonese, Jan Stolk [CC-BY-2.0
(www.creativecommons.org/licenses/by/2.0)], via Wikimedia
Commons
455
Hirschsprung Disease
• Congenital intestinal aganglionosis
• 80% rectosigmoid
• 15-20% to sigmoid (long segment)
• 5% entire colon
Pratap et al. BMC Pediatrics 2007 7:5
doi:10.1186/1471-2431-7-5[CC-BY-2.0
• 5:1 males to females
(http://creativecommons.org/licenses/by/2.0)
• Major gene RET (AD loss of function)
• Many other genes involved rarely (GDNF, NRTN, EDNRB,
EDN3, ECE1, NRG1, SEMA3C, SEMA3D)
• 12% occur as component of syndrome (some chromosomal,
especially Down syndrome)
Alagille Syndrome
• Clinical
• Deficiency/atresia intrahepatic bile ducts
• Cholestasis, neonatal jaundice
• Skeletal anomalies, ocular anomalies (posterior
embryotoxon)
• Characteristic facial appearance
• Genetics
• AGS1- 94%
• JAG1 (Jagged-1 is ligand for Notch receptor)
• Microdeletion 20p12 in 7%
• AGS2
• NOTCH2
Bile Pigment Metabolism
• Loss of bilirubin conjugation – Crigler-Najjar syndrome
• UGT1A1 deficiency
• CN1 – bili 20-40 mg/dl
• CN2 – bili 5-20 mg/dl (partial loss of function
mutations)
• Gilbert syndrome – reduced expression due to promoter
polymorphism mild/intermittent hyperbili
• Dubin-Johnson syndrome – benign MRP2 (ABCC2) deficiency
• Progressive familial intrahepatic cholestasis (ATP8B1,
ABCB11, ABCB4)
456
Hemochromatosis
• Excessive Fe absorption
• Fe overload in tissues
• cirrhosis
• diabetes mellitus
• heart failure
• more severe manifestations in males
• Treat with phlebotomy
• HFE
• Autosomal recessive, 1/400 Caucasians
• 85% C282Y
• Affected genotypes: C282Y/C282Y, C282Y/H63D
• TRF2
• Younger age of onset
• Juvenile hemochromatosis: HFE2, HEPC
Familial Hypercholesterolemia
• Deficiency of LDL receptor – LDLR -60-80%
• Other genes: APOB, PCSK9
liver
• Collectively 1:200 – 1:250
• Heterozygotes: hypercholesterolemia,
atherosclerosis, xanthomas
B-100
cholesterol
• Homozygotes: xanthomas, premature
atherosclerosis
LDL
cholesterol
amino acids
• Treatment with diet, statins, other cholesterol lowering
drugs, PCSK9 inhibitors
Lipoprotein Disorders
Disorder
Gene(s)
Hypercholesterolemia
LDLR, APOB, PCSK9, AD/AR
LDLRAP1
Inheritance
Hypercholesterolemia, xanthomas, atherosclerosis;
A/hypobetalipoproteinemia
MTP, APOB
Diarrhea, vomiting, abdominal distension, deficiency of
fat soluble vitamins
Chylomicron retention
SAR1B
Tangier disease
ABCA1
AR
LCAT Deficiency
LCAT
AR
Chylomicronemia
LPL, APOC2
Hypertriglyceridemia
APOA5, LMF1,
GP1HBP1
AR
Elevated triglycerides
Dysbetalipoproteinemia
APOE2
AR
Elevated cholesterol and triglycerides; vascular disease
Cholesterol ester storage
disease (Wolman)
LIPA
AR
Hypercholesterolemia, hepatomegaly – Wolman disease
is severe childhood form with hepatosplenomegaly,
steatorrhea, anemia – lysosomal acid lipase deficiency
AR
Phenotype
Unable to synthesize intestinal apoB-48; like
hypobetalipoproteinemia
Orange tonsils, hepatosplenomegaly, corneal opacities,
coronary artery disease, neuropathy; lack of HDL
Anemia, renal failure, corneal opacities, low HDL
Abdominal pain, nausea, vomiting, pancreatitis,
hepatosplenomegaly; elevated TG
457
Retinitis Pigmentosum
• Clinical
• Degeneration of photoreceptors or retinal pigment
epithelium
• Night blindness, progressive visual loss
• Abnormal ERG and visual fields
• Posterior subcapsular cataracts
• Genetics
• Multiple genes/modes of inheritance
• RHO 20-30% AD
• RPGR 80% X linked
• Digenic inheritance – PRPH2, ROM1
Other Retinal Disorders
• Usher Syndrome
• RP plus congenital sensorineural hearing impairment, vestibular dysfunction
• Type 1 – profound congenital deafness, later onset RP (MYO7A, USH1C, CDH23, PCDH15, USH1G, C1B2)
• Type 2 – mild – moderate congenital deafness, adolescent/adult RP (ADGRV1, WHRN, USH2A)
• Type 3 – slowly progressivevariable sensorineural hearing loss, RP (CKRB1, HARS)
• Leber Congenital Amaurosis
• Onset first year
• AR – multiple genes
• Poor vision, RP, eye rubbing
• Gyrate Atrophy
• Patches of choroidal, retinal atrophy
• Increased ornithine, deficient ornithine-ketoacid aminotransferase
• Choroidermeia
• X-linked
• Stippling, atrophy of fundus
• Cone-Rod Dystrophy
• Loss of cone function, reduced rod function
• Alstrom, Bardet-Biedl, neuronal ceroid lipofuscinoses, Joubert syndrome
Red-Green Color Blindness
• X-linked recessive
• 8% European males,
3-4% African ancestry males
• Severe
• Protanopia – no red cones
• Deuteranopia – no green cones
• Milder
• Protanomaly – anamolous green cones
• Deuteranomaly – anomalous red cones
458
Deafness
• Sensorineural vs. conductive
• 50% prelingual deafness genetic, 30% of which nonsyndromic
• Syndromic
• AD: Waardenburg, Branchio-oto-renal, Stickler, NF2
• AR: Usher, Pendred, Jervell & Lange-Nielsen, Biotinidase def, Refsum
• XL: Alport, Mohr-Tranedjaerg (deafness, dystonia, optic atrophy)
• Mitochondrial
• Nonsyndromic
• DFNA: AD
• DFN3A: GJB2
• DFNA3B: GJB6
• DFNA11: MYO7A (also Usher syndrome and DFNB2 AR deafness)
• DFNB: AR
• DFNB1: GJB2 (may also be compound het for GJB2 mutation & GJB6 del)
• DNFB2: MYO7A
Autism Spectrum Disorders: DSM 5
• Autism spectrum disorder – no subgroups
• Symptom domains
• Social communication impairment
• Restricted interests/repetitive behaviors
• Symptoms can be present or reported in past history
• Describe in terms of genetic cause, level of language
and intellectual disability, presence of other medical
problems
• Social Communication Disorder – no repetitive
behaviors
Autism-Spectrum Disorders
• Associated Features
• Regressive onset in 30%
• Seizures 25%
• Dysmorphology 15-20%
• Microcephaly 5-15%
• Macrocephaly 30%
• Genetics
• Genetic cause found in 20-25%
• If cause unknown, sibling risk 5-10% for ASD,
10-15% for milder abnormalities
459
ASD: Chromosomal Abnormalities
• Cytogenetically visible change 5%
• 15q11-q13 duplication maternally derived: 1-3% (supernumerary isodicentric 15q)
• Trisomy 21 (7%)
• 45,X Turner syndrome
• CNVs
• 2q37
• 7q11.23 (Williams syndrome)
• 8q
• 15q11 (Prader-Willi syndrome)
• 16p11.2
• 17p11.2 deletion (Smith-Magenis)
• 17p11.2 duplication (Potocki-Lupski)
• 22q13.3 (Ohelan-McDermid – SHANK3)
• Xp22.3
ASD: Single Gene Disorders
• Syndromes
• Fragile X
• PTEN macrocephaly syndrome
• Sotos syndrome
• Rett syndrome
• Tuberous sclerosis complex
• Metabolic disorders (mitochondrial, PKU,
adenylosuccinate lyase, creatine deficiency disorders,
Smith-Lemli-Optiz
• Isolated single gene mutations
Psychiatric Disorders
• Schizophrenia
• Delusions, hallucinations, disorganized speech and behavior, flattened affect, poverty
of speech, avolition, failure to maintain daily functions of life
• Affective disorders
• Major depression (MDD): dysphoria, changes in sleep, appetite, loss of sense of selfworth, guilt, suicidal ideation, no manic episodes
• Bipolar disease (BPD): periods of mania and depression
• Genetics
• Multifactorial
• Some candidate genes may overlap between disorders
• 22q11.2 deletion and schizophrenia
• CNVs in families with psychiatric disorders
• Genetic counseling based on analysis of patient and family history
• Pharmacogenetic considerations for treatment
460
Other Neuropsychiatric Conditions
• Specific reading disability
• unexplained difficulty in learning to read/write
• Familial clustering – multiple candidate loci
• Attention deficit/hyperactivity
• Excessive inattention, hyperactivity/impulsivity
• High heritability (0.76)
• Candidate loci in catecholamine and serotonin systems
• Addictive disorders
• Heritability 40-70%
• Multiple candidate loci (MAOA and COMT)
Cardiomyopathy
• Clinical
• Decrease in cardiac output
• Heart failure and arrhythmia
• Dilated vs. hypertrophic
• Genetics
• Mutations in contractile apparatus proteins
• Genetic and non-genetic etiologies
• All modes of inheritance
• Infectious
• Storage disorders (hemochromatosis)
• Treatment
• ACE inhibitors
• Beta blockers
Hypertrophic Cardiomyopathy
• Hypertrophy of myocardium
• Enlarged myocytes, disarray, fibrosis
• Signs/Symptoms – age of onset varies
• Congestive heart failure
• Chest pain
• Arrhythmia
• Sudden death
• LV hypertrophy by echo/ECG
• AD inheritance
• Contractile apparatus – MYH7, TNNT2/3, MPBPC3, TPM1, ACTC1,
MYL3, MYL2, others
461
Dilated Cardiomyopathy
• Congestive heart failure
• Arrhythmias/abnormal conduction
• Thromboembolic events (left ventricular mural
thrombus)
• Various modes of inheritance
• Surveillance every 2-5 years for first degree relatives
(exam, ECG, echocardiogram)
Syndromic vs. Non-syndromic
• Syndromic
• Hemochromatosis
• Muscular dystrophies
• Carvajal syndrome (palmoplantar keratoderma &
wooly hair) – AR
• Barth syndrome (neutropenia, lactic acidosis,
growth retardation, 3-methylglutaconic acid – Xlinked
• Mitochondrial
Genetics of DHM
• More than 30 genes account for 40-50% cases
• TTN mutations in 20% cases
• Copy number changes in some genes
• AD: TTN, LMNA, MYH7, MYH6, SCN5A, MYBPC3,
TNNT2, BAG3, ANKRD1, RBM20, TMPO, LDB3, TCAP,
VCL, TPM1, TNNI3, TNNC1, ACTC1, ACTN2, CSRP3,
DES, NEXN, PSEN1, PSEN2, SGCD, EYA4, PLN, DSG2
• XL: DMD, TAZ
462
Long QT
Clinical
Syncope, cardiac arrest
Associated with increase in sympathetic activity
Treated with β-blockers, pacemakers, sympathetic ganglionectomy
Genetics
Romano-Ward: KVLQT1
Homozygous mutations cause Jervell & Lange-Nielsen
syndrome (hearing loss)
Other genes:
HERG (K channel)
SCN5A (Na channel)
minK (K channel)
MiRP1 (K channel)
Hereditary Hemorrhagic Telangiectasia
• Clinical
• Cutaneous and visceral AV
malformations
• Epistaxis
• GI bleeding
• Pulmonary AVM – stroke and brain
abscess
• Genetics
• AD
• ENG, ACVRL1, SMAD4, GDF2
Proteus syndrome
• Progressive segmental
overgrowth
• Skeleton
• Skin
• Adipose tissue
• CNS
• AKT1 mosaic mutations in
90%
• C.49G>A (p.Glu17Lys)
• sporadic
By Darryl Leja, NHGRI [Public domain], via
Wikimedia Commons
463
Epigenetics
Normal development
One male, one female pronucleus
Failure to develop
Two male pronuceli
Two female pronuceli
DNA Methylation
Gene
Ac vator
Promoter
methyltransferase
cytosine
Me
CG
Me
Me
CG CG
Promoter
5-methylcytosine
Gene
464
Methylation Marks Erased in Germ Cells
Rett Syndrome
• Clinical
• Developmental regression
• Seizures
• Loss of motor coordination
• Stereotypies
• Atypical Rett syndrome – variable severity
• Genetics
• XLD
• MeCP2 loss of function mutations
Willard and Hendrich, Nature
Genetics 23, 127 - 128 (1999)
Prader-Willi and Angelman Syndromes
465
PWS/AS Mutations
Beckwith-Wiedemann Syndrome
• Clinical
• Macrosomia
• Macroglossia
• Omphalocele
• Hemihyperplasia
• Dysmorphism
• Risk of tumors
• Hepatoblastoma
• Wilms’ tumor
• Genetics
• 85% sporadic
• Some AD
• 1:13,000 births
BWS Region Chromosome 11
466
IC1 Mutations in BWS
IC2 Mutations in BWS
UPD in BWS
467
Paternal 11p Duplication
Russell-Silver Syndrome
• Clinical
• Low birth weight
• Relative macrocephaly
• FTT in infancy
• Delayed growth & bone age
• Skeletal asymmetry
• Urogenital anomalies
• Some with developmental delay
• Genetics
• Usually sporadic
• <5% Maternal UPD7
• 11p imprinting disorder
11p Duplication in RSS
468
Hypomethylation in RSS
GNAS
ֱ
֯
NESP55
NESP55
XL Sɲ
XL Sɲ
Exon1A
Exon1A
Gs ɲ
Exons 2-13
Gs ɲ
Exons 2-13
tissue-specific
expression
Albright hereditary osteodystrophy (intellectual disability and
subcutaneous calcification): paternal transmission of Gsɲ mutation
Pseudohypoparathyroidism-Ia (AHO and resistance to multiple
hormones): maternal transmission of Gsɲ mutation
Pseudohypoparathyroidism-Ib (renal parathyroid resistance): loss
of maternal methylation at exon 1A
Transient Neonatal Diabetes
• Clinical
• IUGR
• Severe neonatal diabetes that regresses around
12 weeks
• May relapse at times of stress
• Genetics
• TNDM1 locus on 6q24: PLAGL1 and HYMA1
• Paternally expressed, maternally methyated
• UPD(6)pat – 40%
• Dup(6q24)pat – 32%
• Maternal hypomethylation TNDM1 – 28%
469
470
Systems-Based Disorders II
SYSTEMS-BASED
DISORDERS II
John A. Phillips, III, MD, FACMG
David T. Karzon Professor of Pediatrics
Professor of Pathology, Microbiology and Immunology and
Professor of Medicine
Director Division of Medical Genetics and Genomic Medicine
Vanderbilt University School of Medicine
John A. Phillips, III, MD, FACMG
Division of Medical Genetics
Vanderbilt University School of Medicine
DD-2205 Medical Center North
Nashville, TN 37232-2578
(615) 322-7602 Telephone
(615) 343-0959 Fax
[email protected]
473
474
Systems Based Disorders II
John A Phillips III
David T Karzon Prof of Pediatrics
Vanderbilt University Medical Center
Site Investigator: 1) PKU: BMN 015 &165, 2) Achondroplasia: BMN 111-901, 201 &
202 Clinical Trials BioMarin Pharmaceutical Inc. & 3) FAOD: UX007 Ultragenyx
Pharmaceuticals, Inc.
PI: TN State Genetics Contract & Member TN Genetics Advisory Committee
Co-PI: Vanderbilt Undiagnosed Disease Network (UDN) Clinical Center
Co-I: Dr Blackwell’s PPG “Mechanisms of Familial Pulmonary Fibrosis”.
4/21/2017
Learning Objectives
• Pulmonary: CF, HPAH, IPF & Primary Ciliary Dyskinesia
• Immunogenetics: ADA, CVID, Hyper IgE/M & SCID
• Hematology: Hemophilia A/B, SS, ɲThalassemia/XL ID & ɴ
Thalassemia
• Endo: Androgen Insensitivity Syndrome, XL Adrenal Hypoplasia,
Antley Bixler Syndrome, CAH, Familial Hyperinsulinism,
Hypophosphatasia, GnRH def & PROP1 Deficiency
• Connective: Achondroplasia, Hypo/Pseudo-achondroplasia; MED, COL
(OI; Achondrogenesis, Kniest, SED & Stickler); CHH, Diastrophic
Dysplasia, EDS, Cutis Laxa, ɎX Elasticum & Marfan/LDS
• Renal: BOR, Lowe Syndrome, Polycystic Kidney Disease & AD
Tubulointestinal Diseases
475
Pul: Cystic Fibrosis (CFTR)
• Clin: Respiratory, exocrine pancreas, intestine, vas
deferens, hepatobiliary & sweat glands
• 15-20% neonates meconium ileus
• >95% CF males infertile
• CAVD in men w/o pulm or GI problems
• DX: 2 sweat Cl > 60 mEq/L OR 2 CFTR disease causing muts
(AR); ȴF508 ~0.7 of ~1500 CF alleles
• Rx: antibiotics, bronchodilators, mucolytic
(pulmonzyme, mucomyst), ? Kalydeco/ivacaftor G551D &
chest PT; avoid smoking, resp viruses & dehydration.
Pul: Cystic Fibrosis (CFTR)
Pul: Cystic Fibrosis- Mutation Specific Rx
476
Pul: Heritable Pulmonary Arterial Hypertension (HPAH)
• Clin: dyspnea (60%), fatigue
(19%), chest pain (7%),
palpitation (5%) or edema
(3%); PA pressure>25 mmHg
(rest)/ >30 (exercise) & other
causes of PAH excluded;
• Increased PA pressure causes
right heart failure & death
within 3 yrs of Dx
Pul:Heritable Pulmonary Arterial Hypertension (HPAH)
• Gen: ~6% of PAH cases familial, AD with ~10% penetrance,
variable age onset & ? anticipation, 2.4 females/male; ~75%
BMPR2 & ACVRL1, BMPR1B, CAV1, ENG & SMAD9 all rare
• Rx: sc/iv Treprostinil; po Bosentan, Sildenafil; neb Iloprost & iv
epoprostenol; avoid hypoxia, amphetamines & estrogens
Pul: Idiopathic Pulmonary Fibrosis (IPF)
• Clin: Bibasilar reticular anomalies/nodules on high res
CT, abnl lung func (VC), Dx usually 50-70 yrs; +/- lung
Ca; 30-50% 5 yr survival
• Gen: TERT, TERC & RTEL1 (short telomeres) or SFTPC in
8-15% multiplex & 3% simplex; all AD reduced
penetrance; PF also occurs in Hermansky Pudlak (AR) &
Dyskeratosis Congenita (AD, AR, XL)
• Rx: Supp O2, lung transplant &
no smoking
477
Pul: Primary Ciliary Dyskinesia (PCD)
• Clin: Abnl situs, sperm motility & cilia structure/function;
chronic otosinopulmonary disease, > 75% have ‘neonatal
respiratory distress’ & adults have bronchiectasis. Males
usually infertile.Situs inversus totalis 40%-50%
& heterotaxy ~12%.
• Gen: AR, Dx: clinical, ciliary structure or DNA
(~2/3 have 2 variants in 1/32 PCD causing genes.
• Rx: Immunizations (influenza/pneumococcal vaccines); hand
washing, avoid sick contacts, clean respiratory devices;
antibiotics for respiratory. Monitor lung function, sputum
cultures & hearing. Avoid: cough suppressants, smoking & air
pollutants.
Learning Objectives
• Pulmonary: CF, HPAH, IPF & Primary Ciliary Dyskinesia
• Immunogenetics: ADA, CVID, Hyper IgE/M & SCID
• Hematology: Hemophilia A/B, SS, ɲThalassemia/XL ID & ɴ
Thalassemia
• Endo: Androgen Insensitivity Syndrome, XL Adrenal Hypoplasia,
Antley Bixler Syndrome, CAH, Familial Hyperinsulinism,
Hypophosphatasia, GnRH def & PROP1 Deficiency
• Connective: Achondroplasia, Hypo/Pseudo-achondroplasia; MED, COL
(OI; Achondrogenesis, Kniest, SED & Stickler); CHH, Diastrophic
Dysplasia, EDS, Cutis Laxa, ɎX Elasticum & Marfan/LDS
• Renal: BOR, Lowe Syndrome, Polycystic Kidney Disease & AD
Tubulointestinal Diseases
Immun: Adenosine Deaminase Def
• Clin: SCID with FTT, opportunistic infections, marked
lymphocytopenia; absent humoral & cellular; usually Dx< 6/12
• Gen: <1% ADA activity (purine metabolism) or 2 known ADA
causing muts (AR)
• Rx: antibiotic, antifungal, IV immunoglobulin (IVIg),
Pneumocystis prophylaxis; bone marrow/stem cell transplant;
PEG ADA ERT
478
Immun: Common Variable Immune Def (CVID)
• Clin: Humoral immun def after 2yrs (often
young adults), sinopulmonary (Strep, H flu,
Kleb pn), meningitis after bacterial infections,
chronic diarrhea, malabsorption,+/- lymphoid
hyperplasia, autoimmune, lymphomas
• Gen: IgG<100 mg/dL to low, poor response Pneumovax; loss
TAC1, CD19, BAFFER protein; TNFRSF13B(TAC1) (10-15%), ICOS
(<1%) muts (AD, AR)
• Rx: Immune globulin (IVIg), antibiotics, monitor lymphoma,
thyroid function
Immun: AD Hyper IgE Syndrome
• Clin: Boils, cyst forming pneumonia & very high IgE;
characteristic face, Chiari malform,+/- eczema, candiasis,
osteopenia, fractures, scoliosis, arterial tortuosity &
aneurysms
• Gen: IgE >2000 IU/mL (~15x); STAT3 (AD)
• Rx: antibiotics to prevent Staph absecess/pn
Immun: X Linked Hyper IgM Syndrome
• Clin: IgG & A low; IgM due to abnl B & T cell function; 50%
onset by 1yr, >90% by 4yr, recurrent respiratory bacterial,
recurrent diarrhea with FTT; neutro & thrombopenia, anemia;
10-15% CNS infections; liver, GI, pancreatic tumors; lymphoma
(Hodgkins) & EBV
• Gen: CD40LG (aka TNFSF5/CD154)
muts in 95% affected males (XL)
• Rx: Allogenic hematopoietic cell
transplantation (HCT), recombinant
granulocyte stim factor (G-CSF) for
neutropenia, antibiotics & Pneumocystis prophylaxis
479
Immun: XL SCID
• Clin: Severe combined cellular & humoral immuno-deficiency
due to absent B & T cells, present 1-3/12 with FTT, oral/diaper
candidiasis, absent tonsils & lymph nodes, recurrent &
persistent infections despite Rx
• Gen: NBS (TRECs) in 34 states, IL2RG
muts in >99% of affected males
• Rx: Antibiotics (Pneumocystis), IVIg,
avoid live viral vaccines; bone marrow
transplantation ASAP, gene therapy?
Learning Objectives
• Pulmonary: CF, HPAH, IPF & Primary Ciliary Dyskinesia
• Immunogenetics: ADA, CVID, Hyper IgE/M & SCID
• Hematology: Hemophilia A/B, SS, ɲThalassemia/XL ID & ɴ
Thalassemia
• Endo: Androgen Insensitivity Syndrome, XL Adrenal Hypoplasia,
Antley Bixler Syndrome, CAH, Familial Hyperinsulinism,
Hypophosphatasia, GnRH def & PROP1 Deficiency
• Connective: Achondroplasia, Hypo/Pseudo-achondroplasia; MED, COL
(OI; Achondrogenesis, Kniest, SED & Stickler); CHH, Diastrophic
Dysplasia, EDS, Cutis Laxa, ɎX Elasticum & Marfan/LDS
• Renal: BOR, Lowe Syndrome, Polycystic Kidney Disease & AD
Tubulointestinal Diseases
Hem: Hemophilia A
• Clin: Prolonged oozing after trauma, tooth
extractions or surgery, age at DX relates to
F8 activity, severe joint /deep muscle bleeds
<2yrs; 10% carriers bleed
• Gen: Low F8 clotting activity with nl von
Willebrand factor level; F8 muts in 98% of
affected males (XL), IVS 22 inversion in 48%
of severe cases & dels/dups in 6%
• Rx: Hemophilia center, IV F8, DDAVP; avoid ASA, IM injections,
impact sports & activities & always Rx before circumcision
480
Hem: Hemophilia B
• Clin: Prolonged oozing after trauma, tooth
extractions or surgery, age at DX relates to
F9 activity, severe joint /deep muscle bleeds
<2yrs, 10% carriers bleed
• Gen: Low F9 clotting activity; F9 muts in
~100% of affected males (XL), dels/dups 3%
• Rx: Hemophilia center, IV F9; avoid ASA, IM
injections, impact sports & activities; always
Rx before circumcision
Ryan White
Hem: Sickle Cell Disease
• Clin: Intermittent vaso-oclusive events &
hemolytic anemia; dactylitis, splenic
infarction/asplenia, cholelithiasis, PAH &
leg ulcers
• Gen: HBB muts ɴS (Glu6Val), ɴC,ɴPunjab,
ɴOArab (AR); SS 60-70%; SS & SC <3.6%
& Sɴthal >3.6% Hb A2
• Rx: Hydration, transfusion, penicillin,
hydroxyurea; Rx PAH phosphodiesterase
inhibs/nitric oxide; monitor CBC, retics,
Fe; liver & renal function
481
Hem: Alpha Thalassemia
• Clin: Significant Hb Bart hydrops fetalis
(Hb Bart Syn) & HbH disease with 90 vs
5% Hb Bart
• Gen: Hb Bart syn, HbH, ɲ thal trait,
Silent carrier & nl have deletions of 4,
3, 2, 1 & 0 ɲ globin genes, respectively;
dels 90% & 10% point muts; ɲ thal
moderates SS
• Rx: Hb Bart syn fatal, HbH transfuse prn,
avoid excess Fe Rx & sulphonamides
Hem: Alpha Thalassemia XL Intellectual Disability Syndrome (ATRX)
• Clin: Microcephaly, telecanthus, coarse facies,
genital anomalies, hypotonia, ID & ɲ thal. Global
delays in infancy & may never walk or develop
speech.
• Gen: PE, ID, hypotonia, HbH, FHx suggesting XL
with 95% sequence & 5% del/dup ATRX variants.
• Rx: High caloric formula; anticholinergics,
botulinum toxin injection of salivary glands &/redirection of
submandibular ducts for excessive drooling; antibiotic
prophylaxis & vaccinations (pneumococcal/meningococcal) in
those with asplenia. Anemia rarely requires treatment.
Hem: Beta Thalassemia
• Clin: Reduced ɴ globin causes microcytic
hypochromic anemia & HbA; ɴ thal major
> severe anemia & hepatosplenomegaly <2 yrs
; marrow expansion
• Gen: RBC indices, Hb A &
months; nucleated RBCs
Hb F >12
• Rx: Regular transfusion, Fe chelation, bone
marrow transplant, folic acid & splenectomy?
;monitor endocrine function; avoid ETOH &
iron meds
482
Learning Objectives
• Pulmonary: CF, HPAH, IPF & Primary Ciliary Dyskinesia
• Immunogenetics: ADA, CVID, Hyper IgE/M & SCID
• Hematology: Hemophilia A/B, SS, ɲThalassemia/XL ID & ɴ
Thalassemia
• Endo: Androgen Insensitivity Syndrome, XL Adrenal Hypoplasia,
Antley Bixler Syndrome, CAH, Familial Hyperinsulinism,
Hypophosphatasia, GnRH def & PROP1 Deficiency
• Connective: Achondroplasia, Hypo/Pseudo-achondroplasia; MED, COL
(OI; Achondrogenesis, Kniest, SED & Stickler); CHH, Diastrophic
Dysplasia, EDS, Cutis Laxa, ɎX Elasticum & Marfan/LDS
• Renal: BOR, Lowe Syndrome, Polycystic Kidney Disease & AD
Tubulointestinal Diseases
Endo: Androgen Insensitivity Syndrome
• Clin: undermasculinization external genitalia at birth, abnl
secondary sexual devel in puberty & infertility with 46,XY
karyotype. AIS spectrum: Complete (CAIS), with typical female
external genitalia, Partial (PAIS) with predominantly female,
male, or ambiguous genitalia & Mild (MAIS) with typical male
external genitalia
• Gen: 46,XY; XL due to AR variants. Dx: external genitalia,
spermatogenesis, absent/rudimentary Müllerian structures,
normal or testosterone (T), dihydroT
& luteinizing hormone (LH).
• Rx: CAIS removal of testes?, vaginal
dilation; assignment of sex of rearing.
Endo: XL Adrenal Hypoplasia Congenita (AHC)
• Clin: Acute adrenal insufficiency by 3 wks in ~60%;
vomiting, hypoglycemia & salt wasting with
hyperkalemia & ACTH; +/- crytorchidism;
• Gen: NROB1(DAX1) dels in 100%
with glycerol kinase def +/- DMD;
but point mutations in nearly all
isolated AHC
• Rx: IV glucose & NaCl; gluco &
mineralocorticoids; oral NaCl
483
Endo: Cytochrome P450 Oxidooreductase Def (Antley Bixler)
• Clin: Steroidogenic defect ranging from cortisol
deficiency to Antley Bixler syndrome (ABS) with
ambiguous genitalia, craniosynostosis, choanal atresia,
radio humeral synostosis & cortisol
• Gen: sterol/steroid abnormalities, POR muts (AR)
• Rx: Cortisol, tracheostomy,
surgery for craniosynostosis
& hypospadias
Endo: Congenital Adrenal Hyperplasia
• Clin: > 90% CYP21OHD, impaired cortisol synthesis by adrenal
cortex, simple virilizing (25%) & salt wasting (low cortisol AND
inadequate aldosterone) (75%), NBS of neonates lowers risk
for initial fatal salt wasting crisis
• ACTH causes adrenal hyperplasia &
over-production of 17 OHP & sex hormones
• Gen: CYP21A2 sequencing
panel or del/dups detects
80-98% (AR)
• Rx: glucocorticoid (increase
with stress), salt wasting add
mineralocorticoid & NaCl
Endo: Familial Hyperinsulinism
• Clin: Hypoglycemia (ranges from
severe neonatal to mild childhood
onset)
• Gen: ~45% ABCC8 (97% in
Ashkenazi Jews) & 5% KCNJ11
(AR), ~5% GLUD1 & 5% HNF4A
(AD with anticipation)
• Rx: IV glucose, diazoxide,
diet , pancreatic resection
& avoid fasting
484
Endo: Hypophosphatasia
• Clin: Prenatal: hypomineralization bone +/or teeth & limb deformaties.
Range: stillbirth to lower extremity fractures in adults. Six forms: 1)
Perinatal (severe) respiratory insufficiency & hypercalcemia, 2) Perinatal (benign) prenatal skeletal changes slowly improve, 3) Infantile
onset birth-6/12 of FTT, rickets & Sx, 4) Childhood (juvenile) low bone
density, fractures & premature loss of teeth, 5) Adult stress fractures of
lower extremities in middle age & 6) Odontohypophospha-tasia
premature loss of primary teeth +/or severe caries without skeletal
problems.
• Gen: AR serum alkaline phosphatase (ALP) & 1-2
pathogenic ALPL variants &ј phosphoethanolamine.
• Rx: Infantile/childhood types: ERT (asfotase alfa),
respiratory care, Rx hypercalcemia, sz with B6 &
craniosynostosis. Other types: dental care by 1 yr;
NSAIDs for osteoarthritis, bone pain & osteomalacia.
Avoid: Bisphosphonates & excess vit D.
Endo: Isolated Gonadotropin Releasing Hormone (GnRH) Def
• Clin: Low testosterone in males & estradiol females; LH & FSH
with hypogonadism, +/- micropenis/ cryptorchidism, small
testes, absent puberty, 60% anosmia (aka Kallmann or KS),
bimanual synkinesis
• Gen: CHD7, FGF8, FGFR1, KAL1, ROK2,
PROKR2 muts ~25% (AD), KAL1 ~10%
KS (XL); > 15 genes cause ~1/2 of
Normosomic isolated GnRH Deficiency
• Rx: Testosterone, hCG in males
& estrogen, progestins in females to
induce pubertal changes
Endo: WZKWϭ Related Combined Pituitary Hormone Def
• Clin: Combined Pituitary Hormone Deficiency (CPHD)
with GH, TSH, LH, FSH & PrL +/- ACTH deficiencies;
short stature, FTT in childhood
• Gen: PROP1 muts >
98% (AR)
Nature Gen 18: 147-149, 98
• Rx: GH until 17 yrs,
L thyroxine, +/testosterone or
estrogens, +/hydrocortisone
485
Learning Objectives
• Pulmonary: CF, HPAH, IPF & Primary Ciliary Dyskinesia
• Immunogenetics: ADA, CVID, Hyper IgE/M & SCID
• Hematology: Hemophilia A/B, SS, ɲThalassemia/XL ID & ɴ
Thalassemia
• Endo: Androgen Insensitivity Syndrome, XL Adrenal Hypoplasia,
Antley Bixler Syndrome, CAH, Familial Hyperinsulinism,
Hypophosphatasia, GnRH def & PROP1 Deficiency
• Connective: Achondroplasia, Hypo/Pseudo-achondroplasia; MED, COL
(OI; Achondrogenesis, Kniest, SED & Stickler); CHH, Diastrophic
Dysplasia, EDS, Cutis Laxa, ɎX Elasticum & Marfan/LDS
• Renal: BOR, Lowe Syndrome, Polycystic Kidney Disease & AD
Tubulointestinal Diseases
Conn Tissue: Achondroplasia & Hypo-chondroplasia (&'&Zϯ)
• Clin: Rhizomelic short stature, macrocephaly/ ICP,
obstructive apnea; kyphosis, lordosis; narrowing
interpedicular distance; trident hand, genu varum;
cranio-cervical compression (CC) & spinal stenosis
• Dx: Signs & X rays; FGFR3: Achondroplasia 98% G>A
transition due to CpG results in Gly380Arg vs
Hypochondroplasia 49% Asn540Lys (C>A) & 21% C>G
• Rx: CNS shunt for ICP; sleep & CC apnea; otitis; orthopedics
for gibbus, genu varum & spinal stenosis; avoid trampoline
& gymnastics
Conn Tissue: Achondroplasia & Hypo-chondroplasia (&'&Zϯ)
486
Conn Tissue: Pseudoachondroplasia (KDW)
• Clin: Nl length at birth, nl facies, decline growth ~ 2 yrs, brachydactyly; loose hands, knees ankles but restricted elbows &
hips; joint pain & ~1/2 lumbar lordosis
• Dx: Signs & X rays show delayed epiphyses ossification &
anterior beaking of vertebrae; COMP point muts
• Rx: Surgery for lower limb malalignment, scoliosis & C1-2
fixation; watch for
odontoid hypoplasia;
avoid trampoline
& gymnastics
Conn Tissue: COL1A1/2 Related Osteogenesis Imperfecta
(OI types 1-4)
• Clin: Fx after min trauma, +/- dentinogenesis imperfecta
(gray/brown) & hearing loss (adults)
• OI type 1: Non-deforming with blue sclerae
• OI type 2: Perinatal lethal
• OI type 3: Progressively deforming
• OI type 4: Variable OI with nl sclerae
• Dx: Family Hx (fx/signs), X rays (fractures, wormian bones,
codfish vertebrae & osteopenia) & COL1A1/2 molecular
testing (90% sensitivity) & / or biochemical analysis of type 1
collagen (98% sensitivity) in OI types 1-4
Conn Tissue: COL1A1/2 Related Osteogenesis Imperfecta
487
Conn Tissue: COL1A1/2 Related Osteogenesis Imperfecta
• Gen: Molecular testing COL1A/2 detects > 90% of
OI types 1- 4 (AD, > 95% seq changes & 2%
del/dup); biochemical detects 90, 98, 84 & 84% of
OI types 1- 4 & 60, 100, ~100% of types 1-3 de
novo, respectively
• Rx: Orthopedic & Otolaryngology management,
periodic dental & hearing eval; bisphosphonates,
oral alendronate or risedronate & GH may reduce
fractures, increase bone density & improve growth
Conn Tissue: Osteogenesis Imperfecta Types 5-10
• OI Types 5-7: Fx; no dentinogenesis (D) or hearing loss (HL);
abnl vertebrae & hyperplastic callous; OI Type 5: IFITM5
(AD); OI Type 6: SERPINF1 (AR); OI Type 7: rhizomelic
shortening of all limbs; CRTAP (AR)
• OI Type 8: Fx, no D or HL; short
limb dwarfism; gracile long bones;
LEPRE1 (AR)
• OI Type 9: Fx, white/gray sclerae, short
limb dwarfism, bowed limbs; PPIB (AR)
• OI Type 10: similar to OI Type 1; caused by SERPINH1 (AR)
Conn Tissue: Multiple Epiphyseal Dysplasia (AD)
• Clin: Joint pain (hips & knees) in early childhood,
decreased ROM, early arthritis & adult Ht lower
nl or mild short
• Dx: Clin & X ray (small, irregular epiphyses but
nl spine except for Schmorl nodes);
• Gen: COMP, COL9A1-3, MATN3 but 10-20% neg
• Rx: Orthopedic, avoid sports involving
joint overload & caution with NSAIDs
488
Conn Tissue: Multiple Epiphyseal Dysplasia (AR)
• Clin: Joint pain (hips & knees), malformations
(hands, feet & knees) & scoliosis;
50% had clubfoot, clinodactyly
or CP at birth; adult Ht 150-180 cm
• Dx: Clin & X ray; Gen: SLC26A2
seq variants in ~100% (AR)
• Rx: Orthopedic, avoid sports
involving joint overload &
caution with NSAIDs
Conn Tissue: Type II Collagenopathies (Achondrogenesis 2, Kniest,
SED & Stickler)
• Achondrogeneis Type 2 (Langer Saldino): micromelic dwarfism,
CP, short ribs & abnl vert; stillborn/neonatal death;
COL2A1(AD) de novo
• Kniest: Short stature & trunk (platyspondyly); hearing loss
(HL); myopia & retinal detach (MRD) & cataract; COL2A1 (AD)
• Spondyloepiphyseal Dysplasia (SEDC): flat face/CP, MRD, abnl
vert, cervical myelopathy; COL2A1 (AD)
• Stickler: Flat face, MRD & cataract; HL, CP +/- Robin; COL2A1,
COL11A1-2 (AD); COL9A1-3 (AR); 80-90% COL2A1, 10-20%
COL11A1, others rare
Conn Tissue: Type 2 Collagen Disorders
Achondrogenesis
Kniest / SED
Stickler
489
Conn Tissue: Cartilage Hair Hypoplasia (Anauxetic Dysplasia
Spectrum)
• Clin: Severe short limbs, joint hypermobility, fine
silky hair, immunodeficiency, anemia, GI dysfunction & risk for malignancy
• Gen: metaphyseal dysplasia, +/- epiphyseal
& vertebral dysplasia; RMRP (AR)
• Rx: Transfusions, surgery cervical vert & kyphosis,
antibiotics (neutropenia), immunoglobulin if
IgG low, BMT (SCID); avoid live vaccines; varicella
can be lethal; 11% malignancies
Conn Tissue: Diastrophic Dysplasia
• Clin: short limbs, nl skull, hitchhiker thumbs, spine
(scoliosis, lordosis, kyphosis), joint
contractures & osteoarthritis; CP 1/3,
cystic ears 2/3, clubfeet
• Gen: Clinical & radiologic confirmed by
SLC26A2 >90% have sequence
variants (AR)
• Rx: PT, casting, ortho surgery with
caution as deformities tend to recur,
watch C spine for cord compression
Conn Tissue: Ehlers Danlos Syndrome
• Classic (I & II): skin hyperext, abl wound
healing & joint hypermobility
• Hypermobility (III): soft skin, dislocations,
pain +/- aortic dilation
• Vascular (IV): thin, translucent skin; bruising,
vascular rupture (12% death secondary to
arterial/Uterine rupture in pregnancy);
GI perforation
• Kyphoscoliotic (VI): friable, hyper extensible skin; scars,
bruising, hypotonia, progressive scoliosis & fragile sclerae
490
Conn Tissue: Ehlers Danlos Syndrome
Ehlers Danlos Syndrome Gen & Rx
• I & II ~50% have COL5A1/2 seq, del/dup (AD)
• III TNXB haploinsufficiency very rare (AD)
• IV COL3A1 seq >95%, del/dup 2% (AD)
• VI deoxypyridinoline/pyridinoline ratio in urine (HPLC) due
to deficient lysyl hydroxylase 1 (PLOD1) activity (fibroblasts);
PLOD1 seq?, del/dups ~18% (AR)
• Rx: I/II care with sutures, ascorbic acid? avoid ASA & contact
sports (CS); III watch aorta & avoid CS/joint hyperextension;
VI ortho, opthal, pregnancy risks & avoid CS; IV avoid CS &
arteriography
Conn Tissue: Cutis Laxa
• Clin: Skin is “doughy”, furrowed especially neck, axillae &
groin, droopy on face & extends without hyper elasticity;
myopia & hernias
• Gen: ATP6VOA2 (AR) thick cortex & cerebellar anomalies;
EFEMP2 aka FBLN4 (AR) arterial tortuosity & aneurysms;
FBLN5 (AR, AD) pulmonary emphysema & peripheral PS
• Rx: avoid smoking
491
Conn Tissue: Pseudoxanthoma Elasticum
• Clin: Affects elastic tissue of skin, eye, CV & GI
systems; skin (papules); retina (streaks or
hemorrhage); GI bleeds, angina/ claudication
• Gen: Skin & eye findings & skin biopsy;
ABCC6 seq ~90% both alleles with dels
in 5-30% (AR)
• Rx: Intraocular injections for macular
degeneration; avoid contact sports, ASA,
NSAIDs; retinal exams in pregnancy
Conn Tissue: Marfan Syndrome
• Clin: Ocular (myopia; ectopia in 60%); skeletal
(dolichostenomelia, pectus, scoliosis); aortic
dilation/ tear/ rupture/ MV prolapse;
Systemic Score (thumb, wrist, pectus, pneumothorax, etc)
• Gen: Fam Hx, exam (especially ectopia lentis & aneurysm)
FBN1 seq 70-93%, dels/dups? (AD) 25% de novo (mosaicism?)
• Rx: Lens surg, ɴ blockers/losartan for
aortic dilation, surgery 5 cm or if 1
cm/yr; yearly opthal & ECHO; avoid
contact sports, isometric; CV stimulants
& LASIK;
CV risk with pregnancy
Conn Tissue: Loeys Dietz Syndrome
• Clin: Vascular (CNS, thoracic & abdominal arterial
aneurysms/dissections) & skeletal (pectus excav/
carin; scoliosis, lax joints, arachnodactyly & clubfeet.
• Continuum; 75% LDS type I have (hypertelorism,
bifid uvula/CP & craniosynostosis) & 25%
LDS Type II lack craniofacial findings
• Gen: TGFBR1/2 in 95% (AD), ¾ de novo
• Rx: Aortic dissection at smaller diameters
than Marfan, ɴ blockers, C spine instability; avoid contact
sports & CV stimulants
492
Learning Objectives
• Pulmonary: CF, HPAH, IPF & Primary Ciliary Dyskinesia
• Immunogenetics: ADA, CVID, Hyper IgE/M & SCID
• Hematology: Hemophilia A/B, SS, ɲThalassemia/XL ID & ɴ
Thalassemia
• Endo: Androgen Insensitivity Syndrome, XL Adrenal Hypoplasia,
Antley Bixler Syndrome, CAH, Familial Hyperinsulinism,
Hypophosphatasia, GnRH def & PROP1 Deficiency
• Connective: Achondroplasia, Hypo/Pseudo-achondroplasia; MED, COL
(OI; Achondrogenesis, Kniest, SED & Stickler); CHH, Diastrophic
Dysplasia, EDS, Cutis Laxa, ɎX Elasticum & Marfan/LDS
• Renal: BOR, Lowe Syndrome, Polycystic Kidney Disease & AD
Tubulointestinal Diseases
Renal: Branchiootorenal Spectrum Disorders
• Branchiootorenal (BOR) ear pits, tags, anomalies causing
deafness > 90%; branchial fistulae/cysts & renal
hypoplasia, dysplasia or agenesis
• Branchiootic Syn (BOS) is BOR without
renal anomalies
• Gen: Clin criteria EYA1 40% (AD);
SIX5 & SIX1 ~5% (AD)
• Rx: Excision fistulae/cysts, ear surgery,
aids, cochlear implants
Renal: Lowe Syndrome
• Clin: Eyes: cataracts, glaucoma & poor vision; CNS:
hypotonia, absent DTRs & 75-90% mild to severe ID
& Renal: Fanconi, RTA, renal rickets, phosphaturia,
aminoaciduria & renal failure by 10-20 yrs
• Gen: OCRL seq variants in 95%
affected males & carriers (XL);
enzyme in fibroblasts
• Rx: Remove cataracts, GERD;
oral Na/KHCO3, PO4 &
calcitriol; avoid contact lens
493
Renal: Polycystic Kidney Disease, AD
• Clin: Late onset, bilateral renal cysts; liver & pancreatic
cysts; CNS/aortic aneurysms & MVP; renal pain,
hypertension, renal failure by 60 yrs
• Dx: Renal US ш 3 cyts PPV = 100%; Gen: 85% PKD1 & 15%
PKD2 most seq but few del/dups (AD); contiguous PKD1 &
TSC2 del > PKD in utero & Tuberous Sclerosis; rare early
onset PKD with neg FH due to hypo morph PKD1 in trans
• Rx: Hypertension, pain, cyst decompression,
nephrolithiasis, clip small & aortic replacement for large
aneurysms; avoid nephrotoxic, caffeine & smoking
Renal: Polycystic Kidney Disease, AD
Renal: Polycystic Kidney Disease, AR
• Clin: Neonates with enlarged echogenic kidneys: ~50%
hepatomegaly, dilated bile ducts & echogenicity;
pulmonary hypoplasia with 30% dying by 1 yr of resp
insuff & >50% have renal failure in first decade
• Dx: Clin findings without renal cysts
in parents; ~80% PKHD1 seq
variants, dups/dels seen
• Rx: Resp failure, hypertension;
avoid NSAIDS, aminoglycosides
& caffeine
494
Renal: AD Tubulointestinal Kidney Disease
• Clin: Hyperuricemia & gout from
renal excretion uric acid, hyperuricemia
& gout as early as teens; creatinine
5-40 yrs & renal failure >40 yrs;
isosthenuria may exacerbate bouts of
dehydration
• Gen: UMOD, REN or MUC1 (AD)
• Rx: Allopurinol/probenecid for gout;
nephrology, peritoneal dialysis,
renal transplant; avoid nephotoxic
meds, dehydration & meat
References
Genetic Home References @ https://ghr.nlm.nih.gov
GeneReviews @ https://ghr.nlm.nih.gov
OMIM @ https://www.omim.org
495
496
Reproductive Genetics II
REPRODUCTIVE GENETICS II
Louise E. Wilkins-Haug, MD, PhD, FACMG
Division Director, Maternal Fetal Medicine and
Reproductive Genetics
Brigham & Women’s Hospital
Professor of Obstetrics, Gynecology and
Reproductive Biology
Harvard Medical School
Louise E. Wilkins-Haug, MD, PhD, FACMG
Division of Maternal Fetal Medicine
Brigham & Women’s Hospital
75 Francis Street
Boston, MA 02115
(617) 732-4208 Telephone
(617) 264-6310 Fax
[email protected]
499
500
Reproductive Genetics 2
Louise Wilkins-Haug
MFM Division Director
Brigham and Women’s Hospital
Disclosure(s) - None
Case – Returns for 18 week ultrasound
මWhat is it? Omphalocele
මCould it have been seen sooner?
Possibly
මRisk of undiagnosed aneuploidy?
About 10%
මRisk of genetic etiologies?
මHow to proceed?
501
Beckwith Wiedemann Syndrome
LGA, macroglossia,
omphalocele, hypertrophy,
increased risk of cancer
මIsolated omphalocele on
fetal US
ම20% have BWS
(Wilkins-Haug 2009)
Beckwith Wiedemann Syndrome
Several causes – 50% due to imprinting error on
chromosome 11
ම Will require specialized testing
Association with Assisted Reproductive Technologies
(ART)
ම 6 fold increase in ART (5% vs 1%)
ම Among ART pregnancies, 1/4000 risk of BWS
ම Majority have an imprint abnormality
ම Loss of methylation on maternal 11p.15
(DeBaun, 2003;Maher, 2003; Gicquel, 2003; Halliday, 2004)
Female germ cell development
Preimplantation
Ovarian stimulation
Oocyte freezing
IVF
Imprints completed at MII
Imprints completed by birth
Male germ cell development
Preimplantation
(Denomme M, 2012)
502
Role of ART – Animal Studies
Animal studies suggest a
contribution of the ART
ම Large offspring syndrome
ම Ovulation induction, oocyte
manipulation, culture media
ම LOS associated with birth defects
Extremes of imprint
deregulation in cloned animals
Imprinting assures male/female
contribution / biologic diversity
Alternative Hypothesis – Imprinting
Associated with Subfertility
Parental Questionnaire - children with Angleman,
Beckwith Weidemann and Prader Willi Syndrome and
controls
Children
ART
Subfertile
AS, BWS, PWS
Control children
6.4%
2.1%
6.8%
3.5%
Same 3 fold higher rate for subfertility without ART
among children with imprinting disorders
(Doornbos, M, 2007)
Epimutations, Imprinted Genes and “Adult
Onset” Disease
Developmental Origins of Health and Disease
(DOHaD)
ම“Fetal programming of adult onset disease”
මNutritional state and exposures during pregnancy
increase adult onset disease – metabolic syndrome,
diabetes, cardiovascular disease
මDue to placental and fetal genetic reprogramming
ම“Barker hypothesis”
(Batcheller A, 2011)
503
Epimutations, Imprinted Genes and “Adult
Onset” Disease
ART children/teens are at higher risk of
cardiometabolic disorders, specifically
ම Elevated systolic and diastolic blood pressure
ම Higher fasting glucose, triglycerides
ම Increased body fat composition
Persists when
ම Sibling groups compared
ම Corrected for parental variables (age, BMI, medical conditions)
ම Corrected for newborn birth weight
(Batcheller A, 2011; Liu, 2015; Pontisilli, 2015; Varoom, 2016 )
Epimutations, Imprinted Genes and “Adult
Onset” Disease
Mouse models
මWider spread epigenomic changes produced by
media changes
මAssociated with superovulation in some but not all
studies
මART in mouse models produces specific
epimutations leading to endothelial damage
(Ramierz-Perez, 2014, Varoomam, 2016)
Epimutations, Imprinted Genes and “Adult
Onset” Disease
Placenta
මFour of the 6 imprinted genes (CEBPA, MEST, NNAT and
SERPINF1) with aberrant hypomethylation following ART
මlinked to adipocyte development, insulin signaling and/or
obesity
Cord blood – epigenome alterations
Blastocysts
ම> 50% had epimutations of imprinted genes
(Katari S, 2009; Sakain, 2015. Estill, 2016, White 2015)
504
Case – How to get to an answer ?
Diagnostic Modalities
Chorionic villus
sampling
Amniocentesis
Percutaneous umbilical
blood sampling
Risks of Invasive Fetal Diagnostic Procedures
I - MISCARRIAGE
III - TECHNICAL
II - FETAL MORBIDITY
Rupture of membranes
Malformation
Infectious
Isoimmunization
No sample obtained
Misdiagnosis
Risk of Miscarriage – CVS and
Amniocentesis Compared
Cochrane review of risks
මTransabdominal CVS = 2nd trimester amniocentesis
මTranscervical CVS = slightly higher risk of miscarriage
Recent systematic review - miscarriage
මLoss rates transabdominal CVS compared to amniocentesis
Loss within 2
wks
Loss up to 4
wks
Loss duration
over pregnancy
CVS - TA
0.7%
1.3%
2.0%
Amniocentesis
0.6%
0.9%
1.9%
Publications Committee of the Society for Maternal Fetal Medicine, 2012
505
Risk of Miscarriage – CVS/ Amniocentesis
vs No Procedure
o Nonrandomized observational studies
o TA CVS or amniocentesis loss rates similar to no
invasive procedures
o Amniocentesis loss rates appear no higher than 1/3001/500 or even 1/1000
o May be even lower in experienced centers
Publications Committee of the Society for Maternal Fetal Medicine, 2012
Risk of Miscarriage
Why was CVS cited with higher loss rates ?
ම CVS less commonly performed, numerous centers to get
sample size (> 30)
ම Each with small numbers
ම Learning curve - high rate of “difficult” procedures, > 1 insertions
ම Comparison to amniocentesis
ම No correction for earlier gestational age at CVS
ම
Adjusting for confounding variables, and data > 1998, CVS
loss rate not significantly increased above the background
Publications Committee of the Society for Maternal Fetal Medicine, 2012
Patient Specifics and Amniocentesis Loss
Retrospective cohorts, <34 yo, loss < 28 wks
Loss < 28 wks
Prior loss
(> 3)
Prior vaginal
bleeding
No predisposing
factors
Cases
(n=3,910)
2.1 %
Controls
(n=5,324)
1.5 %
P
6.9%
3.5%
<0.04
5.9%
3.8%
<0.04
0.96%
0.93%
NS
<0.01
(Antsaklis, 2000)
506
Invasive procedures - Morbidity
Limb-reduction defects after CVS
මCVS between 10 and 13 weeks does not increase
the risk of limb reduction defects
Hemangioma
මMay be increased following TC - CVS
Invasive procedures - Morbidity
Leakage of amniotic fluid(AF) after
amniocentesis
1.7% leak AF (0.4% controls)
Most reseal in 1 week
oAF volume normal in 3 weeks
oDelivery < 37 weeks 50%; IUGR 30%
oPerinatal survival in > 90% of cases
Spontaneous 2nd trimester premature rupture of membranes
oHigh fetal loss rate (80%), low chance of survival (< 10%)
oNo clear benefit to amniotic “plugging” and fluid
replacement
(Borgida,2000)
Invasive procedures - Morbidity
Fetomaternal hemorrhage: Placental disruption
from the invasive testing
මSmall relative to the total fetoplacental blood volume
මUnsensitized Rh negative - receive anti-D
immunoglobulin
මAmniocentesis a better alternative with
isoimmunization
මRisk of worsening with CVS
507
Invasive procedures - Morbidity
Infectious risks
oHBV transmission – low unless active disease
oHCV is unknown
oHIV transmission lowest with antiretroviral therapy
(HAART)
oAmniocentesis only on HAART, preferably when the
viral load is undetectable
oCVS – theoretically avoid
Technical Risks - “Misdiagnosis" on CVS
Evaluation only by direct analysis
මSynchiotrophoblast and mesenchymal core cells may
differ
Maternal cell contamination
Confined placental mosaicism
ම1-2% of CVS samples
ම1/3 are fetal mosaicism, 2/3 are confined to placenta
මRisks of uniparental disomy, fetal growth restriciton
Twins
Misdiagnosis on Amniocentesis
Slow growing
cultures
Maternal Cell
Contamination
Sampling technique
Maternal DNA
polymorphisms
FISH studies
Direct DNA
analyses
1) Discard 1.0 ml
- MCCԜ>Ԝ5% in 26%
vs 2% in first 1 ml discarded
2) Avoid placenta
Weida J, 2016
508
Analysis of Fetal DNA
Whole chromosome aneuploidy detection
Microarray del/dup analysis
Disease specific genetic panels, specialized
studies
Whole exome sequencing
Microarray Utility in Obstetrics -Ultrasound
Abnormalities
(Hillman, 2013)
CMA and specific anomalies
Anomaly
% with CMA
anomaly
Isolated
- CNS (holoprosencephaly, cerebellar
hypoplasia),
15 – 17 %
- Skeletal (club foot, hand , other)
13 – 14 %
Multiple
- Cardiac (HLHS, Tetrology)
20 – 27 %
- CNS (posterior fossa)
23 %
- Cystic hygroma
17%
Shaffer , 2012
509
Microarray Utility – Obstetrics Nondividing
Cells
SABs
මadditional 9.8% with clinical relevance
Stillbirths
ම13% abnormal with prior normal or
unobtainable karyotypes
Preimplantation Genetic Diagnosis
(Raca, 2009)
Microarray Utility – Invasive testing
Greater disease detection
ම Structural/growth abnormality – 6.0%
ම AMA (amniocentesis)
ම0.5% known pathogenic changes
ම1.2% potential for clinical significance
Now supported as first line analysis for
invasive prenatal diagnostic testing (ACOG)
(Wapner, 2012)
Single Gene Disorders – Targeted Panels vs
Whole Exome Sequencing
Targeted gene panels
ම Chondrodysplasia, CNS, arthrogryposis
ම Advantage of targeted, more complete analysis of suspected
regions
ම Disadvantage – requires US findings of syndrome be recognized
Whole exome sequencing
ම Wider range of detection but in less robust detail
ම Advantages in detection of conditions with atypical, unknown
prenatal presentations
ම Disadvantage – higher rate of variants of unknown significance
510
WES and Prenatal Diagnosis
Small series
ම 30 fetuses/neonates with congenital abnormalities
ම 10% genetic diagnosis in 3; potentially significant sequence variants in 5
more
ම 24 fetuses with US anomalies
ම 25% total detection rate, definitive diagnosis in five and plausible
diagnosis in one
Similar to pediatric literature
ම 25% of patients with a suspected genetic disorder and prior negative
genetic testing
(Carrs, 2014; Drury 2014)
Case Continued
Couple declined invasive testing, infant delivers at 39 weeks
and has a normal microarray and negative BWS testing.
Undergoes successful repair.
2 years later - questions about another pregnancy
What else places them at risk for birth defects?
Susan is 39 years old, John is 40
ම Healthy, no prior surgeries, no current medications
ම Non-smokers, deny recreational drug use and consume alcohol with meals
ම Family history is non contributory for congenital anomalies, notable for diabetes / cardiovascular
disease on both sides; both are of Northern European ancestry
Exam notable for BMI = 40, otherwise benign
Human Teratogen Characteristics
(J Wilson, 1959) “Teratology” (David Smith, 1960s)
1) Susceptibility depends on genotype and adverse environmental
factors
2) Susceptibility varies with the developmental stage of exposure
3) Teratogenic agents act on developing cells/tissues to initiate
sequences of abnormal developmental events
4) The access of adverse influences to developing tissues depends on
the teratogenic agent
5) There are four manifestations - Death, Malformation, Growth
Retardation and Functional Defect
6) Dose response relationship
511
Human Teratogen Characteristics
(T Shepard, 1994)
Essential (1,2,3) or (1,3,4)
1) Exposure at critical time in pregnancy
ම Coumadin and first trimester exposure
2) Two or more epidemiologic studies of high quality
ම Valproic acid and neural tube defect
3) Clinical delineation of specific defects
4) Rare exposure associated with rare outcome
ම Cataracts in children exposed to rubella
Helpful
5) Teratogenicity in animal studies
6) Makes biologic sense
7) Proof in an experimental system
Teratogen References
90% of medications have inadequate data for
pregnancy
Journals
Books –Briggs, Shepard
Web-based
ම REPROTOX (https://reprotox.org)
ම TERIS (Teratogen Information System – online Shepard’s)
ම http://depts.washington.edu/terisweb/teris/
ම OTIS (Organization of Teratology Information Specialists)
ම http://www.teratology.org/OTIS_fact_Sheets.asp
ම www.mothertobaby.org/
Maternal Conditions of Concern for
Teratogenicity
Obesity
Diabetes
මType 1
මType 2
Alcohol Use
30% of women entering
pregnancy
11 % of women
7.6% pregnant women,
1.4% binge
(Reproductive age women - 51%,
15% binge)
512
Obesity and Congenital Anomalies
Overweight
BMI 25 -30
Obese
BMI > 30
NTD
1.2 (1.04 – 1.38)
1.87 (1.62-2.15)
Cardiovascular
1.17 (1.03-1.34)
1.3 (1.12 – 1.51)
Cleft lip +/- palate
1.0 (0.87-1.15)
1.20 (1.03- 1.40)
Risk begins with overweight
(Stothard K, 2009,)
Risk by Category of Obesity
NTD
Cardiac
defects
Orofacial
clefts
(Class I – III)
BMI 3034.9
BMI 35 –
39.9
BMI > 40
1.80
(1.29-2.54)
1.11
(1.03-1.19)
2.07
(1.13-3.48)
1.26
(1.12-1.42)
4.08
(1.87-7.75)
1.49
(1.24-1.80)
1.09
(0.91-1.30)
1.62
(1.26-2.09)
1.90
(1.27-2.86)
(Blomberg M, 2010)
Altered Folate with Increased BMI
Known link of congenital anomalies and folate (NTD, CHD)
Increased BMI associated with decreased folate levels
ම Altered absorption / metabolism
ම Altered dietary intake (High fat diets low in folate)
Supplementation studies
ම Primary occurrence – obese women, NTD risk remains despite folic acid
supplements (400 mcg/d)
ම Pre and post food supply fortification with folate - highest BMI quartile had
least NTD reduction Recurrence - folate protection only among women < 70 kg
(Shaw G,1996, Werler M,1996)
513
Diabetes and Birth Defects
Diabetes associated with birth defects
මType 1
මInsulin deficient, often childhood onset
මType 2
මInsulin resistant
Process of hyperglycemia, altered lipid
metabolism, oxidative stress and cell death
Major Malformations and 1st Trimester HbA1C
Joslin Clinic 1984-1992
45
40
HbA1C
35
< 6.1
6.1-9.0
9.1-12.0
12.1-15.0
> 15.0
30
%
25
20
15
10
5
0
RR
(CI)
1.0
1.4
2.2
8.6
11.1
(0.8-3.2) (0.9-5.3) (4.2-17.3) (4.8-25.4)
(Greene, 1994)
Diabetic Embryopathy
aOR
95% CI
130
34 -100
Renal Agenesis
12
3-46
Hydrocephaly/
MCA
12
3.7-40
Cardiac / MCA
11
6.2-19
All cardiac
4.6
2.9-7.5
Anencephaly
3.4
1.1-10
Sacral agenesis
(Correa, 2008)
514
Diabetes Type 2 and Obesity
Type 2 diabetes
ම“adult onset diabetes”
ම90% of diabetes diagnoses
මAssociated with obesity, decreased exercise,
increased abdominal girth
මEstimated 20% of affected individuals undiagnosed
(Langer O, 2000)
Diabetes Type 2, Obesity and Birth Defects
Significant risk for birth defects present only among obese GDM women
Estimated 10-15% of GDM are Type 2 with abnormal fasting
ම Associated with obesity
(Martinez-Frias M,2005)
Diabetes
Cohort analyses
of congenital
anomalies to all
diabetic
pregnancies
(Sheffield, J, 2002,)
515
Alcohol Exposure - FAS, FADS
(FAE), ARND, and ARBD
Fetal Alcohol Syndrome
ම 6 – 9 / 1000
ම facial dysmorphia, delayed growth, central nervous
system (CNS) impact
Fetal Alcohol Disease Spectrum
(Fetal Alcohol Effect)
ම 2-5% of school age children
ම Intellectual disabilities, behavior and learning
challenges and congenital malformations.
ම Alcohol – Related Neurodevelopment Disabilities
ම Alcohol-related Birth Defects (ARBD)
(May, P 2014; Cannon, 2016; CDC 2016)
Alcohol screening,
prevention
ම ACOG - screen first
prenatal visit and
preconception
ම Increase overall
awareness
ම 3 in 4 women “who
want to get
pregnant” consume
alcohol
(Behavioral Risk Factor Surveillance System (BRFSS), 2013)
Alcohol as a Teratogen
Absolute Alcohol per Day in Ounces
4.5
4
3.5
3
2.5
2
1.5
1
0.5
0
re
ad
in
g
pt
s)
(5
-7
ම Binge drinking (> 4
drinks in 2 hours or 1
day)
ම Early, first trimester
exposure
IQ
w
ei
gh
t
bi
rth
FA
S
ounces of
absolute alcohol
Drivers of alcohol
induced effects on
offspring
516
Alcohol as a Teratogen
: Genetic Susceptibility
Presentation varies by exposure levels but also
between women with same exposure levels
Dizygotic twins similarly exposed with discordant
phenotypes
In mice, strain specific phenotypic effects can be
replicated
Viral Infection as a Teratogen
Common features include CNS involvement,
placental infection and subtle longer term
concerns in live-borns
Toxoplasmosis – uncommon in US
Rubella – uncommon post vaccination era
CMV - omnipresent
Zika - epidemics of 2015 – 2016
Shepard's Criteria and Zika
Essential
1) Exposure at critical time in pregnancy - met
ම 1st trimester microcephaly
ම 3rd trimester exposure – IUGR, IUFD, CNS complications evolving
in neonate
2) > 2 epidemiologic studies of high quality - emerging
ම Ultrasound anomalies
ම Zika +
ම Zika -
12/42 (29%)
0/16
ම 1% microcephaly risk in 2013-14 French Polynesian Zika outbreak
(50 fold increase)
(Rasmussen, 2016)
517
Shepard's Criteria and Zika
3) Specific defects (vs range of congenital
anomalies) - met
ම Severe microcephaly, CNS calcifications,
ocular findings, redundant scalp skin,
arthrogryposis
ම Fetal brain disruption syndrome with unique
scalp redundancy (not usually seen with
microcephaly)
4) Rare exposure with rare outcome - met
ම Microcephaly a rare event
ම Zika not rare in Brazil; disease in women with
limited travel to the region is rare
(Epes, 2017; Rasmussen, 2016, Moore, 2017)
Shepard's Criteria and Zika
Helpful
5) Teratogenicity in animal studies – not
met
මBut shown to be neurotropic
6) Makes biologic sense - met
මSimilar to other fetal viral infections
මRecovered from fetal CNS tissue,
destructive / cell death as a mechanism
for microcephaly.
(Rasmussen, 2016)
Zika and Teratogenicity
1) Delineate full spectrum
ම Similar to Rubella and CMV - cataracts, hearing loss & developmental disability
2) Modifiers of infection
ම Coinfection with another virus
ම Preexisting immune response to another flavivirus
ම Genetic background of the mother or fetus
ම Severity of infection
3) Prevention
ම Avoidance
ම Vaccine
518
Return to Case
Their biggest question though is since they are
doing IVF, should they also have preimplantation
genetic testing?
a
c
b
d
Preimplantation Genetic Diagnosis (PGD)
Translocations
ම Array CGH replacing FISH
Single Gene disorders (over 300 to date)
ම Autosomal recessive, dominant, x-linked
ම Sex determination for X-linked disorders
Genetic disorders of adult onset
ම Predisposition to cancer risk (BRCA1/2)
HLA compatibility
ම Siblings with ALL, AML or Diamond Blackfan
anemia requiring bone marrow transplants
(Munne, 2012, Curr Genet 13: 463-7)
PGS – Preimplantation Genetic Screening
With increasing maternal age,
increasing aneuploidy
ම Day 5-6 (blastocyst)
ම 35-37 yo
44.2% aneuploidy
ම 41-42 yo
76.3% aneuploidy
3 color FISH
First approach – PGS-#1
ම Increasing probes increases “false positive”
ම 2007-2010 RCTs – Day 3 (FISH)
ම No benefit
ම Harm with decreased implantation
5 color FISH
519
Preimplantation Genetic Screening v2
New molecular approaches for comprehensive
chromosome screening (CCS)
ම aCGH, SNP array, qPCR, next generation sequencing
ම Karyomapping
ම Simultaneous detection of single gene disorder and molecular aneuploidy in
one assay (SNP analysis)
Further analysis for mosaicism
ම High levels at day 3, a portion correct by day 5
(Dahdouh, 2015,
Wells, Fert Sterl, 2014)
PGS#2 and Outcomes
Meta-analysis 4 RCT and 7 cohort studies
ම RCTs - about 250 cycles each arm
ම Cohorts – 2,338 women, 729 CCS, remainder controls
RCT studies
(RR)
95% CI
Cohort studies
(RR)
95% CI
Implantation
1.32
1.18 - 1.47
1.74
1.35 - 2.24
Clinical
Pregnancy rate
1.26
0.83 - 1.93
1.48
1.20 - 1.83
Ongoing
pregnancy rate
1.31
0.64 - 2.66
1.61
1.30 - 2.00
Livebirth
1.26
1.05 - 1.50
1.35
0.85 – 2.13
(Dahdouh, 2015, Chen, 2015 Wells, 2014)
PGS #2 Confounders
Data from RCT and cohorts
ම Combines indications (AMA, reproductive pregnancy loss)
ම Cleavage stage and blastocyst biopsies
ම Various molecular techniques not equal in efficacy
ම Biased toward women with “good prognosis”
ම PGS on day 5 blastocysts favors those who make it to day #5
Analysis based on cycles started
ම PGS significantly underperformed compared to non-PGS when
starting point was cycle initiated
(Gleicher,2014; Kushnir, 2016)
520
Current State of PGS
ESHRE Study into The Evaluation of oocyte Euploidy by
Microarray analysis (ESTEEM)
ම multicentre, randomized double-blind controlled trial with an intention-to-treat
analysis including women with advanced maternal age
Policy Statements for PGS - ineffective in improving clinical
pregnancy rates and decreasing miscarriage (Based on PGS
#1)
ම American Society of Reproductive Medicine
ම European Society for Human Reproduction and Embryology
ම British Fertility society
Case
They call your office 1.5 years later
Maternal weight decreased through lifestyle
management programs, ceased alcohol
consumption
Spontaneously conceived and just delivered a
healthy girl
Thank You For Your Attention
[email protected]
521
REFERENCES
Alfirevic Z, Gosden CM, Neilson JP. 2000. Chorion villus sampling versus amniocentesis for
prenatal diagnosis. Cochrane Database Syst Rev. 2):CD000055.
DeBaun MR, Niemitz ELFeinberg AP. (2003). Association of in vitro fertilization with
Beckwith-Wiedemann syndrome and epigenetic alterations of LIT1 and H19. Am J Hum
Genet. 72(1):156-60.
American College of Obstetricians and Gynecologists. Microarrays and next-generation
sequencing technology: the use of advanced genetic diagnostic tools in obstetrics and
gynecology. ACOG Committee opinion no. 682. (2016) Obstet Gynecol,128:e262-8
Denomme, M. M. and M. R. Mann (2012) Genomic imprints as a model for the analysis of
epigenetic stability during assisted reproductive technologies. Reproduction 144(4): 393-409.
Antsaklis A, Papantoniou N, Xygakis A, Mesogitis S, Tzortzis EMichalas S. 2000. Genetic
amniocentesis in women 20-34 years old: associated risks. Prenat Diagn. 20(3):247-50.
Batcheller, A., E. Cardozo, et al. (2011) Are there subtle genome-wide epigenetic alterations in
normal offspring conceived by assisted reproductive technologies? Fertil Steril 96(6): 130611.
Borgida AF, Mills AA, Feldman DM, Rodis JFEgan JF. 2000. Outcome of pregnancies
complicated by ruptured membranes after genetic amniocentesis. Am J Obstet Gynecol.
183(4):937-9.
Carss KJ, Hillman SC, Parthiban V, et al. (2014) Exome sequencing improves genetic diagnosis
of structural fetal abnormalities revealed by ultrasound. Human molecular
genetics.23(12):3269-3277.
Doherty AS, Mann MR, Tremblay KD, Bartolomei MSSchultz RM. (2000) Differential effects of
culture on imprinted H19 expression in the preimplantation mouse embryo. Biol Reprod.
62(6):1526-35.
Doornbos, M. E., S. M. Maas, et al. (2007). Infertility, assisted reproduction technologies and
imprinting disturbances: a Dutch study. Hum Reprod 22(9): 2476-80.
Dugoff L, Norton ME, Kuller JA. For Society for Maternal-Fetal Medicine (SMFM) (2016)The
use of chromosomal microarray for prenatal diagnosis. Consult Series 41. Am J Obstet
Gynecol, 215:B2-9.
Drury S, Williams H, Trump N, et al. (2015). Exome sequencing for prenatal diagnosis of fetuses
with sonographic abnormalities. Prenatal diagnosis. 35(10):1010-1017.
Carmichael, S. L., S. A. Rasmussen, et al. (2010). Prepregnancy obesity: a complex risk factor for
selected birth defects. Birth Defects Res A Clin Mol Teratol 88(10): 804-10.
Eppes C, Rac M, Dunn J, Versalovic J, Murray K, Suter M, Sanz Cortes M, et al (2017). Testing
for Zika virus infection in pregnancy. Am J Obstet Gynecol 2017. Testing for Zika virus
infection in pregnancy: key concepts to deal with an emerging epidemic. AJOG March 2017
epub
Chang AS, Moley KH, Wangler M, Feinberg AP, Debaun MR. (2005). Association between
Beckwith-Wiedemann syndrome and assisted reproduction technology: a case series of 19
patients. Fertil Steril 83(2):349-54
Gicquel, C., V. Gaston, et al. (2003). In vitro fertilization may increase the risk of BeckwithWiedemann syndrome related to the abnormal imprinting of the KCN1OT gene. Am J Hum
Genet 72(5): 1338-41.
Dahdouh E, Balayla J, Garcia-Velasco J. (2015). Impact of blastocyst biopsy and comprehensive
chromosome screening technology on preimplantation genetic screening: a systematic review
of randomized controlled trials. Reprod Biomed Online 30(3) 281-9.
Gilboa, S. M., A. Correa, et al. (2010). Association between prepregnancy body mass index and
congenital heart defects. Am J Obstet Gynecol 202(1): 51 e1-51 e10.
Davies M., V. M. Moore, et al. (2012). Reproductive technologies and the risk of birth defects. N
Engl J Med 366(19): 1803-13.
Hansen M, Kurinczuk JJ, Bower CWebb S. (2002). The risk of major birth defects after
intracytoplasmic sperm injection and in vitro fertilization. N Engl J Med. 346(10):725-30.
Hansen, M., C. Bower, et al. (2005). Assisted reproductive technologies and the risk of birth
defects--a systematic review. Hum Reprod 20(2): 328-38.
Hansen, M., J. J. Kurinczuk, et al. (2013). Assisted reproductive technology and birth defects: a
systematic review and meta-analysis. Hum Reprod Update.
Hillman, S. C., S. Pretlove, et al. (2011). "Additional information from array comparative
genomic hybridization technology over conventional karyotyping in prenatal diagnosis: a
systematic review and meta-analysis." Ultrasound Obstet Gynecol 37(1): 6-14.
Hillman SC, Willams D, Carss KJ, McMullan DJ, Hurles ME, Kilby MD. (2015). Prenatal exome
sequencing for fetuses with structural abnormalities: the next step. Ultrasound Obstet
Gynecol.45(1):4-9.
Katari, S., N. Turan, et al. (2009). DNA methylation and gene expression differences in children
conceived in vitro or in vivo. Hum Mol Genet 18(20): 3769-78.
Langer, O. (2008). Type 2 diabetes in pregnancy: exposing deceptive appearances. J Matern Fetal
Neonatal Med 21(3): 181-9.
Ludwig, M., A. Katalinic, et al. (2005). Increased prevalence of imprinting defects in patients
with Angelman syndrome born to subfertile couples. J Med Genet 42(4): 289-91.
Halliday, J., K. Oke, et al. (2004). Beckwith-Wiedemann syndrome and IVF: a case-control
study. Am J Hum Genet 75(3): 526-8.
Papantoniou NE, Daskalakis GJ, Tziotis JG, Kitmirides SJ, Mesogitis SA, Antsaklis AJ. 2001.
Risk factors predisposing to fetal loss following a second trimester amniocentesis. Bjog.
108(10):1053-6.
Reefhuis, J., M. A. Honein, et al. (2009). Assisted reproductive technology and major structural
birth defects in the United States. Hum Reprod 24(2): 360-6.
Shaw, G. M., E. M. Velie, et al. (1996). Risk of neural tube defect-affected pregnancies among
obese women." JAMA 275(14): 1093-6.
Shaffer L, Rosenfeld JA, Dabell MP, et al. (2012) Detection rates of clinically significant
genomic alterations by microarray analysis for specific anomalies detected by ultrasound.
Prenatal Diagnosis.32(10):986-995.
Shepard T. (1994) “Proof of human teratogenicity. Teratology 50:97-98.
Sheffield, J. S., E. L. Butler-Koster, et al. (2002). Maternal diabetes mellitus and infant
malformations. Obstet Gynecol 100(5 Pt 1): 925-30.
Stothard, K. J., P. W. Tennant, et al. (2009). Maternal overweight and obesity and the risk of
congenital anomalies: a systematic review and meta-analysis.JAMA 301(6): 636-50.
Towers CV, Asrat TRumney P. (2001). The presence of hepatitis B surface antigen and
deoxyribonucleic acid in amniotic fluid and cord blood. Am J Obstet Gynecol. 184(7):1514-8;
discussion 1518-20.
Maher E R. (2005). Imprinting and assisted reproductive technology. Hum Mol Genet 14:133-8.
van den Veyver IB, Eng CM. (2015).Genome-Wide Sequencing for Prenatal Detection of Fetal
Single-Gene Disorders. Cold Spring Harbor perspectives in medicine. 5(10)
Martinez-Frias, M. L., J. P. Frias, et al. (2005). Pre-gestational maternal body mass index predicts
an increased risk of congenital malformations in infants of mothers with gestational diabetes.
Diabet Med 22(6): 775-81.
Waller DK, Mills JL, Simpson JL, S et al. (1994). Are obese women at higher risk for producing
malformed offspring? Am J OB Gyn: 170:541-8.
Wapner, R. J., Martin C.L., et al. (2012). "Chromosomal microarray versus karyotyping for
prenatal diagnosis." N Engl J Med 367(23): 2175-84.
Wells D. (2014). Next-generation sequencing: the dawn of a new era for preimplantation genetic
diagnostics. Fertil Steril 101 (5) 1250-1.
Werler, M. M., C. Louik, et al. (1996). "Prepregnant weight in relation to risk of neural tube
defects." JAMA 275(14): 1089-92.
Wilkins-Haug L., A. Porter, et al. (2009). Isolated fetal omphalocele, Beckwith-Wiedemann
syndrome, and assisted reproductive technologies. Birth Defects Res A Clin Mol Teratol
85(1): 58-62.
522
Reproductive Genetics – Preconception References
Alfirevic Z, Gosden CM, Neilson JP. 2000. Chorion villus sampling versus amniocentesis for
prenatal diagnosis. Cochrane Database Syst Rev. 2):CD000055.
American College of Obstetricians and Gynecologists. Microarrays and next-generation
sequencing technology: the use of advanced genetic diagnostic tools in obstetrics and
gynecology. ACOG Committee opinion no. 682. (2016) Obstet Gynecol,128:e262-8
Anthony S, Buitendijk SE, Dorrepaal CA, Lindner K, Braat DDden Ouden AL. (2002)
Congenital malformations in 4224 children conceived after IVF. Hum Reprod. 17(8):208995.
Antsaklis A, Papantoniou N, Xygakis A, Mesogitis S, Tzortzis EMichalas S. 2000. Genetic
amniocentesis in women 20-34 years old: associated risks. Prenat Diagn. 20(3):247-50.
Batcheller, A., E. Cardozo, et al. (2011) Are there subtle genome-wide epigenetic alterations in
normal offspring conceived by assisted reproductive technologies? Fertil Steril 96(6): 130611.
Blomberg, M. I. and B. Kallen (2010) Maternal obesity and morbid obesity: the risk for birth
defects in the offspring.Birth Defects Res A Clin Mol Teratol 88(1): 35-40.
Borgida AF, Mills AA, Feldman DM, Rodis JFEgan JF. 2000. Outcome of pregnancies
complicated by ruptured membranes after genetic amniocentesis. Am J Obstet Gynecol.
183(4):937-9.
Carss KJ, Hillman SC, Parthiban V, et al. (2014) Exome sequencing improves genetic diagnosis
of structural fetal abnormalities revealed by ultrasound. Human molecular
genetics.23(12):3269-3277.
Carmichael, S. L., S. A. Rasmussen, et al. (2010). Prepregnancy obesity: a complex risk factor for
selected birth defects. Birth Defects Res A Clin Mol Teratol 88(10): 804-10.
Chang AS, Moley KH, Wangler M, Feinberg AP, Debaun MR. (2005). Association between
Beckwith-Wiedemann syndrome and assisted reproduction technology: a case series of 19
patients. Fertil Steril 83(2):349-54
Dahdouh E, Balayla J, Garcia-Velasco J. (2015). Impact of blastocyst biopsy and comprehensive
chromosome screening technology on preimplantation genetic screening: a systematic review
of randomized controlled trials. Reprod Biomed Online 30(3) 281-9.
Davies M., V. M. Moore, et al. (2012). Reproductive technologies and the risk of birth defects. N
Engl J Med 366(19): 1803-13.
DeBaun MR, Niemitz ELFeinberg AP. (2003). Association of in vitro fertilization with
Beckwith-Wiedemann syndrome and epigenetic alterations of LIT1 and H19. Am J Hum
Genet. 72(1):156-60.
523
Denomme, M. M. and M. R. Mann (2012) Genomic imprints as a model for the analysis of
epigenetic stability during assisted reproductive technologies. Reproduction 144(4): 393-409.
Doherty AS, Mann MR, Tremblay KD, Bartolomei MSSchultz RM. (2000) Differential effects of
culture on imprinted H19 expression in the preimplantation mouse embryo. Biol Reprod.
62(6):1526-35.
Doornbos, M. E., S. M. Maas, et al. (2007). Infertility, assisted reproduction technologies and
imprinting disturbances: a Dutch study. Hum Reprod 22(9): 2476-80.
Dugoff L, Norton ME, Kuller JA. For Society for Maternal-Fetal Medicine (SMFM) (2016)The
use of chromosomal microarray for prenatal diagnosis. Consult Series 41. Am J Obstet
Gynecol, 215:B2-9.
Drury S, Williams H, Trump N, et al. (2015). Exome sequencing for prenatal diagnosis of fetuses
with sonographic abnormalities. Prenatal diagnosis. 35(10):1010-1017.
Eppes C, Rac M, Dunn J, Versalovic J, Murray K, Suter M, Sanz Cortes M, et al (2017). Testing
for Zika virus infection in pregnancy. Am J Obstet Gynecol 2017. Testing for Zika virus
infection in pregnancy: key concepts to deal with an emerging epidemic. AJOG March 2017
epub
Gicquel, C., V. Gaston, et al. (2003). In vitro fertilization may increase the risk of BeckwithWiedemann syndrome related to the abnormal imprinting of the KCN1OT gene. Am J Hum
Genet 72(5): 1338-41.
Gilboa, S. M., A. Correa, et al. (2010). Association between prepregnancy body mass index and
congenital heart defects. Am J Obstet Gynecol 202(1): 51 e1-51 e10.
Halliday, J., K. Oke, et al. (2004). Beckwith-Wiedemann syndrome and IVF: a case-control
study. Am J Hum Genet 75(3): 526-8.
Hansen M, Kurinczuk JJ, Bower CWebb S. (2002). The risk of major birth defects after
intracytoplasmic sperm injection and in vitro fertilization. N Engl J Med. 346(10):725-30.
Hansen, M., C. Bower, et al. (2005). Assisted reproductive technologies and the risk of birth
defects--a systematic review. Hum Reprod 20(2): 328-38.
Hansen, M., J. J. Kurinczuk, et al. (2013). Assisted reproductive technology and birth defects: a
systematic review and meta-analysis. Hum Reprod Update.
Hillman, S. C., S. Pretlove, et al. (2011). "Additional information from array comparative
genomic hybridization technology over conventional karyotyping in prenatal diagnosis: a
systematic review and meta-analysis." Ultrasound Obstet Gynecol 37(1): 6-14.
Hillman SC, Willams D, Carss KJ, McMullan DJ, Hurles ME, Kilby MD. (2015). Prenatal exome
sequencing for fetuses with structural abnormalities: the next step. Ultrasound Obstet
Gynecol.45(1):4-9.
Katari, S., N. Turan, et al. (2009). DNA methylation and gene expression differences in children
conceived in vitro or in vivo. Hum Mol Genet 18(20): 3769-78.
524
Langer, O. (2008). Type 2 diabetes in pregnancy: exposing deceptive appearances. J Matern Fetal
Neonatal Med 21(3): 181-9.
Ludwig, M., A. Katalinic, et al. (2005). Increased prevalence of imprinting defects in patients
with Angelman syndrome born to subfertile couples. J Med Genet 42(4): 289-91.
Maher E R. (2005). Imprinting and assisted reproductive technology. Hum Mol Genet 14:133-8.
Martinez-Frias, M. L., J. P. Frias, et al. (2005). Pre-gestational maternal body mass index predicts
an increased risk of congenital malformations in infants of mothers with gestational diabetes.
Diabet Med 22(6): 775-81.
Papantoniou NE, Daskalakis GJ, Tziotis JG, Kitmirides SJ, Mesogitis SA, Antsaklis AJ. 2001.
Risk factors predisposing to fetal loss following a second trimester amniocentesis. Bjog.
108(10):1053-6.
Reefhuis, J., M. A. Honein, et al. (2009). Assisted reproductive technology and major structural
birth defects in the United States. Hum Reprod 24(2): 360-6.
Shaw, G. M., E. M. Velie, et al. (1996). Risk of neural tube defect-affected pregnancies among
obese women." JAMA 275(14): 1093-6.
Shaffer L, Rosenfeld JA, Dabell MP, et al. (2012) Detection rates of clinically significant
genomic alterations by microarray analysis for specific anomalies detected by ultrasound.
Prenatal Diagnosis.32(10):986-995.
Shepard T. (1994) “Proof of human teratogenicity. Teratology 50:97-98.
Sheffield, J. S., E. L. Butler-Koster, et al. (2002). Maternal diabetes mellitus and infant
malformations. Obstet Gynecol 100(5 Pt 1): 925-30.
Stothard, K. J., P. W. Tennant, et al. (2009). Maternal overweight and obesity and the risk of
congenital anomalies: a systematic review and meta-analysis.JAMA 301(6): 636-50.
Towers CV, Asrat TRumney P. (2001). The presence of hepatitis B surface antigen and
deoxyribonucleic acid in amniotic fluid and cord blood. Am J Obstet Gynecol. 184(7):1514-8;
discussion 1518-20.
van den Veyver IB, Eng CM. (2015).Genome-Wide Sequencing for Prenatal Detection of Fetal
Single-Gene Disorders. Cold Spring Harbor perspectives in medicine. 5(10)
Waller DK, Mills JL, Simpson JL, S et al. (1994). Are obese women at higher risk for producing
malformed offspring? Am J OB Gyn: 170:541-8.
Wapner, R. J., Martin C.L., et al. (2012). "Chromosomal microarray versus karyotyping for
prenatal diagnosis." N Engl J Med 367(23): 2175-84.
Wells D. (2014). Next-generation sequencing: the dawn of a new era for preimplantation genetic
diagnostics. Fertil Steril 101 (5) 1250-1.
525
Werler, M. M., C. Louik, et al. (1996). "Prepregnant weight in relation to risk of neural tube
defects." JAMA 275(14): 1089-92.
Wilkins-Haug L., A. Porter, et al. (2009). Isolated fetal omphalocele, Beckwith-Wiedemann
syndrome, and assisted reproductive technologies. Birth Defects Res A Clin Mol Teratol
85(1): 58-62.
526
Genomic Medicine
GENOMIC MEDICINE
Bruce R. Korf, MD, PhD, FACMG
Wayne H. and Sara Crews Finley Chair in Medical Genetics
Professor and Chair, Department of Genetics
Director, Heflin Center for Genomic Sciences
University of Alabama at Birmingham
Bruce R. Korf, MD, PhD, FACMG
Department of Genetics
University of Alabama at Birmingham
Kaul 230, 1530 3rd Avenue South
Birmingham, AL 35294-0024
(205) 934-9411 Telephone
(205) 934-9488 Fax
[email protected]
529
530
Genomic Medicine
Bruce R. Korf, MD, PhD
Professor and Chair, Department of Genetics
University of Alabama at Birmingham
Disclosure(s)
Relationship
Entity
Grant Recipient
Novartis
Advisory Board
Accolade, Genome Medical
Board of Directors
American College of Medical Genetics and Genomics
Children’s Tumor Foundation
Advisor
Neurofibromatosis Therapeutic Acceleration Project
Founding Member
Envision Genomics
Salary
University of Alabama at Birmingham
Outline
• Pharmacogenetics
• Genetic Risk Assessment
• Genome Sequencing
531
Individuality
Experience
Culture
Environment
Ancestry
• Risk of disease
• Response to treatment
• Side effects
Pharmacogenetics and Personalized Medicine
Administer the right drug at
the right dose to the right
person at the right time.
Drug Metabolism
• Absorption
• GI
• Tissue spaces
Phase I
• Metabolism
• Activation
• Inactivation
• Excretion
• Kidney
• Liver
Phase II
532
Pharmacogenetic Nomenclature: Diplotypes
A1
A2
B1
B2
Haplotypes: A1-B1 or A2-B2
Diplotype: A1-B1/A2-B2
Star System – most common
diplotype is *1; other
diplotypes *2, *3, etc.
Cytochrome Oxidases
• CYP2D6 (debrisoquine
hydroxylase) – poor,
intermediate, extensive,
ultrarapid metabolizers
• CYP2C9 – Warfarin metabolism
• CYP2C19 – clopidigrel prodrug
• CYP3A4
• CYP3A5
Class
Examples
Analgesics
Codeine, hydrocodone, tramadol
Antiarrhythmics
Encainide, mexiletine, propafenone
Antidepressants
Amitriptyline, desipramine, fluoxetine,
fluvoxamine, imipramine, nortriptyline,
paroxetine
Antihistamines
Chlorpheniramine, diphenhydramine,
promethazine
Antipsychotics
Haloperidol, thioridazine
Beta Blockers
Carvedilol, metoprolol, propranolol, timolol
Cough suppressants
Codeine, dextromethorphan
CYP2D6
excretion
Warfarin
7-hydroxywarfarin
CYP2C9
polymorphisms
S-Warfarin
NADH
NAD+
VKORC1
Vitamin K
oxidized
Vitamin K
reduced
activated clotting factors
533
Clopidigrel
Clopidigrel
prodrug
Cyp2C19
Clopidigrel
Active form
Thiopurine S-methyltransferase
• S-methylation of heterocyclic
sulfhydryls
• 6-mercaptopurine, 6-thioguanine
• Leukemia, autoimmune diseases
• TPMT involved in inactivation of drugs
• 10% subjects have intermediate
activity, 0.3% low activity
• Low or intermediate activity
associated with increased toxicity and
requires lowering of dose
N-acetyltransferase 2
• Transfer of acetyl groups to amine and
hydrazine substrates
p-Aminosalicylic acid
• Slow acetylators
• ~50% population
• Homozygous for allelic variants
• Increased toxicity from isoniazid
(tuberculosis therapy), hydralazine
(hypertension therapy), and other
drugs (chemotherapeutics,
clonazepam, nitrazepam,
procainamide, sulfasalazine)
Acetyl
CoA
p-Aminoacetylsalicylic acid
N-acetyltransferase
CoA
534
Drug Transporters
• Efflux of drugs and toxins from cells
• Role in chemotherapeutic drug transport
• ABCB1, ABCC1, ABCC2, ABCG2
• OATPs – membrane-bound influx transporters
• Antibiotics, cardiac glycosides, chemotherapeutic drugs
• OATP1 – SLCO1B1*5 – statin induced myopathy
• OCTs
• SLC22A gene family
• Renal proximal tubule
• Metformin, procainamide
HLA Variants
• HLAB*1502 – Stevens-Johnson syndrome (carbamazepine)
• HLAB*5701 – Hypersensitivity to abacavir
Malignant Hyperthermia
• Sustained muscle contraction, hyperthermia
• Triggered by halogenated anesthetics with
succinylcholine
• Autosomal dominant inheritance
• Genes
• RYR1 (ryanodine receptor) -70%
• Also myopathies
• CACNA1S (calcium channel) – 1% also
hypokalemic periodic paralysis
• others
535
Pharmocogenetics Knowledgebase
Clinical Pharmacogenetics Implementation Consortium (CPIC) – PharmGKB and
Pharmacogenetics Research Network – peer reviewed guidelines
Choice of Therapy
• Matching the treatment to pathophysiology of disease
• Genetic variants that predict response
• Patterns of gene expression that reveal disease subtypes
• Examples:
• Herceptin and EGFR amplification
• Erlotinib or gefinitib and EGFR mutation
• Vemurafinib and V600E in melanoma
Cancer Genomes
Normal
Tumor
Sequence
Difference =
cancer-speciÀc genetic
changes
536
Treatment of Genetic Disease
ivacaftor
Potentiator
Corrector
Everolimus Treatment in TSC
hamartin
tuberin
Rheb GDP
mTOR
SEGA
Everolimus
Renal AML
growth & proliferation
Molecular Therapy
Normal
Stop Mutation
splice switching oligomers
Aminoglycoside
Readthrough
exon skip
537
Genome Editing
Non-homologous
end joining
Mutation at site of
repair
Homology-directed
repair
Repair with
Donor DNA
Clinical Trials
Drug Discovery
538
The Diagnostic Odyssey
Clinical
Problem
History &
Physical
Interpretation
Genetic
Testing
Differential
Diagnosis
Exome –Capture Technology
(2007)
Exon 1
Exon 2
Exon 3 Exon 4
Exon 5
Fragment and anneal to
oligo probes
Elute
HTS
Analyze
27
539
Analysis of Genomic Data
Evidence Levels
• Pathogenic
•
•
•
•
PVS1 – very strong
PS1-4 – strong
PM1-6 - moderate
PP1-5 – supporting
• Benign
• BA1 – stand alone
• BS1-4 – strong
• BP1-6 – supporting
540
Diversity of category of germline
results that can be found WES clinical
reports
Gene
Mutation
Example
Related to
Patient’s Phenotype
Other
Medically
Actionable
Pathogenic
Pathogenic Pathogenic
ARID1A
VUS
Rare MECP2
missense
SCN5A
mut
mtDNA
MELAS
mut
PCG
Genes
Recessive
Carrier
Genes
FDA
Indication
Pathogenic
CYP2A
mut
CFTR
ΔF508
Modified from Scollon et al., Genome Medicine, 2014
541
Human Phenotype Ontology
Yang et al., :D͕2014 – Description
of 2000 proband only WES clinical
cases
• 1780 (89%) are pediatric
patients (<18 yrs)
• 1440 (72%) have intellectual
disability, seizure disorder or
autism
• Diagnostic rate ~25% for
patients referred for
diagnostic WES
542
Molecular Diagnosis Rate 25%:
Varies for Different Phenotypic Groups
Overall (n=2000)
Non-neurologic (n=244)
Neurologic plus (n=1147)
Neurologic only (n=526)
Specific Neurologic (n=83)
0%
10%
20%
30%
40%
50%
Diagnostic rate (+/- 95% CI)
Yang, et al., JAMA, 2014
Mutations in Positive WES Cases
1.1% 0.8%
0.7% 0.1%
0.1%
0.1%
missense
frameshift
8.1%
nonsense
splice
18.9%
48.9%
in-frame
large deletions
start codon
stoploss
21.0%
promoter region
mitochondrial
708 Mutant Alleles in the 504 Positives, 409 (58%) novel at
time of reporting
Most AD Mutant Alleles Arose de Novo
(AD: 74%; XL: 62%)
X-LINKED, 13%
Mito, 0.2%
de novo,
74%
AD, 53%
AR, 36%
unknown, 14%
inherited, 11%
543
Genomic Newborn Screening
Genomic Prenatal Diagnosis
Genomic Carrier Screening
544
Incidental Findings
• Constitutional mutations on minimal list should be reported
regardless of age of patient
• Laboratories should seek and report specific types of
mutations on list
• Ordering clinician responsible for pre- and post-test
counseling
• Patients may opt out of learning
about incidental findings
ACMG Secondary Findings
Type
Genes
Tumor Predisposition
Breast/ovarian, Li-Fraumeni, Peutz-Jeghers, Lynch, Polyposis, Von HippelLindau, MEN1/2, Medullary thyroid cancer, PTEN hamartoma syndrome,
Retinoblastoma, Paraganglioma/pheochromocytoma, Tuberous sclerosis
complex, WT1-related Wilms’ tumor, NF2
BRCA1/2, TP53, STK11, MLH1, MSH2,
MSH6, PMS2, APC, MUTYH, BMPR1A,
SMAD4, VHL, MEN1 RET, PTEN, RB1,
SDHD, SDHAF2, SDHC, SDHB, TSC1,
TSC2, WT1, NF2
Connective Tissue Dysplasia
Ehlers-Danlos vascular type, Marfan, Loeys-Dietz, Familial aortic
aneurysms and dissections
COL3A1, FBN1, TGFBR1, TGFBR2,
SMAD3, ACTA2, MYH11
Cardiac
Hypertrophic cardiomyopathy, dilated cardiomyopathy, Arrhythmia
MYBPC3, MYH7, TNNT2, TNNI3, TPM1,
MYL3, ACTC1, PRKAG2, GLA, MYL2,
LMNA, RYR2, PKP2, DSP, DSC2,
TMEM43, DSG2, KCNQ1, KCNH2, SCN5A
Metabolic
Hypercholesterolemia, Wilson disease, Ornithine transcarbamylase
deficiency
LDLR, APOB, PCSK9, ATP7B, OTC
Malignant Hyperthermia
RYR1, CACNA1S
Personal Genomes
Carrier Status
Mendelian
Disorder
Pharmacogenetics
Risk Assessment
545
WGS Workflow
546
Syndromes Every
Geneticist Shoud Know
SYNDROMES EVERY
GENETICIST SHOULD
KNOW
Originally prepared by Amy Roberts, MD, FACMG
Boston Children’s Hospital
Updated by Bruce R. Korf, MD, PhD, FACMG
University of Alabama at Birmingham
549
550
Syndromes Every Geneticist Should Know
Originally prepared by Amy Roberts, MD, Boston
Children’s Hospital; Updated by Bruce R. Korf, MD, PhD,
University of Alabama at Birmingham
22q11 DELETION SYNDROME
CARDIOVASCULAR SYSTEM
(DiGeorge, Velocardiofacial syndrome, Shprintzen syndrome)
Responsible genes: TBX1
Proteins: T box 1 transcription factor C
Cytogenetic locus: 22q11.2
Inheritance: AD; 93% de novo
Clinical Features and Diagnostic Criteria: congenital heart disease (74%) (TOF,
IAA B, conotruncal defects), immune dysfunction, palate abnormalities (69%), feeding
problems, developmental delay, learning problems (70-90%), hypocalcemia (50%),
renal anomalies (37%), psychiatric disorders, medial deviation of the internal carotids
Clinical Tests: serum Ca, PTH, T/B Cell subsets, Ig’s, post vaccine Ab’s, renal US,
video laryngoscopy
Molecular Tests: FISH, microarray, or MLPA for DGCR deletion (95%). 3-Mb
deletion most common; no clear genotype-phenotype relationship to del size. (A small
% with S/Sx 22q11 del without a DGCR deletion have 10p13-p14 deletion)
Disease Mechanism: Abnormal development of the pharyngeal arches related to
TBX1 dosage
Treatment/Prognosis: Standard Tx for CHD, palate repair, pharyngeal flap, Ca
replacement, no live vaccines if immunodeficient
22q11 DELETION SYNDROME
CARDIOVASCULAR SYSTEM
(DiGeorge, Velocardiofacial syndrome, Shprintzen syndrome)
(Lin et al, Genet in Med, 2009)
3
551
CARDIOVASCULAR SYSTEM
ALAGILLE SYNDROME
Responsible genes: JAG1, NOTCH2
Proteins: Jagged 1, Neurogenic locus notch homolog protein 2
Cytogenetic locus (loci): 20p12, 1p13-p11
Inheritance: AD, 50-70% de novo
Clinical Features and Diagnostic Criteria: Dx: Bile duct paucity on liver bx + any
three of: cardiac defects (most often PA disease, TOF), cholestasis, skeletal
abnormalities (butterfly vertebrae), eye (posterior embryotoxin), or characteristic
facial features. Also developmental delay, growth failure
Clinical Tests: Bile duct paucity on liver bx,
Molecular Tests: seq JAG1 (>89%), JAG1 20p12 del FISH (~7%), NOTCH2 seq
(1-2%)
Disease Mechanism: JAG1:Truncated protein product rendering it unable to bind
to the cell membrane resulting in functional haploinsufficiency
Treatment/Prognosis: Liver transplant, cardiac and renal anomalies treated in
standard manner, evaluate head injuries and CNS symptoms for vascular
accidents, fat soluble vitamins, monitor growth and development,
4
ALAGILLE SYNDROME
CARDIOVASCULAR SYSTEM
HE staining of liver specimen showing
paucity of the interlobular ducts
(www.cmj.org/Periodical/PaperList.asp?id=LW200)
Facial Features:
Prominent forehead
Deep-set eyes with moderate hypertelorism
Pointed chin
Saddle or straight nose with a bulbous tip
(http://www.icampus.ucl.ac.be/PEDIHEPA/)
5
BRUGADA SYNDROME
Responsible gene: SCN5A
CARDIOVASCULAR SYSTEM
(Pathogenic variants in 22 other genes : ABCC9, CACNA1C, CACNA2D1, CACNB2, FGF12, GPD1L, HCN4, KCND2,
KCND3, KCNE5, KCNE3, KCNH2, KCNJ8, PKP2, RANGRF, SCN1B, SCN2B, SCN3B, SCN10A, SEMA3A, SLMAP, and TRPM4, each <1%)
Protein: Sodium channel protein type 5 subunit alpha
Cytogenetic locus: 3p21 (SCN5A)
Inheritance: AD (except KCNE5 XLR)
Clinical Features and Diagnostic Criteria: Syncope or nocturnal agonal respiration. STsegment abnormalities in leads V1-V3 on the ECG and a high risk of ventricular
arrhythmias and sudden death. Manifests primarily during adulthood (range 2 days to 85
yrs). Mean age of sudden death: 40 yrs. May present as SIDS or the sudden unexpected
nocturnal death syndrome (a typical presentation in individuals from Southeast Asia). May
have FH sudden cardiac death.
Clinical Tests: ECG
Molecular Tests: SCN5A scanning/seq (15-30% of cases)
Disease Mechanism: Gene mutations cause lack of expression of or acceleration in the
inactivation of cardiac sodium channels.
Treatment/Prognosis: Implantable defibrillators, isoproterenol, avoid inducing
medication (vagotonic agents, alpha adrenergic antagonists, beta adrenergic antagonists,
TCA, first generation antihistamines, cocaine, class 1C antiarrhythmic drugs, class 1A
agents (procainamide, disopyramide)
6
552
CARDIOVASCULAR SYSTEM
BRUGADA SYNDROME
7
CARDIOVASCULAR SYSTEM
CARDIO-FACIO-CUTANEOUS
SYNDROME
Responsible genes: BRAF, MAP2K1, MAP2K2, KRAS
Proteins: B-Raf proto-oncogene serine/threonine-protein-kinase, Dual specificity mitogenactivated protein kinase 1 and 2, GTPase KRas
Cytogenetic loci: 7q34, 15q22.31,19p13.3, 12p12.1
Inheritance: AD (majority de novo)
Clinical Features and Diagnostic Criteria: Cardiac abnormalities (pulmonic stenosis,
septal defects, hypertrophic cardiomyopathy, arrhythmia), distinctive facial features, and
cutaneous abnormalities (xerosis, hyperkeratosis, ichthyosis, eczema, ulerythema
ophyrogenes), mild-moderate intellectual disability, neoplasia in some, most often ALL
Clinical Tests: echocardiogram, renal ultrasound, cognitive testing
Molecular Tests: gene sequencing
Disease Mechanism: sustained activation of the Ras MAPK pathway downstream
effectors: MEK and/or ERK
Treatment/Prognosis: Standard cardiac care, dermatology consultation, early intervention
and individualized education plans
8
CARDIO-FACIO-CUTANEOUS
SYNDROME
•High forehead with bitemporal
constriction
CARDIOVASCULAR SYSTEM
•Posteriorly rotated ears with think
helices
•Hypertelorism with down slanting
palpebral fissures
•Epicanthal folds and ptosis
•Depressed nasal bridge with anteverted
nares
•Highly arched palate
•Cupids Bow Lips
•More coarse features and more
dolichocephaly than Noonan syndrome
www.cfcsyndrome.org
9
553
CARDIOVASCULAR SYSTEM
COSTELLO SYNDROME
Responsible genes: HRAS
Proteins: GTPase HRas
Cytogenetic loci: 11p15.5
Inheritance: AD (majority de novo)
Clinical Features and Diagnostic Criteria: feeding issues, developmental delay,
intellectual disability, coarse facial features, loose, soft skin, hypertrophic
cardiomyopathy, pulmonary stenosis, arrhythmia, high rate of cancer (bladder
cancer, rhabdomyosarcoma and neuroblastoma)
Clinical Tests: echocardiogram, neurocognitive testing
Molecular Tests: gene sequencing
Disease Mechanism: Missense mutations lead to constitutive activation of the
abnormal protein product resulting in increased signaling through the Ras MAP
Kinase pathway
Treatment/Prognosis: Standard cardiac care, dermatology consultation, early
intervention and individualized education plans, may require assisted feeding
(nasogastric
or gastric tube), cancer screeing
10
COSTELLO SYNDROME
CARDIOVASCULAR SYSTEM
Coarse facial features (full
lips, large mouth, full nasal
tip)
Curly or sparse, fine hair
Loose, soft skin with deep
palmar and plantar creases
Papillomata of the face and
perianal region
Diffuse hypotonia and joint
laxity with ulnar deviation of
the wrists and fingers
costellosyndromeusa.org
Cancer risk
11
CARDIOVASCULAR SYSTEM
HEREDITARY HEMORRHAGIC
TELANGIECTASIA
Responsible genes: ACVRL1, ENG, GDF2, SMAD4
Proteins: Serine/threonine-protein kinase receptor R3, Endoglin, Growth/differentiation
factor 2, Mothers against decapentaplegic homolog 4
Cytogenetic loci: 12q11-q14, 9q34.1, 10q11.22,18q21.2 Inheritance: AD
Clinical Features and Diagnostic Criteria: nosebleeds, mucocutaneous
telangiectases (lips, oral cavity, fingers, and nose), visceral AV malformation (pulmonary,
cerebral, hepatic, spinal, gastrointestinal). Hemorrhage is often the presenting symptom
of cerebral AVM. Most visceral AVM’s present as a result of blood shunting through the
abnormal vessel and bypassing the capillary beds.
Clinical Tests: Stool for occult blood, CBC (anemia or polycythemia), contrast echo to
find pulmonary AVM, Head MRI for cerebral AVM, US for hepatic AVM,
Molecular Tests: Sequence analysis ACVRL1, ENG (60-80%), duplication/deletion
analysis (10%); SMAD4 in patients with juvenile polyposis
Disease Mechanism: HHT is assumed to be the result of haploinsifficiency
Treatment/Prognosis: Transcatherter embolization of pulmonary AVM >3.0mm. OCP
can decrease/eliminate bleeding. Liver transplant if hepatic AVM is causing heart
failure.
12
554
HEREDITARY HEMORRHAGIC
TELANGIECTASIA
CARDIOVASCULAR SYSTEM
Mucocutaneous telangiectases
13
CARDIOVASCULAR SYSTEM
HOLT-ORAM SYNDROME
Responsible gene: TBX5, SALL4 (related disorder)
Protein: T-box transcription factor TBX5, Sal-like protein 4
Cytogenetic loci: 12q24.1, 20q13.2 Inheritance: AD (85% de novo)
Clinical Features and Diagnostic Criteria: Malformation of the carpal bone(s) and,
variably, the radial and/or thenar bones (left often more severe than right). 100% have
carpal bone abnormality. 75% have CHD, most often multiple ASD or VSD, arrhythmia
(even if no CHD)
Clinical Tests: hand xray
Molecular Tests: TBX5 sequencing (>70%), Del/Dupl analysis (<1%). Rarely due to
SALL4 mutations resulting in similar syndrome
Disease Mechanism: The TBX5 protein product is a transcription factor with an important
role in both cardiogenesis and limb development. TBX5 mutations lead to mutant TBX5
mRNAs that are rapidly degraded or to transcripts with diminished DNA binding- both of
which result in decreased gene dosage.
Treatment/Prognosis: Pacemaker if severe heart block, standard cardiac surgery,
pollicization may be indicated if thumb aplasia/hypoplasia. Annual ECG, annual Holter if
h/o abnormal ECG
14
CARDIOVASCULAR SYSTEM
HOLT-ORAM SYNDROME
Thumb anomaly
http://www.emedicine.com/ped/images/1038ASDbj.jpg
15
555
CARDIOVASCULAR SYSTEM
Noonan syndrome with Multiple Lentigines
(NSML) formerly known as LEOPARD Syndrome
Responsible gene: PTPN11, RAF1, BRAF, MAP2K1 Protein: SHP2 , RAF
proto-oncogene serine/threonine-protein kinase, B-Raf, mitogen-activated
protein kinase 1
Cytogenetic locus: 12q24, 3p25
Inheritance: AD
Clinical Features and Diagnostic Criteria: Lentigines, Electrocardiographic
conduction abnormalities, Ocular hypertelorism, Pulmonary stenosis,
Abnormalities of the genitalia, Retardation of growth, sensorineural
Deafness. Hypertrophic cardiomyopathy in majority
Clinical Tests: Audiogram, ECG, echocardiogram
Molecular Tests: PTPN11 sequencing (90%), RAF1 (<5%), others rare
Disease Mechanism: Loss of function PTPN11 mutations (versus Noonan
syndrome PTPN11 mutations which are gain of function)
Treatment/Prognosis: Treat cardiac defects, deafness
16
CARDIOVASCULAR SYSTEM
Noonan syndrome with Multiple Lentigines
(NS-ML) formerly known as LEOPARD Syndrome
Facial Features:
-Hypertelorism
-Down slanting palpebral
fissures
-Low set ears
-Multiple lentigines
(Sarkozy EJHG 2004)
NOONAN SYNDROME
CARDIOVASCULAR SYSTEM
Responsible genes: PTPN11, SOS1, KRAS, RAF1, NRAS, CBL, SHOC2, BRAF, RIT1,
SOS2, MAP2K1
Proteins: SHP2, Son of sevenless homolog 1, GTPase KRAS, RAF proto-oncogene
serine/threonine-protein kinase, NRAS, CBL, SHOC2, B-raf proto-oncogene serine/threonineprotein kinase, Ric-like protein w/o CAAX Motif 1, Son of sevenless homolog 2
Cytogenetic loci: 12q24.1, 2p22-p21, 12p12.1, 3p25, 1p13.2, 11q23.3, 10q25, 7q35,
1q22, 14q21.3 Inheritance: AD
Clinical Features and Diagnostic Criteria: Characteristic facial features, short stature,
feeding problems, pulmonary valve stenosis, HCM (RAF1,RIT1 most enriched),
cryptorchidism, renal malformation, lymphedema, bleeding disorders, myeloproliferative
disorder, incl. risk of leukemia and learning disabilities
Clinical Tests: Echocardiogram, renal ultrasound, bleeding studies
Molecular Tests: PTPN11 sequencing (50%), SOS1 sequencing (10%), RAF1, RIT1
(10% each), SHOC2 (2%), KRAS, SOS2 (1% each), NRAS/CBL/BRAF/MAP2K1 (<1%)
Disease Mechanism: Gain of function mutations that lead to constitutive activation of the
Ras MAP Kinase pathway
Treatment/Prognosis: Standard cardiac care, orchiopexy, early intervention, GH
replacement
18
556
NOONAN SYNDROME
GENE
PTPN11
CARDIAC
>PS
CARDIOVASCULAR SYSTEM
<HCM
SOS1/SOS2
GROWTH
>Short Stature
>Lower IGF-1
levels
DEVELOPMENT
SKIN/HAIR
OTHER
N308D/S mild
or no dev. delay
Verbal/NV
IQ<SOS1
<Short Stature
Verbal/NV IQ>
PTPN11
<ID
CFC-like
RAF1
RIT1
>HCM
>HCM
<Short Stature
SHOC2
MVP
>Short Stature
Inc. pigment
Hyperactivity
ASD/VSD
>GH Deficiency
Ichthyosis
Hypernasal
voice
Eczema
KRAS
More severe
delays?
NRAS
CBL
Enlarged LA
Ventricular
dysrhythmia
Mitral
Insufficiency
Delayed Brain
Myelination
Sparse, fragile,
thin hair
CFC-like
More severe
medical
problems?
Pre-disposition
to JMML
Cerebellar
Vermis
Hypoplasia
19
CARDIOVASCULAR SYSTEM
WILLIAMS SYNDROME
Responsible gene: Contiguous gene deletion syndrome, ELN in the critical region
Protein: Elastin
Cytogenetic locus: 7q11.23
Inheritance: AD, majority of cases de novo
Clinical Features and Diagnostic Criteria: CV any artery may be narrowed, supravalvar
aortic stenosis (SVAS) most common (75%). Distinctive facial features. CT: hoarse voice,
hernia, rectal prolapse, joint limitation or laxity. ID. Overfriendly, anxiety d/o, ADD. Endo:
hypercalcemia, hypercalciuria, hypothyroidism, FTT infancy
Clinical Tests: Serum and urine calcium and creatinine, TFTs, hearing and vision
evaluation, renal US, echocardiogram
Molecular Tests: FISH, microarray, or MLPA for 7q11.23 critical region (~99%). Point
mutations in ELN cause AD isolated SVAS
Disease Mechanism: Elastin deletion causes the CV and CT problems, LIMK1 has been
linked to the visuospatial construction cognitive deficit
Treatment/Prognosis: PT, OT, ST. Monitor adults who are at risk for MVP, AI, arterial
stenosis, SNHL, hypothyroidism, DM. Monitor for hypercalciuria. Aggressive
management of constipation
20
CARDIOVASCULAR SYSTEM
WILLIAMS SYNDROME
(www.thefencingpost.com/mary/images/intheairtiny.JPG)
(www.wehi.edu.au/media/images/williamskid.jpg)
Facial features: Broad brow, bitemporal narrowness, periorbital fullness, a
stellate/lacy iris pattern, strabismus, short nose, full nasal tip, malar
hypoplasia, long philtrum, full lips, wide mouth, malocclusion, small jaw, and
prominent earlobes
21
557
CHROMOSOME BREAKAGE DISORDERS
ATAXIA-TELANGIECTASIA
Responsible gene: ATM
Protein: Serine-protein kinase ATM
Cytogenetic locus: 11q22.3
Inheritance: AR (carriers have increased risk of breast, colon and pancreatic)
Clinical Features and Diagnostic Criteria: Progressive cerebellar ataxia (onset age
1-4y), oculomotor apraxia, conjunctival telangiectasia, immunodef, choreoathetosis,
ionizing radiation sensitivity, risk cancer (lymphoma and leukemia)
Clinical Tests: AFP, decreased ATM kinase activity, 7;14 translocation (5-15% of
lymphocytes after PHA stimulation)
Molecular Tests: ATM sequencing (>95%). Amish founder mutation
Disease Mechanism: Most mutations are null leading to no protein product. The
normal protein finds double strand dsDNA breaks and coordinates cell cycle
checkpoints prior to repair
Treatment/Prognosis: IVIG if immunodeficient, PT to reduce contractures, wheelchair
usually by age 10, supportive therapy for drooling, choreoathetosis, and ataxia. Avoid
ionizing radiation. Regular medical visits to monitor for S/Sx of malignancy
22
CHROMOSOME BREAKAGE DISORDERS
ATAXIA-TELANGIECTASIA
23
(http://www.nature.com/embor/journal/v5/n8/images/7400210-f1.jpg)
CHROMOSOME BREAKAGE DISORDERS
BLOOM SYNDROME
Responsible gene: BLM
Protein: Bloom syndrome protein
Cytogenetic locus: 15q26.1
Inheritance: AR (1/100 carrier freq in Ashkenazi Jewish)
Clinical Features and Diagnostic Criteria: IUGR, hyper and
hypopigmentation, butterfly distribution sun sensitive telangiectasia,
microcephaly, high pitched voice, normal intelligence, immunodeficiency,
azoospermia, POF, increased risk of cancer (wide distribution of type and
site (colon most common), often multiple primary tumors).
Clinical Tests: Chromatid/chromosome breaks; triradial and quadriradial
figures
Molecular Tests: BLM 2881 del6ins7 (97% mutant allele in AJ)
Disease Mechanism: Abnormal DNA replication and repair leading to
genomic instability.
Treatment/Prognosis: Increased cancer surveillance, decrease exposure
to UV light
and x-ray, BMT, colon cancer surveillance
24
558
CHROMOSOME BREAKAGE DISORDERS
BLOOM SYNDROME
Butterfly
distribution sun
sensitive
telangiectasia
(www.medicalrealm.net)
25
CHROMOSOME BREAKAGE DISORDERS
FANCONI ANEMIA
Responsible genes (Protein and Cytogenetic locus): FANCA, FANCB, FANCC, FANCD2,
FANCE, FANCF, FANCG (Fanconi anemia group A, B, C, D2, E, F, and G protein; 16q24.3,
Xp22.3, 9q22.3, 3p25.3, 6p22-21, 11p15, and 9p13); FA-D1 - BRCA2 (Breast cancer type 2
susceptibility protein, 13q12.3); BRIP1 (Fanconi anemia group J protein, 17q22); FANCL (E3
ubiquitin-protein ligase FANCL).
Inheritance: AR, AD (RAD51), XLR – FA-B
Clinical Features and Diagnostic Criteria: Short stature; abnl pigmentation; radial, GU, ear,
heart, GI, or CNS malformation; hearing loss, hypogonadism, developmental delay. Progressive
bone marrow failure, aplastic anemia, myelodysplastic syndrome, AML, solid tumor of head,
neck, esophagus, cervix, vulva, or liver at unusually young age.
Clinical Tests: DEB or MMC induced chromosome breakage, macrocytic rbcs, immunoblot
assay of FANCD2 monoubiquitination, increased % of cells in G2 arrest by cell sorting.
Molecular Tests: Seq and Del/Dup analysis FANCA (66%), Seq analysis FANCB (0.8%),
FANCC (9.6%), FANCD1, FANCD2, FANCE, FANCF (~3% each), FANCG (8.8%), FANCL (0.4%)
and BRCA2. Numerous additional subtypes with only a few patients described.
Disease Mechanism: >5 of the FA proteins are assembled in a nuclear complex. In response to
DNA damage, this complex activates monoubiquitination of FANCD2 and FANCI protein with
subsequent FA proteins involved in direct DNA repair.
Treatment/Prognosis: Androgens, blood transfusions, growth hormone, BMT, cancer prevention
(avoid toxic agents and sun exposure), cancer surveillance
26
CHROMOSOME BREAKAGE DISORDERS
FANCONI ANEMIA
Preaxial polydactyly
(radswiki.net)
27
559
CONNECTIVE TISSUE DISORDERS
CONGENITAL CONTRACTURAL
ARACHNODACTYLY (Beals Syndrome)
Responsible gene: FBN2
Protein: Fibrillin-2
Cytogenetic locus: 5q23-q31
Inheritance: AD
Clinical Features and Diagnostic Criteria: Marfanoid appearance, long slender fingers
and toes, crumpled ears, major joint contracture, muscle hypoplasia, kyphosis/scoliosis,
Severe/lethal: aortic dilation, ASD, VSD, IAA, duodenal or esophageal atresia,
malrotation
Clinical Tests: x-ray, echocardiogram, UGI with SBFT
Molecular Tests: FBN2 sequencing (75%)
Disease Mechanism: Fibrillin 2 is a glycoprotein of the extracellular matrix microfibrils, it
is co-distributed with fibrillin 1 in many tissues. The precise function is not known.
Treatment/Prognosis: PT for joint contracture, contracture surgical release, bracing
and/or surgical correction of kyphoscoliosis. Echo every 2 years until it is clear the aorta
is not involved. Annual exam for scoliosis/kyphosis.
28
CONNECTIVE TISSUE DISORDERS
CONGENITAL CONTRACTURAL
ARACHNODACTYLY (Beals Syndrome)
Crumpled prominent ears
Major joint contracture
(www.scielo.br)
(www.indianpediatrics.net)
29
CONNECTIVE TISSUE DISORDERS
EHLERS-DANLOS SYNDROME
CLASSIC TYPE (Type I and Type II)
Responsible genes: COL5A1 and COL5A2
Proteins: Collagen alpha-1 and alpha-2 (V) chain
Cytogenetic loci: 9334.2-q34.3 and 2q31
Inheritance: AD
Clinical Features and Diagnostic Criteria: skin hyperextensibility, widened atrophic scars, joint
hypermobility, smooth velvety skin, molluscoid pseudotumors (heaped up scar-like lesions over
pressure points), subcutaneous spheroids (cyst-like lesions, feel like grains of rice, over bony
prominences of legs and arms, they are fibrosed and calcified fat globules), joint
sprains/dislocations/subluxations, hypotonia, easy bruising, hernia, chronic pain, aortic root
dilation
Clinical Tests: Ultrastructural studies by EM suggest disturbed collagen fibrillogenesis
(“cauliflower” deformity is characteristic).
Molecular Tests: COL5A1 “null” allele testing on cultured fibroblasts (30%), COL5A1 and
COL5A2 sequencing (50%)
Disease Mechanism: Dominant negative activity of abnormal Collagen alpha-1 or alpha-2 (V)
chains (interfere with the utilization of normal protein from the normal allele)
Treatment/Prognosis: PT, non-weight-bearing muscular exercise, dermal wounds repaired with
two layer closure without tension, if possible avoid joint surgery , baseline echo for aortic root
assessment
30
560
EHLERS-DANLOS SYNDROME
CLASSIC TYPE (Type I and Type II)
CONNECTIVE TISSUE DISORDERS
Skin hyperextensibility
Joint hypermobility
(www.meddean.luc.edu )
(www.nlm.nih.gov)
31
CONNECTIVE TISSUE DISORDERS
EHLERS-DANLOS SYNDROME,
HYPERMOBILITY TYPE (Type III)
Responsible gene: unknown Protein: Unknown
Cytogenetic locus: 6p21.3
Inheritance: AD
Clinical Features and Diagnostic Criteria: Joint hypermobility, soft or velvety skin with
normal or slightly increased elasticity, absence of skin or soft tissue fragility, recurrent joint
dislocation/subluxation, chronic joint or limb pain, easy bruising, high narrow palate, dental
crowding, and low bone density. Kids less than age 5 are often very flexible and therefore
are hard to assess. Reported instances of aortic root dilation and MVP.
Clinical Tests: The biochemical etiology is unknown in most cases.
Molecular Tests: Not done (rare cases of Tenascin-X deficiency)
Disease Mechanism: Abnormal dermal elastic fibers
Treatment/Prognosis: Improve joint stability with low-resistance exercise to inc muscle
tone, avoid joint hyperextension, avoid high impact sports, wide writing utensils to avoid
strain on finger and hand joints, joint bracing, pain management specialist, delay joint
surgery in favor of PT and bracing. Baseline echocardiogram
32
CONNECTIVE TISSUE DISORDERS
EHLERS-DANLOS SYNDROME,
HYPERMOBILITY TYPE (Type III)
Beighton Hypermobility
Score
(www.physiopro.co.za/wp-content/uploads/2012/09/beightonscale.png)
33
561
CONNECTIVE TISSUE DISORDERS
EHLERS-DANLOS SYNDROME,
VASCULAR TYPE (Type IV)
Responsible gene: COL3A1
Protein: Collagen alpha-1 (III) chain
Cytogenetic locus: 2q31
Inheritance: AD
Clinical Features and Diagnostic Criteria: Major criteria: arterial rupture, intestinal rupture,
uterine rupture during pregnancy, FH of Vascular EDS. Minor criteria: thin, translucent skin, easy
bruising, thin lips and philtrum, small chin, thin nose, large eyes, aged appearance of hands,
small joint hypermobility, tendon/muscle rupture, varicose veins, AV carotid-cavernous sinus
fistula, pneumothorax, CHD, clubfoot, gum recession
Clinical Tests: Cultured dermal fibroblasts: amount of type III procollagen synthesized, the
quantity secreted into the medium, and the electrophoretic mobility are assessed (95% of cases
of vascular EDS)
Molecular Tests: cDNA or genomic DNA COL3A1 sequence analysis (98-99%)
Disease Mechanism: Abnormalities of type III procollagen production, intracellular retention,
reduced secretion, and/or altered mobility
Treatment/Prognosis: Minimization of surgical exploration and intervention, prompt surgery
for bowel rupture, distal colectomy if recurrent bowel rupture, high risk obstetrical care. Minimize
lifting and weight training, no contact sports, no arteriograms; investigational treatment celiprolol
34
EHLERS-DANLOS SYNDROME,
VASCULAR TYPE (Type IV)
CONNECTIVE TISSUE DISORDERS
Facial Features:
Thin lips
Thin philtrum
Small chin
Thin nose
Large eyes
Thin skin
with visible
vessels
www.ehlersdanlosnetwork.org
35
CONNECTIVE TISSUE DISORDERS
EHLERS-DANLOS SYNDROME,
KYPHOSCOLIOTIC TYPE (Type VI)
Responsible gene: PLOD1
Protein: Procollagen-lysine,2-oxoglutarate 5-dioxygenase 1
Cytogenetic locus: 1p36.3-p36.2
Inheritance: AR
Clinical Features and Diagnostic Criteria: Major features: friable, hyperextensible skin,
thin scars, easy bruising, generalized joint laxity, severe muscle hypotonia, progressive
scoliosis, scleral fragility and rupture of the globe. Minor features: widened atrophic
scars, marfanoid habitus, rupture of medium sized arteries, mild to moderate delay of
attainment of gross motor milestones
Clinical Tests: Crosslinked telopeptides are excreted in urine as a byproduct of
increased collagen turnover. Inc ratio of deoxypyridinoline to pyridinoline by urine HPLC
is highly sensitive and specific. Enzyme activity in cultured fibroblasts (<25% activity is
abnormal)
Molecular Tests: PLOD1 seq research only, unknown frequency
Disease Mechanism: Enzyme deficiency leads to deficiency in hydroxylysine-based
pyridinoline crosslinks in types I and III collagen.
Treatment/Prognosis: Surgical correction of scoliosis is not contraindicated), PT,
echocardiogram and standard treatment for MVP or aortic root dilation, aggressive control
of BP, routine eye exams
36
562
CONNECTIVE TISSUE DISORDERS
EHLERS-DANLOS SYNDROME,
KYPHOSCOLIOTIC TYPE (Type VI)
Kyphoscoliosis
www.thefetus.net
37
CONNECTIVE TISSUE DISORDERS
LOEYS-DIETZ SYNDROME
Responsible gene: TGFBR1, TGFBR2, SMAD3, TGFB2
Protein:TGF-beta recepor type-1 and type-2, Mothers against decapentaplegic homolog 3,
Transforming growth factor beta-2
Cytogenetic locus: 9q22.33, 3p24.1, 15q22.33, 1q41
Inheritance: AD
Clinical Features and Diagnostic Criteria: vascular findings (cerebral, thoracic, and abdominal arterial
aneurysms and/or dissections) and skeletal manifestations (pectus excavatum or pectus carinatum,
scoliosis, joint laxity, arachnodactyly, talipes equinovarus). 75% have LDS type I with craniofacial
manifestations (ocular hypertelorism, bifid uvula/cleft palate, craniosynostosis); 25% have LDS type II
with cutaneous manifestations (velvety and translucent skin; easy bruising; widened, atrophic scars).
Clinical Tests: Echocardiogram, MRA or CT scan for arterial aneurysm/tortuosity, spinal xrays
Molecular Tests:gene sequencing and del/dup testing
Disease Mechanism: data demonstrate increased TGFɴ signaling in the vasculature of persons with
LDS
Treatment/Prognosis: Regular surveillance imaging of the vasculature, Beta blockers/Losartan for
aortic root dilation, bracing/surgery for scoliosis
38
CONNECTIVE TISSUE DISORDERS
LOEYS-DIETZ SYNDROME
Marked arterial
tortuosity
(Morris SA and Lacro RV Circ 2011)
39
563
CONNECTIVE TISSUE DISORDERS
MARFAN SYNDROME
Responsible gene: FBN1
Protein: Fibrillin 1
Cytogenetic locus: 15q21.1
Inheritance: AD
Clinical Features and Diagnostic Criteria: Major involvement of 2 body systems and minor
involvement of a 3rd. Major Criteria CV: Dilation or dissection of the ascending aorta Skeletal: pectus
carinatum or excavatum, reduced upper:lower segment or arm span:ht, scoliosis, pes planus, high
palate, reduced elbow extension, protrusio acetabulae, Eye: ectopia lentis, Dura: lumbosacral dural
ectasia, FH: pathogenic FBN1 mutation, 1st degree relative with Marfan syndrome. Minor Criteria CV:
MV, MR, dilation of main PA, mitral annulus calcification, dilation or dissection of the descending
thoracic or abdominal aorta ate age <50yrs, Skeletal: moderate pectus excavatum, joint hypermobility,
high palate with crowding of teeth, typical facial features Eye: flat cornea, increased length of globe,
decreased pupillary miosis, Lung: pneumothorax, apical lung blebs, Skin: skin striae, hernia
Clinical Tests: CXR: apical blebs, Echocardiogram: MVP, aortic measurements, CT or MRI: dural
ectasia
Molecular Tests: FBN1 seqencing (70-90%)
Disease Mechanism: Dominant negative effect of mutant forms of fibrillin
Treatment/Prognosis: Beta blockers/Losartan for aortic root dilation, bracing/surgery for scoliosis,
annual dilated eye exam, glasses for myopia
40
CONNECTIVE TISSUE DISORDERS
MARFAN SYNDROME
Positive thumb sign
Ectopia Lentis
Pectus excavatum
www.medstudents.com.br
www.chicagohs.org
www.gfmer.ch
41
DERMATOLOGIC DISORDERS
HIDROTIC ECTODERMAL
DYSPLASIA 2
Responsible gene: GJB6
Protein: Gap junction beta-6 protein (Connexin 30)
Cytogenetic locus: 13q12
Inheritance: AD
Clinical Features and Diagnostic Criteria: malformed, thickened, small
nails; hypotrichosis (partial or total alopecia), palmoplantar
hyperkeratosis, normal sweating and teeth.
Clinical Tests: None
Molecular Tests: Four GJB6 mutations (p.Gly11Arg, p.Ala88Val,
p.Val37Glu, p.Asp50Asn) account for 100% of identified mutant alleles
Disease Mechanism: Helps form a gap junction channel which mediates
ion diffusion. Mutations are thought to affect trafficking of the protein and
thus the formation of the gap junction
Treatment/Prognosis: No specific treatment. Skin emollients for
hyperkeratosis
42
564
DERMATOLOGIC DISORDERS
HIDROTIC ECTODERMAL
DYSPLASIA
Plantar
Hyperkeratosis
vgrd.blogspot.com/2012/09/hidrotic-ectodermal-dysplasia.html
43
DERMATOLOGIC DISORDERS
HYPOHIDROTIC ECTODERMAL
DYSPLASIA
Responsible genes: EDA, EDAR, EDARADD
Proteins: Ectodysplasin-A, Tumor necrosis factor receptor superfamily member EDAR,
ectodysplasin A receptor-associated adapter protein
Cytogenetic loci: Xq12-q13.1, 2q11-q13, 1q42.2-q43
Inheritance: XL (EDA:95%), AD or AR (5%)
Clinical Features and Diagnostic Criteria: At birth: peeling skin and perioral
hyperpigmentation. Hypotrichosis (sparse scalp and body hair), hypohidrosis (inability to
sweat in response to heat leads to hyperthermia), hypodontia (usually only5-7 teeth develop,
teeth are smaller with conical crowns. Carriers of XL HED show mosaic pattern of sweat pore
function and some degree of hypodontia.
Clinical Tests: dental xray.
Molecular Tests: EDA sequencing (~95% XL HED), EDAR and EDARADD sequencing
Disease Mechanism: Defective ectodysplasin A cannot be activated to mediate the cell-tocell signaling that regulates morphogenesis of ectodermal appendages. Defective EDAR
cannot bind ectodysplasin. The protein encoded by EDARADD is co-expressed with EDAR.
Treatment/Prognosis: During hot weather maintain hydration and keep down body temp with
A/C, “cooling vests”, and/or spray bottle of water. Tooth restoration, orthodontics, and/or
dentures may be necessary
44
DERMATOLOGIC DISORDERS
HYPOHIDROTIC ECTODERMAL
DYSPLASIA
Hypodontia
and
abnormally
shaped teeth
www.wsahs.nsw.gov.au
45
565
DERMATOLOGIC DISORDERS
INCONTINENTIA PIGMENTI
Responsible gene: IKBKG (aka NEMO)
Protein: NF-kappaB essential modulator
Cytogenetic locus: Xq28
Inheritance: XLD (most male fetuses miscarry)
Clinical Features and Diagnostic Criteria: Major: Four stages of skin
changes: erythema->blister->hyperpigmented streaks->atrophic skin
patches. Minor: hypo/andontia, small or malformed teeth, alopecia,
woolly hair, nail ridging or pitting, retinal neovascularization causing
retinal detachment. ID is rare.
Clinical Tests: Free melanin granules if hyperpigmented streak biopsied.
Molecular Tests: Southern blot: Exon 4-10 deletion (80%). Skewed X
inactivation in females (not diagnostic).
Disease Mechanism: Lack of NF-kappa beta activation leads to cells
that are sensitive to proapoptotic signals and apopose easily.
Treatment/Prognosis: Regular retinal exams in first 1-2 yrs. Cosmetic
dentistry.
Normal life expectancy.
46
DERMATOLOGIC DISORDERS
INCONTINENTIA PIGMENTI
IP Stage 2:
blistering
www.fi.cnr.it
IP Stage 4: atrophic
patches
www.utmedicalcenter.org
47
DERMATOLOGIC DISORDERS
OCULOCUTANEOUS ALBINISM
Responsible genes: TYR (OCA1), OCA2, TRYP1, SLC45A2, GPR143 (Ocular)
Proteins: Tyrosinase protein, P protein, Membrane-assoc. transporter protein, Gprotein coupled receptor 143
Cytogenetic loci: 11q14-q21 (OCA1), 15q11.2-q12 (OCA2) (in PWS/Angelman region),
5p13.2, Xp22.2 (Ocular)
Inheritance: AR, XLR (GPR143)
Clinical Features and Diagnostic Criteria: OCA1A (no melanin synthesis) nystagmus,
dec iris pigment, foveal hypoplasia, dec visual acuity, strabismus, white hair and skin,
translucent iris. OCA1B (some melanin synthesis) milder eye and skin manifestation
than OCA1. OCA2 ocular problems same as OCA1 but better vision, range of skin and
eye pigmentation from minimal to near normal
Clinical Tests: Skin and eye exam, VEP
Molecular Tests: TYR sequencing (OCA1A: 2 mutations 83%; OCA1B: 2 mutations
37%). 2kb OCA2 deletion testing (most of Sub-Saharin African heritage), SLC45A2 and
GPR143 seq
Disease Mechanism: Lack of melanin production
Treatment/Prognosis: Yearly eye exam, sun screen and monitoring for skin cancer.
48
566
DERMATOLOGIC DISORDERS
OCULOCUTANEOUS ALBINISM
Hair and skin hypopigmentation
http://www.positiveexposure.org/home.html
49
ENDOCRINE SYSTEM
X-LINKED ADRENAL
HYPOPLASIA CONGENITA
Responsible gene: NROB1
Protein: Nuclear receptor 0B1
Cytogenetic locus: Xp21.3-p21.2
Inheritance: X-LR
Clinical Features and Diagnostic Criteria: acute onset adrenal insufficiency
(hyperkalemia, acidosis, hypoglycemia, shock), cryptorchidism, delayed puberty. Carrier
females: may have adrenal insufficiency or hypogonadotropic hypogonadism
1/3 contiguous gene deletion with glycerol kinase, DMD del
2/3 isolated CAH (half are de novo)
Clinical Tests: Dec Na+, Inc K+, acidosis, inc ACTH with low cortisol, dec 17
hydroxyprogesterone. If GKD: serum triglyceride, urine glycerol. If DMD: elevated CK
Molecular Tests: NROB1 FISH deletion (100%)
Disease Mechanism: OB1 is a negative regulator of nuclear receptor pathways
Treatment/Prognosis: Treat adrenal crisis, replacement steroids and stress dosing, HRT
for hypogonadism,
50
ENDOCRINE SYSTEM
21-HYDROXYLASE-DEFICIENT CAH
Responsible gene: CYP21A2 Protein: Cytochrome P450 XXI
Cytogenetic locus: 6p21.3
Inheritance: AR
Clinical Features and Diagnostic Criteria: virilized female, precocious puberty or
adrenarche, childhood virilization in males, infant with Na+ losing crisis at birth. Nonclassic
form: moderate enzyme deficiency with variable postnatal virilization, no salt wasting, but
rare cortisol def.
Clinical Tests: Elevated serum 17-OHD at baseline or after ACTH stim, elevated
testosterone and adrenal androgen precursors in females and prepubertal males. Part of
NBS (17-OHD level)
Molecular Tests: CYP21A2 common mutation and deletion panel detects 80-98%
Disease Mechanism: cortisol production pathway is blocked-> accumulation of 17-OHP>shunted into the intact androgen pathway->17,20-lyase enzyme converts the 17-OHP to –
androstenedione->converted into androgens. The mineralocorticoid pathway requires
minimal 21-hydroxylase activity->salt wasting
Treatment/Prognosis: Hydrocortisone (monitor closely: too little will have excess
androgen, too much causes Cushing;s, skeletal maturation), stress dose steroids
51
567
ENDOCRINE SYSTEM
21-HYDROXYLASE-DEFICIENT CAH
52
ENDOCRINE SYSTEM
ANDROGEN INSENSITIVITY
SYNDROME (Testicular Feminization)
Responsible gene: AR
Protein: Androgen receptor
Cytogenetic locus: Xq11-q12 Inheritance: XLR
Clinical Features and Diagnostic Criteria: Evidence of feminization (i.e.,
undermasculinization) of the ext. genitalia, abnl secondary sexual development, and infertility in
those with a 46,XY karyotype. Spectrum: complete androgen insens. syndrome (CAIS), with typical
female genitalia; partial androgen insens. syndrome (PAIS) with predominantly female,
predominantly male, or ambiguous genitalia; and mild androgen insens. syndrome (MAIS) with nl
male genitalia.
Clinical Tests: impaired spermatogenesis, absent or rudimentary müllerian structures, evidence of
normal or increased synthesis of testosterone and its normal conversion to dihydrotestosterone,
normal or increased LH, and deficient or defective androgen-binding activity of genital skin
fibroblasts
Molecular Tests: AR sequence analysis (>95% CAIS, <50% PAIS, unknown % MAIS)
Disease Mechanism: Impaired androgen binding
Treatment/Prognosis: To prevent testicular malignancy, treatment of CAIS includes either
removal of the testes after puberty when feminization is complete or prepubertal gonadectomy
accompanied by estrogen replacement therapy. Systematic disclosure of the diagnosis of AIS in an
empathic environment
53
ANDROGEN INSENSITIVITY
SYNDROME (Testicular Feminization)
ENDOCRINE SYSTEM
Disorders of Sexual Development (DSD)
(Kim and Kim, Korean J Urol, 2012)
54
568
ENDOCRINE SYSTEM
KALLMAN SYNDROME
TYPE 1 and 2
Responsible genes: KAL, FGFR1
Proteins: Anosmin 1, fibroblast growth factor receptor 1
Cytogenetic loci: Xp22.3, 8p11.1-11.2
Inheritance: XLR, AD
Clinical Features and Diagnostic Criteria: Type 1and 2: hypogonadotropic
hypogonadism and anosmia. Usually present with delayed pubertal development. Type 1
can also include mirror hand movements, ataxia, GU anomaly, high palate, pes cavus.
Type 2 ID, CL/P, cryptorchidism, choanal atresia, CHD, SNHL.
Clinical Tests: Low FSH and LH; low testosterone in males; low estradiol in females. MRI:
hypo/aplasia olfactory bulbs and tracts.
Molecular Tests: Sequencing KAL (5-10%), FGFR1 (8-16%)
Disease Mechanism: Lack of anosmin stops olfactory axons from interecting with their
target. It is thought that FGFR1 may play a role in olfactory bulb formation and possibly
interacts with anosmin
Treatment/Prognosis: Normalize gonadal steroid levels.
55
ENDOCRINE SYSTEM
KALLMAN SYNDROME
TYPE 1 and 2
(humupd.oxfordjournals.org/content/14/4/293/F2.expansion.html)
56
ENDOCRINE SYSTEM
KLINEFELTER SYNDROME
Clinical Features and Diagnostic Criteria: Tall stature, slightly delayed
motor and language skills, inc learning probs, testosterone plateaus age 14,
small fibrosed testes, azoospermia and infertility, gynecomastia, inc
cholesterol, slightly inc risk of autoimmune disorders and mediastinal germ
cell tumors (1% risk). Increased risk of male breast cancer.
Clinical Tests:
Molecular Tests: karyotype, at least one extra chromosome to a 46,XY
Karyotype
Disease Mechanism: 1st or 2nd meiotic division nondisjunction of either
parent. Maternal>paternal origin. +AMA effect
Treatment/Prognosis: Testosterone in mid-late adolescence for bone
density, secondary sex characteristic development, muscle mass, cholesterol,
increase libido, improved energy. Can do testicular biopsy and use any
retrieved sperm for ICSI (inc risk sex chrom abnormality so follow with PGD
57
569
ENDOCRINE SYSTEM
KLINEFELTER SYNDROME
health.yahoo.com/media/healthwise/nr551770
58
ENDOCRINE SYSTEM
MCCUNE-ALBRIGHT SYNDROME
Responsible gene: GNAS
Protein: Guanine nucleotide-binding protein G(s), alpha subunit
Cytogenetic locus: 20q13.2
Inheritance: sporadic
Clinical Features and Diagnostic Criteria: polyostotic fibrous dysplasia, pathologic
fracture, cranial foramina thickening->deafness and blindness, large irregular café au lait
(“coast of Maine”), precocious puberty, hyperthyoidism, inc GH, PRL, or PTH, ovarian
cysts
Clinical Tests: x-ray, pelvic US, vision and hearing testing, pituitary hormone analysis
Molecular Tests: Targeted mutation analysis
Disease Mechanism: Activating mutations (a stimulatory G-protein) leads to persistently
high cAMP (de-activating mutations cause Albright Heredity Osteodystrophy)
Treatment/Prognosis: Aromatase inhibitor to block testosterone, bisphosphonate for
fibrous dysplasia, anti-thyroid meds, octreotide (somatostatin analog) and bromocriptine
(dopamine receptor agonist)
59
MCCUNE-ALBRIGHT SYNDROME
Polyostotic fibrous
dysplasia
ENDOCRINE SYSTEM
Irregular “coast of Maine” café au lait
history.nih.gov
(brighamrad.harvard.edu)
60
570
ENDOCRINE SYSTEM
TRANSIENT NEONATAL DIABETES MELLITUS
Responsible genes: HYMAI, PLAGL1
Proteins: unknown (HYMAI), zinc finger protein PLAG1
Cytogenetic loci: 6q24 (HYMAi and PLAG1)
Inheritance: UPD isodisomy chromosome 6, paternal 6q24 duplication, or 6q24
methylation defect
Clinical Features and Diagnostic Criteria: DM in the first six weeks of life, resolves by 18
months, severe IUGR, dehydration, hyperglycemia. Occassional macroglossia and
umbilical hernia.
Clinical Tests: High serum glucose and low plasma insulin, no islet cell antibodies, no
ketoacidosis. 2% have a visible 6q24 duplication
Molecular Tests: UPD6 (35%), 6q24 duplication (35%), imprinting mutation (20%)
Disease Mechanism: PLAGL1 and HYMAI are normally only expressed on the paternal
allele, unclear why overexpression causes DM. HYMAI may regulate PLAGL1 expression
Treatment/Prognosis: Rehydration, IV insulin and then subcutaneous insulin within two
weeks, close blood glucose monitoring. Inc risk to later develop type II DM during illness,
puberty or during pregnancy
61
ENDOCRINE SYSTEM
TURNER SYNDROME
Responsible genes: X genes that escape inactivation, SHOX
Proteins: SHOX: Short stature homeobox protein
Cytogenetic locus: SHOX: Xpter-p22.32
Inheritance: sporadic
Clinical Features and Diagnostic Criteria: congenital lymphedema, growth failure,
normal intelligence (10% sig delays), coarctation of the aorta, bicuspid aortic valve, HLHS,
hyperlipidemia, gonadal dysgenesis (10% 45,X go into puberty), hypothyroidism, diabetes,
strabismus, recurrent OM, SNHL, Crohns, renal malformation, osteoporosis.
Clinical Tests: echo, renal US, TFTs, GH testing, FISH SRY
Molecular Tests: Karyotype
Disease Mechanism: SHOX: thought to act as a transcription regulator with many downstream targets that modify growth and stature. SHOX protein has been id’ed in the growth
plate from 12 weeks GA to late childhood.
Treatment/Prognosis: GH, HRT, gonadectomy if Y chromosome mosaicism (risk for
gonadoblastoma). Need lifelong cardiac follow-up, at risk for aortic dilation and dissection
with bicuspid aortic valve.
62
TURNER SYNDROME
ENDOCRINE SYSTEM
Low posterior hairline
neck webbing
and
Hypertelorism and
ears
low set
www.healthofchildren.com
www.tsregistry.org/images
63
571
DISORDERS OF HEARING and/or VISION
BLEPHAROPHIMOSIS, PTOSIS,
and EPICANTHUS INVERSUS
Responsible gene: FOXL2
Protein: Forkhead Box Protein L2
Cytogenetic locus: 3q23
Inheritance: AD (50% de novo)
Clinical Features and Diagnostic Criteria: blepharophimosis, ptosis, epicanthus inversus,
and telecanthus. BPES type I includes the four major features and premature ovarian failure
(POF); BPES type II includes only the four major features. Can also see: lacrimal duct
anomalies, amblyopia, strabismus, and refractive errors. Minor features: broad nasal bridge,
low-set ears, and a short philtrum.
Clinical Tests: FOXL2 sequencing, deletion/duplication analysis
Molecular Tests: Combination of seq analysis and deletion testing
Disease Mechanism: FOXL2 is a transcriptional repressor of granulosa cell differentiation;
mutations cause accelerated differentiation of granulosa cells and secondary depletion of the
primordial follicle pool
Treatment/Prognosis: Surgical correction of eye anomalies, ovum donation if POF
64
DISORDERS OF HEARING and/or VISION
BLEPHAROPHIMOSIS, PTOSIS,
and EPICANTHUS INVERSUS
Facial Features:
Blepharophimosis
Ptosis
Epicanthus inversus
Telecanthus
http://bpes.blogg.se
65
DISORDERS OF HEARING and/or VISION
CONGENITAL HEARING LOSS Connexin 26 and 30
Responsible genes: GJB2 (Cx26), GJB6 (Cx30)
Proteins: Gap junction proteins 2 and 6
Cytogenetic loci: 13q11-12
Inheritance: AR
Clinical Features and Diagnostic Criteria: Congenital mild-profound SNHL. Rare
patients can have AD Cx26 HL which can include skin findings: palmar-planter
keratoderma, KID syndrome (keratitis-ichthyosis-deafness)
Clinical Tests: Newborn hearing screen, ABR diagnostic, monitor with standard
audiometry.
Molecular Tests: GJB2: sequencing of exon 2 and exon 1 for splice site mutation (4th
most common mutation). 35delG common in Caucasians, 235delC in Asians, 167delT,
del35Gand Cx30 gene deletion in Ashkenazi Jewish. GJB6-D13S1830 deletion: deletion
that includes Cx30, causes HL if homozygous or combined with single Cn26 mutation.
Disease Mechanism: Loss of gap junction prevents recycling of toxic ions and
metabolites away from hair cells leading to their death
Treatment/Prognosis: No treatment. Some have progressive HL. Habilitation with
hearing aids or cochlear implants.
66
572
DISORDERS OF HEARING and/or VISION
CONGENITAL HEARING LOSS
Connexin 26 and 30
Cochlear
Implant
http://www.yrsddcd.org.uk/images/cochlea1.jpg
&imgrefurl
67
DISORDERS OF HEARING and/or VISION
HERMANSY-PUDLAK SYNDROME
Responsible gene (protein, cytogenetic locus): HPS1 (10q23.1, HPS 1 protein), AP3B1
(5q14.1, AP-3 complex subunit beta), HPS3,4,5,6,7and 8 (3q24, 22q11.2-q12.2, 11p15-p13,
10q24.3, 6p22.3, 19q13, HPS 3,4,5,and 6 proteins, dysbindin, and biogenesis of lysosomerelated organelles complex -1sununit2), HPS9 (BLOC1S6)(15q21.1)
Inheritance: AR
Clinical Features and Diagnostic Criteria: Findings of oculocutaneous albanism and a
bleeding diathesis: hypopigmentation of the skin and the hair, nystagmus, reduced iris
pigment, reduced retinal pigment, foveal hypoplasia, increased crossing of optic nerve fibers.
Can develop skin cancer, pulmonary fibrosis, colitis
Clinical Tests: Absent platelet dense bodies (sine qua non) on platelet EM. Prolonged
bleeding time.
Molecular Tests: Del/Dup analysis HPS1 (~75% Puerto Rican HPS), HPS3 (~25% Puerto
Rican HPS). Targeted mutation analysis HPS3 (~5% non Puerto Rican HPS)
Disease Mechanism: The HPS genes protein products have unknown fcn
Treatment/Prognosis: DDAVP prior to dental work, thrombin-soaked gel foam for minor
cuts, skin protection, annual eye exam, skin exam, and in adulthood PFT’s.
68
DISORDERS OF HEARING and/or VISION
HERMANSY-PUDLAK SYNDROME
Hypopigmentation of
the skin, hair, and iris
http://www.positiveexposure.org/hps/title.jpg
69
573
DISORDERS OF HEARING and/or VISION
JERVELL and LANGE-NIELSEN
SYNDROME
Responsible gene: KCNQ1 and KCNE1
Protein: Voltage-gated K+ channel protein KvLQT1; K+ voltage-gated channel subfamily E
member 1
Cytogenetic loci: 11p15.5, 21q22.1-q22.2
Inheritance: AR (Heterozygotes at risk for AD long QT a.k.a. Romano Ward syndrome)
Clinical Features and Diagnostic Criteria: Congenital severe-profound bilateral SNHL and
prolonged QT interval. At risk for arrhythmia, syncope, and sudden death
Clinical Tests: Hearing tests (ABR, audiogram)
Molecular Tests: KCNQ1 sequencing (90%), KCNE1 (10%)
Disease Mechanism: In cardiac cells: abnormal repolarization of the ventricular action
potential. In cochlear cells: abnormal depolarization of the auditory nerve
Treatment/Prognosis: Cochlear implants for HL, beta blockers, cardiac pacemakers, and/or
implantable defibrillators. Avoid QT prolonging drugs (http://www.arizonacert.org/). If left
untreated, over ½ of children with JLNS die prior to age 15 yrs
70
DISORDERS OF HEARING and/or VISION
JERVELL and LANGE-NIELSEN
SYNDROME
http://genedx.com/site/system/files/LQT.jpg
71
DISORDERS OF HEARING and/or VISION
LEBER HEREDITARY OPTIC
NEUROPATHY
Responsible genes: MTND1, MTND4, MTND6
Proteins: Complex I subunits of the mitochondrial respiratory chain
Cytogenetic loci: Mitochondrial
Inheritance: Mitochondrial
Clinical Features and Diagnostic Criteria: Blurred or clouded vision
progressing to degeneration of the retinal nerve and then optic atrophy.
Fundus: vascular tortuosity of central retinal vessels, circumpapillary
telangiectatic macroangiopathy, and swelling of the retinal nerve fibers
Clinical Tests: Visual field assessments, ERG, VEP
Molecular Tests: Targeted mutation analysis: G11778A (70% cases), G3460A,
T14484C (15%)
Disease Mechanism: Focal degeneration of the retinal ganglion cell layer and
optic nerve
Treatment/Prognosis: No treatment available, worsened by smoking or EtOH
72
574
DISORDERS OF HEARING and/or VISION
LEBER HEREDITARY OPTIC
NEUROPATHY
Acute fundal appearance in
Leber hereditary optic
neuropathy showing disc
hyperaemia, swelling of the
parapapillary retinal nerve fiber
layer and retinal vascular
tortuosity.
(Yu-Wai-Man P et al. J Med Genet 2009;46:145-158)
73
DISORDERS OF HEARING and/or VISION
PENDRED SYNDROME
Responsible gene: SLC26A4 (PDS) most common, FOX11, KCNJ10 in rare cases
Protein: solute carrier 26A4
Cytogenetic locus: 7q31
Inheritance: AR
Clinical Features and Diagnostic Criteria: bilateral severe SNHL, temporal bone
abnormalities, vestibular abnormalities, goiter in 75% though only 10% have abnormal thyroid
function.
Clinical Tests: Hearing test. CT/MRI: dilation of the vestibular aqueduct with or without
cochlear hypoplasia (Mondini malformation)
Molecular Tests: l236P, T416P, H723R, IVS8+G>A represent 50% of all mutations.
SLC26A4 sequencing available.
Disease Mechanism: SLC26A4 is a chloride/iodide exchanger in the inner ear and thyroid,
mutation leads to inner ear malformation and abnormal iodide processing in the thyroid
Treatment/Prognosis: Hearing aids, cochlear implant, monitor thyroid function
74
DISORDERS OF HEARING and/or VISION
PENDRED SYNDROME
Enlarged vestibular
aqueduct in Pendred
syndrome
(http://www.nidcd.nih.gov/health/hearing/pages/vestAque.aspx#diagram)
75
575
DISORDERS OF HEARING and/or VISION
USHER SYNDROME
Responsible genes: multiple genes, majority of cases due to MYO7A, USH2A Proteins:
Myosin-VIIa, Usherin Cytogenetic loci: 11q13.5, 1q41 Inheritance: AR
Clinical Features and Diagnostic Criteria: Type I congenital profound HL, congenital
balance problems, retinitis pigmentosa (RP) onset pre-puberty. Type II congenital mild-severe
HL, normal balance, RP onset in teens-20’s, Type III progressive later onset HL, progressive
balance problems, variable onset RP.
Clinical Tests: hearing tests, ERG, eye exam for pigment changes
Molecular Tests: Type I MYO7A sequence analysis (40-50%) Type II USH2A sequencing
(65%)
Disease Mechanism: RP is caused by degeneration of rod and cone functions of the retina.
For at least some gene, inner hair cell function and structure are affected in the ear.
Treatment/Prognosis: RP is progressive, bilateral, and symmetric resulting in progressively
constricted visual fields though not complete blindness. Vitamin A may slow progression. HL
is complete in Usher Type I and progressive in types II and III. Cochlear implants and
hearing aids for HL
76
DISORDERS OF HEARING and/or VISION
USHER SYNDROME
webvision.med.utah.edu
77
DISORDERS OF HEARING and/or VISION
WAARDENBURG SYNDROME
Responsible gene: PAX3
Protein: Paired box protein Pax-3
Cytogenetic locus: 2q35
Inheritance: AD
Clinical Features and Diagnostic Criteria: WS1: SNHL, heterochromic irides, white
forelock, early graying, leukoderma, dystrophia canthorum, neural tube defect. WS2: WS1
without dystrophia canthorum WS3: WS1 features and limb hypoplasia or contracture, carpal
bone fusion, or syndactyly WS4: WS1 with Hirschprung disease
Clinical Tests: ABR, audiogram, calculation of W-index to identify dystopia canthorum
Molecular Tests: PAX3 gene sequencing (90% WS1)
Disease Mechanism: Haploinsufficiency. PAX3 is a homeobox transcription factor involved
in melanocyte development.
Treatment/Prognosis: Hearing aids or cochlear implants. Folic acid supplementation of
pregnancies at risk for WS1 related neural tube defect
78
576
DISORDERS OF HEARING and/or VISION
WAARDENBURG SYNDROME
White forelock and
dystrophia canthorum
W Index
X = (2a - 0.2119c - 3.909)/c
Y = (2a - 0.2479b – 3.909)/b
W = X + Y + a/b. WS type I if all
affected family members ≥ 1.95.
www.emedicine.com
79
HEMATOLOGIC DISORDERS
ACUTE INTERMITTENT PORPHYRIA
Responsible gene: HMBS Protein: Porphobilinogen deaminase
Cytogenetic locus: 11q23.3 Inheritance: AD
Clinical Features and Diagnostic Criteria: Onset after puberty, acute
attacks, abdominal pain, muscle weakness, neuropathy, hysteria, anxiety,
hepatocellular carcinoma, NO CUTANEOUS FINDINGS
Clinical Tests: increased urine delta-amonolevulinic acid (ALA) and
porphobilinogen (PBG) during acute attack
Molecular Tests: HMBS gene sequencing (>98%)
Disease Mechanism: toxicity of ALA
Treatment: Stop or treat precipitant (medication, infection, EtOH,
dehydration, smoking, poor caloric intake); intubate if bulbar paralysis; IV
dextrose; IV hemin (repress ALAS-N enzyme activity); pain control; liver
transplantation
80
ACUTE INTERMITTENT PORPHYRIA
HEMATOLOGIC DISORDERS
Photograph of urine from
a normal subject (left)
and a subject with acute
intermittent porphyria
(middle). The colors are
compared with a dilute
aqueous solution of red
wine (right). Provided by
Shigeru Sassa, MD, PhD
http://terrycomeau.com/Porphyria/Urine_in_AIP.jpg
81
577
HEMATOLOGIC DISORDERS
ALPHA THALASSEMIA
Responsible genes: HBA1, HBA2 Protein names: Hemoglobin subunit alpha 1 and 2 Cytogenetic
locus (loci): 16pter-p13.3
Inheritance: AR; if parents Alpha Thal trait, risk for HbH disease if one parent’s mutations are in cis, at
risk for HB Bart if both parents in cis
Clinical Features and Diagnostic Criteria: HB Bart: loss or dysfunction of all 4 alpha thal alleles,
hydrops fetalis, severe hypochromic anemia, death in neonatal period; HbH: loss or dysfunction of 3 of
4 alpha thal alleles, microcytic hypochromic hemolytic anemia, HSM, jaundice Alpha Trait:loss or
dysfunction of 2 alpha thal alleles, low MCV, low MCH, nl levels Hgb A2 and F; Alpha “silent” carrier:
loss or dysfunction of one alpha thal allele, none or mild thalassemia-like effect
Clinical Tests: MCV, MCH, peripheral smear, reticulocyte count, hemoglobin electrophoresis. Prenatal
screen at risk populations!
Molecular Tests: Targeted mutation analysis for common deletions (90%); gene sequencing (10%)
Disease Mechanism: Inability to form normal Hb A (normally composed of two alpha and two beta
chains)
Treatment/Prognosis: No tx for HB Bart, rec termination due to maternal complications with hydrops;
intrauterine blood transfusions, hematopoietic stem cell transplant emerging. Hb H: prbc transfusions
during hemolytic crisis, anemia causing cardiac sx, or severe bony changes; splenectomy with abx
prophylaxis (if <5y) for splenomegaly
82
HEMATOLOGIC DISORDERS
ALPHA THALASSEMIA
http://sickle.bwh.harvard.edu/alpha_two.gif
83
BETA-THALASSEMIA
HEMATOLOGIC DISORDERS
Responsible gene: HBB Protein: Hemoglobin subunit beta
Cytogenetic locus: 11p15.5 Inheritance: AR
Clinical Features and Diagnostic Criteria: severe anemia and HSM. Without Tx: severe FTT and shortened
life expectancy. Thal. intermedia: present later, milder anemia, only rarely requires transfusion; at risk for iron
overload due to inc intestinal absorption of iron. The clinical severity of the beta-thal syndromes depends on the
extent of globin alpha chain/non-globin alpha chain imbalance. At risk pop’s: Mediterranean, middle eastern, Indian,
Thai, Chinese, African, African American.
Clinical Tests: microcytic hypochromic anemia, an abnl peripheral blood smear with nucleated RBCs, and
reduced amounts of hemoglobin A (HbA) on hemoglobin analysis. Carriers: reduced MCV, MCH, and RBC
morphologic changes that are less severe than in affected individuals.
Molecular Tests: Mutation scanning/sequencing. In each at-risk population, 4-10 mutations account for the large
majority of HBB disease. Compound heterozygosity for a mild/silent mutation and a severe mutation produces a
variable phenotype, ranging from thalassemia intermedia to thalassemia major.
Disease Mechanism: Absence of globin beta chains. The non-assembled globin alpha chains that result from
unbalanced globin alpha chain/non-globin alpha chain synthesis precipitate in the form of inclusions which damage
the erythroid precursors in the bone marrow and spleen, causing ineffective erythropoiesis.
Treatment/Prognosis: Treat with a regular transfusion program and chelation therapy (to reduce transfusion iron
overload), allows for normal growth and development and extends life expectancy into the third to fifth decade; bone
marrow transplantation is curative
84
578
HEMATOLOGIC DISORDERS
BETA-THALASSEMIA
http://web2.iadfw.net/uthman/hemoglobinopathy/thal_pathogenesis.gif
85
HEMATOLOGIC DISORDERS
FACTOR V LEIDEN THROMBOPHILIA
Responsible gene: F5 Protein: Coagulation factor V
Cytogenetic locus: 1q23
Inheritance: AD (moderately inc. risk VTE), AR (significantly inc .risk VTE)
Clinical Features and Diagnostic Criteria: inc. risk venous thromboembolism (VTE), most commonly
deep venous thrombosis (DVT). Heterozygous: at most modest inc. in VTE recurrence risk, 2-3x inc RR
pregnancy loss. Homozygous: Inc. chance VTE recurrence. Arterial thrombosis, MI, and stroke not
associated with factor V Leiden.
Clinical Tests: APC resistance assay, sensitivity and specificity for factor V Leiden approaches 100%
Molecular Tests: F5 G to A substitution at nt 1691 (100%)
Disease Mechanism: The G>A substitution affects an APC cleavage site and the mutant factor V
Leiden is inactivated 10x more slowly and persists longer in circulation-> inc. thrombin generation
Treatment/Prognosis: Risk of VTE compounded by coexisting thromboembolic d/o, malignancy, travel,
central venous catheters, pregnancy, OCP, HRT, advancing age, surgery, organ transplant.
Heterozygotes with first VTE with no id’ed risk factor or a persistent risk factor require longer course of
anticoagulation than those with a transient risk factor (eg surgery). Long term anticoagulation with
LMW Heparin or Warfarin if recurrent VTE, multiple thrombophilic d/o, coexistent circumstantial risk
factors, and factor V Leiden homozygotes
86
HEMATOLOGIC DISORDERS
HEMOPHILIA A
Responsible gene: F8 Protein: Coagulation Factor VIII
Cytogenetic locus: Xq28 Inheritance: XLR
Clinical Features and Diagnostic Criteria: hemarthrosis or intracranial bleed with mild or
no trauma; deep muscle hematomas; prolonged or renewed bleeding after trauma, surgery,
tooth extraction, nose bleeds, mouth injury, or circumcision, excessive bruising.
Clinical Tests: Prolonged PTT, severe hemophilia: <1%, moderate: 1-5%, and mild
hemophilia 6-35% Factor VIII activity. 10% of carrier females have Factor VIII activity
<35%.
Molecular Tests: Severe: F8 intron 22-A gene inversion (45%), F8 intron 1 gene inversion
(3%), F8 gene del or rearrangement, frameshift, splice junction, or nonsense mutations
(40%), missense mutation (10%). Mild-moderate: missense mutation (97%)
Disease Mechanism: Normal Factor VIII circulates as an inactivated clotting cofactor
activated by thrombin. Severe mutations lead to absent protein, mild-mod mutations to
abnormal protein.
Treatment/Prognosis: IV Factor VIII prophylactically 3x/wk in severe cases and after
trauma, avoid IM injection. Consider HIV, Hep A, B, and C testing if history of receiving
blood products; DDAVP in mild cases
87
579
HEMATOLOGIC DISORDERS
HEMOPHILIA A
http://scielo.isciii.es/img/revistas/medicorpa/v12n5/10.htm29.gif
88
HEMATOLOGIC DISORDERS
HEMOPHILIA B
Responsible gene: F9 Protein: Coagulation factor IX
Cytogenetic locus: Xq27.1-q27.2
Inheritance: XLR
Clinical Features and Diagnostic Criteria: hemarthrosis or intracranial bleed with mild or
no trauma; deep muscle hematomas; prolonged or renewed bleeding after trauma, surgery,
tooth extraction, nose bleeds, mouth injury, or circumcision, excessive bruising.
Clinical Tests: Prolonged PTT, severe hemophilia: <1%, moderate: 1-5%, and mild
hemophilia 6-30% Factor IX activity. 10% of carrier females have Factor VIII activity <30%.
Molecular Tests: F9 sequence analysis (99%). Large gene deletions, nonsense mutations,
and most frameshift mutations cause severe disease.
Disease Mechanism: Factor IX activates Factor X which is a critical early step that can
regulate the overall rate of thrombin generation in coagulation.
Treatment/Prognosis: Recombinant factor IX concentrate 2-3x/wk for severe deficiency
and within one hour of trauma. Avoid IM injection. Consider HIV, Hep A, B, and C testing if
history of receiving blood products.
89
HEMATOLOGIC DISORDERS
HFE-ASSOCIATED HEREDITARY
HEMOCHROMATOSIS
Responsible gene: HFE
Protein: Hereditary hemochromatosis protein
Cytogenetic locus: 6p21.3 Inheritance: AR (penetrance is low, a large fraction of homozygotes
never develop symptoms.
Clinical Features and Diagnostic Criteria: Inappropriately high iron absorption by the GI mucosa
leads to excessive iron storage in the liver, skin, pancreas, heart, joints, and testes. Early Sx:
abdominal pain, weakness, lethargy, and weight loss.
Clinical Tests: Inc. fasting transferrin-iron saturation (men >60%, women >50%; some use >45% as
cutoff for both men and women) on at least 2 occasions, inc. serum ferritin concentration (nonspecific
for HHC), quantitative phlebotomy to determine iron quantity., liver biopsy, hepatic MRI
Molecular Tests: Targeted mutation testing (60-90% C282Y/C282Y; 3-8% C282Y/H63D.
Disease Mechanism: HFE protein binds transferrin receptor 1 and is thought to reduce cellular iron
uptake- mutation leads to inc. iron uptake
Treatment/Prognosis: If untreated: hepatic fibrosis or cirrhosis, increased skin pigmentation, DM,
CHF and/or arrhythmias, cardiomyopathy, arthritis, or hypogonadism. Treat with phlebotomy if
symptomatic, aim for ferritin <50, transferrin-iron saturation <50%
90
580
DISORDERS OF THE IMMUNE SYSTEM
X-LINKED AGAMMAGLOBULINEMIA
(Bruton’s Agammaglobulinemia)
Responsible gene: BTK
Protein: BTK
Cytogenetic locus: Xq21.3-q22
Inheritance: X-LR
Clinical Features and Diagnostic Criteria: recurrent OM,
pneumonia, sinusitis <5yrs; sepsis, meningitis, or cellulitis,
paucity of lymphoid tissue
Clinical Tests: Low but measureable IgG, <1% B Cells (CD19)
Molecular Tests: 90% BTK sequence variant, 10% del/dupl/inv
Disease Mechanism: Immune deficiency; BTK protein
expressed in myeloid cells, platelets, B lineage cells
Treatment/Prognosis: Monthly IV or weekly SC gammaglobulin
91
DISORDERS OF THE IMMUNE SYSTEM
X-LINKED AGAMMAGLOBULINEMIA
(Bruton’s Agammaglobulinemia)
(www.tmd.ac.jp/english/press-release/20120227/)
92
DISORDERS OF THE IMMUNE SYSTEM
FAMILIAL MEDITERRANEAN FEVER
Responsible gene: MEFV
Protein: Pyrin
Cytogenetic locus: 16p13
Inheritance: AR
Clinical Features and Diagnostic Criteria: Type 1 recurrent febrile episodes with
peritonitis, synovitis, or pleuritis, recurrent erysipelas-like erythema, AA type amyloidosis,
favorable response to continuous colchicine treatment, at risk ethnic group (Armenian,
Turkish, Arab, North African Jewish, Iraqi Jewish, Ashkenazi Jewish). Type 2 amyloidosis as
first clinical presentation
Clinical Tests: Inc ESR, leukocytosis, inc serum fibrinogen, proteinuria
Molecular Tests: MEFV targeted mutation analysis (70-90% depending on panel and
ethnicity), MEFV sequencing (90% all ethnic groups)
Disease Mechanism: Mutations result in less IL-1beta activation and as a result inc IL-1
responsiveness-> inc inflammatory attacks
Treatment/Prognosis: M694V homozygotes or compound heterozygotes with another FMF
allele treated with daily colchicine for life. Colchicine decreases inflammatory attacks and
deposition of amyloid.
93
581
DISORDERS OF THE IMMUNE SYSTEM
FAMILIAL MEDITERRANEAN FEVER
(healthlineinfo.com/familial-mediterranean-fever.html)
94
MULTIPLE CONGENITAL ANOMALIES
AARSKOG SYNDROME
Responsible gene: FGD1
Protein: Rho/Rac guanine nucleotide exchange factor
Cytogenetic locus: Xp11.22
Inheritance: XLR (some AR, AD cases reported)
Clinical Features and Diagnostic Criteria: hypertelorism, shawl scrotum,
brachydactyly, short stature, cryptorchidism, cervical vertebral
abnormalities, ID (30%), milder manifestations in females
Clinical Tests: xray
Molecular Tests: FGD1 sequencing (7-20%)
Disease Mechanism: unclear, FGD1/Rho GTPase Cdc42 implicated in
cytoskeletal organization, and potentially in skeletal formation and
morphogenesis
Treatment/Prognosis: orchiopexy, growth hormone trials have not been
successful
95
MULTIPLE CONGENITAL ANOMALIES
AARSKOG SYNDROME
Int. braz j urol. vol.32 no.4, 2006
96
582
MULTIPLE CONGENITAL ANOMALIES
ANTLEY-BIXLER SYNDROME
Responsible gene: POR
Protein: NADPH-cytochrome P450 reductase
Cytogenetic locus: 7q11.2
Inheritance: AR
Clinical Features and Diagnostic Criteria: Ambiguous genitalia, enlarged cystic ovaries,
poor masculinization in males, maternal virilization during pregnancy with an affected fetus.
Craniosynostosis, choanal stenosis or atresia, stenotic external auditory canals,
hydrocephalus. Neonatal fractures, bowing of the long bones, joint contracture, renal
malformations
Clinical Tests: Sterol or or steroid abnormalities using GC-MS, increased urinary
pregnenolone and progesterone metabolites
Molecular Tests: POR sequence variants
Disease Mechanism: Disorder of steroid and cholesterol synthesis due to cytochrome P450
reductase deficiency
Treatment/Prognosis: Airway management, hydrocortisone replacement, stress dose
steroids, surgical correction of genital abnormalities, VP shunt for significant hydrocephalus,
PT to minimize joint contracture
97
MULTIPLE CONGENITAL ANOMALIES
ANTLEY-BIXLER SYNDROME
Antley-Bixler Facial Features
Frontal bossing
Severe midface hypoplasia
Short bulbous nose
Depressed nasal bridge
Small mouth
Dysplastic ears that may be low set
http://pediatricneuro.com/alfonso/newa10l.jpg
98
MULTIPLE CONGENITAL ANOMALIES
BARDET-BIEDL SYNDROME
Responsible genes: BBS1, BBS10 (multiple additional genes id’ed)
Proteins: BBS1 protein, BBS10 protein Cytogenetic loci: 11q13, 12q21.2
Inheritance: AR (though 10% BBS thought to be tri-allelic)
Clinical Features and Diagnostic Criteria: cone-rod dystrophy, truncal obesity, postaxial
polydactyly, cognitive impairment, male hypogonadotrophic hypogonadism, complex female
genitourinary malformations, and renal dysfunction. Night blindness by age 7-8 yrs, legally blind by
age 15.5 yrs. A majority have significant learning difficulties, only a minority have severe impairment.
Renal disease is a major cause of morbidity and mortality.
Clinical Tests: atypical pigmentary retinal dystrophy with early macular involvement, renal
anomalies on US
Molecular Tests: Targeted mutation analysis: p.M390R BBS1 (18%-32% of BBS) and C91fsX95
BBS10 (10% of BBS).
Disease Mechanism: Defects in cilia or intraflagellar transport (IFT)
Treatment/Prognosis: visual aids and educational programs for the visually impaired; diet,
exercise, and behavioral therapies for obesity; surgery to remove accessory digits; surgical repair of
hydrocolpos, vaginal atresia, or hypospadias; HRT for hypogonadism.
99
583
MULTIPLE CONGENITAL ANOMALIES
BARDET-BIEDL SYNDROME
Beales P L et al. J Med Genet 1999;36:437-446
100
MULTIPLE CONGENITAL ANOMALIES
BRANCHIO-OTO-RENAL
SYNDROME
Responsible gene: EYA1, SIX1, SIX5
Proteins: Eyes absent homolog 1, Homeobox protein SIX1 and SIX5
Cytogenetic loci: 8q13.3, 14q23, 19q13.32
Inheritance: AD
Clinical Features and Diagnostic Criteria: malformations of the outer, middle, and inner ear
associated with conductive, sensorineural, or mixed hearing impairment; branchial fistulae
and cysts; and renal malformations, ranging from mild renal hypoplasia to bilateral renal
agenesis
Clinical Tests: Temporal bone CT, hearing test, renal US
Molecular Tests: Mutation scanning (30%), Dupl/del testing (10%)
Disease Mechanism: EYA1 encodes products important for inner-ear, kidney, and branchialarch development. Some mutations encode proteins that are rapidly degraded. Expression of
SIX1 is necessary for normal development of the inner ear, nose, thymus, kidney, and skeletal
muscle
Treatment/Prognosis: excision of branchial cleft cysts/fistulae, fitting with appropriate aural
habilitation, hearing impaired education programs. End-stage renal disease may require
dialysis or renal transplantation. Surveillance includes semiannual examination for hearing
impairment and annual audiometry to assess stability of hearing loss and semiannual/annual
examination by a nephrologist if indicated
101
MULTIPLE CONGENITAL ANOMALIES
BRANCHIO-OTO-RENAL
SYNDROME
Branchial
fistula
Trummer T et al. J Med Genet 2002;39:71-73
102
584
MULTIPLE CONGENITAL ANOMALIES
CHARGE SYNDROME
Responsible gene: CHD7
Protein: Chromodomain-helicase-DNA-binding protein 7 Cytogenetic locus: 8q12.1
Inheritance: AD
Clinical Features and Diagnostic Criteria: 4/7: eye coloboma, heart anomaly (conotruncal
defects, arch abnormalities), choanal atresia, growth and mental retardation, genitourinary
malformations (microphallus), ear anomalies (ossicular malformations, Mondini defect of the
cochlea) and/or deafness. Facial palsy, cleft palate, TE fistula, and dysphagia are commonly
associated. 20-25% mortality in the first year
Clinical Tests: echocardiogram, audiology evaluation, temporal bone CT, renal ultrasound
Molecular Tests: CHD7 sequencing (60-65%)
Disease Mechanism: Haploinsufficiency. This class of proteins is thought to have pivotal
roles in early embryonic development by affecting chromatin structure and gene expression
Treatment/Prognosis: Assess for airway compromise, swallowing problems, typical surgical
correction of heart and GI malformations
MULTIPLE CONGENITAL ANOMALIES
CHARGE SYNDROME
www.ncbi.nlm.nih.gov/.../bin/chargeFig1.jpg
104
MULTIPLE CONGENITAL ANOMALIES
COFFIN-LOWRY SYNDROME
Responsible gene: RPS6KA3
Protein: Ribosomal protein S6 kinase alpha-3
Cytogenetic locus: Xp22.2-p22.1
Inheritance: XLD
Clinical Features and Diagnostic Criteria: severe to profound ID in males, short, soft fleshy
hands, tapering fingers with small terminal phalanges, males <3% in height, microcephaly,
stimulus induced drop episodes, kyphoscoliosis, characteristic facial features in older males,
normal to profound ID in females.
Clinical Tests: x-ray: thickened skull, anterior vertebrae beaking, metacarpal
pseudoepiphyses
Molecular Tests: RPS6KA3 sequencing (35-40%)
Disease Mechanism: unclear, RPS6KA3 is a member of the Ras signaling cascade and
participates in cellular events such as proliferation and differentiation
Treatment/Prognosis: Medication for drop episodes, Rispieridone for self-injurious behavior,
annual cardiac exam with echo every 5-10 years.
105
585
COFFIN-LOWRY SYNDROME
MULTIPLE CONGENITAL ANOMALIES
Coffin Lowry Facial Features
Prominent forehead and eyebrows
Full supraorbital ridges
Marked ocular hypertelorism with
downslanting palpebrae
Low nasal bridge, blunt tip, and thick
alae nasi and septum
Large mouth, usually held open
Patulous lips with everted lower lip
Prominent ears
http://clsf.info/Images/2005_July.JPG
106
MULTIPLE CONGENITAL ANOMALIES
CORNELIA DE LANGE SYNDROME
Responsible gene: NIPBL, SMC1A, SMC3, HDAC8, RAD21
Protein: Nipped-B-like protein, Structural maintenance of chromosomes protein 1A and 3,
histone deacetylase 8, RAD21
Cytogenetic loci: 5p13.1, Xp11.22-p11.21, 10q25.2, Xq13.1, 8q24.11
Inheritance: AD (NIPBL and SMC3), XLR (SMC1L1)
Clinical Features and Diagnostic Criteria: pre/postnatal growth retardation, low anterior
hairline and synophrys, diaphragmatic hernia, upper limb anomalies (hypoplastic middle
phalanx of the index finger, hypoplastic thenar eminence), ptosis, nystagmus, mod-severe ID,
pulmonary valve stenosis and/or VSD
Clinical Tests: non are diagnostic
Molecular Tests: NIPBL sequencing (~50%), SMC1L1 sequencing (4%), SMC3 (<1%)
Disease Mechanism: Unknown, the majority of mutations are truncating, likely leading to
protein haploinsufficiency
Treatment/Prognosis: Treat individual medical and developmental issues
107
MULTIPLE CONGENITAL ANOMALIES
CORNELIA DE LANGE SYNDROME
Facial Features
Microbrachycephaly
Synophrys, arched eyebrows
Long, thick eyelashes
Low-set posteriorly rotated and/or hirsute
ears with thickened helices
Depressed or broad nasal bridge,
upturned nasal tip with anteverted nares,
and prominence of the lateral aspects
Long smooth philtrum, thin vermillion
border of the upper lip with a midline
"drip" appearance, downturned corners
of the mouth
High and arched palate with clefts
Small widely-spaced teeth
Micrognathia
Short neck
http://www.emedicine.com/ped/images/289503
108
586
MULTIPLE CONGENITAL ANOMALIES
CRI-DU-CHAT (5p MINUS SYNDROME)
Responsible gene(s): RPS14?, microRNA 145 and 146a?
Protein(s):
Cytogenetic locus: 5p15.2
Inheritance: 12% due to unequal segregation of a translocation or recombination involving a
pericentric inversion in one of the parents, 85% sporadic de novo deletions (80% are on the
paternal chromosome)
Clinical Features and Diagnostic Criteria: Cat-like cry (abnormal laryngeal development),
slow growth, microcephaly, ID, hypotonia, strabismus, characteristic facial features. Cat-like
cry only when deletion limited to band 5p15.32
Molecular Tests: Most are visible, a few are submicroscopic and diagnosed by FISH for the
critical region.
Disease Mechanism: A study of 50 patients with deletions ranging from 5p15.2 to 5p13 and
found no correlation with size of deletion and degree of mental impairment
Treatment/Prognosis: Supportive care
109
MULTIPLE CONGENITAL ANOMALIES
CRI-DU-CHAT (5p MINUS SYNDROME)
Facial Features
Microcephaly
Round face
Hypertelorism
Micrognathia
Epicanthal folds
Low-set ears
www.specialchild.com/archives/poster-child023
110
MULTIPLE CONGENITAL ANOMALIES
FRYNS SYNDROME
Responsible gene(s): unknown
Protein(s): unknown
Cytogenetic locus (loci): unknown
Inheritance: AR
Clinical Features and Diagnostic Criteria: LGA, coarse face,
CL/CP, diaphragmatic defect, distal digital hypoplasia, ID in
survivors, agenesis of the CC, optic and olfactory tract hypoplasia,
encephalocele, GU malformation
Clinical Tests: symptomatic
Molecular Tests: none
Disease Mechanism: unknown
Treatment/Prognosis: The majority are stillborn or die in early
neonatal period, 14% survive
111
587
MULTIPLE CONGENITAL ANOMALIES
FRYNS SYNDROME
Hypoplastic nails
Fronal encephalocele and lissencephaly
112
MULTIPLE CONGENITAL ANOMALIES
GREIG CEPHALOPOLYSYNDACTYLY
Responsible gene: GLI3
Protein: Zinc finger protein GLI3
Cytogenetic locus: 7p13
Inheritance: AD
Clinical Features and Diagnostic Criteria: Major findings: macrocephaly, ocular
hypertelorism, preaxial polydactyly, cutaneous syndactyly. Developmental delay, ID, or
seizures (<10%)- more common in those with large (>300 kb) deletions including GLI3.
Allelic with Pallister-Hall syndrome (caused by GLI3 frame shifting mutations).
Clinical Tests: 500-600 band karyotype 7p13 translocation or interstitial deletion (5-10%)
Molecular Tests: GLI3 sequence analysis (70%)
Disease Mechanism: GLI proteins regulate genes distal to Sonic Hedgehog in the SHH
pathway. Pathogenesis of GCPS is haploinsufficiency
Treatment/Prognosis: Surgical correction of polydactyly and syndactyly as indicated. CNS
imaging if HC increasing faster than normal to r/o hydrocephalus
113
MULTIPLE CONGENITAL ANOMALIES
GREIG CEPHALOPOLYSYNDACTYLY
Macrocephaly, hypertelorism, and polysyndactyly
http://www.ojrd.com/content/figures/1750-1172-3-10-1.jpg
114
588
MULTIPLE CONGENITAL ANOMALIES
JOUBERT SYNDROME
Responsible genes: NPHP1, AHI1, CEP290, TMEM67 and others (19 genes)
Proteins: Nephrocystin-1, Jouberin, Centrosomal protein Cep290, Meckelin
Cytogenetic loci: 2q13, 6q23.3, 12q21.32, 8q21.1-q22.1
Inheritance: AR (19 genes with rare-4% prevalence)
Clinical Features and Diagnostic Criteria: Hypotonia in infancy leading to ataxia later,
DD/ID, alternating tachypnea and/or apnea), pigmentary retinopathy, oculomotor apraxia or
difficulty in smooth visual pursuits and jerkiness in gaze tracking. M:F, 2:1. Renal disease
seen in those with retinal involvement. Rarely hepatic fibrosis.
Clinical Tests: Molar tooth sign (cerebellar vermis hypoplasia) on MRI, ERG, renal US, LFT’s
Molecular Tests: NPHP1 FISH or deletion analysis (1-2%), Sequencing AHI1 (11%), CEP290
(10%), TMEM67 (10%)
Disease Mechanism: The CEP290 protein product modulates ATF4, a transcription factor
implicated in renal cyst formation. Meckelin localizes to the primary cilium and plasma
membrane of renal and biliary epithelial cells and other ciliated cells
Treatment/Prognosis: apnea monitoring, ST, G tube if severe dyspahgia, surgery as needed
for eye disease, dialysis for nephronophthisis
115
MULTIPLE CONGENITAL ANOMALIES
JOUBERT SYNDROME
Molar tooth
sign
116
MULTIPLE CONGENITAL ANOMALIES
KABUKI SYNDROME
Responsible genes: KMT2D (66%), KDM6A
Proteins: MLL2, Lysine-specific demethylase 6A
Cytogenetic loci: 12q12-q14, Xp11.3
Inheritance: AD, XLD
Clinical Features and Diagnostic Criteria: unique facial features, fetal finger pads, IQ<80,
joint laxity, high palate, hypotonia, short stature, CHD, CL/P, scoliosis, renal anomalies,
hearing loss, speech delay
Clinical Tests: echocardiogram, renal ultrasound, eye exam, neuropsychological testing
Molecular Tests: MLL2 gene sequencing, KDM6A gene sequencing and deletion testing
Disease Mechanism: MLL2 encodes a protein that is part of the SET family of proteins,
important to the epigenetic control of active chromatin states. Mutations are predicted to
truncate the polypeptide chain before translation of the SET domain. H3K4 methylation by
MLL2 is linked to the demethylation of H3K27 by KDM6A.
Treatment/Prognosis: Individual medical problems are treated as in the general population.
GH for short stature if deficient. At risk for immunodeficiency.
117
589
MULTIPLE CONGENITAL ANOMALIES
KABUKI SYNDROME
Facial Features
Elongated palpebral fissures
Eversion of the lateral third
of the lower eyelid
Arched and broad eyebrows
Short columella with
depressed nasal tip
Large, prominent, or cupped
ears
http://kabukisyndrome.com
118
MULTIPLE CONGENITAL ANOMALIES
MONOSOMY 1p36
Responsible genes: unknown Proteins: unknown
Cytogenetic locus: 1p36
Clinical Features and Diagnostic Criteria: The most common terminal
deletion syndrome. Hypotonia, developmental delay, growth retardation,
obesity, microcephaly, orofacial clefting, typical facial features. Also minor
cardiac malformations, cardiomyopathy, seizures, ventricular dilation, SNHL
Clinical Tests: Brain CT/MRI
Molecular Tests: The deletion can be detected by HR karyotype,
confirmatory FISH required in most cases. The majority are maternally
derived.
Disease Mechanism: contiguous gene deletion syndrome
Treatment/Prognosis: symptomatic treatment
119
MULTIPLE CONGENITAL ANOMALIES
MONOSOMY 1p36
Facial Features:
Straight eyebrows
Deep-set eyes
Midface hypoplasia
Broad and flat
nasal root/bridge
Long philtrum
Pointed chin
Microbrachycephal
y Epicanthal folds
Posteriorly rotated,
low-set, abnormal
ears.
www.theanswers.com
120
590
MULTIPLE CONGENITAL ANOMALIES
PRADER-WILLI SYNDROME
Responsible genes: Paternally expressed genes within the imprinted locus
on 15q11-13 (SNURF-SNRPN, MKRN3, MAGEL2, and NDN )
Cytogenetic locus: 15q11-13
Inheritance: autosomal, expressed from paternal Ch 15
Clinical Features and Diagnostic Criteria: Hypothalamic insufficiency,
neonatal hypotonia, developmental delay, hyperphagia leading to obesity,
short stature, small hands and feet, hypogonadism, ID
Molecular Tests: 3-5 Mb deletion of 15q11.2-q13 (~70%), matUPD (15%),
PWS imprinting center defect (1-2%)
Disease Mechanism: unknown
Treatment/Prognosis: Monitor for feeding problems in infancy, obesity, OCD,
psychosis, scoliosis, obstructive sleep apnea, diabetes, osteopenia
121
MULTIPLE CONGENITAL ANOMALIES
PRADER-WILLI SYNDROME
http://www.bcpwsa.com/images/header.jpg
Arch Dis Child 2008;93:341-345 doi:10.1136/adc.2007
122
MULTIPLE CONGENITAL ANOMALIES
RUBENSTEIN-TAYBI SYNDROME
Responsible gene: CREBBP, EP300
Protein: CREB-binding protein, histone acetyltransferase-p300
Cytogenetic locus: 16p13.3, 22q13
Inheritance: AD though only a few cases of affected parent and child
Clinical Features and Diagnostic Criteria: microcephaly, beaked nose, broad thumbs and
toes, cryptorchidism, growth delay, severe ID (35-50), congenital heart defect, strabismus,
ptosis, sleep apnea, tumors (meningioma, pilomatrixoma, leukemia), behavior problems
Clinical Tests: ERG, echocardiogram, deletion or translocation occasionally seen on
karyotype
Molecular Tests: FISH CREBBP (~10%), direct sequencing CREBBP (40-60%), EP300
(~3%)
Disease Mechanism: Some CREBBP mutations lead to abnormal acetylation of histones,
an important step in transcription activation
Treatment/Prognosis: Standard care for vision, hearing loss, heart defects, feeding
problems. Some require thumb/toe surgery, behavior modification programs
123
591
MULTIPLE CONGENITAL ANOMALIES
RUBENSTEIN-TAYBI SYNDROME
Broad, deviated thumbs
and first toes
European Journal of Human Genetics (2006) 14, 981–985
124
MULTIPLE CONGENITAL ANOMALIES
SMITH-MAGENIS SYNDROME
Responsible gene: RAI1
Protein: Retinoic acid-induced protein 1
Cytogenetic locus: 17p11.2
Inheritance: AD (sporadic unless secondary to a parental balanced translocation)
Clinical Features and Diagnostic Criteria: mild-moderate infantile hypotonia, feeding
problems and FTT, short stature, brachydactyly, ophthalmologic and ORL abnormalities,
early speech delay with or without hearing loss, peripheral neuropathy, sleep problems, and
stereotypic maladaptive behaviors (self-injurious behaviors, inattention+hyperactivity,
impulsivity, disobedience, the “self-hug” and “lick and flip” page turning motion), mild-mod ID,
coarsening face over time
Clinical Tests: Renal US, echo, spine x-ray, FISH, CMA for 17p11.2 deletion (~90%)
Molecular Tests: RAI1 sequencing (5-10%)
Disease Mechanism: The RAI1 gene product is thought to function in transcriptional
regulation
Treatment/Prognosis: ST, sensory integration, psychotropic meds for attention issues,
behavioral therapies, melatonin may help with sleep, monitoring for hypercholesterolemia.
Annual team eval, TFTs, fasting lipid profile, UA, scoliosis check, eye exam
125
MULTIPLE CONGENITAL ANOMALIES
SMITH-MAGENIS SYNDROME
Facial Features:
Brachycephaly
Midface retrusion
Relative prognathism with
age
Broad, square-shaped face
Everted, "tented“ vermilion
of the upper lip
Deep-set, close-spaced eyes
http://www.nature.com/ejhg/journal/v16/n4/images/5202009f2.jpg
126
592
MULTIPLE CONGENITAL ANOMALIES
TRIPLOIDY
Cytogenetic abnormality: 69,XXY>69,XXX (69,XYY very rare)
Inheritance: Sporadic without inc risk of recurrence
Clinical Features and Diagnostic Criteria: >99% lost in first trimester, accounts for 6-10%
of all SAb’s and 16-20% of all chromosomally abnormal SAb’s. Dysplastic calvaria with large
posterior fontanelle, ¾ finger syndactyly, ASD, VSD, hydrocephalus, holoprosencephaly.
Parent of origin effect: If Maternal: small placenta, severe asymmetric IUGR with a large head
If Paternal: hydropic large placenta, well grown to mod symmetric IUGR, nl or microcephalic
head
Clinical Tests: Prenatal US, maternal serum hCG low
Molecular Tests: Karyotype
Disease Mechanism: Gynogenic triploidy (digyny): NDJ producing diploid oocyte, fertilization
of ovulated primary oocyte, or polar body retention. Androgenic triploidy (Diandry) NDJ
producing a diplod sperm or dispermy (most common)
Treatment/Prognosis: Very poor prognosis, may be better if triploid mosaic
127
MULTIPLE CONGENITAL ANOMALIES
TRIPLOIDY
Classic 3/4
finger
syndactyly of
triploidy
library.med.utah.edu
128
MULTIPLE CONGENITAL ANOMALIES
TRISOMY 13, PATAU SYNDROME
Inheritance: 20% due to a translocation
Clinical Features and Diagnostic Criteria: The least common of the live
born trisomy disorders. Holoprosencephaly, polydactyly, seizures, HL,
microcephaly, midline CL/P, omphalocele, cardiac and renal anomalies, ID.
Mosaic Tri 13: very broad phenotype from typical features of full trisomy to
more mild ID and physical features and longer survival.
Clinical Tests: Brain MRI, EEG, audiogram, echo, renal US
Molecular Tests: Karyotype is diagnostic
Disease Mechanism: 75% are due to maternal nondysjunction, 20% to a
translocation, and 5% to mosaicism. Defect in fusion of the midline
prechordial mesoderm in the first three weeks of gestation cause the major
midline dysmorphic features.
Treatment/Prognosis: 44% die in the first month, >70% die within one year.
Severe ID exists in all survivors.
129
593
MULTIPLE CONGENITAL ANOMALIES
TRISOMY 13, PATAU SYNDROME
Cutis Aplasia
www.prenatalpartnersforlife.org
130
MULTIPLE CONGENITAL ANOMALIES
TRISOMY 18, EDWARDS SYNDROME
Inheritance: Less than 1% due to a translocation
Clinical Features and Diagnostic Criteria: clenched hand,
fingers 2/5 overlap 3/4, IUGR, rocker bottom feet, micrognathia,
prominent occiput, microphthalmia, VSD, ASD, PDA, generalized
muscle spasm, renal anomalies, ID. Mosaic Tri 18 has variable
but usually somewhat milder expression.
Clinical Tests: Echo, abdominal US. Maternal serum screen: low
AFP, hCG, and UE3.
Molecular Tests: karytype is diagnostic
Disease Mechanism: Maternal nondysjunction (90%), mosaicism
(10%)
Treatment/Prognosis: 50% die in first week, 90% die by one year
131
MULTIPLE CONGENITAL ANOMALIES
TRISOMY 18, EDWARDS SYNDROME
Typical digit 2 over 3 and 5
over 4 of Trisomy 18
Typical rocker bottom foot of
Trisomy 18
www.gfmer.ch/.../gendis
132
594
MULTIPLE CONGENITAL ANOMALIES
TRISOMY 21, DOWN SYNDROME
Cytogenetic locus (loci): (21.22.1-22.2 has been called the DS critical region though there
have been cases of duplication outside of this region who manifest DS
Inheritance: 95% de novo, 5% due to Robertsonian translocation or isochromosome 21
Clinical Features and Diagnostic Criteria: mild-mod ID, hypotonia, growth delay,
strabismus, adult cataracts, myopia, conductive HL, macroglossia, hypodontia, joint
hyperflexibility, hypogenitalism, congenital heart defect, duodenal atresia, hirschprung,
thyroid disease, early onset Alzheimers, transient myeloproliferation, ALL
Clinical Tests: prenatal US abnormalities detected in 50%, maternal serum screen: high free
beta HCG, low PAPP-A,
Molecular Tests: maternal fetal free DNA testing, karyotype is diagnostic
Disease Mechanism: 90% due to maternal meiosis nondisjunction (¾ MI error, ¼ MII error)
Treatment/Prognosis: Supportive care, overall life expectancy is reduced
133
MULTIPLE CONGENITAL ANOMALIES
TRISOMY 21, DOWN SYNDROME
www.fetalcenter.com/images/Trisomy_21
134
MULTIPLE CONGENITAL ANOMALIES
VACTERL (VATER) ASSOCIATION
Responsible genes: unknown (HOXD13 21 bp deletions: 1 case report), FGF8?, PTF1A?
Proteins: unknown
Cytogenetic locus: unknown
Inheritance: Isolated
Clinical Features and Diagnostic Criteria: Vertebral anomalies, Anal atresia, Cardiac
malformations (VSD, PDA, TOF, TOV), Treacheoesophageal fistula, Esophageal atresia,
Renal anomalies, and Limb anomalies (polydactyly, humeral hypoplasia, radial aplasia,
proximally placed thumb). Diagnosis requires 3 of 7 features and it is a diagnosis of
exclusion. A variant is VACTERL with hydrocephalus which can be AR or XL.
Clinical Tests: echo, spinal x-ray, limb x-ray, and renal US
Molecular Tests: There isn’t a molecular test but rule out aneuploidy with karyotype,
Fanconi anemia with DEB testing, and consider SALL1 sequencing to rule out TownesBrocks syndrome.
Disease Mechanism: unknown
Treatment/Prognosis: Severe cardiac malformation, anal atresia, TE fistula, and EA
require surgical repair in the neonatal period
135
595
MULTIPLE CONGENITAL ANOMALIES
VACTERL (VATER) ASSOCIATION
Approximate days post-conception during which anatomic
structures in VACTERL form
(Stevenson, Mol Syndromol 2013;4:7–15 )
136
MULTIPLE CONGENITAL ANOMALIES
WOLF-HIRSCHORN SYNDROME
(4p minus, Monosomy 4p)
Responsible genes: 4p deletion, critical region includes two genes, WHSC1 and WHSC2 of
unknown significance
Protein: unknown
Cytogenetic locus: 4p; critical region: 165-kb region between markers D4S166 and
D4S3327
Inheritance: 87% de novo, 13% due to unbalanced translocation from a balanced parent
Clinical Features and Diagnostic Criteria: “greek warrior helmet appearance”,
microcephaly, pre and postnatal growth deficiency, ID of variable degree, seizures, facial
asymmetry, ptosis, IgA deficiency, structural brain anomalies, CL/P, CHD
(ASD>PVS>VSD>PDA>AI>TOF), renal US
Clinical Tests: Distinctive EEG, Brain MRI, echo, plasma IgA level
Molecular Tests: HR karyotype for 4p16.3 deletion (60-70%), FISH/array CGH for critical
region deletion (>95%)
Disease Mechanism: The function of WHSC1, WHSC2, and LETM1 in normal development
and in WHS patients is not known
Treatment/Prognosis: 2/3 develop valproate responsive atypical absence seizures,
standard treatment of other medical problems
137
MULTIPLE CONGENITAL ANOMALIES
WOLF-HIRSCHHORN SYNDROME
(4p minus, Monosomy 4p)
Facial Features:
'Greek warrior helmet appearance' of
the nose (the broad bridge of the nose
continuing to the forehead)
Microcephaly
High forehead with prominent glabella
Ocular hypertelorism
Epicanthus
Highly arched eyebrows
Short philtrum
Downturned mouth
Micrognathia
Poorly formed ears with pits/tags
medgen.genetics.utah.edu
138
596
NEUROLOGIC DISORDERS
X-LINKED ADRENOLEUKODYSTROPHY
Responsible gene: ABCD1
Protein: ATP-binding cassette sub-family D member 1
Cytogenetic locus: Xq28
Inheritance: X-LR
Clinical Features and Diagnostic Criteria:
a. Childhood cerebral: ADHD->total disability within 2 yrs
b. Adrenomyeloneuropathy: late 20’s progressive paraparesis, sphincter
disturbance, adrenocortical dysfunction
c. Adrenocortical insufficiency (only); majority by age 7.5 (seen in 20% carrier
females
Clinical Tests: Brain MRI, VLCFA (not reliably abnl in carrier females)
Molecular Tests: ABCD1 seq (92%); ABCD1 del/dup (6%)
Disease Mechanism: Peroxisomal disorder, accumulation of saturated VLCFA
Treatment/Prognosis: Corticosteroid replacement, BMT if diagnosed after
changes visible on brain MRI but before significant neuropsych problems
develop (Lorenzo’s Oil)
139
X-LINKED ADRENOLEUKODYSTROPHY
NEUROLOGIC DISORDERS
Clinical Features of Various Phenotypes of X-linked ALD
*POW = parieto-occipital white matter.
(Kim and Kim, RadioGraphics, 2005)
140
NEUROLOGIC DISORDERS
EARLY ONSET FAMILIAL
ALZHEIMER DISEASE
Responsible genes: PSEN1, APP, PSEN2
Proteins: Presenelin-1, Amyloid beta A4, Presenilin-2
Cytogenetic loci: 14q24.3, 21q21, 1q31-q42
Inheritance: AD
Clinical Features and Diagnostic Criteria: Dementia, confusion, poor judgment, language
disturbance, agitation, withdrawal, and hallucinations. Early onset: <age 60
Clinical Tests: Gross cerebral cortical atrophy. Post mortem neuropath: A beta-amyloid neuritic
plaques, intraneuronal neurofibrillary tangles, and amyloid angiopathy
Molecular Tests: Seq.: PSEN1 (20-70%), APP (10-15%), PSEN2 (rare)
Disease Mechanism: ?chromosomal instability and breakage at nonrandom sites? Triple
dose of APP may explain Alzheimer’s in Tri 21.
Treatment/Prognosis: Death from general inninition, malnutrition, and pneumonia. Clinical
duration 8-10 yrs (range 1-25 yrs)
EOFAD 1-6% OF ALL Alzheimer’s, 60% of which is familial, and 13% inherited in an AD
manner. (<2% of all Alzheimer’s)
LOFAD: Appears to be an assoc with APOE e4 but not sensitive or specific- supports the dx.
APOE e2 may be protective.
597
NEUROLOGIC DISORDERS
EARLY ONSET FAMILIAL
ALZHEIMER DISEASE
Beta-amyloid plaques: senile plaques
appear as small collections of dark,
irregular, thread-like structures often
with a brownish material in the center.
The central core is represented by
amyloid and the irregular, beaded
linear structures represent abnormal
neurites (small dendrites and axons
with degenerative changes).
Nature. 1999 Jul 8;400(6740):173-7.
142
NEUROLOGIC DISORDERS
ANGELMAN SYNDROME
Responsible gene: UBE3A
Protein: Ubiquitin protein ligase E3A
Cytogenetic locus: 15q11-q13
Inheritance: loss of the maternally imprinted contribution in the 15q11.2-q13 (AS/PWS)
region
Clinical Features and Diagnostic Criteria: severe developmental delay or ID, severe
speech impairment, gait ataxia and/or tremulousness of the limbs, and an inappropriate
happy demeanor that includes frequent laughing, smiling, and excitability, microcephaly and
seizures
Clinical Tests: acquired microcephaly by age two years,
Seizures before age three, abnl EEG: large amp. slow-spike waves
Molecular Tests: 4-6 Mb del (65-75%), UBE3A mutation (11%), imprinting defect (2.5%),
unbal chrom transloc (<1%), Pat UPD 15 (<1%), del of imprinting center (0.5%)
Disease Mechanism: Disruption of E6AP ultimately causes an abnormality in the ubiquitin
protein degradation pathway, but no clear AS-causing target protein yet identified
Treatment/Prognosis: Typical care for medical issues, PT, OT, ST, and individualized
education and behavior program.
143
NEUROLOGIC DISORDERS
ANGELMAN SYNDROME
Facial features:
Protruding tongue
Prognathia
Wide mouth
Widely spaced teeth
Strabismus
Light hair and eye color
http://www.psychnet-uk.com/dsm_iv/pictures/angel.jpg
144
598
NEUROLOGIC DISORDERS
CADASIL
(Cerebral Autosomal Dominant Arteriopathy with
Subcortical Infarcts and Leukoencephalopathy)
Responsible gene: NOTCH3
Protein: Neurogenic locus notch homolog protein 3
Cytogenetic locus: 19p13.2-p13.1 Inheritance: AD
Clinical Features and Diagnostic Criteria: Stroke-like episodes before age 60, cognitive
disturbance, behavioral abnormalities, migraine with aura
Clinical Tests: Skin Bx EM: e- dense granules in media of arterioles. Brain MRI: T2 signal
abnormalities in the WM of the temporal pole and external capsule, subcortical lacunar lesions
(groups of rounded lesions at the junction of GM and WM. WM changes seen as early as age
21 yrs.
Molecular Tests: NOTCH3 sequencing (>90%)
Disease Mechanism: NOTCH genes encode transmembrane receptors involved in cell fate
specification during development. The functional consequences of NOTCH3 mutations in the
abnormal protein are not known.
Treatment/Prognosis: supportive care, angiography and anticoagulants may precipitate CVA,
smoking increases risk of stroke. Mean age to walk with asst.: 60yrs, bedridden by 64yrs, med.
age of death 68 yrs.
145
NEUROLOGIC DISORDERS
CADASIL
(Cerebral Autosomal Dominant Arteriopathy with
Subcortical Infarcts and Leukoencephalopathy)
Natural history of the main clinical manifestations of CADASIL
146
(Chabriat, The Lancet Neurology, 2009)
NEUROLOGIC DISORDERS
CANAVAN DISEASE
Responsible gene: ASPA Protein: Aspartoacylase
Cytogenetic locus: 17pter-p13 Inheritance: AR
Clinical Features and Diagnostic Criteria: Macrocephaly, lack of head
control, developmental delays by age 3-5 mos, severe hypotonia, never sit,
walk, or speak. Hypotonia evolves to spasticity.
Clinical Tests: High urine N-acetyl aspartic acid (NAA)
Molecular Tests: 3 common mutations account for 99% of disease-causing
alleles in Ash. Jewish, 50-55% in non-jewish.
Disease Mechanism: Absence of aspartoacylase leads to build up of NAA
in the brain leading to demyelination
Treatment/Prognosis: Supportive care: nutrition, hydration, managing
infectious disease, protecting airway. Life expectancy to the teens.
147
599
NEUROLOGIC DISORDERS
CANAVAN DISEASE
Macrocephaly
and Hypotonia
www.canavanresearch.org
148
NEUROLOGIC DISORDERS
FAMILIAL DYSAUTONOMIA
Responsible gene: IKBKAP
Protein: IkappaB kinase complex-associated protein
Cytogenetic locus: 9q31
Inheritance: AR
Clinical Features and Diagnostic Criteria: Progressive, GI dysfunction, vomiting crises,
recurrent pneumonia, altered sensitivity to pain and temperature, CV instability, autonomic
crises, hypotonia, broad based ataxic gate deteriorates, decreased life expectancy. Dec
taste and absence of fungiform papillae of the tongue, dec or absent DTR’s, absence of
overflow tears with crying
Clinical Tests: Pupillary hypersensitivity to parasympathetic agents, absence of axon flare
response after intradermal histamine injection
Molecular Tests: IVS20 (+6T>C); R696P in IKBKAP (>99% Ashkenazi Jewish population)
Disease Mechanism: Abnormal development and survival of sensory, sympathetic and
parasympathetic neurons
Treatment/Prognosis: Aspiration precautions, hydration and elastic stockings for orthostatic
hypotension, protect cornea with artificial tears, PT for contracture
149
NEUROLOGIC DISORDERS
FAMILIAL DYSAUTONOMIA
Smooth tongue without typical vascularized
fungiform papillae
(Axelrod and Gold-von Simson Orphanet Journal of Rare Diseases 2007)
150
600
NEUROLOGIC DISORDERS
FRAGILE X
Responsible gene: FMR-1
Protein: FMRP (Fragile X Mental Retardation Protein)
Cytogenetic locus: Xq27.3
Inheritance: X-linked triplet repeat
Clinical Features and Diagnostic Criteria: Delayed motor and verbal development, ID (mod-severe
in boys, milder in girls), prominent jaw and forehead, high activity, autistic features. Carrier females:
anxiety, OCD, depression, 20% have POF. Carrier Males: (>30% of males >50y), progressive intention
tremor, ataxia, parkinsonism, and autonomic dysfunction. Two other loci: FraXE: only ID, FraXF: no
phenotype
Clinical Tests: None
Molecular Tests: CGG triplet repeat detection. Southern Blot: good for small or large expansions,
doesn’t give repeat #. PCR: Better quantification of repeat number, subject to allele dropout with large
expansions. NL: 5-44 repeats, Intermediate: 45-58 repeats (gray zone), Pre-mutation: 59-200 repeats,
Mutation: >200 repeats
Disease Mechanism: >200 repeats leads to silencing by methylation. POF and ataxia thought to be
due to toxic gain of function.
Treatment/Prognosis: No specific treatment.
151
NEUROLOGIC DISORDERS
FRAGILE X
Facial Features:
Long face,
Prominent
forehead
Large ears
Prominent jaw
152
(suzannebalvanz.blogspot.com/2007_07_01_archive.html)
NEUROLOGIC DISORDERS
HUNTINGTON DISEASE
Responsible gene: HD
Protein: Huntington
Cytogenetic locus: 4p16.3
Inheritance: AD
Clinical Features and Diagnostic Criteria: progressive motor disability involving both
involuntary and voluntary movement (chorea, dysarthria, dysphagia progress to bradykinesia,
rigidity, and dystonia) , cognitive decline (problems with planning or organization), psychiatric
disturbances (personality change, affective psychosis, or schizophrenic psychosis. Mean age
of onset 35-44 yrs (juvenile onset <20yrs ~10%).
Clinical Tests: CT or MRI: characteristic atrophy of caudate and putamen. PET scan: dec
uptake and metab. of glucose in the caudate nucleus (often abnl before MRI or CT).
Molecular Tests: Targeted mut. analysis: trinucleotide CAG repeat expansion >36. 27-35:
no symptoms but, if male, risk of expansion in children (6-10% risk of expansion with 35
repeats). 36-39: reduced penetrance, may never develop symptoms. >40: fully penetrant.
>60 repeats: juvenile onset.
Disease Mechanism: Unknown
Treatment/Prognosis: Tx is symptomatic: neurolepetics, benzo’s, psychotropics, Median
survival time: 15-18 yrs after onset, average age of death is 55 yrs. Suicide in 12%.
153
601
NEUROLOGIC DISORDERS
HUNTINGTON DISEASE
www.Nature.com
154
NEUROLOGIC DISORDERS
KRABBE DISEASE
Responsible gene: GALC
Protein: Galactocerebrocidase
Cytogenetic locus: 14q31
Inheritance: AR
Clinical Features and Diagnostic Criteria: Infantile form: irritability to sensory stimuli,
muscle hypertonicity, progressive neurologic deterioration, peripheral neuropathy, white
matter disease, elevated CSF protein. Later onset (6 mos to 5th decade): weakness, vision
loss, intellectual regression.
Clinical Tests: CT: nonspecific- diffuse cerebral atrophy of grey and white matter. MRI:
demyelination of the brainstem and cerebellum. Dec GALC enzyme activity (0-5% of
normal activity). Abnl EEG, low nerve conduction velocity,
Molecular Tests: GALC targeted mutation analysis: GALC 30-kb deletion (45% of
Europeans, 35% of Mexicans); 809G>A mutation (50% of late onset Krabbe). GALC
sequencing (virtually 100%)
Disease Mechanism: Missense mutations result in unstable protein that is rapidly
degraded
Treatment/Prognosis: Hematopoietic stem cell transplant decreases morbidity and
mortality when given to infants before they show symptoms. Supportive care to control
irritability and spasticity if diagnosed when symptomatic. Infantile form: average age of
death is 13 mos due to infections or resp failure.
155
NEUROLOGIC DISORDERS
KRABBE DISEASE
(http://disorders.eyes.arizona.edu/disorders/krabbe-disease)
156
602
NEUROLOGIC DISORDERS
NEUROFIBROMATOSIS TYPE I
Responsible gene: NF1
Protein: Neurofibromin
Cytogenetic locus: 17q11
Inheritance: AD
Clinical Features and Diagnostic Criteria: 2 or more of: 6x5mm (prepubertal) or 6x15mm
(postpubertal) café au lait, 2 or more neurofibromas, one plexiform neurofibroma, axillary or
inguinal freckling, optic glioma, 2 or more Lisch nodules, sphenoid dysplasia or thickening of
long bone cortex, 1st degree relative with NF-1
Clinical Tests: x-ray, eye exam, brain MRI
Molecular Tests: >500 mutations reported, usually unique to a particular family
Disease Mechanism: Loss of function mutations impair ras GTPase mediated cellular
proliferation and tumor suppression
Treatment/Prognosis: The majority live normal lifespan. Surgery for bone malformations or
painful or disfiguring tumors; clinical trials and use of MEK inhibitors for plexiform neurofibromas.
Risk of malignant peripheral nerve sheath tumors in adolescence and young adulthood.
157
NEUROLOGIC DISORDERS
NEUROFIBROMATOSIS TYPE I
runkle-science.wikispaces.com
158
NEUROLOGIC DISORDERS
PARKINSON DISEASE
Responsible gene: Multiple, main gene PARK2
Protein: Parkin
Cytogenetic locus: 6q25.2-q27
Inheritance: AD, AR, multifactorial
Clinical Features and Diagnostic Criteria: bradykinesia, rigidity, and tremor,
asymmetric limb involvement. Juvenile Onset AR PARK2 mutations, typical
features, onset 20-40yrs.
Clinical Tests: Good response to L-Dopa
Molecular Tests: PARK2 sequencing
Disease Mechanism: Unclear but thought to be due to loss of function by absent
protein or protein inactivation
Treatment/Prognosis: Dopamine therapy, PT, OT, ST. Some patients may benefit
from palliodotomy or deep brain stimulation of the subthalamic nucleus.
159
603
NEUROLOGIC DISORDERS
PARKINSON DISEASE
160
http://www.holisticonline.com/images/PD-ama-schematic1.GIF
NEUROLOGIC DISORDERS
RETT SYNDROME
Responsible genes: MECP2
Proteins: MECP2
Cytogenetic loci: Xq28
Inheritance: XLD
Clinical Features and Diagnostic Criteria: ID, developmental regression (especially
language and hand use), acquired microcephaly, stereotypical wringing hand movements,
hyperventilation, bruxism, paroxysmal laughing, prolonged QT, scoliosis
Clinical Tests: EEG (nonspecific for Rett), ECG
Molecular Tests: MECP2 sequencing (>80%), Need to test parents if a novel variant found.
MECP2 MLPA or quantitative PCR testing for deletion (~16%).
Disease Mechanism: Decreased function of loss-of-function of MECP2. Normally MECP2
binds methylated CpG islands.
Treatment/Prognosis: Seizures are often difficult to manage, SSRI’s for agitation, monitor
for scoliosis, periodic ECG to monitor for long QT
Small subset have CDKL5 mutations and present atypically with early onset seizures
161
RETT SYNDROME
NEUROLOGIC DISORDERS
Genetic testing
strategy
www.lbl.gov
162
(Williamson and Christodoulou EJHG 2006)
604
NEUROLOGIC DISORDERS
WILSON DISEASE
Responsible gene: ATP7B Protein: Copper-transporting ATPase 2
Cytogenetic locus: 13q14.3-q21.1 Inheritance: AR
Clinical Features and Diagnostic Criteria: Can present age 3-50 yrs. Liver
disease: jaundice, self-limited hepatitis-like illness, autoimmune hepatitis,
hepatic failure, chronic liver disease. Neurologic presentation: movement
disorder, disorganization of personality
Clinical Tests: Kayser-Fleisher rings on corneal exam, low serum Cu and
ceruloplasmin, inc urinary copper excretion. Liver bx: inc copper storage.
Molecular Tests: ATP7B sequencing (98%). H1069Q (35-45% Europeans),
R779L (57% Asians), H714Q and delC2337 (40% Russians).
Disease Mechanism: Loss of ATP7b function impairs holoceruloplasmin
biosynthesis and biliary copper excretion with resultant copper-mediated
oxidative damage, activation of cell death pathways, leakage of copper into
plasma and eventual tissue copper overload.
Treatment/Prognosis: Chelating agents, liver transplant
163
NEUROLOGIC DISORDERS
WILSON DISEASE
Kayser-Fleisher ring
www.kellogg.umich.edu
164
NEUROMUSCULAR DISORDERS
AMYOTROPHIC LATERAL SCLEROSIS
Responsible genes: SOD1 (rare: SETX, VAPB, BSCL2, VCP, ALS2, SPG20, others)
Protein: Superoxide dismutase
Cytogenetic locus: 21q22
Inheritance: AD (AR ALS2 and SPG20)
Clinical Features and Diagnostic Criteria: UMN: hyperreflexia, extensor plantar response,
inc muscle tone, and weakness. LMN: weakness, muscle wasting, hyporeflexia, muscle
cramps, and fasciculations. Frontotemporal dimentia
Clinical Tests: EMG; Path: (1) degeneration and loss of the motor neurons in the anterior
horns and in the motor nuclei of cranial nerves VII, X, and XI and most commonly the
hypoglossal nucleus; and (2) axonal loss with decreased myelin staining in the lateral and
anterior corticospinal tracts
Molecular Tests: SOD1 mutation (20% familial, 3% sporadic ALS- 50% have the A4V Exon 1
mutation)
Disease Mechanism: Toxic gain of function, not enzyme deficiency (SOD1 prevents
oxidative damage to cells)
Treatment/Prognosis: Primarily palliative, Riluzole (glutamate inhibitor) FDA-approved drug.
Mean age of onset: 46 yrs if familial, 56 yrs if sporadic. Death usually caused by resp.
muscle compromise.
165
605
NEUROMUSCULAR DISORDERS
AMYOTROPHIC LATERAL SCLEROSIS
http://www.esperanzapeptide.net/images/treatment-mnd.jpg
166
NEUROMUSCULAR DISORDERS
CHARCOT MARIE TOOTH DISEASE
CMT1: Abnormal myelin, AD, 50% of all CMT, PMP22 (17p11.2), MPZ (1q22), LITAF (16p13.1-p12.3),
EGR2 (10q21.1-q22.1), NEFL (8p21)
CMT2: Axonopathy, AD, 20-40% of all CMT, KIF1B and MFN2 (1p36.2), RAB7 (3q21), LMNA (1q21.2),
GARS (7p15), NEFL (8p21), HSPB1 (7q), MPZ (1q22), GDAP1 (8q12-q21.1)
CMT Intermediate Form: Combination of myelinopathy and axonopathy, AD, rare cause of CMT, DNM2
(19p12-p13.2), YARS (1p34-p35)
CMT 4: Either myelinopathy or axonopathy, AR, rare cause of CMT, GDAP1 (8q13-q21.1), MTMR2
(11q22), CMT4B2 (11p15), SH3TC2 (5q32), NDRG1 (8q24.3), EGR2 (10q21.1-q22.1), PRX (19q13.1q13.2
CMTX: Axonopathy with secondary myelin changes, XLD, 10-20% of all CMT, GJB1 (Xq13.1).
Clinical Features and Diagnostic Criteria: slowly progressive weakness and atrophy of distal muscles
in the feet and/or hands beginning in the 1st-3rd decade; hearing loss; pes cavus foot deformity, hip
dysplasia.
Clinical Tests: nerve conduction studies, nerve biopsy
Molecular Tests: Gene sequencing, deletion/duplication analysis
Disease Mechanism: Abnormal peripheral myelination
Treatment/Prognosis: orthopedic surgery, TCA’s, carbamazepine, or gabapentin for neuropathic pain.
167
NEUROMUSCULAR DISORDERS
CHARCOT MARIE TOOTH DISEASE
Pes cavus foot deformity
Finger contracture
168
http://findmeacure.com/2011/03/22/charcot-marie-tooth-diseasecmt/
ACMG Genetics and Genomics Review
Course June 18-21, 2015
606
NEUROMUSCULAR DISORDERS
DUCHENNE AND BECKER MUSCULAR DYSTROPHY
Responsible gene: DMD
Protein: Dystrophin
Cytogenetic locus: Xp21.2
Inheritance: XLR
Clinical Features and Diagnostic Criteria: DMD: Symptoms present before age 5, progressive
symmetrical muscular weakness, proximal>distal, calf hypertrophy, dilated cardiomyopathy (DCM).
BMD: Later onset, less severe, weakness of quadriceps may be only sign, activity induced cramping.
Preservation of neck flexor muscles (unlike DMD). DCM can occur in isolation
Clinical Tests: CK 10x nl in DMD, 5x nl in BMD. Unreliable test for carrier females, tends to decrease
with age.
Molecular Tests: Multiplex PCR: DMD gene deletion (65% DMD, 85% BMD). Southern or quantitative
PCR for gene duplication (6% DMD), DMD sequencing for small del/ins or point mutations (30% DMD)
Disease Mechanism: Dystrophin binds actin and other membrane proteins. Mutations that lead to lack
of dystrophin expression: DMD, those that lead to abnormal quality or quantity of dystrophin: BMD.
Treatment/Prognosis: Supportive therapy, steroids may prolong walking 2-3 yrs. DMD: wheelchair
dependent by age 13, ventilator by age 20, survival into 20’s. BMiDs: Wheelchair after age 16 (if at all),
survival 40-50’s. Carrier females at risk for DCM. Exon skipping therapies, stop mutation readthrough
under investigation.
169
NEUROMUSCULAR DISORDERS
DUCHENNE AND BECKER MUSCULAR DYSTROPHY
http://img.orthobullets.com/Pediatrics/Neuromuscular%20problems/Duchenes%20Muscular%20Dystroph
y/Images/dystrophin_stains.jpg
170
NEUROMUSCULAR DISORDERS
FRIEDREICH ATAXIA
Responsible gene: FRDA
Protein: Frataxin
Cytogenetic locus: 9q13
Inheritance: AR
Clinical Features and Diagnostic Criteria: Progressive degeneration of the dorsal root
ganglia, posterior columns, corticospinal tracts, and the dorsal spinocerebellar tracts of the
spinal cord and cerebellum. There is progressive limb and gait ataxia before age 25 yrs,
absent tendon reflexes in the lower extremities. Within 5 years of disease onset: dysarthria,
arefelxia, pyrimidal weakness of the legs, extensor plantar responses and distal loss of joint
position and vibration sense. Also, scoliosis, pes cavus, optic nerve atrophy, hypertrophic
cardiomyopathy, DM or glucose intolerance
Clinical Tests: electrophysiologic evidence of axonal sensory neuropathy
Molecular Tests: GAA triplet repeat expansion in FRDA intron 1 (96% homozygous) Normal
5-33, premutation 34-65, and disease causing: 66-1700 repeats.
Disease Mechanism: It is believed that GAA expansion forms a stable DNA structure that
interferes with transcription
Treatment/Prognosis: Treatment is supportive: psychological, prostheses, walking aids,
wheelchairs, PT, and ST
171
607
NEUROMUSCULAR DISORDERS
FRIEDREICH ATAXIA
Model of
alteration in iron
uptake in
Friedreich's
ataxia.
Richardson D R et al.
PNAS 2010;107:1077510782
172
NEUROMUSCULAR DISORDERS
HEREDITARY NEUROPATHY WITH
LIABILITY TO PRESSURE PALSIES
Responsible gene: PMP22
Protein: Peripheral myelin protein 22
Cytogenetic locus: 17p11.2
Inheritance: AD
Clinical Features and Diagnostic Criteria: adult with recurrent focal
pressure palsies, mild polyneuropathy, absent ankle reflexes, reduced DTRs,
mild-mod pes cavus deformity
Clinical Tests: Prolongation of distal nerve conduction latencies (virtually
100%), normal general motor nerve conduction velocities, demyelination and
tomaculous (focal nerve enlargement) on sural nerve biopsy
Molecular Tests: PMP22 sequencing (20%), 1.5-Mb PMP22 deletion (80%)
Disease Mechanism: HNPP is associated with decreased mRNA message
for PMP22 and decreased peripheral myelin protein 22 in peripheral nerve.
Treatment/Prognosis: Bracing, AFO for foot drop, unclear if surgical nerve
decompression is helpful, avoid risk factors for pressure palsy: prolonged
sitting with legs crossed, repetitive wrist movements, prolonged leaning on
elbows, and rapid weight loss.
173
NEUROMUSCULAR DISORDERS
LIMB-GIRDLE MUSCULAR
DYSTROPHY
Responsible gene (protein, cytogenetic locus): CAPN3 (Calpain 3, 15q15.1-q21.1), FKRP
(Fukutin related protein, 19q13.1), LMNA (Lamin-A/C, 1q21.2), SGCA (alpha sarcoglycan,
17q12), SGCB beta sarcoglycan, 4q12), SGCD (delta-sarcoglycan, 5q33), SGCG (gammasarcoglycan, 13q12), DYSF (Dysferlin, 2p13.3)
Inheritance: most AR, some rare AD subtypes
Clinical Features and Diagnostic Criteria: AR Sarcoglycan LGMD: proximal limb
weakness, difficulty running and walking, calf hypertrophy, onset age 3-15 (68% of childhood
onset, 10% adult onset) Calpain AR LGMD proximal limb weakness, difficulty running and
walking, calf atrophy, onset 2-40 yrs (10-30% AR LGMD). Dysferlin AR LGMD problems
running and walking, foot drop, distal and/or pelvic weakness, transient calf hypertrophy,
onset 17-23 yrs
Clinical Tests: Inc serum CPK, dystrophic changes on muscle biopsy, sarcoglycan protein
staining
Molecular Tests: Gene sequencing (80-99%)
Disease Mechanism: Sarcoglycanopathies disrupt dystrophin-dystroglycan complex,
calpainopathy: unknown, dysferlinopathy: may be die to abnl membrane fusion
Treatment/Prognosis: Supportive care to promote mobility and ambulation. Monitor for
respiratory and orthopedic complications and for cardiomyopathy
174
608
NEUROMUSCULAR DISORDERS
MYOTONIC DYSTROPHY TYPE 1
Responsible gene: DMPK
Protein: Myotonin-protein kinase
Cytogenetic locus: 19q13.32 Inheritance: AD
Clinical Features and Diagnostic Criteria: Multisystem disorder of skeletal and smooth
muscle, eyes, heart, endocrine system, and CNS.
MILD cataract and mild myotonia (50-150 repeats)
Classic muscle weakness and wasting, myotonia, cataract, and arrhythmia (100-1000
repeats). Have grip myotonia (sustained muscle contraction leads to inability to quickly
release a hand grip)
Congenital hypotonia and severe generalized weakness at birth often with resp. insufficiency
and early death, MR is common (>2000 repeats)
Clinical Tests: EMG, serum CK, muscle biopsy (internal nuclei, ring fibers, sarcoplasmic
masses, type I fiber atrophy, inc # intrafusal muscle fibers), slitlamp exam
Molecular Tests: CTG triplet repeat at the 3’-UTR of the DMPK (100%). PCR: detect repeats
up to ~100, southern blot (detect repeats>100)
Disease Mechanism: Cause thought to be due to gain of function RNA mechanism- the
CUG repeats alter alternative splicing of other genes, including a CL- channel, resulting in
myotonia
Treatment/Prognosis: Symptomatic only
175
NEUROMUSCULAR DISORDERS
MYOTONIC DYSTROPHY TYPE 1
Three ring
fibers (one
marked),
atrophic
myofibers,
and central
nuclei
http://neuropathology.neoucom.edu/chapter13/chapter13cDystrophy.html
176
NEUROMUSCULAR DISORDERS
NEMALINE MYOPATHY
Gene (protein, chromosomal locus): ACTA1 (Actin, alpha skeletal muscle, 1q42.1), NEB
(Nebulin, 2q22), TNNT1 (Troponin T, slow skeletal muscle, 19q13.4), TPM2 (Tropomyosin
beta chain, 9p13.2-p13.1), TPM3 (Tropomyosin alpha-3 chain, 1q22-q23), RARE: CFL2
(Cofilin-2, 14q13.1), KBTBD13 (Kelch repeat and BTB domain containing protein 13,
15q22.31), KHLH40 (Kelch-like protein 40, 3p22.1), and KHLH41 (Kelch-like protein 41,
2q31.1)
Inheritance: AR or AD
Clinical Features and Diagnostic Criteria: weakness, hypotonia, and depressed or absent
DTR’s. Weakness is usually most severe in the face, neck flexors, and proximal limb
muscles. Age of onset: congenital, childhood, or adulthood.
Clinical Tests: Muscle biopsy: the diagnostic hallmark is the presence of rod-like inclusions,
nemaline bodies, in the sarcoplasm of skeletal muscle fibers with trichrome staining.
Molecular Tests: ACTA sequencing: 15-25% of NM, ACTA Del/dup analysis: Exon 55.
Disease Mechanism: NM is a disorder of thin filament anchoring proteins
Treatment/Prognosis: No definitive correlation between # of rods and severity of the
myopathy. Walking prior to 18 months is predictive of survival.
177
609
NEMALINE MYOPATHY
NEUROMUSCULAR DISORDERS
Nemaline inclusion
www.pathology.vcu.edu
178
NEUROMUSCULAR DISORDERS
SPINAL MUSCULAR ATROPHY
Responsible genes: SMN1, SMN2
Proteins: survival motor neuron protein 1 and 2
Cytogenetic loci: 5q12.2-q13.3
Inheritance: AR
Clinical Features and Diagnostic Criteria: arthrogryposis multiplex congenita, peripheral
nerve hypomyelination. SMA I onset 0-6mo, muscle weakness, tongue fasiculations, absent
DTRs SMA II muscle weakness onset after 6 months, finger trembling, low tone, absent
DTRs, SMA III Weakness leads to frequent falls or trouble with stairs, onset 2-3yrs, proximal
weakness (legs>arms), SMA IV adult onset
Clinical Tests: EMG: denervation and diminished motor action potential amplitude. Muscle
Bx: atrophy of type 1 and type 2 fibers
Molecular Tests: Targeted mutation analysis: deletion of SMN1 exon 7 deletion (95-98%),
SMN1 sequencing (2-5%). Carriers who have two copies of SMN1 in cis (~4% of the
population) will be misdiagnosed as non-carriers. SMN2 copy # modifies the severity. 2
copies SMN2- SMA I, 3 copies- SMA II, 4-8 copies- SMA III. Absence of both SMN genes:
lethal
Disease Mechanism: Mutant SMN lacks the splicing-regeneration activity of wild type.
Treatment/Prognosis: Optimize feeding and nutrition, PFT’s, sleep study for OSA, treat
contractures, dislocations, and scoliosis; Nusinersen treatment
179
NEUROMUSCULAR DISORDERS
SPINAL MUSCULAR ATROPHY
Diagnostic algorithm for SMA
(Lunn and Wang, The Lancet, 2008)
180
610
NEUROMUSCULAR DISORDERS
SYNDROMIC CONGENITAL
MUSCULAR DYSTROPHY
(Fukuyama (FCMD), Muscle-Eye-Brain (MEB), Walker-Warburg (WWS), Congenital Muscular Dystrophy Type 1D
(MDC1D)
Responsible gene (protein, cytogenetic locus): FCMD; FCMD (Fukutin, 9q31); MEB: POMGNT1 (protein Omannosidase beta-1,2-N-acetylglucosaminyltransferase, 1p34-p33); WWS: POMT1 and POMT2 (Protein Omannosyltransferase 1 and 2, 9q34.1, and 14q24.3); MDC1D (LARGE, glycosyltransferase-like protein LARGE,
22q12.3-q13.1
Inheritance: AR
Clinical Features and Diagnostic Criteria: Muscle weakness present at birth. Hypotonia and weakness. Joint
contracture (MEB and WWS: elbow, FMD: hip, knee, ankle elbow).
Clinical Tests: Muscle bx: dystrophic or myopathic pattern; inc serum CK; Muscle Bx: immunostaining; Brain MRI:
Cobblestone complex (enlarged lat ventricles, flat brainstem, cerebellar hypoplasia)
Molecular Tests:
Disease Mechanism: Disruption of alpha dystroglycan (an integral component of the dystrophin-glycoprotein
complex)
Treatment/Prognosis: Weight control, PT, assist devices for ambulation, surgical correction of orthopaedic
problems, monitoring of respiratory function
181
NEUROMUSCULAR DISORDERS
SYNDROMIC CONGENITAL
MUSCULAR DYSTROPHY
Head lag due to
hypotonia
http://neuromuscular.wustl.edu/syncm.html
NEUROMUSCULAR DISORDERS
TAY-SACHS DISEASE
Responsible gene: HEXA
Protein: Hexosaminidase A
Cytogenetic locus: 15q23-q24
Inheritance: AR
Clinical Features and Diagnostic Criteria: Infantile weakness starts at 6 mo, exaggerated
startle, seizures and vision loss by the end of the first year, neurodegeneration continuesdeaf, cannot swallow, weakening of muscles, and eventual paralysis, death in toddler years.
Juvenile muscle coordination problems, seizures, and vision problems starting as young
children. Chronic and adult onset start later, progress more slowly, more rare.
Clinical Tests: HEXA enzyme activity, cherry red spot on eye exam
Molecular Tests: Follow enzyme testing with DNA testing (some with a positive enzyme
assay have a pseudodeficiency allele that does not cause Tay Sachs). HEXA 6 common
mutation panel: 92% of Ashkenazi Jewish
Disease Mechanism: Accumulation of GM2 gangliosides in the brain
Treatment/Prognosis: Supportive only
183
611
NEUROMUSCULAR DISORDERS
TAY-SACHS DISEASE
Cherry red spot
of the macula
http://themedicalbiochemistrypage.org/images/cherryredspot.jpg
184
ONCOLOGIC DISORDERS
BRCA1 and BRCA2
Hereditary Breast/Ovarian Cancer
Responsible genes: BRCA1 and BRCA2
Proteins: Breast cancer type 1 and 2 susceptibility protein
Cytogenetic loci: 17q21, 13q12.3 Inheritance: AD
Clinical Features and Diagnostic Criteria: BRCA 1 and 2: Br, ovarian, prostate cancer. BRCA2:
pancreatic
Clinical Tests: mammography, MRI, BRCA1-related breast tumors show an excess of medullary
histopathology, are of higher histological grade, and are more likely to be estrogen receptor-negative
and progesterone receptor-negative. BRCA1-related ovarian cancer: excess of serous
adenocarcinomas
Molecular Tests: Full gene sequencing and deletion analysis (3-5%). Overall, about 3-5% of reports
have variants of uncertain clinical significance). Ashkenzi founder 187delAG and 5382insC (BRCA1),
and 6174delT (BRCA2) mutations are found in 20-30% of Jewish women with early breast cancer and in
45-60% of Jewish women diagnosed with ovarian cancer. Dutch women with early br or ovarian ca:
often one of 3 large BRCA1 deletions. BRCA2 999del5 occurs in 7.7% of women and 40% of men with
breast cancer from Iceland. Three Ashkenazi founders found in 2.4% of individuals of Ashkenazi Jewish
descent.
Disease Mechanism: BRCA1 and 2 are tumor suppressor genes
Treatment/Prognosis: Prophylactic salpingo-oopherectomy, breast MRI, chemoprevention trials, and
options for prophylactic surgery. 85% will develop Br ca by age 70 yrs.
185
ONCOLOGIC DISORDERS
BRCA1 and BRCA2
Hereditary Breast/Ovarian Cancer
186
612
ONCOLOGIC DISORDERS
FAMILIAL ADENOMATOUS
POLYPOSIS
Responsible gene: APC
Protein: Adenomatous polyposis coli protein
Cytogenetic locus: 5q21-22
Inheritance: AD (15-30% new mutation)
Clinical Features and Diagnostic Criteria: adenomatous colonic polyps (100-1000) in
childhood to adolescence, abdominal desmoid tumors, jaw osteoma,
absent/supernumerary/malformed teeth, hepatoblastoma, thyroid cancer, epidermoid cysts.
Attenuated FAP: fewer polyps, more proximal in the colon. Gardner syndrome: colonic
adenomatous polyposis, osteomas, and soft tissue tumors. Turcot syndrome: colon cancer
and CNS tumors (medulloblastoma more common in FAP)
Clinical Tests: Clinical findings on colonoscopy
Molecular Tests: APC sequence analysis abnormal (~90%), deletion (~5%)
Disease Mechanism: Decrease in APC protein results in lack of b-catenin proteasome
degradation and high levels of nuclear b-catenin protiein, binds to a transcription factor Tcf-4
or Lef-1 (T cell factor-lymphoid enhancer factor), and may activate the oncogenes c-Myc and
cyclin D1
Treatment/Prognosis: Without colectomy, colon cancer is inevitable, and prophylactic
colectomy is recommended in late teen-age years. The mean age of cancer in untreated
individuals is 39 years.
187
ONCOLOGIC DISORDERS
FAMILIAL ADENOMATOUS
POLYPOSIS
www.oncolink.com
Colon with multiple adenomas
188
HEREDITARY NONPOLYPOSIS
COLON CANCER (LYNCH SYNDROME)
ONCOLOGIC DISORDERS
Responsible gene (protein and cytogenetic locus): MLH1 (3p21.3, DNA mismatch repair protein MLH1),
MSH2 (2p22-p21, DNA mismatch repair protein Msh2), MSH6 (2p16, DNA mismatch repair protein MSH6), and PMS2
(7p22, PMS1 protein homolog 2) Inheritance: AD
Clinical Features and Diagnostic Criteria: HNPCC-related tumors: colon, endometrium, stomach, ovary,
hepatobiliary tract, urinary tract, small bowel, brain/CNS. Amsterdam II Criteria: 3 or more family members (at least
one 1st degree of the other 2) with HNPCC related cancers; 2 successive affected generations; 1 or more of the
HNPCC-related cancers diagnosed before age 50; exclusion of FAP. Bethesda 2004: CRC diagnosed under age
50yrs, 2 HNPCC related tumors at once, CRC with high MSI in someone <age 60yrs, CRC in one or more 1st degree
relatives with and HNPCC related tumor with 1 cancer diagnosed before age 50yrs, or CRC diagnosed in 2 or more 1st
or 2nd degree relatives (any age)..
Clinical Tests: Microsatellite instability (MSI) of tumor tissue, immuno-histochemistry of tumor tissue for the
presence or absence of DNA mismatch repair proteins MLH1, MSH2, MSH6 and PMS2.
Molecular Tests: Sequencing of all genes MLH1 (90-95%), MSH2 (50-80%). Deletion analysis MLH1 (5-10%),
MSH2 (10-20%), MSH6 and PMS2 (less than 10% of Amsterdam criteria families combined). EPCAM deletion.
Disease Mechanism: These proteins work in a recessive manner at the cellular level- LOH leads to absence of any
functional protein and dysfunctional mismatch repair.
Treatment/Prognosis: 70-80% lifetime risk of CRC. Colonoscopy every 1-2yrs by age 20-25. Current guidelines do
not recommend endometrial sampling unless symptomatic. If CRC present, full colectomy with ileorectal anastomosis
considered. Tumors respond better to immune checkpoint inhibitors.
Recessive form – note rare AR constitutional mismatch repair deficiency syndrome with childhood cancer risk.
189
613
ONCOLOGIC DISORDERS
HEREDITARY NONPOLYPOSIS
COLON CANCER (LYNCH SYNDROME)
www.mdanderson.org/images/hnpcc
190
ONCOLOGIC DISORDERS
LI-FRAUMENI SYNDROME
Responsible genes: TP53
Proteins: Cellular tumor antigen P53
Cytogenetic locus: 17p13
Inheritance: AD
Clinical Features and Diagnostic Criteria: Proband with sarcoma <age 45 yrs, 1st deg
relative with cancer <45 yrs, and 1st or 2nd deg relative with any cancer <45yrs or a sarcoma at
any age. Increased risk of multiple primary tumors: bone, cartilage, and soft tissue sarcoma;
early onset breast cancer; brain tumors (including choroid plexus carcinoma), childhood
adrenocortical tumors, also GI malignancies, lung cancer and neuroblastoma.
Clinical Tests: Pathology
Molecular Tests: TP53 sequencing and deletion analysis
Disease Mechanism: Abnormal DNA repair and genomic instability: P53 protein plays a role
in determining whether cells undergo arrest for DNA repair or apoptosis
Treatment/Prognosis: Toronto protocol (extensive screening with whole body MRI, brain MRI,
breast MRI and abdominal U/S, biochemical screening for adrenal tumors) shown to improve
mortality. Avoid or minimize exposure to radiation.
191
ONCOLOGIC DISORDERS
LI-FRAUMENI SYNDROME
Journal of Clinical Oncology, Vol 27, No 26 (September 10), 2009: pp e108-e109
192
614
ONCOLOGIC DISORDERS
MULTIPLE ENDOCRINE NEOPLASIA TYPE 1
Responsible gene: MEN1
Protein: Menin
Cytogenetic locus: 11q13
Inheritance: AD
Clinical Features and Diagnostic Criteria: MEN1= tumor in 2 of: parathyroid, enteropancreatic
endocrine tissue, or anterior pituitary OR Tumor in one and 1st degree relative with MEN1. Facial
angiofibroma, collagenoma, café au lait, lipoma
Clinical Tests: Parathyroid function studies, anterior pituitary hormone abnormalities, Brain MRI
Molecular Tests: MEN1 sequencing (70-90% familial, 65% sporadic), Dup/del testing (1-3%)
Disease Mechanism: MEN1 is a tumor suppressor gene by regulating transcription of proteins
involved in the regulation of cell proliferation and development
Treatment/Prognosis: biochemical testing of serum concentrations of calcium (from age 8 yrs),
gastrin (from age 20 yrs), pancreatic polypeptide (from age 10 yrs), prolactin (from age 5 yrs),
abdominal CT or MRI (from age 20 yrs) and head MRI (from age 5 yrs).
193
ONCOLOGIC DISORDERS
MULTIPLE ENDOCRINE NEOPLASIA TYPE 1
my.clevelandclinic.org/disorders/familial_multiple_endocrine_neoplasia
194
ONCOLOGIC DISORDERS
MULTIPLE ENDOCRINE NEOPLASIA Type 2
Responsible gene: RET
Protein: proto-oncogene tyrosine-protein kinase receptor ret
Cytogenetic locus: 10q11.2
Inheritance: AD
Clinical Features and Diagnostic Criteria: MEN2A two or more of medullary thyroid
carcinoma, pheochromocytoma, or parathyroid adenoma/hyperplasia in a single person or
close relatives. MEN2B mucosal neuromas of the lips and tongue, medullated corneal nerve
fibers, Marfanoid habitus, and medullary thyroid carcinoma
Clinical Tests: Calcitonin, catecholamines, catecholamine metabolites, Ca, PTH
Molecular Tests: RET sequencing: Exon 10 and 11 (95% MEN2A), Exon 16 (95% MEN2B)
Disease Mechanism: Gain of function mutations in RET lead to constitutive activation of
tyrosine kinase
Treatment/Prognosis: Prophylactic thyroidectomy (age dependent on specific mutations – by
age 1 for MEN2B, by age 5 for most of MEN2A, screen for pheochromocytoma annually and
prior to any surgery, annual calcitonin stim test, annual PTH screening.
195
615
ONCOLOGIC DISORDERS
NEUROFIBROMATOSIS Type 2
Responsible gene: NF2
Protein: Neurofibromin-2 (Merlin)
Cytogenetic locus: 22q12.2
Inheritance: AD
Clinical Features and Diagnostic Criteria: Benign nerve tumors (schwannomas,
meningiomas, ependymonas, astrocytoma). Hallmark is bilateral acoustic schwannoma,
onset age 18-24 yrs, hearing loss, tinnitus, balance problems. Also cataracts,
mononeuropathy, café-au-lait (fewer than in NF1).
Clinical Tests: MRI/CT, BAER, audiology evaluation, eye exam
Molecular Tests: NF2 sequencing (75%), dupl/del testing (10-15%)
Disease Mechanism: NF2 is a tumor suppressor, 2nd hit leads to complete loss of function
when one germline mutation present
Treatment/Prognosis: Symptomatic tumors removed surgically (XRT may induce tumor
formation). Bevacizumab may shrink vestibular tumors and stabilize hearing in some patients.
196
ONCOLOGIC DISORDERS
NEUROFIBROMATOSIS Type 2
www.nfcalifornia.org/DiagnosticNF2
197
ONCOLOGIC DISORDERS
PTEN HAMARTOMA TUMOR SYNDROME
Responsible gene: PTEN
Protein: Phosphatidylinositol-3,4,5-trisphosphate 3-phosphatase and dual-specificity protein
phosphatase
Cytogenetic locus: 10q23 Inheritance: AD
Clinical Features and Diagnostic Criteria: Cowden: Presents 2nd/3rd decade: mucocutaneous facial
and oral papules, gingival cobblestoning, acral keratosis; dystrophic and adenomatous multinodular
goiter, GI polyps, adenosas and fibrocystic breast lesions, macrocephaly, dolichocephaly, lipomas, GU
anom. High risk for breast, thyroid, and endometrial cancer. Bannayan-Riley-Ruvalcaba (BRR)
macrocephaly, polyposis, lipomas, pigmented macules of the glans penis. Clinical Tests: Lesion
pathology, MRA/MRI, CT
Molecular Tests: PTEN seq (80%), promoter region mutations (10%)
Disease Mechanism: Wild-type protein is a major lipid phosphatase that downregulates the PI3K/Akt
pathway to cause G1 arrest and apoptosis
Treatment/Prognosis: Annual derm exam, annual breast exam, annual breast MRI and mammography
starting age 30, annual thyroid US starting age 14,
Proteus syndrome: Distinct disorder - CT nevi, disprop. overgrowth, dysregulated adipose tissue,
vascular malformation, risk of ovarian or parotid tumor in 2nd decade – somatic variants in AKT1
198
616
PTEN HAMARTOMA TUMOR SYNDROME
ONCOLOGIC DISORDERS
Cowden Syndrome
Proteus syndrome
www.uveitis.org/images/Image1.jpg
http://www.childrenshospital.org/az/Site1965
199
ONCOLOGIC DISORDERS
TUBEROUS SCLEROSIS COMPLEX
Responsible genes: TSC1 and TSC2 Proteins: Hamartin and Tuberin
Cytogenetic loci: 9q34, 16p13
Inheritance: AD (2/3 de novo)
Clinical Features and Diagnostic Criteria: Skin: hypomelanotic macules, facial
angiofibroma, shagreen patch, ungual fibromata. CNS: subependymal glial nodules, cortical
tubers, giant cell astrocytoma, seizures. Renal: angiomyolipomas, epithelial cysts, <1%
malignant transformation Heart: cardiac rhabdomyoma, tend to regress in infancy without
intervention. Lung: lymphangiomatosis (TSC2, women aged 20-40 yrs) Eye: hamartomas or
achromic patches. There is a TSC2/PCKD contiguous gene deletion syndrome with features
of TS and PKD.
Clinical Tests: brain MRI, echo, renal ultrasound, Wood’s lamp exam, eye exam, EEG
Molecular Tests: TSC1 sequencing (30% familial, 15% sporadic) and TSC2 sequencing
(50% familial and 60-70% sporadic)
Disease Mechanism: Abnormal tumor suppressor activity
Treatment/Prognosis: Renal US q1-3 yrs, renal CT/MRI if numerous lesions on US,
semiannual renal US if angiomyolipomas <3.5-4.0 cm, chest CT if pulmonary symptoms;
everolimus for renal angiomyolipoma or subependymal giant cell astrocytoma. mTOR
inhibitors, everolimus and others, used for TSC-associated tumors.
200
ONCOLOGIC DISORDERS
TUBEROUS SCLEROSIS COMPLEX
www.uwo.ca/.../pictures/tuberousclinical.jpg
201
617
ONCOLOGIC DISORDERS
VON HIPPEL-LINDAU SYNDROME
Responsible gene: VHL
Protein: Von Hippel –Lindau disease tumor suppressor
Cytogenetic locus: 3p25
Inheritance: AD
Clinical Features and Diagnostic Criteria: Hemangioblastoma (cerebellum, retina, spinal
cord), pheochromocytoma (hypertension), renal cell carcinoma (40%), endolymphatic sac
tumor. Somatic VHL mutations frequently seen in sporadic VHL associated tumors.
Clinical Tests: Dilated eye exam, CT or MRI, urine catecholamine metabolites, renal US
Molecular Tests: VHL sequencing (72%), Partial or complete gene deletion (28%)
Disease Mechanism: Abnormal tumor suppressor function. Truncating or missense
mutations that grossly disrupt protein folding lead to VHL Type I: low risk for pheo. Other
missense mutations lead to VHL Type II: high risk of pheo. Reduced risk of renal cancer in
those with complete gene deletion.
Treatment/Prognosis: Annual eye exam from early childhood, Starting at age 5: fractionated
metanephrines, BP. Starting age 15 every other year abdominal US, every 2-3 years brain
and spine MRI. Temporal bone MRI if documented hearing loss or tinnutus.
202
ONCOLOGIC DISORDERS
VON HIPPEL-LINDAU SYNDROME
Lancet. 2003 Jun 14;361(9374):2059-67
203
XERODERMA PIGMENTOSUM
Responsible genes: Most common subtypes: XPA, XPC, ERCC2, POLH
Proteins: DNA-repair protein complementing XP-A cells, DNA-repair protein complementing XP-C cells,
ONCOLOGIC DISORDERS
TFIIH basal transcription factor complex helicase subunit, DNA polymerase theta
Cytogenetic loci: 9q22.3, 3p25, 19q13.2-q13.3, 6p21.1-p12
Inheritance: AR
Clinical Features and Diagnostic Criteria: severe sun sensitivity, UV exposure to conjunctiva, cornea,
and lids-> severe keratitis, progressive neurologic deterioration: acquired microcephaly, dec/absent
DTR’s, prog. SNHL, cognitive impairment. > 1000x inc. risk of skin and eye neoplasms
Clinical Tests: Cellular UV hypersensitivity (a post UV exposure cellular survival plot reflecting capacity
for DNA repair.
Molecular Tests: Research only direct DNA testing of XPA (25%), XPC (25%), ERCC2 (15%), POLH
(21%)
Disease Mechanism: Impaired ability to sense, excise, and repair UV-induced DNA damage
Treatment/Prognosis: Regular detailed skin and eye exam, regular audiometry, protection of all body
surfaces from UV light, UV meter to detect unexpected sources of high levels of UV light (eg halogen
lamps).
204
618
ONCOLOGIC DISORDERS
XERODERMA PIGMENTOSUM
http://phobos.ramapo.edu/~pbagga/xp.JPG
205
BECKWITH-WIEDEMANN SYNDROME
OVERGROWTH DISORDERS
Responsible genes: CDKN1C, H19, KCNQ1OT1
Proteins: cyclin-dep kinase inhib 1C, H19 maternally expressed untranslated mRNA, potassium
voltage-gated channel, KQT-like subfamily, member 1
Cytogenetic locus: 11p15.5
Inheritance: AD (15%)
Clinical Features and Diagnostic Criteria: hemihyperplasia, macrosomia, macroglossia,
visceromegaly, embryonal tumors (e.g., Wilms tumor, hepatoblastoma, neuroblastoma,
rhabdomyosarcoma), omphalocele, neonatal hypoglycemia, ear creases/pits, adrenocortical
cytomegaly, and renal abnormalities
Clinical Tests: AFP, abdominal CT
Molecular Tests: Cytogenetically detectable abnormalities of 11p15 (<1%); loss of methylation at
DMR2 (50%); gain of methylation at DMR1 (2% -7%); pat. UPD for 11p15 (10-20%); mutations in the
CDKN1C (40% of familial cases and 5-10% of sporadic cases)
Disease Mechanism: imprinted genes including growth factors and tumor suppressor genes in the
11p15.5 region
Treatment/Prognosis: Screening for embryonal tumors: abdominal US every three months until eight
years. Serum AFP concentration is monitored in the first few years of life for hepatoblastoma.
206
OVERGROWTH DISORDERS
http://imaging.cmpmedica.com/consultantlive/images/photo_clinic
BECKWITH-WIEDEMANN SYNDROME
Facial features:
Anterior linear ear
lobe creases
Posterior helical ear
pits
Macroglossia
Hemihyperplasia
Facial nevus
flammeus
Midface hypoplasia
Infraorbital creases
207
619
OVERGROWTH DISORDERS
SOTOS SYNDROME
Responsible gene: NSD1
Protein: Histone-lysine N-methyltransferase, H3 lysine-36 and H4 lysine-20 specific
Cytogenetic locus: 5q35 Inheritance: AD
Clinical Features and Diagnostic Criteria: classic: macrocephaly, pointed chin, tall stature
and increased body mass, delayed motor skills, delayed cognitive, verbal, and social
development, advanced BA. Less common: phobias, aggression, OCD, ADD, abnormal EEG
and seizure, chronic OM and constipation, congenital heart defects, strabismus,
hyper/hypothyroidism, possible inc risk of tumors (saccrococcygeal teratoma and
neuroblastoma).
Clinical Tests: Bone age. Brain MRI or CT may show inc ventricles
Molecular Tests: MLPA or FISH for 5q35 microdeletion including NSD1: ~15% (70% in
Japanese). NSD1 sequencing: 27-93% (12% in Japanese)
Disease Mechanism: Haploinsufficiency of NSD1. May be related to genes affecting growth.
Treatment/Prognosis: Supportive treatment, most end up of ave adult Ht, IQ ranges from
normal to ID. Cancer screening is not rec. (risk ~1%)
208
OVERGROWTH DISORDERS
SOTOS SYNDROME
www.gfmer.ch
Facial features: malar flushing, sparse frontotemporal hair, high bossed
forehead, downslanting palpebral fissures, a long narrow face, and prominent
narrow jaw; the head is said to resemble an inverted pear
209
CONNECTIVE
PREMATURE AGING
TISSUESYNDROMES
DISORDERS
ATAXIA WITH OCULOMOTER APRAXIA
TYPE 1 and TYPE 2
Responsible genes: APTX, SETX
Proteins: Aprataxin, Probable Helicase Senataxin
Cytogenetic loci: 9p13.3, 9q34
Inheritance: AR
Clinical Features and Diagnostic Criteria: childhood onset of slowly progressive cerebellar
ataxia, followed by oculomotor apraxia and a severe primary motor peripheral axonal motor
neuropathy. Oculomotor apraxia progresses to external ophthalmoplegia.
Clinical Tests: Cerebellar atrophy, axonal neuropathy on EMG and biopsy, low serum
albumin, high cholesterol. Type 2: Inc AFP
Molecular Tests: sequencing APTX (Inc incidence in Portugal and Japan) and SETX.
Mutation detection rate unknown.
Disease Mechanism: There is direct involvement of aprataxin in the DNA single-strand break
repair mechanisms; mutations in the APTX gene destabilize aprataxin and cells from
individuals with AOA1 are characterized by enhanced sensitivity to agents that cause singlestrand breaks in DNA
Treatment/Prognosis: PT, wheelchair by age 15-20 yrs, educational support, high protein
low cholesterol diet
210
620
CONNECTIVE
PREMATURE AGING
TISSUESYNDROMES
DISORDERS
ATAXIA WITH OCULOMOTER APRAXIA
TYPE 1 and TYPE 2
Sagittal T2-weighted
section showing severe
cerebellar atrophy
predominantly in the
vermis
(Le Ber et al, Brain, 2003)
211
CONNECTIVE
PREMATURE AGING
TISSUESYNDROMES
DISORDERS
COCKAYNE SYNDROME
Responsible genes: ERCC6, ERCC8
Proteins: DNA excision repair protein ERCC-6 and ERCC-8
Cytogenetic loci: 10q11, Chromosome 5
Inheritance: AR
Clinical Features and Diagnostic Criteria: CS Type I: normal prenatal growth, severe FTT
in first 2 years, progressive deterioration of vision, hearing, CNS, and peripheral nervous
syndrome. Type II: growth failure at birth, little or no postnatal neurological development,
kyphosis, scoliosis, joint contracture. Type III: normal growth and development or late onset.
Xeroderma Pigmentosum-CS: facial freckling, early skin cancer, ID, spasticity, short stature,
hypogonadism (no demyelination).
Clinical Tests: Brain MRI: leukodystrophy. Eye exam: pigmentary retinopathy, cataracts,
demyelinating peripheral neuropathy. Abnormal DNA repair on skin fibroblasts
Molecular Tests: Gene sequencing ERCC6 (75%), ERCC8 (25%)
Disease Mechanism: Abnormal transcription-coupled nucleotide excision repair (preferential
removal of UV-induced pyrimidine dimers and other transcription blocking lesions)
Treatment/Prognosis: PT, dental exams, skin exams, sunscreen if photosensitive. Death in
1st-2nd decade Type I, by age 7 yrs Type II.
212
CONNECTIVE
PREMATURE AGING
TISSUESYNDROMES
DISORDERS
COCKAYNE SYNDROME
www.ufowijzer.nl
213
621
CONNECTIVE
PREMATURE AGING
TISSUESYNDROMES
DISORDERS
HUTCHINSON-GILFORD
PROGERIA SYNDROME
Responsible gene: LMNA
Protein: Lamin-A/C
Cytogenetic locus: 1q21.2
Inheritance: AD (all de novo, paternal age effect)
Clinical Features and Diagnostic Criteria: short stature, wt<<ht, head large for face, diminished sc fat,
prominent scalp veins, generalized alopecia, delayed and crowded teeth, delayed fontanelle closure,
pear shaped thorax, small chin, thin limbs, tight joints, wide based shuffling gate. Sclerodermatous skin
changes over lower abdomen and thighs
Clinical Tests: Elevated urine hyaluronic acid (unreliable for Dx). ECG, echo, and carotid duplex scans
for stenosis. X-ray for clavicular absorption, acroosteolysis, coxa valga
Molecular Tests: LMNA G608G Exon 11 (100%)
Disease Mechanism: G608G leads to abnl splicing and the mutant form of prelamin A that results is
thought have a dominant negative effect leading to progressive defects in nuclear architecture
Treatment/Prognosis: Optimize nutrition, age appropriate schooling, PT, aspirin. Annual ECG, echo,
carotid duplex, lipid profiles, dental exam and x-ray. Hip x-rays every few yrs to evaluate for avascular
necrosis of the femoral head. Severe atherosclerosis develops even with nl lipid profiles, usually die of
MI or CVA (ave lifespan 13 yrs); clinical trials
214
CONNECTIVE
PREMATURE AGING
TISSUESYNDROMES
DISORDERS
HUTCHINSON-GILFORD
PROGERIA SYNDROME
seattlepi.nwsource.com
215
PULMONARY SYSTEM
ALPHA-1-ANTITRYPSIN DEFICIENCY
Responsible gene: SERPINA1
Protein: AAT
Cytogenetic locus: 14q32.1
Inheritance: AR
Clinical Features and Diagnostic Criteria: Adult COPD, childhood and
adult liver disease (obstructive jaundice and raised transaminases in kids,
cirrhosis and fibrosis in adults). Age of onset 40-50y if a smoker, 60’s if not.
Clinical Tests: Low plasma AAT (also low in other resp d/o inc CF),
Demonstration of deficient variant of the AAT protein by protease inhibitor
typing
Molecular Tests: Targeted mutation testing of SERPINA (95% E42K)
Disease Mechanism: Loss of sufficient protease inhibition by AAT
Treatment/Prognosis: Liver transplant is a cure (donor liver produces AAT).
Intravenous augmentation therapy
216
622
PULMONARY SYSTEM
ALPHA-1-ANTITRYPSIN DEFICIENCY
(commons.wikimedia.org/wiki/File:Conditions_associated_with_Alpha-1_Antitrypsin_Deficiency.png)
217
PULMONARY SYSTEM
CFTR-RELATED DISORDERS
Responsible gene: CFTR
Protein: cystic fibrosis transmembrane conductance regulator
Cytogenetic locus: 7q31.2
Inheritance: AR
Clinical Features and Diagnostic Criteria: Cystic fibrosis (CF): chronic airway infection, chronic
sinusitis, meconium ileus, malabsorption due to pancreatic insufficiency, male infertility due to
azoospermia. Progression to end stage lung disease. Congenital bilateral absence of the vas deferens
(CBAVD) occurs in men without pulm. or GI Sx of CF.
Clinical Tests: sweat test, decreased semen volume with low pH, high [citric acid], high [acid
phosphatase], low [fructose]
Molecular Tests: Common mutation testing or full gene sequencing. Intron 8 5T variant: variably
penetrant, test for if R117H mutation. 5T with 12 or 13 TG tract (just 5’ of 5T) has the strongest
adverse effect on proper intron 8 splicing. deltaF508: 30-80% of mutant alleles depending upon ethnic
group.
Disease Mechanism: CFTR forms a regulated cell membrane chloride channel. 4 mutation classes: I.
reduced or absent synthesis, II. block in protein processing, III. block in regulation of CFTR chloride
channel, IV. altered conductance of CFTR chloride channel.
Treatment/Prognosis: antibiotics, bronchodilators, steroids, mucolytics, chest PT, lung transplant,
pancreatic enzymes, fat soluble vitamins, microscopic sperm aspiration; some examples of mutationspecific therapies.
218
PULMONARY SYSTEM
CFTR-RELATED DISORDERS
Cysticfibrosis.com
219
623
RENAL DISORDERS
ALPORT SYNDROME AND THIN
BM NEPHROPATHY
Responsible genes: XL: COL4A5, AR: COL4A3 and COL4A4, AD: COL4A3 and COL4A4
Proteins: Collagen alpha-3(IV) chain, Collagen alpha-4(IV) chain, Collagen alpha-5(IV) chain
Cytogenetic loci: 2q36-q37 (COL4A3 and COL4A4), Xq22.3 (COL4A5)
Inheritance: 80% X linked, 15% AR, 5% AD
Clinical Features and Diagnostic Criteria: Spectrum from progressive renal disease with cochlear and
ocular abnormalities (Alport) to isolated hematuria with a benign course (thin BM nephropathy).
Clinical Tests: Microhematuria, eventually proteinuria. Anterior lenticonus virtually pathognomonic, EM
on renal biopsy
Molecular Tests: Sequencing and deletion testing COL4A3, COL4A4, COL4A5 (80-100%)
Disease Mechanism: Type IV Collagen is found ubiquitously and is the major collagen component of
BMs. Alport due to abnl secretion of collagen alpha 3,4,and 5 (IV) chains
Treatment/Prognosis: ESRD: 60% by 30 yrs and 90% by 40 yrs and Deafness: 80-90% SN deafness
by age 40 in males, later in life in females with XL Alport. Renal progression and deafness is slower in
AD Alport and ocular lesions uncommon. Juvenile onset HL in AR Alport.
220
RENAL DISORDERS
ALPORT SYNDROME AND THIN
BM NEPHROPATHY
www.ncbi.nlm.nih.gov/books/NBK22265/
221
RENAL DISORDERS
POLYCYSTIC KIDNEY DISEASE
Responsible genes: PKD1, PKD2 and PKHD1
Proteins: Polycystin-1, Polycystin-2, Fibrocystin
Cytogenetic loci: 16p13.1, 4q21, and 6p21.1-p12
Inheritance: AD (PKD1, PKD2) AR (PKHD1)
Clinical Features and Diagnostic Criteria: AD PKD Enlargement of both kidneys, renal
cysts, hematuria, polyuria, flank pain, renal stones, urinary infection. Cysts in liver, pancreas,
and intestine; heart valve defects, intracranial aneurysm. AR PKD Fetal or neonatal death,
impaired lung formation, pulmonary hypoplasia due to oligohydramnios, renal failure, hepatic
fibrosis. Most present prenatally or early infancy
Clinical Tests: Abdominal US, prenatal US, MRI
Molecular Tests: PKD1 and PKD2 sequence analysis (85%). Large deletion including PKD1
and TSC2: manifestations off PKD and tuberous sclerosis
Disease Mechanism: Unclear, decreased amount of functional protein?
Treatment/Prognosis: PKD2 mutations show later onset and slower rate of progression.
ESRD age 60 yrs
222
624
RENAL DISORDERS
POLYCYSTIC KIDNEY DISEASE
Liver and
kidney
cysts
www.learningradiology.com
223
SKELETAL DYSPLASIA
ACHONDROPLASIA
Responsible gene: FGFR3
Protein: Fibroblast growth factor recepter 3
Cytogenetic locus: 4p16.3
Inheritance: AD; 80% de novo
Clinical Features and Diagnostic Criteria: short stature, rhizomelic
shortening, trident hand, frontal bossing, midface hypoplasia, macrocephaly,
OSA, spinal cord compression
Clinical Tests: Narrowing of interpediculate distance, caudal spine; notch-like
sacroiliac groove, circumflex or chevron seat on the metaphysis
Molecular Tests: 98% FGFR3 G1138A; ~1% FGFR3 G1138C
Disease Mechanism: Constitutive activation of FGF R (GOF mutations)activation of negative growth control
Treatment/Prognosis: achondroplasia growth curves, surgery or CPAP for
OSA, role of GH unclear, leg lengthening, suboccipital decompression, spinal
fusion, LPA support group
224
ACHONDROPLASIA
SKELETAL DYSPLASIA
Rhizomelic shortening, frontal bossing, midface
hypoplasia, macrocephaly
myweb.lsbu.ac.uk
medgen.genetics.utah.edu
Genu varum
225
625
SKELETAL DYSPLASIA
CLEIDOCRANIAL DYSPLASIA
Responsible gene: RUNX2
Protein: Runt-related transcription factor 2
Cytogenetic locus: 6p21
Inheritance: AD (high proportion de novo)
Clinical Features and Diagnostic Criteria: delayed closure of the cranial sutures, hypoplastic
or aplastic clavicles, multiple dental abnormalities. Abnormally large wide open anterior
fontanel, midface hypoplasia, brachydactyly, recurrent OM, hearing loss, normal intellect.
Clinical Tests: X-ray: clavicular hypoplasia, open sutures, wormian bones, poor or absent
sinus pneumatization, hypoplastic scapulae, wide symphysis pubis and sacroiliac joints, large
femoral neck and epiphyses, pseudoepiphyses of the metacarpals and metatarsals, deformed
and short middle phalanges, osteopenia.
Molecular Tests: RUNX2 sequencing and array for microdeletions (60-70%).
Disease Mechanism: Independently mediates DNA binding and protein heterodimerization;
mutations abolish DNA binding
Treatment/Prognosis: Hearing test, dental referral, ear tubes, helmets if large skull defects
226
SKELETAL DYSPLASIA
CLEIDOCRANIAL DYSPLASIA
Hypoplastic/
absent clavicles
allow opposition
of the shoulders
anteriorly
Large skull
defects
(www.lab3d.odont.ku.dk)
(www.pediatriconcall.com)
227
SKELETAL DYSPLASIA
DIASTROPHIC DYSPLASIA
Responsible gene: SLC26A2
Protein: Sulfate transporter
Cytogenetic locus: 5q32-q33.1
Inheritance: AR
Clinical Features and Diagnostic Criteria: limb shortening, normal-sized skull, hitchhiker thumbs,
small chest, large joint contracture, cleft palate, cystic ear swelling, ulnar deviation of fingers, clubfoot,
low tone, normal IQ
Clinical Tests: x-ray: cervical kyphosis, incomplete thoracic vertebrae ossification, coronal clefting of
lower thoracic and lumbar vertebrae, narrowed interpedicular distance L1 to L5, distal humerous can be
bifid or v shaped, rounded distal femur, advanced bone age. Cartilage histopathology: paucity of
sulfated proteoglycans in cartilage matrix. Abnormal incorporation of sulfate into macromolecules in
cultured chondrocytes
Molecular Tests: SLC26A2 targeted mutation analysis (65% one of 5 mutations), SLC26A2
sequencing(>90%)
Disease Mechanism: Undersulfation of proteoglycans affects the composition of the extracellular matrix
and leads to impaired proteoglycan deposition which is necessary for proper enchondral bone formation
Treatment/Prognosis: Maintain joint positioning and mobility as much as possible, clubfoot deformities
tend to recur after surgical correction, scoliosis surgery best if postponed until after puberty. Joint
contractures and spine deformity worsen with age. Total arthroplasy may diminish joint pain.
228
626
SKELETAL DYSPLASIA
DIASTROPHIC DYSPLASIA
Limb shortening
Normal-sized skull Hitchhiker
thumbs
Small chest
Large joint contracture
Ulnar deviation of fingers
Clubfoot
www.thefetus.net
229
FGFR-RELATED CRANIOSYNOSTOSIS
SKELETAL DYSPLASIA
(Pfeiffer, Apert, Crouzon, Beare-Stevenson, FGFR2-related Isolated Coronal
Synostosis, Jackson-Weiss, Crouzon with Acanthosis Nigricans, and Muenke)
Responsible genes: FGFR1, FGFR2, FGFR3
Proteins: Basic fibroblast growth factor receptor 1, 2, and 3
Cytogenetic loci: 8p11.2-p11.1, 10q26, 4p16.3
Inheritance: AD
Clinical Features and Diagnostic Criteria: All but Muenke and FGFR2-related Isolated Coronal
craniosynostosis are associated with bicoronal craniosynostosis or cloverleaf skull, distinctive facial
features, and variable hand and foot anomalies (broad and/or syndactylous). Developmental delay/ID,
hearing loss, and visual impairment common.
Clinical Tests: Brain CT or MRI for hydrocephalus, spinal x-rays for vertebral anomalies
Molecular Tests: FGFR1 sequencing (5% Pfeiffer 1); FGFR2 sequencing (100% Crouzon, JacksonWeiss, Apert, Pfeiffer 2 and 3, and FGFR2-related isolated coronal synostosis); FGFR3 sequencing
(100% Crouzon with Acanthosis Nigricans); FGFR3 targeted mutation analysis (100% Muenke)
Disease Mechanism: Mutations cause increased R affinity thought to promote excessive receptor
down-regulation.
Treatment/Prognosis: Coordinated neurosurgical, ORL, and dental care, follow for scoliosis, limb
anomalies rarely benefit from surgery
230
FGFR-RELATED CRANIOSYNOSTOSIS
SKELETAL DYSPLASIA
(Pfeiffer, Apert, Crouzon, Beare-Stevenson, FGFR2-related Isolated Coronal
Synostosis, Jackson-Weiss, Crouzon with Acanthosis Nigricans, and Muenke)
http://www.ida.org.in/Infor
mation/newimages/cranios
ynostosis.1.jpg
http://webspace.webring.com/people/jc/crouzonsyndrom
e/family2007c.jpg
http://static.guim.co.uk/sys-images
231
627
SKELETAL DYSPLASIA
HEREDITARY MULTIPLE
OSTEOCHONDROMAS SYNDROME
Responsible genes: EXT1, EXT2
Proteins: Exostosin-1, Exostosin-2
Cytogenetic loci: 8q24.11, 11p11.2
Inheritance: AD
Clinical Features and Diagnostic Criteria: Exostoses (benign cartilage-capped bony
growths) arising from the growth plate of the long bones or from the surface of flat bones
(scapula). Limb length inequity and bowed long bones can develop. Short metacarpals. Can
have mass effect compression of nerves and blood vessels.
Clinical Tests: x-ray may detect mildly affected individuals
Molecular Tests: EXT1 and EXT2 sequencing: >70% detection rate, del/dup studies: 20%
Disease Mechanism: EXT1/2 encode glycosyltransferases, mutations lead to actin
accumulation and cytoskeltal abnormalities
Treatment/Prognosis: Growth ceases after skeletal maturation. 0.5-2% of cases degenerate
to chondrosarcoma. Treatment is surgical resection.
232
SKELETAL DYSPLASIA
HEREDITARY MULTIPLE
OSTEOCHONDROMAS SYNDROME
Exostoses
www.learningradiology.com
233
SKELETAL DYSPLASIA
HYPOCHONDROPLASIA
Responsible gene: FGFR3
Protein: Fibroblast growth factor receptor 3
Cytogenetic locus: 4p16.3
Inheritance: AD
Clinical Features and Diagnostic Criteria: Short stature, stocky build, rhizo- or mesomelia,
limited elbow extension, brachydactyly, mild joint laxity, macrocephaly, scoliosis, genu varum,
lumbar lordosis, mild-mod ID, LD, adult onset osteoarthritis
Clinical Tests: x-ray: elongated distal fibula, short lumbar pedicles, short distal ulna, chevron
deformity of distal femur metaphysis, flattened acetabular roof
Molecular Tests: Targeted mutation analysis: N540K (C1620A) (49%), N540K (C1620G)
(21%). Exon 9, 10, 13, or 15 sequencing (80%)
Disease Mechanism: unknown but mouse models suggest FGFR3 is a negative regulator of
bone growth
Treatment/Prognosis: Monitor for S/Sx spinal cord compression (MRI or CT foramen
magnum), sleep study id history c/w OSA, ortho eval if severe genu varum impairs walking.
234
628
SKELETAL DYSPLASIA
HYPOCHONDROPLASIA
Short stature
Stocky build
Rhizo- or mesomelia
Limited elbow
extension
Brachydactyly
Macrocephaly
Scoliosis
Genu varum
Lumbar lordosis
(www.nature.com/.../n12/fig_ta
b/5201700f4.html)
235
SKELETAL DYSPLASIA
OSTEOGENESIS IMPERFECTA
Responsible genes: COL1A1 and COL1A2
Proteins: Collagen alpha 1(I) chain, Collagen alpha 2(I) chain
Cytogenetic loci: 17q21.33, 7q21.3 Inheritance: AD and rare AR
Clinical Features and Diagnostic Criteria: Fractures with little or no trauma, relative short
stature, blue sclera, dentinogenesis imperfecta, post-pubertal HL, ligamentous laxity, easy
bruising. OI Type II: perinatal lethal, palpable callus formation on ribs, hips in “frog-leg”
position, short bowed extremities. OI Type III: severe, skull descends on cervical spinebrainstem compression, obstructive hydrocephalus, syringomyelia
Clinical Tests: x-ray: fractures of varying ages, spinal compression fracture, wormian bones,
protrusio acetabuli, osteopenia. Cultured fibroblasts (98% Type II, 87% all others)
Molecular Tests: COL1A1 and COL2A1 sequencing: ~100% Type I, 98% Type II, 60-70%
Type III, 0-80% Type IV
Disease Mechanism: Type I: premature stop codon->unstable mRNA->dec amount type I
collagen. Types II, III, IV: mutations alter collagen structure
Treatment/Prognosis: Bisphosphonate to decrease bone resorption, GH to increase linear
growth and bone formation
236
SKELETAL DYSPLASIA
OSTEOGENESIS IMPERFECTA
Bowing of
the femur
folding.stanford.edu
237
629
SKELETAL DYSPLASIA
SAETHRE-CHOTZEN SYNDROME
Responsible gene: TWIST1
Protein: Twist-related protein 1
Cytogenetic locus: 7p21
Inheritance: AD
Clinical Features and Diagnostic Criteria: coronal synostosis, facial asymmetry, ptosis, 2/3
hand syndactyly, mild-moderate developmental delay in a minority, short stature, parietal
foramina, vertebral fusions, radioulnar synostosis, cleft palate, maxillary hypoplasia,
congenital heart defect
Clinical Tests: echo, x-ray for vertebral abnormalities, audiologic testing, and karyotype:
translocations, inversions, or ring chromsome 7 have been reported
Molecular Tests: TWIST1 sequencing: >50%, del/dup testing: complete deletion of the
TWIST1 gene 11-28%
Disease Mechanism: haploinsufficiency by gene deletion, rapid degradation of abnormal
protein, or altered subcellular localization of abnormal protein
Treatment/Prognosis: endocrine eval if plateau in growth, craniofacial team management,
surgical repair of cleft palate and craniosynostosis, eye exams to monitor for evidence of
increase ICP
238
SKELETAL DYSPLASIA
SAETHRE-CHOTZEN SYNDROME
Facial Features:
Coronal synostosis
Facial asymmetry
Ptosis
Maxillary
hypoplasia
(The Cleft Palate-Craniofacial Journal: May 2010, Vol. 47, No. 3, pp. 318-321)
239
630
Writing Exam Questions
WRITING EXAM
QUESTIONS
Debra Schindler, PhD
Senior Education Specialist, Office of Curricular Affairs
Saint Louis University School of Medicine
633
634
Designing Single Best Answer Multiple Choice Questions
This is a short summary of the main points to consider in writing single best answer
multiple choice examination questions. It is intended as a guide and checklist to facilitate
what is a very difficult task: crafting a good examination that assesses your students’
knowledge and problem-solving abilities, and not their test-taking skills. Most of the
material here comes straight from Case and Swanson’s Constructing Written Test
Questions For the Basic and Clinical Sciences.1
There are three basic questions that you should be able to answer by the time you finish
reading these materials.
x How do I construct the question?
x How do I construct the answer choices?
x How do I construct an exam with a balance of difficulty levels?
Only with practice, however, will you be able to apply the basic principles of question
writing described here. A well-crafted exam question is usually the product of several
iterations of less “ideal” exam questions, and testing the item with students.
How to Use This Guide
First Steps. The section titled First Steps is the place to start. Use the three steps on this
page to review and revise each of your current exam questions. If you find that your
questions and your exam meet the criteria listed under First Steps, then your exam is
probably in good shape. If you don’t read anything else, read this section.
The Structure of Questions and Answers / Question Difficulty. These two sections
provide a more detailed explanation of the “how” and “why” of the First Steps.
Second Steps- Polishing Vignettes. When you are ready to add new questions, modify
existing questions to include more patient-based problems, or improve your existing
vignettes, refer to this section.
Case and Swanson’s Item Review. This is a more detailed checklist to use in reviewing
your test items. If your test item meets the “bare bones” criteria of the First Steps, then
use this checklist to check for any additional areas of difficulty.
Content Integration. The last page of this document contains an example of how one
patient vignette can be used in four different disciplines, providing a means of integrating
knowledge across course and discipline boundaries.
1
Susan M. Case and David B. Swanson, Constructing Written Test Questions For The Basic and Clinical
Sciences (Philadelphia: National Board of Medical Examiners, 1998).
Prepared by the Office of Curricular Affairs, St. Louis University School of Medicine,
September 28, 2004. Debra L. Schindler, Ph.D., Evaluation Coordinator
635
First Steps
Writing good test questions is hard and takes time. Here are some first steps to take in
reviewing and revising your existing test questions.
1. Cover the options. Can you still answer the question? If not, rewrite the question to
make it complete. Why? Ideally, you would like your students to answer a question
without having any hints (i.e. answer choices)- ideally, they should be able to
generate the answer from the knowledge they have inside their heads. So- reinforce
the cognitive structure of their medical knowledge by asking a question that requires
accessing the knowledge base.
HINT: Don’t use the phrase “Which of the following…” if possible. Use the full range of
question words available in English: who, what, when, where, why, how, which. Using
these question words almost always forces you to write a complete question.
2. Order the options. Make sure that you can order your options (including the correct
answer) from
o Best to Worst
o First to Last
o Most Likely to Least Likely
o Etc.
Why? When your students generate answers in their heads, you would like those possible
answers to be logically structured. We must help students build the scaffolding required
to organize all of the knowledge and skills they need to acquire, and then teach them,
through question and answer, how to retrieve and apply that knowledge in a structured
way. A clearly focused question, with options that can be ordered logically, reinforces
both the structure of knowledge that students are building, and the thought process
needed to retrieve that knowledge.
If your options cannot be ordered in any logical manner, they probably aren’t
homogenous, and your question probably isn’t clearly focused.
3. Provide a balanced level of difficulty and challenge your students. They work hard
and want to demonstrate their knowledge and skills. On every exam, try to provide a
balance between questions that ask students to recall facts, questions that ask students
to apply those facts, and questions that ask students to generate new solutions.
Using questions that assess different levels of cognitive ability can produce important
feedback for both students and faculty. Students can identify the types of questions (and
thought processes) that they have the most difficulty with, and focus their efforts in those
areas. Faculty can identify areas of teaching and/or content that may need strengthening
or may actually need less time in class.
2
Prepared by the Office of Curricular Affairs
636
The Structure of Questions and Answers2
Single best answer style questions should have the following structure:
Component
A 32-year old man has a 4-day history of progressive
weakness in his extremities. He has been healthy except for an
upper respiratory tract infection 10 days ago. His temperature
is 37.8 C (100 F), blood pressure is 130/80 mm Hg, pulse is
94/min, and respirations are 42/min and shallow. He has
symmetric weakness of both sides of the face and the
proximal and distal muscles of the extremities. Sensation is
intact. No deep tendon reflexes can be elicited; the plantar
responses are flexor.
Stem (e.g., a clinical or
laboratory case
presentation). This need
not be a long
presentation, and may in
fact, elicit simple recall.
What is the most likely diagnosis?
Lead-in question
A.
B.
C.
D.
E.
One correct answer +
four distractors
Acute disseminated encephalomyelitis
Guillain-Barré syndrome
Myasthenia gravis
Poliomyelitis
Polymyositis
The Question/Stem
The stem and lead in question should provide enough information to allow the student to
answer the question without looking at the options. More information on creating a
vignette is provided on pages 10-11. Case and Swanson provide many examples of leadin questions for basic sciences on pages 39-40, and examples for clinical science items
Health and Health Maintenance, Mechanisms of Disease, Diagnosis, and Management)
on pages 61-65.
2
Material on writing questions and answers is borrowed and adapted from Susan M. Case and David B.
Swanson, Constructing Written Test Questions For The Basic and Clinical Sciences (Philadelphia: National
Board of Medical Examiners, 1998).
Prepared by the Office of Curricular Affairs
637
3
The Answer/Options
Note that the incorrect options in the example above are not totally wrong. The options
can be diagrammed as follows:
D
Least Correct
C
A
E
B
Most Correct
Even though the incorrect answers are not completely wrong, they are less correct than
the "keyed answer."
The examinee is instructed to select the "most likely diagnosis"; experts would agree that
the most likely diagnosis is B; they would also agree that the other diagnoses are
somewhat likely, but less likely than B.
Using a continuum provides a more realistic context for deriving the correct answer.
Physicians must choose, for example, the best drug from among several possible choices:
under different circumstances each of the choices may be the ‘best.” The circumstances
of the case that you provide to students, however, should dictate the “best” choice, and
thus reflect the correct decision-making process.
As long as the options can be laid out on a single continuum, in this case from "Most
Likely Diagnosis" to "Least Likely Diagnosis," options in single best answer questions do
not have to be totally wrong. Other option sets can include next steps, tests, treatments,
prognoses, etc., as long as all of the options in a set are homogeneous.
The next example illustrates the most common type of flaw in single best answer
questions: non-homogeneous option sets.
Component
Which of the following is true about pseudogout?
Stem
A.
B.
C.
D.
E.
One correct answer +
four distractors
It occurs frequently in women
It is seldom associated with acute pain in a joint.
It may be associated with a finding of chondrocalcinosis
It is clearly hereditary in most cases
It responds well to treatment with allopurinol
After reading the stem, the examinee has only the vaguest idea what the question is
about. In an attempt to determine the "best" answer, the examinee has to decide whether
"it occurs frequently in women" is more or less true than "it is seldom associated with
acute pain in a joint." This is a comparison of apples and oranges. In order to rank-order
the relative correctness of options, the options must differ on a single dimension or else
all options must be absolutely 100% true or false.
4
Prepared by the Office of Curricular Affairs
638
The options for this test item can be diagrammed as follows:
Gender (A)
Rx (E)
Inheritance (D)
False
(B) Associations (C)
True
The options are heterogenous and deal with miscellaneous facts; they cannot be rankordered from least to most true along a single dimension. Although this question appears
to assess knowledge of several different points, its inherent flaws preclude this. The
question by itself is not clear; the item cannot be answered without looking at the options.
Prepared by the Office of Curricular Affairs
639
5
Question Difficulty
The three questions below are examples of the evolution of questions on the USMLE
Step 1 exam, beginning with question type 1 from the 1980s, question type 2 from the
1990s, and ending with question type 3, a format that has been added to the currently
administered Step 1 exam.3 Each question type represents a different level of difficulty
and requires a different thought process to arrive at the correct answer. As faculty in
Phase 1 and 2 courses build banks of test questions, questions at all three levels should be
constructed and questions from all three levels should be included on every exam. Exams
may contain very few type 3 questions at the highest level of cognitive ability, but such
questions can help identify students who are truly at the top of their class (as measured by
the exam only). Distinguishing between types of questions can also help identify students
who are comfortable with repeating directly from the course material, but who have
trouble with integrating information and forming independent conclusions. Just as the
USMLE examination contains all three types of questions, so should SLUSOM course
examinations.
Type 1:
Comprehension,
Knowledge
Parkinson’s disease is caused by loss of neurons in which
of the following areas of the brain?
A) Caudate nucleus
B) Cerebral cortex
C) Mamillary bodies
D) Substantia nigra
E) Subthalamic nucleus
Type 2:
Application
A 72-year-old man presents complaining of shaking in his
right hand and trouble starting movements. On physical
examination, the patient has a resting tremor of the right
hand that decreases with active movement. The man’s face
is expressionless, and his voice is very soft. Ratchet-like
resistance to passive movement is noted in both arms. He
also has a slightly stooped posture, and a slow, shuffling
gait. Which of the following areas of the brain is most
likely affected in this patient?
A) Caudate nucleus
B) Cerebral cortex
C) Mamillary bodies
D) Substantia nigra
E) Subthalamic nucleus
Type 3:
Analysis,
Synthesis,
Evaluation
A 72-year-old man presents complaining of shaking in his
right hand and trouble starting movements. On physical
examination, the patient has a resting tremor of the right
hand that decreases with active movement. The man’s face
is expressionless, and his voice is very soft. Ratchet-like
resistance to passive movement is noted in both arms. He
also has a slightly stooped posture, and a slow, shuffling
gait. Which of the following areas of the labeled brain
below is most likely affected in this patient?
A) A
B) B
C) C
D) D
E) E
3
The examples in this table have been reproduced from a Kaplan Medical Power Point presentation made
November 21, 2002 at Saint Louis University School of Medicine, with permission from Kaplan, Inc. It
may not be reproduced or altered without their permission.
6
Prepared by the Office of Curricular Affairs
640
The table below provides several additional ways to think about question item
construction and difficulty.
Test Question
Level of Difficulty
1
Easy to Moderate
2
Moderate to
Difficult
3
Very Difficult
Analysis
Synthesis
Evaluation
Bloom's Taxomony
Comprehension
Knowledge
Walvoord and
JohnsonAnderson's criteria
for assigning
difficulty4
The information in
the question is
directly from the
course presentation,
although it may
have changes in
wording or
phraseology.
Course materials
provide all of the
background
information needed
to answer the
question. There is a
direct, visible
connection between
the course material
and the question.
Course materials
provide all of the
background
information needed
to answer the
question, although
there is no clear,
visible connection
between the course
material and the
question.
Type 1:1980s style
questions
Type 2: 1990s style
questions
Type 3: Current
style of questions
USMLE Step 1
Question types (as
illustrated on page
7)
Application
4
Walvoord, B.E, and Johnson-Anderson, V. (1998) Effective Grading. A Tool for Learning and
Assessment. San Francisco: Jossey-Bass.
Prepared by the Office of Curricular Affairs
641
7
Second Steps- Polishing Vignettes
As you work on your existing questions, you may want to write new questions, and refine
your old questions to include more patient-centered problems, such as patient, clinical,
and laboratory vignettes. Case and Swanson recommend using templates to help generate
questions. Their explanations are summarized below, and references are provided to
guide to the specific sections of their book which contain more information.
Basic Science Questions
Template: 5 A (patient description) is unable to (functional disability). Which (structure)
is most likely to have been injured?
Example:
A 65-year-old man has difficulty rising from a seated position and straightening his trunk,
but he has no difficulty flexing his leg. Which muscle is most likely to have been injured?
A.
B.
C.
D.
E.
Gluteus maximus
Gluteus minimus
Hamstrings
Lliopsoas
Obturator internus
Patient vignettes for basic science questions may include some or all of the following
information: age, gender, site of care, presenting complaint, duration, patient history
(with or without family history), physical findings, +/- results of diagnostic studies, +/initial treatment, subsequent findings, etc.
Clinical Science Questions
Clinical vignettes should begin with the presenting problem, history, physical findings,
results of diagnostic studies, initial treatment, subsequent findings, etc. The vignette may
include all or only part of this information, but the information should be presented in this
order. There should be one clearly formulated problem, which the student should be able
to answer without looking at the options.
Example:
A 52-year-old man has had increasing dyspnea and cough productive of purulent sputum
for 2 days. He has smoked one pack of cigarettes daily for 30 years. His temperature is
37.2 C (99 F). Breath sounds are distant with a few rhonchi and wheezes. His leukocyte
count is 9000/mm3 with a normal differential. Gram’s stain of sputum shows numerous
neutrophils and gram-negative diplococci. X-ray films of the chest show hyperinflation.
What is the most likely diagnosis?
5
Page 39 in the Case and Swanson book list many other templates appropriate for basic science questions.
8
Prepared by the Office of Curricular Affairs
642
Tips:
x Don’t use “real patients”—they are too complicated, their findings may contain
“red herrings,” and they sometimes lie. Use incidental findings when appropriatestudents should know how to filter information for what is most relevant in any
given situation. A paper and pencil test can’t capture the visual and verbal cues
that a physician gets from a patient, so avoid trying to write those cues into the
vignette. For example, rather than writing “The patient claims to smoke one pack
of cigarettes per day,” write “the patient smokes one pack of cigarettes per day.”
Tailor the patient and his or her vignette to include accurate information, focused
on the problem-solving or decision-making behavior that you want students to
demonstrate.
x Provide reference materials (e.g,. a table of normal laboratory values), if in real
life, the physician would use such reference materials to obtain information or
help make a decision.
Prepared by the Office of Curricular Affairs
643
9
Case and Swanson’s Item Review
Susan Case and David Swanson recommend the following guidelines for writing these
types of test questions in the context of medical school assessment. 6 Subject each
question to the five “tests” implied by rules below. If a question passes all five, it is
probably well phrased and focused on an appropriate topic.
The Basic Rules for One-Best Answer Items
ƒEach item should focus on an important concept, typically a common or potentially
catastrophic problem. Don’t waste testing time with questions assessing knowledge of
trivial facts. Focus on problems that would be encountered in real life. Avoid trivial,
“tricky,” or overly complex questions.
ƒEach item should assess application of knowledge, not recall of an isolated fact. The
item stems may be relatively long; the options should be short. Vignettes provide a good
basis for a question. For medical students, patient vignettes are useful, even for beginning
students with little exposure to clinical problems. These vignettes should begin with the
presenting problem of a patient, followed by the history (including duration of signs and
symptoms), physical findings, results of diagnostic studies, initial treatment, subsequent
findings, etc. Vignettes may include only a subset of this information, but the information
should be provided in this specified order. For some students in some topic areas,
vignettes may be very brief; in other areas, longer, more complicated vignettes are more
appropriate.
ƒThe stem of the item must pose a clear question, and it should be possible to arrive at an
answer with the options covered. To determine if the question is focused, cover up the
options and see if the question is clear and if the examinees can pose an answer based
only on the stem. Rewrite the stem and/or options if they could not.
ƒAll distractors (i.e., incorrect options) should be homogeneous. They should fall into the
same category as the correct answer (e.g., all diagnoses, all tests, all treatments, all
prognoses, all “next steps in solving the problem”). Rewrite any dissimilar distractors.
Avoid using “double options” (e.g., do W and Y; do Y because of Z) unless the correct
answer and all distractors are double options. Rewrite double options to focus on a single
point. All distractors should be plausible, grammatically consistent, logically compatible,
and of the same (relative) length as the correct answer. Order the options in logical order
(e.g. numeric), or in alphabetical order.
ƒAvoid technical item flaws that provide special benefit to testwise examinees or that
pose irrelevant difficulty. Do NOT write any questions of the form “Which of the
following statements are correct?” or “Each of the following statements is correct
EXCEPT.” These questions are unfocused and have heterogeneous options.
6
Susan M. Case and David B. Swanson, Constructing Written Test Questions For The Basic and Clinical
Sciences (Philadelphia: National Board of Medical Examiners, 1998) 33.
10
Prepared by the Office of Curricular Affairs
644
Content Integration7
To lighten everyone’s burden in writing exams, consider sharing good vignettes across
courses, and maybe working in some curricular integration at the same time.
This example illustrates how a single vignette can be used in combination with questions
from multiple disciplines, allowing integration of content from across the curriculum in a
single examination. The questions posed after the vignette are application type questions
(using Bloom's taxonomy), requiring two steps in the thought process needed to arrive at
an answer: students must first draw a conclusion, and then apply knowledge of an
associated fact.
1. A 72-year-old man presents complaining of shaking in his right hand and trouble
starting movements. On physical examination, the patient has a resting tremor of the
right hand that decreases with active movement. The man’s face is expressionless,
and his voice is very soft. Ratchet-like resistance to passive movement is noted in
both arms. He also has a slightly stooped posture and a slow, shuffling gait.
Pharmacology
Which of the following treatments for this
disease acts by inhibiting monoamine
oxidase B?
Pathology
Which of the following pathologic findings
is characteristic of this disease?
(A) Amantadine
(B) Benztropine
(C) Bromocriptine
(D) Levodopa
(E) Selegiline
(A) Amyloid plaques
(B) Howell-Jolly bodies
(C) Kaiser-Fleischer rings
(D) Lewy bodies
Behavioral Science
The neurotransmitter that is deficient in
this disease is thought to be involved in the
pathogenesis of which of the following
psychiatric disorders?
Biochemistry
The neurotransmitter involved in the
pathogenesis of this disease is derived from
which of the following amino acids?
(A) Bipolar disorder
(B) Borderline personality disorder
(C) Major depressive disorder
(D) Obsessive-compulsive disorder
(E) Schizophrenia
(A) Arginine
(B) Glycine
(C) Histidine
(D) Tryptophan
(E) Tyrosine
7
The example in this table has been reproduced from a Kaplan Medical Power Point presentation made
November 21, 2002 at Saint Louis University School of Medicine, with permission from Kaplan, Inc. It
may not be reproduced or altered without their permission.
Prepared by the Office of Curricular Affairs
645
11
646
Sample Exam Questions &
Answers
SAMPLE EXAM
QUESTIONS & ANSWERS
649
650
2017 Genetics and Genomics Review Course
Sample Questions & Answers
This document contains a consolidation of sample questions with the answers following
each section. All questions have been organized by category for ease of review.
Section
Title
A
Genetic Transmission
B
Biochemical Genetics
C
Clinical Genetics
D
Neurogenetics
E
Molecular Genetics
F
Genetic Counseling
G
Cancer Genetics
H
Cytogenetics
I
Prenatal/Reproductive Genetics
J
Mendelian Genetics
K
Developmental Genetics
L.
Genomic Medicine
651
A. Genetic Transmission (Quantitative Genetics) Questions A1-A35
A1.
Assume that 500,000 serial newborns were examined for new mutation cases of a dominant
disorder with 100% penetrance. Ten affected infants were found with unaffected parents. Which
of the following represents the calculated mutation rate?
A.
B.
C.
D.
E.
A2.
A couple are both carriers for an autosomal recessive disorder. They have four children. What is
the probability that at least one child is affected?
A.
B.
C.
D.
E.
A3.
40/256
81/256
160/256
175/256
216/256
What is the coefficient of inbreeding (F) for a mating of half-siblings?
A.
B.
C.
D.
E.
A4.
1 x 10-6
2 x 10-6
1 x 10-5
2 x 10-5
4 x 10-5
1/4
1/8
1/16
1/32
1/64
A couple requests genetic testing for their newborn child (arrow) who is at risk of a sex-linked
recessive disorder that affects the child’s brother and uncle. A linkage study is performed using a
marker closely linked to the disease locus. There are two alleles at the marker site: A and B, and
the distance between the marker locus and the disease locus is estimated to be 5 cM. What is the
risk of this child developing signs of the disorder, assuming complete penetrance?
A.
B.
C.
D.
E.
0.0475
0.0500
0.0950
0.0975
0.1000
652
A5.
I-1 was affected with Hemophilia (Factor VIII deficiency). What is the probability that the
pregnancy (IV-4) will result in a boy affected with Hemophilia?
A.
B.
C.
D.
E.
1/4
1/8
1/9
1/18
1/36
I
II
1
2
1
2
III
2
1
IV
1
2
3
P
4
A6.
A couple presents for prenatal counseling and you obtain the family history documented in the
pedigree below. They have two healthy sons and she is pregnant again. What is the approximate
probability that her fetus will be severely affected with hydrocephalus?
A.
B.
C.
D.
E.
A7.
1/5.
1/8.
1/10.
1/20.
1/32.
Assume a three allele autosomal DNA polymorphism with gene frequencies of 0.50, 0.25, and
0.25. What proportion of the population will be heterozygous for these polymorphisms?
A.
B.
C.
D.
E.
4/16 (0.25)
6/16 (0.375)
8/16 (0.5)
10/16 (0.625)
12/16 (0.75)
653
A8.
A woman has a child with an autosomal recessive disorder that affects 1:40,000 in the population.
She remarries and the couple requests recurrence risk counseling. What is their approximate risk
of having an affected child?
A.
B.
C.
D.
E.
A9.
What is the probability that a couple who are both heterozygous for the (F508 mutation for CF
will have two affected and two unaffected with CF if they have four children?
A.
B.
C.
D.
E.
A10.
9%
18%
27%
36%
45%
Assume you discover a new X-linked platelet antigen detected by a monoclonal antibody.
Heterozygous and homozygous positive females are antigen positive. You find 20% of males to
be antigen positive and 80% to be antigen negative. Assuming Hardy-Weinberg equilibrium,
what percent of females would you predict to be antigen positive?
A.
B.
C.
D.
E.
A12.
9/32
9/64
9/128
27/128
27/256
Assume the following frequencies for the ABO blood group. The A allele = 0.30; the B allele =
0.10; and the O allele = 0.60. What percent of people should be blood group A?
A.
B.
C.
D.
E.
A11.
1/500
1/400
1/200
1/100
1/50
16%
20%
32%
36%
40%
A recessive condition has a population frequency of 1:90,000. Choose the number below that is
closest to the number of generations required to reduce the allele frequency by half, assuming that
all homozygous individuals are unable to reproduce?
A.
B.
C.
D.
E.
50
75
150
300
600
654
A13.
There is a three allele DNA polymorphism. A 10 kb fragment has an allele frequency of 0.10;
a 7 kb fragment has an allele frequency of 0.10; and a 5 kb fragment has an allele frequency of
0.80. Assuming Hardy-Weinberg equilibrium, what percent of individuals should be
heterozygous for this polymorphism?
A.
B.
C.
D.
E.
A14.
A GT repeat polymorphism has five alleles all with a frequency of 0.20. What proportion of
people should be heterozygous (informative)?
A.
B.
C.
D.
E.
A15.
0.10
0.20
0.50
0.80
0.90
You find an autosomal (two allele) RFLP and 25% of people are homozygotes for the upper
band. What should be the allele frequency for that band?
A.
B.
C.
D.
E.
A16.
34%
36%
50%
64%
66%
0.05
0.10
0.125
0.25
0.50
A couple seeks counseling regarding their risk of having a child with an autosomal recessive
condition that affects the brother of one partner (see pedigree). The population frequency of the
disorder is 1:400. What is the risk to one of their offspring of being affected?
A. 1/40
B. 1/60
C. 1/80
D. 1/100
E. 1/120
655
A17.
What is the coefficient of inbreeding (F; equal to the chance of homozygosity by descent) for the
child of the mating shown below?
A.
B.
C.
D.
E.
A18.
A couple comes to you for genetic counseling. The wife had two siblings that died from WerdnigHoffman syndrome (or spinal muscular atrophy 1, SMA1), an autosomal recessive disease with a
prevalence of approximately 1 in 20,000 livebirths. The husband reports a negative family
history. What is the probability that this couple will have a child with SMA1?
A.
B.
C.
D.
E.
A19.
1 in 70
1 in 120
1 in 240
1 in 360
1 in 440
A 35-year old man presents for genetic counseling because his mother recently died from an
autosomal dominant late-onset progressive neuromuscular disorder. By age 35 years, 75% of
individuals carrying the mutant gene have begun to show symptoms of muscle weakness and
slurred speech. A thorough evaluation by a neurologist did not detect any signs of this condition.
The gene for this condition is unknown and there are no available carrier tests. What is the
likelihood that this man inherited the mutant gene from his mother?
A.
B.
C.
D.
E.
A20.
1/4
1/8
1/16
1/32
1/64
1 in 5
1 in 10
1 in 20
1 in 30
1 in 50
Which of the following numbers below is the closest estimate of the mutation rate for an
autosomal dominant disorder with a population frequency of 1:5,000 in which reproductive
fitness is approximately 0.3?
A. 0.0001
B. 0.0002
C. 0.0003
D. 0.0004
E. 0.0005
656
A21.
Linkage is a violation of which of the following genetic concepts?
A.
B.
C.
D.
E.
A22.
The presence of an allele or genotype at one locus being necessary for the expression of a
genotype at a second locus is described by which of the following genetic concepts?
A.
B.
C.
D.
E.
A23.
Hardy-Weinberg equilibrium
Mendel’s law of independent assortment
Mendel’s law of segregation
Recombination
Variability of expression
pleiotropy
penetrance
linkage disequilibrium
variable expression
epistasis
A couple has a son with a multifactorial disorder in which it is more common for females to be
affected. Which of the following statements about recurrence risk is true?
A. Recurrence is higher for a next child than if a daughter had been affected, and would be higher
for a daughter than a son
B. Recurrence is higher for a next child than if a daughter had been affected, and would be higher
for a son than a daughter
C. Recurrence is lower for a next child than if a daughter had been affected, and would be higher
for a daughter than a son
D. Recurrence is lower for a next child than if a daughter had been affected, and would be higher
for a son than a daughter
E. Recurrence is 50% since a son is affected.
A24.
Mutations in the CFTR gene can affect multiple organ systems, including respiratory,
gastrointestinal, and reproductive systems. This is an example of which of the following genetic
concepts?
A.
B.
C.
D.
E.
A25.
pleiotropy
penetrance
linkage disequilibrium
variable expression
epistasis
Males with two mutations in the CFTR gene can have classic CF (lung involvement, pancreatic
insufficiency, male infertility) or only infertility. This is an example of which of the following
genetic concepts?
A.
B.
C.
D.
E.
pleiotropy
penetrance
linkage disequilibrium
variable expression
epistasis
657
A26.
A GeneReviews article notes the incidence of an autosomal recessive disease is 1 in 10,000. A
man has an affected child and has remarried and his new wife is pregnant. His mutation is known
and is found to be transmitted to the fetus. Carrier detection for the new wife is impractical.
What is the chance the fetus is affected?
A.
B.
C.
D.
E.
A27.
A woman has a son with an X-linked recessive lethal disorder and no other family history of the
condition. Her daughter has one healthy son. What is the daughter’s risk of being a carrier?
A.
B.
C.
D.
E.
A28.
1/2
1/3
1/4
1/6
1/12
What is the approximate odds ratio of a disorder in a person who carries an allele that is seen in
30% of those with the disorder but only 20% of controls?
A.
B.
C.
D.
E.
A30.
0.10
0.20
0.33
0.50
0.66
A couple’s first child is a boy with a very severe X-linked disease. There is no carrier testing for
this disease and the incidence is 1 in 100,000 boys. Neither partner has a family history of this
disease. The probability that the couple will have another affected child is about?
A.
B.
C.
D.
E.
A29.
1 in 25
1 in 50
1 in 64
1 in 100
1 in 200
1.5
0.8
1.7
1.9
2.4
Elizabeth’s maternal grandfather has hemophilia due to a factor VIII deficiency. Elizabeth has
married an unaffected man and has four healthy sons. Elizabeth is pregnant again with a male
fetus. What is the approximate probability that this fetus will develop hemophilia?
A.
0
B. 1/4
C. 1/25
D. 1/33
E. 1/50
658
A31. Leevi, a 32-year-old man of Finnish descent had a sibling who died at the age of 10 from cystic
fibrosis (CF). Leevi, who does not himself have CF, has recently married and would like to start a
family, but is concerned about the risk of passing CF to his children. DNA samples are not
available from his brother or from his parents, who are both deceased. He undergoes DNA testing
for the 70 most common CFTR mutations – a panel that detects 90% of the mutations found
among individuals of northern European descent. He tests negative for all 70 mutations. Given
this information, what is the probability that he is a heterozygous carrier for CF?
A.
B.
C.
D.
E.
A32.
2/3
1/4
1/6
1/15
1/25
Rob's father has Huntington's disease. Rob's mother has no family history of Huntington's
disease. Although Rob does not want to know if he has inherited the mutation, he is concerned
about the potential risks for his children. Rob's wife, who has no family history of Huntington's
disease, is pregnant. Prenatal testing is performed, using the alleles of an STR marker linked
(q = 0.0) to the Huntington gene. The allele results are shown below on the pedigree of Rob's
family.
Using this information, what is the approximate chance that the fetus has inherited the mutated
(expanded) copy of the Huntington gene?
A. 0
B. 1/4
C. 1/2
D. 3/4
E. 1
A33.
A 22-year-old man is at 50% risk of having inherited a mutation for a dominantly inherited
disorder. He has no signs of the disorder, but the condition displays age-dependent penetrance, so
that 20% of individuals at age 22 would expect to be affected. What is his approximate risk of
having inherited the mutation?
A.
B.
C.
D.
E.
0.86
0.52
0.44
0.24
0.20
659
A34.
Mr. and Mrs. Smith come to see you in your clinic very distressed. At 38 years old, Mrs. Smith is
pregnant with her fifth child. Her firstborn died from complications of cystic fibrosis. The
couple’s three other children are not affected with this condition, but Mr. and Mrs. Smith are
concerned the fetus may have CF. They have come to you for prenatal diagnosis. You screen the
Smiths for the 25 most common CF mutations, but they do not have any of these mutations. You
conclude they are carriers of rare CF mutations. Fortunately, there is a microsatellite marker very
tightly linked (theta = 0.00) to the CF-causing gene. You obtain DNA from all the surviving
family members and the fetus (via CVS) and genotype them with respect to this linked marker.
The corresponding alleles are shown below:
9,3
9,4
?
9,9
3,4
3,9
9
9,4
4
Based on these results, what is the chance that the fetus has CF?
A.
0
B. 1/4
C. 1/2
D. 3/4
E. 1
A35.
What is the approximate risk that the woman indicated by the arrow in the pedigree below will
have a son with the X-linked recessive lethal disorder that affects her brother?
A.
B.
C.
D.
E.
0.50
0.25
0.10
0.05
0.01
660
A36.
A case control study is done in which a specific allele is tested in affected and unaffected
individuals. The allele is found in 40% of affected individuals but only 10% of controls. Which
of the following is the best estimate of the odds ratio of disease given the presence of the allele
compared with the allele not being present?
A.
B.
C.
D.
E.
A37.
2
4
6
8
10
A couple seek counseling regarding their risk of having a child with a specific recessive disorder
that occurs in 1:40,000 births in the mother’s ancestry and 1:10,000 in the father’s. Which of the
following is the chance that they would have an affected child?
A.
B.
C.
D.
E.
1:5,000
1:10,000
1:20,000
1:400,000,000
1:1,600,000,000
A38. 37. You are evaluating a large family depicted in the pedigree below. Which of the following
modes of inheritance is most compatible with this family history?
A.
B.
C.
D.
E.
Autosomal recessive
Digenic
Maternal
X-linked dominant
X-linked recessive
661
Answers to Genetic Transmission (Quantitative Genetics) Questions A1-A38
A1.
For the direct method of estimating the mutation rate,
Number of sporadic cases with condition
ȝ [numbers of individuals examined x #sporadic cases
The correct answer is C
A2.
The easiest approach is to calculate the probability that none of the children is affected which is
(3/4)4. This is 81/256. The difference (175/256) is the probability that at least one child will be
affected. It is not as easy to calculate the probability, for example, that two children would be
affected and two children would be unaffected. This would involve using the binomial expansion
or making very careful calculations to consider all the possible birth orders and multiplying these
out.
The correct answer is D.
A3.
Expect a question on coefficient of inbreeding. You can memorize the usual numbers, calculate
the probability that the offspring of the mating will be homozygote by descent at a given locus in
simple cases, or use the method of path coefficients as described below:
Using Sewall Wright’s path coefficient method, the coefficient of relationship of the parents is
(1/2)n where n is the number of links through common ancestors on a path through the pedigree.
Here n = 2 for the path through from the father through his mother to his half-sister (mother of
child). The coefficient of relationship of the parents is therefore 1/4. The coefficient of
inbreeding for the child is ½ the coefficient of relationship for the parents or 1/8.
The correct answer is B.
A4.
Based on the affected sibling, the mother most likely has the A allele in coupling with the
disorder. The risk to the child is therefore 2T – 2T2 = 0.0950.
The correct Answer is C.
A5.
Use Bayesian calculation that III-1 is a carrier.
Prior
Conditional
Joint
Posterior
Carrier
1/2
1/2x1/2x1/2
1/16
1/9
Carrier risk x 1/4 = risk of affected boy = 1/36.
The correct answer is E.
662
Non-Carrier
1/2
1
8/16
8/9
A6.
Aqueductal stenosis can be X-linked and appears to be so in this family. The probability she is a
carrier is a Bayesian calculation.
Carrier
1/2
1/2x1/2
1/8
1/5
Prior
Conditional
Joint
Posterior
Non-Carrier
1/2
1
4/8
4/5
The chance she is a carrier (1/5) times the chance of an affected male (1/4) = 1/20.
The correct answer is D.
A7.
It is easiest to calculate the frequency of each of the homozygotes which is simply the sum of the
square of the gene frequencies. Thus the frequency of homozygotes would be 1/4 + 1/16 + 1/16 =
6/16. Thus the proportion of the population which would be heterozygous would be 10/16.
The correct answer is D.
A8.
Explanation: q
1
40,000
1
2q = 1/100 = risk partner is a carrier
200
Risk = 1(1/100)(1/4) = 1/400
The correct Answer is B.
A9.
The probability for two of each is 1/4 x 1/4 x 3/4 x 3/4 but this can occur in any of six orders. The
affecteds may be 1 + 2, 1 + 3, 1 + 4, 2 + 3, 2 + 4, or 3 + 4. Therefore the answer is 6(1/4)2 (3/4)2 =
27/128. You could use binomial expansion.
The answer is D.
A10.
Individuals of genotype AA would be 0.3 X 0.3 = 0.09. Individuals of genotype AO would be 2
X 0.3 X 0.6 = 0.36. Adding these together gives 0.45.
The correct answer is E.
A11.
The male data gives a gene frequency for antigen negative of p = 0.8. Antigen positive
homozygotes would be 0.2 X 0.2 = 0.04. Heterozygous females would be 2 X 0.2 X 0.8 = 0.32.
Adding these two together gives the correct answer, D.
A12.
Explanation:
q
1
90, 000
1
300
n = 1/q = 300
The correct Answer is D.
A13.
It may be easier to calculate the frequency of homozygotes which would be each allele frequency
squared. The sum of each
allele frequency squared is 0.66. This would be the frequency of
homozygotes. The frequency of heterozygotes would be 0.34 = 34%.
The correct answer is A.
663
A14.
Similarly for this question, the frequency of homozygotes would be each gene frequency squared.
Since they are all same, one can calculate 5 X 0.2 X 0.2 = 0.2 for the frequency of homozygotes.
Thus the heterozygotes would be 0.80.
The correct answer is D.
A15.
The allele frequency would be the square root of the frequency of the homozygotes which would
be 0.5.
The correct answer is E.
A16.
The male in the couple has a 2/3 risk. The frequency of the allele is 1/20 (square root of 1/400),
so the carrier frequency is approximately 1/10. Hence their risk of having an
affected child is
(2/3)(1/10)(1/4)= 1/60
The correct Answer is B.
A17.
Using Sewall Wright’s path coefficient method, the coefficient of relationship of the parents is
(1/2)n where n is the number of links through common ancestors on a path through the pedigree.
Here n = 5 for the path through from the father through his great-grandfather to the mother and
also n = 5 from the father through his great-grandmother to the mother. The coefficient of
relationship of the parents is therefore 1/32+1/32 = 1/16. The coefficient of inbreeding for the
child is ½ the coefficient of relationship for the parents or 1/32.
The correct answer is D.
A18.
The wife has a 2/3 risk of being a carrier. The allele frequency is the square root of 1/20,000 =
1/141. The carrier frequency is twice that, about 1/70. Hence the risk to a child is 2/3 x 1/70 x ¼
= 1/420.
E is closest to this value.
A19.
This is best handled as a Bayesian calculation:
Prior
Conditional
Joint
Posterior
Inherited Gene
1/2
1/4
1/8
1/5
Did Not Inherit Gene
1/2
1
4/8
The correct answer is A.
A20.
Explanation:
2p = 1/5,000
p = 1/10,000
ľűŴġľġĩIJİIJıĭıııĪĩıįĸĪġľġıįııııĸġſġıįıııIJġĩųŦŮŦŮţŦųġŵũŢŵġŴġľġIJ-F)
The correct Answer is A.
A21.
The correct answer is B.
A22.
The correct answer is E.
664
A23.
This is an example of the Carter effect. Since the disorder is more common in daughters, the fact that a
son was affected says that the liability in the couple must be higher than if a daughter had been affected.
This makes the recurrence risk higher for the next child, but recurrence for this trait is always higher for a
daughter.
The correct Answer is A.
A24.
The correct answer is A.
A25.
The correct answer is D.
A26.
We are told essentially that q2 equals 1/10,000 (q=1/100). The chance that the fetus will be
affected is the chance that the mother is a carrier (essentially 2q) times the ½ chance that she will
pass on the altered gene. Therefore 1/100 x 2 x 1/2 = 1/100.
The correct answer is D.
A27.
Explanation:
The
The mother has a 2/3 risk of being a carrier. For the daughter:
Prior
Conditional
Joint
Posterior
Is carrier
1/3
1/2
1/6
1/5
not carrier
2/3
1
2/3
4/5
correct Answer is B.
A28.
The correct answer is D. Answer: 2/3 chance of being a carrier and 1/4 chance of having any
affected child = 1/6.
A29.
Explanation:
The correct Answer is C.
A30.
The correct answer is D. – Begin with the prior risk that Elizabeth is a carrier for hemophilia.
Her mother is an obligate carrier and Elizabeth’s carrier risk is ½. Then use Bayesian analysis to
calculate the risk given Elizabeth has four unaffected sons.
Prior
Conditional
Joint
Posterior
Carrier
1/2
1/2x1/2x1/2x1/2
1/32
1/17
Non-Carrier
1/2
1
16/32
16/17
So, given that she had four unaffected sons, Elizabeth’s new risk of being a carrier is 1/17. She is
pregnant with a male fetus so the chance she passes along the mutant chromosome becomes 1/17
x ½ = 1/34 ~ 1/33 probability.
665
A31.
The correct answer is C. – Begin with the prior risk that Leevi is a carrier for CF. Because he is
not affected with the disorder, the risk he is a carrier is 2/3. Then use Bayesian analysis to
calculate the risk given his negative testing results.
Prior
Conditional
Joint
Posterior
Carrier
2/3
1/10
2/30
1/6
Non-Carrier
1/3
1
10/30
5/6
Conditional (results of negative screening test)
1/10 (the chance he has a mutation not detected by the panel)
1 (if he is not a carrier, you expect him to test negative)
So, given that he tested negative for the CF panel, his new risk of being a carrier for CF is 1/6.
A32.
The correct answer is A. – Although Huntington's disease can be tested for directly, Rob does
not want to know if he has inherited the mutation. By testing the fetus for a linked marker, this
can be avoided. The goal of this type of approach is to determine whether the fetus has inherited
the copy of Huntington gene that originated in Rob's father. Note that we do not know if this is
the expanded or normal copy. In the example given above, the fetus inherits the copy of
Huntington gene that originated in Rob's mother, which is of normal size. This puts the chance of
Huntington's disease closest to zero for this fetus.
A33.
Explanation:
prior
conditional
joint
posterior
0.5
0.8
0.4
0.4/0.9
0.5
1
0.5
The correct Answer is C.
A34.
The correct answer is E. – CF is an autosomal recessive disease and since the first child was
affected with CF, you know both parents are carriers. The common mutation panel doesn’t
identify either parent’s mutation, but you can use the linked marker in a diagnostic manner. The
alleles shown are the alleles present at the linked marker. A priori you do not know which allele
of the marker is on the same chromosome as the copy of the CF gene with the mutation. You can
look at the alleles present in the unaffected children however, to get an idea of the combinations
that do not result in CF. Thus by process of elimination, you can deduce that “9,9”, “3,4”, “3,9”
are not associated with homozygosity for the mutation. That leaves only “9,4” as the combination
that represents both copies of the mutation and the probability that the fetus will have CF is
closest to 1. Note that it is the “9” allele from Dad and the “4” allele from Mom. (Mom has a
“9,4” combination, but in her family, the mutation is associated with the “4” allele, not the “9”
allele).
666
A35.
Explanation:
Mother:
prior
conditional
joint
posterior
4P
1/16
P
1-4Pγ 1
P
P
1/2
0.05
0.05/0.95 = 0.0526
0.90
1
0.90
Daughter:
prior
conditional
joint
posterior
The correct Answer is D.
A36.
CORRECT ANSWER: C
SOURCE OF ITEM TOPIC: Lecture/Syllabus
KEYWORDS: odds ratio, multifactorial
EXPLANATION: The odds of disease given the allele are 40/10 = 4; the odds of disease given
not having the allele = 60/90 = 2/3; the odds ratio is 4 divided by 2/3 = 6
A37.
CORRECT ANSWER: C
SOURCE OF ITEM TOPIC: Lecture/Syllabus
KEYWORDS: population genetics, risk assessment
EXPLANATION: The allele frequency in the mother’s population is 1/200, so her risk of being a
carrier is 1/100. The allele frequency in the father’s population is 1/100, so his risk of being a
carrier is 1/50. Their risk of having an affected child is 1/100 x 1/50 x 1/4 = 1/20,000.
Remember to calculate the carrier frequency, not the allele frequency.
A38.
CORRECT ANSWER: D
SOURCE OF ITEM TOPIC: Lecture/Slides
KEYWORDS: Mendelian inheritance
EXPLANATION: The founding male transmits to all his daughters but not to his son; his
daughters transmit to half their offspring. It is possible for autosomal dominant to explain this
pedigree, but that choice is not provided. It is not autosomal recessive, as it is transmitted from
generation to generation; it is not maternal, as the founding transmitting individual is male; it is
not likely X-linked recessive, since many females are affected. Digenic inheritance is unlikely
given the many couples who have had affected offspring.
667
B. Biochemical Genetics/Newborn Screening Questions B1-B72
The vignette for the next two questions is the same:
B1. New parents bring their 1-week-old baby boy to the pediatrician's office and report that he is not
feeding as well over the last 24 hours and that his diapers have an odor like sweaty feet. Which of
the following disorders is the most likely cause of this child's problems?
A.
B.
C.
D.
E.
Isovaleric acidemia
Maple syrup urine disease.
Methylmalonic acidemia.
Phenylketonuria
Propionic acidemia
B2. Which of the following laboratory tests is most likely to provide a definitive diagnosis?
A.
B.
C.
D.
E.
Comprehensive Metabolic Panel
Plasma amino acids
Plasma ammonium level
Urine amino acids
Urine organic acids
The vignette and response options for the next 2 items are the same.
Select one answer for each item in the set.
For each description, select the associated glycogen storage disorder (A-D).
.
Glycogen storage diseases result from several different enzyme deficiences, a few of which are
listed below. Match the clinical description below with the appropriate diagnosis from this list.
You may use an answer once, more than once, or not at all.
A.
B.
C.
D.
Type la is glucose-6-phosphatase deficiency or von Gierke.
Type II is (alpha-glucosidase (acid maltase) deficiency or Pompe.
Type III is debrancher deficiency or Forbes.
Type V is muscle phosphorylase deficiency or McArdle
B3.
A six-month-old boy presents with hepatomegaly, renomegaly, hypoglycemia and lactic acidosis.
B4.
A six-month-old boy presents with severe hypotonia, massive cardiomegaly, progressive
weakness and markedly elevated CPK.
668
B5.
A four-day-old infant boy is brought to the emergency room lethargic and no longer taking his
formula. A metabolic profile reveals metabolic acidosis and his ammonium level is normal.
Which of the following diagnoses is most likely to be found with subsequent metabolic screening
studies?
A. Citrullinemia
B. Maple syrup urine disease
C. Methylmalonic acidemia
D. Ornithine transcarbamylase deficiency
E. Proprionic acidemia
B6.
A 6 year old girl is referred to genetics following an episode of pancreatitis. During her
hospitalization, she received hyperalimentation and became obtunded. An ammonia level during this
episode was 200 meq/L. Following discontinuation of the hyperalimentation and treatment of the
pancreatitis, the girl was discharged home on a low fat diet. She has not had any further episodes of
pancreatitis or lethargy. You suspect she has a specific metabolic disorder. Which of the following
laboratory evaluations is the BEST way to obtain a specific diagnosis?
A.
B.
C.
D.
E.
B7.
You are seeing a couple whose first child is a Duarte/classical Galactosemia genetic compound
heterozygote. His parents (who have not been tested) ask about the risk for their next child to
have classical galactosemia (Gal/Gal). Knowing that the frequencies of the Duarte (1/27) and Gal
(1/278) alleles, which of the following do you think is their risk to have a Gal/Gal child?
A.
B.
C.
D.
E.
B8.
Gene sequencing
Plasma amino acids
Plasma ammonia level
Protein challenge
Urine orotic acid
1/278
1/278 x 2/3
1/278 x ½
1/278 x 2/3 x ½
1/278 x ½ x ½
You are notified by the state newborn screening laboratory that a seven-day-old neonate, born at
33-weeks gestation and is receiving antibiotics for possible sepsis and total parenteral nutrition
(TPN). He was found to have a phenylalanine level (Phe) of 4.8 mg/dL (291 (M). Which of the
following interventions is the most appropriate next step in evaluating or managing this child's
care?
A.
B.
C.
D.
E.
Stop (TPN) briefly and remeasure Phe on glucose and IV fluids.
Modify antibiotic coverage patient is probably receiving
Monitor the phenylalanine level weekly over next month.
Restrict the infant's phenylalanine intake
Obtain urine for assessment of biopterin metabolite levels.
669
B9.
A 3 year old boy is referred to genetics clinic for evaluation of hypotonia, seizures, and developmental
delay. His older brother, age 8, was also hypotonic and has been diagnosed with autism. You suspect a
disorder of creatine metabolism. Which of the following studies is the best option to confirm a
diagnosis for this child?
A.
B.
C.
D.
E.
B10.
A nine- month-old infant contracts a viral illness and is unable to take her usual amount of formula
over the previous 24 hours. She is found dead in her bed the following morning. Which of the
following disorders accounts for approximately 5% of cases of sudden infant death syndrome
(SIDS) and is the most likely cause of this unfortunate infant’s death?
A.
B.
C.
D.
E.
B11.
Glutaric aciduria Type I
Glycogen storage disease Type III
MCAD deficiency
OTC deficiency
Propionic academia
A couple comes for genetic counseling because the husband’s sister lost a child to Tay-Sachs (TSD)
disease. The wife has a negative family history for TSD, is not Jewish, and is currently 6 weeks
pregnant.
A.
B.
C.
D.
E.
B12.
Measure plasma guanidinoacetoacetate
Measure plasma creatine to creatinine ratio
Perform a brain MRI
Perform plasma amino acids to measure an ornithine level
Sequence the SLC6A8 creatine transporter gene
Perform molecular TSD analysis on the wife.
Perform molecular TSD analysis on both the husband and the wife.
Perform no testing since the risk is low for having a child with TSD
Perform serum HexA and molecular TSD analysis on the husband.
Perform serum HexA testing on the wife
A 9-year-old boy is referred by a neurologist for evaluation of a 6-month history of decreasing
intellectual performance. An MRI shows prominent white matter changes. Serum biochemistries
reveal evidence of mild adrenal dysfunction. Which of the following biochemical pathway
components is most likely to be encoded by the gene for this disorder?
A.
B.
C.
D.
E.
Lysosomal enzyme
Mitochondrial enzyme involved in fatty acid oxidation
Peroxisomal enzyme involved in fatty acid oxidation
Peroxisomal membrane protein
Transcription factor
670
B13.
You are asked to evaluate a one month old asymptomatic girl who had an elevated tyrosine on
newborn screening. Succinylacetone testing was subsequently positive. Which of the following
interventions is the best initial course of action for this infant?
A.
B.
C.
D.
E.
B14.
You are asked to evaluate a newborn baby in the neonatal intensive care unit who is noted to have
proximal shortening of the humeri (and to a smaller degree the femurs) along with punctate
calcifications in cartilage, coronal clefting of the several vertebrae and congenital cataracts. You
suspect a peroxisomal disorder as the likely etiology. Which of the following biochemical
abnormalities is most likely to be found when appropriate peroxisomal studies are performed?
A.
B.
C.
D.
E.
B15.
Low plasmalogen levels
High very long chain fatty acid levels
High 7-dehydrocholesterol levels
Low phytanic acid levels
Low very long chain fatty acid levels
Hyperammonemia is a cardinal finding in the newborn period of several metabolic disorders,
however late onset hyperammonemia can also occur. Which of the following disorders is most
likely to present outside the newborn period with progressive neurologic findings and mild
hyperammonemia?
A.
B.
C.
D.
E.
B16.
Place the infant on a low phenyalanine diet
Place the infant on NTBC and a low tyrosine diet
Reassure the family that the infant had transient tyrosinemia of the newborn
Refer the infant for evaluation for a liver transplant
Repeat tyrosine levels at monthly intervals
Arginase deficiency
Citrullinemia
OTC deficiency
Propionic academia
Sepsis
Pycnodysostosis, a skeletal dysplasia, is caused by mutations in which of the following proteins?
A.
B.
C.
D.
E.
Cartilage-specific parathyroid related protein
Cathepsin K, a lysosomal enzyme
Collagen Type IX
Dystosin, a fibrillar structural protein
FGFR4
671
B17.
You are asked to evaluate a 10 month old boy with macrocephaly, seizures, and a leukodystrophy
pattern on MRI. You suspect Canavan disease. Which of the following analyses is the best screening
test to confirm your assumption?
A.
B.
C.
D.
E.
B18.
Acute intermittent prophyria, a defect in heme biosynthesis, is characterized by which of the
following inheritance patterns?
A.
B.
C.
D.
E.
B19.
Non-paternity
Pseudodeficiency
Unreliability of the assay for carrier testing
Decline of enzyme activity with age
His being affected with the disease but asymptomatic
Specific diagnosis of Glycogen Storage Disease Type IA is available by which of the follwoing
sets of laboratory analyses?
A.
B.
C.
D.
E.
B21.
A digenic disorder in 2 sequential enzymes in the heme biosynthesis
As an autosomal dominant disorder
As an autosomal recessive disorder
As an X-linked disorder affecting only males
Maternally since the enzyme is encoded by the mitochondrial genome
A 15 month old boy with hypotonia, optic atrophy, and neurologic regression is found to have
metachromatic leukodystrophy with 0.3 nmol/hr/mg protein Arylsulfatase A activity in white
blood cells. Activity in the mother’s and father’s white blood cells is 8.6 nmol/hr/mg and 1.2
nmol/hr/mg protein, respectively, with a normal control range of 11-25 nmol/hr/mg protein. The
low activity in the father is most likely the result of which of the following genetic issues?
A.
B.
C.
D.
E.
B20.
Acyl carnitine profile
Lysosomal enzyme screening on white blood cells
Plasma amino acids
Urine organic acids
Very long chain fatty acids
Enzyme assay of liver or muscle
Enzyme assay of skin fibroblasts or liver
Enzyme assay of liver or molecular analysis of the glucose-6-phosphatase gene
Molecular analysis of the glucose-6-translocase gene only
Western blotting of muscle with specific antibody to the protein
A new autosomal recessive disorder of N-linked glycosylation and sterol metabolism has recently been
described and involves a defect in an enzyme involved in the synthesis of the dolichol, a polyprenol
required for the synthesis and transfer of dolichol-linked monosaccharides to proteins. Which of the
following screening tests is most likely to detect this new disorder?
A.
B.
C.
D.
E.
Enzyme assay of the new protein
Plasma cholesterol
Plasma sterol analysis
Serum transferrin isoelectric focusing
Urine oligosaccharide chromatography
672
B22.
The metabolic defect in Fabry disease is a deficiency of which of the following enzymes?
A.
Į-galactosidase
B.
Į-glucosidase
C.
E-galactosidase
D.
E-glucosidase
E.
cerebroside E-galactosidase
B23.
Which of the following is a major biochemical abnormality in X-linked adrenoleukodystrophy?
A.
B.
C.
D.
E.
B24.
Malignancy is a major risk in which of the following metabolic disorders?
A.
B.
C.
D.
E.
B25.
Cystinosis
Hurler syndrome
Maple syrup urine disease
Propionic acidemia
Tyrosinemia
A 35 year woman is referred for evaluation of ptosis, limitation of eye gaze, and a cardiac arrhythmia.
You suspect a mitochondrial disorder. Which of the following mutations would be most compatible
with her clinical features?
A.
B.
C.
D.
E.
B26.
cerebroside E-galactosidase deficiency
dicarboxylic aciduria
pipecolic acidemia
plasmalogen deficiency
very long chain fatty acid excess
A de novo dominant mutation in a nuclear encoded gene for an OXPHOS subunit
A heteroplasmic deletion involving a portion of the mitochondrial genome
A homoplasmic missense mutation in a mitochondrial gene for an OXPHOS subunit
A heteroplasmic single base change in a mitochondrial rRNA gene
A heteroplasmic single base change in a mitochondrial tRNA gene
The best diagnostic approach for screening for Zellweger syndrome in an infant with
organomegaly and dysmorphic features is which of the following tests?
A.
B.
C.
D.
E.
Chromosome analysis
CT scan of the brain
Plasma carnitine levels
Plasma very long chain fatty acids levels
Radiographs of entire infant skeleton
673
B27.
Which of the following abnormalities is the most likely diagnosis in a 3-day-old infant who develops
lethargy, vomiting, respiratory alkalosis and hyperammonemia with undetectable plasma citrulline
and massive orotic aciduria?
A.
B.
C.
D.
E.
B28.
An asymptomatic infant who is found to have hyperphenylalaninemia on newborn screening should
be tested for which of the following associated problems?
A.
B.
C.
D.
E.
B29.
D1-antitrypsin deficiency
hepatitis C
hepatorenal tyrosinemia
tyrosine aminotransferase deficiency
acetyl CoA-carboxylase deficiency
A 37 year old woman is referred for a history of unexplained stroke. During her evaluation she was
found to have proteinuria. Her 42 year old brother has been on renal dialysis for 2 years and is
awaiting a renal transplant. Which of the following is the most likley diagnosis in this family?
A.
B.
C.
D.
E.
B31.
biopterin synthetic defects
catechol-o-methyltransferase
liver disease
microdeletion deficiency of chromosome 12
porphyria cutanea tarda
Which of the following disorders is the most likely diagnosis in an infant with progressive liver
failure, elevated serum alpha-fetoprotein and succinylacetone in the urine?
A.
B.
C.
D.
E.
B30.
argininosuccinate synthetase deficiency
carbamylphosphate synthase I deficiency
nonketotic hyperglycinemia
ornithine transcarbamylase deficiency
propionic acidemia
Alport syndrome
Fabry disease
MELAS syndrome
Von Hippel Lindau syndrome
X-linked Adrenoleukodystrophy
Which of the following abnormalities is shared by the hyperornithinemia-hyperammonemiahomocitrullinuria (HHH) syndrome and gyrate atrophy of the choroid and retina?
A.
B.
C.
D.
E.
hyperammonemia
hyperglutaminemia
hyperornithinemia
orotic aciduria
progressive retinal degeneration
674
B32.
Intermittent treatment with metronidazole is recommended in patients with propionic academia or
methylmalonic acidemia for which of the following therapeutic effects?
A.
B.
C.
D.
E.
B33.
Neonatal hypotonia, seizures, apnea and hiccups are features of which of the
following inborn errors of metabolism?
A.
B.
C.
D.
E.
B34.
Cobalamin complementation studies on the patient’s fibroblasts
Measurement of serum B12 level and total homocysteine in the mother
Measurement of serum B12 level and total homocysteine in the sister
Measurement of transcobalamin II levels in the patient
Sequencing the methylmalonyl CoA mutase gene in the patient
Very long branch-chain fatty acids undergo E-oxidation in which of the cellular organelles?
A.
B.
C.
D.
E.
B36.
citrullinemia
galactosemia
isovaleric acidemia
maple syrup urine disease
nonketotic hyperglycinemia
A six month old girl is referred for megaloblastic anemia. She is otherwise healthy and has been
exclusively breast fed. Metabolic screening reveals mildly elevated methylmalonic acid in her urine
and a plasma total homocysteine level of 27 uM (nl 5-8). The mother states that her older daughter,
now age 3, who is the patient’s maternal half-sister, had similar anemia as an infant, but is “now fine.”
Which of the following assessments is the best course of action in the evaluation of this patient?
A.
B.
C.
D.
E.
B35.
gut flora produce substantial amounts of propionate
it promotes normal bowel function
it provides a source of reducing equivalents
it reduces the incidence of sepsis
reduction of enteric bacteria synthesis of valine and isoleucine
endoplasmic reticulum
Golgi apparatus
mitochondria
nuclear envelope
peroxisomes
Transferrin isoelectric focusing is useful in diagnosing which of the following disorders?:
A.
B.
C.
D.
E.
Acute Intermittent Porphyria
Congenital disorders of glycosylation
Hemophilia B
Iron-deficiency anemia
Marfan syndrome
675
B37.
Which of the following mucopolysaccharidoses includes preserved cognitive function?
A.
B.
C.
D.
E.
B38.
A seven month old boy is referred for coarse facial features and hepatomegaly. His x-rays demonstrate
beaking of the lumbar vertebrae, a J-shaped sella tursica, and ribs that widen anteriorly. Lysosomal
enzyme screening from blood reveals elevated levels oIDU\OVXOIDWDVH$DQGȕ-glucuronidase. Whichof
the following diagnoses is most likely to explain this infant’s clinical findings?
A.
B.
C.
D.
E.
B39.
Androgen receptor deficiency
Aromatase deficiency
Cholesterol desmolase deficiency
5-Į-Reductase deficiency.
21-Hydroxylase deficiency
Thrombosis and strokes are a frequent complication of which one of the following diseases?
A.
B.
C.
D.
E.
B41.
Hurler syndrome
I-cell disease
Metachomatic leukodystrophy
MPS VII, Sly disease
Pseudo-Hurler polydystrophy
Metabolic defects that cause Congenital Adrenal Hyperplasia usually involve which of the
following enzyme deficiencies?
A.
B.
C.
D.
E.
B40.
Hunter syndrome (MPS II)
Hurler syndrome (MPS IH)
Maroteaux-Lamy syndrome (MPS VI)
Sanfilipo syndrome (MPS III)
Sly disease (MPSVII)
Congenital Adrenal Hyperplasia
Galactosemia
Homocystinuria
Medium Chain AcylCoA Dehydrogenase Deficiency
Phenylketonuria
Which of the following NBS acylcarnitine profiles is most suggestive of MCAD deficiency?
A.
B.
C.
D.
E.
Increased C3 and C4
Increased C5-OH
Increased C6, C8 and C8/ C10 ratio
Increased C14-OH, C16-OH, C18-OH and C18:1-OH
Increased C14:1 and C14:1/ C12:1 ratio
676
B42.
The MS/MS result of newborn screening sample number 1257 (shown at the bottom) is most
consistent with which of the following disorders?
d3-Leu
100
%
Ala
Leu
d4-Ala
Phe
Normal
d5-Phe
Tyr d6-Tyr
d -Met
Met 3
0
Phe
100
Elevated Phenylalanine
PKU 1257
Sample
%
0
140
160
180
200
220
240
260
m/z
280
A. d5 Phe deficiency
B. Hyperphenylalaninemia
C. Maple Syrup Urine Disease
D. Normal result
E. Tyrosine deficiency
B43.
A 20 month old girl is referred for hypotonia, developmental delay and difficult to control seizures. As
part of her evaluation you recommend a lumbar puncture to be sent for special biochemical analyses,
including neurotransmitter studies. You wish to exclude a disorder where a medical treatment can
result in a dramatic improvement in symptoms. Which of the following disorders would meet this
criterion and be applicable in this patient?
A.
B.
C.
D.
E.
adenylosuccinate lyase deficiency
Cerebral folate deficiency
GLUT1 deficiency
Menkes disease
Nonketotic hyperglycinemia
677
B44.
The results of BH4 challenge tests are shown below for individuals A-D who have MHP, Mild
PKU, or PKU. Which individuals appear to respond to BH4?
B
A
C
D
Blau N.PKU & BH4 SPS Verlagsgesellschaft mbH, Heilbronn 2006; p 409.
A.
B.
C.
D.
E
B45.
Individuals A, and B but not C and D appear to respond .
Individuals A, and D but not B and C appear to respond.
Individuals A, B, C and D appear to respond.
Individuals A, B and C but not D appear to respond
Individuals C, and D but not A and B appear to respond
A neonate, Sam, was referred for an elevated Gal-1-P and E Coli sepsis at 8 days of age. Sam is
four-weeks-old and he is on Prosobee (lactose free) formula. Which of the following laboratory
tests is most likely to yield information that will guide dietary recommendations for Sam?
A.
B.
C.
D.
E.
Detection of urinary carbohydrate by Clinitest
Galactose-1-P uridyltransferase activity level
N1314D and -119GTCA deletion genotyping
Q188R genotyping
Total galactose level in his blood
678
B46.
Sarah is a five-week-old girl who is brought to clinic by her foster parents. She is on regular
formula. Her foster parents just received Sarah’s newborn screening results that show Sarah had
phenylalanine levels of 660 and 1220 μmol/L at 3 and 32 days of age, respectively. After obtaining
plasma amino acids, which of the following interventions is the best management plan in this
situation?
A.
B.
C.
D.
E.
B47.
Your state Newborn Screening (NBS) Laboratory sends you a batch report of five abnormal NBS
results on neonates discharged from your facility two days ago. Which of the following abnormal
NBS results warrants your immediate attention for finding the patient and performing definitive
testing?
A.
B.
C.
D.
E.
B48.
Biotinidase enzyme level was 1.2 times the normal cutoff
C6 and C8 levels were 1.2 times the normal cutoff
GALT enzyme level was 1.2 times the normal cutoff
Immunoreactive Trypsinogen level 0.8 times the normal cutoff
Phenylalanine level was 1.2 times the normal cutoff
You are asked to evaluate a 34-week fetal demise with fetal hydrops. Congenital heart defects,
arrhthymias, and immune causes have been excluded. Which of the following inborn errors of
metabolism is most likely to present this way?
A.
B.
C.
D.
E.
B49.
Start Sarah on a phenylalanine-free formula
Start Sarah on large neutral amino acids
Start Sarah on PEGylated phenylalanine ammonium lyase injections
Obtain urine pterins and start Sarah on a phenylalanine-free formula
Obtain urine pterins and start Sarah on tetrahydrobiopterin (Sapropterin)
Maple syrup urine disease
OTC deficiency
Sialidase deficiency
Very long chain acyl CoA dehydrogenase deficiency
Zellweger syndrome
You are treating a one year-old with propionic acidemia diagnosed by newborn screening.
Despite optimal management, the infant is not growing well, requires G-tube feeds, and has
frequent hospitalizations with intercurrent illnesses and mild hyperammonemia. You decide to
treat the infant with carglumic acid (N-carbamylglutamate) at the beginning of the next illness to
try to prevent the hyperammonemia. Which of the following mechanisms best describes the
action of carglumic acid in preventing elevated NH3?
A.
B.
C.
D.
E.
Acting as a synthetic required cofactor for carbamyl phosphate synthetase I
Increasing the renal excretion of urea
Inhibiting N-acetylglutamate synthase
Inhibiting the production of organic acids that inhibit the urea cycle
Removal of nitrogen by an alternate pathway
679
B50.
You are asked to perform a consult for an internist regarding a 46-year-old man with significant
liver disease and early cirrhosis. He has a 10-year history of significant alcohol consumption and
darkened complexion in skin-exposed areas. You suspect porphyria cutanea tarda. Which of the
following patterns of laboratory porphyrins or porphyrin precursors would be consistent with this
diagnosis?
A.
B.
C.
D.
Elevated red blood cell (RBC) free protoporphyrin and stool protoporphyrin
Elevated RBC free and Zinc (Zn) protoporphyrin and stool protoporphyrin
Elevated urine aminolevulinic acid (ALA), porphobilinogen (PBG) and uroporphyrin
Elevated urine ALA, PBG, and coproporphyrin III and stool coproporphyrin III and
protoprphyrin
E. Elevated urine uroporphyrin and stool isocoproporphyrin
B51.
You are asked to consult about a 10-month-old girl with hypotonia and developmental delay. This
is the couple’s first child, and there is no known consanguinity in the family. Her plasma lactate
is elevated (6 mM/L, normal <2). Her MRI shows basal ganglia lesions and white matter changes.
You suspect Leigh syndrome. Which of the following mutations would be most compatible with
this diagnosis?
A.
B.
C.
D.
E.
B52.
A 2½-year-old toddler has neurologic regression, seizures, and a very low level of serum
hexosaminidase A activity. On ophthalmologic exam, which of the following clinical findings would
you expect to find?
A.
B.
C.
D.
E.
B53.
A de novo autosomal dominant mutation in a nuclear-encoded gene for an OXPHOS subunit
A heteroplasmic deletion involving a portion of the mitochondrial genome
A heteroplasmic single base change in a mitochondrial tRNA gene
A homoplasmic missense mutation in a mitochondrial gene for an OXPHOS subunit
Compound heterozygous mutations in a nuclear encoded gene for an OXPHOS subunit
Cherry red spot in the macula
Corneal clouding on slit lamp examination
Optic atrophy consistent with white matter disease
Raised intraocular pressure
Retinitis pigmentosa at the periphery of the retina
You are asked to consult about a 2-month-old girl with hypotonia, seizures, and an elevated
plasma lactate (8 mM/L, normal <2). Brain MRI shows a thin corpus callosum but no other
abnormalities. You suspect pyruvate dehydrogenase deficiency. Which of the following is the
most likely mode of inheritance in this infant?
A.
B.
C.
D.
E.
Autosomal dominant
Autosomal recessive
Mitochondrial
Sporadic
X-linked
680
B54.
A 6-month-old infant presents with FTT, enlarged liver, hypotonia, and developmental delay.
You diagnose Carbohydrate Deficient Glycoprotein syndrome Type Ib based on transferrin
glycosylation and gene sequencing. Which of the following interventions is the most appropriate
treatment for this disorder?
A.
B.
C.
D.
E.
B55.
You are asked to evaluate a 2-year-old boy with coarse facies, developmental delay, and
hepatomegaly. You note that the corneas are clear; there is mixed sensorineural and conductive
hearing loss; and dysostosis multiplex on skeletal survey. Which of the following
mucopolysaccharidoses (MPS) is the most likely diagnosis?
A.
B.
C.
D.
E.
B56.
Development of sensorineural or conductive hearing loss
Lack of development of antibodies in treated patients
Loss of visual acuity and retinal detachment
Normal gross motor skills after 12 months of therapy
Progressive cardiac enlargement
A couple is 9 weeks pregnant. A previous child died shortly after birth with a severe presentation
of Smith-Lemli-Opitz disease. Which of the following clinical/laboratory findings is most likely
to be found with an affected fetus in the current pregnancy?
A.
B.
C.
D.
E.
B58.
Hunter syndrome (MPS II)
Hurler syndrome (MPS I)
Maroteaux-Lamy syndrome (MPS VI)
Morquio syndrome (MPS Type IVA)
Sanfilippo syndrome (MPS III)
Early enzyme replacement therapy has dramatically altered the prognosis for infantile Pompe
disease. Which of the following represents an unexpected clinical finding in long-term treated
survivors?
A.
B.
C.
D.
E.
B57.
Enzyme replacement therapy
Oral mannose supplementation
Supportive care only
Bone marrow transplant
Low carbohydrate elemental formula
Abnormal microarray with small deletion on chromosome 11q
Abnormal nuchal translucency at 10 weeks on prenatal ultrasound
High maternal serum alpha-fetoprotein level
Low maternal serum estriol level
Polyhydramnios on prenatal ultrasound at 24 weeks
You evaluate a 6-day-old term girl in the emergency room with an unremarkable prenatal and family
history. She was discharged home at 2 days of age on regular infant formula. For 24 hours she has been
eating poorly (<1 ounce per feed) and seems “sleepy.” She has vomited once. Her CBC is normal, her
electrolytes show a metabolic acidosis and the ammonia level is 500 mg/dl. Which of the following
metabolic laboratory results are you the most likely to find?
A.
B.
C.
D.
E.
Elevated plasma glycine with 3-OH-propionate, methylcitrate present on urine organics acids
Low citrulline on plasma amino acids along with high orotic acid in urine
Markedly elevated citrulline on plasma amino acids and normal urine organic acids
Markedly elevated plasma glycine with normal urine organic acids
Normal plasma amino acids and large suberylglycine peak on urine organic acids
681
B59.
An 8-year-old boy and his family recently moved to the United States from Russia. His parents report their
son has a genetic disorder and needs a special diet. Newborn screening was never performed. On physical
examination you note macrocephaly and choreoathetotic movements. Height and weight are 25-30th centile
for age. The parents note he has some mild cognitive deficits, but attended regular school in Russia. Which
of the following metabolic disorders is the most likely diagnosis?
A.
B.
C.
D.
E.
B60.
A couple, who had a previous child with classic Hurler syndrome who died of disease complications,
comes to you for pre-conception counseling. Each parent has been molecularly confirmed to carry a known
pathogenic mutation. Which of the following interventions will likely provide the best outcome for a future
affected pregnancy?
A.
B.
C.
D.
E.
B61.
Canavan disease
Glutaric academia Type I
Isovaleric acidemia
Medium chain acyl-CoA dehydrogenase deficiency
Phenylketonuria
HLA matched (66%; 4/6) unrelated donor bone marrow transplant in the early postnatal period
HLA matched liver transplant when the infant reaches a weight of 10 kilograms
Institution of chaperone therapy as early as possible in the postnatal period
Institution of enzyme replacement therapy at the first sign of neurologic regression
Unrelated donor, HLA matched (100%; 6/6) cord blood transplant in the early postnatal period
A 15-year-old healthy girl is running long distances as a member of her high school track team. The
morning after a 10-mile run on a hot day, she notices a red color to her urine. She is referred to you to
exclude an inborn error of metabolism. Her creatine kinase is markedly elevated at 15,000 U/L and urine
myoglobin is positive. Which of the following disorders of fatty acid oxidation is most compatible with
this presentation?
A.
B.
C.
D.
E.
Carnitine palmitoyltransferase I (CPT I) deficiency
Carnitine palmitoyltransferase II (CPT II) deficiency
Long chain 3-OH Acyl-CoA dehydrogenase deficiency
Medium chain acyl-CoA dehydrogenase deficiency
Short chain acyl-CoA dehydrogenase deficiency
NEWBORN SCREENING
B62. You are seeing a neonate, Katie, who has a positive Newborn Screening (NBS) test result. The NBS test
has a sensitivity of 0.95, a specificity of .90 and a positive predictive value of 0.10. What is the probability
that Katie actually has the disorder?
A.
B.
C.
D.
E.
B63.
95%
90%
10%
9.5%
5%
A Newborn Screening (NBS) test of 1000 neonates yielded 100 who tested positive. Subsequent diagnostic
testing showed that only 10 of the 100 who tested positive and none of those who tested negative on NBS
actually had the disease. Which of the following is the best estimate of the sensitivity of the NBS test?
A.
B.
C.
D.
E.
90/990
10/100
100/990
900/990
10/10
682
B64.
A neonate, Billy, failed his newborn hearing screening test and was also found to have a 17
hydroxyprogesterone (17OHP) level that is 1.5 times the upper range of normal. Which of the following
disorders do you think Billy is most likely to have?
A.
B.
C.
D.
E.
B65
ACTH Deficiency
Congenital Adrenal Hyperplasia
Pendred Syndrome
Primary Congenital Hypothyroidism
Secondary Congenital Hypothryroidism
A 6-day-old neonate, Sammy, presents with acidosis, ketonuria, hyperammonemia, neutropenia
and thrombocytopenia. Sammy’s Newborn Screening (NBS) test shows an isolated elevation of
C5. Which of the following disorders is Sammy most likely to have?
A.
B.
C.
D.
E.
Biotinidase Deficiency
Holocarboxylase Deficiency
Isovaleric Acidemia
Methylmalonic Acidemia
Propionic Acidemia
Source: Newborn Screening Slides 42-43 and 49
Keywords: acidosis, ketonuria, hyperammonemia, neutropenia, thrombocytopenia, C5
Explanation: Isovaleric Acidemia presents with acidosis, ketonuria, hyperammonemia,
neutropenia, thrombocytopenia, an elevated C5 and the odor of sweaty feet. Biotinidase
Deficiency presents with alopecia, rash and elevated C5-OH. Holocarboxylase Deficiency
presents with elevated C5-OH. Methylmalonic Acidemia and Propionic Acidemia present with
elevated C3.
B66.
A newborn infant is diagnosed with Medium Chain AcylCoA Dehydrogenase Deficiency (MCADD) based
on newborn screening (NBS) results. While awaiting the molecular analysis to confirm the NBS result,
which of the following complications should you warn the parents about during their initial visit to your
medical genetics clinic?
A.
B.
C.
D.
E.
B67.
Hyperammonemia
Hypoglycemia
Ketosis
Neutropenia
Thrombocytopenia
You are adjusting the cutoffs for a Newborn Screening (NBS) test for a metabolic condition. Your
adjustments increase the number of true positive results and reduce the number of false positive results.
Both of these changes will increase which of the following NBS test parameters?
A.
B.
C.
D.
E.
False negative rate
False positive rate
Positive predictive value
Sensitivity of the test
Specificity of the test
683
B68.
As you evaluate a 4-day-old boy with a blood sugar of 37, an anion gap of 21, 4+ urinary ketones and an
ammonia level of 197, the State Newborn Screening (NBS) Lab contacts you to report an emergency NBS
result. The emergency result is most likely to reveal an increase in which of the following levels?
A.
B.
C.
D.
E.
B69.
You are seeing a 4-day-old girl with abnormal Newborn Screening (NBS) result. She has a 2-day history of
emesis and diarrhea. On physical examination she is quite icteric with hepatomegaly. Which of the
following abnormalities is most likely to be found on the NBS report?
A.
B.
C.
D.
E.
B70.
Cystic Fibrosis
Galactosemia
Homocystinuria
Pendred Syndrome
Phenylketonuria
You evaluate a 6-day-old girl whose phenylalanine (Phe) level was increased on her Newborn Screening
(NBS) test obtained at 36 hrs of age. Which of the following factors decreases the positive predictive value
of her results?
A.
B.
C.
D.
E.
B72.
C0 acylcarnitine level is decreased
C5DC level is increased
Citrulline level is decreased
GALT level is decreased
Phenylalanine level is increased
You evaluate a 3-day-old boy with an abnormal Newborn Screening (NBS) result. He has a 2-day history
of poor feeding, emesis and diarrhea. On physical examination he is somewhat icteric and has
hepatomegaly. Which of the following disorders is most likely to be implicated by the abnormal NBS
result?
A.
B.
C.
D.
E.
B71.
Biotinidase enzyme activity
C3 acylcarnitine
C6, C8 & C8/C10
GALT enzyme activity
Succinylacetone
Early rise of Phe level
Fasting before heel stick
Increased Phe/Tyr ratio
Rapid rise of Phe level
Total Parenteral Nutrition
A new screening (NBS) test was evaluated on stored NBS blood spots from 200 individuals confirmed to
have Pompe Disease and 200 normal controls. Of these 400 samples 160 tested positive and 240 tested
negative. Which of the following is the maximum sensitivity of the new screening test for Pompe Disease?
A. 160/400 or 0.40
B. 240/400 or 0.60
C. 160/200 or 0.80
D. 40/200 or 0.20
E. 160/240 or 0.67
684
Answers to Biochemical Genetics/Newborn Screening B1-B72
B1.
The odor of sweaty feet is classically associated with isovaleric acidemia. The odor of untreated
PKU is sometimes described as having a mousy odor. The odor of untreated maple syrup urine
disease is as suggested by the name of the disease. Odor is not a prominent feature for propionic
or methymalonic.
The correct answer is A.
B2.
Urine organic acids will provide a characteric profile that allows for clinical diagnosis.
The correct answer is E
B3.
Patient A has the typical phenotype of type la GSD.
The correct answer is A
B4
Patient B has a typical phenotype of Type II GSD.
The correct answer is B
B5.
High ammonia would be a feature of all of these disorders except maple syrup urine disease.
The correct answer is B.
B6.
The clinical vignette is suspicious for heterozygous OTC deficiency. Female carriers can become
symptomatic when ill or if they receive a high protein load (such as hyperalimentation). In the past, a
protein challenge or allopurinol loading test followed by measurement of urine orotic acid was often
used to make a diagnosis. Today gene sequencing is the best method. Urine orotic acid alone or
ammonia level will not be abnormal unless the patient is in the midst of an acute crisis.
The correct answer is A
B7.
One parent must be a Gal carrier and the other a Duarte carrier. There is a possibility that the
parent who is a Duarte carrier is also a Gal carrier. Such individuals have about 25% of normal
gal activity, which would not produce clinical signs of galactosemia. The risk that the other allele
in this parent is Gal is 1/278. In this case the risk to a child would be ¼. Therefore the overall
risk is 1/278 x ¼.
The correct answer is E.
B8.
There are two reasons why this infant may have an elevated phe on newborn screening –
immaturity of the HPPD enzyme and TPN. The best way to exclude an inherited disorder in phe
metabolism would be to stop the TPN briefly (~4-6 hr) and provide calories with glucose and
recheck the phe off the TPN.
The correct answer is A
B9.
The history is fairly classic for X-linked creatine transporter deficiency which can account for up to
~1% of X-linked MR. Other features include hypotonia, seizures, and autism. Screening can be
performed by urine guanidinoacetoacetate (GAA) which is elevated or by finding a low creatine peak
on brain MRI with spectroscopy. It is not specific on plain MRI. While the disorder is technically a
defect in ornithine metabolism, ornithine levels are normal. Plasma creatine/creatinine ratio is used to
detect the 2 other disorders of creatine metabolism (AGAT and GAMT deficiency).
The correct answer is E
685
B10.
MCAD presents with hypoketotic hypoglycemia and can resemble Reye syndrome or SIDs.
Although initially it was thought that a higher percentage of cases of SIDS were likely due to
defects in fatty acid oxidation, prospective and retrospective studies now demonstrate that this
accounts for ~5% of SIDs. While OTC and propionic acidemia patients can occasionally die
unexpectedly, there are usually some warning signs – vomiting, lethargy, irritability. GA I
presents with MR, a movement disorder, and macrocephaly. GA II could be a rare cause of
sudden death.
The correct answer is C.
B11.
Depending on how rapidly one could obtain results, the BEST answer might be serum HexA on
husband (+/- molecular) and WBC hexA on the wife. The major points of this question are (1)
serum hexA is not accurate during pregnancy and one needs to do WBC hexA determinations and
(2) molecular testing is not helpful in the non-Ashkenazi Jewish population since most mutations
will not be detected.
The correct answer is D.
B12.
The clinical features here are strongly suggestive of X-linked adrenoleukodystrophy. Although
biochemically this disorder is characterized by accumulation of VLCFA, the gene defect is NOT
an enzyme involved in fatty acid metabolism. Rather, it is a peroxisomal membrane protein which
may be involved in transport of the enzyme.
The correct answer is D.
B13.
The elevated tyr + succinylacetone is diagnostic for Type I tyrosinemia. Current standard treatment is
NTBC + low tyr diet (tyrex formula). The advent of newborn screening and NTBC has resulted in a
markedly reduced need for liver transplantation.
The correct answer is B
B14.
Rhizomelic chondrodysplasia patients have normal VLCFA and normal numbers of peroxisomes.
Most patients have elevated phytanic acid levels from impaired phytanic acid oxidase acitivity.
The correct answer is A.
B15.
Arginase deficiency does not present with crises in the newborn period. It presents typically with
spastic paraplegia and MR although later episodes of mild hyperammonemia can occur.
The correct answer is A
B16.
Pycnodysostosis is actually a lysosomal storage disease caused by Cathepsin K deficiency.
The correct answer is B.
B17.
While gene sequencing would be a definitive test, screening can be performed by urine organic acids
which will detect elevated N-acetylaspartic acid (NAA). An elevated NAA peak could also be seen on
MRI spectroscopy.
The correct answer is D.
B18.
AIP is an autosomal dominant enzyme deficiency. Most patients are asymptomatic.
The correct answer is B
B19.
Pseudodeficiency, with 5-15% of normal enzyme activity, is found in ~2% of European
Caucasian alleles. It results from 2 single base “polymorphisms” in the Arylsulfatase A gene and
can complicate carrier testing and prenatal diagnosis. For families where this is found, molecular
testing or use of sulfatide loading as a natural substrate is the preferred assay. The slightly low
level of activity in the mother would be typical for an obligate carrier.
The correct answer is B.
686
B20.
The glucose-6-phosphatase enzyme is expressed only in liver. Prior to isolation of the gene,
prenatal diagnosis was not possible. Translocase deficiency causes GSD Ib.
The correct answer is C.
B21.
Since this involves a defect in N-linked glycosylation, serum transferrin isoelectric focusing should
produce an abnormal pattern. This would be the most efficient screening test. Cholesterol and sterol
patterns may be normal; insufficient information is provided concerning their utility. Urine
oligosaccharide chromatography is used for lysosomal storage disorders such as mannosidosis and
fucosidosis.
The correct answer is D.
B22.
Expect some simple questions about basic metabolic defects in the general exam. The defect in
Fabry disease was referred to as Į-galactosidase A in the past.
The correct answer is A.
B23.
Pipecolic acidemia and plasmalogen deficiency can occur in various peroxisomal disorders.
Cerebrosidase E-galactosidase deficiency is the defect in Krabbe disease. Dicarboxylic aciduria is
characteristic of some of the defects of fatty acid oxidation. The hallmark metabolic abnormality
in adrenoleukodystrophy is accumulation of very long chain fatty acids, although this abnormality
also occurs in other peroxisomal disorders.
The correct answer is E.
B24.
Some autosomal recessive disorders with substantial risk of malignancy are glycogen storage disease
type I, tyrosinemia, hemochromatosis, and others less prominently. Immunodeficiency disorders and
DNA repair disorders also relevant.
The correct answer is E.
B25.
The features are classic for Kearn-Sayre syndrome which involves heteroplasmic deletions or
duplications of portions of the mitochondrial genome.
The correct answer is B.
B26.
Chromosome analysis and plasma carnitine levels would not be helpful in any specific way to
make the diagnosis of Zellweger syndrome. Skeletal X-rays might detect stippling and CT scan of
the brain might show abnormalities of gyral formation. However, plasma very long chain fatty
acid analysis would be by far the best choice In this test and would be abnormal in the vast
majority of cases.
The correct answer is D.
B27.
The clinical features are classic for a urea cycle disorder. Levels of citrulline on amino acids are key
to deciding which disorder is present. Undetectable citrulline is found in males with OTC and CPS
deficiencies. In OTC, carbamyl phosphate is shunted to pyrimidine synthesis resulting in high orotic
acid. In CPS deficiency, no carbamyl phosphate is made and so there is no elevation in orotic acid.
(see pathway)
The correct answer is D.
B28.
Tetrahydrobiopterin is a cofactor for phenylalanine hydroxylase (along with oxygen) that converts
phe to tyr ~2% of infants with inherited hnyperphenylalaninemia have a defect in the synthesis or
recycling of the biopterin cofactor. All infants with confirmed hyperphe should be tested for a
biopterin defect by blood and urine pterin screening.
The correct answer is A.
687
B29.
Hepatorenal tyrosinemia is also called tyrosinemia type I and results from a defect in last step of
tyrosine degradation fumarylacetoacetate hydrolase (FAH).
The correct answer is C.
B30.
In the past several years, it has been recognized that most female Fabry carriers develop symptoms.
Proteinuria develops in late childhood although only ~10% develop frank renal failure. Most affected
males (disorder is X-linked) develop renal failure if untreated, with onset of symptoms around 30
years of age. Fabry accounts for ~5% cryptogenic strokes and can be seen. Alport is often X-linked
and has renal disease (nephritis) and hearing loss, but not strokes. MELAS has strokes but not this
pattern of renal disease. In Von Hippel Lindau, vascular malformatons are present in multiple tissues
and there can also be renal cysts.
The correct answer is B.
B31.
Gyrate atrophy of the retina is caused by deficiency of ornithine aminotransferase (OAT). It is
diagnosed by finding elevated ornithine on plasma amino acids.
The correct answer is C.
B32.
Metronidazole is an antibiotic used against anaerobic bacteria and protozoa. Some gut flora can
generate propionate (which can be converted to MMA). Thus, some doctors advocate using
medtronidazole to reduce gut flora and propionate production in PA or MMA.
The correct answer is A.
B33.
These are classic findings in nonketotic hyperglycinemia, particularly the seizures and hiccups,
which may occur prenatally.
The correct answer is E.
B34.
This is likely a transient B12 deficiency in the patient and her sister secondary to B12 deficiency in the
mother. The fact that the sisters have different fathers makes a defect in cobalamin metabolism itself in
the patient or her sister much less likely. While transcobalamin II deficiency can produce
megaloblastic anemia in infants (often with FTT and other sx), this is very rare and is also an
autosomal recessive disorder. The reason for B12 deficiency in the mother is often undiagnosed
pernicious anemia, but it can also occur in strict vegans.
The correct answer is B.
B35.
The correct answer is E.
B36.
The correct answer is B.
B37.
Intellectual function is usually normal in MPSVI. It is also preserved in MPS IVA (Morquio) and
the Scheie variant of Hurler (MPS IS).
The correct answer is C.
B38.
The features are suggestive of a mucopolysaccharidosis or storage disorder with significant somatic
involvement. Elevation of multiple lysosomal enzymes occurs in I-cell disease (ML II) and pseudoHurler polydystrophy (MLIII) because the enzymes are not targeted to the lysosome properly and are,
instead, secreted. The two disorders are allelic with I-cell being more severe and presenting earlier.
The defect is in a sugar phosphotransferase found in the Golgi that properly targets enzymes to the
lysosome.
The correct answer is B.
688
B39.
21-Hydroxylase deficiency causes ~95% of cases of CAH. Aromatase deficiency causes inability
to synthesize estrogen and affected females have ambiguous genetalia and primary amenorrhea.
5-Į-Reductase deficiency converts testosterone into the more potent dihydrotestosterone.
Affected males can have pseudovaginal perineoscrotal hypospadias Androgen receptor
deficiency causes androgen insensitivity and feminization of affected males.
The correct answer is E.
B40.
The correct answer is C. Thromboembolic events cause the death of 50% of individuals
affected by Homocystinuria by 20 yrs.
B41.
The correct answer is C. Increased C6, C8 and C8/ C10 ratio is the profile for MCAD and
these are derived from medium chain fatty acids. Increased C14-OH, C16-OH, C18-OH and
C18:1-OH is the profile for LCHAD Deficiency. Increased C14:1 and C14:1/ C12:1 ratio is the
profile for VLCAD deficiency. Increased C3 and C4 These are seen in Propionic academia and
SCAD deficiency, respectively.
B42.
The peak for phe is markedly elevated.
The correct answer is B
B43.
Cerebral folate deficiency is an autosomal recessive disorder due to mutations in a folate receptor,
FOLR1. It results from brain-specific folate deficiency, and, if recognized, can be treated with high
dose oral folinic acid. This often leads to a dramatic decrease in seizures and marked improvement in
tone and development. There is no good treatment for the other disorders although some therapies are
available (Menkes, NKH). Menkes is also X-linked, and these symptoms would not be expected in a
female carrier.
The correct answer is B.
B44.
Both A and B show a decrease in phe levels following administraton of BH4. The other two
graphs do not (C,D).
The correct answer is A
B45.
Sam’s GPUT (GALT) enzyme level should clarify if he has classical galactosemia which is
associated with both an elevated Gal-1-P and E Coli sepsis. A. and E.— Since Sam may have
been on a lactose free formula for several weeks his blood galactose and urinary carbohydrate
level by Clinitest are likely to be normal and will not clarify if he has classical galactosemia. C.
and D.— Determining if he has Duarte (N1314D and -119GTCA deletion) or Gal (Q188R)
genotypes alone would only clarify dietary recommentations for Sam if he should he be found to
be homozygous for the Q188R mutation.
The correct answer is B.
B46.
Sarah’s Phenylalanine level of 1220 μmol/L strongly suggests that she has Phenylketonuria
(PKU). You should obtain a confirmatory test (plasma amino acid levels of Phe & Tyr), exclude
BH4 disorders (urine pterins) and start a Phe free formula while waiting for the test results. A.
Starting a Phenylalanine free formula doesn’t exclude BH4 disorders. B. or C. Similarly starting
her on large neutral amino acids or PEGylated Phenylalanine Ammonium Lyase rather than a Phe
free formula are also not standard care. E. Starting her on BH4 (Sapropterin) at 20 mgm/kg/day
rather than a Phe free formula is not standard care.
The correct answer is D.
689
B47.
C6 and C8 levels at 1.2 times the normal cutoffs suggests that the baby may have MCAD
Deficiency which can cause lethal hypoketotic hypoglycemia following fasting, gastroenteritis or
poor feeding. This baby should be evaluated as soon as possible, the NBS repeated, the parents be
given instructions for what to do if the baby has poor feeding or lethargy, and diagnostic tests for
MCAD considered. A. Increased biotinidase enzyme levels do not indicate biotinidase deficiency.
C. Increased GALT enzyme levels do not indicate galactosemia. D. Decreased Immunoreactive
Trypsinogen levels do not indicate cystic fibrosis. E. A mildly increased Phe level should be
repeated but PKU is not a life-threatening disorder like MCAD.
The correct answer is B.
B48.
The correct answer is C.
Keywords: fetal hydrops, inborn error of metabolism
Several lysosomal storage disorders have severe forms with prenatal onset, including sialidase
deficiency.
B49.
The correct answer is A.
Keywords: propionic acidemia, carglumic acid
Carglumic acid is a synthetic analog of N-acetylglutamate (NAG), an obligate cofactor
for carbamyl phosphate synthetase I (CPSI). In organic acidemias, NAG synthase or
CPSI reactions are compromised, probably by the organic intermediates that build up
during acute illnesses. While designed to treat NAG synthase deficiency, carglumic acid
has been found to be effective in reducing hyperammonemia in organic acid disorders,
such as PA, MMA, IVA, by increasing CPSI activity acting as a cofactor.
B50.
The correct answer is E.
Keywords: cirrhosis, porphyria cutanea tarda, porphyrins
Different porphyrias have different sites and patterns of porphyrin metabolite and/or product
accumulation. All of the erythropoietic cutaneous porphyrias have some accumulation in RBCs.
The only hepatic porphyria with RBC accumulation is ADP (ALA-dehydratase-deficient
porphyria) which is extremely rare. Other hepatic porphyrias accumulate metabolites in urine or
urine + stool. AIP, the most common has early metabolites (ALA, PBG) in urine only (answer C).
PCT is occasionally an inherited or, more often, acquired deficiency of URO-decarboxylase and
accumulation of uroporphyrin is found in urine. The presence of increased stool
isocoproporphyrin is diagnostic.
B51.
The correct answer is E.
Keywords: lactate, Leigh syndrome
Most mitochondrial disease in infants affect nuclear genes. Leigh syndrome has multiple
etiologies including nuclear genes in complexes IV or I and nuclear SURF1 mutations that
encode a protein involved in the assembly of Complex IV. These variants of Leigh syndrome are
typically inherited in an autosomal recessive fashion.
B52.
The correct answer is A.
Keywords: hexosaminidase A, cherry red spot
Answer: A. Hex A is missing in Tay-Sachs disease. The classic eye finding is a cherry red spot
which is present by the time neurologic symptoms begin. The macula is normal and the cherry
red spot is due to lipid accumulation in the surrounding neural retina.
690
B53.
The correct answer is E.
Keywords: Leigh syndrome, lactate
X-OLQNHG0XWDWLRQVLQWKH(Įsubunit are the most common. This gene is on the X
chromosome and severe cases are usually lethal in hemizygous males.
B54.
The correct answer is B.
CDG Type Ib is the only type currently known with a specific treatment. The defect is
mannose-6-phosphate isomerase.
B55.
The correct answer is A.
Keywords: Hunter syndrome, coarse facies, hepatomegaly
MPSII is X-linked and found mostly in males. Features include coarse facies,
hepatomegaly and dysostosis multiplex like Hurler syndrome, but there is hearing loss
and usually no corneal clouding (may occasionally see by slit lamp but not clinically
significant.)
B56.
The correct answer is A.
Keywords: Pompe disease, hearing loss
Many patients have hearing loss which was not appreciated previously because of their
early death. Osteopenia is also found. Cardiac disease usually improves quickly. Motor
skills improve but residual weakness is typical. >90% of patients develop IgG antibodies
but this does not cause inhibitory activity against the recombinant enzyme.
B57.
The correct answer is D.
Keywords: Smith-Lemli-Opitz, prenatal
Cholesterol is a precursor for the synthesis of steroid hormones and many fetuses affected
with SLOS are associated with low maternal serum estriol.
B58.
The correct answer is A.
This should be an organic acid disorder because of acidosis, type and timing of
presentation, and high ammonia. Option A is consistent with propionic academia. Option
B would be consistent with OTC and one would not expect heterozygous females to
present in neonatal period. Option C would be typical of urea cycle and expect normal
pH or mild alkalosis early in course. Option D would be consistent with nonketotic
hyperglycinemia which has early seizures and normal routine labs including chemistries
and ammonia. Option E is diagnostic for MCAD, which almost never presents in first
week of life.
B59.
The correct answer is B.
The macrocephaly and movement disorder with relatively intact development is typical
of treated GAI.
691
B60.
The correct answer is E.
Early matched umbilical cord blood or matched bone marrow transplant has best
outcome.
B61.
The correct answer is B.
Late onset CPTII deficiency typically presents as intermittent rhabomyolysis with
strenuous exercise. Option A - CPTI presents with liver disease. Option C - LCHAD has
liver, cardiac and acute muscle symptoms, usually presenting earlier. Option D - MCAD
does not have muscle involvement. Option E - SCAD is usually not disease-associated
and if pathologic does not have skeletal muscle involvement.
B62.
NEWBORN SCREENING
The correct answer is C.
Source: Newborn Screening Slides 7-10
Key Words: Sensitivity, specificity, positive predictive value
Explanation: Positive Predictive Value (PPV) is the fraction of those who have a +
screen results who are affected = True Positive/ (True Positive +False Positive). If the
PPV= 0.10, then 10% of those with a + screening result will be affected.
B63.
The correct answer is E.
Source: Newborn Screening Slides 7-8
Key Words: Screening, sensitivity
Explanation: Sensitivity is the fraction of affecteds who screen positive = True Positive/
(True Positive +False Negative). In this case there were 10 TPs (affecteds who screen +)
and none of those who were affected screened negative. Since none of the remaining 90
who screened + were affected, there were no FNs. Thus sensitivity = 10/ (10+0) = 10/10.
B64.
The correct answer is B.
Source: Newborn Screening Slides 52 and 61-63
Keywords: newborn hearing screening, 17OHP
Explanation: Of the choices only Congenital Adrenal Hyperplasia (CAH) causes an
elevation in 17OHP. CAH is potentially life threatening and an elevated 17OHP on NBS
is an emergency. ACTH is elevated rather than low in CAH and low ACTH doesn’t cause
an elevated 17OHP. Pended Syndrome does cause deafness in newborns but does not
cause either an elevated 17OHP or congenital hypothyroidism. It does cause euthyroid
goiter in adolescence.
B65.
The correct answer is C.
Source: Newborn Screening Slides 42-43 and 49
Keywords: acidosis, ketonuria, hyperammonemia, neutropenia, thrombocytopenia, C5
Explanation: Isovaleric Acidemia presents with acidosis, ketonuria, hyperammonemia,
neutropenia, thrombocytopenia, an elevated C5 and the odor of sweaty feet. Biotinidase
Deficiency presents with alopecia, rash and elevated C5-OH. Holocarboxylase Deficiency
presents with elevated C5-OH. Methylmalonic Acidemia and Propionic Acidemia present with
elevated C3.
692
B66.
The correct answer is B.
Source: Newborn Screening Slides 39-41
Keywords: MCADD, NBS
Explanation: Children with MCADD who are fasted or have poor po intake are prone to develop
hypoketotic hypoglycemia. Before NBS for MCADD the first crisis was fatal in 25% of affected
children. The other answers are not typical for MCADD.
B67.
The correct answer is C.
Positive predictive value is the fraction with a positive screen that are affected. Since the PPV =
TP/ (TP+FP) both increasing the TP and reducing the FP will increase the PPV. A. False
negative rate (FNR) is the fraction of affecteds that test negative. Since the FNR = FN/ (FN+TP)
increasing TP will decrease the FNR. B. False positive rate (FPR) is the fraction of unaffected
that test positive. Since the FPR = FP/ (FP+TN) decreasing the FP will decrease the FPR. D.
Sensitivity of the test is the fraction of affecteds who screen positive. Since Sensitivity =
TP/ (TP+FN) increasing TP can increase the sensitivity but reducing FP has no additional effect
as it does on the PPV. E. Specificity of the test is the fraction of unaffected who screen negative.
Specificity = TN/ (TN+FP) reducing the FP can increase the sensitivity but increasing the TP has
no additional effect as it does on the PPV.
B68.
The correct answer is B.
This neonate has hypoglycemia, ketoacidosis and hyperammonemia. These are all consistent with
an organic acidemia. C3 acylcarnitine is elevated in methylmalonic acidemia and propionic
acidemia. A. Deficiencies rather than increases in Biotinidase enzyme activity cause disease. C.
Increases in these analytes are seen in MCAD deficiency which is usually not associated with
ketoacidosis. D. Deficiencies rather than increases in GALT enzyme activity cause galactosemia.
E. Increases in Succinylacetone are seen in tyrosinemia type 1 which usually doesn’t present this
early.
B69.
The correct answer is D.
This neonate has emesis, diarrhea, icterus and hepatomegaly. The early onset and all of these
signs and symptoms are seen in galactosemia which is caused by GALT deficiency. A. C0
acylcarnitine level is decreased in carnitine transporter deficiency that can have hepatomegaly
due to cardiac failure but usually not emesis and diarrhea. B. C5DC level is increased in GA1
which presents with macrocephaly and hypotonia. C. Citrulline level is decreased in OTC which
is X linked and doesn’t present with above. E. Phenylalanine level is increased in PKU which
doesn’t present with above.
B70.
The correct answer is B.
This neonate has poor feeding, emesis, diarrhea, icterus and hepatomegaly. The early onset and
all of these signs and symptoms are seen in galactosemia. A. Cystic Fibrosis can present with
diarrhea but not usually with poor feeding, emesis or hepatomegaly. C. Homocystinuria is
usually asymptomatic in neonates. D. Pendred Syndrome can present with hearing loss but not
usually with poor feeding, emesis or hepatomegaly. E. Phenylketonuria is usually asymptomatic
in neonates.
B71
The correct answer is E.
TPN can cause Phe elevations which represent false positives that decrease the PPV=TP/
(TP+FP). A. Early rise of Phe level is seen in PKU and it increases TP and the PPV. B. Fasting
before heel stick decreases FP and increases the PPV. C. Increased Phe/Tyr ratio increases TP
and the PPV. D. Rapid rise of Phe level is seen in PKU and it increases TP and the PPV.
693
B72.
The correct answer is C.
Source: Lecutre/Slides
Keywords: Sensitivity, True positive, False negative
Explanation: Sensitivity is the ratio of True positive/True positive + False negative results.
There were 200 confirmed cases and 200 normal control samples. If all 160 positives were from
the 200 confirmed cases then the maximum sensivity was 160/200 or 0.80.
A. 160/400 or 0.40 is the ratio of Positives/True positives + False negatives + True Negatives
B. 240/400 or 0.60 is the ratio of False negatives + True negatives / True positives +False
negatives + True Negatives
C. 160/200 or 0.80 is the maximum ratio of True positives / True positives + False negatives
D. 40/200 or 0.20 is the ratio False negatives / True positives + False negatives
E. 160/240 or 0.67 is the ratio of Positives / Negatives
694
C. Clinical Genetics Questions C1-C98
C1.
A concerned mother brings her 18-month-old daughter to the pediatrician's office because she has
been intermittently constipated. Her mother became alarmed while changing her diaper after
straining to have a stool, she found some red, tissue bulging from the rectum. Which of the
following procedures or laboratory studies is most like to identify the appropriate diagnosis?
A.
B.
C.
D.
E.
C2.
Guanine nucleotide binding proteins play regulatory role in many signal transduction pathways
within the cell. Abnormalities of guanine nucleotide binding protein have been associated with
several clinical disorders. Which of the following disorders results from inactivating mutations of
the guanine nucleotide binding protein Gs?
A.
B.
C.
D.
E.
C3.
Albright hereditary osteodystrophy
insulin unresponsiveness
McCune-Albright syndrome
nephrogenic diabetes insipidus
type II diabetes mellitus
An infant dies at ten days of age, and autopsy reveals clinical features that include: agyria,
cerebellar hypoplasia, Dandy-Walker cyst, microphthalmia, and retinal detachment with retinal
dysplasia. Which of the following syndromes if the most likely diagnosis?
A.
B.
C.
D.
E.
C4.
Colonoscopy for Crohn disease.
Molecular analysis for Duchenne muscular dystrophy
Rectal biopsy Hirschsprung disease
Renal ultrasound for Beckwith-Wiedemann syndrome
Sweat chloride test for cystic fibrosis
Meckel-Gruber syndrome
Miller-Dieker syndrome
Neu-Laxova syndrome
Pallister-Hall syndrome
Warburg syndrome
Prader-Willi is a genetic syndrome characterized by failure to thrive during the first year of life,
developmental delay in most patients, small hands and feet (more noticeable in late
childhood), crytorchidism in males, a facial gestalt that is recognizable in many patients and foodseeking behaviour after age two. Even so clinical diagnosis can be difficult because of the
variability in the phenotype and laboratory studies have become the mainstay of diagnosis.
Which of the following laboratory analyses is most likely to reveal a positive finding in a young
child where the constellation of clinical features above is not readily identifiable?
A.
B.
C.
D.
E.
FISH analysis for missing SNRPN locus
High-resolution chromosome analysis
Imprinting assessment with DNA methylation assay
Routine chromosome analysis and karyotype
Subtelomeric probe analysis by FISH studies
695
C5.
A newborn infant has the Robin sequence. The mother was diagnosed with arthritis in childhood
and has had a retinal detachment. Which of the following is the most likely syndromic diagnosis?
A.
B.
C.
D.
E.
C6.
Radial ray anomalies distinguished by the presence or absence of thumbs can provide a clue
toward an appropriate diagnosis. In which of the following syndromes are thumbs consistently
present?
A.
B.
C.
D.
E.
C7.
Thrombocytopenia and absent radius syndrome
Fanconi anemia
Holt-Oram syndrome
VATER association
de Lange syndrome
Soft cystic masses in the auricle which develop into hypertrophic cartilage are a typical feature of
which of the following skeletal dysplasias?
A.
B.
C.
D.
E.
C8.
Acromesomelic dysplasia
Rieger syndrome
Stickler syndrome
Treacher Collins syndrome
Warburg syndrome
Achondroplasia
Camptomelic dysplasia
Chondroectrodermal dysplasia
Diastrophic dwarfism
Larsen syndrome
A 28-year-old woman underwent amniocentesis for chromosomal analysis when her fetus was
found to have limbs that were short for the assigned dates. The karyotype was 46,XY. At birth
the baby had female appearing genitalia. These clinical findings are most consistent with which
of the following syndromes ?
A.
B.
C.
D.
E.
Achondrogenesis type IA
Achondrogenesis type II
Campomelic dysplasia
Jeune thoracic dystrophy
Thanatophoric dysplasia
696
C9.
A 1-year-old boy presents with an episode of fever on a 90 degree day in July and there is no
evidence of an infection. His parents note that he never seems to sweat in the extreme heat of the
summer. He has extremely sparse hair. Skull x-rays show absence of most of the primary tooth
buds. Which of the following statements reflects the most likely inheritance pattern for this
disorder?
A.
B.
C.
D.
E.
C10.
A newborn infant has extreme hypotelorism, microcephaly, midline cleft, poor temperature
regulation, seizures, and no anomalies below the neck. Which of the following syndromes is best
described by these findings?
A.
B.
C.
D.
E.
C11.
Ellis van Creveld syndrome
Holoprosencephaly
Meckel-Gruber syndrome
Trisomy 13
Trisomy 18
A newborn infant has cleft lip and palate, postaxial polydactyly, microphthalmos, and congenital
heart disease. Which of the following syndromes is best described by these findings?
A.
B.
C.
D.
E.
C12.
He likely has an autosomal dominant disorder.
He likely has an autosomal recessive disorder.
He likely has a sporadically occurring disorder.
He likely has an X-linked dominant disorder.
He likely has an X-linked recessive disorder.
Ellis van Creveld syndrome
Holoprosencephaly
Meckel-Gruber syndrome
Trisomy 13
Trisomy 18
Haploinsufficiency is the mechanism underlying the genotype/phenotype relationship for which
of the following disorders?
A.
B.
C.
D.
E.
achondroplasia
acute intermittent porphyria
Huntington disease
multiple endocrine neoplasia, type 2A
transthyretin amyloidosis
697
C13.
Reproductive fitness is a measure of an individual’s likelihood of being able to have offspring
where a fitness level of 1 is equivalent to the normal population and a fitness of 0 represents an
inability to reproduce. In the relationships outlined below the disorders are abbreviated as
follows: Huntington disease (HD), tuberous sclerosis (TS), neurofibromatosis type 1 (NF), and
osteogenesis imperfecta type II (OI). Which of the following relationships best describes the
reproductive fitness of these disorders from greatest to least?
A.
B.
C.
D.
E.
C14.
A six-year-old girl presents to clinic with a history of cleft palate, tetralogy of Fallot, height at the
3rd centile, and mental retardation. Which of the following syndromes is the most likely
diagnosis?
A.
B.
C.
D.
E.
C15.
Cardiofaciocutaneous syndrome
Down syndrome
Otopalatodigital syndrome
Stickler syndrome
Velocardiofacial syndrome
A 7-year-old boy with cleft palate, height at the 3rd centile, myopia, and a family history of retinal
detachment presents to orthopedic clinic with hip pain and a referring diagnosis of Legg-Perthes
disease. Which of the following sysndromes is most consistent with this patient’s clinical
presentation?
A.
B.
C.
D.
E.
C16.
HD>NF>TS>OI
HD>TS>NF>OI
NF>HD>OI>TS
NF>OI >HD>TS
TS >NF>OI>HD
Coincidental Legg-Perthes disease in a child with nonsyndromic cleft palate
Otopalatodigital syndrome
Stickler syndrome
Struger’s multiple epiphyseal dysplasia with cleft palate syndrome
Velocardiofacial syndrome
A newborn baby is found to have hydrocephalus, spina bifida, and clubbed feet. Which of the
follwoing genetic etiologies provides the best description of this baby’s clinical features?
A.
B.
C.
D.
E.
A multiple congenital anomalies syndrome
A single primary malformation
A probable teratogenic syndrome
A probable chromosome abnormality
An association
]
698
C17.
You are evaluating a newborn with bilateral absence of the radii, with intact thumbs. Which of the
following clinical findings is most likley to be associated with this baby’s presentation?
A.
B.
C.
D.
E.
C18.
A child with hypoplastic left heart syndrome is being evaluated prior to consideration of a heart
transplant. Which of the following karyotypes is most likely to be found on chromosomal analysis?
A.
B.
C.
D.
E.
C19.
Aarskog
Noonan
Robinow
Pfeiffer
Velocardiofacial
A man with a right-sided triphalangeal thumb and left sided absent thumb has a child with bilateral
absence of the radius and an atrioventriculoseptal defect. Which of the following genetic concepts
accounts for this clinical history?
A.
B.
C.
D.
E.
C21.
45,X
46,XX
46,XX,del(22)(q11.2q11.2)
46,XY.ish del(7)(q11.23q11.23)(ELN-)
47,XXY
You are asked to evaluate a 6-year-old boy with mild developmental delay and a distinctive
dysmorphic facial appearance. In taking the history, you are told by his mother that his father has
the exact same appearance. Which of the following syndromes are you most likely to remove from
your differential diagnosis based on this clinical history?
A.
B.
C.
D.
E.
C20.
Cytogenetic testing would show an extra copy of an E group chromosome.
Hypoplasia of the mammary tissue is a common associated finding.
Thrombocytopenia is a common complication.
Tracheo-esophageal fistula is a common associated finding.
When DEB is added to the media, increased chromosomal breakage is seen.
Genetic heterogeneity
Multifactorial inheritance
Phenotypic pleiotropy
Variable expression with complete penetrance
Variable penetrance with incomplete expressivity
A newborn boy manifests macrosomia (birth weight 5.2 Kg), coarse facies, and post-axial
polydactyly. Which of the following clinical findings are you most likely to encounter when
caring for this child?
A.
B.
C.
D.
E.
Cardiac conduction defects
Delayed bone age
Father to son transmission
Late onset retinitis pigmentosa
Uniparental disomy for chromosome 11p.
699
C22.
A four-year-old girl is found to have relative macrocephaly, a broad forehead, a large open
anterior fontanelle, and Wormian bones on X-ray. Her parents were told that she suffered bilateral
clavicular fractures at birth, and they have never healed. Her father has a broad forehead, wide set
eyes, and a history of dental problems, including supernumerary and ectopic teeth. Which of the
following clinical findings has been associated with the underlying disorder?
A.
B.
C.
D.
E.
C23.
Cytogenetic testing is needed to confirm the diagnosis.
Most affected individuals have an adult height under 5 feet.
Retinal detachment is a long-term risk.
There is a lifetime increase risk for fractures.
There is an increase rate of Caesarian section for affected women.
A 3-year-old girl has hypodontia, alopecia, and mild developmental delay. Examination reveals
hyperpigmented hyperkeratotic streaks. Her sister has severe developmental delay with seizures.
Her mother has partial adontia and atrophic scalp hair. Which of the following clinical findings is
most commonly associated with this disorder?
A.
B.
C.
D.
E.
Affected females can only have unaffected males or affected females.
Chromosome analysis of fibroblasts reveal an abnormality in 90% of cases.
Germline mosaicism is a common etiology.
One sees fewer than expected males in affected families.
67% of cases represent new mutations
C24.
A 3-year-old boy manifests short stature, “Hitchhiker” thumbs, and “cauliflower” ears. What is the
recurrence risk for his unaffected parents of having a subsequent child with the same condition?
A.
<1%
B.
5-7%
C.
12.5%
D.
25%
E.
50%
C25.
Which of the following genes is most likely to be mutated in an individual with a
hypercoagulable state?
A.
B.
C.
D.
E.
C26.
Which of the follwoing symptoms is the most common presenting complaint in individuals with
hemochromatosis?
A.
B.
C.
D.
E.
C27.
Factor VIII
Factor IX
Prothrombin
Thrombin
von Willebrand factor
joint pain
fatigue
change in skin pigmentation
nausea and vomiting
decreased libido
Hereditary angioedema is due to deficiency in which of the following systems?
700
A.
B.
C.
D.
E.
C28.
Which of the following is responsible for a phase I reaction in drug metabolism?
A.
B.
C.
D.
E.
C29.
Allergy
Arrhythmia
Hair loss
Hepatitis
Seizure
You are asked to evaluate a well infant girl with apparently isolated Pierre Robin sequence.
Which of the following diagnostic tests is most likely to yield a useful clinical finding?
A.
B.
C.
D.
E.
C33.
Asthma
Chronic myelogenous leukemia
Coronary artery disease
Diabetes
Hypertension
A sodium channel polymorphism can be associated with which of the following side effects of
medication?
A.
B.
C.
D.
E.
C32.
Hypertension
Asthma
Tuberculosis
Leukemia
Diabetes
Imatinib is used in the treatment of which of the following disorders?
A.
B.
C.
D.
E.
C31.
ABC transporter
Acetylase
Glucuronyl transferase
P450 enzyme
Transcarbamylase
The thiopurine methyltransferase polymorphism is important in treatment of which of the
following disorders?
A.
B.
C.
D.
E.
C30.
Antibodies
B-cells
Complement
Macrophages
T-cells
Chromosome analysis
Electrocardiogram (ECG)
Eye exam
FISH for del22q11.2
Skeletal survey
Chromosome analysis would be most helpful in diagnosis of which of the following syndromes?
701
A.
B.
C.
D.
E.
C34.
Which of the following would be most useful in differentiating non-accidental injury (NAI) from
osteogenesis imperfecta in a 5-month-old girl with unexplained fractures?
A.
B.
C.
D.
E.
C35.
Blue sclerae
Healing fractures of different ages
Low socioeconomic status of the parents
Retinal hemorrhages
Type I collagen analysis
A 3 year old girl has severe developmental delay, microcephaly, no speech, grand mal seizures,
and ataxic limb movements. Chromosome analysis and FISH testing for a 15q11 deletion are
normal. Which of the following genetic tests would be most appropriate to order in trying to
determine the underlying etiology?
A.
B.
C.
D.
E.
C36.
Marfan syndrome
Neurofibromatosis 1
Smith-Magenis syndrome
Pfeiffer syndrome
Williams syndrome
Cholesterol/7-delhydrocholesterol ratio
FMR1 analysis
MECP2 sequence analysis
Methylation studies of chromosome 11p15
Uniparental disomy for chromosome 7
Assuming complete penetrance for a point mutation, this pedigree is most consistent with
A. autosomal dominant inheritance of a gene imprinted (not
expressed) for the maternal allele
B. autosomal dominant inheritance of a gene imprinted (not
expressed) for the paternal allele
C. mitochondrial inheritance
D. X-linked dominant inheritance
E. X-linked recessive inheritance
C37.
Which of the following is associated with increased sensitivity to treatment of small cell lung
cancers with gefitinb?
A.
B.
C.
D.
E.
C38.
BCR-ABL translocation
EGF receptor mutation
N-myc amplification
RAS mutation
RET mutation
Which of the following hematologic disorders is commonly associated with a gene inversion?
702
A.
B.
C.
D.
E.
C39.
Which of the following immune deficiency disorders is associated with thrombocytopenia?
A.
B.
C.
D.
E.
C40.
Hemophilia A
Hemophilia B
Protein S deficiency
Thalassemia
Von Willebrand disease
Adenosine deaminase deficiency
Ataxia-telangiectasia
Severe combined immune deficiency
Velocardiofacial syndrome
Wiscott-Aldrich syndrome
Which of the following mutations is the most common mutation associated with
hemochromatosis?
A.
B.
C.
D.
E.
C282Y
H63D
I105T
Q283P
R330M
C41 – C43: For each consultand described below, choose the chance that the consultand might have a
child with a neural tube defect (in the US) from the percentages listed in A-E. You may choose an
answer once, more than once, or not at all.
Percentages
A.
< 0.1%
B.
0.1% to 1.9%
C.
2% to 4%
D.
5% to 10%
E.
>10%
C41.
A man with asymptomatic L-5 spina bifida occulta discovered on X-ray following a car accident
C42.
A woman whose maternal aunt's daughter had a baby with anencephaly
C43.
A normal couple whose first two children have isolated spina bifida
703
C44 – C46: For each situation described below, choose the coping response that best describes the
consultands behavior, from those listed in A-E. You may choose an answer once, more than once, or not
at all.
A.
B.
C.
D.
E.
Anger
Denial
Intellectualization
Projection
Reaction formation
C44.
A father who is preoccupied with learning all about the embryology of his newborn son's cleft lip
C45.
A mother who insists that her 4 year old daughter with spina bifida is not walking because of
her many hospitalizations
C46.
A mother who ascribes her son's fetal alcohol syndrome to her obstetrician's negligence
C47.
Which of the following proteins represents the defective protein responsible for familial
hypercholesterolemia?
A.
B.
C.
D.
E.
C48.
ApoB-100
ApoE
ApoB-48
Lipoprotein lipase
LDL receptor
Which of the following skeletal disorders has been associated with defective Wnt signaling?
A.
B.
C.
D.
E.
Osteogenesis imperfecta
Osteopetrosis
Pseudohypoparathyroidism
Achondroplasia
Osteoporosis
DEVELOPMENTAL GENETICS
C49.
During development, specification and determination involve the stepwise acquisition of a stable
cellular phenotype of gene expression specific to the particular fate of each cell, and regulation of
gene expression depends on epigenetic changes. Which of the following is NOT an epigenetic
change?
A.
B.
C.
D.
E.
Transcription complex stabilization
Transposon insertion
Modification of histones in chromatin
DNA methylation
Alternative promoter usage
704
C50.
You are asked to consult on a newborn girl in the NICU. She was born with shortened fingers on
her right hand with syndactyly of the 3rd and 4th right fingers. On examination, you find an
absence of the right pectoral muscle. Which of the following mechanism is the most likely cause
of these anomalies?
A.
B.
C.
D.
E.
C51.
A 4-month-old boy presented with new onset seizures. His physician ordered an MRI,
and he is found to have agyria. The patient is referred to you. On examination, the child
has no dysmorphic features. Family history is remarkable for a seizure disorder in his
mother, who is a stay at home mother who finished high school. Which of the following
molecular alterations is the most likely reason for these findings in this patient?
A.
B.
C.
D.
E.
C52.
Deformation
Disruption
Dysplasia
Malformation
Migration
ARX mutation
DCX mutation
Deletion of the terminal end of 17p
EMX2 mutation
LIS1 mutation
A newborn male infant in the NICU is found to display hypotelorism, small head
circumference, thin nose and his mother has a single maxillary incisor. Which of the
following genetic defects is the most likely cause of this newborn’s phenotype?
A.
B.
C.
D.
E.
Chromosomal anomaly
DNA repair enzyme mutation
Fibroblast growth factor receptor mutation
RAS pathway component mutation
Sonic hedgehog mutation
705
C53.
You are asked to evaluate a small child with asymmetric head shape, midface hypoplasia
and digital anomalies. You obtain a head CT with 3-D reconstruction that demonstrates
the finding below.
Which of the following genetic defects is the most common cause of the radiographic
finding seen on this 3-D CT image?
A.
B.
C.
D.
E.
C54
Chromosomal anomaly
DNA repair enzyme mutation
Fibroblast growth factor receptor mutation
RAS pathway component mutation
Sonic hedgehog mutation
The correct answer is E
A nasal sample was sent on a patient that was seen in clinic who you suspect has a
problem with ciliary function. The results found abnormalities in the dynein arms of the
cilia. Which of the following diagnoses is most consistent with this finding?
A. Bardet-Biedl syndrome
B. Grieg cephalopolysyndactyly syndrome
C. Holoprosencephaly
D. Joubert syndrome
E. Situs inversus
MENDELIAN TRANSMISSION
C55. Consider a disorder inherited as an autosomal dominant with complete penetrance in
which reproductive fitness is 0.8 and the disease frequency is 1/50,000. What is the rate
of new mutation?
A. 1/1,000,000
B. 1/500,000
C. 1/250,000
D. 8/500,000
E. 8/1,000,000
706
C56.
A man with hemochromatosis is married to a woman who is not affected, but is of the
same ethnicity as he. Before doing genetic testing, what is closest to the risk of their
having a child homozygous for a hemochromatosis mutation? Assume a population
frequency of homozygosity to be 1/400.
A. 1/5
B. 1/10
C. 1/20
D. 1/40
E. 1/80
C57.
A woman has retinitis pigmentosa (RP) and is found to be heterozygous for two unlinked
genes, each of which has been implicated in RP. Her partner does not have RP and is
unrelated. The chance for a child of this couple to have RP is?
A. 0
B. 1/8
C. 1/4
D. 1/2
E. 1
C58.
What is the risk that a person with phenylketonuria (PKU) would have a child with PKU,
assuming his partner is not related but is of the same ethnicity? Assume a population
frequency of 1/10,000.
A. 1/1,000
B. 1/500
C. 1/200
D. 1/100
E. 1/50
C59.
A couple who are first cousins request counseling regarding their risk of having a child
with alpha-1-antitrypsin deficiency, a rare autosomal recessive trait. Their grandfather is
affected with the disorder. What is the risk to their child of being homozygous for a
mutation for the condition?
A. 1/4
B. 1/8
C. 1/16
D. 1/32
E. 1/64
707
C60.
A couple who are first cousins ask about their risk of having a child with a rare autosomal
recessive disorder that affected the sister of their common grandmother. The
grandmother was not affected by this condition that exhibits complete penetrance. What
is the risk to their child?
A.
B.
C.
D.
E.
C61.
You are counseling a couple where both parents are affected with NF1. They have just
had a baby, who at birth has no signs of the disorder. You explain that although NF1
follows is autosomal dominant disorder, features such as café-au-lait spots may not
appear at birth, so their child still needs to be followed clinically, and that genetic testing
is possible. You also note that homozygotes for NF1 mutation do not survive in utero.
The couple asks you to estimate the chance that the baby has inherited an NF1 gene
mutation. You quote the family which of the following risks?
A.
B.
C.
D.
E.
C62.
1/128
1/96
1/32
1/24
1/6
1/4
1/3
1/2
2/3
1
A genetic trait is associated with a fitness of 0.8 and is found in 1 in 10,000 individuals in
the population. Assuming mutation/selection balance, what is the mutation rate?
A.
B.
C.
D.
E.
0.00001
0.00002
0.00008
0.002
0.008
PHARMACOGENETICS
C63. A 70 year old woman has had a heart valve replacement and needs to be treated with a
drug to reduce platelet function. You are considering starting her on clopidigrel. Which
of the following genetic tests would be appropriate to inform this decision?
A. CYP2C19
B. CYP2D6
C. N-acetyl transferase
D. Thiopurine methyltransferase
E. VKORC1
708
C64.
A patient you are following learns of a clinical trial for the disorder. It is a phase I trial.
What should the patient expect if he participates?
A. The trial will determine whether the treatment is effective.
B. The trial will determine whether the treatment is more effective than alternatives.
C. The trial will determine the maximum tolerated dose of drug and pharmakokinetics.
D. The trial will monitor for long-term side effects of treatment.
E. The trial will determine whether the drug affects fertility.
C65.
The parent of a child with cystic fibrosis has read about a new medication, ivacaftor,
which can help some with the disorder. Which of the following mutations would need to
be present in the child to justify treatment?
A. 3849+10kbC>T
B. G551D
C. G542X
D. phe508del
E. R117H
C66.
A case-control study reveals an allele in 700/1000 cases and 300/1000 controls. What is
closest to the odds ratio of a carrier of the allele having the disease compared with a noncarrier having the disease?
A. 1
B. 3
C. 5
D. 7
E. 10
C67.
A 60 year old man develops severe respiratory depression after a dose of codeine. Which
of the following genes is most appropriate to test in an effort to find an explanation?
A. CYP2C19
B. CYP2D6
C. N-acetyl transferase
D. Thiopurine methyltransferase
E. VKORC1
709
SYSTEM-BASED SINGLE GENE DISORDERS
C68. A boy is seen with Hirschsprung disease. No apparent syndrome is found. Which of the
following genes is most likely to be mutated if a genetic test is done?
A. AKT1
B. APC
C. PTEN
D. RAS
E. RET
C69.
A child with autism spectrum disorder is found to have a deletion at 15q13.3. For which
of the following clinical problems is the child at greatest risk?
A. congenital heart defect
B. deafness
C. epilepsy
D. obesity
E. retinitis pigmentosum
C70
Exome sequencing reveals a heterozygous pathogenic mutation in MYO7A. Which of the
following phenotypes is most likely?
A.
B.
C.
D.
E.
C71.
AD deafness
AR deafness
Congenital myopathy
Ectodermal dysplasia
Usher syndrome
A 2 year old girl is seen with streaky patches of hyperpigmentation. She had had seizures
in the newborn period, which subsequently resolved. She had multiple skin lesions in the
early weeks of life that began with vesicular lesions and later evolved. A genetic test is
done. Which of the following types of mutation is most likely to be found?
A. Amino acid substitution
B. Deletion
C. Frameshift mutation
D. Mosaic trisomy
E. Stop mutation
C72.
A newborn has failed her hearing screen. An electrocardiogram is ordered and is found
to be abnormal. Which of the following findings is most likely?
A. Elevated P wave
B. Elevated R wave
C. Elongated PR interval
D. Elongated QT interval
E. Right bundle branch block
710
C73.
A man has 3 schwannomas, no vestibular schwannomas, and a normal exam. His
schwannomas came to attention because of pain. Which of the conditions best explains
this presentation?
A. NF1
B. NF2
C. schwannomatosis
D. tuberous sclerosis complex
E. von Hippel Lindau syndrome
C74.
A newborn is having multiple seizures and you learn that there is a family history of the
same in multiple relatives on the father’s side. Which of the following type of gene is
most likely to be responsible for this history?
A. chloride channel encoding gene
B. gaba receptor encoding gene
C. nicotinic acid receptor encoding gene
D. potassium channel encoding gene
E. sodium channel encoding gene
C75.
33. A 64-year-old man is seen with tremor, unsteady gait, and memory loss. Which of
the following genetic tests would be most likey to be informative?
A. ApoE
B. ATM
C. DYT1
D. FMR1
E. LRRK2
C76.
40. A 40 year old man with tuberous sclerosis complex has a progressively enlarging
renal angiomyolipoma. Which of the following is the most appropriate approach to
therapy?
A. treatment with bevacizumab
B. treatment with everolimus
C. treatment with imatinib
D. nephrectomy
E. treatment with rapamycin
711
C77.
45. A newborn with severe weakness is evaluated in the nursery. He is intubated and
moves very little. Exam reveals little movement but weakly elicitable deep tendon
reflexes. On taking a family history it is noted that his mother has visible mild facial
weakness. What is the most likely diagnosis for the infant?
A. Charcot-Marie-Tooth disease
B. congenital muscular dystrophy
B. Duchenne muscular dystrophy
C. myotonic dystrophy
D. spinal muscular atrophy
C78.
You are seeing Ms Andrews who is a 36-year-old woman who recently developed
dyspnea and fatigue. She was found to have a mean pulmonary arterial pressure of 33
mm Hg on an otherwise normal echocardiogram. Her father is healthy. Her mother was
also healthy until she developed right heart failure during pregnancy. Ms Andrews’
pulmonologist asks you “Which of the following genes is most likely to bear a mutation
that is the cause of her condition?”
A.
B.
C.
D.
E.
C79.
ACVRL1
BMPR1B
BMPR2
EAG
SMAD9
You are seeing Joey, a child who had an elevated Immunoreactive Trypsinogen newborn
screening result. Since Joey was found to have a Phe508del/G551D CFTR genotype
which of the following medications is likely to potentiate his CFTR function?
A.
B.
C.
D.
E.
Gleevec (Imatinib)
Kalydeco (ivacaftor)
Mucomyst (acetylcysteine)
Pulmozyme (dornase)
Tobi (tobramycin)
712
C80.
You are seeing Seth who is a 4-month-old boy who has had recurring respiratory
infections, diarrhea and failure to thrive since one month of age. Seth had two maternal
uncles who died at 4 and 7 months of recurring bacterial infections. Which of the
following genes would you test first to find the cause of Seth’s problems?
A.
B.
C.
D.
E.
C81.
Sammy is a 12-year-old boy is referred to you for genetic testing to determine the cause
of his Common Variable Immune Deficiency (CVID). His was recently diagnosed
because of recurring Streptococcal, then Klebsiella pneumonia which was complicated
by meningitis at 6 years of age. An abnormality in which of the following genes is the
one that is most likely to cause Sammy’s CVID?
A.
B.
C.
D.
E.
C82.
ADA
IL2RG
STAT3
TNFR5R13B
ZAP70
You are seeing a 32-year-old man, Todd, who has recently been found to have an area of
aortic dilation. Which of the following disorders is least likely to be the cause of Todd’s
aortic aneurysm?
A.
B.
C.
D.
E.
C83.
ADA
FOXP3
IL2RG
STAT3
ZAP70
Ehlers Danlos Syndrome
Familial Thoracic Aortic Aneurysm and Aortic Dissection (TAAD)
Loeys-Dietz Syndrome
Marfan Syndrome
Pseudoxanthoma Elasticum
You are seeing a brother and sister (Will and Cindy) who are 15 and 19 years of age.
Both have recently been found to have multiple renal cysts. You are asked to determine
the genetic cause of their renal cysts. Which of the following genes is most likely to show
a mutation on molecular testing for this disorder?
A.
B.
C.
D.
E.
PKD1
PKD2
PKHD1
OCRL
TSC2
713
C84.
You are evaluating Sarah, a 13-month-old girl, who has microcytic anemia (MCV=56)
with 5% Hb Bart. Which of the following hemoglobinopathies do you think is the most
likely cause of Sarah’s findings?
A.
B.
C.
D.
E.
C85.
You are evaluating Paul who is a 5-day-old boy. Paul has a large hematoma at his
Vitamin K injection site and he continues to ooze after initial bleeding from his
circumcision three days ago. He has a normal prothrombin time, von Willebrand and F9
factor levels and no family history of coagulation problems. You suspect hemophilia A.
Which of the following mutations in the F8 do you predict is most likely to explain
Paul’s problems?
A.
B.
C.
D.
E.
C86.
Deletion/duplication
IVS1 inversion
IVS22 inversion
Missense
51 promoter
A 4-year-old girl has sparse hair, abnormal teeth, and absence of sweating. Which of the
following genes is most likely to be responsible for this disorder?
A.
B.
C.
D.
E.
C87.
Į-Thalassemia rait
ȕ-Thalassemia trait
Hb Bart syndrome
Hb E disease
Hb H disease
EDA1
EDAR
GJB6
MSX1
SLC45A2
A skin biopsy is done on a young woman with severe scarring due to epidermolysis
bullosa. Which of the following sites is most likely to demonstrate skin separation in
this biopsy?
A.
B.
C.
D.
E.
Below basement membrane
Epidermis
Multiple layers in skin
Sub-dermis
Within basement membrane
714
C88.
A 30-year-old man develops congestive heart failure and has left ventricular hypertrophy
by echocardiogram. If this clinical finding is genetically determined, which of the
following is the most likely mode of inheritance?
A.
B.
C.
D.
E.
C89.
Autosomal dominant
Autosomal recessive
Mitochondrial
X-linked dominant
X-linked recessive
You evaluate a 2-year-old boy, Ethan, with multiple fractures after mild trauma. The
family pedigree (below) reveals additional family members with a similar clinical
phenotype. Ethan’s two sisters have no history of fractures.
The most likely cause of fractures in this family is a mutation in which of the following
genes?
A.
B.
C.
D.
E.
C90.
COL1A1/2
IFITM5
LEPRE1
RPIB
SERPINF1
You evaluate an 8-year-old girl with eczema since 3 months of age, recurring boils and
cyst forming pneumonia since 1 year of age but no significant problems with diarrhea.
Which of the following disorders is the most likely cause of her problems?
A.
B.
C.
D.
E.
AR Severe Combined Immune Deficiency
Common Variable Immune Deficiency
Hyper IgE Syndrome
Hyper IgM Syndrome
XL Severe Combined Immune Deficiency
715
C91.
You evaluate an 8-year-old boy with “very soft skin” and myopia. On physical
examination you find his skin is quite soft, especially on his face; he has bilateral inguinal
hernias, and normal joint mobility. Which of the following disorders is the most likely
diagnosis?
A.
B.
C.
D.
E.
C92.
You evaluate a 19-year-old man (Ralph) with chronic hip and knee pain. His mother had
hip and knee replacements at 42 years of age. On your physical examination you find
Ralph’s height is at the 10th percentile, he has decreased range of motion of both hips and
knees, but no scoliosis. Which of the following disorders is the most likely diagnosis?
A.
B.
C.
D.
E.
C93.
Ehlers Danlos Syndrome
Hypochondroplasia
Marfan Syndrome
Multiple Epiphyseal Dysplasia
Osteogenesis Imperfecta
You are seeing a child who presents with small teeth and dystrophic nails. Her hair is
normal, she has normal skin, and she is able to sweat normally. Which of the following
genes is the most likely to explain her disorder?
A.
B.
C.
D.
E.
C94.
Cutis laxa Syndrome
Classic Ehlers-Danlos Syndrome
Loeys Dietz Syndrome
Marfan Syndrome
Pseudoxanthoma Elasticum
EDA
EDAR
GJB6
MSX1
WNT10A
You are evaluating a 17-year-old girl, whose primary care physician (PCP) reports has
primary amenorrhea but is otherwise normal. The primary care physician obtained a
pelvic ultrasound which showed a “blind vagina and no uterus”. Laboratory results
showed low 17-OHP but high testosterone levels. Which of the following diagnoses
provides the best explanation of her clinical findings?
A.
B.
C.
D.
E.
Congenital adrenal hyperplasia
Complete androgen insensitivity
Luteinizing hormone deficiency
Turner syndrome
46, XY sex reversal, SRY related
716
C95.
You are seeing a 6-month-old girl who has short stature, rachitic skeletal changes and
serum biochemistries which included: sodium 138, potassium 4.6, chloride 105,
bicarbonate 22, BUN 8, creatinine 0.3, glucose 87, calcium 10.4, ionized calcium 4.4,
magnesium 2.2, phosphorus 6.4 mg/dL (Nl: 3.0 – 6.0 mg/dL) alkaline phosphatase 44
U/L (Nl: 110 – 320 U/L) and vitamin B6 (pyridoxal 5’-phosphate) level 2449 nmoles/L
(Nl: 10 – 110 nmoles/L). Which of the following clinical findings is associated with her
disorder?
A.
B.
C.
D.
E.
C96.
A woman is referred for genetic testing because of a severe disorder of skin blistering and
scarring. There is no known prior family history of the condition. Biopsy reveals
scarring in the dermis below the basement membrane of the skin. Which of the following
genes is the most likely to explain her condition?
A.
B.
C.
D.
E.
C97.
COL7A1
EXPH5
KRT14
LAMB3
TGM5
You are seeing an eight-year-old boy with microcephaly, telecanthus, coarse facies,
genital anomalies, hypotonia, intellectual disability & mild anemia thought to be due to
thalassemia. Which of the following genes is the best candidate to explain his findings?
A.
B.
C.
D.
E.
C98.
Craniosynostosis
Hypocalemia
Hypocalciuria
Osteopetrosis
Tall stature
ATRX
HBA1
HBA2
HBB
HBG1
You are evaluating a young boy with proportionate short stature, relative macrocephaly,
and hypospadias. His bone age is delayed. You arrange for testing of imprinting defects
on chromosome 11. Which of the following changes is the most likely to explain his
features?
A.
B.
C.
D.
E.
CDKN1C mutation
Gain of methylation of the maternal IC1 site
Hypomethylation of the paternal IC1 site
Microdeletion of the maternal IC1 site
Paternal uniparental disomy
717
Answers to Clinical Genetics Questions C1-C98
C1.
Answer: E. Rectal prolapse is a frequent feature of CF and should raise concern about that
diagnosis.
C2.
Answer: A. Mutations causing deficiency of G proteins occur in Albright osteodystrophy while
activating mutations in the same gene cause McCune-Albright.
C3.
Answer: E. This is a typical description for a patient with Warburg syndrome. A patient wlth
Meckel-Gruber syndrome would more likely have encephalocele, polydactyly, and polycystic
kidney disease. The Pallister-Hall syndrome is characterized by hypothalamic hamartoblastoma,
hypopitutarism, imperforate anus, and postaxial polydactyly. The Neu-Laxova syndrome is
characterized by microcephaly or lisssencephaly, elfin-facies with exophthalmos, and syndactyly
with subcutaneous edema. The Miller-Dieker syndrome is characterized by lissencephaly.
C4.
Answer C. Consensus clinical diagnostic criteria are accurate, but the mainstay of
diagnosis is DNA-based methylation testing to detect abnormal parent-specific
imprinting within the Prader-Willi critical region (PWCR) on chromosome 15; this
testing determines whether the region is maternally inherited only (i. e., the paternally
contributed region is absent) and detects more than 99% of affected individuals.
Methylation-specific testing is important to confirm the diagnosis of PWS in all
individuals, but especially those who have atypical findings or are too young to manifest
sufficient features to make the diagnosis on clinical grounds. (www.genereviews.org)
C5.
Answer: C. This would be a reasonably good story for Stickler syndrome. The arthropathy can
simulate juvenile rheumatoid arthritis. Retinal detachment is a feature. Robin sequence can occur
in childhood. The other disorders would all be quite different.
C6.
Answer: A. The thumbs may be absent in any of these disorders except TAR where they are
consistently present.
C7.
Answer: D. This description of the changes in the ear is typical of diastrophic dwarfism. It is a
very distinctive change but is usually not present in the neonatal period.
C8.
Answer: C. Female appearing genitalia in an XY infant is a common feature of campomelic
dysplasia, but is not a feature of the other disorders.
C9.
Answer: E. This clinical description is very suggestive of anhidrotic ectodermal dysplasia, which
is usually X-linked. Many other forms with different inheritance occur. This story is typical for
the X-linked disorder but is not diagnostic of that pattern of inheritance.
C10.
Answer: B. This is a typical description of holoprosencephaly. The trisomies and Meckel-Gruber
syndrome would likely be associated with additional features below the neck. The CNS
abnormalities would not be a feature of Ellis van Creveld although cardiac defect and limb
abnormalities would be present.
C11.
Answer: D. This is a typical clinical description for Trisomy 13.
718
C12.
Answer: B. All the conditions listed are autosomal dominant disorders. Missense mutations with
some type of aberrant function are involved in MEN2A and achondroplasia. The mechanism is
not clear for Huntington disease, but there is no evidence that deletion of this region causes the
disease, and the evidence favors some type of gain of function. A harmful effect of the mutant
protein is involved in amyloidosis. Many of the mutations in acute intermittent porphyria and in
some of the other porphyries are obvious loss of function mutations indicating that
haploinsufficiency is the mechanism of dominance for AIP.
C13
Answer: A. Reproductive fitness is relatively normal in HD. It is somewhat reduced, but not
severely so in NF. Fitness is substantially reduced, but not to 0, in TSC. OI type II is lethal
neonatal OI and reproductive fitness is 0. Keep in mind that a reproductive fitness of 1 is equal to
the normal population and a reproductive fitness of 0 is no ability to reproduce.
C14.
Answer: E. Conotruncal defects are most common in Down syndrome and clefting would be less
typical. It is unlikely a child with Trisomy 13 or 18 would survive to age 6 years. Congenital
heart defect would not be expected with otopalatodigital syndrome and it is much less common
than VCFS. Clefting and TOF are not features of CFC. All of the features listed are common in
VCFS.
C15.
Answer: C. The retinal detachment is the unique feature in this question- it is only found in
Stickler syndrome.
C16.
Answer: B. This is a single primary malformation because the other malformations are a direct
result of the first. A MCA syndrome includes anomalies that arise independently, not
causatively. An association is a group of anomalies that frequently arise together (e.g.
VACTERL) without an apparent relationship to each other or a known single gene cause. The
vast majority of neural tube defects are isolated and multifactorial in origin, not chromosomal or
teratogenic.
C17.
Answer: C. Absent radii is associated with absent or abnormal thumbs except in one syndromeThrombocytopenia-Absent Radius syndrome. These children have a high rate (as the name
suggests) of developing thrombocytopenia. The other answers suggest other syndromes that
commonly manifest radial hypoplasia, but in all the thumbs are also absent: Trisomy 18 (answer
B), Fanconi syndrome (answer D), and VATER (answer E). Answer C suggests ulnar mammary
syndrome, in which ulnar ray (not radial ray) defects are associated with mammary hypoplasia.
C18.
Answer: B. The most common cytogenetic finding seen with hypoplastic left heart (HLH) is a
normal karyotype. Turner syndrome (45,X) is among the most common cytogenetic abnormalities
seen with HLH, but is still not as common as a normal karyotype. Other syndromes seen with
HLH include Down syndrome, and Smith Lemli Opitz.. HLH is not seen at an increased
frequency in Williams syndrome [46,XY.ish del(7)(q11.23q11.23)(ELN-)], VCFS/DiGeorge
[46,XX,del(22)(q11.2q11.2)], or Klinefelter syndrome [47,XXY].
C19.
Answer: A. While all the syndromes have a distinctive facial appearance, Aarskog syndrome is an
X-linked disorder, so father-to-son transmission would not be seen. The other syndromes are
autosomal dominant.
C20.
Answer: D. The description fits Holt-Oram syndrome, and represents variable expression with
complete penetrance.
719
C21.
Answer: A. The child in this scenario has Simpson-Golabi-Behmel syndrome, an X-linked
overgrowth syndrome caused by mutations in Glypican 3 on Xq26. There is no parent of origin
effect, and male to male transmission is not seen. These children have a high rate of cardiac
conduction defects, which may account for the observation of unexplained death. Bone age is
advanced initially, then becomes normal. Retinitis pigmentosa is not a common long-term
complication.
C22.
Answer: E. This scenario describes a child with Cleidocranial dysplasia (CCD). Women with
CCD have an increased rate of needing a Caesarian section in childbirth. They do not have an
increased fracture rate. Mild short stature can be seen, but adult heights are usually at the low end
of the normal range. Retinal detachment is not a manifestation. Cytogenetic testing is not helpful.
Some children have a microdeletion on 6p21, but it is not cytogenetically visible. These children
usually have developmental delay/mental retardation in addition to the usual CCDS findings.
C23.
Answer: D. The condition describes is Incontinentia Pigmenti (IP), an X-linked male lethal trait.
In such conditions one sees fewer than expected males in a given pedigree, as ½ of male fetuses
(those affected) are lost. Germline mosaicism is not common in IP. For an X-linked lethal trait,
approximately 1/3rd of cases are new mutations. There is no cytogenetic abnormality associated
with IP. Chromosome analysis of fibroblasts demonstrates an abnormality in approximately 40%
of cases of Hypomelanosis of Ito, a related but clinically distinct disorder. Affected females can
have unaffected males and affected and unaffected females.
C24.
Answer: D. This child’s description is classic for Diastrophic Dysplasia, which is an autosomal
recessive trait caused by mutations in the DDST gene.
C25.
Answer: C. The prothrombin G20210A mutation is associated with the hypercoagulable state.
Mutation of any of the other listed proteins would lead to bleeding tendency, not increased
tendency to form blood clots.
C26.
Answer: B. Fatigue is the most common presenting complaint, although all of the listed items
are features of hemochromatosis.
C27.
Answer: C. Hereditary angioedema is due to C1q inhibitor deficiency in the complement
system.
C28.
Answer: D. Phase I reactions are oxidations, reductions, and hydrolysis reactions; one of the
major enzyme systems involved in phase I reactions is the P450 system.
C29.
Answer: D. The TPMT polymorphism affects metabolism of drugs such as 6-mercaptopurine, 6thioguanine, etc., which have a role in treatment of leukemia and autoimmune disorders.
C30.
Answer: B. Imatinib targets the BCR-ABL kinase that results from the Philadelphia
chromosome, which is associated with chronic myelogenous leukemia.
C31.
Answer: B. The sodium channel polymorphism can lead to arrhythmia upon exposure to specific
drugs.
720
C32.
Answer: C. In approximately 50% of cases, Pierre Robin sequence (PRS) is part of a genetic
syndrome, and half of that is Stickler syndrome (SS). The majority of those with SS have eye
involvement (only those cases caused by COL11A2 mutations do not). The eye findings of SS are
usually evident in the newborn period. Chromosome abnormalities are unlikely in an otherwise
normal baby with PRS. Del22q11.2 is the next most common syndromic cause of PRS, so it
should be ordered in every baby with PRS. However, del22q11.2 is only about half as common as
SS as the cause of PRS. Cardiac conduction abnormalities are unlikely in this setting.
C33.
Answer: C. SMS is caused by deletions of 17p11.2 visible with routine chromosome analysis
(>550 bands), although deletions may be missed on first inspection. There is evidence that the
majority of the manifestations may be attributable to haploinsufficiency of the RAI1 gene. While
a small percentage of Neurofibromatosis 1 (NF1) is caused by deletions of the NF1 gene, the
majority of cases are caused by point mutations in the NF1 gene. Furthermore, a cytogenetically
visible deletion is very rare. Williams syndrome is caused by submicroscopic deletions of 7q11.2,
encompassing the elastin gene and others as well. The Pfeiffer syndrome (PS) phenotype can be
caused by a variety of different point mutations in FGFR1 (mild type 1 PS), FGFR2 (more severe
type 1 as well as types 2 and 3 PS), and FGFR3 (some cases of FGFR3-related craniosynostosis,
or Muenke syndrome, can look like PS). Marfan syndrome is generally caused by point mutations
in fibrillin-1 on chromosome 15q21.
C34.
Answer: D. Blue sclerae are seen in OI types 1, 2 and 3, but are also seen in most healthy babies
as a normal variant. Healing fractures of different ages can be seen in OI as well as NAI. Type I
collagen analysis is considered the ‘gold standard’ for the diagnosis of OI, but it can be normal in
over 10% of OI, so a negative analysis does not rule out OI. NAI can be seen in families of any
level of socioeconomic status of the parents. Retinal hemorrhages are not a feature of OI, but do
suggest a diagnosis of NAI.
C35.
Answer: C. Mutations in MECP2 cause Rett syndrome, but also can cause an Angelman
syndrome-like phenotype, as described above. Uniparental disomy (UPD) 7 causes idiopathic
short stature and a small percentage of cases of Russell-Silver syndrome. FMR1 is the gene for
Fragile X syndrome, and does not typically present with these findings. The ratio of cholesterol/7dehydrocholesterol is used to diagnose Smith-Lemli-Opitz syndrome.
C36.
Answer: B. The disorder was not expressed in the mother of the two affected children, since these
women inherited the gene from their father. Their sons each inherited the mutation from a
female, though, and therefore are affected.
C37.
Answer: B. EGF receptor mutations are found in small cell lung cancers that are likely to
respond to gefitinib.
C38.
Answer: A. Hemophilia A is commonly associated with an inversion of the factor VIII
gene.
C39.
Answer: E.
C40.
Answer: A.
721
C41.
The correct answer is B. Although this man probably does not have an increased risk to have a
child with an NTD, the question requires you to recognize that there is a "background" incidence
of NTDs in the population and to know that the incidence is about 1 per 1000 in the U.S.
Although this man's risk is not increased, it would not be lower than the population incidence, the
correct answer is (B)
C42.
The correct answer is B. In the U.S. and Canada, data on risk to first cousins of an affected range
from 0.3 to 0.9%, but a second cousin (the relationship between this woman’s fetus daughter and
her cousin’s baby with anencephaly) don’t have a risk significantly above background. There
does appear to be a higher risk when the proband is related through the mother rather than
through the father. Even though this consultand may have a slightly higher risk than the first
man, the correct answer is still (B).
C43.
The correct answer is D. The data for risk following two affecteds differ between the U.S. and the
U.K., reflecting the lower population incidence here. However, in both places, the risk is at least
twice what it would be after only 1 affected. Iin the U.S., this is about 6.4%; in the U.K., at least
10%. Since the question asks about U.S. figures, the correct answer is (D).
C44.
The correct answer is C. The father in (J7) is trying to achieve some mastery over a situation over
which he has no control by intellectualizing.
C45.
The correct answer is B. The mother in (J8) is denying the reality of her daughter's disability by
attributing it to a cause that, at age 2 or 3 years old might still have been legitimate.
C46.
The correct answer is D. The mother in J9, who presumably has been counseled regarding her
son's diagnosis and it's cause, is projecting her own guilt onto the obstetrician.
C47.
The correct answer is E.
C48.
Answer: E. A mutation in LDL receptor protein (LRP6) has been found in a family with a number
of conditions, including osteoporosis; and an LRP5 mutation in association with pseudogloma
osteoporosis syndrome. Both are involved in the Wnt signaling pathway.
DEVELOPMENTAL GENETICS
C49.
The correct answer is B.
Keywords: Epigenetic change, transposon insertion
Transposon insertion is a genomic change, not an epigenetic change. Epigenetic changes are not
inherited, and are not permanent changes in DNA. The other choices are all examples of
epigenetic changes.
C50.
The correct answer is B.
Keywords: Poland anomaly, syndactyly
Explanation: The clinical vignette describes a child with Poland anomaly. Poland anomaly is
believed to result from a disruption of blood flow during development before birth. This
disruption is thought to occur at about the sixth week of embryonic development and affect blood
vessels that will become the subclavian and vertebral arteries.
722
C51.
The correct answer is B.
Keywords: Agyria, lissencephaly,
Agyria is lissencephaly. Mutations in ARX, DCX and LIS1, as well as terminal 17p
deletions cause lissencephaly. EMX2 mutations cause schizencephaly, ruling our D.
Since there are no dysmorphic features, 17p deletions (C, which cause Miller-Dieker
syndrome with craniofacial dysmorphisms) can be ruled out. Similarly, since the patient
has normal genitalia, that rules out ARX mutations (A). Isolated lissencephaly can be
caused by LIS1 or DCX mutations. Since the mother has seizures, but clearly no
lissencephaly, and the patient is male, the most likely explanation is (B) DCX mutation,
not (E) LIS1 mutation.
C52.
The correct answer is E.
Keywords: holoprosencephaly, sonic hedgehog
The clinical case described a child with the outward appearance of holoproencephaly.
This is supported by the single maxillary incisor in his mother. The most common cause
of holoprosencephaly are mutations in sonic hedgehog
C53.
The correct answer is C.
Keywords: Craniosynostosis, fibroblast growth factor receptor (FGFR)
The CT scan displays craniosynostosis. The most common causes of craniosynostosis are
mutations in FGFR1, 2 and 3.
C54
The correct answer is E
Bardet-Biedl syndrome (BBS genes), Grieg cephalopolysyndactyly syndrome (GLI3),
Holoprosencephaly (Sonic hedgehog and other genes) and Joubert syndrome (multiple
genes) are caused by defects in signal transduction in the cilia, but they do not display
structural cilia defects. Situs inversus is often caused by mutations in ciliary dyneins,
which can be detected in electron micrographs as structural defects in the dynein arms.
MENDELIAN TRANSMISSION
C55. The correct answer is B
Source: Genetic Transmission lecture
Keywords: population genetics, dominant, selection, mutation
Explanation: P = ps 2p = 1/50,000, hence p = 1/100,000; s = 0.2 (1-fitness); therefore
m = (1/100,000)(0.2) = 1/500,000
C56.
The correct answer is C.
Source: Genetic Transmission lecture
Keywords: autosomal recessive, pseudodominance, risk assessment
Explanation: The carrier frequency is 2(1/20) = 1/10; the risk to a child is 1/20; we are
ignoring the possibility that the mother is a homozygote, which is possible, but adds little
to the risk.
723
C57.
The correct answer is C.
Source: Genetic Transmission Lecture
Keywords: genetic transmission, Mendelian, digenic
Explanation: This is an example of digenic inheritance. The woman has a 1/2 chance of
transmitting each of the mutant alleles, so a 1/4 chance of transmitting both to an
offspring.
C58.
The correct answer is D.
Source: Genetic Transmission Lecture
Keywords: autosomal recessive, Hardy-Weinberg, genetic counseling, risk assessment
Explanation: The frequency of the gene is the square root of 1/10,000 = 1/100. The
carrier frequency is therefore 1/50. The chance of having an affected child is
1(1/50)(1/2) = 1/100
C59.
The correct answer is C.
Source: Genetic Transmission Lecture
Keywords: consanguinity, risk assessment, autosomal recessive
Explanation: Both of the parents of the couple are obligate carriers. Each partner
therefore has a risk of ½ of having inherited a mutation. Each in turn would have a ½
chance of passing the mutation on to a child. The risk to a child is therefore
(1/2)(1/2)(1/2)(1/2) = 1/16.
C60.
The correct answer is B.
Keywords: inbreeding; population genetics; genetic risk assessment
Explanation: The grandmother has a 2/3 risk of being a carrier. There is a 1/4 chance
that she passed the mutation on to either parent and hence a 1/16 chance that she passed it
to both. If both are carriers, they have a 1/4 chance of having an affected child. Hence
the risk here is (2/3)(1/16)(1/4) = 1/96
C61.
The correct answer is D.
Keywords: genetic transmission, Mendelian genetics, genetic risk
Explanation: The parents are both heterozygous for an NF1 mutation. There is a one in
four chance that a fetus will be homozygous and therefore will die in utero, a one-in four
chance that an offspring will be homozygous for the wild type gene, and a one in two
chance for being heterozygous for either one of the parent’s mutations. Since the
homozygous mutation embryo will not survive to birth, a live born child has a 2/3 risk of
being affected with NF1.
C62.
The correct answer is B.
Keywords: population genetics; Hardy-Weinberg; mutation rate
Explanation: P = sq2; s = 1 – 0.8 = 0.2; hence P = (1/10,000)(0.2) = 0.0001(0.2) =
0.00002
724
PHARMACOGENETICS
C63. The correct answer is A.
Source: Genomic Medicine lecture
Keywords: pharmacogenetics
Explanation: CYP2C19 is required to activate clopidigrel, so would be appropriate to
test.
C64.
The correct answer is C.
Source: Genomic Medicine lecture
Keywords: clinical trials
Explanation: A phase I trial looks at maximum tolerate dose and side effects. A phase II
trial looks at efficacy and a phase III trial compares the treatment to alternatives. Longterm effects are monitored in phase IV.
C65.
The correct answer is B.
Source: Genomic Medicine lecture
Keywords: treatment, cytic fibrosis
Explanation: Ivacaftor targets the mutant protein due to the G551D mutation increasing
Cl- flow through the mutant CFTR.
C66.
The correct answer is C.
Source: Genomic Medicine lecture
Keywords: genetic association, case-control study
Explanation: The odds of a carrier having disease are 700/300 = 7/3; the odds of a noncarrier having disease are 300/700 = 3/7. The odds ratio is (7/3)/(3/7) = 49/9§5
C67.
The correct answer is B.
Source: Genomic Medicine lecture
Keywords: pharmacogenetics
Explanation: Codeine is converted to morphine by the CYP2D6 enzyme; a rapid
metabolizer would produce too much morphine at a standard dose and might have this
reaction
SYSTEM-BASED SINGLE GENE DISORDERS
C68. The correct answer is E.
Source: Systems I lecture
Keywords: Hirschsprung disease, genetic testing, RET gene
Explanation: RET mutations are the most common associated with non-syndrome
Hirschsprung disease.
C69.
The correct answer is C.
Source: Systems I lecture
Keywords: autism, chromosome 15, deletion
Explanation: Those with deletions at 15q13.3 are at risk for epilepsy, in addition to
autism.
725
C70.
The correct answer is A.
Source: Systems I lecture
Keywords: deafness, Usher syndrome, genetic heterogeneity, exome sequencing
Explanation: MYO7A mutation can lead to AD deafness, AR deafness, or Usher
syndrome; the fact that heterozygous mutation was found here suggests an AD mode of
inheritance, and hence deafness.
C71.
The correct answer is B.
Source: Systems I lecture
Keywords: mutation, incontinentia pigmenti
Explanation: The clinical history and exam suggest incontinenti pigmenti. This is
associated with IKBKG mutation, which in 80% of cases is a deletion.
C72.
The correct answer is D.
Source: Systems I lecture
Keywords: Long QT, deafness, Jervell and Lange-Nielsen syndrome
Explanation: Jervell and Lange-Nielsen syndrome is associated with congenital deafness
and long QT interval.
C73.
The correct answer is C.
Source: Neurogenetics lecture
Keywords: neurofibromatosis, phakomatosis
Explanation: multiple schwannomas plus pain without vestibular tumor is characteristic
of schwannomatosis; mosaicism for NF2 would be a less likely possibility. None of the
other disorders is likely to present with multiple schwannomas.
C74.
The correct answer is D.
Source: Neurogenetics lecture
Keywords: epilepsy, genetic testing
Explanation: The history is most compatible with benign neonatal convulsions, which
are associated with mutations in a potassium channel encoding gene.
C75.
The correct answer is D.
Source: Neurogenetics lecture
Keywords: FXTAS, dementia, tremor
Explanation: The clinical history is most compatible with fragile X ataxia tremor
syndrome.
C76.
The correct answer is B.
Source: Neurogenetics lecture
Keywords: tuberous sclerosis complex, treatment, angiomyolipoma
Explanation: everolimus is FDA approved for treatment of progressive angiomyolipoma
associated with tuberous sclerosis complex
726
C77.
The correct answer is D.
Source: Neurogenetics lecture
Keywords: neuromuscular, anticipation
Explanation: The weakness, preserved reflexes, and occurrence of milder weakness in
the mother are all suggestive of myotonic dystrophy. Charcot-Marie-Tooth would not
present at this age and deep tendon reflexes would be absent in spinal muscular atrophy.
Congenital muscular dystrophy would not explain the mother’s weakness.
C78.
The correct answer is C.
Source: Systems Based Disorders II slides 7- 8
Key Words: pulmonary arterial pressure, heart failure
Explanation: pulmonary arterial pressure of 33 mmHg and otherwise normal
echocardiogram indicates pulmonary arterial hypertension (PAH). Maternal history of
right heart failure during pregnancy suggests Heritable PAH. Slide 8 states that ~75% of
HPAH cases are due to BMPR2 mutations and that ACVRL1, BMPR1B, EAG and
SMAD9 are all rare causes of HPAH.
C79.
The correct answer is B
Source: Systems Based Disorders II slide 6 and Newborn Screening slide 50
Key Words: CFTR, G551D, potentiates
Explanation: Kalydeco (ivacaftor) is an allele specific drug that potentiates CFTR
function in individuals who have G551D CFTR mutations.
C80.
The correct answer is C.
Source: Systems Based Disorders II slides 10, 12 and 14-16
Key Words: recurring infections, onset 1-3 months, maternal uncle
Explanation: Recurring infections beginning at 1 month, FTT, and maternal uncle who
died of recurring infections all suggest XL SCID. IL2RG mutations are found in > 99%
of males with SCID. ADA is AR, FOXP3 causes IPEX syndrome (XL but very different
phenotype), STAT3 causes Hyper IgE (AD) and ZAP70 causes AR SCID.
C81. The correct answer is D.
Source: Systems Based Disorders II slides 10-12 and 15-16
Key Words: Recurring pneumonia, complicating meningitis, 12 years
Explanation: Sammy’s onset of recurring Streptococcal, then Klebsiella pneumonia
complicated by meningitis after 2 years of age is classic for CVID. TNFRSF13B (TAC1)
is currently the gene that is most often found to bear mutations causing CVIB. TAC1
mutations are found in 10-15% and ICOS mutations in 1% of CVIB cases. ADA causes
ADA Deficiency, IL2RG causes XL SCID, STAT3 causes Hyper IgE (AD) and ZAP70
causes AR SCID.
727
C82.
The correct answer is E.
Source: Systems Based Disorders II slides 41, 44-45 and 47
Key Words: Aortic root dilation
Explanation: Pseudoxanthoma Elasticum frequently causes mineralization of the internal
elastic lamina resulting in arterial narrowing but not aortic dilation. All the other answers
are associated with aortic dilation.
C83.
The correct answer is A.
Source: Systems Based Disorders II slides 50-53
Keywords: Multiple renal cysts, teenage
Explanation: AD Polycystic Kidney Disease (PKD) is characterized by 3 or more
(unilateral or bilateral renal cysts) in an individual aged 15-39 years. PKD1 mutations
cause 85% of cases and PKD2 causes 15%. PKHD1 causes AR Polycystic Kidney
Disease which presents in neonatal period. OCRL causes Lowe Syndrome which does not
have renal cysts and is XL. TSC2 can be deleted with PKD1 resulting in PKD in utero &
Tuberous Sclerosis but such contiguous TSC2- PKD1 deletions are rare.
C84.
The correct answer is E.
Source: Systems Based Disorders II slides 22-23
Key Words: microcytic anemia, MCV=56, 5% Hb Bart.
([SODQDWLRQ+E+'LVHDVHLVXVXDOO\FDXVHGE\GHOHWLRQRIĮJORELQJHQHVDQGLWFDXVHV
PLFURF\WLFDQHPLDRIWKLVVHYHULW\ZLWKa+E%DUWĮDQGȕWKDOWUDLWDQG+E(GRQRW
cause Hb Bart. Hb Bart Syndrome is usually lethal before or just after birth and is
associated with ~90% Hb Bart.
C85.
The correct answer is C.
Source: Systems Based Disorders II slides 18 and 20
Key Words: Hematoma, oozing, prothrombin, von Willebrand, F9
Explanation: Prolonged oozing after trauma, normal prothrombin; and normal von
Willebrand and F9 factor levels leave F8 deficiency or Hemophilia A as most likely
problem. All the answers are known F8 mutations but IVS22 inversions are found in 48%
of severe cases and often as new mutations.
C86.
The correct answer is B
Keywords: ectodermal dysplasia
Explanation: This is most likely to be a form of non-X-linked hypohydrotic ectodermal
dysplasia, which could be associated with EDAR mutation. EDA1 is associated with the
X-linked form; GJB6 with hydrotic ectodermal dysplasia; MSX1 with Witkop syndrome;
SLC45A2 with oculocutaneous albinism.
C87.
The correct answer is A.
Keywords: epidermolysis bullosa
Explanation: Separation occurs below the basement membrane in dystrophic EB; it
occurs in the epidermis in simplex EB, within the basement membrane in junctional EB,
and at multiple layers in Kindler syndrome.
728
C88.
The correct answer is A.
Keywords: cardiomyopathy
Explanation: Hypertrophic cardiomyopathy is usually autosomal dominant if it is due to
a monogenic cause.
C89.
The correct answer is A.
Ethan, and his brother, mother and maternal grandfather all had many fractures after mild
trauma. This is consistent with and AD form of OI which is most likely OI Type 1 which
is caused by mutations in COL1A1/2 in >90% of cases. IFITM5, LEPRE1, RPIB, and
SERPINF1 are genes that cause AR forms of OI.
C90.
The correct answer is C
This 8-year-old girl has had eczema since infancy and recurring boils and cyst forming
pneumonia since 1 year of age but no significant problems with diarrhea. The triad of
recurrent skin boils, cyst forming pneumonia and high serum IgE is classic for Hyper IgE
syndrome. A. AR Severe Combined Immune Deficiency presents with recurrent
infections, diarrhea, and Pneumocystis, but not cyst forming pneumonia. B. Common
Variable Immune Deficiency presents with recurrent sinopulmonary infections and
diarrhea, but not cyst forming pneumonia. D. Hyper IgM Syndrome presents with
recurrent infections, diarrhea and neutropenia but not cyst forming pneumonia. E. XL
Severe Combined Immune Deficiency presents with candidiasis, absent tonsils and
persistent infections. It almost always only affects males.
C91.
The correct answer is A.
He has “very soft and loose skin”, especially on his face, myopia, bilateral inguinal
hernias, but normal joint mobility. This is most consistent with Cutis Laxa. B. Classic
Ehlers Danlos Syndrome has hyper mobile joints. C. Loeys Dietz Syndrome doesn’t have
soft skin or hernias. D. Marfan Syndrome doesn’t have soft skin or normal joint mobility.
E. Pseudoxanthoma Elasticum does not have myopia or lax skin and has emphysema.
C92.
The correct answer is D.
Ralph has chronic hip and knee pain and decreased ROM of his hips and knees. His
mother had early hip and knee replacements. This is most consistent with Multiple
Epiphyseal Dysplasia. A. Ehlers-Danlos syndrome has increased ROM of joints. B.
Hypochondroplasia has short stature. C. Marfan Syndrome has scoliosis and infrequently
requires hip or knee replacements. E. Osteogenesis Imperfecta presents with recurring
fractures rather than chronic pain.
C93.
The correct answer is D.
SOURCE: Lecture/Slides
KEYWORDS: Ectodermal dysplasia
729
EXPLANATION: The child’s abnormal teeth and nails, with normal skin, hair, and
sweating, are compatible with Witkop syndrome, associated with MSX1 mutations.
EDA and EDAR are both associated with hypohidrotic ectodermal dysplasia. GJB6 is
associated with Clouston syndrome, which leads to hyperkeratosis and abnormal hair in
addition to abnormal nails. WNT10A is associated with Odonto-Onycho-Dermal
Dysplasia, which also leads to hair and skin abnormalities.
C94.
CORRECT ANSWER: A
SOURCE OF ITEM TOPIC: Lecture/Slides
KEYWORDS: Amenorrhea, blind vagina and testosterone.
EXPLANATION: Complete androgen insensitivity is associated with 46, XY; absent or
rudimentary müllerian structures, normal or elevated testosterone , dihydrotestosterone &
luteinizing hormone and it is XL due to AR variants. CAH would have high 17-OHP
levels, LH deficiency causes low testosterone levels, Turner syndrome and 46, XY sex
reversal females, SRY related should not have a blind vagina or absent uterus.
C95.
CORRECT ANSWER: A
SOURCE: Lecture/Slides
KEYWORDS: Hypophosphatasia
EXPLANATION: The child’s short stature, rachitic skeletal changes and a low serum
ALP are all typical of Hypophosphatasia and craniosynostosis is often found in the
perinatal or infantile form. Hypercalcemia, hypercalciuria, low bone density and short
stature are typical rather than hypocalcemia, hypocalciuria, osteoporosis and tall stature.
C96.
CORRECT ANSWER: A
SOURCE: Lecture/Slides
KEYWORDS: epidermolysis bullosum
EXPLANATION: Dystrophic epidermolysis bullosum with scarring in the dermis can be
AD or AR and is associated with COL7A1 mutations. LAMB3 is associated with
junctional EB, where scarring is at the basement membrane, and the others with EB
simplex, with scarring in the basal layer.
C97.
CORRECT ANSWER: A
SOURCE: Lecture/Slides
KEYWORDS: Microcephaly, telecanthus, coarse facies, genital anomalies, hypotonia,
intellectual disability & thalassemia.
EXPLANATION: ATRX pathogenic variants cause Alpha Thalassemia XL Intellectual
Disability Syndrome. The other genes encode alpha, beta or gamma globin.
C98.
CORRECT ANSWER: C
SOURCE OF ITEM TOPIC: Lecture/Slides
KEYWORDS: Russell-Silver syndrome, imprinting
EXPLANATION: Hypomethylation of the paternal IC1 site explains about 45% of cases
of Russell-Silver syndrome. Hypomethlation leads to blockage of expression of the IGF2
locus. The other mechanisms are all associated with Beckwith-Wiedemann syndrome.
730
D. Neurological Genetics Questions D1-D40
D1.
A 10-year-old boy visits an ophthalmologist for the first time after complaining to his parents that
he is having trouble seeing the interactive whiteboard at school. The ophthalmologist finds a
mild refractive error, but is most concerned about finding Lisch nodules. The presence of iris
Lisch nodules is a helpful diagnostic feature of which of the following syndromes?
A.
B.
C.
D.
E.
D2.
A young girl with new onset seizure disorder is examined by a neurologist and found to have
non-traumatic periungual and subungual fibromas. These clinical features are a primary
diagnostic feature of which of the following syndromes?
A.
B.
C.
D.
E.
D3.
Gardner syndrome
Neurofibromatosis type 1
Sturge-Weber syndrome
Tuberous sclerosis complex
von Hippel-Lindau disease
A 30-year-old woman with a complicated medical history dies of renal cell carcinoma. Renal cell
carcinoma is a primary cause of death in which of the following syndromes?
A.
B.
C.
D.
E.
D4.
Gardner syndrome
Neurofibromatosis type 1
Sturge-Weber syndrome
Tuberous sclerosis complex
von Hippel-Lindau disease
Gardner syndrome
Neurofibromatosis type 1
Sturge-Weber syndrome
Tuberous sclerosis
von Hippel-Lindau disease
Two sisters are affected with cataracts in infancy, intellectual impairment, microcephaly,
nystagmus, and moderate growth deficiency. Which of the following syndrome is the most likely
diagnosis?
A.
B.
C.
D.
E.
Abetalipoproteinemia
Ataxia telangiectasia
Friedreich ataxia
Marinesco-Sjogren syndrome
Type 3 Gaucher disease
731
D5.
A 40-year-old man presents with pain in his leg associated with a palpable mass. Biopsy reveals
a schwannoma. He previously had a schwannoma removed from his arm. An MRI of the brain
shows no abnormality and an ophthalmological examination is normal. There is no known family
history of similar problems. Molecular analysis of which one of the following genes would be
most likely to reveal a mutation that might explain his phenotype?
A.
B.
C.
D.
E.
D6.
A 45 year old presents with ringing in the ears and hearing loss. He is found to have a vestibular
schwannoma. Concerned that he might have NF2, you arrange an eye examination. Which of the
following would be supportive of this diagnosis?
A.
B.
C.
D.
E.
D7.
maternal inheritance of the CAG repeat expansion
maternal uniparental disomy of the CAG repeat expansion
new mutation of the CAG repeat expansion
paternal inheritance of the CAG repeat expansion
paternal uniparental disomy of the CAG repeat expansion
Which of the following disorders is associated with duplication of gene on chromosome 17?
A.
B.
C.
D.
E.
D9.
Lisch nodules
optic atrophy
papilledema
posterior subcapsular cataract
thickened corneal nerve
An eight-year-old girl develops signs and symptoms of Huntington disease, presenting with
rigidity. Early onset is most likely the result of which of the following genetic abnormalities?
A.
B.
C.
D.
E.
D8.
NF1
NF2
SPRED1
INI1
PTPN11
Charcot-Marie-Tooth disease
Hereditary liability to pressure palsies
Miller-Dieker syndrome
NF1
Spinal muscular atrophy
Which of the following is the most common mutation responsible for infantile spinal muscular
atrophy?
A.
B.
C.
D.
E.
gene conversion of SMNT to SMNC
deletion of NAIP gene
deletion of SMNT gene
point mutation of SMNT gene
duplication of SMNC gene
732
D10.
Which of the following genes includes a polymorphism that is associated with risk of Alzheimer
disease?
A.
B.
C.
D.
E.
D11.
You are seeing a child with tuberous sclerosis and examine both parents. The child's mother is
found to also have hypopigmented macules. Genetic testing is performed and reveals a TSC2
mutation in both mother and child. The mother reports being in good health, with no history of
seizures, normal development, and normal activity. Which of the following tests is most likely to
be normal even if she does have tuberous sclerosis complex?
A.
B.
C.
D.
E.
D12.
Basal cell nevus syndrome
NF1
NF2
TSC1
TSC2
The most common autosomal dystonia 1 (DYT1) mutation is found in which ethnic group?
A.
B.
C.
D.
E.
D14.
brain MRI
pulmonary CT
ophthalmological examination
echocardiogram
renal ultrasound
Which of the following disorders is most likely to be associated with renal cysts?
A.
B.
C.
D.
E.
D13.
amyloid precursor protein
apoB-48
apoE
presenilin
prion protein
African-Americans
Ashkenazi Jews
Finns
Mennonites
Southeast Asians
Pantothenate kinase associated neurodegeneration is associated with basal ganglia accumulation
of which of the following metals?
A.
B.
C.
D.
E.
Calcium
Copper
Iron
Magnesium
Zinc
733
D15.
The triplet repeat expansion responsible for Huntington disease is located in which of the
following areas of the Huntingtin gene?
A.
B.
C.
D.
E.
.
D16.
Individuals with CADASIL are at risk for which of the following complications?
A.
B.
C.
D.
E.
D17.
abnormal methylation
chromosomal abnormality
gene conversion
single base pair mutation
triplet repeat expansion
The CTG repeat expansion responsible for myotonic dystrophy affects which of the following
molecular processes for the DMPK gene or gene product?
A.
B.
C.
D.
E.
D19.
Berry aneurysm
cavernous hemangiomas
hemangioblastomas
multiple strokes
stroke-like episodes
A newborn has weakness, hypotonia, absent reflexes, and tongue fasciculations. Which of the
following genetic mechanisms is most likely responsible for this clinical presentation?
A.
B.
C.
D.
E.
D18.
at an intron-exon border
in an exon
in an intron
in the 3’ untranslated region
near the promoter
Level of transcription
protein degradation in the proteosome
RNA binding protein
RNA editing
RNA splicing
A 55-year-old man is seen in Neurology Clinic because of unsteady gait. On examination he is
found to have ataxia and a tremor. Brain MRI reveals white matter lesions in the basal ganglia.
The patient has no children, but he has a sister who has a son with impaired cognitive
development. Genetic testing of the patient is most likely to reveal which of the following
findings in the FMR1 gene?
A.
B.
C.
D.
E.
37 CAG repeats
45 CAG repeats
60 GAA repeats
75 CGG repeats
250 CGG repeats
734
D20.
The most likely mutation in an individual with hereditary neuropathy with predisposition to
pressure palsies is the result of which of the following molecular mechanisms?
A.
B.
C.
D.
E.
D21.
A 17-year-old man is evaluated for weakness in the lower extremities that has been gradually
getting worse over the past 3 years. He is found to have pes cavus and his knee and ankle jerk
reflexes cannot be elicited. His exam is otherwise normal. Family history reveals that his father
was affected similarly. Which of the following diagnoses is most likely to explain his clinical
findings?
A.
B.
C.
D.
E.
D22.
A cleft in the brain
A double band of cortex
Lack of cerebral gyri
Large numbers of small cerebral gyri
Small numbers of large cerebral gyri
Mutations in the D-synuclein gene are most likely to result in which of the following neurologic
findings?
A.
B.
C.
D.
E.
D24.
Charcot-Marie-Tooth disease
Friedreich ataxia
Hereditary amyotrophic lateral sclerosis
Limb-girdle muscular dystrophy
Myotonic dystrophy
Lissencephaly is characterized by which of the following anatomic abnormalities?
A.
B.
C.
D.
E.
D23.
Single base change
Deletion
Duplication
Inversion
Imprinting change
Ataxia
Dementia
Spasticity
Tremor
Weakness
A 60-year-old man presents to neurology clinic with a history of late-onset, progressive cerebellar
ataxia and intention tremor. A family history reveals his daughter has a son with fragile X
syndrome. Which of the following alterations in the FMR1 gene is most likely to be found in this
patient?
A.
B.
C.
D.
E.
Point mutation
30 CGG repeats
80 CGG repeats
250 CGG repeats
Deletion of the FMR1 gene
735
D25.
You see a couple for genetic counseling because of a family history of Duchenne muscular
dystrophy. See pedigree below. She is now 30 weeks pregnant and ultrasound reveals a male;
she asks whether it will be possible to test her son at birth for the disorder. Which of the
following is the best way to establish whether the child is affected at birth?
A.
B.
C.
D.
E.
D26.
Which of the following neuromuscular diseases is associated with deletion of a repeated sequence
in the chromosome 4q subtelomeric region?
A.
B.
C.
D.
E.
D27.
physical examination
testing the dystrophin gene for intragenic deletions
sequencing the dystrophin gene
muscle biopsy
serum creatine phosphokinase testing
Charcot-Marie-Tooth Disease
Congenital fiber type disproportion
Duchenne muscular dystrophy
Facio-scapulo-humeral muscular dystrophy
Limb girdle muscular dystrophy
Patients with basal cell nevus syndrome are most likely to develop which of the following brain
tumors?
A.
B.
C.
D.
E.
Glioblastoma
Hemangioblastoma
Medulloblastoma
Optic glioma
Pilocytic astrocytoma
736
D28.
Which of the following disorders results from a triplet repeat expansion in the 3’ untranslated
region of the causative gene?
A.
B.
C.
D.
E.
D29.
An individual at risk for Huntington disease is found to have 40 CAG repeats. Which of the
following allele types best describes the implications of this result?
A.
B.
C.
D.
E.
D30.
Fragile X syndrome
Friedreich ataxia
Huntington disease
Myotonic dystrophy
Spinocerebellary ataxia
Disease allele
Intermediate allele
Normal allele
Reduced penetrance allele
Polymorphic allele
Which of the following is associated with mutation in a GTPase activating protein?
A.
B.
C.
D.
E.
NF1
NF2
TSC1
TSC2
Schwannomatosis
D31. Which of the following movement disorders is associated with a GAG deletion?
A.
B.
C.
D.
E.
D32.
Parkinson disease
Huntington disease
Spinocerebellar ataxia
Tourette syndrome
Torsion dystonia
Notch3 gene mutations cause which of the following neurologic complications?
A.
B.
C.
D.
E.
Ataxia
Deafness
Dystonia
Seizures
Strokes
737
D33.
First-time parents report that their 8-month-old infant son has recurrent vomiting episodes,
feeding problems and does not appear to shed tears when upset and crying. Which of the
following neurologic disorders is the most likely diagnosis?
A.
B.
C.
D.
E.
D34.
Myasthenia gravis affects which of the following components of the neuromuscular system?
A.
B.
C.
D.
E.
D35.
Central core myopathy
Duchenne muscular dystrophy
Myotonic dystrophy
Nemaline myopathy
Spinal muscular atrophy
A 7-year-old with NF1 develops an acute left hemiparesis. Which of the following clinical
findings is the most likely cause of acute stroke in NF1?
A.
B.
C.
D.
E.
D37.
Cardiac muscle
Extracellular matrix surrounding muscle
Muscle cytoskeleton
Neuromuscular junction
Sarcomere
A massively elevated CK (creatine kinase) level is found in a 4-year-old boy with progressively
worsening gross motor delay. This finding is typical for which of the following neuromuscular
disorders?
A.
B.
C.
D.
E.
D36.
Charcot-Marie Tooth disease
Familial dysautonomia
Friedreich ataxia
Hereditary dystonia
Von Hippel Lindau syndrome
Blood clotting abnormality
Cardiac rhabdomyoma
Embolus from cardiac arrhythmia
Moyamoya syndrome
Optic glioma
You are following a 30-year-old pregnant woman with von Hippel-Lindau syndrome.
Which of the following findings is the most important to monitor her for during the
pregnancy?
A.
B.
C.
D.
E.
Endolymphatic sac tumor
Pheochromocytoma
Renal Cancer
Retinal hemangioblastoma
Vestibular schwannoma
738
D38.
A 50 year old man presents with pain and is found to have multiple schwannomas. A brain MRI
shows no evidence for vestibular schwannoma. Which pair of genes would be appropriate to test
to determine whether he might have schwannomatosis?
A.
B.
C.
D.
E.
D39.
A 50-year-old woman has a direct-to-consumer genetic test that reveals she is homozygous for
the E4 allele of ApoE. Which of the following conditions has the highest relative risk of
occurring given this result?
A.
B.
C.
D.
E.
D40.
NF1 and NF2
NF2 and TSC2
NF1 and TSC2
TSC2 and LZTR1
SMARCB1 and LZTR1
Alzheimer disease
Ataxia telangiectasia
Hereditary spastic paraplegia
Parkinson disease
von Hippel-Lindau syndrome
After his younger sister (III-2) was diagnosed as having Gaucher disease, a 25-year-old man
(III-I) is found to be carrier for Gaucher disease as seen in the pedigree below. Which of the
following conditions has the highest relative risk given his carrier status?
A. Alzheimer disease
B. Charcot-Marie Tooth Disease
C. Myotonic Dystrophy
D. Parkinson Disease
E. PKAN Associated Neurodegeneration
D41.
A 40-year-old man presents with ataxia and MRI reveals a cerebellar hemangioblastoma. He is
also found to have a retinal hemangioblastoma by ophthalmological exam and sensorineural
hearing loss by audiology. Which of the following lesions is the most likely to be responsible for
his hearing loss?
A.
B.
C.
D.
E.
brainstem hemangioblastoma
endolymphatic sac tumor
glioma
meningioma
vestibular schwannoma
739
Answers to Neurological Genetics Questions D1-D40
D1.
Answer: B. Lisch nodules are a very helpful clinical feature of neurofibromatosis type 1
and are an important part of family evaluations.
D2.
Answer: D. The primary diagnostic criteria for tuberous sclerosis include cortical
hamartomas, multiple retinal hamartomas, and angiofibromas or periungual fibromas.
Other criteria include infantile spasms, hypomelanotic patches, single retinal hamartoma,
subependymal or cortical calcifications, multiple renal tumors, cardiac rhabdomyoma,
and first-degree relative with TS.
D3.
Answer: E. The von Hippel-Lindau disease has the most significant association with
renal cell carcinoma. Retinal angiomas, angiomas of the cerebellum, and
pheochromocytomas are other features.
D4.
Answer: D. This is a typical clinical description for Marinesco-Sjogren syndrome.
Ataxia telangiectasia would be characterized by the telangiectasia and the absence of
cataracts. The onset would be later with less mental impairment early on. Friedreich
ataxia would have a later onset and not be associated with many of the findings listed
here. Type 3 Gaucher disease would not be associated with cataracts or early growth
deficiency and would likely have associated splenomegaly. Abetalipoproteinemia would
have a quite different presentation with none of these dysmorphic features.
D5.
Answer D. The occurrence of an multiple schwannomas presenting with pain is most
typical of schwannomatosis, associated with INI1 mutation. Schwannomas could also be
associated with NF2, but one would expect bilateral vestibular schwannomas and cataract
in individuals with NF2.
D6.
Answer: D. Posterior subcapsular cataract is characteristic of NF2; other features listed
are typical of NF1 (papilledema is optic nerve swelling in the setting of increased
intracranial pressure).
D7.
Answer: D. Paternal inheritance in some cases is associated with the juvenile
presentation of Huntington disease, with rigidity.
D8.
Answer: A. Chariot-Marie Tooth disease is associated with PMP22 duplication;
hereditary neuropathy with liability to pressure palsies is associated with deletion of this
gene.
740
D9.
Answer: C. Deletion of the SMNT gene is associated with SMA; sometimes the mutation
consists of gene conversion of SMTC to SMNT
D10.
Answer: C.
D11.
Answer D. Adults with tuberous sclerosis complex are unlikely to have cardiac
rhabdomyoma, since this lesion tends to regress with time. The other studies are
indicated for screening, even in the absence of signs or symptoms.
D12.
Answer: E. Renal cysts occur when a contiguous deletion occurs between the TSC2 and
PKD1 genes.
D13.
Answer: B.
D14.
Answer: C.
D15.
Answer: B. The triplet repeat is within an exon and encodes a polyglutamine tract in the
protein.
D16.
Answer: D.
D17.
Answer: C. This is a description of spinal muscular atrophy, which in some cases is due
to a gene conversion event of SMNC to SMNT
D18.
Answer: C.
D19.
Answer D. The patient has signs of FXTAS (fragile X tremor-ataxia syndrome),
associated with premutation expansion of the CGG repeat in the FMR1 gene.
D20.
Answer: B. Hereditary neuropathy with predisposition to pressure palsies is due to PMP22
deletion; Charcot-Marie-Tooth disease occurs when the gene is duplicated.
D21.
Answer A. Pes cavus and lack of deep tendon reflexes is suggestive of peripheral
neuropathy, and the family history suggests dominant inheritance. Friedreich ataxia
would produce other signs and is recessive; the exam is not suggestive of the other
diagnoses.
D22.
Answer: C. Lissencephaly is characterized by a smooth brain surface without gyri.
Polymicrogyria is the occurrence of a large number of small gyri; pachygyria is the occurrence of
a small number of large gyri; schizencephaly is the occurrence of a cleft in the brain.
741
D23.
Answer: D. D-synuclein mutations result in an inherited form of Parkinson disease, which is
characterized by tremor, rigidity, and bradykinesia.
D24.
Answer: C. Fragile X tremor/ataxia syndrome occurs in males with FMR1 premutation alleles.
D25.
Answer E. Creatine phosphokinase levels will be elevated at birth for a boy with
Duchenne muscular dystropy, well before muscle weakness will be apparent. Genetic
testing could be done, but the family mutation is not known, so a negative test will be
difficult to interpret, and should not be necessary. Physical exam will be normal at this
age and muscle biopsy, besides being invasive, will likely miss pathology this early in
life
D26.
Answer: D. FSH dystrophy is due to deletions of a repeat region in the 4q subtelomere.
D27.
Answer: C. Optic gliomas (which are pilocytic astrocytomas) are characteristic of NF1;
hemangioblastomas are seen in von Hippel Lindau syndrome.
D28.
Answer: D. Myotonic dystrophy is due to a CTG repeat expansion in the 3’ untranslated region
of the DMPK gene. The expansion for fragile X occurs in the promoter region; for Friedreich
ataxia in an intron; for Huntington disease and spinocerebellar ataxia in exons.
D29.
Answer: A. Normal alleles are 10-26 repeats; intermediate alleles 27-35 repeats; disease
alleles are >36 repeats with reduced penetrance from 36-39.
D 30. Answer: A. The NF1 gene product, neurofibromin, is a GTPase activating protein for
Ras.
D31.
Answer E. Torsion dystonia is most commonly associated with GAG deletion in the
DYT1 gene.
D32.
Answer: E. Notch3 mutations are associated with CADASIL (cerebral autosomal
dominant arteriopathy with subcortical infarcts and leukoencephalopathy).
D33.
Answer B. Familial dysautonomia includes absent tearing, autonomic neuropathy,
episodic vomiting, feeding disorder, and absent fungiform papillae.
D34.
Answser: D. Myasthenia gravis is a disorder of the neuromuscular junction; usually it is
autoimmune, but there are rare inherited forms.
D35.
Answer: B. Duchenne and Becker dystrophies are associated with major CPK
elevations. The other disorders either do not cause elevation, or the elevation is typically
minor.
D36.
Answer: D.
Keywords: NF1, stroke, vascular
Explanation: Individuals with NF1 are at increased risk of internal carotid occlusion and
associated moyamoya syndrome.
742
D37.
Answer B.
Keywords: von Hippel-Lindau syndrome
Explanation: Women with VHL should be followed during pregnancy for development
of pheochromocytoma. The other matters are not specifically critical during pregnancy,
and vestibular schwannoma is not a feature of VHL.
D38.
Answer E.
Keywords: neurofibromatosis, schwannomoatosis
Explanation: SMARCB1 and LZTR1 are both associated with schwannomatosis.
Schwannomas can be found in NF2 patients, but usually in association with vestibular
schwannomas and other features. TSC2 is associated with tuberous sclerosis complex,
not schwannomatosis.
D39.
CORRECT ANSWER: A
SOURCE OF ITEM TOPIC: Lecture/Syllabus
KEYWORDS: Alzheimer disease, ApoE
EXPLANATION: The ApoE E4 allele is associated with an increased risk of Alzheimer
disease. None of the other conditions is associated with the ApoE4 genotype.
D40.
CORRECT ANSWER: D
SOURCE OF ITEM TOPIC: Lecture/Slides
KEYWORDS: Gaucher, glucocerebrosidase, Parkinson disease
EXPLANATION: Having one N370S glucocerebrosidase allele has been indicated as a
risk factor for Parkinson disease. None of the other conditions is associated with carrier
status for glucocerebrosidase.
D41.
CORRECT ANSWER: B
SOURCE OF ITEM TOPIC: Lecture/Slides
KEYWORDS: von Hippel-Lindau, hearing loss
EXPLANATION: The presence of cerebellar and retinal hemangioblastoma is indicative
of von Hippel-Lindau syndrome, which is also associated with endolymphatic sac tumor,
that can cause hearing loss. A brainstem hemangioblastoma is not likely to cause hearing
loss, though it could occur in VHL. The other tumors are not associated with VHL;
vestibular schwannoma is typical for NF2.
743
E. Molecular Genetics Questions E1-E79
E1.
You have cloned a candidate gene for an autosomal recessive disorder. You have identified a
dinucleotide repeat polymorphism in intron 1 of the gene. Solid symbol is affected. Three alleles
for the repeat are found in this family.
Which of the following assessments are you most likely to provide regarding the likelihood that
the gene you identified is the disease gene in this family?
A.
B.
C.
D.
E.
Very encouraging
Slightly encouraging
No evidence either way
Slightly discouraging
Virtually rule it out
1
2
3
E2.
The diagram below represents linkage analysis for an autosomal dominant disorder where the
grandfather is deceased. A dinucleotide repeat with a marker within the disease gene is shown
with three alleles. What is the probability that the fetus (P) is affected by this disease?
A.
B.
C.
D.
E.
1.0
0.75
0.50
0.25
0.0
P
1
2
3
E3.
For this CF family, DNA results are as indicated. Assume analysis for 20 mutations with
detection of 90% CF mutant chromosomes. Neg = negative for 20 mutations. Assume carrier
frequency of 1 in 25. Which of the following probabilities represents the likelihood that
individual #6 is a carrier based on the data shown?
A.
B.
C.
D.
E.
1
1in 7
1 in 11
1 in 16
1 in 20
1 in 24
5
2
6
Neg
744
3
4
7
E4.
For this CF family, DNA results are as indicated. Assume analysis for 20 mutations with
GHWHFWLRQRI&)PXWDQWFKURPRVRPHVǻ) ǻ)1HJ QHJDWLYHIRUPXWDWLRQV8. =
unknown (untested) CF allele, Nl = presumptive normal allele. Assume carrier frequency of 1 in
25. What is the probability that individual #6 is a carrier based on the data shown?
A.
B.
C.
D.
E.
1
1 in 2
1 in 4
1 in 9
1 in 10
1 in 11
5
2
3
6
Neg
4
7
'F/Nl
UK/Nl
'F/UK
E5.
A dinucleotide repeat analysis for a polymorphism within a gene is analyzed for an autosomal
recessive disorder as shown below. What is the probability that the pregnancy (P) is affected?
A.
B.
C.
D.
E.
1.0
0.75
0.50
0.25
0.0
P
1
2
3
E6
A 20-year-old male presents with proximal muscle weakness but is ambulatory, has frequent
falls, and has significant fatigue during mild exercise. SMN1 deletion analysis is ordered, and
testing is done using quantitative PCR analysis. The results indicate that the patient has one
SMN1 allele detected. Which of the following rationales provides the best plan of action in
response to this result?
A. The patient has a both a gene deletion and a point mutation (or small deletion/insertion) and
should have SMN1 gene sequence analysis.
B. The patient has juvenile amyotrophic lateral sclerosis and would benefit by testing ALS2
gene.
C. The patient has X-linked spinal muscular atrophy and should be tested for a mutation in
UBA1 gene.
D. The patient is deleted for both copies of SMN2 and should be tested by MLPA to more
accurately assess number of gene copies.
E. The patient is deleted for both SMN1 genes but has a false negative test due to interference of
multiple copies of the pseudogene SMN2 an no additional testing is warranted.
745
E7.
Fred is an 8-year-old boy referred because his physician was concerned he might have Fragile X
Syndrome (FXS). The lab report of Fred’s FXS study says that his DNA yielded hybridizing
fragments of 5.2 and 2.8 kbs indicating methylated and unmethylated FMR1 alleles. Which of the
following explanations accounts for these test results?
A.
B.
C.
D.
E.
E8.
DNAs from the patient and another individual were swapped
The patient has a normal FXS test result
The patient has Klinefelter Syndrome
The patient is mosaic for Fragile X Syndrome
The restriction endonuclease digestion was incomplete
A DNA analysis using a dinucleotide repeat within a gene for an autosomal dominant disorder is
presented below. The father and deceased grandfather are affected. . What is the probability that
the pregnancy (P) is affected?
A.
B.
C.
D.
E.
1.0
0.75
0.5
0.25
0.0
P
1
2
3
E9.
This is a family with X-Iinked Duchenne muscular dystrophy. A Southern blot is depicted using a
fragment of the cDNA for the gene as a probe. Bands indicate presence or absence with no
attempt to distinguish intensity (dosage). Assume no gonadal mosaicism. Which of the following
conclusions can be drawn about this family?
A.
B.
C.
D.
E.
It is uncertain if the mother is a
carrier or not
The mother and only the mother
is a carrier
The mother and the grandmother
are carriers
The mother is definitely not a
carrier
The mother, the grandmother
and the aunt are all carriers.
Normal male
and female
pattern
746
E10.
Protein truncation testing is best suited for detecting which of the following types of mutations?
A.
B.
C.
D.
E.
E11.
triplet repeat expansions.
promoter mutations.
missense mutations.
large deletions.
frameshift mutations.
A researcher has a candidate gene for an autosomal recessive disorder and studies a dinucleotide
repeat within intron 1 in the family depicted below. What are the odds favoring that this is the
disease gene relative to the chances of a result such as this favoring linkage occurring by chance?
A.
B.
C.
D.
E.
16 to 1
32 to 1
64 to 1
128 to 1
256 to 1
AB
AC
E12.
AC
AC
AC
The R117H mutation (Arg to His at position 117) in the cystic fibrosis gene is associated with
considerable phenotypic heterogeneity. This is explained at least in part by which of the
following types of polymorphism?
A.
B.
C.
D.
E.
E13.
AC
CD
A polymorphism involving a splice site.
A polymorphism involving the polyadenylation site.
A polymorphism involving the promoter.
A polymorphism involving the signal sequence.
A polymorphism mapping to a locus other than CFTR.
Mary is 35-year-old and has a 4-year-old boy with developmental delay and large ears. She has 6year-old daughter with attention deficit hyperactivity disorder. Mary is being seen by her OB for
ovarian insufficiency. Her father at age 61 has recently developed tremors. Which of the
following explanations accounts for the clinical information found in this family history?
A. Her father and children have a point mutation in the FMR1 gene that shows variable
expressivity.
B. Her father and children have unrelated disorders, Parkinson disease and nonspecific
developmental delay.
C. Her father and her daughter have small CTG expansions in the premutation range, while her
son has a full CTG expansion, demonstrating anticipation.
D. Her father carries a premutation of >80 CGG repeats and has FXTAS, while the son has
inherited an expansion of >200 CGG repeats and has Fragile X.
E. Her father carries a reduced penetrance CAG repeat expansion, explaining his milder
symptoms and later age of onset.
747
E14.
A patient’s malignant melanoma biopsy specimen is tested for the BRAF V600E mutation to
guide chemotherapy. Which of the following findings would be the most likely cause of a false
negative result?
A.
B.
C.
D.
E.
E15.
The genetic length of the human genome is best represented by which of the following
pairings?
A.
B
C
D.
E.
E16.
8800 cM for male map and 2700 for female map.
8800 cM for male map and 5400 for female map.
8800 cM for female map and 5400 for male map.
4400 cM for male map and 2700 for female map.
4400 cM for female map and 2700 for male map.
A woman has a brother and son affected with Duchenne dystrophy. No deletion could be found,
so linkage analysis was used to test her current pregnancy (P) which is a male fetus. An STR at
the 5' end of the gene and another at the 3' end of the gene were tested as shown below. What is
the best estimate of the probability that the fetus is affected?
A.
B.
C.
D.
E.
E17.
A chromosomal deletion
A chromosomal duplication
A low level of somatic mosaicism
A sample mix up
A trinucleotide repeat expansion
close to 0%
close to 25%
close to 50%
close to 75%
close to 100%
P
5’STR alleles
3’STR alleles
1
5
1/2
5/6
1
5
1
6
Mutations can affect the function of a gene in a number of ways. Which of the following is the
term used for a mutation that causes substitution of a single amino acid?
A.
B.
C.
D.
E.
frameshift
in-frame deletion
missense
nonsense
splicing
748
E18.
ȕ-thalassemia and cystic fibrosis are autosomal recessive disorders in which a large number of
mutations have been discovered. Which of the following characteristics of a gene is most useful
in designing a sensitive mutation assay?
A.
B.
C.
D.
E.
E19.
Indirect detection of an abnormal gene can be used for diagnosis in situations where the diseaseproducing mutation is unknown. Which of the following characteristics of a DNA marker is most
important for accuracy in linkage studies?
A.
B.
C.
D.
E.
E20.
can be detected using PCR
highly polymorphic
low recombination rate with disease locus
physical distance from disease gene is known
polymorphisms are absent at the marker site
Which of the following investigative assays would be the most appropriate method to use for
testing 50 variants including point mutations, deletions and insertions in 10 different genes as a
screening test for the reproductive population?
A.
B.
C.
D.
E.
E21.
distribution frequency of mutations among different ethnic/racial groups
distribution frequency of mutations among exons
frequency of missense, nonsense, frameshift and splicing mutations
frequency of single nucleotide polymorphisms with a gene.
frequency of structural rearrangements (e.g.insertions/deletions)
Fluorescence Resonance Energy Transfer
Liquid Bead Array
Multiplex Ligation-dependent Probe Amplification
Next generation sequencing
Sanger sequencing
You have clinically diagnosed achondroplasia in an infant whose parents are unaffected. You
submit the infant’s DNA for molecular analysis. The laboratory reports an FGFR3 GLY380ARG
(GGG to AGG) substitution due to an 1138G-A change. This change was reported in 96% of 17
sporadic and 6 familial cases of achondroplasia studied. Which of the following mechanisms is
most likely to have caused this change in the DNA?
A.
B.
C.
D.
E.
A trinucleotide repeat expansion
Gene conversion
Methylation of a CG
Recombination
Slipped misspairing
749
E22.
In order to test for the homozygous deletion in patients affected with SMA which of the following
procedures must be accomplished initially?
A.
B.
C.
D.
E.
E23.
Which of the following is the most accurate test for the diagnosis of Fragile X syndrome?
A.
B.
C.
D.
E.
E24.
Allele specific amplification
Chemical cleavage
Protein truncation
Reverse dot blotting
SSCP
Which of the following characteristics is the most likely mechanism of action of the triplet repeat
expansion in Huntington disease?
A.
B.
C.
D.
E.
E26.
Allele specific oligonucleotides
Allele specific PCR
Protein truncation
Quantitative PCR
Southern blotting
Which of the following molecular methods is a non-gel based assay for the detection of point
mutations?
A.
B.
C.
D.
E.
E25.
perform multiplex PCR for SMN exons 1-4
distinguish the telomeric (SMN1) SMN gene from the centromeric SMN (SMN2) gene
perform a dosage assay on the SMN1 gene
use Southern blotting to identify the common SMN1 junction fragment
determine a loss of heterozygosity using closely linked markers inside the SMN1 gene
affects the overall chromatin configuration
decreases gene methylation
inhibits gene expression
inhibits protein translation
works by a gain of function
Advantages of the microsatellite repeats compared to restriction fragment length polymorphisms
for indirect testing include which of the following characteristics?
A.
B.
C.
D.
E.
Microsatellites repeats have negligible recombination errors
Microsatellites repeats are intronic
Microsatellites repeats are multi-allelic
Microsatellites are invariable in length
Microsatellites repeats are amenable to PCR detection
750
E27.
A 50-year-old woman with aplastic anemia was found to have a nonsense and a silent mutation in
the FANCA gene, confirming the diagnosis of Fanconi anemia. All of her siblings were tested for
both mutations as potential bone marrow donors for transplant. Her asymptomatic older sister was
found to have both mutations in her fibroblasts but only the silent mutation in her blood. Which
of the following explanations provides the most likely reason for this finding?
A. A gene conversion event has occurred in the asymptomatic sister’s fibroblast cells.
B. The asymptomatic sister has had a reversion event in her bone marrow, explaining the
difference in blood.
C. The asymptomatic sister is not affected with Fanconi anemia and therefore no further
monitoring is required.
D. The diagnosis is incorrect, because the silent mutation does not change an amino acid in the
protein and thus, it is really a benign polymorphism.
E. There is a lab error in her sister’s blood test result because these are germline variants and all
tissues within each individual will carry the same variants.
E28.
You submitted a DNA sample to determine if there is a mutation in an arginine residue “hotspot”
in the NRAS gene. Which of the following alterations is most likely to result from a substitution
that changes this arginine codon from AGG to ATG?
A.
B.
C.
D.
E.
E29.
Which of the following codons each result the in termination of translation?
A.
B.
C.
D.
E.
E30.
AUG & UAA
AUG, UAA, UGA & UAG
UUU, UAG, UGA & UAA
UUU, UAA & UAG
UGA, UAA & UAG
Which of the following genes is commonly associated with intragenic inversion?
A.
B.
C.
D.
E.
E31.
Creates a frameshift downstream
Creates a nonsynonymous substitution
Creates a start codon
Creates a synonymous substitution
Creates a termination codon
Factor VIII deficiency
Factor IX deficiency
Factor XI deficiency
Von Willebrand disease
Factor V Leiden
Which of the following events typically occurs when ultraviolet light damages DNA?
A.
B.
C.
D.
E.
Double strand breaks.
Purines dimerize within a strand
Purines dimerize between strands.
Pyrimidines dimerize within a strand
Single strand breaks.
751
E32.
You have performed linkage analysis using restriction fragment length polymorphisms (RFLPs) to
follow the inheritance pattern of defective beta-globin genes in a family with beta-thalassemia
(shown below). Assuming that the RFLP does not show recombination with the beta-globin locus,
what is the most likely diagnosis for the daughter (indicated by the open circle) who is at risk for
this autosomal recessive disorder?
A.
B.
C.
D.
E.
E33.
The patient has a fully penetrant HD allele.
The patient has a nonpenetrant allele, ruling out the diagnosis of HD.
The patient has a reduced penetrance allele, confirming the diagnosis of HD.
The patient’s personality disorder is caused by another gene defect.
This result is normal so another disorder must be responsible.
Mrs. Soft has a history of multiple fractures, mild short stature and hearing loss. She brings her
two-month-old son, Carl, to you for evaluation. Carl has no history of fractures. He has a normal
physical exam. Which of the following evaluations is the most accurate way to determine if Carl
has the same condition as his mother?
A.
B.
C.
D.
E.
E35.
?
A 54-year-old woman with mild neurological symptoms of ataxia and personality disorder was
found to have a 38 CAG repeat in the huntingtin (HTT ) gene. Which of the following
explanations provides the most likely reason for this finding?
A.
B.
C.
D.
E.
E34.
Daughter is heterozygous for beta thalassemia
Daughter is heterozygous for a beta globin gene deletion
Daughter is homozygous normal
Pattern indicates non-paternity
The results are not informative
Determine if Carl has a mutation in his COL1A1 or COL1A2 genes
Determine if Carl has an abnormal ABR or OAE test
Determine if Carl’s skull radiographs demonstrate Wormian bones
Determine if Mrs. Soft and Carl share a COL1A1/COL1A2 mutation
Determine if radiographs of Carl’s extremity bones show fractures
Core promoter elements that direct transcription include which of the following motifs?
A.
B.
C.
D.
E.
CAAT and Acceptor splice sequences
CAAT box and Initiator sequence
CAAT and TATA boxes and Poly-A addition signal
TATA box and Branch point consensus
TATA boxes and Donor splice sequences
752
E36.
Trans-acting transcriptional regulatory sequences include which of the following DNA
components?
A.
B.
C.
D.
E.
E37.
PCR is most useful for which of the following DNA analyses?
A.
B.
C.
D.
E.
E38.
1:200
1:500
1:1000
1:1500
1:2000
A male is found to have 400 CGG repeats. Which of the following disorders is the most likely
diagnosis?
A.
B.
C.
D.
E.
E40.
RFLP analysis
VNTR analysis
Short Tandem Repeat (STR) analysis
CGG full expansion analysis in males
CTG full expansion analysis in females
A disorder is recessive and has a carrier frequency of 1:100 in Ashkenazi Jews. The test includes
one mutation and the test sensitivity is estimated at 90%. What is the probability that an
Ashkenazi Jewish individual with a negative test result is a carrier for this disorder?
A.
B.
C.
D.
E.
E39.
DNA methylation of CpG dinucleotides
CAAT boxes
Promoter enhancers
Transcription factors
Response elements
Fragile X syndrome
Friedreich ataxia
FXTAS
Huntington disease
Myotonic dystrophy
A male is found to have 46 CAG repeats. Which of the following disorders is the most likely
diagnosis?
A.
B.
C.
D.
E.
Fragile X syndrome
Friedreich ataxia
FXTAS
Huntington disease
Myotonic dystrophy
753
E41.
Your lab is testing a patient, with brain iron accumulation on MRI showing the “eye of the tiger”
sign, for the PANK2 gene using sequence analysis and identified a missense variant in exon 3
along with a small insertion in exon 5. The PANK2 database shows the insertion to be pathogenic
but has no information about the missense variant. Both Polyphen and SIFT predict the missense
mutation to be possibly damaging. The predicted amino acid change is from serine to
phenylalanine. dbSNP lists an rs number with unknown frequency for the missense variant. 1000
Genomes indicates a frequency of 0.001 for the missense variant. There are no deletions or
duplications detected by MLPA. There are no other symptomatic individuals in the family. Which
of the following interpretations would best explain these results?
A.
B.
C.
D.
E.
E42.
A 50-year-old man is found to have 120 CGG repeats. Which of the following disorders is the
most likely diagnosis?
A.
B.
C.
D.
E.
E43.
Fragile X syndrome
Friedreich ataxia
FXTAS
Huntington disease
Myotonic dystrophy
A male fetus is found to have 1600 CTG repeats. What is the most likely diagnosis?
A.
B.
C.
D.
E.
E44.
The missense variant is benign.
The missense variant is pathogenic.
The missense variant is a VUS (variant of unknown significance).
These findings confirm the diagnosis.
These findings make the diagnosis unlikely.
Fragile X syndrome
Friedreich ataxia
FXTAS
Huntington disease
Myotonic dystrophy
An individual is found to have a 33 and a 300 GAA repeat. Which of the following results is the
most likely interpretation?
A.
B.
C.
D.
E.
Affected with Huntington disease
Affected with Friedreich ataxia
Affected with myotonic dystrophy
Carrier of Friedreich ataxia
Female carrier of Fragile X with symptoms
754
E45.
An 18-year-old young man is found to have thousands of colon polyps and his blood sample is
sent for sequence analysis of the APC gene. The results are negative. Which of the following
conclusions is the best explanation of his findings and the next step in his clinical management?
A.
B.
C.
D.
E.
E46.
Which of the following findings would be interpreted as a positive test result for Spinal Muscular
Atrophy?
A.
B.
C.
D.
E.
E47.
Melting curve analysis by FRET.
Multiplex ligation probe amplification.
Oligonucleotide assay.
Quantitative PCR.
Sanger sequence analysis.
In one form of familial isolated growth hormone (GH) deficiency, GH mutations cause skipping of
exon 3. In heterozygotes, the GH protein product lacking exon 3 accumulates and kills the GH
secreting cells to prevent secretion of normal GH molecules from the normal GH gene. This result
is best described by which of the following genetic concepts?
A.
B.
C.
D.
E.
E49.
A homozygous deletion of exon 7 in the cenSMN gene.
A homozygous deletion of exon 7 in the telSMN gene.
A homozygous duplication of exon 8 in the telSMN gene.
A homozygous duplication of exon 8 in the cenSMN gene.
A homozygous duplication of both exon 7 and exon 8 in the cenSMN gene.
Which of the following molecular tests would be most useful method for diagnosis of a 5-yearold boy with progressive neuromuscular disease, a positive Gower sign, and elevated creatine
phosphokinase values?
A.
B.
C.
D.
E.
E48.
He definitely does not have FAP but he should undergo colectomy soon.
He is a good candidate for MYH polyposis gene analysis and testing should be done
He is a good candidate for HNPCC analysis and testing should be done
He has a mutation in the regulatory region of the gene and testing should be done
He may have a large gene deletion in the APC gene and dosage analysis should be done
Dominant negative effect
Gain of function mutation
Haploinsufficiency
Loss of function mutation
Protein genocide
Achondroplasia has a high mutation rate and is most likely the result of which of the following
genetic concerns?
A.
B.
C.
D.
E.
Highly repetitive sequence
Large gene size
Maternal age effect
Methylated CG dinucleotide
Paternal age effect
755
E50.
Which of these disorders is most likely due to loss of function of the protein product?
A.
B.
C.
D.
E.
E51.
Which of the following disorders is most likely due to a gain of function in the protein product?
A.
B.
C.
D.
E.
E52.
Fragile X
Friedreich Ataxia
FXTAS
Myotonic dystrophy
Spinobulbar Muscular Atrophy
Which of the following disorders is most likely due to a gain of function toxic RNA?
A.
B.
C.
D.
E.
E53.
Fragile X syndrome
FXTAS
Huntington Disease
Myotonic Dystrophy
Spinobulbar Muscular Atrophy
Fragile X
Friedreich Ataxia
Huntington Disease
Myotonic Dystrophy
Spinalbulbar Muscular Atrophy
The figure below represents a methylation Southern blot analysis performed on a symptomatic
individual. The probe is SNRPN and methylation sensitive and insensitive enzymes were used.
Which of the following is the best interpretation of the result for the individual in lane 2?
1
2
3
maternal
paternal
A.
B.
C.
D.
E.
Angelman Syndrome due to the absence of the maternal allele
Angelman Syndrome due to the absence of the paternal allele
Normal individual based on the single targeted band in the blot.
Prader Willi Syndrome due to absence of the maternal allele
Prader Willi Syndrome due to the absence of the paternal allele
756
E54-E57. The following table should be used in answering questions E54-57:
Disease-Positive
70
30
100
Test Positive
Test Negative
Total
E54.
5%
30%
70%
95%
100%
In the above 2x2 table, what is the false negative rate?
A.
B.
C.
D.
E.
E58.
5%
30%
70%
95%
100%
In the above 2x2 table, what is the false positive rate?
A.
B.
C.
D.
E.
E57.
5%
30%
70%
95%
100%
In the above 2x2 table, what is the test specificity?
A
B.
C.
D
E.
E56.
Total
75
125
Given the 2x2 table, what is the test sensitivity?
A.
B.
C.
D.
E.
E55.
Disease-Negative
5
95
100
5%
30%
70%
95%
100%
A metastatic colorectal cancer patient is being tested to determine whether he is a good candidate
for imatanib therapy. Which of the following test results would be most favorable for
administering EGFR inhibitor therapy?
A.
B.
C.
D.
E.
A positive result for a mutation in BRAF.
A negative result for a mutation in FLT3.
A negative result for a mutation in KRAS.
A positive result for a mutation in NPM.
A positive result for a mutation in NRAS.
757
E59.
Your lab is validating a new assay for an autosomal recessive disorder using 100 positive and 100
negative control samples. The assay detected 98 positive samples out of the positive controls and
got 1 positive result in the negative controls. The targeted mutation panel of the assay is designed
to detect 90% of mutations reported in the database of clinically affected individuals. Which of
the following assessments provides the best interpretation of your data?
A.
B.
C.
D.
E.
.
E60.
What is the recommended nomenclature for the major mutation in the CFTR gene in Caucasian
populations? (Pick the best answer)
A.
B.
C.
D.
E.
E61.
'F508
c.F508del
deltaF508
p.F508del
phe508del
If you are screening a colorectal cancer population to detect HNPCC, which of the following
evaluation plans is the most efficient strategy?
A.
B.
C.
D.
E.
E62.
The analytical sensitivity is ~99%.
The analytical specificity is ~98%.
The clinical sensitivity is ~80%.
The clinical sensitivity is ~90%.
The negative predictive value is ~99%.
Family history using Bethesda criteria followed by gene testing MLH1 & MSH2
Full gene sequencing for MLH1, MSH2, MSH6 in all patients
Immunohistochemical analysis of MLH1, MSH2 & MSH6, followed by sequencing of
the indicated gene
Methylation analysis of the MLH1 promoter using blood specimens
Microsatellite instability (MSI) followed by MLH1 & MSH2 sequencing
According to a recent national survey of laboratory directors, which of the following laboratory
descriptions was the strongest predictor of laboratory quality (and fewer errors)?
A.
B.
C.
D.
E.
ABMG Board certification of the laboratory director
Academic setting in a medical school
Laboratory certification by CAP
Participation of the laboratory director in professional societies
Participation of the laboratory in proficiency testing programs
758
E63.
Mary’s sister’s 2-year-old son was recently diagnosed with Duchenne muscular dystrophy
(DMD). He was tested by microarray analysis and found to have a deletion of exons 44-52. Mary
is 6 weeks pregnant and wants to know her DMD carrier status. Her blood sample is sent to a
different lab that uses MLPA for dystrophin carrier testing. The lab does not find the exon 44-52
deletion but instead finds an exon 3 deletion. Which of the following explanations most likely
accounts for these results?
A.
B.
C.
D.
E.
E64.
You see a couple with a history of several miscarriages in prenatal genetics clinic and
you take a detailed medical history. You recommend molecular/cytogenetic analyses
and after blood is drawn from each of them, you complete the appropriate requisition
forms to accompany each of the specimens to the lab. Which of the following items
listed on the requisition is most important to provide the laboratory staff enabling them to
set up the appropriate study?
A.
B.
C.
D.
E.
E65.
Mary’s DNA has a variant at the probe binding site causing a false positive result.
Mary’s mother is a germline mosaic for two different dystrophin mutations.
One of the labs had a sample mix up, misidentifying the sample.
The exon 3 deletion represents a new mutation that occurred in Mary.
The microarray analysis produced false positive results due to incomplete hybridization.
Clinical indication for the molecular/cytogenetic analyses
Documentation that patient fasted for at least 8 hours
Patient’s family’s ethnic and ancestral origins
Patient’s primary medical insurance provider
Source of the tissue sample to be used in the study
A clinical laboratory has received funding from their institution to implement next
generation sequencing in their laboratory. The laboratory is tasked to evaluate the
instruments and sequencing chemistries on the market and provide a justification for
selection of a particular instrument and chemistry. Amongst the parameters being
evaluated the laboratory is looking at Phred scores. A Q40 Phred Score reflects which of
the following likelihoods that the base call is inaccurate.
A.
B.
C.
D.
E.
1:40
1:4,000
1:100
1:1,000
1:10,000
759
E66.
A clinical laboratory has developed next generation sequencing panel for a disorder, which has
shown to be associated with mutations in several genes. The laboratory has completed test
validation is preparing educational material for the test launch. Which of the following is not an
accurate description of the gene panel?
A.
B.
C.
D.
E.
E67.
9. A clinical laboratory has been advised to develop next generation sequencing based tests using
paired end sequencing instead of single read sequencing. Paired end sequencing can be helpful in
increasing the ability to correctly map short sequence reads when which of the following
molecular analysis issues is present?
A.
B.
C.
D.
E.
E68.
The library fragment size is larger than the size of the repetitive region
The library fragment size is smaller than the size of the repetitive region
The library fragment size is equal to the size of the repetitive region
Longer sequencing primers are used
Longer molecular barcodes are used
10. Next Generation sequencing technology is being widely adopted in clinical laboratories. As a
part of the guidance documents from CAP, CDC and ACMGG tests are evaluated for several
parameters, including analytical specificity. The analytical specificity of a test is a reflection of
which of the following statistical measures?
A.
B.
C.
D.
E.
E69.
Detection of point mutation, small insertion or deletion in a gene
Confirmation of an established clinical diagnosis when single gene sequencing is negative
Detection of point mutation in promoters and introns
Detection of intragenic copy number variation within the genes
Perform mutation analysis in an affected individual with differential diagnosis
False negative rate
Positive prediction value
False positive rate
Negative prediction value
Standard deviation
Next Generation Sequencing technology is being widely adopted in clinical laboratories. As a
part of the guidance documents from CAP, CDC and ACMGG tests are evaluated for several
parameters, including analytical specificity.The analytical sensitivity of a test is a reflection of
which of the following statistical measures?
A.
B.
C.
D.
E.
False negative rate
Positive prediction value
False positive rate
Negative prediction value
Standard deviation
760
E70.
You provide a clinical description of a patient to the molecular cytogeneticist at your
facility including: a prominent nasal root, bulbous nasal tip, hypocalcemia,
immunodeficiency, and conotruncal heart abnormality. Which of the following
laboratory techniques is best used to confirm the suspected diagnosis in this patient?
A.
B.
C.
D.
E.
E71
A family with an affected father and two affected siblings with an undiagnosed
autosomal dominant condition is examined by a clinical geneticist. Since the clinical
presentation is not indicative of a specific disorder the clinical geneticist decides to order
whole exome sequencing. Whole exome sequencing interrogates approximately what
fraction of the exome?
A.
B.
C.
D.
E.
E72.
Next-generation sequencing
Routine cytogenetic testing
Southern Blot
Fluorescence-in-situ hybridization (FISH) or DNA microarray
Sanger sequencing of the TBX1 gene
100%
92%
70%
50%
20%
Known pathogenic variants are found in a variety of genetic databases available
electronically through the internet. In searching for the implications of a de novo variant
found on exome sequencing of a patient with unexplained developmental delay and
dysmorphic features you find the exact same variant reported in the Human Genetic
Mutation Database (HGMD) associated with a milder phenotype. However, a search of
the Online Mendelian Inheritance in Man (OMIM) database and LOVD (Locus Specific
database) does not confirm this relationship. Which of the following is the best
explanation for the differences in the information available in these databases?
A. Cancer causing changes (somatic mutations) are not included in these databases.
B. Each database has a unique way of identifying which variants to include and call
pathogenic
C. Mitochondrial mutation data is only included in some of these databases.
D. Sources of identified variants are the same across these databases.
E. Splice-site and regulatory regions of human nuclear genes are not included.
761
E73.
A clinical laboratory is undergoing validation of next generation sequencing (NGS)
technology using CAP molecular check list. They are validating gene panels and exome
sequencing but will be outsourcing the “bench work” to a CLIA certified core sequencing
facility and performing in house bioinformatics analysis. Which is most appropriate file
to be returned to the laboratory for analysis?
A.
B.
C.
D.
Sequence + images
Short sequence reads aligned to the reference sequence
Sequence + quality scores
Variants + quality scores
E. Variant calls and images
E74.
Next Generation Sequencing (NGS) technology is being widely adopted in clinical
laboratories. As a part of the guidance documents from CAP, CDC and ACMGG tests are
evaluated for several parameters which include ability to detect insertion and deletion.
NGS based tests tend to have a reduced analytical sensitivity for small insertions and
deletions (in/dels) as a result of which of the following characteristics of in/dels
detection?
A.
B.
C.
D.
E.
E75.
In/dels negatively influence cluster generation on the flow cell
In/dels lead to altered chromatin structure
In/dels tend to have strand bias and reads containing them are filtered out
Short reads containing in/dels are more difficult to map unambiguously
The polymerases used do not sequence efficiently through in/dels
Based on the clinical presentation and nuclear study, a diagnosis of Hereditary
Paraganglioma-Pheochromocytoma syndrome was considered in a 36-year-old woman.
The eight exons of the succinate dehydrogenase complex, subunit B (SDHB) gene were
sequenced and the variant c.434 C>T (p.R115X) was detected in the fourth exon of this
gene. What is the most likely outcome of the mRNA transcribed from this allele?
A. mRNA will be translated into truncated protein that has deleterious gain of function
B. mRNA will be translated into truncated protein that has dominant negative effect
C. mRNA will undergo nonsense mediated mRNA decay (NMD)
D. mRNA will be translated into full-length protein
E. This variant will inhibit the transcription of mRNA from this allele
762
E76.
A 5-year-old boy was referred for delays in language development. He had no functional
communication, but was able to repeat words and phrases. He displayed some repetitive behavior
and did not play with toys in the typical manner. Testing for Autism Spectrum Disorder was
recommended. Which of the following analyses is likely the first tier test to be performed on this
child?
A. Chromosomal analysis
B. Expanded CGG repeat in FMR1 (Fragile X)
C. Expanded CCG repeat in FMR2 (FRAXE)
D. Non-syndromic autism gene panel
E. Syndromic autism gene panel
E77.
American College of Medical Genetics and Genomics published new sequence interpretation
guidelines in 2015. These guidelines are now widely used for interpretation of variants in genes
associated with disease. The guidelines give five categories for variant classification (Pathogenic,
Likely Pathogenic, Variant of uncertain Significance, Likely Benign and Benign) with rules to
classify them. Which of the following are these rules a best fit for when interpreting a variant in a
clinical setting?
A. Genes without evidence as causing disease
B. Genes with moderate to strong evidence as being causative of disease and a well-established
reported variant spectrum
C. Genes with only strong evidence of being causative of disease and a well-established reported
variant spectrum
D. Sequence variants of all types irrespective of the gene/ disease causality evidence
E. Truncating variants only (nonsense, frameshift and splice site)
E78.
A new gene is reported in the medical literature in 2013, associated with an inherited autosomal
dominant form of colon cancer. Multiple genes have been reported to be causative of inherited
forms of colon cancer. Subsequent publications establish the causal relationship of gene: colon
cancer occurs with evidence a loss of function effect from the variants of the gene. A founder
pathogenic variant is reported in the Chinese population and most Chinese patients are found to
have this pathogenic variant, with only two other pathogenic variants reported from a publication
coming from China. A laboratory in China has made a decision to develop a test for this gene.
Which of the following technical approaches is preferred for this gene in order to identify
pathogenic variants causative of disease?
A. Exome sequencing to detect variation in all ~22,000 genes in the human genome.
B. Gene panel approach using next generation sequencing to detect variants in all genes known to
causative of inherited form of colon cancer including this newly reported covering all exon and
flanking intron/exon boundaries
C. Next generation full sequencing single gene assay covering the entire genomic region of the
gene to detect all types of variants
763
D. Sanger sequencing single gene assay covering all exons and flanking exon/intron boundaries
to detect all types of variants
E. Targeted pathogenic variant detection approach using methods such DHPLC, or PCR followed
by restriction enzyme to detect the common Chinese pathogenic variant
E79.
A silent sequence variant was identified, in a Caucasian individual with no reported family
history of skin disease, in a gene associated with an autosomal dominant form of skin disease.
This silent variant occurred at the last base of the exon 3 (ATG start in exon 1) and has not been
reported in the literature. ExAC, GnomAD and dbSNP, and 1000G have a collective allele
frequency of 0.00005% for this variant. The laboratory reported the variant using the ACMG
sequence interpretation guidelines as a Variant of Uncertain Significance (VUS). Which of the
following steps should the laboratory take classify the variant as pathogenic or benign?
A. Perform reanalysis of the variant, by obtaining new blood samples from patient and family
along with performing functional analyses on patient skin biopsy fibroblasts.
B. Recommend exome or genome sequencing to identify other possible causes of the skin d
disease.
C. Report the case as” negative” and conduct no further investigations into the nature of this
silent variant.
D. Sign up for Google Alerts to be notified when this specific variant is reported in the medical
literature by a reputable research group.
E. The patient’s physician should search the medical literature to try to find a research group
working on this gene in order to help the family
764
Answers to Molecular Genetics E1-E79
E1.
There are two noteworthy issues. The mother fails to give an allele to the affected and maybe she
has a deletion for the locus. Second, the other child who fails to receive an allele from the mother
supports that this phenomenon is real but gets the opposite allele from the father and is
unaffected. The other unaffected sibs have genotypes different from the affected. This is very
encouraging!
The correct answer is A.
E2.
The phase cannot be deduced in the father of the pregnancy and the analysis is of no help. The
risk for the fetus is 0.5.
The correct answer is C.
E3.
This is best calculated on a computer program such as MLINK but can be estimated by a
Bayesian calculation.
The correct answer is B.
Prior
Conditional (Neg.)
Joint
Post.
Carrier
5/10
1/10
5/100
5/55 = 1/11
Noncarrier
5/10
1
50/100
50/55 = 10/11
E4.
Since the analysis definitely is not detecting the mutation on the paternal side, the risk is
unchanged by DNA analysis and is 1 in 2.
The correct answer is A.
E5.
This is an intercross result for an autosomal recessive disorder and the data on the grandparents
are irrelevant. If the fetus were homozygous 11 or 22, it would be a definite carrier. Since it is 12,
there is a 50% probability for affected and 50% for noncarrier.
The correct answer is C.
E6.
Both copies of SMN1 are deleted in approximately 95% of patients. The remaining 5% of
patients have one deleted gene and a nonsense, frameshift or missense mutation in the other gene
and would require sequence analysis. All other options were distractors.
The correct answer is A.
E7.
The hybridizing fragments of 5.2 and 2.8 kb represent methylated, non-expanded (inactive
FMR1) and non-methylated, non-expanded (active FMR1) alleles, respectively. The most likely
reason for a male to have both methylated and non-methylated, non-expanded FRM1 alleles is
that he has an XXY Karyotype and Klinefelter syndrome. A) Swapping DNA samples is possible
but this should have resulted in a sample from a female generating a male pattern. Also a repeat
sample should be requested and its analysis will eliminate the possibility that his FXS sample was
exchanged with that of an XX female. b) FXS result for a normal male would be 2.8 kb only. d)
Mosaicism for FXS would yield 2.8 kb and a smear of methylated, expanded FRM1 alleles >~6.6
kb in size. e) An incomplete endonuclease digestion should have been noticed on exam of the
digestion products after gel electrophoresis and is likely to not generate discrete hybridizing
bands. The correct answer is C.
765
E8.
The phase in the father can be deduced to be allele 2 with the disease allele since both had to
come from the grandfather. The fetus receives allele 3 from the father and allele 2 from the
mother and is unaffected.
The correct answer is E.
E9.
There is an abnormal junction fragment in this blot. The affected child has the junction fragment.
The mother also has the junction fragment and is definitely a carrier. None of the other female
individuals has the abnormal fragment and therefore all are predicted to be noncarrier.
The correct answer is B.
E10.
The protein truncation assay is based on the detection of truncated peptides due to any mutation
that causes premature termination of translation.
The correct answer is E.
E11.
The first of the five affected children in this sibship sets the phase. For each subsequent affected
child, the chances of receiving this same dinucleotide genotype is 1 in 4. Therefore the
probability that all of the subsequent affected children would inherit the same dinucloeotide
genotype by chance is ¼ x ¼ x ¼ x ¼ = 1/256. Therefore the odds of obtaining a result favoring
linkage by chance would be 1 in 256.
The correct answer is E.
E12.
The phenotype associated with the R117H mutation is influenced by the 5T/7T/9T polymorphism
in intron 8, which affects whether exon 9 is included in the transcript.
The correct answer is A.
E13.
FXTAS is caused by large premutations in older individuals and the symptoms include tremors
and ataxia, explaining the grandfather. The boy is affected with fragile X syndrome due to a CGG
expansion in FMR1. The mother inherited the premutation from her father and passed an
expanded full mutation to her son. All other options were distractors.
The correct answer is D.
E14.
DNA for analysis is isolated from tissue blocks or slides that contain portions of tumor
and possibly adjacent, non tumor tissue. If the portion of the sample selected for DNA
isolation contains little tumor but abundant normal tissue, molecular analysis may be
false negative. a) A Chromosomal deletion including BRAF would not cause a false
negative result because the V600E mutated allele would remain if the deletion was trans
and if the deletion is in cis, V600E is not there to detect. d) A sample mix up (exchange
of samples) is possible but selection of sections of tumor for DNA isolation makes this
unlikely. e) The BRAF V600E mutation is a missense mutation rather than a trinucleotide
repeat expansion.
The correct answer is C.
E15.
You should know that the female map is greater than the male map, and that the sex-average map
is about 3700 cM.
The correct answer is E.
766
E16.
These data are consistent with a recombination event occurring between the two markers for the
current pregnancy. This means that there is little ability to use the linkage data to predict the
outcome of the pregnancy and the a priori risk of 50% would be the most appropriate.
The correct answer is C.
E17.
The correct answer is C.
E18
The correct answer is A.
E19.
The correct answer is C.
E20.
The correct answer is B. The liquid bead array can be highly multiplexed and is frequently used
for large targeted assays. The FRET assay is not a multiplex and is well suited to a single target.
MLPA is for detection of deletions and duplications, not point mutations. NGS and Sanger
sequencing would not be suitable for large scale testing for a highly multiplexed panel.
The correct answer is B.
E21.
The FGFR3 GLY380ARG (GGG to AGG) substitution that was reported to cause 96% of
Achondroplasia cases changes the G of a CG dinucleotide to an A by methylation of the C on the
complementary strand that is paired to the G of the forward strand. Methylation of Cs on the
forward strand cause CG to TG transitions. Methylation of Cs on the complementary (reverse)
strand causes CG to CA transitions on the forward strand. a) GGG to AGG is not a trinucleotide
expansion. b) While a GGG to AGG substitution might be caused by gene conversion, CG
transitions are much much more common. d or e) This is not an example of either recombination
or slipped misspairing.
The correct answer is C.
E22.
It is imperative to separate the SMN1 gene from the SMN2 gene before testing for the
homozygous deletion. This is most commonly done by using a restriction enzyme site present in
the SMN2 gene, but lacking in the SMN1 gene.
The correct answer is B.
E23.
Southern blotting detects both the large CGG expansions and the methylation status
The correct answer is E .
E24.
A major advantage of dot blotting techniques is that gels are not used which allows one to screen
many samples.
The correct answer is D.
E25.
The CAG repeats all seem to work by a gain of function mechanism.
The correct answer is E.
E26.
Microsatellites repeats are multi-allelic and therefore are often highly informative.
The correct answer is C.
767
E27.
Reversion is a common event in the FANCA gene. Both blood and fibroblasts are tested for
breakage analysis and then for DNA to identify the mutation. A number of patients show
differences in the mutations identified in the blood and fibroblasts. The reversion event most
likely occurs in bone marrow. Note that a silent mutation can be either benign or disease-causing.
In this case the silent mutation leads to the creation of an alternate splice site.
The correct answer is B.
E28.
Changing base 2 of a codon always produces a missense (nonsynonymous) substitution.
Changing base 1 almost always does so, while changing base 3 causes a missense
mutation much less frequently. a) AGG to ATG doesn’t create a frameshift. c) While
AGG to ATG might create a start codon, remember this is an internal codon and it is
unlikely to be embedded in a Kozak sequence (GCCPuCCAUGG). Without the
surrounding Kozak sequence the ATG encodes an internal Methionine. d) Changing base
2 of a codon never produces a synonymous substitution. e) AGG to ATG doesn’t create a
termination codon.
The correct answer is B.
E29.
The correct answer is E.
E30.
Factor VIII deficiency is often due to inversion within the factor VIII gene.
The correct answer is A.
E31.
The most common lesions in DNA caused by UV damage are pyrimidine dimers. The exact
position of the new bonds can vary. These can be T-T, T-C, or C-C dimers. The dimers involve
adjacent bases on the same strand.
The correct answer is D.
E32.
The correct answer is B.
E33.
To answer this question you have to know the size categories for CAG repeats in HD. A 38 repeat
allele is in the reduced penetrance category (36 to 39 repeats). A symptomatic individual with a 38
CAG repeat in HTT would be interpreted as the results are consistent with a diagnosis of HD.
The correct answer is C.
E34.
Mrs. Soft most likely has Osteogenesis Imperfecta (OI) type 1 which is caused by missense
mutations, small insertions or deletions, or exon-skipping mutations of COL1A1 or COL1A2.
You should test Mrs. Soft to confirm that she has a detectable COL1A1 or COL1A2 OI causing
mutation. You can then test her son for her mutation. If he is positive, then he is at great risk to
develop findings of OI. If he is negative for the mutation, then he is at population risk to develop
OI. a) His not having an OI causing COL1A1 or COL1A2 mutation could result from Mrs. Soft
not having OI Type 1, he has her mutation but it is not detectable, or he does not have her
mutation. Thus if he has a negative result you are in doubt. b, c and e) These options are all
looking for pleiotropic effects of OI that can have reduced penetrance and thus could be negative
at two months of age.
The correct answer is D.
E35.
The correct answer is B.
768
E36.
The correct answer is D.
E37.
The correct answer is C. RFLPs, while some can be detected by PCR, many cannot and require
Southern analysis. VNTRs are large repeating units and require Southerns, as do full expansions
for Fragile X and myotonic dystrophy.
E38.
The correct answer is C. This requires a Bayesian risk calculation.
Prior
Conditional
Joint
Posterior
Carrier
1/100
10/100
10/10000
1/991 (~1/1000)
Non-Carrier
99/100
1
99/100
E39.
The correct answer is A. The only CGG expansion disorders in this list is Fragile X and FXTAS.
A large expansion of this size would lead to FRAXA, while a premutation would cause FXTAS.
The size of the CGG repeat is consistent with Fragile X symptoms, not FXTAS. Myotonic is
CTG, FXTAS
E40.
The correct answer is D. HD is the only disorder in this list that has CAG expansions.
E41.
While the missense variant has not been reported, it is predicted to be pathogenic by multiple
predictive algorithms and is found at a very low frequency in the population, and is seen in this
affected individual along with a known pathogenic mutation. Thus, it may be pathogenic.
However, clinically, it would probably be reported as a variant of unknown clinical significance
until additional evidence, such as family studies or functional analysis provided evidence for reclassifying it. Caveats to the report may include that it is suspected pathogenic.
The correct answer is C.
E42.
The correct answer is C. FXTAS is caused by premutations in FRAXA.
E43.
The correct answer is E. Myotonic dystrophy is caused by CTG expansions. Large expansions
are found in congenital myotonic dystrophy and are maternally derived.
E44.
The correct answer is D. Only 4% of FRDA patients will have an expansion and a point mutation.
Most FRDA carriers will have an expansion and a normal allele. If this were a symptomatic
individual, one would recommend the point mutation analysis. For diagnostic testing, finding one
expansion confirms that the individual is at least a carrier but does not confirm the diagnosis,
although it is strongly suggestive.
E45.
The correct answer is E. All of these are possibilities, however, the best NEXT STEP that should
be done after sequence analysis for APC is gene deletion analysis. Then, MYH gene testing
would be indicated. At present clinical labs are not testing for mutations in the regulatory region
of APC. If APC and MYH are ruled out, then he would most likely not have FAP. It is unlikely
that he has HNPCC due to the thousands of polyps.
769
E46.
The correct answer is B. The important concept is that there are two SMN genes, one centromeric
and one telomeric. The pseudogene is centromeric, while the gene that causes SMA is telomeric.
Both genes are very similar at the molecular level and can be distinguished by SNPs in exons 7
and 8. Clinical laboratory testing is based on finding a homozygous deletion of exon 7 in the
SMNtel gene, so it is critical to be able to separate the SMNtel from the SMNcen. In most SMA
cases, a homozygous deletion of both exons 7 and 8 is found. However, the homozygous absence
of exon 7 is found in almost all SMA cases.
E47.
The correct answer is B. The patient clearly has DMD. The majority of mutations in the
dystrophin gene are deletions and duplications. The only test listed which detects
deletions and duplications is MLPA.
E48.
The correct answer is A
E49.
The correct answer is D (repeat question (see #21) without vignette and different distractors)
E50.
The correct answer is A. The only disorder listed that is due to loss of function of protein product
is FRAXA. The function of the normal protein product is an RNA-binding protein.
E51.
The correct answer is E. SBMA is due to a gain of function protein that most likely affects
transcription.
E52.
The correct answer is D. Myotonic dystrophy is most likely due to the gain of function of a toxic
RNA that interferes with normal splicing in a number of RNAs leading to faulty protein products.
E53.
The correct answer is E. Lane 1 is a normal individual with both maternal and paternal
contributions. Lane 2 is Prader Willi Syndrome patient lacking a paternal contribution; lane 3 is
Angelman Syndrome.
E54.
The correct answer is C. The clinical test sensitivity is calculated by determining the number of
individuals who have disease and are positive for the test divided by the total number of
individuals with disease who were tested (both positive & negative).
E55.
The correct answer is D. The clinical test specificity is calculated by determining the number of
individuals who do not have disease and are negative for the test divided by the total number of
individuals without disease who were tested (positive and negative).
E56.
The correct answer is A. The false positive rate is determined by the number of individuals
without disease who tested positive over the total number of individuals without disease who
were tested.
E57.
The correct answer is B. The false negative rate is the number of individuals with disease who
tested negative over the total number of individuals with disease who were tested.
770
E58.
The correct answer is D. KRAS is downstream from EGFR. Auto-activating mutations that occur
in KRAS result in unregulated growth, thus bypassing EGFR regulation by inhibitor drugs. Prior
to prescribing EGFR inhibitors, patients are tested for KRAS mutations. Only in the absence of a
KRAS mutation is the mCRC patient placed on the EGFR inhibitor drug.
E59.
The correct answer is C. The clinical sensitivity refers to the percent of patients that the
assay will correctly identify. In this case, the disorder is autosomal recessive, indicating
that two mutations would be found in each patient. The assay detects 90% of the variants
found in the patient database. The clinical sensitivity is 0.9 x 0.9 = 0.81, or approximately
80%.
E60.
The correct answer is D. The p stands for protein, the F for phenylalanine, 508 is the codon, and
del is the deletion. I did not give the cDNA nomenclature at the nucleotide level because that
would not be the common name that most people would recognize on the general exam. It is
recommended to give both proper and common nomenclature on reports. The proper
nomenclature should always be used in describing a novel variant and in sequence analysis and
data base reporting. The molecular folks should be able to name a mutation using current
nomenclature standards, given the sequence information.
E61.
The correct answer is C. A combination of all of these testing strategies can used in HNPCC
screening in a large colorectal cancer population. The only incorrect statement is that promoter
methylation and BRAF analysis is done on the tumor, not in blood (germline). The most efficient
strategy listed includes IHC followed by the indicated gene analysis. It is efficient in that it
decreases the number of multiple gene analyses.
E62.
The correct answer is E. The strongest indicator of high quality and low error rate was laboratory
participation in proficiency testing for all analytes tested.
E63.
The correct answer is A. The greatest technical concern about the MLPA is regarding
SNPs at the probe site that would interfere with binding of the probe and ligation, which
produce false positive results. Generally, this would result in only a single exon being
apparently deleted. For this reason, single exon deletion results should be confirmed
using another technique. While some of the distractors could also happen, by far, the
most likely explanation is as described above.
E64.
The correct answer is A.
Keywords: clinical indication, miscarriages
Explanation: The clinical indication will allow the laboratory staff to select the
appropriate first study to perform. The patient’s family’s ethnic/ancestral origins may
impact what is targeted in some studies, but in the instance of multiple miscarriages, is
less likely to play a significant role. Fasting is not an issue with molecular/cytogenetic
analyses. The patient’s health insurance provider should not impact the decision of the
most appropriate study to perform. The source of tissue while important for some
studies, is obviously blood in this instance.
771
E65.
The correct answer is E
Keywords: Phred score, sequence chemistry
Explanation: Phred scores are indicative of the quality of the sequence data and the
higher the Phred score lower the error rate in the sequence data. A Phred score of Q40 is
considered the highest and best Phred indicative of an error rate of 1:10,000 in the
sequence data.
E66.
The correct answer is D.
Keywords: mutations, clinical diagnosis
Explanation: Gene panels can be easily developed using next generation sequencing
(NGS) technology. These panels are useful to confirm the clinical diagnosis and also
when the disorder is in the list of differential diagnosis. At this time NGS can detect only
point mutations and small insertions and deletion in any regions of the gene. NGS cannot
detect intragenic copy number variation.
E67.
The correct answer is B
Keywords: paired end sequencing, single read sequencing
Explanation: NGS technology is based on short read sequencing. Single read sequencing does not
generate high fidelity data as it does not read from both ends of the sequence and therefore does
not interrogate repetitive region effectively. Paired end sequencing can effectively address this
issue.
E68.
The correct answer is C.
Keywords: False positive rate, False negative rate
Explanation: Analytical specificity is defined as the ability of an analytical method to measure
only the sought-for analyte or measurand. Numerically characterized by determination of
interferences and non-specific responses to other analytes or materials.
E69
The correct answer is A.
Keywords: False positive rate, False negative rate
Explanation: Analytical sensitivity is defined as the ability of an analytical method to detect small
quantities of the measured component. Numerically characterized by determination of detection
limit.
E70.
The correct answer is D
Keywords: clinical description, cytogenetics
Explanation: The clinical description is specific and indicative of a targeted cytogenetic
abnormality, which can be identified by FISH analysis. Methodologies such as next generation,
routine cytogenetic testing are useful when a specific condition is not indicated from the clinical
presentation. Southern blot analysis is labor intensive method to detect the abnormality whereas
sequencing will not detect the common mutation in the indicated abnormality.
E71
The correct answer is B.
Keywords: Whole exome sequence, coverage
Explanation: Certain regions of the exomes are still refractory to the exome analysis so whole
exome sequencing has “Holes” in it. These holes contain regions not yet sequenced by the Human
Genome project, difficult to sequence regions such as repeat regions and complex sequences. Still
a large amount of the exome is covered by exome sequencing.
772
E72.
The correct answer is B.
Keywords: human variation, databases
Explanation: The methods for identifying variants differs from database to database.
Some databases rely on searching the medical literature for published reports of variants
and others allow for investigators to submit the variants they have identified in an online
report. Some databases include mitochondrial and/or somatic mutations.
E73.
The correct answer is C.
Keywords: sequence analysis, bioinformatics
Explanation: The file generated by NGS should contain all the relevant information
required for analysis using NGS analysis pipeline. The image files are large and not
needed for sequence analysis. The reference sequence should be a standard sequence
using by a clinical laboratory to validate their analysis pipeline so it important the
laboratory performing the analysis chooses the reference sequence. The laboratory should
generate their own variant file using their reference sequence instead of a pre computed
variant file which could lead to errors.
E74.
The correct answer is D.
Keywords: in/del detection, analytical sensitivity
Explanation: In contrast to the commonly used Sanger sequencing methodology which
generates long sequence reads NGS generates short sequence reads. The short read
sequencing technology is fast and allows generation of large amount of data using
sophisticated sequence chemistries, which involve cluster generation in a flow, high
fidelity polymerases. A drawback of the short sequence reads is that they can map nonspecific regions of the genome and thus cause errors in the in/del detection.
E75.
The correct answer is C.
Nonsense-mediated mRNA decay (NMD) is a post-transcriptional surveillance
mechanism that degrades transcripts with nonsense mutations in their open reading frame
(ORF). Mutations that UHVLGHDWOHDVWQWƍWRDQH[RQMXQFWLRQGLUHFWWKHDIIHFWHG
mRNA to rapid decay. In contrast, the nonsense mutations within the last exon do not
activate NMD and yield a stable mRNA that directs the synthesis of C-terminally
truncated polypeptides.
E76.
The correct answer is A.
Chromosomal analysis as a cytogenetic abnormality accounts for ~15% of genetic causes
related to ASD.
773
E77.
CORRECT ANSWER B:
SOURCE: Lecture/Syllabus
KEYWORDS: sequence interpretation, variant classification
EXPLANATION: Genes with moderate to strong evidence as being causative of disease
and a well-established reported variant spectrum. The ACMG sequence variant
guidelines clearly state that it is important to look into reported gene: disease causality
with applying the sequence interpretation guidelines. This includes the reported evidence
for gene: disease relationship, variant spectrum, effect of the pathogenic variant (eg Loss
of function) and any contradictory evidence.
E78.
CORRECT ANSWER B:
SOURCE: Lecture/Syllabus
KEYWORDS: colon cancer, gene panel, founder, pathogenic variant
EXPLANATION: A gene panel approach using next generation sequencing to detect
variants in all genes known to causative of inherited form of colon cancer including this
newly reported gene, covering all exon and flanking intron/ exon boundaries is the best
answer. This approach will ensure all types of variation are detected in the newly
reported gene (since two other pathogenic variants have been reported) and other
previously known genes are also covered. A next generation sequencing approach is the
most cost effective approach.
E79.
CORRECT ANSWER: A
SOURCE OF ITEM TOPIC: Lecture/Syllabus
KEYWORDS: silent variant, Variant of Uncertain Significance (VUS), splice site defect
EXPLANATION: The laboratory should perform reanalysis of the variant, contact
Physician/ Counselor to obtain additional family members and try to obtain a new blood
samples/skin biopsy to perform functional analysis to investigate a possible splice site
defect. The clinical report should clearly state the findings on the gene and recommended
testing for additional family members to aid in interpretation of the variant. If possible,
the laboratory should perform cDNA seq or collaborate with a research laboratory to
resolve the variant.
774
F. Genetic Counseling Questions F1-F40
F1.
Which of the following lists accurately describes the risks for an Ashkenazi Jewish couple to
have a child affected with one of the following disorders? (CF = cystic fibrosis)
A. CF > Tay Sachs = Gaucher
B. Gaucher > CF > Tay Sachs
C. Tay Sachs > Gaucher > CF
D. Gaucher > Tay Sachs = CF
E. Tay Sachs > CF > Gaucher
F2.
An uncle and his niece have consensual sex and she becomes pregnant. The uncle’s sister (maternal
aunt of the niece) has a rare autosomal recessive disorder which is fully penetrant. Which of the
following risks best describes the likelihood that the pregnancy shown is affected with the condition?
A. 1 in 12
B. 1 in 16
C. 1 in 18
D. 1 in 24
E. 1 in 32
P
F3.
This family is affected with an X-linked disorder where penetrance is complete in hemizygous males,
and heterozygous females are asymptomatic. What is the probability that individual III, 4 is a carrier.
A.
B.
C.
D.
E.
1/2 or 0.50
1/4 or 0.25
1/5 or 0.20
1/8 or 0.125
1/10 or 0.10
I
1
II
2
1
3
2
3
4
1
2
3
III
775
4
F4.
This family is affected with an autosomal recessive disorder. What is the probability of the disease in
the current pregnancy? Two sisters married two brothers.
A.
B.
C.
D.
E.
0.125
0.0625
0.052
0.039
0.035
P
F5.
An autosomal recessive disease can be diagnosed with molecular detection of mutations. The current
test identifies 80% of carriers. This test identifies a man as a definite carrier of this disease. His
spouse’s test is negative. Assuming a population carrier risk of 1 in 25, what is the probability that
this man's spouse is a carrier?
A.
B.
C.
D.
E.
F6.
Twin, adoption and family studies for pyloric stenosis suggest that genetic factors are
important in the etiology of this birth defect, and it appears to follow a multifactorial pattern
of inheritance. The incidence among male babies is 5/1000, and the incidence among female
babies is 1/1000. Your patient, Dorothy, was born with pyloric stenosis, and she is
currently pregnant. Another patient, Eleanor, is also pregnant and the father of the baby was
born with pyloric stenosis. There is no other family history of pyloric stenosis in either
family. Among the following individuals, who is at highest risk to have pyloric stenosis?
A.
B.
C.
D.
E.
F7.
slightly greater than 0.032
exactly 0.032
slightly less than 0.032
slightly greater than 0.008
exactly 0.008
Dorothy’s daughter
Dorothy’s son
Eleanor’s daughter
Eleanor’s son
Each has the same risk
Which of the following viral agents is the most common identifiable cause of congenital viral
infections?
A.
B.
C.
D.
E.
Cytomegalovirus
Herpes simplex
Mumps
Rubella
Varicella
776
F8.
Which of the following clinical findings is most likely in infants with congenital CMV infection?
A.
B.
C.
D.
E.
F9.
A woman comes to clinic because she had a CT of the lower spine and then discovered she was 3-4
weeks pregnant at the time. The radiologist says the dose was less than 0.01 gray (Gy). Her risk for
fetal malformation and/or mental retardation from the X-ray exposure is closest to which of the
following estimates?
A.
B.
C.
D.
E.
F10.
1 in 10 or more
1 in 50
1 in 100
1 in 500
1 in 1000 or less
Which of the following congenital heart defects carries the highest recurrence risk?
A.
B.
C.
D.
E.
F11.
Cataracts
Congenital heart disease
Intracranial calcification
Hydrocephalus
Asymptomatic child
Pulmonary atresia
Truncus arteriosus
Tricuspid atresia
Ebstein's anomaly
Endocardial fibroelastosis
For the pedigree below, assume complete penetrance and that solid symbols identify individual who
are affected with the disorder. The pedigree below is most consistent with which of the following type
of inheritance?
A.
B.
C.
D.
E.
Imprinted gene expressed only from the maternal allele
Imprinted gene expressed only from the paternal allele
Sex limited autosomal dominant inheritance
X-linked dominant inheritance
X-linked recessive inheritance
777
F12.
A 30 year-old woman elects to undergo maternal serum marker screening for aneuploidy.
The test results indicate a 1 in 1,000 risk for Down syndrome. An ultrasound examination is
subsequently performed at 18 weeks gestational age. There is a singleton fetus with adequate
growth. Amniotic fluid volume is normal. All anatomic findings are normal. Which of the
following choices would be the most appropriate management for this patient?
A.
B.
C.
D.
E.
F13.
Amniocentesis
Fetal echocardiogram
No further testing
Repeat serum marker testing
Ultrasound to determine the Nuchal Thickening (NT) measurement
A pedigree with two individuals affected with autosomal recessive polycystic kidney disease is
depicted below. Two brothers married two sisters. The brothers had an affected deceased sister and
one of the couples had an affected child. The other couple is pregnant (P). The probability that the
fetus is AFFECTED is closest to which of the following fractional statements?
A.
B.
C.
D.
E.
1 in 4
1 in 9
1 in 12
1 in 16
1 in 32
P
F14.
The results of prenatal study for an autosomal recessive disorder are depicted below. A dinucleotide
repeat within the gene for the disease was analyzed in the parents, an affected child, an unaffected
child, and the fetus. Which of the following percentages represents the probability that the fetus is
AFFECTED?
A. 100%
B. 67%
C. 50%
D. 25%
E. 0%
P
778
F15.
A woman has two brothers and a maternal uncle institutionalized with a rare form of severe X-linked
mental retardation (X-MR). She has two normal sons and is pregnant again with a male fetus. What is
the probability that the fetus is affected with the X-MR disorder?
A.
B.
C.
D.
E.
F16.
1 in 8
1 in 10
1 in 12
1 in 14
1 in 16
The chance that the mother (not retarded) of a boy with fragile X mental retardation will have both
triplet repeats on her two X chromosomes in the normal size range is closest to which of the
following percentages?
A.
B.
C.
D.
E
F17.
The border between normal and expanded triplet repeat size for Huntington disease is closest to
which of the following numbers of triplet repeats.
A.
B.
C.
D.
E
F18.
50%
25%
10%
5%
0%
35 repeats
45 repeats
55 repeats
65 repeats
75 repeats
Older children and adults with normal hemoglobin typically have 97% Hemoglobin A, 2%
Hemoglobin A2, and < 1% Hemoglobin F. Which of the following results would you expect
to see on the quantitative hemoglobin electrophoresis for a beta-thalassemia carrier?
A.
B.
C.
D.
E.
F19.
Ĺ+HPRJORELQ$Ļ+HPRJORELQ$Ļ+HPRJORELQ)+HPRJORELQ6
Ļ+HPRJORELQ$Ĺ+HPRJORELQ$Ĺ+HPRJORELQ)+HPRJORELQ6
Ļ+HPRJORELQ$Ļ+HPRJORELQ$Ĺ+HPRJORELQ)+HPRJORELQ6
Ĺ+HPRJORELQ$Ļ+HPRJORELQ$Ļ+HPRJORELQ)+HPRJORELQ6
Ļ+HPRJORELQ$Ļ+HPRJORELQ$Ļ+HPRJORELQ)+HPRJORELQ6
The recurrence risk is higher if the index case is female for which of the following disorders?
A.
B.
C.
D.
E.
cleft palate
lethal osteogenesis imperfecta
neural tube defect
profound childhood deafness of unknown cause
pyloric stenosis
779
F20. The risk of a severe abnormality or mortality related to a genetic disorder for the first child of a
cousin mating is closest to which of the following percentages?
A.
B.
C
D.
E.
F21.
first
1%
3%
6%
9%
12%
Which of the following explanations is MOST likely to account for the great variation in severity that can
occur between full siblings with cystic fibrosis?
A. epistatic interaction with modifying genes
B. linkage disequilibrium
C. multiple loci for the cystic fibrosis gene
D. multiple alleles for the cystic fibrosis gene
E. uniparental disomy
F22. A couple whose first son has isolated hydrocephalus with aqueductal stenosis asks about their risk to have
another affected child. There are no other affected individuals in the family, and the boy has a normal
karyotype. Which of the following percentages represents the approximate chance that their next child would
be affected?
A.
B.
C.
D.
E.
F23.
1%
3%
6%
12%
25%
A screening program has determined that a severe autosomal recessive condition occurs in 1/10,000
newborns in a population. A new test is devised to screen for carriers for this disease using tears collected
on filter paper. For those testing positive, carrier status can reliably be confirmed with a serum enzyme
assay. An experimental heterozygote screening program yields the following results:
Total screened: 5,000; screen positive: 135; confirmed heterozygotes: 85
The sensitivity of this screening test is closest to which of the following percentages?
A. 1%
B. 2%
C. 15%
D. 85%
E. 99%
780
F24.
A couple is referred for prenatal counseling during the 18th week of pregnancy because of
a positive 2nd trimester screen result, which revealed an increased risk for trisomy 18.
They state at the start of the session that they are uncertain whether to proceed with
amniocentesis. The pregnant patient later states that she wishes to have an amniocentesis,
but that her husband is reluctant. Which of the following management activities would be
the most appropriate next step in genetic counseling for this couple?
A. Arrange for an amniocentesis appointment as soon as possible as long as the wife wants
the procedure.
B. Refer the couple for couples counseling so that they can resolve the issue with an impartial
mediator.
C. Suggest the couple defer making a decision about whether to have the procedure, and set
up a return appointment for the following week.
D. Explore with the couple the possible outcomes associated with deciding to have (or not
have) the amniocentesis.
E. Use confrontation to clarify for the couple the source of their disagreement so that their
true feelings are known.
F25.
Which of the following reasons is the MOST important reason to screen newborns for sickle cell anemia?
A. determine the incidence of the disease in various populations
B. identify other at-risk family members
C. prevent the subsequent birth of affected siblings
D . provide genetic counseling to the parents
E.. reduce morbidity and mortality in affected infants
F26.
A man has a child with an extremely rare autosomal recessive disorder. His first wife dies and he now
marries the half sister of his first wife. The new couple is pregnant. The chance the fetus is affected
with the condition is about
A.
B.
C.
D.
E.
1 in 4
1 in 8
1 in 16
1 in 32
1 in 64
P
781
F27.
You are scheduled to see a patient whose medical records indicate that he has mild, unilateral hearing
loss, heterochromia, and a white, frontal streak in his hair. These findings would be most consistent
with which of the following syndromes?
A.
B.
C.
D.
E.
F28.
A normal-appearing infant boy who passed his initial newborn hearing screen but remained in the
NICU for an infection develops severe hearing loss after treatment with streptomycin. The baby’s
mother had hearing loss in her early 20s and the baby’s maternal grandmother became completely
deaf by her late 40s. Molecular testing of which of the following genes is most likely to shed light on
this family’s hearing loss?
A.
B.
C.
D.
E.
F29.
Alport syndrome
Pendred syndrome
Treacher Collins syndrome
Usher syndrome
Waardenburg syndrome
USH2A
GBJ2
GNJ6
MT-RNR1
MT-TS1
Which of the following types of craniofacial clefting is associated with the highest sibling risk of
recurrence (assuming non-syndromic)?
A.
B.
C.
D.
E.
bilateral cleft lip and palate
cleft soft palate alone
cleft hard palate alone
unilateral cleft lip alone
unilateral cleft lip and palate
782
F30.
A 50-year old woman presents for genetic counseling because her father (age 75) has
recently been diagnosed with Alzheimer’s disease. The woman is concerned about this
family history because her 30-year old daughter is currently pregnant with her first child.
She states that she wants to pursue genetic testing, in order to find out whether her daughter
and grandchild are at risk. Which of the following interventions is the best response to this
woman?
A. Arrange genetic testing to determine the ApoE genotype for this 50-year old patient.
B. Arrange genetic testing for the 75-year old father to determine whether he carries a ApoE
genotype associated with increased risk, before offering testing to your patient.
C. Arrange a genetic counseling appointment for the 30-year old daughter, since she is
currently pregnant and it will be quickest to start with offering testing for her.
D. Decline to arrange genetic testing in this family, because of the limited clinical validity and
utility.
E. Determine what this 50-year patient hopes to gain from having testing, and what the
interpretation will be for each potential result.
F31.
In the U.S., when a woman with a singleton pregnancy has had a positive high MSAFP of 3.5 MoM
between 15 - 20 weeks gestation which of the following outcomes for the baby is most likely to
occur?
A.
B.
C.
D.
E.
F32.
have an open neural tube defect
have an abdominal wall defect
miscarry or be stillborn
be small-for-dates
be normal at birth
The likelihood that a child resulting from incest between two first-degree relatives will have a
significant abnormality (e.g., birth defect, AR disorder, intellectual impairment) is estimated to be
closest to which of the following percentages?
A.
B.
C.
D.
E.
5%
10%
15%
25%
40%
783
F33
After a genetic counseling appointment with a pregnant patient, a genetic counselor
recognizes that she is feeling angry towards her patient because the patient is continuing to
use alcohol and marijuana during the pregnancy. The counselor seeks out a colleague to
discuss the case, and realizes that her reaction stems from her own recent experience with
miscarriage despite attempting to maintain a healthy lifestyle. Which of the following
concepts best describes the counselor’s reaction to this prenatal patient?
A.
B.
C.
D.
E.
F34.
Counter-transference
Transference
Confrontation
Advanced empathy
Self-reflection
Your office receives a phone call from a woman who says that she wants to self-refer herself
for genetic counseling and testing because of a family history of Huntington disease. She
also states that she wants to know more about what protection there is against possible
discrimination in insurance coverage. Which of the following statements represents the
clearest description of the protection provided by the Genetic Information Nondiscrimination
Act of 2008 (GINA)?
A. GINA prohibits health insurers from making decisions about insurance coverage based on
whether someone is affected and manifests a genetic condition.
B. GINA prohibits health insurers from using the results of genetic testing to make decisions
about insurance coverage.
C. GINA prohibits health insurers from using the results of genetic testing to make decisions
about insurance coverage, except for group policies at companies which employ fewer than
15 people.
D. GINA prohibits insurance companies from using the results of genetic testing in decisions
about insurance coverage.
E. GINA requires insurance companies to provide coverage for genetic testing.
F35.
A woman has a son with an X-linked recessive lethal disorder, and her maternal half-brother
had the same X-linked recessive lethal disorder. There is no other family history of the
condition. Her daughter has one healthy son. What is the daughter’s chance to be a carrier?
A. 1/4
B. 1/3
C. 1/2
D. 2/3
E. 5/6
784
F36.
You are scheduled to see a woman whose family history is consistent with inheritance of an
autosomal dominant RB1 mutation with reduced penetrance. Your patient’s brother,
maternal uncle, and maternal grandmother all had bilateral retinoblastoma. However, your
patient and her mother have both had a normal ophthalmologic exam. Review of the
literature reveals that RB1 mutations with 90% penetrance have been documented. If an
RB1 mutation with 90% penetrance is segregating in this family, what is the chance for your
patient’s offspring to inherit the RB1 mutation?
A.
B.
C.
D.
E.
F37.
0
1/40
1/22
1/11
½
You are scheduled to see a patient at 17 weeks gestation because of a positive second
trimester maternal serum screening result which showed a maternal serum alphafetoprotein
(AFP) level of 2.59 multiples of the median (MOM). During the initial contracting process,
your patient tells you that she is certain that she wants to proceed with amniocentesis,
because of the likelihood of fetal anomalies. Which of the following options is the most
appropriate next step?
A. Ask the patient to describe for you what she has been told about her positive screening
result.
B. Determine the patient’s primary sources of emotional support during her pregnancy.
C. Educate the patient about the possible causes of an increased maternal serum AFP result.
D. Explain the risks and limitations associated with amniocentesis.
E. Provide the patient with the informed consent form to sign, indicating that she is choosing
to proceed with amniocentesis.
F38.
A couple comes to you for genetic counseling because the husband had two siblings who
died from cystic fibrosis (CF; an autosomal recessive disease with an incidence of
approximately 1 in 2,500 livebirths). No molecular testing was done for either of his
siblings. Both members of the couple choose to have carrier testing, using a mutation panel
test which detects 90% of CF mutation carriers in their population. You are surprised to find
that the wife receives a positive result, and is a carrier of the Phe508del mutation. However,
the husband’s test is negative; no mutations are identified. What is probability that their first
child will be affected?
A.
B.
C.
D.
E.
1/2500
1/150
1/44
1/24
1/6
785
F39.
You see a 4-year-old boy and his parents for a follow-up visit in genetics clinic. He was
initially referred from craniofacial clinic for a sub-mucous cleft, delayed speech, a ventricular
septal defect, and characteristic facial features. The parents were told these features may be
consistent with a deletion of chromosome 22q11.2, and the syndrome was briefly described.
The chromosomal microarray analysis you obtained following his initial visit revealed a
22q11.2 deletion and the family comes to discuss the results. After disclosing the
confirmation of the diagnosis to the parents, which of the following actions is the most
appropriate next step?
A. Assess the parents’ reaction to this diagnosis, then ask if they would like to hear more
information.
B. Describe the 22q11.2 deletion syndrome and the clinical features associated with this
diagnosis.
C. Explain how 22q11.2 deletion syndrome is inherited and discuss testing for other family
members.
D. Inform the parents of the limitations and potential benefits of chromosomal microarray
testing.
E. Provide support and reassurance for the parents that everything will be okay.
F40. Mr. and Mrs. B are referred for genetic counseling because of a family history of cystic fibrosis.
Mr. B and Mrs. B are first cousins (their mothers are sisters), and Mrs. B’s brother was affected
with cystic fibrosis. No molecular testing was done, and there is no further family history of
CF. The chance that Mr. and Mrs. B’s first child will be affected with CF is closest to which
of the following probabilities?
A.
B.
C.
D.
E.
F41.
1/32
1/24
1/18
1/8
1/6
A woman (II-4) had a half-brother who died of a lethal disorder in his early twenties. Her
15-year-old son (III-5) has inherited the same disorder. Her daughter (III-4) has one healthy
son (IV-1). There is no other family history of the condition. Based on the most likely
inheritance pattern consistent with the pedigree below, what is the daughter’s (III-4) chance
to be a carrier?
A. 1/4
B. 1/3
C. 1/2
D. 2/3
E. 5/6
786
Answers to Genetic Counseling Questions F1-F40
F1.
Gaucher of the nonneuronopathic type is clearly the most frequent at about 1 in 600-1,000. Tay
Sachs is generally quoted at about 1 in 3,600, and the incidence for CF is not well documented
but is probably about the same or less frequent than Tay Sachs and definitely would not be more
frequent than Tay Sachs.
The correct answer is D.
F2.
The chance the man is a carrier is 2/3. The chance his niece (sexual partner) is a carrier is about
1/3. The chance of an affected is about 2/3 x 1/3 x 1/4 = 1/18.
The correct answer is C.
F3.
This is a simple Bayesian calculation.
Prior
Conditional
Joint
Post
0.5
0.5 x 0.5
0.125
0.2
0.5
1.0
0.5
0.8
Thus the probability that individual II-4 is a carrier is 0.2 and the probability that individual III-4
is a carrier is 0.1.
The correct answer is E.
F4.
The prior probability that the pregnant couple are both carriers is 0.25. Calculations to take into
account that the two normal offspring are provided below.
Prior
Conditional
Joint
Post
0.25
0.75 x 0.75
0.14
0.157
0.75
1.0
0.75
0.843
The posterior probability that they are both carriers is 0.157. This number times 0.25 times the
risk for an affected of 0.039.
The correct answer is D.
F5.
One way to make this calculation is to use a Bayesian method. The point is that the correct
answer is not exactly 0.008 which is the probability that a person is a carrier times the probability
that a carrier will have a negative test. The proper calculation is shown below.
The correct answer is D.
Prior
Conditional
Joint
Post
1/25
0.20
0.008
0.0083
787
24/25
1.0
0.96
0.9917
F6.
The correct answer is B.
F7.
The most frequent identifiable congenital viral infection is with CMV, although the majority of
these infants are asymptomatic.
The correct answer is A.
F8.
As mentioned above, the majority of infants with congenital CMV are asymptomatic.
The correct answer is E.
F9.
According to Harper p. 277, the risk would be 1 in 1000 or less with this exposure.
The correct answer is E.
F10.
According to Harper p.219, the recurrence risk for all of these cardiac malforrmations is relatively
low (1.0-1.3%).The recurrence risk for endocardial fibroelastosis is substantially higher at 3.8%.
The correct answer is E.
F11.
This pedigree is not consistent with X-linked inheritance and complete penetrance because of the
evidence of male to male transmission of the trait. Sex limited autosomal dominant inheritance
would be a possibility except for the statement to assume complete penetrance and the
transmission through an asymptomatic male. The pedigree would fit well for a mutation in an
imprinted gene where the gene is expressed only from the maternal allele, and the phenotype
occurs only when the mutant gene is inherited from the mother.
The correct answer is A.
F12.
The correct answer is C.
F13.
The probability that the father of the current pregnancy would be a carrier is 2/3. The probability
that the mother is a carrier is ½. If they are both carriers, the probability that the fetus would be
affected is ¼. Thus, 2/3 x ½ x ¼ = 1/12.
The correct answer is C.
F14.
This is an intercross data set for the affected child and the fetus. The linkage analysis indicates
that the fetus is either homozygous affected or is a noncarrier. A heterozygous result for the
disease would involve homozygosity for the upper or lower dinucleotide allele as is the case for
the unaffected sib. There is a 50-50 probability that the fetus is affected.
The correct answer is C.
F15.
This requires a Bayesian calculation. The woman’s mother is an obligate carrier of this disorder.
Prior
Conditional (2 nl sons)
Joint
Post
Is carrier
1/2
1/4
1/8
1/5
Is not carrier
1/2
1
1/2
Hence she has a 1/5 risk of being a carrier, so the chance that her male fetus is affected is
1/5 x ½ = 1/10.
The correct answer is B.
788
F16.
All mothers of males with fragile X full mutations have at least a pre-mutation allele.
Hence the correct answer is E, 0.
F17.
The correct answer is A.
F18.
The correct answer is B.
F19.
The correct answer is E.
F20.
The background risk for the general population is about 3%; the incremental risk for first cousins
is another 3%.
The correct answer is C.
F21.
The correct answer is A.
F22.
About ¼ of males with aqueductal stenosis have the X-linked form; the risk to the next child is
P[XL] x P[male] x P[affected] = ¼ x ½ x ½ .
The correct answer is C.
F23.
q2 = 1/10,000; q = 1/100, 2pq = 1/50. Therefore should have 100 carriers among 5,000
individuals; found 85, so 85% sensitivity.
The correct answer is D.
F24.
The correct answer is D.
F25.
The correct answer is E.
F26.
The man is a definite carrier. There is a 1 in 2 chance that his double mother-in-law is a carrier
and a 1 in 2 chance that she passed the altered gene to his new spouse ( ½ X ½). The chance that
the father will pass on an altered gene is ½ . The chance that the mother will pass on an altered
gene to the fetus is ½ X ½ X ½ . The chance the fetus is affected is about 1 in 16.
The correct answer is C.
F27.
Patient has classic features of Waardenburg’s and, apart from deafness, no apparent signs of the
other syndromes: Alport (e.g., glomerulonephritis, lenticonus), Usher (e.g., RP, vestibular
dysfunction), Treacher-Collins (e.g., mandibulo-facial dysostosis; cleft palate; Pendred (e.g.,
goiter, vestibular dysfunction).
The correct answer is E
F28.
Baby’s deafness has apparently arisen in reaction to aminoglycoside exposure. Later age of
hearing loss in (presumably unexposed) mother and maternal grandmother is consistent both with
maternal inheritance and effects of mtDNA mutation: A1555G in MT-RNR1; a) is one of the
Usher genes and is AR, not accounting for the three-generation HL hx.; b) is connexin 26—
usually AR, though sometimes AD, but not likely to have been associated with initially normal
NB hearing screen with loss after streptomycin exposure; c) is connexin 30, presumably AR, and
e) the other mtDNA mutation, is a heteroplasmic mutation and associated with progressive,
childhood-onset SN HL with skin changes, but not with aminoglycoside toxicity.
The correct answer is D
789
F29.
The correct answer is A. a) = 5.7-8% b) = 3.8%, c) = 5.4%, d) = 1.6-2.5%, and
e) = 3.3-4.2%.]
F30.
The correct answer is E.
F31.
With an MSAFP 3.5 MoM, 18% will have problem found with US or amnio; if US and amnio are
normal, and additional 28.5 will have problem later in pregnancy, but the baby is still most likely
to be normal at birth. At a 2.5 MoM cut-off, about 2-3% of women will have a positive high
MSAFP. Of these, 0.1% of all women would be expected to have a fetus with a NTD, and about
an equal percentage will have a fetus with ventral wall defect. For an MSAFP between 3 and 4.9
MoM, the overall risk for poor outcome is about 41%. With MSAFP >5 MoM, the risk for an
adverse outcome is about 91%.
The correct answer is E
F32.
The correct answer is E. 40%
F33.
The correct answer is A.
F34.
The correct answer is B
GINA prohibits health insurers from requesting genetic information of an individual or
the individual’s family members, or using it for decisions regarding coverage, rates, or
preexisting conditions. Answer A is incorrect, because GINA does not does not
prohibit the health insurer from determining eligibility or premium rates for an individual
based on the manifestation of a disease or disorder in that individual. Answer C is
incorrect; it is the employment provisions of GINA which do not apply if an employer
has fewer than 15 employees. Answer D is incorrect, because the insurance provisions of
GINA only cover health insurance; they do not extend to long-term care, life, or disability
insurance. Answer E is incorrect, because GINA does not mandate coverage for any
particular genetic test or treatment.
F35.
The correct answer is B
Keywords: Bayesian calculation
Explanation: The woman is herself an obligate carrier, since she has an affected son and
an affected maternal half-brother. So the daughter’s prior (Mendelian) risk to be a
carrier is 1/2. Use Bayesian calculation to calculate her chance to be a carrier given that
she has a healthy son.
Prior
Condition (healthy son)
Joint
Posterior
Carrier
1/2
1/2
1/4
1/3
Non-carrier
1/2
1
1/2
2/3
The probability that the daughter is a carrier is 1/3.
790
Ȉ F36.
The correct answer is C
Keywords: Retinablastoma, reduce penetrance
Explanation: Your patient’s mother is an obligate carrier of the RB1 mutation in the
family. Use a Bayesian calculation to determine the chance that your patient is a carrier,
given that she has a normal ophthalmologic exam.
Prior
Condition (nl ophth.
exam)
Joint
Posterior
Carrier
1/2
1/10
Non-carrier
1/2
1
1/20
1/11
10/20
10/11
Ȉ Probability your patient is a carrier = 1/11.
Probability your patient’s child will inherit the mutation = 1/22
F37.
The correct answer is A
Keywords: Maternal serum screen, genetic counseling
Explanation: The most appropriate next step will be to assess the patient’s current
understanding of the implications of the positive screening result. This will allow you to
provide the further counseling information and support in a manner that will be most
helpful to your patient. Clearly the session will also include providing information about
the possible causes of a positive screen result (C) and her options for further testing (D);
however, this should follow an assessment of her current understanding. During the
session, you will also assess her sources of emotional support (B), but this would not be
done during the initial contracting process.
F38.
The correct answer is D
Keywords: Cystic fibrosis recurrence risk, Bayesian calculation
Explanation: The probability that the wife is a carrier is 1. Use Bayesian calculation to
calculate the probability that the husband is a carrier.
Prior
Condition (neg. carrier
test)
Joint
Posterior
Carrier
2/3
1/10
Non-carrier
1/3
1
Ȉ 2/30
1/6
10/30
5/6
The probability that their child will be affected is 1/6 * 1 * ¼ = 1/24
791
F39.
The correct answer is A
It is important to assess what the parents’ reaction is, before providing further
information. Your plan for the session will also include further discussion of the
syndrome including natural history and inheritance pattern and other testing that may be
indicated for the patient or family members, and you should also have a plan in place for
providing support for the family. However, in order to achieve those goals, it will be
important to first assess how the parents have heard the diagnostic information that you
have provided, and what their reaction is.
F40.
The correct answer is B
Solution: Probability (Mrs. B is a carrier) = 2/3, because both of her parents are carriers,
and she is unaffected. Probability (Mr. B is a carrier) = 1/4, because his aunt is an
obligate carrier, and his mother has a 1/2 chance to be a carrier. Probability (affected
child) = 2/3 x 1/4 x 1/4 = 1/24.
F41.
CORRECT ANSWER: B
Keywords: Bayesian calculation
Explanation: The woman is herself an obligate carrier, since she has an affected son and
an affected maternal half-brother. So the daughter’s prior (Mendelian) risk to be a
carrier is 1/2. Use Bayesian calculation to calculate her chance to be a carrier given that
she has a healthy son.
Prior
Condition (healthy son)
Joint
Posterior
Carrier
1/2
1/2
1/4
1/3
Non-carrier
1/2
1
1/2
2/3
The probability that the daughter is a carrier is 1/3.
792
Ȉ G. Cancer Genetics Questions G1 – G47
G1.
Which of the following cancers is the most common malignancy after colorectal cancer seen in
families with hereditary non-polyposis colon cancer (Lynch syndrome)?
A.
B.
C.
D.
E.
G2.
A four-year-old boy is diagnosed with pulmonary pleuroblastoma and the father has multicystic
goiter? In taking a family history of cancer which of the malignancies listed below might be
expected to be found given this history?
A.
B.
C.
D.
E.
G3.
Adrenal corticocarcinoma
Breast cancer
Lung cancer
Rhabdomyosarcoma
Testicular cancer
A newborn boy is found to be small for gestational for both weight and length and has radial
aplasia. Fanconi anemia is confirmed by laboratory studies. While counseling the parents you
are most likely to mention which of the following time periods as the average age of clinical
evidence for bone marrow failure in Fanconi anemia?
A.
B.
C.
D.
E.
G4.
Ovarian cancer
Bile duct cancer
Endometrial cancer
Breast cancer
Pancreatic cancer
1 to 2 months
6 to 7 months
2 to 3 years
6 to 8 years
12 to 14 years
A 2-year-old boy presents to his pediatrician with mild dysmorphic features (frontal bossing) and
a family history reveals a mother with basal cell nevus syndrome (Gorlin syndrome). In addition
to skin cancer which of the following malignancies is the most common pediatric malignancy
typically identified in this disorder?
A.
B.
C.
D.
E.
Glioblastoma
Medulloblastoma
Neuroblastoma
Wilms’ tumor
Leukemia
793
G5.
A twelve-year-old child presents with leukoplakia, abnormal nails and evidence of bone marrow
failure. Your hematology colleague is concerned about a possible underlying genetic diagnosis.
Which of the following tests is most likely to yield a diagnosis?
A.
B.
C.
D.
E.
G6.
Based on the pedigree documented below, which of the following genes is most likely to be
mutated causing the hereditary cancer syndrome in this family?
A.
B.
C.
D.
E.
G7.
Comparative genomic hybridization
Diepoxybutane breakage analysis
Quantitative radiation sensitivity
Sister chromatid exchange
Telomere length measurement
ATM
BRCA1
BRCA2
P53
PTEN
Approximately 10% of children with hepatoblastoma have which of the following disorders?
A.
B.
C.
D.
E.
Ataxia telangiectasia
Familial Adenomatous Polyposis
Li Fraumeni
Neurofibromatosis I
Von Hippel Lindau
794
G8.
A five-year-old child presents with rash on the cheeks, very short stature and she was recently
diagnosed with leukemia. Your hematology colleague is concerned about a possible underlying
genetic diagnosis. Which of the following tests is most likely to yield a diagnosis?
A.
B.
C.
D.
E.
G9.
Missense mutations are the most common type of constitutional mutation found in which
syndrome?
A.
B.
C.
D.
E.
G10.
Familial Adenomatous Polyposis
Li Fraumeni Syndrome
Multiple Endocrine Neoplasia Type I
Multiple Endocrine Neoplasia Type IIA
Multiple Endocrine Neoplasia Type IIB
As a geneticist following patients with NF1, which of the following clinical features is the most
likely to be detected only after a patient has become a teenager?
A.
B.
C.
D.
E.
G12.
Cowden Syndrome
Familial Adenomatous Polyposis
Hereditary breast ovarian cancer
Neurofibromatosis I
von Hippel Lindau Syndrome
Prophylactic thyroidectomy by age 5 is recommended for children who are diagnosed with which
of the following heritable cancer syndromes?
A.
B.
C.
D.
E.
G11.
Comparative genomic hybridization
Diepoxybutane breakage analysis
Quantitative radiation sensitivity
Sister chromatid exchange
Telomere length measurement
Acoustic schwanoma
Axillary freckling
Learning disabilities
Malignant peripheral nerve sheath tumor
Optic pathway tumor
In women who carry a BRCA1 mutation, the lifetime risk for ovarian cancer is closest to which
of the following percentage ranges?
A.
B.
C.
D.
E.
1-2%
5-15%
15-20%
25-60%
75-90%
795
G13.
In men who carry a BRCA2 mutation, the lifetime risk of male breast cancer is closest to which
of the following percentage ranges?
A.
B.
C.
D.
E.
1-2%
5-10%
15-25%
40-50%
70-80%
Questions G14-15
G14. You are asked to see the parents of a child newly diagnosed with unilateral retinoblastoma. The
mother is pregnant and is concerned about the risk of having a second child with retinoblastoma.
The parents have normal eye exams and have no family history of retinoblastoma. What is the
approximate likelihood that the future sibling will develop retinoblastoma?
A.
B.
C.
D.
E.
G15.
<0.1%
0.1%
1.0%
6.0%
45%
The parents in question 14 return to see you again after their child with unilateral retinoblastoma
underwent enucleation and genetic testing. The results of the tumor and blood analysis at the
RB1 locus are shown below. Now what is the likelihood that the future sibling will develop
retinoblastoma?
Allele 1
Allele 2
Tumor Promoter Methylation 683delC
Blood
A.
B.
C.
D.
E.
G16.
Normal
Normal
<0.1%
0.1%
1.0%
6%
45%
Which of the following lists gives the correct order for the risk of developing colorectal cancer in
these syndromes? (HNPCC – hereditary non-polyposis colon cancer; FAP – familial
adenomatous polyposis; JPC – juvenile polyposis coli; PJS – Peutz-Jeghers syndrome)?
A.
B.
C.
D.
E.
FAP>HNPCC>JPC>PJS
FAP>HNPCC>PJS>JPC
FAP>JPC>HNPCC>PJS
HNPCC>FAP>JPC>PJS
HNPCC>JPC>FAP>PJS
796
G17.
Molecular testing identifies a nonsense mutation in the RB1 gene of a two-year-old boy with
bilateral retinoblastoma. Parental molecular analyses are normal. Based on the pedigree below
which of the following percentages most closely approximates the parents’ risk of recurrence in
their next child?
A.
B.
C.
D.
E.
G18.
A number of genome wide association studies have been performed to identify susceptibility
genes for breast cancer. These involve using large case and control cohorts to identify which
polymorphisms spread throughout the genome are differentially found in cases or controls by
genotyping on SNP arrays. Which of the following breast cancer susceptibility loci was
discovered using this method?
A.
B.
C.
D.
E.
G19.
ATM
CHEK2
FGFR2
PALB2
PTEN
Families demonstrating low penetrance retinoblastoma are most likely to have which of the
following types of mutation identified in the RB1 gene?
A.
B.
C.
D.
E.
G20.
1%
6%
15%
20%
25%
Deletions
Promoter mutations
Nonsense mutations
Splice site mutations
Translocations
Mutation or disruption of an imprinted locus has been associated with which of the following
syndromes?
A.
B.
C.
D.
E.
Familial Adenomatous Polyposis
Beckwith Wiedemann Syndrome
Gorlin Syndrome
Neurofibromatosis II
Von Hippel Lindau
797
G21.
Overall, children with Beckwith-Weidemann syndrome have an increased risk of developing
Wilms tumor. Molecular testing can help to clarify the cancer risk. Which of the following
results on analysis of a blood sample from a child with BWS has the highest risk for Wilms
tumor?
A.
B.
C.
D.
E.
G22.
Which of the following autosomal dominant cancer predisposition syndromes results from a
mutation which activates an oncogene?
A.
B.
C.
D.
E.
G23.
Costello Syndrome
Gorlin Syndrome
Juvenile Polyposis Coli
Neurofibromatosis 2
von Hippel Lindau Syndrome
An otherwise healthy woman with early onset breast cancer has genetic testing of the BRCA1 and
BRCA2 genes. The result includes a missense mutation in BRCA2 that is a variant of uncertain
significance (VUS). The testing laboratory provides you additional information about this
variant. Which of the following data provides the strongest evidence against the VUS being
associated with hereditary breast/ovarian cancer syndrome?
A.
B.
C.
D.
E.
G24.
Abnormal methylation of H19/IGF2
Abnormal methylation of LIT1/KCNQ10T1
Paternal duplication of 11p15
Point mutation in CDKN1C
Uniparental disomy of 11p15
The finding of a frameshift mutation in BRCA1 in the same patient.
The finding of a frameshift mutation in BRCA2 in the same patient.
The VUS has not been previously reported in control populations.
The VUS occurs in a consensus splice site.
The VUS is a nonsense mutation.
In evaluating a family for hereditary non-polyposis colon cancer which test of the tumor should
be performed prior to deciding about testing for constitutional mutations?
A.
B.
C.
D.
E.
Comparative genomic hybridization
FISH analysis at MSH2
Loss of heterozygosity at the MSH2 locus
Microsatellite instability
Tumor cell karyotype
798
G25.
A 25-year-old woman has invasive colon cancer and hundreds of polyps. She has a total
colectomy. In addition to routine care for the general population, which of the following medical
surveillance evaluations will she continue to require?
A.
B.
C.
D.
E.
G26.
Which of the following explanations accounts for the increased risk of skin cancer for patients
with Xeroderma Pigmentosum?
A.
B.
C.
D.
E.
G27.
Amplification of chromosome 2 including the ALK locus
Inversion of chromosome 2 involving the ALK gene
Deletion of the ALK gene
Missense mutation of the ALK gene
Translocation between ALK and EML4
Genetic analysis of the tumor specimen of a fifty-five year old man with colon cancer reveals
microsatellite instability. The likelihood that analysis of a blood sample from this patient will
reveal a germline mutation in one of the mismatch repair genes is closest to which of the
following percentages?
A.
B.
C.
D.
E.
G29.
Their cells are deficient in mismatch repair.
Their cells are deficient in nucleotide excision repair of UV-induced DNA lesions.
Their cells are deficient in transcription-coupled repair of UV-induced DNA lesions.
Their cells are sensitive to crosslinking agents.
Their cells have a DNA repair gene mutation and lose the second copy frequently.
The ALK gene undergoes a variety of different mutational events in cancers. Molecular analysis
of a blood sample from a child with neuroblastoma is most likely to reveal the following?
A.
B.
C.
D.
E.
G28.
Annual ophthalmic exam
Breast MRI
Endometrial biopsy every 2 years
Screening skin exam for melanoma yearly
Upper GI endoscopy every 2-3 years
<1%
1%
5%
25%
50%
A child is diagnosed with pulmonary pleuroblastoma. You order genetic testing. Which specific
gene test is most likely to be positive?
A.
B.
C.
D.
E.
CTNNB1
DICER1
PTCH1
SMARCB1
SUFU
799
G30.
Heterozyous mutations in CDH1 predispose to diffuse gastric cancer. About which additional
cancer risk do carriers of CDH1 mutations need to be informed?
A.
B.
C.
D.
E.
G31.
A 45-year-old man is diagnosed with chromophobe histology of kidney cancer. Considering
genetic condition(s) which predispose to this type of cancer, about what other medical problem
would you ask?
A.
B.
C.
D.
E.
G32.
Embryonal rhabdomyosarcoma
Chromophobe renal cell cancer
Lobular breast cancer
Papillary thyroid cancer
Small cell lung cancer
Pulmonary arteriovenous malformation
Pulmonary fibrosis
Pulmonary hemangiomas
Recurrent pneumonias
Spontaneous pneumothorax
16-year-old presents to the emergency room with intussusception. After the patient is
stabilized you are asked to see the patient and recommend genetic testing? Which gene
test do you order?
A.
B.
C.
D.
E.
APC
BMPR1A
PTEN
SMAD4
STK11
800
G33.
A 37-year-old woman was recently diagnosed with breast cancer and comes for a
consultation. Which option describes the most appropriate use of the current models and
guidelines?
A. The BRCAPro algorithm can be used to predict the likelihood that she carries a
BRCA1/2 mutation.
B. The Chompret criterion requires additional family history data to determine the
likelihood that she carries a BRCA1/2 mutation.
C. The Claus tables can be used to predict her likelihood of developing ovarian cancer.
D. The Gail model can be used to predict her likelihood of developing a second breast
cancer.
E. The NCCN guidelines suggest reflex TP53 testing if BRCA1/2 testing is negative.
G34.
Alterations in genes encoding transcription factors play an important role in the
development of hematopoietic disorders. Consider an infant with Trisomy 21 who
develops transient myeloproliferative disorder. A somatic mutation in which gene is most
likely to be found when examining the bone marrow?
A.
B.
C.
D.
E.
G35.
AKT
ETV6
GATA1
IKZF1
RUNX1
A 6-month-old with unilateral retinoblastoma has genetic testing performed. The results
of the tumor and blood analysis at the RB1 locus are shown below. What is the
likelihood that a future sibling will develop retinoblastoma?
Retinoblastoma
tumor
Blood
A.
B.
C.
D.
E.
Allele 1
c.7510C>T (p.R261X)
Allele 2
c.7510C>T (p.R261X)
normal
normal
Homozygous C>T substitution that causes an immediate
termination codon in exon 8 of tumor DNA
<1%
1%
6%
25%
50%
801
G36.
A four-year-old girl presents to the emergency room with rectal bleeding and rectal
prolapse. The gastroenterologist removes a bleeding polyp and stabilizes the patient. A
geneticist evaluates the patient and recommends genetic testing for a next generation
panel test for polyposis/GI cancer genes. Which of the following gene tests is most likely
to be positive given the child’s clinical presentation?
A.
B.
C.
D.
E.
G37.
APC
MUTYH
MSH2
PTEN
SMAD4
You are asked by your hematology colleague to consult on a 16-year-old young man with
recent diagnosis of bone marrow failure. Examination of this patient demonstrates nail
dystrophy and oral leukoplakia (also present by history in an uncle who died from a
different cancer). What diagnostic test is most likely to yield a positive result?
A.
B.
C.
D.
E.
Antibody diversity
Diexpoxybutane breakage
Ionizing radiation sensitivity
Sister chromatid exchange
Telomere flow FISH
802
G38.
A 34-year-old woman was recently diagnosed with breast cancer and comes for a
consultation. Which of the following options describes the most appropriate use of the
current models and guidelines?
A. The Amsterdam criteria can be used to predict the likelihood that she carries a PALB2
mutation.
B. The Chompret criterion including additional family history data can determine the
likelihood that she carries a BRCA1/2 mutation.
C. The Claus tables can be used to predict her likelihood of developing ovarian cancer.
D. The Gail model can be used to predict her likelihood of developing a second primary
breast cancer.
E. The NCCN guidelines suggest reflex TP53 testing if BRCA1/2 testing is negative.
G39.
A 20-year-old woman presents with a pheochromcytoma of the adrenal gland. Further
work-up reveals hearing loss and an endolymphatic sac tumor. Genetic testing using an
available pheochromcytoma/paraganglioma panel is ordered. A positive mutation in
which of the following genes is most likely to be identified?
A.
B.
C.
D.
E.
G40.
NF1
RET
SDHB
SDHD
VHL
The 35-year-old woman shown in the pedigree (noted by a star) is concerned about her
risk of kidney cancer based on her family history of a brother who died of kidney cancer
(clear cell RCC) at age 45. Given the pedigree you order genetic testing using a kidney
cancer panel. Which of the following genes is most likely to have a pathogenic
mutation?
A.
B.
C.
D.
E.
BHD
FH
MET
PTEN
VHL
803
G41.
The FDA approved use of Crizotinib requires which of the following molecular tests of
the tumor specimen?
A.
B.
C.
D.
E.
G42.
Amplification of p53 in neuroblastoma
Deletion of RB1 in osteosarcoma
Inversion of ALK in lung cancer
Missense mutation of SRC in breast cancer
Translocation of ABL in chronic myelogenous leukemia
You are asked by your hematology colleague to evaluate an 8-year-old boy with recent
diagnosis of bone marrow failure shown in the pedigree. Examination of this patient
demonstrates absence of the radius and short stature and his sister died of head and neck
cancer at age 23. Which of the following diagnostic tests is most likely to yield a positive
result?
A.
B.
C.
D.
E.
Antibody diversity
Diexpoxybutane breakage
Ionizing radiation sensitivity
Sister chromatid exchange
Telomere flow FISH
804
G43.
A child is diagnosed with malignant rhabdoid tumor of the kidney. You order genetic
testing. Which of the following specific gene test is most likely to be positive?
A.
B.
C.
D.
E.
G44.
A 45-year-old woman is diagnosed with secondary glioblastoma after a prior astrocytoma
and the tumor is found to carry an IDH2 missense mutation. Analysis of a germline
sample from this patient is most likely to reveal?
A.
B.
C.
D.
E.
G45.
A second germline IDH2 mutation in addition to the tumor mutation
Amplification of IDH2
Deletion of one copy of IDH2
Normal sequence of the IDH2 gene
The same IDH2 missense mutation in the heterozygous state
Analysis of the total number of somatic mutations in a colon cancer specimen undergoing
whole exome sequencing demonstrates thousands of somatic mutations. Germline
genetic testing revealed a pathogenic variant in a hereditary colon cancer gene. Which of
the following genes is most likely to result in predisposition to developing a hypermutated tumor?
A.
B.
C.
D.
E.
G46.
CTNNB1
DICER1
PTCH1
SMARCB1
SUFU
APC
MSH2
MUTYH
POLE
STK11
A 25-year-old woman develops a paraganglioma. You order a 10 gene hereditary
pheochromocytome/paraganglioma panel which reveals a nonsense pathogenic variant in
SDHB. Which of the following clinical concerns is most compatible with this molecular
result?
A. The patient will require MRI screening for renal cell carcinoma
B. The patient will require careful monitoring for malignant transformation of the
paraganglioma
C. The patient will require prophylactic thyroidectomy to reduce risk of medullary
thyroid cancer
D. The patient is at decreased risk of having an affected child because SDHB is
maternally imprinted
E. The patient will require careful monitoring of Calcium given risk of parathyroid
tumors
805
G47.
There is increasing use of genetic testing for cancer patients to determine treatment
decisions. Which of the following pairs of genes and medications best describes this new
use of genetic testing?
A.
B.
C.
D.
E.
APC and immune checkpoint inhibitors
BRCA2 and PARP inhibitors
NF1 and angiogenesis inhibitors
TP53 and HER2 inhibitors
VHL and mTOR inhibitors
806
Answers to Cancer Genetics Questions G1-G47
G1.
The cumulative incidence of endometrial cancer in women who carry HNPCC mutations is
estimated to be approximately 70%. Ovarian, bile duct and pancreatic are all increased in
HNPCC. Breast cancer is not.
The correct answer is C.
G2.
Pulmonary pleuroblastoma (PPB) is part of a cancer susceptibility syndrome due to DICER1
germline mutations which includes a variety of other benign (multinodular goiter) and malignant
cancers. Rhabdomyosarcoma is found in families with PPB and DICER1 mutations.
The correct answer is D.
G3.
For this reason the patient may present with congenital anomalies or cancer prior to bone marrow
failure being clinically evident.
The correct answer is D.
G4.
There is an estimated 4-5% risk of developing medulloblastoma in patients with Gorlin
syndrome. This can represent the first clinical manifestation of the disorder and when present
precedes the development of basal cell carcinomas.
The correct answer is B.
G5.
The clinical features given are typical of DysKeratosis Congentia (DKC). Nail abnormalities are
not seen in Fanconi anemia. DKC results from a telomere defect and quantitative measure of
telomere length is the laboratory test which is now used to aid in making the diagnosis.
The correct answer is E
G6.
Cowden Syndrome is due to mutations in the PTEN tumor suppressor gene and is associated with
breast and thyroid cancer. Malignancies less frequently seen in Cowden Syndrome include
endometrial cancer and renal cell carcinoma.
The correct answer is E
G7.
Hepatoblastoma develops in very young children and recent publications recommend screening
for hepatoblastoma in children identified to carry mutations in the APC gene. Approximately
10% of children with hepatoblastoma carry a mutation in the APC gene. Beckwith Weidemann
Syndrome also carries an increased risk of hepatoblastoma.
The correct answer is B.
G8.
The clinical features are typical of Bloom syndrome, including the butterfly rash on the face.
Children with Bloom syndrome are very short. The abnormal recombination that results from
mutations in the BLM gene yields an increased number of sister chromatid exchanges which can
be used as a laboratory test to aid in making diagnosis.
The correct answer is D.
G9.
A significant percentage of patients (over 35%) with VHL carry missense mutations. In
particular, missense mutations are associated with Type II VHL. The Type II families
demonstrate an increased risk of developing pheochromocytomas. The other syndromes listed all
have truncating mutations (nonsense, frameshift and splicing) as the predominant type.
The correct answer is E.
807
G10.
The risk of medullary thyroid cancer in childhood is highest in MENIIA and MENIIB. However, in
MENIIB the age of onset is considerably lower so that prophylactic thyroidectomy is recommended
by age 1. In MENIIA it is recommended to be performed by age 5. FAP is associated with an
increased risk of thyroid cancer compared to the general population but not to an extent to warrant
prophylactic thyroidectomy.
The correct answer is D.
G11.
Options B, C, E are features of NF1 but are normally diagnosed in very young or early school
age children. Malignant peripheral nerve sheath tumors typically first appear during the teenage
or young adult years. Acoustic lesions can be diagnosed in teenagers but occur in NF2 not NF1.
The correct answer is D.
G12.
There significant variation among different published studies, however, most studies have
resulted in estimates consistent with 25-60%.
The correct answer is D.
G13.
Results from the Breast Cancer Linkage Consortia give a lifetime estimate of 6% for men who
carry BRCA2 mutations. Although the lifetime risk is lower, several recent papers have
demonstrated that some men with breast cancer carry mutations in the BRCA1 gene.
The correct answer is B.
G14.
Overall about 15% of unilateral retinoblastoma patients carry a germline mutation. There is the
risk that this mutation may be the result of germline mosaicism in the parents (which for offspring
with bilateral retinoblastoma results in a 5-7% recurrence risk for another child). In the unilateral
setting described here this risk is diminished but is still approximately 1% for each pregnancy that
another child may develop retinoblastoma.
The correct answer is C.
G15.
The results shown above are typical for a sporadic case of retinoblastoma. The frameshift
mutation is not found in the blood sample and thus must have occurred after fertilization.
Promoter methylation is an event which occurs during tumor development and is not usually
inherited. With this result the likelihood that the parents carry a germline RB1 mutation is
extremely low and the recurrence risk is similar to the population risk (1 in 15,000-30,000).
The correct answer is A.
G16.
FAP and HNPCC have significantly higher risks, 95% and 70%, respectively, then JPC and PJS.
Although initially controversial is it now clear that JPC families do have an increased risk of
colorectal cancer (~50%) and that appears to be particularly true of families that carry mutations
in the SMAD4 gene.
The correct answer is A.
G17. There is a significant risk of germline mosaicism in the fathers of children with constitutional RB1
mutations resulting in a 6-7% recurrence risk. Thus, even if parents test negative it is important to
have all subsequent siblings tested at birth for the mutation identified in the affected child in order
to determine if the newborn needs surveillance for retinoblastoma.
The correct answer is B.
808
G18.
Inherited mutations in all five genes result in an increased risk of developing cancer. However, in
GWAS studies you examine polymorphisms that appear in the general population with a minor
allele frequency of about 5% or greater. For ATM, PALB2 and PTEN there are a large number of
individually rare variants that cause an increased risk of cancer. For CHEK2 the recurrent
mutations in the population have a frequency of about 1% and are frameshift and deletions which
are not easily detected by GWAS genotyping assays. A polymorphism in FGFR2 found in 0.38
of the case population is one of the most consistent GWAS hits for breast cancer susceptibility.
The correct answer is C.
G19.
The splice site mutations appear to result in a small amount of correctly spliced product that is
functional. There are also low penetrance families that carry missense mutations in conserved
domain that result in some residual activity of the RB1 protein.
The correct answer is D.
G20.
BWS often results from disruption of imprinting. Evidence for this includes the presence of UPD
in children with BWS as well as families with affected cousins that inherit the mutation through
their mothers.
The correct answer is B.
G21.
The genetic etiology of Beckwith Weidemann syndrome is complex and there are overlapping
clinical features seen in children with different underlying molecular abnormalities. The IGF2
growth factor is thought to be important for growth control and tumor development. Children
with abnormal methylation of the H19/IGF2 locus have a high Wilms tumor risk compared with
the other options given. Clinically, children with hemihypertrophy and organomegaly also have a
high Wilms tumor risk.
The correct answer is A.
G22.
Costello syndrome is due to classic mutations which constitutively activate the H-Ras oncogene.
The second wild-type H-Ras gene is retained in the tumor and transfection assays of the mutant
gene into NIH3T3 cells demonstrate transforming activity.
The correct answer is A.
G23.
The interpretation of VUS results is quite complex. In the case of BRCA2, patients who carry two
deleterious mutations demonstrate a Fanconi phenotype. Thus, the finding of a frameshift
mutation in the same patient argues strongly against pathogenicity. There are a number of
individuals who carry mutations in both BRCA1 and BRCA2 so (A) is not that informative. There
are many rare benign variants in BRCA2 so the lack of data in a control population is not helpful.
Options D and E are both findings for mutations that are associated with hereditary breast/ovarian
syndrome.
The correct answer is B.
G24.
Microsatellite instability is seen in tumors from HNPCC families. This is evidenced by increases
and decreases in the size of a repeat in the tumor specimen when compared to normal DNA from
the same patient. Some laboratories opt to use immunohistochemistry for the expression of the
MSH2 and MLH1 proteins as an alternative to microsatellite instability testing. Methylation
studies of the MLH1 gene are normally performed next as that is a common somatic change
which results in microsatellite instability.
The correct answer is D.
809
G25.
The clinical description is classic for familial adenomatous polyposis (FAP). Adults with FAP
develop gastric polyps and are at increased risk for small bowel carcinomas. Thus, they require
lifelong surveillance of the upper GI tract after removal of the colon.
The correct answer is E.
G26.
Xeroderma pigmentosum (XP) is the classic autosomal recessive cancer predisposition syndrome.
Their cells do not correct UV-induced DNA lesions correctly. In Cockayne syndrome, cells are
deficient for transcription-coupled repair and patients do not have a significant cancer risk. In the
majority of XP subtypes, cells are deficient for nucleotide excision repair with resulting
mutagenesis and cancer predisposition.
The correct answer is B.
G27.
All of the options are rearrangements seen in ALK except C. Deletions are found in tumor
suppressor genes, not oncogenes. The question asks about analysis of a blood sample, so we are
looking for the cause of familial neuroblastoma which is the result of inheriting missense
mutations in ALK. The inversions, translocations and amplifications are somatic rearrangements
in ALK that are found in tumors.
The correct answer is D.
G28.
The correct answer is C.
Source of topic: slides
Keywords: colon cancer, Lynch syndrome, MLH1
Explanation: Approximately 15-18% of colon cancers reveal microsatellite instability. However,
the vast majority of these tumors have loss of mismatch repair function through epigenetic
silencing of MLH1. It is only about 5% of these MIS+ tumors (or 2% of the overall population)
that demonstrate germline mutation in one of the MMR genes.
G29.
The correct answer is B.
Source of topic: slides
Keywords: pulmonary pleuroblastoma, microRNA, tumor suppressor gene
Explanation: The majority of patients (>80%) with pulmonary pleuroblastoma carry a germline
mutation in DICER1, an enzyme involved in microRNA processing. CTNNB1, beta catenin,
undergoes somatic mutation in a number of tumors; PTCH1 mutations result in Gorlin sydrome;
SMARCB1 results in rhabdoid tumors and SUFU mutations result in medulloblastoma.
G30.
The correct answer is C.
Source of topic: slides
Keywords: breast cancer, gastric cancer
Explanation: CDH1 encodes E-cadherin and mutations in this gene were first identified in
familial diffuse gastric cancer. Analysis of these families demonstrated that there was an
unexpected prevalence of lobular breast cancers in these families. Subsequently, analysis of
familial lobular breast cancer kindreds also revealed CDH1 mutations.
G31.
The correct answer is E.
Source of topic: slides
Keywords: renal cell cancer
Explanation: Chromophobe renal cancer is associated with the Birt-Hogg-Dube syndrome.
Individuals with BHD often have a significant history of spontaneous pneumothorax as teen-agers
or young adults. Pulmonary fibrosis is seen in dyskeratosis congenita and pulmonary AVMs can
be seen in hereditary hemorrhagic telangiectasia. Pulmonary hemangiomas are not a typical
810
finding of VHL although hemangioms of the retina or hemangioblastoma of spine and cerebellum
are seen in VHL. Patients with VHL develop clear cell RCC not chromophobe.
G32.
The correct answer is E.
Source of topic: slides
Keywords: polyposis, hamartoma
Explanation: All of the genes listed can result in polyposis syndromes of different types.
However, intussusception is a life-threatening complication of Peutz-Jeghers syndrome
and can occur at much older ages than seen in the general population. Once the diagnosis
of PJS is made it is important to warn patients and parents of this risk and the need for
immediate medical care of symptoms if an acute abdomen develops.
G33.
The correct answer is A.
Source of topic: slides.
Keywords: BRCA1/2, risk prediction
Explanation: BRCAPro was developed to predict the likelihood of BRCA1/2 mutation
and can be used in women with or without a personal history of breast cancer. The
Chompret criterion is helpful in determining the likelihood of TP53 mutations. The
Claus tables are designed to determine the risk of breast cancer in healthy women based
on their family history of breast cancer. The Gail model is also for healthy women (over
age 35) to predict risk of breast cancer. The NCCN guidelines suggest TP53 testing for
any woman diagnosed with breast cancer under age 35 if BRCA1/2 is negative.
G34.
The correct answer is C.
Source of topic: slides
Keywords: trisomy 21, leukemia
Explanation: Somatic GATA1 mutations are seem in almost all cases of TMP which is a
frequent complication of children with Down syndrome. AKT undergoes somatic
mutation in Proteus syndrome and is not a transcription factor, ETV6 is a translocation
partner in AML, IKZF1 mutations and deletions are common in ALL and RUNX1
deletions and mutations cause familial platelet disorder with AML.
G35.
The correct answer is A.
Source of topic: Slides
Keywords: retinoblastoma, genetic testing
Explanation: The test results provided are consistent with sporadic retinoblastoma in the
child given that both hits (a nonsense mutation in the RB1 gene followed by loss of
heterozygosity) were seen in the tumor and no evidence of the nonsense mutation in the
blood sample. Thus, with this information there is much less than 1% risk to subsequent
siblings.
811
G36.
The correct answer is E.
Keywords: polyposis, hamartoma
Explanation: All of the genes listed can result in polyposis syndromes of different types
and are often on gene panels. However, juvenile polyposis is most likely to present first
in young children with rectal bleeding due to a large polyp and thus SMAD4 is the most
likely gene to have a pathogenic mutation in this scenario. The other gene underlying
juvenile polyposis is BMPR1A.
G37.
The correct answer is E.
Source of topic: Slides
Keywords: dyskeratosis congenita, telomeres
Explanation: The pedigree suggests an X-linked pattern of inheritance. Although bone
marrow failure and head & neck cancer is also common in Fanconi anemia, it is very
rarely X-linked and abnormal nails and leukoplakia are key features of dyskeratosis
congenita. The most common form of DKC is X-linked and telomere flow FISH
demonstrating very short telomere is the diagnostic test.
G38.
The correct answer is E.
Keywords: TP53 guidelines, risk prediction
Explanation: The NCCN guidelines suggest TP53 germline testing for any woman
diagnosed with breast cancer under age 35 if BRCA1/2 is negative. A is wrong because
the Amsterdam criteria are designed to determine the likelihood of Lynch syndrome. The
Chompret criterion is helpful in determining the likelihood of TP53 mutations. The
Claus tables are designed to determine the risk of breast cancer in healthy women
(without breast cancer) based on their family history of breast cancer. The Gail model is
also for healthy women (over age 35) to predict risk of breast cancer and can’t be used
once a woman has a diagnosis of breast cancer.
G39.
The correct answer is E.
Explanation: VHL is associated with pheochromocytomas of both the adrenal and extraadrenal locations. Individuals with VHL have a significantly increased risk of
endolymphatic sac tumors that are not seen in any of the other genes listed here. All of
the other genes are associated with pheochromcytoma or paraganglioma.
G40.
The correct answer is B.
Explanation: The clinical description is typical of hereditary leiomyomatosis with renal
cell cancer (HLRCC) which is caused by heterozygous mutations in the fumarate
hydratase (FH). Birt Hogg Dube is associated with oncocytic RCC and not with uterine
lesions; MET is associated with hereditary papillary renal cell cancer and not the other
lesions; PTEN can be associated with RCC and endometrial cancer but the skin lesions
are different; VHL is associated with RCC but no skin or uterine findings.
812
G41.
The correct answer is C.
Keywords: oncogenes, targeted therapeutics
Explanation: C is correct. Crizotinib is an inhibitor of the ALK kinase. FDA approval
was granted for treatment of non-small cell lung cancer patients if the tumor specimen is
demonstrated to contain the EML4-ALK inversion. FDA approval is specific to the gene
and the type of rearrangement tested.
G42.
The correct answer is B.
Keywords: Fanconi anemia, chromosome breakage syndromes
Explanation: The pedigree suggests an autosomal recessive pattern of inheritance. Most
forms of Fanconi anemia are autosomal recessive except one X-linked form (FANCB).
Bone marrow failure and head & neck cancer are common in both Fanconi anemia and
Dyskeratosis congenita. However, radial ray anomalies are a classic feature of Fanconi
anemia (as is short-stature). Thus, the diagnostic test is diepoxybutane testing of a
peripheral blood sample to look for increased breakage and chromosome abnormalities.
G43.
The correct answer is D.
Keywords: rhabdoid predisposition syndrome.
Explanation: Each of genes provided is associated with a specific class of childhood
cancers. Rhabdoid tumors of the solid organs and the CNS (referred to as atypical
rhabdoid/teratoid tumors) are associated with both somatic and germline mutations in
SMARCB1. For the other choices: CTNNB1, beta catenin, undergoes somatic mutation in
a number of tumors, particularly, hepatoblastoma; the majority of patients (>80%) with
pulmonary pleuroblastoma carry a germline mutation in DICER1, an enzyme involved in
microRNA processing; PTCH1 mutations result in Gorlin syndrome and risk of
medulloblastoma; SUFU mutations result in medulloblastoma.
G44.
The correct answer is D.
Keywords: Pattern of mutations in oncogenes
Explanation: The majority of mutations in proto-oncogenes are somatic mutations. The
cancer patient does not inherit a mutation and there is a single mutant allele. Thus,
analysis of the germline sample is most likely to reveal normal sequence and there is a
single heterozygous missense mutation in IHD2 in the tumor sample.
G45.
CORRECT ANSWER: D
SOURCE: Lecture/Slides
KEYWORDS: Hypermutated tumors, hereditary colon cancer
EXPLANATION: All of the genes provided result in an increased risk of colon cancer.
Specific missense mutations in the POLE gene augment the error prone polymerase
encoded by POLE and result in hypermutated tumors, with the highest number of somatic
mutations currently identified.
A - Tumors from patients with FAP caused by mutations in APC gene have multiple
rearrangements but are not hypermutated
B – Tumors from patients with Lynch syndrome were previously thought to be those with
the highest rate of somatic mutation but POLE tumors are higher
C – Tumors from patients with MUTYH also demonstrate diminished base excision
repair but again are not hypermutated
813
E – Patients with Peutz-Jeghers syndrome from STK11 mutations do have an increased
risk of colon cancer but it is not thought to be for hypermutated tumors.
G46.
CORRECT ANSWER: B
SOURCE: Lecture/Slides
KEYWORDS: SDH, hereditary paraganglioma, imprinting
EXPLANATION: One of the most significant clinical concerns for carriers of SDHB
pathogenic variants is the increased risk of malignant tumors which are otherwise rare in
the other syndromes and may result in metastatic spread.
A.– VHL is associated with PHEO and renal cell carcinoma,
C – MEN2 is associated with PHEO and medullary thyroid cancer
D – SDHD is paternally imprinted (not SDHB)
E – MEN1 and MEN2 are associated with an increased risk of parathyroid tumors.
G47.
CORRECT ANSWER: B
SOURCE OF ITEM TOPIC: Lecture/Slides
KEYWORDS: pharmacogenetics, BRCA
EXPLANATION: Tumors that are homologous recombination deficient results in
sensitivity to PARP inhibitors such as Olaparib (FDA approved) which destroys the backup DNA repair pathway.
A – it is Lynch syndrome genes (not APC) that results in increased sensitivity to immune
checkpoint inhibitors
C – it is NF2 (not NF10 and angiogenesis inhibitos like Bevacizumab
D – it is ERBB2 amplified breast cancers which are sensitive to HER2 inhibitors such as
trastuzumab
E – it is TSC1 or TSC2 mutations that lead to sensitivity to mTOR inhibitors.
814
H. Cytogenetics Questions H1-H43
H1.
Which of the following syndromes results from a chromosome deletion in a major proportion of
cases?
A. CHARGE syndrome
B. Fragile X syndrome
C. Mowat-Wilson syndrome
D. Prader-Willi syndrome
E. Charcot-Marie-Tooth syndrome
H2.
Microdeletions and duplications mediated by segmental duplications are recurrent due to their
underlying mechanism. Which of the following disorders is the most common recurrent
microdeletion syndrome?
A.
B.
C.
D.
E.
H3.
Aniridia-Wilms tumor
Retinoblastoma
Prader-Willi syndrome
VCF/DiGeorge syndrome
Cri-du-chat syndrome
A 2-year-old boy presents with a history of retinoblastoma, dysmorphic features and
developmental delay. Which of the following molecular abnormalities is the most likely cause of
his clinical findings?
A. Nonsense mutation in the RB1 gene.
B. Missense mutation in the RB1 gene.
C. RB2 gene mutation
D. Contiguous gene deletion of 13q14
E. Contiguous gene deletion of Xq27.3.
H4.
A young woman with severe mental retardation, short stature, and other malformations has a
karyotype of 45,X/46,X,r(X). Which of the following explanations is most likely to account for
the constellation of clinical features noted above?
A.
B.
C.
D.
E.
Both X chromosomes are likely to be active in the cells with 46 chromosomes.
The normal X is likely to be inactive in most or all of the cells with 46 chromosomes.
The ring X is likely to be acentric explaining its absence in the 45,X cell line.
The ring X is likely to be inactive in most or all of the cells with 46 chromosomes.
The XIST gene will almost certainly be present on the ring X chromosome.
815
H5.
A middle-aged couple visits prenatal clinic for preconception counseling. Based on the wife’s
age their risk of having a child with Down syndrome is increased. The genetic counselor explains
that even if their pregnancy were to result from a trisomy 21 conception there is a reduced
likelihood that the pregnancy would lead to a liveborn infant. Which of the following
probabilities best represents the likelihood that a trisomy 21 conception will result in a liveborn
infant?
A.
B.
C.
D.
E.
H6.
1%
20%
70%
90%
100%
Which of the following chromosome abnormalities is mostly likely constitutional?
A. t(8;21)(q22;q22)
B. t(11;22)(q23;q11.2)
C. del(5)(q22q33)
D. inv(16)(p13q22)
E. t(15;17)(q22;q21)
H7.
Triploidy is the result of three copies of each human chromosome and is one of the most common
causes of spontaneous miscarriage. Which of the following occurrences is the most common
mechanism by which triploidy arises?
A.
B.
C.
D.
E.
H8.
chimerism
duplication of an entire maternal haploid chromosome set
one egg fertilized by two sperm
tetraploid conception with loss of one copy of each chromosome
two eggs fertilized by one sperm
A newborn presents with microcephaly, hypotonia, and a high-pitched cat-like cry? Which of the
following cytogenetic abnormalities is most likely to be found on chromosomal analysis?
A. 4p deletion
B. 5p deletion
C. 5q deletion
D. 8p deletion
E. 11p deletion
H9.
A child is born with a partial deletion and partial duplication involving the same chromosome.
Which of the following parental chromosomal abnormalities is most likely to lead to this
occurrence?
A.
B.
C.
D.
E.
balanced reciprocal translocation
balanced Robertsonian translocation
isochromosome
pericentric inversion
paracentric inversion
816
H10.
Chorionic villus sampling is performed on a 38-year-old woman who is concerned about having a
child with a chromosomal disorder. Which one of the following karyotypes carries the highest
risk for this woman to deliver a chromosomally abnormal livebom?
A.
B.
C.
D.
E.
45,XX,der(13;14)(q10;q10)
45,XY,der (14;21)(q10;q10)
46,XY,inv(2)(p11.2q13)
46,XX,inv(2)(q31q35)
46,XX,t(11;22)(q23;q11.2)
H11. Which of the following prenatal testing methods can detect confined placental mosaicism?
A.
B.
C.
D.
E.
H12.
amniocentesis
chorionic villus sampling
fetal cells in maternal blood
maternal blood
percutaneous umbilical blood sampling
A young woman is found to have a balanced translocation after a family history and pedigree
analysis, during a preconception counseling visit, raised a red flag for a chromosomal
abnormality. Based on this clinical finding, which of the following statements about
chromosomal segregation and possible offspring is the most likely clinical outcome?
A. Offspring resulting from alternate segregation will be chromosomally normal.
B. Offspring resulting from alternate segregation are likely to be phenotypically normal.
C. Offspring resulting from adjacent-1 segregation will be chromosomally normal.
D. Offspring resulting from adjacent-1 segregation are likely to be phenotypically normal.
E. Offspring resulting from adjacent-2 segregation are likely to be phenotypically normal.
H13.
During meiosis, homologs must pair in order for recombination to occur. Which of the following
events allows optimal pairing and normal segregation:
A.
B.
C.
D.
E.
H14.
One or more chiasma per chromosome
Two or more chiasma per chromosome arm
Distal position of chiasma on chromosome arm
Medial position of chiasma on chromosome arm
Centromere pairing of chromosomes
Which of the following microarray confirmation methods can allow the cytogeneticist to
determine the mechanism of an imbalance detected on comparative genome hybridization
studies?
A. Fluorescence In Situ Hybridization (FISH)
B. Junction fragment PCR
C. Multiplex ligation-dependent probe amplification (MLPA)
D. Quantitative PCR
E. Targeted microarray for the abnormal region
817
H15
If you were a T-lymphocyte destined for cytogenetic analysis, some of the treatments you would
experience along the way (in correct order) are:
A.
B.
C.
D.
E.
H16.
Hypotonic solution, acetic acid, Giemsa, trypsin
Hypotonic solution, lysis buffer, trypsin, Giemsa
Phytohemagglutinin, hypotonic solution, fixative, Colcemid, Giemsa
Phytohemagglutinin, Colcemid, hypotonic solution, fixative, trypsin
Trypsin, acetic acid, hypotonic solution, Giemsa
The historical milestones in cytogenetics mark exciting intervals in the development of the field
of clinical genetics. Which of the following series of events follows the chronological order in
which these milestones were attained?
A. Hypotonic treatment of cells > Correct chromosome number is 46 > Y chromosome identified
in humans > Chromosome banding techniques>Fragile sites and fragile X syndrome
B. Correct chromosome number is 46 >Hypotonic treatment of cells > Y chromosome identified
in humans > Chromosome banding techniques> Fragile sites and fragile X syndrome
C. Y chromosome identified in humans >Hypotonic treatment of cells > Correct chromosome
number is 46 > Chromosome banding techniques> Fragile sites and fragile X syndrome
D. Hypotonic treatment of cells > Y chromosome identified in humans> Correct chromosome
number is 46 > Chromosome banding techniques> Fragile sites and fragile X syndrome
E. Y chromosome identified in humans> Hypotonic treatment of cells > Correct chromosome
number is 46 > Fragile sites and fragile X syndrome >Chromosome banding techniques>
H17.
A newborn presents with a cardiac defect, cleft palate and a hypoplastic thymus. Which of the
following chromosome abnormalities is the most likely cytogenetic imbalance in this patient?
A. A deletion of 1q.21.1
B. A deletion of 2q11.1
C. A deletion of 16p11.2
D. A deletion of 22q11.2
E. A deletion of 22q13.3
H18.
The mammalian somatic cell cycle is approximately 24 hours in length. Which of the following
phases of the cell cycle requires the most time?
A. G1
B. S
C. G2
D. Interphase
E. Mitosis
818
H19.
Meiotic recombination is necessary for genetic diversity and rates of genetic recombination are
different in males than females and for different chromosomal regions. The highest rate of
recombination is observed in which of the following situations?
A. in females compared to males near centromeres
B. in males compared to females near centromeres
C. in males compared to females in the middle of each chromosome arm
D. near centromeres compared to near telomeres
E. near telomeres compared to near centromeres
H20.
An MRI shows lissencephaly in your patient, however the patient does not have any
dysmorphic facial features. Which of the following is the most likely mutation?
A.
B.
C.
D.
E.
H21.
G-banding has been the preferred method for cytogenetic analysis since the late 1960s. The
underlying DNA content corresponds to the staining pattern observed by G-banding. Which of
the following descriptions details the difference(s) between G-negative bands and G-positive
bands?
A.
B.
C.
D.
E.
H22.
Point mutation within the LIS1 gene coding region.
Terminal deletion of 17p which includes the LIS1 gene
Terminal deletion of 17q which includes the LIS1 gene
Terminal duplication of 17p which includes the LIS1 gene
Triplet repeat expansion involving the LIS1 gene
G-negative bands are more gene rich than G-positive bands
G-positive bands are GC rich regions and are early replicating
G-negative bands are AT rich and are late replicating
G-negative bands are GC rich and are late replicating
G-positive bands are more gene rich than G-negative bands
Comparative genomic hybridization (CGH) is one method that can be used for copy number
variation (CNV) detection. Which of the following issues is a limitation of this methodology
compared to a G-banded karyotype?
A.
B.
C.
D.
E.
Cannot detect polyploidies
Cannot detect single copy differences
Cannot detect unbalanced translocations
Lower resolution than G-banded karyotype
Requires cells to be cultured
819
CELL BIOLOGY:
H23.
After a normal meiosis I cell division, the resulting cells contain which of the following
number of chromosomes and chromatids?
A.
B.
C.
D.
E.
H24.
Contains 23 chromosomes and 23 chromatids
Contains 23 chromosomes and 46 chromatids
Contains 46 chromosomes and 23 chromatids
Contains 46 chromosomes and 46 chromatids
Contains 46 chromosomes and 92 chromatids
Aneuploidies typically result from meiotic errors but the specific missing or additional
chromosome is not necessarily derived from the maternal or paternal gametes at an equal
rate. Which of the following aneuploidies virtually always arises as a result of maternal
meiotic error?
A.
B.
C.
D.
E.
45,X
47,XY,+16
47,XX,+21
47,XXX
47,XXY
H25. A child is born with a partial deletion and partial duplication for the same chromosome.
Which of the following chromosomal abnormalities represents the likely parental
chromosome abnormality predisposing to this occurrence?
A.
B.
C.
D.
E.
H26.
Balanced insertion
Balanced reciprocal translocation
Balanced Robertsonian translocation
Paracentric inversion
Pericentric inversion
Chromosomal rearrangements can result from a variety of mechanisms during meiosis or
mitosis. Which of the chromosome rearrangements listed below always involves at least
two chromosomes?
A.
B.
C.
D.
E.
Balanced Translocation
Duplication
Interstitial deletion
Isochromosome
Pericentric inversion
820
H27. Carriers of balanced structural chromosomal rearrangements are at risk of having offspring
with an unbalanced chromosome complement. Which of the following statements is true regarding
the risk of unbalanced offspring?
A. Carriers of small pericentric inversions have higher risk than large pericentric
inversions
B. Carriers of paracentric inversions have higher risk than pericentric inversions
C. Carriers of balanced autosomal translocations have a 75% risk of having unbalanced
offspring
D. Female carriers of the 21;21 Robertsonian translocation have a 50% risk of having
trisomy 21 offspring
E. Female carriers of the 14;21 Robertsonian translocation are at higher risk than male
carriers
H28.
The figure below depicts the various phases of the mammalian somatic cell cycle. The
numbers in parenthesis represent each of the phases. What is the correct order of phases?
A.
B.
C.
D.
E.
1=G1, 2=G2, 3=S,
1=G1, 2=S, 3=G2,
1=G1, 2=S, 3=G2,
1=G1, 2=S, 3=G2,
1=S, 2=G1, 3=G2,
4=prophase,
4=prophase,
4=prophase,
4=prophase,
4=prophase,
5=metaphase, 6=anaphase,
5=anaphase, 6=metaphase,
5=metaphase, 6=anaphase,
5=metaphase, 6=telophase,
5=metaphase, 6=anaphase,
821
7=telophase
7=telophase
7=telophase
7=anaphase
7=telophase
H29.
41. A phenotypically normal person carries a reciprocal balanced translocation between
two autosomes. When segregation occurs in this individual during meiosis I, which of the
following statements regarding the eventual offspring appropriately associates the
genotype with the expected phenotype?
A. Alternate segregation always leads to a normal karyotype with a normal phenotype
B. Alternate segregation can lead to an abnormal karyotype with a likely normal
phenotype
C. Adjacent-1 segregation leads to a normal karyotype with an abnormal phenotype
D. Adjacent-1 segregation leads to an abnormal karyotype with a likely normal phenotype
E. Adjacent-2 segregation leads to a normal karyotype with a likely normal phenotype
H30.
Non-allelic homologous recombination (NAHR) mediated by segmental duplications is a
mechanism known to cause recurrent deletions and duplications across the genome.
Deletions of 22q11.2, which have been associated with DiGeorge syndrome and
Velocardiofacial syndrome, are the most common recurrent imbalance. Which of the
following syndromes is also caused by a NAHR-mediated mechanism?
A. 1p36 deletion syndrome
B. Cri du Chat syndrome
C. Miller-Dieker syndrome
D. Potocki-Lupski syndrome
E. Wolf-Hirschhorn syndrome
H31.
During meiosis, homologs must pair in order for recombination to occur. Which of the
following features of homologous chromosome pairing allows optimal pairing and
normal chromosome segregation?
A.
B.
C.
D.
E.
H32.
Centromere pairing
Distal position of chiasma on chromosome arm
Medial position of chiasma on chromosome arm
One or more chiasma per chromosome
Two or more chiasma per chromosome arm
In normal female somatic cells, one chromosome is active and the second remains
condensed and appears in interphase as a Barr body due to X-inactivation. The number of
Barr bodies in an individual depends on the chromosomal complement of the sex
chromosomes. Which of the following karyotypes would result in detection of three Barr
bodies?
822
A. 46,XXY
B. 47,XXX
C. 48,XXXY
D. 48,XXXX
E. 49,XXXYY
H33.
Two unrelated patients are evaluated in genetics clinic. One of the patients has sickle cell
anemia phenotype and the other has a thalassemia phenotype. Molecular analysis
detected homozygous mutations in beta globin for each patient. In the first patient, the
homozygous mutation was consistent with sickle cell anemia (coding for the Glu6Val
substitution), while the second patient was homozygous for a stop codon in the betaglobin gene. Which of the following concepts best describes these findings?
A. Allelic heterogeneity
B. Incomplete penetrance
C. Locus heterogeneity
D. Pleiotropy.
E. Variable expressivity
CLINICAL CYTOGENETICS:
H34.
Imprinting plays a role in determining the phenotype of many different conditions. In
which of the following groups of disorders can all of the diagnoses be associated with
imprinting as a major determinant of phenotype?
A.
B.
C.
D.
E.
Angelman syndrome, Prader-Willi syndrome, DiGeorge syndrome
Ovarian teratoma, Trisomy 13 mosaicism, Beckwith-Wiedemann syndrome
Triploidy, Prader-Willi syndrome, ovarian teratoma
Triploidy, Williams syndrome, Beckwith-Wiedemann syndrome
Prader-Willi syndrome, Trisomy 8 mosaicism, Russell-Silver syndrome
H35. Which of the following recurrent translocations is diagnostic of chronic myeloid leukemia
(CML)?
A.
B.
C.
D.
E.
t(8;21)(q22;q22)
t(15;17)(q22;q21)
t(12;21)(p13;q22)
t(4;11)(q21;q23)
t(9;22)(q34;q11.2)
823
H36.
13. A patient carries an interstitial deletion in the proximal region of the long (q) arm of
the paternal homologue of chromosome 15 [del(15)(q11.2q13.1)]. This patient is
expected to present with which of the following disorders?
A.
B.
C.
D.
E.
H37.
A 10-year-old boy presents with distal muscle weakness and atrophy associated with mild
glove and stocking sensory loss, depressed reflexes, and pes cavus. Array CGH analysis
demonstrates an abnormality in chromosome 17 at band p12 shown in the plot below.
What is the diagnosis of this patient?
A.
B.
C.
D.
E.
H38.
Angelman syndrome
DiGeorge syndrome
Miller-Dieker syndrome
Prader-Willi syndrome
Williams syndrome
Charcot-Marie-Tooth syndrome type 1A (CMT1A)
Hereditary neuropathy with liability to pressure palsies (HNPP)
Miller-Dieker syndrome
Neurofibromatosis type 1 (NF1)
Williams syndrome
Recurrent microdeletions and their reciprocal microduplications are the result of which
molecular mechanism?
A.
B.
C.
D.
E.
Allelic homologous recombination (AHR)
Fork stalling and template switching (FoSTeS)
Non-allelic homologous recombination (NAHR)
Non-homologous end joining (NHEJ)
Replication slippage
824
H39.
Genome-wide chromosomal microarray analysis (CMA) detects losses and gains across
the genome. Various methods have been developed for CMA, including the use of arrays
that contain single nucleotide polymorphism (SNP) probes. Which of the following test
characteristics represents an advantage for detecting genetic abnormalities by using an
array that contains SNP probes, compared to an array that contains only copy number
detection probes?
A. Balanced rearrangements can be detected
B. Exon-level imbalances can be detected
C. Imbalances of the sex chromosomes can be detected
D. Smaller imbalances can be detected
E. Uniparental disomy can be detected
H40.
Your laboratory offers chromosomal microarray testing and you were asked to test an
individual who has a known cytogenetic abnormality, which was detected by G-banding,
to further define the abnormality. Which of the following karyotypes represents an
individual who would not benefit from this additional characterization by microarray
analysis?
A. 47,XY,+mar
B. 47,XY,+21
C. 46,XX,t(3;7)(p24;q22)dn
D. 46,XY,add(11)(q25)
E. 46,XX,der(6)t(1;3)(q32;q27)
H41. Chromosomal microarray analysis (CMA) has replaced conventional G-banding as the firsttier test for cytogenetic analysis. Comparative genomic hybridization (CGH) is one type of
CMA that can be used to detect copy number variants, however there are still limitations
to this method. Which of the following test characteristics is a limitation of this
methodology compared to a G-banded karyotype?
A. All polyploidies cannot be detected
B. Cells need to be cultured for DNA extraction
C. Resolution is lower than a G-banded karyotype
D. Single copy number differences (i.e., loss or gain) cannot be detected
E. Unbalanced translocations cannot be detected
825
H42.
Your laboratory offers chromosomal microarray testing and you were asked to test an
individual who has a known cytogenetic abnormality, which was detected by G-banding,
to further define the abnormality. Which of the following karyotypes represents an
individual who would not benefit from this additional characterization by microarray
analysis?
A. 47,XY,+mar
B. 47,XY,+13
C. 46,XX,t(3;7)(p24;q22)dn
D. 46,XY,add(2)(q37)
E. 46,XX,der(6)t(1;3)(q32;q27)
H43.
Non-allelic homologous recombination (NAHR) mediated by segmental duplications is a
mechanism known to cause recurrent deletions and duplications across the genome.
Deletions of 22q11.2, which have been associated with DiGeorge syndrome and
Velocardiofacial syndrome, are the most common recurrent imbalance. Which of the
following syndromes is also caused by a NAHR-mediated mechanism?
A. 1p36 deletion syndrome
B. Miller-Dieker syndrome
C. Pallister-Killian syndrome
D. Sotos syndrome
E. Wolf-Hirschhorn syndrome
826
Answers to Cytogenetics Questions H1-H43
H1.
Prader-Willi is associated with deletions in 15q. The other disorders are commonly the result of
other mechanisms.
The correct answer is D.
H2.
22q11 deletion syndrome--including DiGeorge syndrome and VCF (velocardiofacial syndrome)
is the most common deletion syndrome in humans, occurring in ~1/4000 births. This is a
common, recurrent microdeletion mediated by NAHR of flanking low-copy repeats (LCRs).
Other LCR genomic disorders (PWS, AS, Williams, Smith-Magenis, Sotos) occur at ~1/10,0001/20,000. Random deletion syndromes such as cri du chat, Wolf-Hirschhorn, Miller-Dieker
syndrome, retinoblastoma, aniridia-Wilms tumor are less common (1/50,000-1/100,000 or less).
The correct answer is D.
H3.
The patient has additional clinical features (dysmorphic features and developmental delay) to
retinoblastoma, suggesting a larger deletion that includes more genes than just the RB1 gene
The correct answer is D.
H4.
The abnormal phenotypes seen in association with mosaic karyotypes of the type indicated are
associated with failure of X inactivation. This is usually also associated with lack of expression
of the XIST gene. This means that both the normal X and the ring X are likely to be active. The
XIST gene is considerably more likely to be deleted than to be present on the ring X, although
deletion of the X inactivation center may be more pertinent than deletion of the XIST gene. As
mentioned above, both X chromosomes are likely to be active. The ring is not likely to be
acentric.
The correct answer is A.
H5.
The probability that a trisomy 21 conception will result in live birth is approximately 20%.
The correct answer is B.
H6.
the 11;22 translocation is the most common non-Robertsonian translocation observed in
constitutional cytogenetic studies
The correct answer is B.
H7.
Polyspermy is the most common cause of triploidy. Chimerism may caused by the fusion of two
embryos and differs from mosaicism. The other options are not likely to be observed.
The correct answer is C.
H8.
The features described fit Cri du Chat syndrome which typically results from terminal deletion 5p
from band 5p15.2 to 5pter. 4p- syndrome results in Wolf-Hirschorn syndrome. 5q- and 8psyndromes do not have alternative names. 11p- deletion is associated with WAGR syndrome.
The correct answer is B.
H9.
The occurrence of a deletion and duplication involving the terminal portions of the same
chromosome is typically the result if one parent carries a pericentric inversion. This occurs if
there is a cross-over within the inversion loop at meiosis.
The correct answer is D.
827
H10.
Both maternal and paternal carriers of rob(13q14q) have about a 1% chance of having a child
with translocation trisomy 13. Fathers who carry rob(14q21q) have less than a 1% chance of
having a child with translocation Down syndrome. Both types of inversions, regardless of the
parental origin, have almost no chance of abnormal offspring. The pericentric inversion (C) is a
common inversion found in the population and the paracentric inversion (D) would result in
nonviable dicentric and acentric products. Mothers who carry the t(11;22) are at a 6% risk for an
abnormal offspring typically have 47 chromosomes with a small extra der(22) and have mental
retardation and other anomalies.
The correct answer is E.
H11.
CVS samples chorionic villi from the placenta and can detect the mosaicism. The other tissues
sampled do not include placental cells and will not allow for detection of mosaicism confined to
the placenta.
The correct answer is B.
H12.
Alternate segregation results in normal gametes and in gametes with balanced chromosomal
translocation. Thus the offspring are phenotypically normal but not chromosomally normal in all
cases. Offspring from adjacent-1 or adjacent-2 segregation will all be unbalanced and all will be
chromosomally abnormal. They would also be phenotypically abnormal, except in very unusual
circumstances. Adjacent segregation-1 is when homologous centromeres segregate, in contrast to
adjacent-2 where the homologous centrmorees go to the same cell. Adjacent-2 is rare compared
to adjacent-1.
The correct answer is B.
H13.
Optimal pairing includes one chiasma per chromosome arm, or two per chromosome.
The correct answer is A.
H14.
Only FISH can show the localization of the signal on chromosomes and allow for a
determination of the mechanism for the imbalance.
The correct answer is A
H15.
T-lymphocytes are stimulated to a blast phase with the red kidney bean extract
phytohemagglutinin (PHA). After 48-72 hours, a mitotic poison such as Colcemid or colchicine is
used to block spindle formation and attachment to “arrest” cells in mitosis, a hypotonic treatment
is used to swell the cells, they are fixed in 3:1 methanol:acetic acid, put onto slides, and then most
often treated with trypsin prior to Giemsa staining to produce G-banding patterns.
The correct answer is D.
H16.
The discovery of the Y Chromosome and the XX/XY mechanism of sex determination in humans
was by T. S. Painter in 1923, hypotonic treatment of cells by T. C. Hsu in 1953, the correct
chromosome number by Tjio and Levan in 1956, chromosome banding techniques in 1969/70,
and fragile sites and fragile X in the late 70s.
The correct answer is C.
H17.
Deletion 22q11.2 causes VCF/DiGeorge syndrome. The three phenotypic characteristics listed are
most consistent with a clinical diagnosis of 22q11.2 deletion.
The correct answer is D.
H18.
The correct answer is D. All of the phases of mitosis take approximately one hour.
828
H19.
Overall, recombination rates in human females are significantly higher (~50%) than males, except
near telomeres, where male recombination rates are higher than female. Human recombination is
about twice as high as that in mouse, perhaps due to our biarmed chromosomes compared to all
telocentric chromosomes in mice (remember the obligatory 1 crossover per chromosome arm for
stable pairing and segregation). Recombination is suppressed near centromeres, and increases
greatly near telomeres in both males and females (but more so in males).
The correct answer is E..
H20.
Point mutations within the LIS1 gene cause Isolated Lissencephaly syndrome. These individuals
can only exhibit lissencephaly without the more dysmorphic features that are associated with
Miller-Dieker syndrome, which is caused by larger deletions of 17p.
The correct answer is A
H21.
R-bands are the reverse of G-bands, so there are the same number of each. G-positive bands are
AT-rich and relatively later replicating.
The correct answer is A
H22.
The correct answer is A.
H23.
The correct answer is D.
Keywords: meiosis
Explanation: Meiosis is a reduction division. In meiosis I, homologous chromosomes
replicate so each chromosome consists of two sister chromatids glued together by cohesins.
In the final phases of meiosis I, homologous chromosomes segregate into separate daughter
cells with each of the four resulting cells containing 23 chromosomes each of which
consists of two sister chromatids.
H24.
The correct answer is B.
Keywords: aneuploidy, nondisjunction
Explanation: Trisomy 16 is a prenatal lethal aneuploidy due to a maternal meiosis I
nondisjunction in 100% of cases. Turner syndrome (45,X) is 70% paternal and 30%
maternal in origin. Trisomy 21 is 92% maternal and 8% paternal in origin. 47,XXX is 90%
maternal and 10% paternal in origin. Klinefelter syndrome (47,XXY) is 54% maternal and
46% paternal in origin.
H25.
The correct answer is E.
Keywords: structural chromosomal abnormalities
Explanation: A pericentric inversion is the only structural chromosome abnormality that
can predispose to a terminal duplication on one chromosome arm and a terminal deletion
on the other chromosome arm in the offspring.
H26.
The correct answer is A.
Keywords: structural chromosomal abnormalities
Explanation: A balanced translocation in the only structural chromosomal abnormality in
the list provided to involve at least two chromosomes. All other listed abnormalities
involve only one chromosome.
829
H27.
The correct answer is E.
Keywords: risk in carriers of balanced chromosomal abnormalities
Explanation: The risk of having a trisomy 21 offspring in female carriers of the 14;21
Robertsonian translocation is ~10-15%, whereas the risk in male carriers is ~0-2%. Carriers
of large pericentric inversions have higher risk than small pericentric inversions. Carriers
of pericentric inversions have higher risk than paracentric inversions. Carriers of balanced
autosomal translocations have a 10-15% risk of having unbalanced offspring. Carriers of
the 21;21 Robertsonian translocation have a 100% risk of having trisomy 21 offspring.
H28.
The correct answer is C.
Keywords: cell cycle, interphase, M-phase, mitosis
Explanation: The cell cycle is divided into interphase and M-phase. In interphase,
dividing cells pass through G1, S, and G2 phases before starting the M-phase. The Mphase consists of mitosis and cytokinesis. Mitosis (or nuclear division) is divided into
prophase, metaphase, anaphase, and telophase. Cytokinesis (or cytoplasmic division)
finally splits the cytoplasm into two.
H29.
The correct answer is B.
Keywords: segregation of a balanced translocation
Explanation: Alternate segregation can produce either normal or balanced translocation in
the offspring. If the carrier is phenotypically normal, then the offspring carrying the same
balanced translocation will most likely be phenotypically normal. Adjacent-1 segregation
can only produce the unbalanced versions of the translocation in the offspring with an
abnormal phenotype. Adjacent-2 segregation is very rare and will also produce
unbalanced products that are phenotypically abnormal.
H30.
The correct answer is D.
Potocki-Lupski syndrome is the only syndrome listed that is mediated by NAHR. All of the
distractors do not have an underlying mechanism that makes the CNVs recurrent and involving
the same unique region across individuals.
H31.
The correct answer is D.
For homologous chromosomes to pair properly, at least one or more chiasma per
chromosome is needed. The distractors offer other mechanisms (centromere), positions
(distal, medial) or number (two or more).
H32.
The correct answer is D.
The number of Barr bodies is one less than the number of X chromosomes. Therefore, the correct
answer is the karyotype with four X chromosomes. All of the other distractors would result in a
different number of Barr bodies (A – 1, B – 2, C – 2, E – 2).
H33.
The correct answer is A.
830
Allelic heterogeneity refers to the situation when different alleles of a single gene
produce different phenotypes, and is the correct answer to this question. Incomplete
penetrance refers to the situation when an individual has a genotype known to cause a
disease but does not display a phenotype. Locus heterogeneity refers to the production of
identical phenotypes by mutations at two or more different loci. Pleiotropy refers to
disorders in which a single gene or gene pair causes multiple phenotypes, especially
when the effects are not obviously related. Expressivity is the extent to which a genetic
defect is expressed, from mild to severe.
H34.
The correct answer is C.
Keywords: imprinting
Explanation: Imprinting plays a major role in the phenotype of Angelman syndrome,
Prader-Willi syndrome, Beckwith-Wiedmann syndrome, Russell-Silver syndrome, ovarian
teratoma, and triploidy. Imprinting does not play any role in the phenotype of Trisomy 13
and Trisomy 8 mosaicisms, DiGeorge syndrome, and Williams syndrome.
H35.
The correct answer is E.
Keywords: cancer cytogenetics, leukemia
Explanation: The t(9;22)(q34;q11.2) is diagnostic of CML and a subset of B-precursor
ALL. The t(8;21)(q22;q22) is diagnostic of AML-M2. The t(15;17)(q22;q21) is diagnostic
of acute promyelocytic leukemia (AML-M3). The t(12;21)(p13;q22) is diagnostic of a
subset of B-precursor ALL. The t(4;11)(q21;q23) is diagnostic of a subset of B-precursor
ALL observed mostly in infants.
H36.
The correct answer is D.
Keywords: microdeletion, genomic disorders
Explanation: 15q11.2q13.1 deletion on the paternal chromosome 15 causes Prader-Willi
syndrome, whereas the same deletion on the maternal chromosome 15 causes Angelman
syndrome. DiGeorge syndrome is due to a 22q11.2 deletion. Miller-Dieker syndrome is
due to a 17p13.3 deletion. Williams syndrome is due to a 7q11.23 deletion.
H37.
The correct answer is A.
Keywords: microdeletion, microduplication, genomic disorders
Explanation: This array CGH plot shows a 17p12 duplication that encompasses the
PMP22 gene. The phenotype of the patient and the 17p12 duplication is consistent with
the clinical diagnosis of Charcot-Marie-Tooth syndrome type 1A (CMT1A). Williams
syndrome is due to a 7q11.23 deletion. HNPP is due to a 17p12 deletion. Miller-Dieker
syndrome is due to a 17p13.3 deletion. NF1 is due to a 17q11.2 deletion.
H38.
The correct answer is C.
831
Keywords: molecular mechanisms, recurrent CNVs
Explanation: Recurrent microdeletions/microduplications are caused by NAHR, which is
mediated by the homologous flanking segmental duplications. FoSTeS, NHEJ, and
replication slippage, all result in non-recurrent copy number changes. AHR is a normal
recombination mechanism that does not result in copy number changes.
H39.
The correct answer is E.
The use of SNP probes allows uniparental disomy (UPD) to be detected. UPD cannot be
detected on arrays that only have DNA probes that do not contain polymorphic markers.
All of the other distractors listed can be detected on array platforms that contain or do not
contain SNP probes.
H40.
The correct answer is B.
Microarray analysis cannot detect the mechanism for a cytogenetic imbalance. Therefore,
since a diagnosis of trisomy 21 by G-banding is already established, microarray analysis
will not provide any additional information about this imbalance. Microarray analysis
could help to: identify the origin of a marker chromosome (A) or additional material on a
chromosome (D), look for an imbalance at the breakpoint of an apparently balanced de
novo translocation (C), or define the size and gene content of an unbalanced translocation
(E).
H41.
The correct answer is A.
CMA using CGH cannot detect all polyploidies since the data is normalized to a copy
number normal reference DNA sample. Therefore, tetraploidies, and many triploidies
cannot be detected using this method. Cells do not need to be cultured, DNA can be
extracted from blood samples. The resolution is higher than a G-banded karyotype and
single copy number difference can be detected. Unbalanced translocations can also be
detected by CMA/CGH.
H42.
The correct answer is B.
Microarray analysis cannot detect the mechanism for a cytogenetic imbalance. The
diagnosis of trisomy 13 by G-banding is established, so microarray analysis will not
provide any additional information about this imbalance. Microarray analysis could help
to: identify the origin of a marker chromosome (A) or additional material on a
chromosome (D), look for an imbalance at the breakpoint of an apparently balanced de
novo translocation (C), or define the size and gene content of an unbalanced translocation
(E).
H43.
The correct answer is D.
Sotos syndrome is the only syndrome listed that is mediated by NAHR. All of the distractors do
not have an underlying mechanism that makes the CNVs recurrent and involving the same unique
region across individuals.
832
I.
Prenatal/Reproductive Genetics Questions I1-I46
I1.
Chorionic villus sampling (CVS) occasionally produces cytogenetic results that are somewhat
ambiguous such as mosaicism. The finding of a mosaic trisomy 15 in a CVS leads to a secondary
amniocentesis, which demonstrated a normal diploid karyotype. In such a case which of the
following clinical options would be the best response?
A.
B.
C.
D.
E.
I2.
Which of the following chromosome studies would most likely be detected in a phenotypically
normal woman with infertility?
A.
B.
C.
D.
E.
I3.
46,XY
45,X
46,XX/45,X
47,XXY
47,XYY
Fragile X testing should be considered during infertility evaluations when you encounter which of
the following clinical findings?
A.
B.
C.
D.
E.
I4.
Consider uniparental disomy molecular cytogenetic studies for chromosome 15
Monitor the pregnancy with targeted ultrasound for cardiac malformations.
Perform cordocentesis (funicentesis) to confirm normal karyotype in fetal blood.
Reassure the parents that the baby will be normal during a discussion of the results.
Suggest consideration of termination of pregnancy for fear of hidden ‘tissue mosaicism”
azoospermia
oligospermia
partial androgen insensitivity
polycystic ovarian syndrome
premature ovarian failure
The risk of pregnancy loss is considered to be the highest with which of the following prenatal
procedures?
A.
B.
C.
D.
E.
amniocentesis - standard (> 15 weeks)
amniocentesis - early (< 12 weeks)
chorionic villus sampling
fetal magnetic resonance imaging study
first trimester nuchal lucency determination
833
I5.
The laboratory at your center is seeking your advice regarding introduction of array
Comparative Genome Hybridization (aCGH) as an option for prenatal diagnostic testing
from chorionic villus sampling and amniocentesis. You can counsel the laboratory
administration that, when compared to conventional karyotype analysis, aCGH is
associated with which of the following features?
A.
B.
C.
D.
E.
I6.
When noted as an isolated finding on second trimester ultrasound, findings such as choroid
plexus cyst, echogenic bowel, echogenic cardiac focus, or short femur are most likely occur in a
fetus with which of the following outcomes?
A.
B.
C.
D.
E.
I7.
cystic fibrosis
Down syndrome
trisomy 18
normal infant
tuberous sclerosis
Among causes of premature ovarian failure, which of the following would be most commonly
encountered by the infertility specialist?
A.
B.
C.
D.
E.
I8.
Increases detection for submicroscopic deletions
Produces more uninterruptible results from stillbirth specimens
Requires a greater volume of amniotic fluid
Requires greater technician skill
Requires longer cell culture time
46, XY female
45, X
fragile X premutation carrier
galactosemia
myotonic dystrophy
Mr and Mrs Smith each are heterozygotes for delta F508 mutation of the CFTR gene and
are undergoing Preimplantation Genetic Diagnosis(PGD) to avoid transfer of embryos
homozygous for delta F508. Mrs Smith who is 39, inquires about her increased of Down
syndrome and asks if screening also can be undertaken for aneuploidy. Which of the
following statements can you report to her about using fluorescence in situ hybridization
(FISH) technologies for aneuploidy detection for Preimplantation Genetic Screening
(PGS) ?
A.
B.
C.
D.
E.
decreases Down syndrome miscarriages
decreases Down syndrome livebirths
increases implantation rates
removes the need for CVS or amniocentesis
has not yet shown any proven benefit
834
I9.
Mrs. Smith has an 11-week ultrasound with a fetus with a nuchal translucency
measurement of 4.5 mm. Her diagnostic testing by chorionic villlus sampling (CVS)
reveals 46, XY. At second trimester ultrasound, the risk is greatest for identification of
anomalies in which of the following systems?
A.
B.
C.
D.
E.
I10.
Among male causes of infertility, which of the following would be most likely to be encountered
in the man with oligospermia?
A.
B.
C.
D.
E.
I11.
Balanced translocation
Fragile X mutation
Delta F508 mutation
Sex chromosome aneuploidy
Sertoli cell only syndrome
Among male causes of infertility, which of the following would be most likely to be encountered
in the man with non-obstructive azoospermia?
A.
B.
C.
D.
E.
I12.
Cardiac
Central nervous system
Gastrointestinal
Genitourinary
Skeletal
Balanced translocation
Fragile X premutation
Delta F508 mutation
Sertoli cell only syndrome
Sex chromosome aneuploidy
Mrs. Smith is scheduled for a chorionic villus sampling (CVS) at 11-weeks gestation as
she and her husband are carriers of cystic fibrosis. They are aware of their 25% risk and
are seeking early diagnosis. However, while she is aware of the risk of miscarriage
associated with CVS, she also inquires as to which of the following complications have
been associated with CVS?
A.
B.
C.
D.
E.
Abruption
Hemangioma
Placenta previa
Preterm delivery
Preterm premature rupture of membranes
835
I13.
Among male causes of infertility, which of the following would be most likely to be encountered
in the man with obstructive azoospermia?
A.
B.
C.
D.
E.
I14.
If a fetus has a nuchal translucency increased to 2 standard deviations at 11-14 weeks, which of
the karyotype results is the most likely to be found on amniocentesis?
A.
B.
C.
D.
E.
I15..
45, X
45, X/46,XX
46,XX
47,XX, +18
47, XX, +21
Mrs Jones is 35-years-old and has an amniocentesis for an increased risk of Down
syndrome based on her maternal serum screening. The ultrasound at the time of the
amniocentesis was unremarkable without structural abnormalities noted in the fetus, nor
ultrasound markers for Down syndrome. The amniotic fluid was normal and the placenta
was posterior without hematomas noted. Two days following the amniocentesis she is
diagnosed with premature rupture of membranes. Which of the following outcomes are
you going to counsel her is the most likely?
A.
B.
C.
D.
E.
I16.
Balanced translocation
DAZ deletion
Delta F508 mutation
Fragile X premutation
Sex chromosome aneuploidy
Chorioamnionitis
Chorionamnion separation
Fetal demise
Miscarriage
Reaccumulation of amniotic fluid
In order to reach 80% detection of trisomy 21 in women under the age of 35 with nuchal
translucency and first trimester serum markers, approximately how many women will have
positive screening results?
A.
B.
C.
D.
E.
<10 %
20-30%
30-40%
40-50%
> 55 %
836
I17.
John and Mary present for an infertility evaluation. John is diagnosed with oligospermia (a low
sperm count). Which of the following genetic conditions is associated with oligospermia?
A.
B.
C.
D.
E.
I18.
As part of an infertility evaluation, Mrs. Smith at age 35 years is diagnosed with
premature ovarian failure based on an elevated follicle stimulating hormone. She is
otherwise healthy with an unremarkable clinical history. Which of the following clinical
disorders is a likely diagnosis to consider in your differential diagnosis?
A.
B.
C.
D.
E.
I19.
Cystic Fibrosis
Huntington Disease
Kennedy Disease
Myotonic Dystrophy
Premutation Fragile X
Susan is being cared for by her gynecologist. She carries a diagnosis of late onset congenital
adrenal hyperplasia based on clinical exam and endocrinologic studies. Treatment with steroids
has regulated her menstrual cycle and she is now contemplating pregnancy. Obtaining her
specific molecular diagnosis is important in order to determine which of the following
management issues?
A.
B.
C.
D.
E.
I20.
Balanced translocation carrier
Cystic fibrosis
Fragile X Syndrome
47,XXY
47,XYY
Correctly adjust her steroid medication during pregnancy
Determine whether she is at risk for severe salt wasting during pregnancy
Establish whether she can become pregnant without assisted reproduction
Identify risks for adverse pregnancy outcomes
Identify the potential risk for a child with classic salt wasting CAH
Congenital bilateral absence of the vas deferens (CBAVD) is present in almost all males with
classic cystic fibrosis. Among men with CBAVD alone, what is the likelihood that they will
have at least one mutation of the CFTR region (including mutations in 5T)?
A.
B.
C.
D.
E.
25%
50%
70%
85%
100%
837
I21. Mr. Jones and his wife are a Northern European couple who have experienced two years of
infertility. Mr. Jones visits his urologist and is diagnosed with bilateral congenital
absence of the vas deferens (CBAVD). He is otherwise well with an unremarkable
clinical history. Analysis of his cystic fibrosis transmembrane receptor gene (CFTR) most
likely reveals which of the following molecular findings?
A.
B.
C.
D.
E.
I22.
Among children born following assisted reproduction, the risk of a major congenital
malformation is approximately how much greater than the general population risk?
A.
B.
C.
D.
E.
I23.
10%
30%
50%
60%
70%
Early amniocentesis (9.0-12.9 weeks) is associated with the highest risk of fetal loss when
compared to any other diagnostic modalities (CVS, standard amniocentesis and PUBS). The
increased fetal loss rate is most likely the consequence of which of the following events
associated with the procedure?
A.
B.
C.
D.
E.
I24.
Absence of mutations on a common 23 mutation panel
Absence of mutations on an expanded mutation panel
Double heterozygosity for classic CFTR mutations
Double heterozygosity for poly T variants
Heterozygosity for a classic CFTR mutation
Difficulty piercing the membranes leading to tears and fluid leaks
Increased susceptibility for infection at early gestational age
Poor ultrasound resolution at early gestation leading to fetal injury
Proportionately a greater volume of amniotic fluid removed
Incomplete conversion of corpus luteal to placental progesterone support
Mrs Smith is a 35 year old G1 at 11 weeks gestation with a fetal ultrasound which
demonstrates a nuchal lucency measurement of 3.0 mm. Which of the following
karyotypes are you most likely to find on analysis of this fetus’ chromosomes?
A.
B.
C.
D.
E.
45, X
46, XX
47,XX,+21
47,XX, +18
47, XX, +13
838
I25.
Assisted reproductive technologies are associated with an increased risk of congenital
malformations. Based on animal and human studies, which of the following categories of
genetic conditions has been reported most frequently?
A.
B.
C.
D.
E.
I26.
Mrs Jones is interested in an early pregnancy, noninvasive method to determine if her
current fetus has Down syndrome. Free fetal nucleic acids are most uniquely
differentiated from fetal cells in the maternal circulation by which of the following
features?
A.
B.
C.
D.
E.
I27.
Chromosomal breakage disorders
DNA expansion disorders
Imprinting disorders
Point mutations
Segmental duplication/deficiency disorders
ability to cross from the fetus to the mother
ability to identify fetal sex
clearance from the mother within hours of delivery
capacity for identifying fetal aneuploidy
identification in the maternal circulation in the first trimester
A 40-year-old woman carries a fetus with increased nuchal translucency of 3.0 mm at her
12-week ultrasound. This clinical finding poses the highest risk for which of the
following fetal abnormalities?
A.
B.
C.
D.
E.
Aneuploidy
Developmental delay
Fetal demise
Growth restriction
Neural tube defect
I28. You are a molecular/cytogenetic laboratory director and would like to convince your
hospital to provide chromosome microarray analysis. Potential benefits include an
increase in the numbers of clinically-significant diagnoses above that obtained from
routine karyotype in several settings in which patients are seen by the obstetrics
department. Which of the following indications for testing will have the lowest increase
in new diagnoses?
A.
B.
C.
D.
E.
Amniocentesis for anomalies
Amniocentesis for maternal age > 35
Chorionic villus sampling for increased nuchal translucency
Miscarriages
Stillbirths
839
I29.
A couple presents to your office and informs you the wife carries a balanced translocation
between chromosomes 5 and 12. Which of the following options would not be an
appropriate reproductive genetic technology to address their risks?
A.
B.
C.
D.
E.
I30.
A 36-year-old patient is now pregnant following 4 years of infertility and three cycles of
IVF. She is concerned about her risks of delivering a child with Down syndrome but is
anxious to avoid a miscarriage. Which of the following screening tests is associated with
the highest positive predictive value in women over 35 years?
A.
B.
C.
D.
E.
I31.
Amniocentesis
Chorionic villus sampling
Noninvasive free fetal DNA
Percutaneous umbilical blood sampling
Preimplantation genetic diagnosis
Combined first trimester screen
Free fetal dna screen
Integrated serum screen
Quad serum screen
Triple serum screen
A couple presents to your office with a pregnancy at 10 weeks gestation. Using assisted
reproductive technology (ART), the pregnancy was conceived by in vitro fertilization as
the husband has oligospermia. They are also each carriers of cystic fibrosis and
preimplantation genetic diagnosis (PGD) was performed for their specific CFTR
mutations. The couple asks “Is this pregnancy at greater risk for birth defects?” Which of
the following responses is most appropriate?
A. No, there is no evidence that either PGD or ART is associated with increased risks of
birth defects.
B. No, your PGD was performed to allow only healthy embryos to develop.
C. Yes, ART is associated with an approximate 30% increase in congenital
malformations
D. Yes, for imprinted gene abnormalities but these can be tested for at amniocentesis.
E. Yes, for imprinted gene abnormalities but these were tested for at PGD.
840
I32.
A 40-year-old woman presents for prenatal care with questions concerning her risk of
delivering a child with Down syndrome. She would like to proceed with the screening
test that has the lowest false positive rate. You should offer which of the following
prenatal screening tests?
A.
B.
C.
D.
E.
I33.
A 30-year-old patient returns from her 18-week ultrasound visit with a verbal report
stating she was told there were “birth defects” noted in her developing fetus. As you are
obtaining the report, your concern for the presence of possible chromosome abnormality
would be highest if the report indicates which of the following ultrasound findings?
A.
B.
C.
D.
E.
I 34.
Atrial septal defect
Gastroschisis
Hydronephrosis
Omphalocele
Upper limit normal ventriculomegaly (10 mm)
A 28-year-old woman with a BMI of 40 presents for her 18-week ultrasound. Her
chances of having an incomplete fetal survey are increased due to her higher BMI. Her
fetal survey should be repeated as she faces increased risks for congenital anomalies. She
is at the highest relative risk is for which of the following anomalies?
A.
B.
C.
D.
E.
I35.
Combined first trimester screen
Free fetal DNA screen
Intergrated serum screen
Quad serum screen
Triple serum screen
Cardiac anomaly
Cleft lip/palate
Gastroschisis
Limb anomalies
Neural tube defect
A couple transfers to your practice and reports a history of two miscarriages during their
ten years of marriage, as well as difficulty conceiving. The initial evaluation reveals
oligospermia. On further study of the husband, which of the following karyotypic
abnormalities are you most likely to find?
A.
B.
C.
D.
E.
A balanced translocation
45, X
45, X/46, XX
47, XXY
47, XYY
841
I36.
You offer your patient noninvasive prenatal testing (NIPT) as a screening test for
aneuploidy at 10 weeks into her pregnancy. Which of the following conditions would
have excluded her from this testing?
A.
B.
C.
D.
E.
I37.
Mrs Smith is a 40-year-old G2 P1 at 18-week gestation, with an ultrasound report
indicating a neural tube defect was found in the fetus. She had a 12-week cell free fetal
DNA (cffDNA) study which was "normal' for chromosomes 13, 18, 21 and X, Y. She
remains concerned about the fetus so she decides to proceed with an amniocentesis.
Which of the following findings is most likely from her amniocentesis?
A.
B.
C.
D.
E.
I38.
Demise of twin gestation at 8 weeks
Diabetes type 1
Diabetes type 2
Miscarriage within past year
Obesity (BMI > 30)
Autosomal aneuploidy (other than chromosome 13, 18, 21)
Autosomal aneuploidy (of chromosome 13, 18, 21)
Microdeletion syndrome
Normal karyotype
Sex chromosome aneuploidy
In your obstetric practice, you obtain a detailed second trimester fetal ultrasound survey
to assess for the risk of Down syndrome. Which of the following ultrasound findings
would have the highest relative risk for trisomy 21?
A. Choroid plexus cyst
B. Echogenic bowel
C. Echogenic cardiac focus
D. Renal pyelectesis (4.0 mm)
E. Shortened femur
I39.
Mr and Mrs Jones are both in their early thirties and seeking prenatal genetic counseling
at 18 weeks gestation. This is their first pregnancy, conceived by IVF following 4 years
of infertility, etiology undetermined. They are worried about the possibility of abnormal
findings on their upcoming ultrasound. If an ultrasound abnormality were identified,
which of the following findings is most likely to be seen on their pending ultrasound?
A.
B.
C.
D.
E.
Dysplastic kidney and absent stomach bubble
Omphalocele and nephromegaly suggesting Beckwith Weideman Syndrome
Shortened long bones consistent with a skeletal dysplasia
Tetralogy of Fallot
Ventriculomegaly and adducted thumbs suggesting X-linked aqueductal stenosis
842
I40.
Susan Smith is a G1 at 22 weeks gestation. Her medical history is significant for obesity
(BMI 40), Type 2 diabetes and anxiety. She drinks a glass of wine with most dinner
meals. She started metformin preconception and has continued on it through the
pregnancy. First trimester hemoglobin A1C was slightly elevated (7.0, normal < 6.1).
She took Ampicillin for 7 days at 10 weeks gestation for a urinary tract infection. She
presents to your office with an ultrasound finding of a fetus with a neural tube defect.
From her history, which of the following factors conveys the highest relative risk for a
fetus with a birth defect, especially a neural tube defect?
A.
B.
C.
D.
E.
I41.
Ms. Jones is 25-years old with her first pregnancy. She is 10 weeks pregnant and elects
cfDNA for non-invasive screening for aneuploidy which returns as negative for
chromosomes 13, 18, 21. At 18 weeks, her ultrasound for a complete fetal survey
indicates the fetus is growing appropriately, no structural abnormalities are identified but
an echogenic focus is noticed in the heart. Which of the following next steps is most
consistent with current guidelines?
A.
B.
C.
D.
E.
I42.
Alcohol consumption during pregnancy
Ampicilin use
Maternal obesity (BMI > 40)
Metformin use
Type 2 diabetes A1C = 7.0
Amniocentesis
Echocardiograph (ECHO)
No further action
Quad serum screen
Third trimester fetal growth ultrasound
17. Mrs. Brown is 40-years old with her first pregnancy. She received counseling on the
various options for screening as well as diagnosis in order to detect cytogenetic
abnormalities. After counseling and reading, she feels that only a diagnostic procedure
will provide her the information she is seeking. She elects to proceed with an
amniocentesis. This is performed at 17 weeks, 20 ml clear fluid is sent for chromosome
microarray analysis. She calls the next morning reporting that on awakening, she noted a
gush of fluid from her vagina. She returns to the office for an ultrasound which reveals a
17-week fetus, positive heart beat and mild oligohydramnios. Clinical exam is positive
for rupture of membranes. Which of the following outcomes is the most likely with
rupture of membranes after an amniocentesis?
A.
B.
C.
D.
E.
Contractures in the neonate
Intrauterine growth restriction
Late preterm delivery
Miscarriage
Pulmonary hypoplasia in the neonate
843
I43.
Mr. and Mrs. Smith present to your office with questions concerning possible exposures
that could result in a child with a birth defect. They are particularly concerned as they
want to conceive soon and their rental apartment was recently painted. Their awareness
of congenital abnormalities has heightened as their neighbor just delivered an infant with
a neural tube defect. Mrs Smith is 25 years old, has a BMI of 28, is healthy, had a RouxN-Y gastric bypass 4 years ago, takes no medication, and reports no remarkable family
history. Mr Smith is also 28 years old, has a BMI of 30, is healthy with no surgeries,
takes a daily SSRI for anxiety/depression and reports no significant family history. Both
are Northern European in ancestry. Both have jobs they describe as “stressful” and they
enjoy a bottle of wine many nights of the week to unwind. Which of the following
exposures to Mrs. Smith has the highest relative risk of contributing to the development
of a birth defect?
A.
B.
C.
D.
E.
I44
Mrs. Smith is a 40-year-old woman at 10 weeks gestation in her first pregnancy. Her
obstetrician discusses screening options and she elects NIPT screening for Down
syndrome. The result returns as uninterpretable, on repeat sampling at 12 weeks, the
NIPT is also uninterpretable. Which of the following circumstances is most compatible
with an uninterpretable NIPT result?
A.
B.
C.
D.
E.
I45.
Alcohol exposure
Folate deficiency
Obesity
Paint fume exposure
Selective serotonin receptor uptake
BMI > 40
Confined placental mosaicism
Maternal malignancy
Maternal mosacism
Vanished twin
Mrs Smith is 36-years old and pregnant with her first child. She has a first trimester
screening evaluation for Down syndrome (nuchal translucency measurement and serum
biomarkers) and the report indicates an increased risk for Down syndrome. Her neighbor,
also 36-years old and expecting her first child, has a noninvasive screening test (NIPT)
from a sample of her blood and the result is also reported as high risk. Which of the
following when comparing screening tests for Down syndrome, is most consistent with
NIPT compared to serum screening?
A.
B.
C.
D.
E.
Better approach for twins
Higher false positive rate
Higher sensitivity rate
Lower chance of non-informative results
Lower positive predictive value
844
I46.
Mrs Jones with her first pregnancy has her 18-week ultrasound planned. She mentions to
you that her sister had a child with congenital cytomegalovirus (CMV) who had deafness,
developmental and growth delay and passed away at age 3. Which of the following
ultrasound findings increases is the chance there is fetal infection with CMV?
A.
B.
C.
D.
E.
Gastroschesis
Large for gestational age fetus
Polydactyly – post axial
Polyhydramnios
Ventriculomegaly
845
Answers to Prenatal/Reproductive Genetics Questions I1-I46
I1.
The correct answer is A. – one etiology of mosaicism is considered to be an initially trisomic
conception which undergoes trisomic rescue, this would put the pregnancy at risk for retention of
two chromosomes from the same parent, uniparental disomy testing would be indicated.
Ultrasound surveillance is indicated for growth restriction but not for congenital anomalies such
as cardiac malforamation. Low level tissue mosaicism can never be disproven in the fetus.
I2.
The correct answer is C. – 46,XX/45,X can present as a phenotypically normal female with
infertility, Answer A could be a phenotypically normal female but genetically be a male with
complete androgen insensitivity and would have amenorrhea.but is less common. Answer A
would present with Turner syndrome phenotype. Answers D and E present as males.
I3.
The correct answer is E. – POF is the only impact on reproductive capacity recognized among
fragile X carriers
I4.
The correct answer is B. – at a loss rate of 2-3%, early amniocentesis out paces the risk of losses
from CVS (1.0%) and standard amniocentesis (0.5%), MRI and nuchal lucency measurement are
noninvasive and not associated with pregnancy loss
I5.
The correct answer is A. -- Implementation of aCGH in the prenatal setting may be advantageous
for several reasons including increased detection of genomic disorders when ultrasound
malformations are present and the conventional karyotype is normal and when poor cell growth is
anticipated (stillbirth or miscarriage). The ability of aCGH to utilize smaller volumes of fluid and
the applicability for automation may also prove valuable components. Current trials are
underway to address aCGH versus conventional karyotype in routine amniocentesis with the issue
of CNV to be addressed.
I6.
The correct answer is D. – each of these findings in isolation can occur in as many as 1-5% of
pregnancies; in the majority of instances the karyotype is normal. As aneuploidy is a relatively
rare event, although the relative risk of aneuploidy is indeed increased (1.2 to 5.0 fold), the
majority of pregnancies are karyotypically normal. Echogeneic bowel is associated with cystic
fibrosis with an increased relative risk but again the majority of infants with echogenic bowel are
normal. None of the findings are seen in fetuses with tuberous sclerosis.
I7
The correct answer is C. – Amenorrhea would be expected with A (Androgen insensitvity) and
B (Turner syndrome) Galactosemia is rare in itself , so an infertility specialist will only rarely
encounter a patient. Myotonic dystrophy may impact fertility but not specifically though PFO.
I8.
The correct answer is E. -- Aneuploidy is known to increase as women age and theoretically
screening the embryos of women of women of advanced maternal age prior to transfer should
decrease their rate of aneuploid conceptions. This should indirectly increase their implantation
rate, lower their miscarriage rate and increase their delivery rate. However, while initial studies
with FISH were promising, randomized controlled trials found no benefit to screening embryos
prior to transfer and even detriment to implantation rates. Advances in technology to allow
application of aCGH to PGS may overcome these difficulties.
846
I9.
The correct answer is A. -- With increasing nuchal lucency in the first trimester the risk of a
structural malformation being detected on the second trimester ultrasound increases. The
association between cardiac anomaly and nuchal edema may be physiologic although there does
not appear to be good correlation between type of cardiac defect and presence of increased
edema. Among fetuses with trisomy 21, increased nuchal edema in the first trimester is not
predictive of a second trimester cardiac anomaly despite the high prevalence of cardiac
malformations in Down syndrome.
I10.
The correct answer is A. – based on frequencies of infertility identified in each category, A
(balanced translocation) would be most commonly encountered with oligospermia. Sex
chromosome aneuploidy and Sertoli cell only syndrome are commonly associated with
azoospermia; Delta F508 mutuation with obstructive azoospermia and CBAVD.
I11.
The correct answer is E. – balanced translocation is associated with oligospermia; cystic fibrosis
mutation associated with obstructive azoospermia (CBAVD) and sex chromosome aneuploidy
associated with non-obstructive azoospermia. Sertolic cell only syndrome is a rare cause of
azoospermia. Fragile X is not associated with male infertility.
I12.
The correct answer is B. -- Adverse pregnancy outcomes with regard to aberrant placentation,
preterm labor or rupture of membranes have not been associated with CVS. However, concern
has been raised as to the possible effects on the fetus and in particular most recently an increase in
hemangiomas. This increased risk in hemangiomas appears to be concentrated in transcervical
CVS and is not gestational age dependent (in contrast to the limb reduction abnormalities which
were concentrated in the < 10 week CVS cases).
I13.
The correct answer is C. - balanced translocation is associated with oligospermia; cystic fibrosis
mutation associated with obstructive azoospermia (CBAVD) and sex chromosome aneuploidy
associated with non-obstructive azoospermia. DAZ deletion is a cause of azoospermia, fragile X
premutation is not a associated with infertility in the male,
I14.
The correct answer is C. – as a screening test, an enlarged NL at the second standard deviation
increases the relative risk for aneuplopidy but will be encountered most commonly in
chromosomally normal pregnancies.
I15.
The correct answer is E. -- Premature rupture of membranes following amniocentesis occurs in
approximately 1% of cases. Reaccumulation of the amniotic fluid occurs in > 90% of patients
though on average takes 3 weeks. The outcomes are generally favorable which is in contrast to
spontaneous rupture of membranes in the second trimester which is associated with fetal demise,
miscarriage and chorioamnionitis.
I16.
The correct answer is A. – highlights the need to have a fairly significant portion of the
population screen positive in order to detect 80% of trisomy 21 fetuses
I17.
The correct answer is A. 3-5% of men with oligospermia carry a balanced translocation. Cystic
fibrosis is associated with male infertility with obstructive azoospermia, Fragile X is associated
with decreased ovarian reserve in female premutation carriers and 47, XXY would classically
present with azoospermia. 47,XYY syndrome individuals are typically fertile.
847
I18.
The correct answer is E. -- Premutation fragile X carriers are noted in 2-5 % of women with
premature ovarian failure varying with the strength of their family history of POF. Myotonic
dystrophy is associated with male infertily, Kennedy Disease is a rare condition not associated
with POF and Cystic Fibrosis is assopciated with male infertility and CBAVD. Huntington
Disease is not associated with infertility.
I19.
The correct answer is E. -- Some women with late onset CAH will carry a classic salt wasting
CAH mutation as well as an atypical allele. If the partner is also a carrier for salt wasting CAH,
the risk of an affected child would be 25%. Her specific molecular diagnosis would not impact
her steroid medication during pregnancy, alter whether she can become spontaneously pregnant
or her pregnancy complications.
I20.
The correct answer is D. -- If 5T is included, approximately 32% of men with CBAVD are
heterozygotic for a mutation in CFTR and an additional 53% are double heterzygotes.
I21.
The correct answer is E. -- The majority of men with a clinical diagnosis of CBAVD who are
otherwise asymptomatic have either a double heterozygosity or heterozygosity for CF mutations.
In individuals who are asymptomatic from a respiratory standpoint, if they are double
heterozygotes then typically non-classic mutations are involved. Double heterozygosity for
classic mutations would be associated with an earlier onset of typical cystic fibrosis symptoms.
I22.
The correct answer is B. -- Recent meta-analysis suggests that once large, studies could be
assessed; an increased congenital malformation rate of approximately 30% over background is
noted following assisted reproduction techonologies.
I23.
The correct answer is D. For an amniocentesis at 10 weeks, the removal of the needed 20 cc
amniotic fluid represents almost half of the total volume. Removal of 20 ccs at 16 weeks
represents only about 12% of the total volume.
I24.
The correct answer is B. -- While enlargement of the nuchal space in the first trimester fetus is a
sensitive marker for aneuploidy, in most fetuses it represent a variation of normal. Thus the need
to add serum markers to first trimester nuchal translucency measurements in order for these
findings to be used as a screening tool without a high screen positive rate
I25.
The correct answer is C. -- Evidence for increased congenital malformations is based on several
population based studies with the highest rates seen for infants with multiple malformations.
Based on animal and human data, work is accumulating that alterations of imprinting may
contribute to this increase as seen in specific recognized syndromes ( Beckwith Weidemann for
example). However, whether the imprinting errors occur related to the ART and ovulation
induction, or are related to the underlying infertility which brings the couple to the ART program
remains an area of investigation.
I26.
The correct answer is C. --. Cell free nucleic acids are ubiquitous in humans and reflect cell
apotosis due either to inflammation, necrosis or programmed cell death. Fetal cell free nucleic
acids are most often of placental origin and are of shorter length then the cell free nucleic acids
derived form the mother’s own cells undergoing cell turnover and death. Fetal cf nucleic acids
rapidly clear from the maternal circulation within hours of delivery which is a differentiating
property form intact fetal cells which may be ahrobroed for decades in the mother. The
persistence of fetal cells in the maternal circulation was one factor which hindered progression of
noninvasive prenatal testing which has been overcome by the rapid clearance of the cell free
nucleic acids
848
I27.
The correct answer is A.
Source : Reproductive Genetics – 1 Preconception
Explanation:
A. Aneuploidy – with an NT of 3.0 approximately 17% of fetuses have an aneuploidy,
this is likely higher given her maternal age of 40.
B. Developmental delay – among fetuses with increased NT who are found to be
euploid and without structural malformations, approximately 6% were found to have
developmental delay at later ages though these children were clustered among those with
markedly enlarged NT (> 6.0 mm)
C. Fetal demise – this can be associated with increased NT but typically at increased
dimensions of > 6.0 and those which persist into the second trrimester
D. Growth restriction – not classically associated with increased NT
E. Neural tube defect - while the risk of congenital malformations are increased in
euploid fetuses with an increased NT, the predominant malformation is cardiac anomaly
I28.
The correct answer is B.
Source : Reproductive Genetics – 2 Prenatal
Explanation:
A. Amniocentesis for anomalies – increases in clinically significant diagnoses beyond
that afforded by the conventional karyotype occur in almost 10% of samples sent for
ultrasound anomalies
B. Amniocentesis for maternal age > 35 – increased clinically significant diagnoses are
reported in approximately 1% of women
C. Chorionic villus sampling for increased nuchal translucency – a portion of increased
diagnoses would be anticipated given the increased association of large NT with later
structural anomalies, especially cardiac and 22q-.
D. Miscarriages - – increases in diagnostics are expected from the application of
chromosome microarray to stillbirths due to both the ability to utilize nonviable tissues
and the greater degree of diagnostic resolution
E. Stillbirths – increases in diagnostics are expected from the application of chromosome
microarray to stillbirths due to both the ability to utilize nonviable tissues and the greater
degree of diagnostic resolution
849
I29.
The correct answer is C.
Source : Reproductive Genetics – 2 Prenatal
Explanation:
A. Amniocentesis - amniocentesis is a earliest diagnostic test later in pregnancy (> 15
weeks) which would also provide definitive information to address the increased risk of
an unbalanced conception to the parent with a balanced translocation
B. Chorionic villus sampling – CVS represents the earliest diagnostic test in pregnancy
which would provide definitive information to address the increased risk of an
unbalanced conception to the parent with a balanced translocation
C. Noninvasive free fetal DNA – currently the available technologies for noninvasive
screening with fetal DNA have focused on the common aneuploides (trisomy 21, 13, 18)
and sex chromosome abnormalities and would not detect abnormalities in other
chromosomes
D. Percutaneous umbilical blood sampling - a earliest diagnostic test late in pregnancy (>
18 weeks) which would also provide definitive information to address the increased risk
of an unbalanced conception to the parent with a balanced translocation. May be chosen
over amniocentesis as while there is a higher risk of miscarriage with PUBs, the rapid
return of a full karyotype (48 hours) may be of greater importance.
E. Preimplantation genetic diagnosis – chromosome rearrangements are ideal candidates
for assessment by PGD either through individually designed FISH probers or the use of
chromosome microarray technologies
I30.
The correct answer is B.
Source : Reproductive Genetics – 2 Prenatal
Explanation:
A. Combined first trimester screen - the chances of a true positive from among the
screen positive population is low, approximately 3%
B. Free fetal DNA screen – among women older than 35 years of age, the chances of a
true positive from among the screen positive population is high, greater than 97%
C. Intergrated serum screen – the chances of a true positive from among the screen
positive population is low, approximately 4%
D. Quad serum screen - the chances of a true positive from among the screen positive
population is low, approximately 2%
E. Triple serum screen – the chances of a true positive from among the screen positive
population is low, approximately 2%
850
I31.
The correct answer is C.
Source : Reproductive Genetics – 1 Preconception
Explanation:
A. No, there is no evidence that either PGD or assisted reproduction is associated with
increased risks of birth defects – less research has been compiled on the population
utilizing PGD but meta-analyses support a 30% increased rate of congenital
malformations with ART.
B. No, your preimplantation genetic diagnosis was performed to allow only healthy
embryos to develop – PGD is a testing modality which tests for specific conditions and
the intention is not to screen for numerous conditions to assure a healthy fetus.
C. Yes, ART is associated with an approximate 30% increase in congenital
malformations – recent meta-analysis supports this estimate of risk although the
contribution of underlying sub/infertility among the couples remains an unanswered
question.
D. Yes, for imprinted gene abnormalities but these can be tested for at amniocentesis imprinted abnormalities are not the only form of genetic disease or congenital anomaly
for which the ART exposed fetus faces an increased risk. As the number of potential
changes to imprinted genes may be numerous and the connection to clinical disease still
evolving, they are not screened for at amniocentesis.
E. Yes, for imprinted gene abnormalities but these were tested for at PGD – imprinted
abnormalities are not the only form of genetic disease or congenital anomalies for which
the ART exposed fetus faces an increased risk. The number of potential changes to
imprinted genes are numerous and the connection to clinical disease still evolving. PGD
is applicable for this testing.
I32.
The correct answer is B.
Source : Reproductive Genetics – 2 Prenatal
Explanation:
A. Combined first trimester screen – ultrasound and serum analytes used to screen for
Down syndrome which moves screening into the first trimester but can have as high as a
15% screen positive rate in > 35 year old women.
B. Free fetal DNA screen – less than 1% of women (0.02% reported in studies of women
> 35 years of age).
C. Integrated serum screen – a combined first and second trimester ultrasound and serum
screening for Down syndrome with the lowest screen positive rate of the serum analyte
programs.
D. Quad serum screen - a second trimester serum analyte screening for Down syndrome
with a screen positive rate in women < 35 of approximately 5%, increased detection of
trisomy 21 compared to triple screen.
E. Triple serum screen - a second trimester serum analyte screening for Down syndrome
with a screen positive rate in women < 35 of approximately 5%.
851
I33.
The correct answer is D.
Source : Reproductive Genetics – 2 Prenatal
Keywords: Birth defects, chromosomal abnormality, ultrasound detection
Explanation:
A. Atrial septal defect – an isolated cardiac defect with < 10 % association with
aneuploidy
B. Gastroschisis – considered a vascular defect typically unassociated with aneuploidy
C. Hydronephrosis – one of the soft markers associated with an increase in aneuploidy in
conjunction with other markers
D. Omphalocele – a single anomaly with a strong association with aneuploidy trisomy
21,18 and 13 (30%)
E. Ventriculomegaly (10 mm) – a finding usually detected in normal fetuses but with an
association with trisomy 21
I34.
The correct answer is E.
Source : Reproductive Genetics – 1 Preconception
Keywords: Neural tube defect, BMI, fetal survey
Explanation:
A. Cardiac anomaly – increased risk among those with obesity but not with relative risks
as high as for NTD
B. Cleft lip/palate – increased risk with higher levels of obesity
C. Gastroschisis – not seen in recent studies
D. Limb anomalies - variably associated at low levels in some studies
E. Neural tube defect – highest relative risks which increase as BMI increases
I35.
The correct answer is A.
Source : Reproductive Genetics – 1 Preconception
Keywords: Infertility, oligospermia
Explanation:
A. A balanced translocation – as sperm levels decrease, men are at increased risk to be
identified as carriers of balanced translocations.
B. 45, X – would present with Turner syndrome phenotype typically
C. 45, X/46, XX - typically presents with Turner syndrome phenotype
D. 47, XXY – azoospermia is typically identified when men carry sex chromosome
abnormalities
E. 47, XYY – not typically associated with alterations in sperm levels
852
I36.
The correct answer is A.
Keywords: Noninvasive prenatal testing (NIPT)
Explanation:
Source : Reproductive Genetics – 2 Prenatal
Explanation:
A. Demise of twin gestation at 8 weeks – a proposed contributor to false positives or
discordant results when the NIPT is positive and the fetal testing does not confirm the
aneuploidy
B. Diabetes type 1 – not shown to affect free fetal DNA levels
C. Diabetes type 2 - not shown to affect free fetal DNA levels
D. Miscarriage within past year – fetal fragments of DNA clear the maternal circulation
within hours of delivery so prior pregnancies do not interfere with the test
E. Obesity (BMI > 30) – shown to decrease free fetal DNA levels as BMI increases, a
reason to note BMI but not withhold testing
I37.
The correct answer is D.
Keywords: cell free DNA, aneuploidy screening, ultrasound anomalies
Explanation: The overall sensitivity and specificity for cell free DNA panels are > 98%
although variation by the specific chromosomes do exist. Thus while it is possible that
the "normal" cffDNA result may represent a false negative, the likelihood of this
occurrence in the setting of a fetus with a neural tube defect is low. Most fetuses with a
NTD are chromosomally normal. Similarly, while microdeletions have been identified in
3-7% of fetuses with ultrasound anomalies, this would not represent the most likely
finding.
I38.
The correct answer is B.
Keywords: aneuploidy screening, ultrasound anomalies
Explanation: Echogenic bowel is represented by gastrointestinal images which appear as
bright as bone. Such a finding has one of the highest likelihood ratios for trisomy 21, on
the order of 5 - 7 fold. This is higher than the likelihood ratios for most other "soft
markers" such as shortened femur, (LR approximately 2.0) and renal pyelectesis and
echogenic intracardiac focus, both of the latter with likelihood ratios < 2.0. Choroid
plexus cyst is a soft marker for trisomy 18 and not a marker for trisomy 21.
853
I39.
The correct answer is D.
Keywords: assisted reproduction, ultrasound anomalies, imprinting disorders
Explanation: Pregnancies conceived by assisted reproduction (ART) are at increased risk
for isolated congenital anomalies, multiple congenital anomalies and disorders of
imprinting. The association of imprinting disorders such as Beckwith Weidman
syndrome with IVF has received attention as a means to understanding the biology during
assisted reproduction. However, the chance of a child with BWS is actually low
(estimated at 1 in 4,000 ART pregnancies). Conversely, the relative risk of isolated
congenital anomalies has been in the 6-8% range when background risks for isolated
congenital anomalies have been in the 5-6% range. For multiple defects, while the
relative risk is larger, their overall background rate is lower (0.5% increasing to 2.0% in
the ART population).
I40.
The correct answer is C.
Keywords: ultrasound anomalies, teratogenicity
Explanation: Teratogenicity from fetal exposure to fairly common agents and conditions
is underappreciated. These include the effects from maternal obesity, hyperglycemia in
poorly controlled diabetes and alcohol exposure. Of the three above - maternal obesity
has the highest odds of producing a congenital malformation in the fetus - as much as
four fold higher in women with a BMI of > 40. Poorly controlled diabetes also increases
risk with hyperglycemia reflected by hemoglobin A1C levels. However even with
elevated levels of 9.0 - 12.0, the associated risk congenital malformation is a doubling of
the background rate. Alcohol consumption during pregnancy is associated most often
with the development with delayed growth, neurocognitive delays and a characteristic
facial appearance. Medication exposures remain relatively poorly studied during
pregnancy. However for more commonly encountered medications, such as a common
antibiotic (ampicillin) and an oral antigylcemic (metformin) , the risks of a birth defect
have not been found to be significantly elevated.
I41.
The correct answer is C.
SOURCE: Lecture/Slides/Syllabus
KEYWORDS: aneuploidy, cell free DNA screening, ultrasound “soft markers”
Ultrasound “soft markers” were introduced at a time when other options for screening for trisomy
21 were limited. “Soft markers” are subtle imaging findings (such as echogenenic intracardiac
focus (EIF), renal pyelectasis, choroid plexus cyst) with an increased association with aneuploidy.
However, these “soft markers” also occur in 1-4% of fetuses with normal chromosomal content.
Given a very low relative risk of trisomy 21 based on the presence of an isolated EIF, with a
negative cell free DNA study, the recommendation is to classify EIF as a normal variant. EIF has
no association with cardiac anomaly, cardiac function or other altered fetal development such as
growth restriction.
A. Amniocentesis – with a negative cfDNA for trisomy 21, an EIF does not contribute further to
the baseline risk and diagnostic testing for trisomy 21 is not recommended. Even prior to the
introduction of cfDNA, EIF in isolation carried a very small increased relative risk and in women
under the age of 35, diagnostic testing was not recommended.
B. ECHO – EIF does not have any association with cardiac anomalies which would be the focus
for an echocardiography study of the fetus
C. Correct answer
854
D. Quad screening is a second trimester maternal serum test (MSAFP, HcG, estriol and inhibin)
with relatively less sensitivity for trisomy 21 (70-80%) than NIPT (> 98%). With the higher
sensitivity (>98%) of cfDNA, screening with a second, lower sensitivity test would not provide
helpful information.
E.Third trimester fetal growth ultrasound – EIF is not associated with other alterations in fetal
growth or development. One “soft marker” however, echogenic bowel, can herald the
development of fetal growth restriction in settings where its presence is likely associated with
placental insufficiency.
I42.
CORRECT ANSWER: C
SOURCE OF ITEM TOPIC: Lecture/Slides/Syllabus
KEYWORDS: amniocentesis, rupture of membranes, oligohydramnios
EXPLANATION:
Premature preterm rupture of membranes (PPROM) following a procedure has a different
prognosis than when membranes rupture spontaneously in the second trimester. The majority of
pregnancies (90%) with PPROM after an amniocentesis deliver at a late preterm (average 34
weeks) gestation. This is in comparison to the higher risk of loss (in some series over 80%) and
very early preterm delivery (< 26 weeks) noted when PPROM occurs spotnaneously. The
difference may reflect the underlying causes with inflammation / infection occurring more
commonly in the spontaneous rupture group.
A. Contractures occur more commonly following spontaneous PPROM as the chance of
reaccumulation of amniotic fluid is lower. With continued oligohydramnios from the time of
spontaneous PPROM to delivery, this places the infant at higher risk of contractures. With post
amniocentesis PPROM, the likelihood of reaccumulation of the amniotic fluid within 1 -2 weeks
is high.
B. Fetal growth restriction may be slightly increased in this population but does not occur in the
majority.
D. Miscarriage is more commonly seen following spontaneous PPROM likely related to the
differences in underlying causation of the membrane rupture. Spontaneous PPROM has a more
inflammatory/infectious nature which is the driver then for the pregnancy loss. With PPROM
after amniocentesis, the majority of women reaccumulate the amniotic fluid and the overall
likelihood of miscarriage is low.
E. Pulmonary hypoplasia occurs after persistent oligohydramnios during the second trimester. As
the amniotic fluid often reacccumulates within a week or two following an amniocentesis related
PPROM, pulmonary hypoplasia is uncommon. The ongoing oligohydramnios with spontaneous
PPROM contributes to a high rate of pulmonary hypoplasia, similar to that seen in other
conditions without amniotic fluid such as those with renal agenesis.
I43.
CORRECT ANSWER: A
SOURCE: Lecture/Slides/Syllabus
KEYWORDS: teratogenicity, environmental exposures
EXPLANATION: There is not a recognized level of alcohol exposure during pregnancy which is
considered safe and without effect on the developing fetus. With higher levels of alcohol
consumption, fetal alcohol syndrome can be seen which is often characterized by facial
dysmorhpia, growth restriction, congenital anomalies and cognitive/behavioral effects. However,
beyond FAS, there is now animal and human data supporting a fetal alcohol disease spectrum.
855
The risks of fetal effects from alcohol are highest with increased volumes, binge drinking and
early first triemster exposures. There is no safe level of alcohol consumption during pregnancy
with increasing efforts focused on preconception / gynecologic / primary care screening and
education.
B. Folate deficiency – although her bypass surgery can place her at risk for iron and other vitamin
deficiencies, lower levels of folate have not been reported. Additionally, many flours and grain
products in the United States are now supplemented with folate. Women are encouraged to take a
prenatal vitamin with folate when attempting conception to further lower their background risk of
about 1 in 1,500.
C. Obesity – a high BMI is associated with over a 2-fold increase in risk of neural tube defect,
and shows an increasing risk profile with increasing BMI. Her BMI currently is not elevated and
her prior obesity prior to her gastric bypass has not been associated with an increased risk of
NTD.
D. Paint fume exposure – the toluene components of aerosolized paint at high levels can produce
features similar to fetal alcohol syndrome. However, an isolated exposure to paint in a well
ventilated room does not reach the levels of concern which are obtained only from ongoing,
recreational inhalation of toluene fumes (“huffing spray paint”).
E. Selective serotonin uptake inhibitors - paternal use of SSRIs would not be expected to have a
teratogenic effect on the fetus.
I44.
CORRECT ANSWER: A
SOURCE OF ITEM TOPIC: Lecture/Slides/Syllabus
KEYWORDS: NIPT, biologic discordancy, low fetal fraction
EXPLANATION: Women with an elevated BMI have a higher rate of uninterruptable results
due to low fetal fraction. Some of the lowering of the fetal fraction may be related to increased
volume distribution in women with increased BMI, similar to the process known to impact the
analytes for serum screening. Additionally, however, as the fetal fraction is a portion of the
maternal fraction of free DNA, in women with an increased BMI, their inflammatory state is
higher resulting in a higher baseline level of cell free DNA from cell apoptosis. This then lowers
the ability to detect the relatively lower level of fetal free DNA.
B. Confined placental mosaicism (CPM) can result in false positive NIPT results when the
aneuploid and not the normal cell line are detected in the maternal circulation (and the fetus is
euploid). With CPM, false negative results can also occur when the normal but not aneuploidy
line is detected in the maternal circulation and the fetus possesses the aneuploid line (seen with
trisomy 18 and 13)
C. NIPT results which are positive for more than one chromosome abnormality have been
associated with maternal malignancy. The malignancies themselves have been shown to be
responsible for the chromosomally abnormal cell free fragments which have a maternal and not
fetal origin.
D. The cell free DNA methodology used by most NIPT programs reflects changes in the expected
number of DNA fragments for each chromosome but does not specifically identify which are
maternal as opposed to fetal. NIPT results with both high and low counts the sex chromosomes
have been reported to also occur with a normal fetal karyotype and a mother with an
unrecognized sex chromosome aneuploidy (47, XXX or 45, X /46, XX).
856
E. Vanished twins occur more commonly than realized. When SNP methodology is used for the
NIPT assay, cell free fragments from two distinct fetal genotypes have been reported. Notably,
the presence of free fetal DNA fragments from a co-twin can persist for more than 8 weeks after
early fetal demise.
I45.
CORRECT ANSWER: C
SOURCE OF ITEM TOPIC: Lecture/Slides/Syllabus
KEYWORDS: noninvasive prenatal testing, aneuploidy screening, first trimester screening
EXPLANATION: The overall sensitivity for cell free DNA panels for trisomy 21 is > 98%;
lower but still high sensitvity is also present for the other common aneuploidies. This exceeds the
sensitivity for serum screening for trisomy 21 (about 80%).
A. NIPT is not currently supported for twins with concerns for unequal levels of cell free fetal
DNA from each placenta. Serum screening in twins has a lower sensitivity than in singletons but
has been validated in clinical studies.
B. Serum screening results in a 5-10% false positive rate (rate at which women are called
positive), this rate is < 1.0% with NIPT.
D. NIPT has rates of noninformative results which can vary between 3-7% and are associated
with a higher rate of aneuploidy; noninformative results from serum screening are rare.
E. The positive predictive value for trisomy 21 in a woman over 35 years of age is over 80% for
NIPT and 4-6% for serum screening.
I46.
CORRECT ANSWER: E
SOURCE OF ITEM TOPIC: Lecture/Slides/Syllabus
KEYWORDS: viral infection, birth defects
EXPLANATION: Following primary cytomegalovirus (CMV) infection during pregnancy, there
is a relatively high rate of transmission of the virus to the fetus and an increased risk of fetal
infection especially during the first and second trimesters. As with other viral infections, the
consequences of CMV fetal infection are often marked by CNS involvement including
ventriculomegaly, abnormal gyration patterns and abnormal development of CNS structures.
Isolated ventriculomegaly can occur as a normal variant but also be associated with aneuploidy,
single gene disorders and genetic syndromes. The prognosis for isolated ventriculomegaly
depends on whether additional CNS involvement is evident on MRI and the underlying etiology.
A. Gastroschisis - this abdominal wall abnormality occurs to the right of the umbilicus, with
extrusion of GI contents without a peritoneal covering. Conditions which increase the risk of
vascular events such as maternal cocaine have been associated with gastroschisis.
B. Large for gestation age – growth parameters > 90% in the second trimester are unusual and
may raise the concern for a genetic overgrowth syndrome. However much of fetal growth and
overgrowth occurs during the third trimester whether due to an overgrowth syndrome, exposure
to maternal diabetes or as part of the normal distribution of the population. CMV fetal infection
typically leads to growth restriction which can present in the second trimester as CMV infection
of the placenta impedes fetal support.
C. Polydactyly – while associated with some chromosome and genetic syndromes, in isolation
postaxial polydactyly is often a familial trait.
D. Polyhydramnios – altered amniotic fluid levels (both high and low) in the second trimester are
more highly associated with underlying fetal concerns then when noted after 28 weeks. High
levels of amniotic fluid may be due to obstruction of the GI tract at several levels
857
(tracheoesophageal fistula, small bowel obstruction). High amniotic fluid levels due to glucose
imbalance from maternal diabetes usually presents in the third trimester. CMV infection can lead
to low amniotic fluid (oligohydramnios) thought to be due to two mechanisms – early
inflammation of renal cells with infection and ongoing placental infection leading to decreased
placental perfusion.
858
J. Mendelian Genetics Questions J1-J29
J1.
In successive pregnancies, two healthy parents have two children with osteogenesis imperfecta,
type 1 There is no family history of this disorder. Which of the following concepts is most likely
to explain this situation in this family?
A.
B.
C.
D.
E.
J2.
Ben and his wife Jennifer come in for preconception counseling. They met at a local benefit for
cystic fibrosis (CF). Ben is healthy, but has a brother with CF and Jennifer has CF. Which of the
following percentages best represents their risk for having a child with CF?
A.
B.
C.
D.
E.
J3.
25%
33%
50%
67%
100%
Mary was recently diagnosed with Duchenne Muscular Dystrophy (DMD), an X-linked disorder.
Her brother died of the condition. Which of the following explanations is the most likely cause of
Mary’s clinical findings?
A.
B.
C.
D.
E.
J4.
Genomic imprinting
Germline mosaicism
Multifactorial inheritance
Parental consanguinity
Uniparental disomy
Her father has germline mosaicism for Duchenne Muscular Dystrophy.
Her father is not her biological father (non-paternity).
Her mother has germline mosaicism for Duchenne Muscular Dystrophy.
She has a comorbid disorder, androgen insensivity syndrome.
She has an X;21 translocation, with the normal X carrying the mutant DMD gene.
A 14-year-old African-American girl has the early signs of a rare neurodegenerative disease that
was first described in members of a small village in Africa. Nothing is known about the cause of
the disease, although it is suspected that it is genetically determined. Her parents are healthy, but
her paternal grandfather, who came to the United States from that same region in Africa, died at
56 from the same disease. Which of the following assumptions about this disease can be
definitively concluded based on this history?
A.
B.
C.
D.
E.
A multifactorial etiology for this disease is likely.
An autosomal dominant inheritance pattern can be excluded.
An autosomal recessive inheritance pattern is likely.
An X-linked recessive inheritance pattern can be excluded.
The underlying cause is certainly genetic.
859
J5.
Billy has Branchio-Oto-Renal syndrome (BOR). He has bilateral renal hypoplasia and requires
dialysis. He is also deaf. His mother has a mild hearing loss and is missing one kidney. Her
80-year-old father is well except for a history of having had a neck cyst resected when he was a
boy. Which of the following genetic concepts best explains this situation?
A.
B.
C.
D.
E.
J6.
Cleidocranial dysplasia (CCD) is caused by mutations in the gene RUNX2. A small number of
CCD patients with a more severe phenotype (including mental retardation), typically have a
microdeletion of the 6p21 region that includes RUNX2. The nature of RUNX2 mutations is best
described by which of the following explanations below?
A.
B.
C.
D.
E.
J7.
RUNX2 mutations result in a loss of function of the protein.
RUNX2 mutations result in a triplet repeat expansion.
RUNX2 mutations result in excessive activity the protein product.
RUNX2 mutations result in the disruption of an imprinted gene.
RUNX2 mutations result in the protein having a novel function.
Triplet repeat disorders can be characterized by several features that become evident when
evaluating an affected family’s pedigree. Which of the following characteristics is a geneticist
most likely to find only within a family harboring a disorder caused by expansion of triplet
nucleotide repeats?
A.
B.
C.
D.
E.
J8.
Allelic heterogeneity
Heteroplasmy
Locus heterogeneity
Pleiotropy
Variable expressivity
The disorder appears to be more likely with increased paternal age.
The disorder is transmitted only through the females in the pedigree.
The disorder may appear to have skipped generations in the family.
The severity of the phenotype worsens in subsequent generations.
There is a reduced number of male offspring compared to female offspring.
Assume that hypophosphatemic rickets is an X-linked dominant trait with normal fitness and
complete penetrance in both sexes. Which of the following ratios represents the expected
male:female sex ratio in the affected population?
A.
B.
C.
D.
E.
1:3
1:2
1:1
2:1
3:1
860
J9.
Robert is a three-month-old infant with ectrodactyly (a rare malformation of the hands and feet)
who is brought to you by his parents, Marsha and Todd. Robert has one older sibling who has
normal hands and feet. Although neither parent has hand or foot abnormalities, other members of
Todd’s family are also affected with ectrodactyly. A family pedigree is below, where symbols
representing individuals affected with ectrodactyly are shaded and symbols representing
individuals without clinical symptoms are unshaded. Which of the following modes of
inheritance is the best described by this pedigree?
Todd
Marsha
Robert
A.
B.
C.
D.
E.
J10.
Autosomal dominant with reduced penetrance
Autosomal recessive with variable expressivity
X-linked dominant with reduced penetrance
X-linked recessive with a high degree of pleiotropy
Y-linked with evidence of allelic heterogeneity
Mark’s father was affected with type 1 albinism, an autosomal recessive disorder that results from
a mutation in tyrosinase. Mark and his wife Annette show no signs of the disease. Annette's
maternal grandmother was also affected with type I albinism. Annette is pregnant. Which of the
following probabilities represents the risk that this fetus will be affected with type 1 albinism?
A.
B.
C.
D.
E.
1/4
1/8
1/16
1/24
1/64
861
J11.
Based on the pedigree below, which of the following inheritance patterns best describes the
disease represented by the shaded members of the family?
A.
B.
C.
D.
E.
J12.
Autosomal dominant
Autosomal recessive
Mitochondrial
X-linked dominant
X-linked recessive
Bobby and Donna are the parents of Stephen, a child affected with a fully penetrant, autosomal
recessive disorder that occurs in the population with an incidence of 1/6400 and is easily
diagnosed at birth. Neither parent has this disorder and their next child, Brad, is born without any
apparent signs of the disease. Brad grows up and marries Anita, a woman with no known family
history of the disorder. Which of the following ratios best represents the chance that a child of
Brad and Anita will be affected with the same disorder that affects Stephen?
A.
B.
C.
D.
E.
1/60
1/120
1/180
1/240
1/320
862
J13.
A significant proportion of nonsyndromic holoprosencephaly is caused by mutations in the sonic
hedgehog gene (SHH), which codes for a secreted signaling protein required for developmental
patterning. Some individuals with SHH mutation have only mild clinical symptoms, such as
midface hypoplasia. Other individuals with SHH mutations may have severe CNS and facial
malformations that are incompatible with life. Which of the following genetic concepts best
describes this spectrum of clinical features?
A.
B.
C.
D.
E.
J14.
Digenic inheritance
Locus heterogeneity
Reduced penetrance
Sex-influenced expression
Variable expressivity
The ABO blood group is coordinated by three alleles (A, B, O) at the blood group locus. The A
and B alleles function in a codominant manner, and both are dominant to the O allele. The
pedigree below shows the blood groups from a three-generation family.
I
Type A
Type A
Type A
Type ?
Type B
Type B
II
III
Type B
Type A
Type O
Based on this information, which of the following allele pairs represents individual II-1’s most
likely genotype at the ABO blood group locus?
A.
B.
C.
D.
E.
AA
AB
AO
BO
OO
863
J15.
Janet and Jim are married and both have achondroplasia. Each always knew they would only
marry someone who also had achondroplasia. This type of choice, which at the population level
can disturb Hardy-Weinberg equilibrium, is an example of which of the following genetic
concepts?
A.
B.
C.
D.
E.
J16.
Bethany is affected with a rare form of retinitis pigmentosa (a progressive retinal degeneration)
that only occurs when individuals are heterozygous for mutations in two different unlinked genes,
ROM1 and peripherin. Individuals who carry a mutation in only one of these genes are not
affected; only patients heterozygous for mutations in both genes develop the disease. Which of
the following genetic concepts explains this complex inheritance pattern?
A.
B.
C.
D.
E.
J17.
Co-dominant inheritance
Digenic inheritance
Genetic heterogeneity
Pleiotropy
Triallelic inheritance
An autosomal recessive disorder has a frequency of 1 per 2500 in the Caucasian population. All
known cases of this disorder result from an identical mutation in the causative gene. Which of the
following percentages represents the approximate proportion of this population with two copies
of the normal allele for this gene?
A.
B.
C.
D.
E.
J18.
Assortative mating
Codominance
Consanguinity
Linkage Disequilibrium
Positive selection
2%
4%
90%
96%
98%
A three allele locus on the X chromosome controls the green-sensitive pigment for color vision.
The normal allele “G” is dominant to the other two alleles, both of which are mutations. The g1
mutation leads to a condition called green shift – the presence of a weakened green-sensitive
pigment where only certain shades of green are indistinguishable from browns. The g1 mutation
is dominant to the g2 mutation, which results in green blindness, where reds, greens and yellows
cannot be distinguished. A man with green shift vision marries a woman with green blindness.
Which of the following types of green vision will the sons born to this couple have?
A.
B.
C.
D.
E.
All will have green blindness
All will have green shift
All will have normal green vision
Half will have green shift; half will have green blindness
Half with have normal vision; half will have green shift
864
J19.
Which of the following modes of inheritance is most likely to explain the findings for this rare
disorder shown in the pedigree below?
A.
B.
C.
D.
E.
J20.
Autosomal dominant with imprinting
Autosomal recessive
Digenic
Mitochondrial
X-linked dominant
Which of the following modes of inheritance is most likely to explain the findings for this rare
disorder shown in the pedigree below?
A.
B.
C.
D.
E.
Autosomal dominant with imprinting
Autosomal dominant with male limitation
X-linked dominant
X-linked recessive
Y-linked
865
J21.
Which of the following modes of inheritance is most likely to explain the findings for this rare
disorder shown in the pedigree below?
A.
B.
C.
D.
E.
J22.
Autosomal dominant with incomplete penetrance
Autosomal dominant with sex limitation
Autosomal recessive
X-linked dominant
X-linked recessive
Which of the following modes of inheritance is most likely to explain the findings for this rare
disorder shown in the pedigree below?
A.
B.
C.
D.
E.
Autosomal dominant with imprinting
Autosomal dominant with incomplete penetrance
Pseudodominant
X-linked recessive
X-linked dominant
866
J23.
Which of the following modes of inheritance is most likely to explain the findings for this rare
disorder shown in the pedigree below?
A.
B.
C.
D.
E.
Autosomal dominant
Autosomal dominant with imprinting
Mitochondrial
X-linked dominant
X-linked recessive
867
J24.
Which of the following modes of inheritance is most likely to explain the findings for this rare
disorder shown in the pedigree below?
A.
B.
C.
D.
E.
J25.
Autosomal dominant with imprinting
Autosomal recessive
Mitochondrial
X-linked dominant
X-linked recessive
Differences in the degree to which different tissues are impaired in an individual with a
mitochondrial disorder is best explained by which of the following genetic concepts?
A.
B.
C.
D.
E.
Germline mosaicism
Heteroplasmy
Heterozygosity
Incomplete penetrance
New mutation
868
J26.
Which of the following modes of inheritance is most likely to explain the findings for this rare
disorder shown in the pedigree below?
A.
B.
C.
D.
E.
J27.
Autosomal dominant with imprinting
Autosomal dominant with sex limitation
Digenic
X-linked dominant
X-linked recessive
Which of the following genetic concepts is most likely to explain the findings for this rare
disorder shown in the pedigree below?
A.
B.
C.
D.
E.
Digenic inheritance
Germline mosaicism
Imprinting
Recurrent new mutation
Triplet repeat expansion
869
J28.
Which of the following disorders is most is most consistent with the findings shown in the
pedigree below?
A.
B.
C.
D.
E.
J29.
Hemochromatosis
Incontinentia pigmenti
MELAS
Neurofibromatosis 1
Tuberous sclerosis
Which of the following disorders is most is most consistent with the findings shown in the
pedigree below?
A.
B.
C.
D.
E.
Duchenne muscular dystrophy
Hemochromatosis
MERFF
Rett syndrome
X-linked hypophasphatemic rickets
870
Answers to Mendielian Genetics Questions J1-J29
J1.
Answer: B. Type I OI is an autosomal dominant condition. Germline mosaicism is a common
explanation for the apparent recessive transmission of a known dominant disorder, and OI is one
of the more common syndromes to manifest germline mosaicism. 1st cousins with Angelman
syndrome are most likely due to an imprinting or UBE3A mutation; father to son transmission of
sickle cell disease is most likely due to pseudodominant inheritance, with the mother being a SS
carrier; deafness in siblings who are heterozygous for a GJB2 mutation can occur due to several
reasons – another genetic cause, a shared environmental agent (e.g., maternal CMV infection), or
compound heterozygosity of the GJB6 deletion (“Digenic” inheritance).
J2.
Answer: B. Jennifer will pass on a CF allele. There is a 2/3rds chance that Ben is a carrier, and 1
in 2 (50%) chance that he will pass on the abnormal allele.
J3.
Answer: D. In androgen insensitivity syndrome an affected individual will have the phenotypic
appearance of a female but the chromosomal compliment (46,XY) of a male. Non-paternity and
paternal germline mosaicism have no bearing, as the altered X is transmitted from the mother;
maternal germline mosaicism cannot cause this to occur either. An X;21 translocation would
cause skewed X-inactivation, but as described the normal X chromosome would be preferentially
inactivated (would have the DMD gene), while the translocated X has the normal dystrophin gene
and would not cause DMD.
J4.
Answer: D. It is unlikely that this disorder is X-linked because of the male to male transmission
(the girl’s grandfather to father) it would require. While it may be genetic, it is still possible that
the disorder is caused by an environmental agent that is shared by members of this village, such
as a particular food. Autosomal dominant and autosomal recessive inheritance are both possible.
J5.
Answer: E. This is a classic example of variable expressivity. While there are several genes that
can cause BOR, it is safe to assume that the same gene and the same mutation are present in all
members in this family. Therefore, neither locus nor allelic heterogeneity would be correct.
Heteroplasmy refers to mitochondrial genomic mutations, which present differently than a
developmental disorder such as BOR. Pleiotropy refers to the wide range of effects caused by a
genetic mutation.
J6.
Answer: A. Mutations that result in premature termination upstream or within the runt domain
produce classic CCD by abolishing the transactivation activity of the mutant protein with
consequent haploinsufficiency. Hypomorphic mutations (Arg391X, Thr200Ala, and 90insC)
result in a clinical spectrum ranging from isolated dental anomalies without the skeletal features
of CCD to mild CCD to classic CCD. Intrafamilial variability is significant (genereviews.org).
J7.
Answer: D. Worsening in the severity of the phenotype in subsequent generations is also called
genetic anticipation, and this is a distinguishing feature of triplet repeat expansion disorders.
Maternal transmission only is seen with mitochondrial inheritance. A reduced number of males
compared to females is seen with X-linked lethal disorders. Increased paternal age is seen with
some autosomal dominant disorders. “Skipped generation” is seen with autosomal dominant
disorders that feature non-penetrance.
J8.
Answer: B. Frequency of males = q. Frequency for females = 2pq + q2 which will generally be
close to 2q for rare traits. Ratio for male to female about 1:2.
871
J9.
Answer: A. Males and females are affected in roughly equal proportions, there are affected
individuals in each generation and there is transmission by individuals of both sexes – all
characteristics of autosomal dominant inheritance. It appears that Todd inherited the mutation
causing ectrodactyly, but did not express clinical symptoms, a finding consistent with reduced
penetranceOther choices: A – E-linked inheritance can be excluded because Todd's sister
inherits ectrodactyly from her father (who passed an X and not a Y chromosome to her). B –
Affected individuals in multiple generations makes this option unlikely, especially given that the
disorder is rare (the chance that four carriers married into this family would be very low). C & D
– The male-to-male transmission (from Todd's father to Todd's brother) excludes X-linked
inheritance patterns.
J10
Answer: B. -Individuals with autosomal recessive disorders have mutations in each copy of the
causative gene. For a child of Mark and Annette to have type 1 albinism, each parent must inherit
one copy of the mutation and then pass it to the child. Mark's father had type 1 albinism so Mark
inherited one copy of the tyrosinase mutation from him. There is a 1/2 chance he will pass this
along to a child. We know Annette’s mother is an obligate carrier for type 1 albinism (she had to
inherit one of the two mutant copies from Annette’s grandmother, who was affected.)
Subsequently, there is a 1/4 chance Annette will pass along a tyrosinase mutation to a child – a
1/2 chance she received the mutation her mother inherited from Annette's grandmother multiplied
by a 1/2 chance that she would pass along that mutation to a child. So there is a 1/8 total chance
the fetus will inherit two copies of the tyrosinase mutation.
J11.
Answer: D. Notice that individuals are affected in each generation – a hallmark of dominant
disorders. In addition, there is no male-to-male transmission, there are approximately twice as
many affected females as males, and affected fathers transmit the disease to all of their daughters
– all of which suggest a sex-linked dominant disorder. Other choices: A – The lack of male-tomale transmission and the presence of affected individuals in each generation make this an
unlikely option. B – Although the pedigree certainly suggests a dominant disorder, the lack of
male-to-male transmission and the excess of affected females suggests a sex-linked dominant
disorder is a better choice. C – In a sex-linked recessive disorder, there is usually an excess of
affected males – females who inherit one copy of the mutation are generally not affected
(although it is possible that under certain conditions some carrier females exhibit clinical
symptoms).
J12.
Answer: D. Three things must occur for Brad and Anita to have an affected child.
1. Brad must be a carrier for the disorder. As Brad doesn't show any symptoms of the disorder
(which is fully penetrant), he does not carry both copies of the mutation. Subsequently, there
are three equally likely remaining genotype options for Brad – homozygous normal,
heterozygous with the mutation received from his mother, or heterozygous with the mutation
received from his father. The overall probability that Brad is a carrier is therefore 2/3.
2. Anita must also be a carrier. Because she has no family history of disorder, we assume her
risk of being a carrier is equivalent to the carrier frequency in the population. This can be
deduced using Hardy-Weinberg equilibrium. The incidence of this disorder in the population
is 1/6400. A rough estimate of the carrier frequency can be obtained by doubling the square
root of the population frequency – § u 1/80 = 1/40. This is the chance that Anita is a carrier.
3. Both Brad and Anita must pass their mutation on to a child. If they are both carriers, the
chance they would have an affected child is 1/4.
To obtain the final probability for this problem, multiply the probabilities from all three events –
2/3 u 1/40 u 1/4 = 2/480 = 1/240.
872
J13.
Answer: E. Variable expressivity refers to the variation observed in clinical symptoms among
individuals with an identical genetic disorder. Other choices: A – Digenic inheritance is a
situation where a disease is the result of double heterozygosity at two different genes acting in an
additive fashion. . B – Locus heterogeneity occurs when a disorder can be caused by mutations
in more than one gene. C – Reduced penetrance describes an individual who inherits a diseasecausing mutation, but exhibits no clinical symptoms whatsoever. This is believed to be the result
of modifying effects of other genes or environmental factors D – Sex-influenced expression refers
to disorders that are more frequently expressed in one sex than another, probably due to the
influence of sex-specific hormones.
J14.
Answer: C. Individual II-1's parents have type A blood. Their genotypes are either AA or AO.
Depending on their specific genotypes, Individual II-1 could either be AA, AO or OO. Individual
II-1's wife has type B blood and is either BB or BO at the blood group locus. Based on her
father's type A blood, we can infer that Individual II-1's wife is BO at the ABO locus – she
inherits a B from her mother and an O from her father. Individual II-1 and his wife have three
children. The second child has type A blood. Individual II-1's wife can pass on an O or B allele to
her child. In order to have type A blood, the child must have inherited an O from his mother and
an A allele from Individual II-1. The third child has type O blood, which can only occur when an
OO genotype is present. So Individual II-1 must also have an O allele. That means Individual II-1
is AO at the blood group locus and has type A blood.
J15.
Answer: A. One of the assumptions underlying Hardy-Weinberg equilibrium is